You are on page 1of 759

1

(Q.-Q.): Criminal conspiracy under the Indian Penal Code (IPC) is a substantive offence in
itself and punishable separately. There have been rare instances where persons have been
tried for commission of the substantive act of criminal conspiracy. However, most
commonly, the charge of criminal conspiracy is slapped on an accused person along with the
charge of a substantive offence under the IPC or any other law which he may be accused of
committing along with other co-conspirators. Criminal conspiracy is hatched to commit an
illegal act which is an offence punishable under law. It is not essential that the accused person
must do an overt act, and mere agreement between two or more persons to commit an illegal
act is sufficient to constitute the offence of criminal conspiracy. It is also not necessary that
the object of the conspiracy should have been achieved for it to be considered as an offence.
Even if the conspiracy fails on account of abandonment or detection before commission of
offence, the very act of entering into an agreement by the co-conspirators is itself an offence
and punishable under the law.

. Ajab with the intention of winning the elections in the Resident Welfare Association spreads
around false information along with Gajab about Bhola, a competition who is also seen as a
viable candidate for the election. This not only leads to Bhola losing the elections but also,
Bhola being portrayed in a bad light in the society. This affected Bhola‘s life, personally and
professionally.

(a) Both Ajab and Gajab will be liable for spreading false information against Bhola.

(b) Both Ajab and Gajab will be liable for committing criminal conspiracy against Bhola.

(c) Both Ajab and Gajab will be liable for defamation of Bhola.

(d) Only Ajab will be liable for defamation and not Gajab.

. Persons A, B, C and D conspire to rob the local bank for which they decide that A and B
would threaten the bank manager at the counter, C would take care of the situation at the gate
and D would keep the car ready for A, B, and C when they successfully rob the bank.

(a) All A, B, C and D are liable for committing criminal conspiracy.

(b) Only A and B will be liable for committing criminal conspiracy.

(c) All A, B, C and D are liable for committing criminal conspiracy, but C and D will be
exempted from punishment.

(d) All A, B, C and D are liable for committing criminal conspiracy, but C and D will be
allowed to plea bargain.

. Referring to the above question, if D did not know of the objective and intention of A, B and
C and was merely a taxi driver who was being paid for his job, then would D be liable under
the offence of criminal conspiracy?

(a) D would not be liable as he did not have knowledge of the main object and purpose for
which A, B and C came together.
(b) D would not be liable as he is not gaining any money in the heist.

(c) D would be held liable as mala fide intention can be formed at a moment, pre meditated
intention is not a necessary requirement.

(d) Both (a) and (b)

. Referring to question 2, what if A and B first decided to rob the bank and later C joined
them, knowing what the agreement is for and having the intention to do the same. Will C
fetch any liability under Indian Penal Code?

(a) No, C will not be held liable as he was not a part of initial plan and joined only later.

(b) No, C will not be held liable as the conspiracy was done by A and B.

(c) Yes, C will be held liable as conspiracy can be developed in successive stage.

(d) Yes, C will be liable as he joined the two after becoming aware of the object and had
developed the same mala fide intention.

. Six persons agreed to commit robbery in the house of Mr. Santosh Jain, a jeweler merchant.
It was agreed that robbery would be committed on the night of 1st May 2006 at about
midnight. On the agreed day at the agreed time, only four of them were present. They
attempted robbery and were caught by the locals. The other two were nowhere to be found
around the crime site. Decide the liability.

(a) The offence of criminal conspiracy has been committed. There are six persons involved.
All will be liable.

(b) The offence of robbery was not committed as they couldn‘t execute their plan properly
and were caught mid-way.

(c) There is conspiracy to commit the offence of Dacoity and all except the two will be liable.

(d) The other two persons are deemed to have participated in the robbery. Hence the offence
under IPC has been committed.

. Which among the following statement cannot be referred in affirmation from the given
passage?

(a) Criminal conspiracy includes any legal act which is done through illegal means by any
person.

(b) Two or more person when commit a murder, will be charged for criminal conspiracy.

(c) An overt act in consonance with a plan to commit an offence is not an essential
requirement.
(d) Even if two or more person fail to commit an offence, yet will be charged for criminal
conspiracy.

(Q.-Q.): Copyright is a form of intellectual property protection granted under Indian law to
the creators of original works of authorship such as literary works (including computer
programs, tables and compilations including computer databases which may be expressed in
words, codes, schemes or in any other form, including a machine-readable medium),
dramatic, musical and artistic works, cinematographic films and sound recordings. Copyright
law protects expressions of ideas rather than the ideas themselves. Under section 13 of the
Copyright Act 17, copyright protection is conferred on literary works, dramatic works,
musical works, artistic works, cinematograph films, and sound recording. For example,
books, computer programs are protected under the Act as literary works. Copyright refers to a
bundle of exclusive rights vested in the owner of copyright by virtue of Section 14 of the Act.
These rights can be exercised only by the owner of the copyright or by any other person who
is duly licensed in this regard by the owner of the copyright. These rights include the right of
adaptation, right of reproduction, right of publication, right to make translations,
communication to public etc. Copyright protection is conferred on all Original literary,
artistic, musical or dramatic, cinematography and sound recording works. Original means,
that the work has not been copied from any other source. Copyright protection commences
the moment a work is created, and its registration is optional. However, it is always advisable
to obtain a registration for better protection. Copyright registration does not confer any rights
and is merely a prima facie proof of an entry in respect of the work in the Copyright Register
maintained by the Registrar of Copyrights. As per Section 17 of the Act, the author or creator
of the work is the first owner of the copyright. An exception to this rule is that the employer
becomes the owner of the copyright in circumstances where the employee creates a work in
the course of and scope of employment. Copyright registration is invaluable to a copyright
holder who wishes to take civil or criminal action against the infringer.

. Adam and Danny are renowned writers in the world of literature. Adam comes out with a
book which is a big hit. Danny decides to write the translation of Adam‘s book. This turns
out to be great. However, Danny fails to take the consent of Adam before publishing the
translation. Adam files a suit challenging it and claims it to be a clear infringement of the
copyright. Decide.

(a) The suit is not maintainable as the translation would not be an exact copy to the original
work.

(b) The suit is not maintainable as it was not necessary to take consent.

(c) The suit is maintainable and it amounts to copyright infringement.

(d) The suit is maintainable but would not be covered under copyright law.

 
. X and Y are best friends. X in good faith and confidence narrates a plot to Y. X expresses
his intention to make a movie on the plot narrated by him. Y takes advantage of the situation
and releases a movie before X could. The movie yields huge financial returns. Aggrieved by
this, X files a claim against Y for copyright infringement. Decide.

(a) The claim will succeed as it the plot belonged to X.

(b) The claim will succeed as the narration was made in good faith.

(c) The claim will succeed as Y failed to take the consent of X.

(d) The claim will not succeed as it was just a mere idea.

. June and Julie are renowned personalities in the film industry. June released a movie on
partition of India just after a week of a similar partition movie released by Julie. Both the
movies had similar scenes of bloodshed, riots etc. Enraged by this, Julie files a complaint
against June for copying the original work of Julie. Decide.

(a) Julie will succeed as his work was published before June.

(b) Julie will succeed as the June‘s movie had similar scenes.

(c) Julie will not succeed as such scenes are integral to partition movies. 

(d) Julie will not succeed as there is possibility that it was June‘s original idea.

. Anand writes romantic plays and is popular for his work among the theatre artists. Recently,
he wrote a play highlighting the struggles of inter-community couples. Anand used to work
for a production house, ASD Deluxe and the chief manager of the ASD Deluxe is still fond of
his work. ASD Deluxe comes out with a movie based on the theme of inter-community
couples highlighting other issues like dowry etc. as well. Anand was disturbed after
witnessing this unfortunate event and decided to file a claim against ASD Deluxe. Decide.

(a) Anand will succeed as it was his original work and this amount to copyright infringement.

(b) Anand will not succeed as the modes of expression were different.

(c) Anand will succeed as they fail to take consent before moving further with such
reproduction of the work.

(d) Anand will not succeed as the theme was similar but not the expression.

. Based on the information provided in the passage, decide whether the book keeping system
to maintain the accounts can be copyrighted by companies based on the arrangement that they
release in their record books?

(a) The system can be copyrighted as it would be unique to every company.


(b) The system cannot be copyrighted as it is a general technique available to all and used by
all.

(c) The system can be copyrighted by the ones who introduced the system.

(d) The system can be copyrighted depending on the aspects of expression and arrangement.

(Q.-Q.): Defamation, which works contrary to the popular rhyme, ―Sticks and stones will
break my bones, but words will never hurt me‖, was derived from the Latin term ‗Diffamare‘
which means ‗spreading evil report about someone‘. The offence of defamation involves
causing damage to the reputation of the other. Justice Cave therefore very rightly defined
defamation as a ―false statement about a man to his discredit.‖ Salmond and Heuston on the
Law of Torts, 20th edition defined a defamatory statement as under- ―A defamatory
statement is one which has a tendency to injure the reputation of the person to whom it refers;
which tends, that is to say, to lower him in the estimation of right thinking members of the
society generally and in particular to cause him to be regarded with feelings of hatred,
contempt, ridicule, fear, dislike or disesteem. The statement is judged by the standard of an
ordinary, right thinking member of the society.‖ The right to reputation, which is recognized
as a dimension of the right to privacy, fundamental right under Article 21 of the Constitution
of India, is what is protected by the law of defamation. Reputation is how a person is
perceived by others, i.e. the opinion of the community against a person. The Supreme Court
in a landmark judgment held that an individual has a right to protect his reputation from being
unfairly harmed and such protection of reputation needs to exist not only against falsehood
but also 20 of 35 certain truths. Intention to cause harm to reputation of a person is sine qua
non of the offense of defamation. Section 4 of the Indian Penal Code, 18 reads as follows-
―Whoever, by words either spoken or intended to be read, or by signs or by visible
representations, makes or publishes any imputation concerning any person intending to harm,
or knowing or having reason to believe that such imputation will harm, the reputation of such
person, is said, except in the cases hereinafter expected, to defame that person.‖ Therefore,
the essential elements which constitute defamation are[1]

1. Publication of words, spoken or written or signs or any visual representation;

2. With the intention or belief that such imputation would lower the estimation of such a
person.

3. Such defamation is done before the right thinking members of the society. Publication
which is the primary sense of communication becomes one of the most important factors in
suit for defamation. It is publication through which the third party gets to hear or come to
know about the alleged defamatory imputations. Such a publication could be done in 2
ways[1]

a. Libel– This is words written

b. Slander– This is words spoken, thus temporary in nature.

. BabuBhaiya tells Raju, who manages Babu Bhaiya‘s kirana shop that – ―Sell nothing to
Devi Prasad unless he pays you ready money, for I have no opinion of his honesty‖. When
Devi Prasad comes to buy ration from the shop, Raju refuses to sell him ration and tells him
about the warning given by Babu Bhaiya. Outraged by this, Devi Prasad files a suit for
defamation against Babu Bhaiya. Will Devi Prasad succeed?

(a) Devi Prasad will succeed as Babu Bhaiya made a defamatory statement against him and
conveyed the same to Raju.

(b) Devi Prasad will succeed as Babu Bhaiya deliberately degraded Devi‘s reputation in front
of Raju and therefore should be charged under IPC.

(c) Devi Prasad will not succeed as Babu Bhaiya made certain imputations in good faith to
save himself from any further losses. (d) Devi Prasad will not succeed as there was no
publication of the defamatory statement.

. Mr. Unjust a sitting judge of sessions court, while rejecting the bail application of Hasina
who is involved in a case of contractual fraud, called her a woman of bad character and made
remarks about her being physically involved with men without marriage. Hasina filed a case
against Mr. Unjust for defaming her. Decide. (a) Hasina will succeed as remarks made by Mr.
Unjust were uncalled for and are defamatory in nature. (b) Hasina will succeed as remarks
made by Mr. Unjust harmed her reputation in the eyes of right thinking people of society. (c)
Hasina will not succeed as she is an accused in an unlawful act and Mr. Unjust said what is
true in reality. (d) Both (a) and (b)

. The Standard, a reputed Newspaper, initiated an ad campaign where it exhibited how


irrelevant information is provided by other newspapers. Seeing the ad, people started creating
mockery of Birmingham Post, rival newspaper of The Standard, believing that ad by The
Standard was meant against Birmingham Post. It was found the ad was created by H.
Salmond, editor of The Standard. In the meantime, Daily Post, another newspaper, also
objected to the ad stating that along with Birmingham Post, it was also referred in the ad
which was clear from the usage of the word "Post". Against this, Birmingham Post also
initiated an ad campaign thereby mentioning how The Standard is maligning it by providing
unauthentic information. This led to the reputation loss of The Standard. Meanwhile, H.
Salmond delivered a speech expressing how several newspapers are providing wrong and
irrelevant information, and that one should refer The Standard only. Birmingham Post filed a
defamation suit against The Standard for the ad campaign. Will Birmingham Post succeed in
its claim? (a) Birmingham Post will succeed since the ad campaign by The Standard was
understood to be against Birmingham Post by reasonable men. (b) Birmingham Post will not
succeed since ad campaign was not against it. (c) Birmingham Post will not succeed since the
ad campaign was not untruthful and false in nature. (d) Birmingham Post will succeed since
ad campaign caused reputation loss to it. 21 of 35

. In the above question, Daily Post also filed a defamation suit against The Standard for the
ad campaign since word "Post" was mentioned in it. Will Daily Post succeed in its claim? (a)
Daily Post will not succeed since there was nothing in the ad which was against Daily Post
unambiguously. (b) Daily Post will succeed since the usage of word "Post" made it clear that
ad campaign was directed towards it. (c) Daily Post will succeed since the ad campaign was
meant to damage the reputation of both Birmingham Post and Daily Post. (d) Daily Post will
succeed since ad campaign did not have any proof for alleging that other newspapers provide
irrelevant information.
. The Standard filed a defamation suit against Birmingham Post for mentioning that The
Standard had been maligning it. Will The Standard succeed in its claim? (a) Yes, The
Standard will succeed since ad campaign of Birmingham Post was truthful and justified. (b)
Yes, The Standard will succeed since ad campaign by Birmingham Post was meant against
The Standard. (c) No, The Standard will not succeed since ad campaign only represented
information which was true. (d) No, The Standard will not succeed since by ad campaign
Birmingham Post was merely exercising freedom of speech.

(Q. – Q.): In the law of torts, if any person commits any wrongful act which causes injury to
another person, he is held liable and has to pay damages or provide some other remedy which
the Court determines, to the victim of such an act. But in some cases even if a person suffers
some loss because of the act of another person, he cannot claim damages from that person
because of the operation of defenses of tort. One such defense available to a defendant is the
defense of volenti non fit injuria in which the plaintiff is not entitled to damages because he
consents to the act which has caused injury to him. Under these defenses, a defendant can
escape liability and volenti non-fit injuria is also one such defense which is available for the
defendant. In case a person gives his consent to doing of an act which leads to him getting
injured, then even if an injury is caused by the other person, he cannot claim any damages
from that person because the act was one for which he voluntarily consented. The consent of
the plaintiff acts as a defense and this defense is called volenti non fit injuria which means to
a willing person no injury happens. For the application of the defence of volenti non fit
injuria there are some essential elements or conditions which should be present in a case and
only when they are fulfilled, this defence can be taken to prevent liability. There are 2
essential elements in this defence: 1. The plaintiff has the knowledge of the risk 2. The
plaintiff with the knowledge of risk has voluntarily agreed to suffer the harm. Thus, whenever
the plaintiff is aware of the possibility of harm which is likely to be caused by an act and
when he still accepts to do that act and therefore agrees to suffer the injury, a defendant is
relieved of his liability. . Amita lives in the city of Jaipur. She is a responsible and caring
towards the fellow beings. At one instance, a fire breaks out in her locality and a child was
stuck inside. She called the fire brigade but they said it would take them an hour to reach the
destination. Worried for the child she goes inside the house and saves the child. However, she
succumbs to the fumes. The family of Amita asks for compensation from the company whose
faulty wiring caused the fire in the house. Decide. (a) The company is not liable as the risk
was willingly accepted by Amita. (b) The company is liable as Amita was consented to
saving the life and not succumbing to fire. (c) The company is liable as it was due to their
faulty wiring. (d) The company is liable as her consent was influenced due to the situation. 22
of 35 . Manoj is a renowned footballer in the city of Bhopal. While playing in a match he
receives a long pass from his teammate and has an open run to the goal having left the
goalkeeper behind. In an attempt to save the goal, the goalkeeper hurls hitting Manoj in the
back and breaking it. Manoj is paralyzed because of this and decides to move against the
goalkeeper. Decide. (a) The goalkeeper is liable as he intentionally caused injuries to Manoj.
(b) The goalkeeper is not liable as Manoj consented to the risks of football. (c) The
goalkeeper is liable as Manoj did not consent to such injuries. (d) The goalkeeper is not liable
as Manoj has knowledge of the risks of the game. . Manish is an employee in a brewery
owned by Shivani. At one instance, he was trying to remove the lid from the boiling vat.
Further, when he made a harder pull the lid came off but he fell back on the cooling vat
which contained a harmful liquid. Aggrieved by this, he decided to claim the amount for
damages but Shivani claimed the defense of Volenti Non Fit Injuria. Decide. (a) The claim is
justified as Shivani failed to provide adequate safety measures. (b) The claim is not justified
as Manish was aware of the risks and voluntarily encountered the same. (c) The claim is
justified as Manish did not consent to the injuries. (d) The claim is justified this was an
outcome of the wrongful act of Shivani due to which Manish became the sufferer. . Meena
decided to visit a doctor on account of lumps being developed around her breasts. The doctor
consulted for a surgery for the same. While the surgery was being performed, the doctor had
not so firm grounds to believe that these lumps were caused by the uterus so he removed it
believing that it would prevent development of lumps in the future. When Meena came to
know about this, she filed a suit. Decide. (a) The suit is not maintainable as she voluntarily
consented to operate. (b) The suit is not maintainable as the doctor to look into the best
interest of Meena. (c) The suit is maintainable as the consent was for the removal of lumps
and not the uterus. (d) The suit is not maintainable as she had the knowledge of the associated
risks of the operation which was to be cured. . Amit was walking down along a narrow train
tunnel on the railway track which was owned and occupied by Railways. The company was
aware that the tunnel was used by the members of the public so they asked the drivers to
signal the people by whistling and slowing down while entering the tunnel. When Amit was
walking through it, the driver failed to follow the aforementioned instructions. Pursuant to
this negligent driving, Amit sustained injuries. He decided to file a claim for compensation
from railways due to the negligence of their driver. Decide. (a) The claim will succeed as the
driver was negligent. (b) The claim will not succeed as he consented to the risk of entering a
narrow tunnel. (c) The claim will succeed as he did not consent to negligent driving of the
driver. (d) The claim will not succeed as the passage was owned by railway authorities and
was not for the public to use.

(Q.–Q.): The laws related to Extortion have been entailed in Section 3- 3 of IPC. The
definition of ‗Extortion‘ is given under Section 3 of IPC. Section 3 IPC contains the penalty
for extortion whereas Section 3 of IPC entails the punishment for an attempt to commit
extortion. It must be noted that Section 3 to 3 go on to the extent of discussing much harsher
punishments for much[1]aggravated forms of extortion. As per Section 3 IPC, extortion is an
act to put someone in fear of injury or any other harm to obtain his property or any other
valuable item. Section 3 IPC reads as follows: ―Whoever intentionally puts any person in
fear of any injury to that person, or to any other, and thereby dishonestly induces the person
so put in fear to deliver to any person any property or valuable security, or anything signed or
sealed which may be converted into a valuable security, commits ―extortion‖.‖ The main
essence of Section 3 is obtaining the delivery of property or any item of value in consequence
of dishonest inducement. To be more precise, an intention to cause wrongful loss to one and
wrongful gain to other is an essential ingredient of extortion. 23 of 35 Essential elements: 1.
Intentionally putting a person in fear of injury: It is a must that the person must have an
intention to cause a wrongful gain to one and wrongful loss to another in a manner where
another person is put under threat. The actual delivery of the property is essential to constitute
extortion. 2. The purpose of which is to dishonestly induce the person put in fear: The
objective of the 0ffence should have been towards the furtherance of the crime of Extortion.
3. To deliver property or valuable security: The actual transaction must have happened to
have the section come into play in its entirety. If at all, for any reason. Section 3 contains the
punishment for extortion. Section 3 is a cognizable offence wherein police may arrest without
warrant and the same is triable by any Magistrate. However, ordinarily, the warrant is
required to be issued in the first instance. It is a non- bailable offence and cannot be
compounded. Section 3 IPC reads as under: ―Whoever commits extortion shall be punished
with imprisonment of either description for a term which may extend to three years, or with
fine, or with both.‖

. MukeshMehra, threatens Om Kapoor to hand him over Om‘s one lakh cash prize which Om
won at Film Fare Awards. When Om refuses to hand over the money, Mukesh Mehra
threatens Om by saying that he will put Shanti (Om‘s love interest) on fire. Om, who is
pyrophobic gets scared of losing Shanti, hands over one lakh rupees to Mukesh Mehra.
Decide.

(a) Mukesh Mehra is liable for extortion as he made Om deliver his property.

(b) Mukesh Mehra is liable for extortion as he caused injury to Om while putting him in fear.

(c) Mukesh Mehra is liable for extortion as he made Om deliver his property by dishonestly
putting him under fear of setting Shanti on fire.

(d) Mukesh Mehra is not liable as he lacked mala fide intention when he put Om under fear.

. Popat and Chopat got to know about Mr. Joss having an extra marital affair. Popat and
Chopat, who are unable to secure job due to pandemic and finding themselves low on cash
decided to make good use of the Mr. Joss‘s situation. They stalked him and clicked few
pictures of Mr. Joss and sent him same over the mail threatening to disclose his extra marital
affair if he doesn‘t meet their demand of Rupees 5 Lakhs. Mr. Joss, who is not much bothered
about his reputation and bad consequences in his marital bliss doesn‘t give a heed to the
threat and refused to hand over the money. Later Mr. Joss filed a case of extortion against
Popat and Chopat. Will Mr Joss succeed?

(a) Mr. Joss will succeed as he was put in fear of injury to his reputation and was subjected to
the demand for Rs 5 Lakhs.

(b) Mr. Joss will succeed as any person on his caliber would get scared and will deliver the
money.

(c) Mr. Joss will not succeed as though there was threat to reputation yet he did not give away
any valuable property.

(d) Mr. Joss will not succeed as threat to reputation is not considered under the definition of
extortion under penal laws.

. Gunther owns a porcelain coffee mug which was gifted to him by his late aunt and is also
Gunther‘s prized possessions. Rachel, whom Gunther loves unrequitedly, wants the porcelain
coffee mug. Rachel understanding Gunther‘s obsession with her, offered to go out on a date
with him if he give her the porcelain coffee mug. Gunther who had to make a difficult choice
went ahead and gave the coffee mug to Rachel. Later, Rachel refused to go out with Gunther
and instead went out with Paolo. Gunther wants to file a case against Rachel for under
Section 3 of IPC.
(a) Gunther will succeed as Rachel ditched him with dishonest intention to get the porcelain
coffee mug from him.

(b) Gunther will not succeed as Rachel did not put him in intentional fear of injury, making
him deliver the coffee mug.

(c) Gunther will not succeed as Rachel lacked dishonest intention. (d) Gunther will not
succeed as it was a mere promise and such trivial acts are not punishable under Indian penal
laws.

. Sam, a batchmate of Anju made a video clip of Anju bathing in a bath tub. Sam blackmails
Anju for sexual favours and puts her in fear by pointing gun towards her. Anju in fear of
losing her life, gives up to the demands of Sam and before any unfortunate act happens, Anju
was saved by her mother Nandini who beat Sam black and blue. Will the act of Sam qualify
under the definition of extortion under Section 3 of IPC?

(a) Sam‘s act does not qualify as extortion though he will be held liable for outraging the
modesty of a woman.

(b) Sam‘s act does not qualify as extortion as there was no demand for delivery of property or
any valuable security.

(c) Sam‘s act of putting gun and causing fear of injury to Anju makes his act an act of
extortion.

(d) Both (a) and (b)

. Mohan, a CLAT aspirant is accused of committing extortion and is under trial.


Understanding that he might not get entry to college as only students with no criminal record
are eligible for admission, Mohan tried to settle the matter outside the court by paying a hefty
amount to the victim. Which among the following is correct?

(a) Mohan can settle the matter as extortion is a compoundable and bailable offence.

(b) Mohan can settle the matter as extortion is a compoundable and non bailable offence.

(c) Mohan cannot settle the matter as extortion is a non compoundable and non bailable
offence.

(d) Mohan cannot settle the matter as extortion is a non compoundable and bailable offence.

(Q.–Q.): In Damayanti v. Union of India, the validity of Hindi Sahitya Sammelan Act, 19 was
challenged as violative of Article 19(1)(c). The petitioner was a member of an association.
The Act changed the composition of the association and introduced new members. The result
of this alteration was that the members who voluntarily formed the association were now
compelled to act in the association with other members in whose admission they had no say.
The Supreme Court held- The Act violated the rights of the original members of the society
to form an association guaranteed under Art 19 (1) (c). ―The right to form an association‖,
the Court said, ―necessarily implies that the person forming the association have also the
right to continue to be associated with only those whom they voluntarily admit in the
association. Any law by which members are introduced in the voluntary association without
any option of being given to the members to keep them out, or any law which takes away the
membership of those who have voluntarily joined it, will be a law violating the right to form
association.‖ The Apex Court also reiterated that fundamental rights (right to association
here) are applicable only to laws made by or administrative actions of the State and do not
apply to actions of private persons. The Hindi Sahitya Sammelan Act does not merely
regulate the administration of the affairs of the original society, what it does is to alter the
composition of the society itself. The result of this change in the composition is that the
members who voluntarily formed the association are now compelled to act in the association
with other members who have been imposed as members by the act and in whose admission
to membership they had no say. Such alteration in the composition of the association itself
clearly interferes with the right to continue to function as members of the association which
was voluntarily formed by the original founders. The Act, therefore, violates the right of the
original members of the society to form an association guaranteed under Article 19 (1)(c).

. Hind Mazdoor Union Limited, a khadi textile cooperative private company, offered an
employment contract of two years to Gyanendra Tiwari. One of the clauses in the
employment contract provided that Gyanendra Tiwari must join Hind Mazdoor Sangh
(HMS), one of the trade unions active in Hind Mazdoor Union Limited. Decide which of the
following propositions can be most reasonably inferred through the application of the given
legal passage:

(a) The employment contract offered to Tiwari to join HMS is legal as it does not restrict his
freedom not to join any association.

(b) The condition requiring Tiwari to join HMS cannot bind him as it impinges on his
freedom not to join any association.

(c) Gyanendra Tiwari cannot claim a fundamental right to freedom of association against
Hind Mazdoor Union Limited and therefore, the contract would bind him even though his
freedom of association is restricted.

(d) The employment contract infringes Gyanendra Tiwari‘s freedom to decide with whom to
associate and therefore is legally not enforceable.

. If Parliament enacts a law which requires every employee to join the largest trade union in
their workplace mandating Gyanendra Tiwari to join HMS, then:

(a) Such a law would merely govern private action to which fundamental rights do not apply.

(b) Such a law would not curtail any individual‘s right to freedom of association.

(c) Neither the employment contract, nor the law of the parliament would be enforceable as
they would curtail the freedom of association.

(d) The law of Parliament would violate an individual‘s freedom not to join any association
and therefore be unconstitutional.
 

. If Hind Mazdoor Union Limited enter into an agreement with HMS wherein the former
agrees to hire only the existing members of HMS as employees, then:

(a) The agreement would be illegal as it would curtail the union members‘ right to decide
with whom they would like to associate.

(b) Such an agreement would infringe the union‘s right to decide with whom to associate and
therefore is legally not enforceable.

(c) The agreement would not be enforceable as it would infringe upon the employer‘s right
not to join an association.

(d) The constitutionality of this agreement cannot be contested on grounds of contravention


of fundamental rights as such rights are not applicable to private persons.

. Which among the following cannot be referred from the given passage?

(a) Right to association includes right to continue being associated with one association.

(b) Right to association also includes right not to associate.

(c) Right to association also includes right to alter the objective of one‘s association.

(d) Right to association can be invoked against policies made by state.

. Recently Gujarat witnessed a 15 day long violent protest by the workers from different
factories in the state demanding safe and secured work environment. Fearing that such protest
might fire up again, government passed a legislation restricting formation of any worker‘s
union and holding gatherings of any existing union. Mohan, the head of the department at a
shirt manufacturing factory wishes to form a worker‘s union to support the cause of the
workers. He was restricted under the newly passed legislation. Mohan challenges the same in
court of law.

(a) Mohan will succeed as it is violation of fundamental right to freedom of association.

(b) Mohan will not succeed as he can join some already existing worker‘s union.

(c) Mohan will not succeed as government can put reasonable restrictions on fundamental
right.

(d) None of the above.

(Q.–Q.): The Uttar Pradesh government has cleared the Uttar Pradesh Prevention of Cow
Slaughter (Amendment) Ordinance, 2020, which imposes stringent penalties for the offence
of cow slaughter. The penalties under the Ordinance include imprisonment ranging from one
year to ten years and fines ranging from Rs. 1 lakh and 5 lakh. Under the existing legislation -
the Uttar Pradesh Prevention of Cow Slaughter Act, 15 - the maximum penalty is seven years'
imprisonment. Additional Chief Secretary (Home) of the Uttar Pradesh government
AwanishAwasthi was quoted as saying that the Ordinance contains a provision that allows
authorities to paste pictures of the accused in public places if they go on the run. As per the
Ordinance, the accused will also have to bear expenditure related to the maintenance of the
seized cattle for a year or till the animals are released, whichever earlier. Penalties for injury
to cattle and inhumane transportation are also envisioned. Persons convicted for offences
under the Ordinance for a second time will be punished for a "double penalty". Back in 2017,
the Central government had notified rules pertaining to cattle slaughter, termed the
Prevention of Cruelty to Animals (Regulation of Livestock Markets) Rules. These Rules
became the subject of challenge in various High Courts, and had eventually reached the
Supreme Court. In July 2017, the Centre told the Supreme Court that it will consider the
representations received from various stakeholders and make necessary amendments to the
Rules.

. Hamid Miyan is into the meat exporting business in Delhi. He bought cows from a market
in Gorakhpur, Uttar Pradesh and was transporting it to his slaughter house in Delhi so that his
servant could utilize the cow for the export business. He was caught before he could cross the
border of UP and was penalized under Uttar Pradesh Prevention of Cow Slaughter
(Amendment) Ordinance, 2020. Decide.

(a) Penalization of Hamid Miyan is against the stated law as he did not slaughter the cow
within the state of Uttar Pradesh.

(b) Penalization of Hamid Miyan is against the stated law as he himself was not involved in
the slaughtering of the cow.

(c) Penalization of Hamid Miyan is as per the stated law because he bought the cattle for the
purpose of slaughtering.

(d) Penalization of Hamid Miyan is as per the stated law as there is no restriction in slaughter
cows in territory of Delhi.

. Assuming the above mentioned facts, Will the answer remain the same, if Hamid Miyan had
his slaughtering business in Kanpur and was about to slaughter the cow that he transported
from Gorakhpur?

 (a) Yes

 (b) No

(c) Can‘t say

(d) None of the above

. Hardeep is into the transport business and was arrested by police for transporting drums
containing animal flesh to the Lucknow airport for supply in other cities of the state.
According to the information provided in the passage, what provision of the Uttar Pradesh
Prevention of Cow Slaughter (Amendment) Ordinance, 2020 did UP police act under?

(a) UP police did not act as per the provisions of Uttar Pradesh Prevention of Cow Slaughter
(Amendment) Ordinance, 2020.

(b) UP Police acted as per the provisions of Uttar Pradesh Prevention of Cow Slaughter
(Amendment) Ordinance, 2020.

(c) UP Police acted as per the provisions of Prevention of Cruelty to Animals (Regulation of
Livestock Markets) Rules.

(d) None of the above.

. Manav was arrested by police and was later sent to judicial remand. He has been accused of
running a chain of shops across the state selling different kinds of meat. In his defence,
Manav argued that his business doesn‘t involve slaughtering and selling of cows. If his
argument is true, what will be the outcome of his case?

(a) Manav will not be held liable if he is able to prove his argument in court of law.

(b) Manav will not be held liable as his business is outside the scope of ordinance passed by
UP government.

(c) Manav will be held liable as there is no proof that he is not involved in cow slaughter.

(d) Manav‘s liability will be decided as per the discretion of the court.

2
Passage (Q.-Q.): Section 46(4) was inserted in the Code of Criminal Procedure, 19
(hereinafter referred to as the code) in the year 2005 and the proviso for (1) of the section was
inserted in the year 2009, this provision describes the procedure for the arrest of a woman,
section 46 of the act is read as under:- ” Arrest how made –

1. In making an arrest the police officer or other person making the same shall actually touch
or confine the body of the person to be arrested unless there is a submission to the custody by
word or action. Provided that where a woman is to be arrested, her submission to custody on
an oral intimation of arrest shall be presumed and, unless the police officer is a female, the
police officer shall not touch the person of the woman for making her arrest.

2. If such person forcibly resists the endeavor to arrest him, or attempts to evade the arrest,
such police officer or other person may use all means necessary to affect the arrest.

3. Nothing in this section gives a right to cause the death of a person who is not accused of an
offense punishable with death or with imprisonment for life.
4. Save in exceptional circumstances, no women shall be arrested after sunset and before
sunrise, and where such exceptional circumstances exist, the woman police officer shall, by
making a written report, obtain the prior permission of the Judicial Magistrate of the first
class within whose local jurisdiction the offense is committed or the arrest is to be made.

Therefore, the requirement of the provisions of sub-section (4) of Section 46 of the said Code
is two[1]fold. If the Police Officer wants to arrest the woman after sunset and before sunrise,
there must exist exceptional circumstances for such arrest. In cases where in such exceptional
circumstances do exist, a Lady Police Officer shall make a written report and obtain the prior
permission of the Judicial Magistrate, First Class in whose jurisdiction the offense is
committed or the arrest is to be made. Also, section A of the code states that –―No arrest
shall be made except in accordance with the provisions of this code or any other law for the
time being in force providing for arrest.‖ Sedition is punishable with death.

. The police had intelligence that a certain car is being used to smuggle drugs from Delhi to
Patiala and they stopped the alleged car in a RTO which was located on the outskirts of
Patiala. The police on findings drugs in the car, tried to arrest the driver who as a matter of
fact was deaf. Despite repeated oral requests from the police officer to enter the police van
she stood mute and still upon which the head constable lost his cool and grabbed her arm in
order to take her to the police car. Can the constable be held liable for violating the aforesaid
provisions?

(a) No, because the woman was deaf, she couldn‘t hear anything he said to her which left him
with no option but to grab her arm.

(b) No, because she tried to evade the arrest by not remaining still and not following the
officer‘s orders. Thus, allowing the police officer to use all means necessary to make the
arrest.

(c) Yes, because the police officer should‘ve waited for a female constable to arrive and make
the arrest.

(d) Yes, because she was a woman and by grabbing her arm he touched her directly violating
the provision mentioned in the paragraph.

. On 30th December the police received reports that a female suicide bomber has check in a
hotel planning to bomb herself in the midnight of 1st January. The female Inspector on
hearing the news immediately sent a written request to the judicial magistrate of the
concerned area at 5pm when it became evident that the arresting her before sunset wouldn‘t
be possible for her team. Since he was not in his office due to a meeting, he didn‘t get to
know about it and thus couldn‘t grant the permission. On not receiving permission for more
than 4 hours, the inspector decided to make the arrest. Can the inspector be held liable for
violating section 46 of the CrPC.

(a) No, since she sent the written request at 4pm before sunset and thus didn‘t violate section
46.
(b) No, because arresting a suicide bomber was very important for safety of the public which
over-rides the CrPC provisions.

(c) Yes since she made the arrest after sunset.

(d) Yes since there was no urgency and the police could‘ve arrested her next morning
because she anyways wasn‘t going to act before midnight leaving police with ample time to
act.

. In question 2, assuming the facts to remain same except that the sub-divisional magistrate of
the same jurisdiction , who is of same rank as the judicial magistrate first class, came to know
about the issue and granted the permission to police so that they can arrest the bomber before
she harms anyone. Is such arrest valid under Section 46 of CrPC.

(a) No, since SDM didn‘t have the power to grant such permission.

(b) No, since the permission wasn‘t granted by judicial magistrate first class.

(c) Yes, because SDM is also a magistrate and that too, of same rank.

(d) Yes, because under such a grave circumstance, it was only logical of the SDM to grant
permission.

. X is a student who made a video explaining how the current government is violating the
constitutional provisions of the country and targeting a certain class of the society calling for
all other students to come out and protest against such actions by the government. On
viewing this video, a lawyer filed a case accusing her of sedition. In furtherance of the
complaint, the police went in to arrest her to which she resisted by hitting a policeman on
head by a baseball bat making the fellow policeman to shoot X which resulted in her death. Is
police liable for her death under section 46?

(a) No, since she hit the policeman first.

(b) No, since she was accused of sedition which is punishable by death.

(c) Yes, because she didn‘t make the seditious video and was wrongly accused of sedition.

(d) Yes, because she was fighting to safeguard the constitutional provisions.

. In question 1.4, had few people edited her video by adding sentences like ―this government
should be overthrown‖, ―army is corrupt‖ and it was a result of this edited video that the
lawyer filed a complaint accusing her of sedition and ultimately caused her death during
arrest, would the answer be any different?

(a) Yes, since she was wrongfully accused of sedition.

(b) Yes, instead people who edited her video shall be liable for sedition
(c) No, the content of video is immaterial to the question

(d) No, since she forcibly arrested her arrest allowing policeman to use all means necessary

Passage (Q.-Q.): The DNA technology regulation bill and the personal data protection bill are
under consideration by two separate parliamentary committees. It is necessary to discuss the
nature of DNA information and to review these bills together to explore whether sufficient
privacy protection has been extended to DNA information. DNA samples are a potential
source of human genetic information and can reveal sensitive health information.
Unauthorized disclosure could lead to genetic profiling and other forms of profiling, leading
to discrimination that can undermine the dignity and the privacy of the person concerned and
cause grave violations of group rights. Profiling of religious, ethnic or linguistic minorities
has in many jurisdictions led to their persecution. It was precisely for this reason that
Information Technology Rules, 2011 was notified under the Information Technology Act,
2000. It referred explicitly to DNA in the definition of biometric information. Additional
privacy safeguards were provided for such sensitive personal information. However, in the
PDP bill, there is no explicit mention of DNA information even though any logical
interpretation of ‗biometric information‘ and ‗genetic information‘ would require the
inclusion of DNA. Such a reference to DNA information within the definitions of biometric
and genetic information would obviate the possibility of illogical interpretations. DNA Bill
creates two categories of people. There is no requirement for obtaining consent of those
arrested for offences punishable with death or by imprisonment of more than seven years.
The second category constitutes those arrested for offences punishable with a lesser penalty.
Procedurally, written consent is required for the second category, but if it is not given, it can
be mandated through the order of a magistrate. The DNA bill falters on endorsing voluntary
consent by opting for procedures such as consent given in writing on the order of the
magistrate instead of advocating for prior informed consent. Involuntary consent, tantamount
to coerced consent, seems to be integral to the DNA bill, whereas the PDP bill provides that
consent necessary for processing personal data be ‗informed, free, specific, clear and capable
of being withdrawn‘. The DNA bill thus needs to replicate the consent provision in the PDP
bill along with a provision to ensure that the person from whom bodily substance is taken for
DNA testing is given the ‗right to be heard‘ by a magistrate. Profiles in the DNA databank
are categorized into five indices — crime scene; suspects/undertrials; offenders; missing
persons; and unknown deceased person. This ensures that searches for DNA profile match
may be performed across all indices as all of them are maintained together in a single DNA
databank. ‗Suspects‘ are not defined under Indian law, leaving the investigating officer with
complete discretion to define a ‗suspect‘. It is unclear whose DNA profile will be uploaded
in the ‗suspects and undertrials‘ index, thus allowing the kind of searches that will lead to
violations of privacy. The DNA bill and the PDP bill have to be read concomitantly and
harmonized in a manner which would ensure robust and effective privacy protection for DNA
information.

. MrAman is guilty of an offence and accordingly is punished with 7 year of imprisonment.


For investigation purposes, his DNA was required, however he did not agree to provide his
DNA. As his DNA was imperative for investigation purposes, police forcibly took his DNA.
He challenges this in court of law. What could be the result if is governed under DNA Bill?
(a) MrAman would be successful as police has not functioned according to law.

(b) MrAman would not be successful as police has functioned according to law.

(c) MrAman would be successful as without his consent no one could take his DNA.

(d) MrAman would not be successful as his DNA was imperative for investigation purposes.

. What would be answer of facts mentioned in above question if has to be governed under
PDA Bill?

(a) It will change.

(b) It will not change.

(c) The information given in passage is not sufficient to answer the question.

(d) None of these.

. Which of the following points about DNA Bill has not been highlighted in the passage?

(a) There is need of a provision which will provide for the free consent as an integral part
while taking DNA of a person.

(b) There could be violation of privacy as certain terms used in DNA Bill are ambiguous and
vague.

(c) In some cases, DNA Bill advocates for Involuntary consent which is equal to coerced
consent.

(d) As suspects‖ are not defined in the Indian law, that particular category of indices should
be removed.

. Among which of the following the author seems to agree the most?

(a) DNA bill and PDP bill have been referred to two separate parliament committees
however, because the nature and subject matter of both of these acts is very much similar,
they should refer to single committee.

(b) The DNA bill and PDP bill should be read collectively and any discrepancy among them
should be removed.

(c) As ―suspects‖ are not defined in the Indian law, that particular category of indices should
be removed.

(d) None of these.


 

. Mr X has been guilty for rape of a child of 7 years. He has charged with offence of Rape,
Abduction, attempt to murder. If proven guilty he would be punished with death penalty.
Whole country is in anguish and demanding punishment to the offender. For the investigation
purposes police need DNA samples of Mr X. However, he denied to give it and this led
police to forcibly take it. Is police right in doing so? Answer according to provision of DNA
Bill?

(a) Police is right as it functioned according to law.

(b) Police is not right as it did not function according to law.

(c) Police is right as it his DNA was necessary for the investigation purposes.

(d) Police is not right as no one can take his DNA without his permission.

. Assuming the facts given in question 5 as same. What would be answer of the question
according to provisions of PDA Bill?

(a) It will change.

(b) It will not change.

(c) The information given in passage is not sufficient to answer the question.

(d) None of these.

Passage (Q.-Q.): Nuisance is an unlawful interference with a person‘s use or enjoyment of


land, or of some right over, or in connection with it. Nuisance is an injury to the right of a
person in possession of a property to undisturbed enjoyment of it and result from an improper
use by another person in his property.Stephen defined nuisance to be ―anything done to the
hurt or annoyance of the lands, tenements of another, and not amounting to a trespass.‖The
distinction between nuisance and trespass is clear. Trespass is direct physical interference
with the plaintiff‘s possession of land through some material or tangible object while
nuisance is an injury to some right accessory to possession but no possession itself. However,
both may overlap when the injury is to possessory as well as to some right necessary to
possession. In order to establish nuisance, there should exist a wrongful act and damage or
loss or inconvenience or annoyance caused to another. Such inconvenience must be such
which the law considers as substantial or material. Nuisance is of two kinds. Public nuisance
is defined in Section 268 of IPC as ―an act or illegal omission which causes any common
injury, danger or annoyance, to the people in general who dwell, or occupy property, in the
vicinity, or which must necessarily cause injury, obstruction, danger or annoyance to persons
who may have occasion to use any public right.‖ Public nuisance can only be subject of one
action, otherwise a party might be ruined by a million suits. In the following circumstances,
an individual may have a private right of action in respect a public nuisance.1. He must show
a particular injury to himself beyond that which is suffered by the rest of public i.e. he must
show that he has suffered some damage more than what the general body of the public had to
suffer.2. Such injury must be direct, not a mere consequential injury; as, where one is
obstructed, but another is left open.3. The injury must be shown to be of a substantial
character, not fleeting or evanescent.

. Abhilaksh‘s neighbor has a mango tree the branches of which spread to his garden. Every
year during the season, the braches shed lots and lots of leaves due to which Abhilaksh is
unable to maintain his own garden which is located there. Decide

(a) This amounts to trespass as plaintiff‘s possession of land is interfered with through some
material object

(b) This amounts to nuisance as this is interference with a right accessory to possession but
not possession itself

(c) This is a case of overlapping of both nuisance and trespass

(d) None of the above

. Please refer to the facts above. Abhilaksh‘s neighbor, near the mango tree has planted some
ayurvedic herbs which smell like ammonia. Irritated, Abhilaksh cannot go near his garden
located right next to the mango tree on his side of the land. Decide

(a) This amounts to trespass as plaintiff‘s possession of land is interfered with through some
material object

(b) This amounts to nuisance as this is interference with a right accessory to possession but
not possession itself

(c) This is a case of overlapping of both nuisance and trespass

(d) None of the above

. Shuk‘s neighbor wishes to grow the most beautiful sunflowers in the world. He watched on
YouTube that the more pollen a sunflower releases, the more beautiful they are. He procures
a special variety of sunflowers which release a lot of pollen and plants them in his kitchen
garden. The entire colony is now fed up of the pollen grains in the air and cannot breathe
properly. Shuk in addition is also allergic to pollen grains and develops rashes on the skin.
The colony collectively files a suit for public nuisance. Shuk files a private action suit too.

(a) Shuk‘s suit shall be entertained for he is suffering additionally from the sunflowers

(b) Shuk‘s suit shall not be entertained as a public nuisance suit has been filed already

(c) Shuk‘s suit shall be entertained for he is in the closest proximity to his neighbor

(d) Shuk‘s suit shall not be entertained as he should not contribute to multiplicity of
proceedings

. Please refer to the facts above. Shuk, due to the pollen grains in the air, developed breathing
difficulties. Consequently, he had to see the doctor and had to get expensive medicines.
When the public nuisance suit was filed against his neighbor, he also filed a private action
suit asking for medical expenses.

(a) Shuk‘s suit shall be entertained for he is suffering additionally from the sunflowers

(b) Shuk‘s suit shall not be entertained as a public nuisance suit has been filed already

(c) Shuk‘s suit shall be entertained for he is in the closest proximity to his neighbor

(d) Shuk‘s suit shall not be entertained as his injury is merely consequential.

Passage (Q.-Q.):The Supreme Court has observed that the beneficiaries of the policy taken
out by the insured are also 'consumers' under the Consumer Protection Act, even if they are
not parties to the contract of insurance. Referring to the definition of 'consumer' under
Section 2(d) of the Act, the bench noted that the definition of consumer under the Act is very
wide and it not only includes the person who hires or avails of the services for consideration
but also includes the beneficiary of such services who may be a person other than the person
who hires or avails of services. It said: "The definition of consumer under Section 2(d)
quoted hereinabove is in 2 parts. Subclause (i) of Section 2(1)(d) deals with a person who
buys any goods and includes any user of such goods other than the person who buys such
goods as long as the use is made with the approval of such person. Therefore, the definition
of consumer even in the 1st part not only includes the person who has purchased but includes
any user of the goods so long as such user is made with the approval of the person who has
purchased the goods. As far as the definition of the consumer in relation to hiring or availing
of services is concerned, the definition, in our view, is much wider. In this part of the section,
consumer includes not only the person who has hired or availed of the services but also
includes any beneficiary of such services. Therefore, an insured could be a person who hires
or avails of the services of the insurance company but there could be many other persons who
could be the beneficiaries of the services. Observing thus, the bench held that the definition
of 'consumer' includes beneficiaries who can take benefit of the insurance availed by the
insured. Consumer Protection Act clearly provides that a beneficiary of the services, other
than the insured is a consumer under the Act, it added. (Source: livelaw.in)

. A cold storage by the name of Sunshine Freezers entered into an insurance contract with an
insurance agency Modified India Insurance. The insurance company under the insurance
policy is liable to indemnify the cold store with regard to the value of goods in case of any
mishap. The cold storage services were availed by some farmers who stored their agricultural
produce in the same cold store. Three months into the contract, the cold storage catches fire
and all the goods stored are destroyed. This caused panic among the farmers. The insurance
company refused to reimburse Sunshine Freezers. The farmers filed a complaint against the
repudiation of the claim of the cold storage by the insurance company. Based on the
understanding of the passage decide accordingly.

(a) No relief will be granted to the Farmers as there was no privity of contract between the
farmers and the insurance company because the policy was taken by the cold store and not by
the farmers and therefore, they cannot be called 'consumers'.

(b) The insurance company is be liable to pay for the damage caused by the fire only to the
Sunshine Freezers.
(c) Since farmers are financially impoverished and have suffered huge losses. They are
entitled to be reimbursed. However, in the lack of a privity of contract they can only claim
damages from Sunshine Freezers.

(d) Since, farmers are also the beneficiaries of the services given by the Insurance company.
They are entitled to receive compensation from the Insurance company.

. Mrs X bought a new I-phone. Within a month of its purchase, she lost it on a train. Mr Y
found it lying on the berth of the train. He tried to look for the owner of the phone but to no
avail. So, he kept the phone for his use. Three months later, while picking up a call, the phone
blew up in the hands of Mr Y. Mr Y filed a complaint against Apple Inc. before the National
Consumer Dispute Resolution Forum.

(a) Mr Y is not a consumer because he did not own the phone in the first place.

(b) Mr Y has been using the phone for a period of three months which is sufficient enough for
him to be deemed as the consumer.

(c) Mr Y is an indirect beneficiary of the services of Apple therefore he is a consumer and is


entitled to a compensation.

(d) Mr Y is not a consumer within the meaning of the Consumer Protection Act.

. Smita bought a washing machine for the purpose of gifting it to her sister-in-law, Priti. After
5 months of use the machine started to act up. Priti tried contacting the company for their
services but they outrightly refused. Smita filed a complaint against the washing machine
company:

(a) Smita cannot file a complaint because she is not the immediate consumer of the product
anymore.

(b) Only Priti can file the complaint because she has been using the machine with the
approval of Smita and is a beneficiary of services of the washing machine company.
Therefore, she is the consumer in the instant case.

(c) Priti cannot be called the consumer of the services of the washing machine company
because there was no consideration on her part.

(d) Smita can file the complaint as she is a consumer.

. Strictly based on the understanding of the passage which of the following will be a
‗Consumer‘?

(a) A person who receives a laptop as a scholarship because he is a beneficiary

(b) A person who shoplifts a phone from the electronics market


(c) A person who found a phone lying on the road

(d) All of the above

Passage (Q.-Q.): Demonetization was a big shock for the whole nation in 2016 as Rs.500 and
0 currency notes ceased to be a legal tender as a prevailing currency in India. Just like the
currency notes, an act that was legal at one time is suddenly no longer legal and instead is
illegal and prohibited by law. These changing circumstances have led to wrongful
punishment of many individuals who did not actually commit any offense that was
punishable by law and according to their knowledge, that act was actually legal. These laws
which criminalize any act or increase the punishment of any offense are called ex post facto
laws and it has been mentioned in Article 20(1) of the Indian Constitution (1). Ex post facto
law is derived from the Latin word ―ex post facto‖ which means ‗out of the aftermath‘, it is
a law that has a retrospective consequence on any act committed, which is not prohibited by
law, before the enactment of a preceding law. Ex post facto laws can either label an act as an
offense with retrospective effect; or increase the punishment that is prescribed for an act
committed in the past. It is important to protect the citizens of India from being punished for
an act done, which was legal when it was done but was criminalized or the punishment for
that act has been enhanced by any act that was formulated later on. The prohibition under
Article 20(1) is only for a conviction or sentence, but not trial procedure. The objection does
not apply to a change of procedure or of court. A literal interpretation of Article – 20(1) of
the Indian Constitution would mean that the safeguards provided under this article are given
against conviction for an act or omission which was not an offense under the law that existed
at the time of the commission and against any increased punishment for the same act for
which the punishment was different at the time of the commission of the act. It is usually
claimed that Article – 20(1) invalidates ex post facto law.

. Chitmann Pandey got married to Mahima in 19. They were happy in their marriage until
Chitmann fell in love with Shanaya in 19 and got married to her in 19. The Bigamy act was
passed in 19 laying down that any person who had two spouse living at the time of enactment
of the law would be punishable with an imprisonment of 7 years. Chitmann was charged
under the aforementioned act for having two wives. Chitmann filed a petition challenging the
validity of the Act on the ground that the act was committed in past and cannot be
criminalized by subsequent law. Decide.

(a) Chitmann will not succeed as the Act does not use the term retrospectively and only
criminalizes the current act,

(b) Chitmann will not succeed as the Act prohibits Bigamy in the interest of the public.

(c) Chitmann will succeed as the past acts cannot be criminalized with retrospective effect.

(d) Chitmann will succeed as the punishment of 7 years is not appropriate to the act
committed by him.

. Badri was arrested for the possession of an AK-47 rifle and was booked under Terrorist and
Disruptive Activities (Prevention) Act (TADA) which provided for an imprisonment of 10
years. After his arrest the Government passed a new law Prevention of Terrorist Activities
Act (POTA) that made the possession of a rifle punishable for 20 years. Badri was convicted
and was sentenced to 20 years in prison. He challenged his conviction before the State High
Court on the ground that punishment for the act cannot be enhanced with retrospective effect.
Decide.

(a) Badri will succeed as the law which retrospectively increases the punishment is
unconstitutional.

(b) Badri will succeed as the punishment of 20 years is not reasonable.

(c) Badri will not succeed as the law does not use the term retrospectively.

(d) Both (a) and (b)

. A new taxation law was introduced by the government of India. This law provided that all
the individuals who evaded tax liability in the past six months would pay a higher penalty to
the concerned tax authorities. Kishan decided to challenge the law to be violative of the
fundamental right enshrined in Article 20 of the Constitution of India. Decide.

(a) The challenge is justified as it has increased the punishment through payment of higher
penalty.

(b) The challenge is not justified as it cannot be extended to imposition of fines.

(c) The challenge is justified as it operates through a retrospective effect.

(d) The challenge is not justified because the State has to punish such tax evaders.

. Mukesh is a sixteen years old boy who is convicted of committing an offence of house
trespass along with outraging the modesty of a girl aged seven years. In pursuance to the
same, he has been sentenced for a rigorous punishment of six months along with a fine of Rs.
50,000. After the judgment was passed, a new legislation came into existence. This provided
that an individual below the age of 21 years shall not be sentenced to imprisonment.
Therefore, Mukesh claims the benefit of the same. Decide.

(a) Mukesh cannot claim the benefit as the law was not in existence when he committed the
act.

(b) Mukesh cannot claim the benefit as the judgment has been passed.

(c) Mukesh can claim the benefit since rigorous imprisonment is not a proportionate
punishment to his act.

(d) Mukesh can claim ex-post facto law could be applied to reduce the punishment.
. In order to control the corrupt practices, Prevention of Corrupt Act (PCA), 10 was
introduced. This provided that if any public servant is found in possession of disproportionate
assets without an explanation they shall be found guilty of criminal misconduct. Sachin
challenged the law on the ground that while calculating the disproportionate assets only the
assets acquired after the date of PCA can be taken into account. If the asset acquired prior to
PCA is taken into account then it shall amount criminalizing a past act thus violative of
Article 20. Decide.

(a) The offence of disproportionate assets is different from its calculation therefore no
violation.

(b) It would be unjust to take past acts into consideration thus violative of Article 20.

(c) The data so available is not sufficient to reach a conclusive end.

(d) The assets acquired from more than ones known sources were made a crime only in 10 so
the property prior to it was irrelevant for PCA.

Passage (Q.-Q.): Road safety is a State subject. The administration of the Motor Vehicles
Act, 19 is under the transport Department. The aforesaid Act provides in detail the legislative
provisions regarding registration of motor vehicles. For exercising the legislative provisions
of the Act, the Government of India made the Central Motor Vehicles Rules 19. Additionally,
there are Rules of Road Regulations, 19. The RTO with which every individual is interacting,
its services are discussed below: Law relating to Registration of Vehicle o Mandatory
Registration: Under section 39 of the Motor Vehicle Act, 19, it prohibits driving of any motor
vehicle or any vehicle, which is not registered or no owner of vehicle should permit driving
of an unregistered vehicle in public place which is not registered under the provision of the
MV Act. Exception to this provision is cars with the dealers. Jurisdiction for Application:
Registration of the vehicle is done by the concerned authority is done on the basis of your
residence or place of residence or place of business, where the vehicle is normally kept. o
Application for Jointly owned Vehicle: In case of joint ownership, the registration of vehicle
can be applied by one of the owners. If a vehicle registered in one state is kept in another
state for more than a period of 12 months, then owner of such vehicle has to approach the
registration authority for assignment of new registration mark within whose jurisdiction the
vehicle is. o Change of Address: If there is change of address, then also, the owner of vehicle
is required to approach the authority within 30 days in whose jurisdiction he has shifted for
recording the change of address. Under section 48 of the motor vehicles act of 19, the
registering authority may, by order, refuse to register any motor vehicle, or renew the
certificate of registration in respect of a motor vehicle (other than a transport vehicle), if in
either case, the registering authority has reason to believe that it is a stolen motor vehicle or
the vehicle is mechanically defective or fails to comply with the requirements of this Act or
of the rules made thereunder, or if the applicant fails to furnish particulars of any previous
registration of the vehicle or furnishes inaccurate particulars in the application for registration
of the vehicle or, as the case may be, for renewal of the certificate or registration thereof and
the registering authority shall furnish the applicant whose vehicle is refused registration, or
whose application for renewal of the certificate of registration is refused, a copy of such
order, together with the reasons for such refusal.
. John purchased a new sports bike. It hasn‘t been yet received its registration no. But John
got too excited to flaunt it before his friends, so he took it out for a zoom. On the en route to a
club, cops stop him and seize the bike. John reasoned that his bike is new and the registration
no. is not yet issued, but the cops give him no ear and ask him to appear before the court.
According to the information provided in the above passage, was this action by the police
correct?

(a) Yes. Being an agency of the state, the police have absolute power to do so.

(b) No. The bike has the provisional registration on it. This action by the police was not legal.

(c) Yes. No owner of vehicle should permit driving of an unregistered vehicle in public place.

(d) No. The police has no power to seize any vehicle.

. Karan Singh resides in New Delhi. He purchased a pre-owned Audi Q7 from Ranbir Kapoor
of Chandigarh. It‘s been eleven months from the purchase, the car yet carries a Chandigarh
registration. One day cops spot the car on the streets of Noida and issue a challan. Karan
challenges this in the court of law and the judge orders the challan-issuing cop to pay penalty
to Karan from his personal account. According to the information provided in the above
passage, why do you think would be the reason for such ruling?

(a) Because Karan still had some time to apply for a local registration.

(b) Because Karan had a VIP registration no. on his car.

(c) Because Noida falls in UP, not in New Delhi.

(d) None of the above.

. Babubhaiya, Raju, and Shyam opened a garage in partnership. They decide to buy a tow
truck to cater the requirements of customers and get a pre-used tow truck. But they are now
stuck at to whose name they shall get it registered, as the RTO authority denied granting the
registration under their business‘s name. According to the information provided in the above
passage, what should they do?

(a) Resell the tow truck as registration is denied.

(b) They should by a new tow truck for the sake of the business.

(c) They should challenge RTO in the court of the law.

(d) They should re-apply for registration under any of the partner‘s name.

. ViratKohli purchased a new modified car and approached the local authority for
registration. But the person in charge denied the registration. ViratKohli took this matter to
the court of law and got the responsible officer suspended. According to the information
provided in the above passage, can you tell why this happened?

(a) Because the concerned officer didn‘t provide reason for the denial.

(b) Because ViratKohli is a celebrity, the case was ruled to pacify him.

(c) Because the concerned officer failed to prove that the car had stolen parts installed in it.

(d) None of the above.

. Based in Bhopal, Amit owned a house and a car. He got transferred to Indore, where he took
his car along. Six months later, his car gets involved in an accident. Though the accident
wasn‘t his mistake, but the insurance company refuses to pay him and the cops denied
releasing the vehicle without a penalty. According to the information provided in the above
passage, why do you think this happened?

(a) Because Amit‘s car was parked in no a parking zone.

(b) Because Amit‘s car was still registered at his old address of Bhopal.

(c) Because the damage wasn‘t covered by the insurance provider.

(d) Can‘t say. it‘s a state matter.

. According to the information provided in the above passage, what allows the transport
department to exercise legislative provisions about registration of vehicles?

(a) Motor vehicle Act 19. 

(b) Rules of Road regulations, 19.

(c) The central motor vehicle rules, 19.

(d) Highway act 19.

Passage (Q.-Q.): In common parlance, the word ‗abet‗ signifies help, co-activity and support
and incorporates within its ambit, illegitimate reason to commit the crime. So as to bring an
individual abetting the doing of a thing under any of the conditions specified under Section
of the Indian Penal Code, it isn‘t just important to demonstrate that the individual who has
abetted has participated in the means of the transactions yet additionally has been associated
with those means of the transaction which are criminal. The offense of abetment by
instigation relies on the intention of the individual who abets and not upon the act which is
finished by the individual who has abetted. The abetment might be by instigation, connivance
or purposeful aid as given under Section of the Indian Penal Code. Abetment under the Penal
Code involves active complicity on the part of the abettor at a point of time prior to the actual
commission of that offence, and it is of the essence of the crime of abetment that the abettor
should substantially assist the principal culprit towards the commission of the offence.
Nowhere, concurrence in the criminal acts of another without such participation therein as
helps to give effect to the criminal act or purpose, is punishable under the Code. For an
individual to be called liable for Abetment, and so as to proceed against an individual for a
criminal offense under Section , prosecution must claim the component of mensrea.
Negligence or carelessness can‘t be named to be abetment in order to punish the liable,
according to the arrangement of penal laws. So as to establish abetment, the abettor must
have appeared to ―deliberately‖ support the commission of the wrongdoing. In such a case
we need to just prove that the wrongdoing charged couldn‘t have been done without the
association as well as intervention of the supposed abettor isn‘t sufficient with the
prerequisites of Section .

. Rajat and Ambika were married. However, Rajat‘s mother Sashi is not fond of Ambika
because of the dowry given by her family. The conduct, language and expression of Sashi are
derogatory towards Ambika suggesting that she comes from a broke family. Ambika is
mentally broken by all this and gets depressed. One sudden day she decided to commit
suicide by hanging herself. Rajat and Sashi are prosecuted for abetting the suicide of Ambika.
Decide.

(a) Rajat is liable as he did not intervene in the cruel treatment of the mother to his wife.

(b) Rajat and Sashi are both liable. The former is liable indirectly while the latter is directly
responsible for the outcome.

(c) Only Sashi is liable as her conduct stimulated the outcome.

(d) Both of them are not guilty as no active steps were taken so as to be held liable under
abetment.

. Amit and Akhila are good friends. At one instance, where they were together Akhila told
Amit about the tussle that is going on between Akhila and her business partner Abhijeet.
Amit suggests the murder of Abhijeet. However, Akhila did not murder Abhijeet and ignored
the instigation. Decide.

(a) Amit is not guilty of abetment as the instigation was ignored.

(b) Amit is guilty of abetment even when it was not acted upon.

(c) Amit is not guilty as the Abhijeet was not murdered.

(d) Amit is not guilty as the words were devoid of mensrea.

. Based on the factual information provided in the aforementioned question, if Akhila stabbed
Abhijeet but he ended up recovering in the hospital, would that be considered abetment?

(a) No, it would not amount to abetment as the desired result was not achieved.

(b) No, it would not amount to abetment as Abhijeet has recovered.


(c) Yes, it would amount to abetment as the requisite effect is not an essential condition.

(d) Yes, it would amount to abetment and the liability would be shared by Amit and Akhila.

. Aditya instigated Bheem to instigate Palash to commit the offence of theft. Palash has
committed the theft and has been arrested for the same. Bheem is also arrested and charged
for the offence of abetment. Decide the liability of Aditya.

(a) Aditya will not be liable as he was not the instigator of Palash.

(b) Aditya will not be liable as he did not intend for the outcome.

(c) Aditya will be liable for abetting Bheem.

(d) Aditya will be liable but not under the offence of abetment.

. Based on the information provided in the passage, decide whether the words uttered in an
angry state would amount to abetment?

(a) Yes, it would amount to abetment but it has to assessed based on the result. 

(b) Yes, it would amount to abetment if such words incite the other person.

(c) No, it would not amount to abetment as it is devoid of intention.

(d) No, it would not amount to abetment but negligence.

Passage (Q.- Q.): A concept borrowed from the Irish Constitution, the roots of the Directive
Principles of State Policy lie in the history of India itself. Since the times of the myriad rulers
India had seen, the responsibility of both initiation and execution of efforts to improve the lot
of the people had lain with the government.Brought up in such an environment, the members
of the Constituent Assembly believed that the responsibility of development of the country
lay with the government. The Directive Principles of State Policy, hereafter referred to as
DPSPs, are enshrined in Part IV (Articles 36-51) of the Constitution of India. Their primary
function is to serve as a guideline for the government policy formation. The government is
expected to take these principles into consideration while performing its legislative functions.
But unlike the Fundamental Rights, contained in Part III of the Constitution, these principles
are not enforceable.But merely because they are not justiciable in a court of law, does not
render them useless. Their importance has increased manifold over the years. They serve not
only as guidelines today, but also keep a check on the governments, even though that check is
not the Court‘s but the citizens‘. The parties that form governments today are not concerned
with the well-being of the nation. They play divisive politics for their personal betterment.
They are concerned with the furtherance of their 26 of 34 ideologies that the nation may not
even share. In this environment, the DPSPs are a yardstick for the government‘s performance
and also a check on arbitrary legislation. Apart from laying the guidelines for government
policy making, the DPSPs also lay down the objectives of the Indian State. They differentiate
between a ‗Welfare State‘ and ‗Police State‘, their presence making India the former. They
provisions in this Part include promotion of welfare of citizens, provision of free legal aid to
the economically disabled, organizing village panchayats, developing a Uniform Civil Code,
raising nutrition levels, etc. They also aim to establish democracy promised in the Preamble.

. Amit was arrested for the charges of smuggling drugs. When the matter went to the court,
the Judge noticed that Amit was not represented by a lawyer. On being questioned about the
same, he expresses his inability to afford a lawyer due to financial reasons. The DPSP
provide for a free legal aid at such instances. Further, the State Bar Council Rules provided
for every lawyer to represent minimum of 4 clients with economic inadequacy. Mr. Tripathi
was asked to represent Amit as his counsel. However, Mr. Tripathi denies stating that he
already had 4 such cases with him and is occupied with a Tax case in the High Court. Can
Mr. Tripathi Refuse?

(a) Mr. Tripathi can refuse as he has already taken the required 4 cases as per the rules.

(b) Mr. Tripathi can refuse since he has a cases being listed in the high court.

(c) Mr. Tripathi can refuse as there is no proof of the financial incapacity of Amit.

(d) Mr. Tripathi cannot refuse as the free legal aid should be provided those who cannot
afford it for economic reasons.

. Manisha was working in a factory owned by Mr. Anirban Bansal but operated and managed
by the State government. Manisha gets pregnant and seeks for maternity leave. However, she
was asked to resign on the maternity grounds with an assurance that she can reapply and it
will be consideration when she reapplies. The DPSP‘s provide for the State to make
provisions for just and humane conditions at work along with maternity relief. Manisha files
a petition against the factory stating that the state has failed to perform its duty of making
provisions for maternity relief. Decide.

(a) Manisha will succeed the State is bound by the duty of providing maternity relief.

(b) Manisha will not succeed as the State is not mandatorily required to provide maternity
relief to her.

(c) Manisha will succeed as the factory even if owned by a private individual but is under the
control of the State.

(d) Manisha will not succeed as the owner is a private person and DPSP is only enforceable
against the State.

. Heena is a widow seeking an employment opportunity on State Bank of India on the ground
of her being the sole bread winner in the family. Heena‘s husband was the employee of the
aforementioned bank at the time of his death. The bank rejects the application of Heena on
the ground that she fails to fulfill the minimum educational qualifications required for the job.
Heena files a petition in the court on the ground that the State has failed to perform its duty to
secure employment enshrined in DPSP‘s. Decide.

(a) The petition will succeed because she can demand employment on compassionate grounds
that the husband was the employee at the time of his death.

(b) The petition will succeed as the State has the obligation to secure employment for its
citizens.

(c) The petition will not succeed as she does not meet minimum educational qualifications.

(d) The petition will not succeed as DPSP‘s are not enforceable.

. Based on the information provided in the passage, decide the nature of directive principles
of state policy from the following:

(a) Judicially acknowledgeable

(b) Only Socialist

(c) Socio-economic

(d) All of the above

3
Passage (Q.68-Q.): The menace of lynching, with disproportionate targeting of Muslims and
Dalits, is a grim reminder of the fair distance that Indian democracy still has to traverse to
realize the promise of ‗constitutional citizenship‘ – in which one's identity is irrelevant to the
realization of rights and equal protection of the law. Apart from the majoritarian backlash,
another index for testing the equal citizenship claim is the state of civil and personal liberties
in the nation, in particular the freedom to dissent. This claim was tested when the State
arrested five human rights activists and critics of the State – calling them ‗Urban Naxals'.
These human rights activists had substantial experience working with marginalized and
disadvantaged communities. Further, they had often been critical of the government in the
past. This sudden arrest by the Pune Police was seen as an attempt to freeze dissent by the
heavy hand of state machinery. In response, five eminent citizens filed a Public Interest
Litigation (PIL) case in the Supreme Court, challenging the arrests and seeking a court-
monitored probe into the investigation. The Court in a 2:1 judgment in Romila Thapar v.
UOIrejected the plea for a Special Investigation Team (SIT) to probe into the investigation,
on the ground that the State had adduced sufficient evidence for the possibility that they are
members of a banned terrorist organization, CPI (M). Note that the petitioners were not
allowed to scrutinize this evidence, as it was submitted in sealed covers – only the judges
viewed it. The lone dissenting judge, DY Chandrachud, called for a court-monitored probe as
he recounted various procedural lapses in the arrest process, signalling States‘ selective
targeting of critics. This case forces one to re-examine the fragile nature of speech protection
when it collides with state power. The standards of proof, required for successful conviction,
need not be met to justify a call for a probe at initial stages. A prima facie case is sufficient to
merit investigation. Further, should the power asymmetry between citizens and the State not
be factored in, when such brazenness is shown in arresting dissenters and critics? Rather than
legitimizing sealed cover jurisprudence, shouldn‘t the Court critically assess the government's
account of the facts?

68. As per the author of the above passage, which of the following is not the index for testing
the equal citizenship claim?

(a) Disproportionate majoritarian backlash on the marginalized community

(b) The state of civil and personal liberties in the nation available to its citizenry.

(c) A balance between free speech and state power, tilting towards the latter.

(d) The state of freedom to dissent available to the citizens.

. As per the claim in the above passage, which of the following might be the reason behind
the arrest of five human rights activists and critics of the State?

I. The human rights activists had substantial experience working with marginalized and
disadvantaged communities.

II. The human rights activists have been critical of the government in the past.

III. The state by arresting them is trying to curb dissent by the heavy hand of state machinery.

(a) Only I and II

(b) Only II and III

(c) Only III

(d) All I, II and III

. As per the above passage, what was the reason in Romila Thapar v. UOIrejected the plea for
a Special Investigation Team (SIT) to probe into the investigation? I. The State had adduced
sufficient evidence for the possibility that they are members of a banned terrorist
organization, CPI (M). II. The petitioners were not allowed to scrutinize this evidence, as it
was submitted in sealed covers – only the judges viewed it. III. There were various
procedural lapses in the arrest process, signalling States‘ selective targeting of critics.

(a) Both I and II

(b) Both I and III

(c) Only I

(d) Neither I, II or III


 

. As per the above passage, the arrest of five human rights activists does not shed light on
which of the following prevailing situation in the nation?

(a) The nature of speech protection becomes fragile when it collides with state power.

(b) The jurisprudence that allows a prima facie case to be sufficient to merit investigation
should be reviewed.

(c) The Court should have critically assessed the government's account of the facts before
rejecting the formation of an SIT.

(d) That the CPI(M) has branched out its membership despite a ban by the central
government.

. Which of the below given statements is the most appropriate justification to the concept of
‗constitutional citizenship‘ as described in the above passage?

(a) Granting of citizenship is based on the person‘s identity.

(b) One's identity is irrelevant to the realization of rights and equal protection of the law.

(c) The realization of rights is intricately connected with the identity.

(d) The state of civil and personal liberties in the nation, in particular the freedom to dissent.

Passage (Q.-Q.): According to the IPC, Section 420 states that whoever cheats and thereby
dishonestly induces the person deceived to deliver any property to any person, or to make,
alter or destroy the whole or any part of a valuable security, or anything which is signed or
sealed, and which is capable of being converted into a valuable security, shall be punished
with imprisonment of either description for a term which may extend to seven years, and
shall also be liable to fine. Under IPC, section 420 the offence is cognizable and non-
bailable. There are few essential elements under section 420 of IPC. Let us understand that- ·
Cheating The term "cheating" has been defined under Section 415 of the Indian Penal Code.
The element of cheating must be present in every offence under Section 420 of I.P.C. Section
415 of IPC states that Whoever, by deceiving any person, fraudulently or dishonestly induces
the person so deceived to deliver any property to any person, or to consent that any person
shall retain any property, or intentionally induces the person so deceived to do or omit to do
anything which he would not do or omit if he were not so deceived, and which act or
omission causes or is likely to cause damage or harm to that person in body, mind, reputation
or property, is said to "cheat". · Acting dishonestly Section 24 defines that what is ―acting
dishonestly‖. When the doing of any act or not doing of any act causes wrongful gain of
property to one person or a wrongful loss of property to a person, the said act is done
dishonestly. · Property The word property may basically be defined as all things which can be
measured in terms of money. The said thing should be capable of being possessed by a
person for the exclusive use or enjoyment as owner of that thing. · Fraudulently Section 25
defines the term "Fraudulently". It says that a person is said to do a thing fraudulently if he
does that thing with intent to defraud but not otherwise. · Mens rea Mens rea is a legal phrase
which used to define the mental state of a person while committing a crime and that should
be intentional. It can refer to a general intent to break the law or a specific prearranged plan
to commit a particular offense. A criminal prosecutor must show beyond any reasonable
doubt to convict an accused person that the suspect actively and knowingly contributed in a
crime that affected another person or their property. · How Cheating is to be proved It must
be shown that there is a failure of the promise which was made. It must be shown that there
was no effort on the part of accused to perform his promise. The test of prudent man must be
applied to appreciate the evidence on record.

. A hires B to renovate his house, and pays a certain amount of money in advance as
consideration for the same. Due to a strike by the transport department B is unable to bring in
the material as promised to A files a case against B for cheating. Will he succeed?

(a) Yes, as B induced him into thinking he will work and then later did not finish it.

(b) Yes, as B, induced him into giving away valuable property, and later defrauded him.

(c) No, as B did not have an intention to cheat, he merely couldn‘t fulfill he contractual
obligation.

(d) No, as B did not take entire sum of consideration against his work, so he did as he was
paid.

. Mister Z and Mr. Y are in a contract wherein the former has promised to create a certain
number of products for the latter and have taken the consideration sum against the same.
Mister Z later, pretends to have completed part of the performance. Has Mr. Z cheated? 

(a) Yes, as he knowingly gave false pretense to induce money under dishonest means. 

(b) No, as he only breached his contract and did not cheat Y.

(c) Yes, as his intention was to lie.

(d) No as his intention was not to deceive, he just failed in his duty.

. A sells and conveys an estate to B. A, knowing that in consequence of such sale he has no
right to the property, sells or mortgages the same to Z, without disclosing the fact of the
previous sale and conveyance to B, and receives the purchase or mortgage money from Z. Is
this cheating?

(a) No, as it‘s the duty of the purchaser to ensure everything about the property.

(b) No, as Z should have been cautious before choosing A as his representative.

(c) Yes, as A misrepresented himself in position, to induce property.

(d) Yes, as B and Z both got defrauded out of their property, and hence both should sue him.

 
. Raj high school student pretends to be older than his age to enter into a club to attend a
concert of his favorite band, ' The Rockstar'. Later his ID is discovered to be fake, and he is
asked to leave the property immediately. Has Raj cheated?

(a) Yes, as he misrepresented himself for dishonest intention.

(b) Yes, as he dishonestly tried to make a wrongful gain.

(c) No, as he did not have any wrongful gain intended out of it.

(d) No, as all the elements of cheating have not been fulfilled.

. Mr. X. pretending to be a civil Servant at a high post, get a discount at the ration shops near
his house. Can Mr. X be accused of cheating?

(a) No as Mr. X can be accused of corruption not of cheating.

(b) No, as what Mr. X discount on certain goods cannot be termed as property.

(c) Yes, as Mr. X deceived someone for wrongful gain of property.

(d) Yes, as no civil servant gets discount on ration anyway.

. An Educational Institute by the name of law-walas started the 3 and 5-year law courses, for
students after their 12th Grade, but were not registered with any governmental recognition.
Rujhan, a graduate of this institute, unknowing of the same, applied for a job with an
esteemed law firm. Seeing his graduation degree in the resume, they filed a case of cheating
against him. Will it sustain?

(a) Yes, as he applied for a job with a fraudulent degree pretending to be a law graduate.

(b) No, as he did not have the intention or knowledge of his degree being illegal.

(c) Yes, as he can then further sue the college for his reimbursement and for cheating.

(d) No, as the company should sue the college but not rujhan.

Passage (Q.-Q.): An act will be called insider trading if such information is provided to a
third party who benefits from his investment activities post receiving such information. It is a
term used commonly in the securities market and usually relates to illegal conduct. However,
insider trading can be both legal and illegal. In generic terms, insider trading means buying
and selling of stocks and shares based on significant information which is publicly not
available. Apart from creating a biased field which disadvantages other investors, insider
trading by corporate employees breach their utmost responsibility to work in the best interest
of the shareholders. The legality of insider trading depends on the time when the insider's
official trades the information. Analysing a global perspective, in most countries, insider
trading is not illegal if the information provided by key personnel of a company in a way
which does not allow an individual to take advantage of insider information. Importantly, the
higher officials of a company are not restricted to trade in their company's shares as contrary
to the general rules; it would be unfair to prevent these officials making company's
significant decision from investing in stocks.Not only directors can be convicted under such
an offence, but the law can also penalize the brokers as well as the family members to misuse
the information. The term ‗insider‘ has been defined under Regulation 2(e) of SEBI
(Prohibition of Insider Trading) Regulations, 19. Basically, the term ‗insider‘ can be
classified into three broad categories, which are: · Persons who are connected to the
company, · Persons who were connected with the company, · Persons who are deemed to be
connected to the company. In order to become an insider a person has to fulfil three elements,
viz; · The person should be a natural person or legal entity; · The person should be connected
person or deemed to be connected; · Acquisition of the unpublished price sensitive
information by virtue of such connection. Unpublished price sensitive information means any
information which refers to the internal matters of the company and ordinarily it is not
disclosed by the company in the regular course of the business. In India, SEBI (Insider
Trading) Regulation, 19 framed under Section 11 of the SEBI Act, 19 intends to curb and
prevent the menace of insider trading in securities. Section 15(G)(i) of SEBI act 2002 says, if
an insider either on its own or on behalf of any person has dealt on behalf of his company any
unpublished information then he may be fined with RS. 25 crores or 3 times the profit made,
whichever is higher.

. There was a company named Manas Ltd., involved in Telecom business. Manas Ltd. had a
manager named Shivani, her work involved communicating with investors, bankers, other
companies, etc. Recently, Manas Ltd got an offer from Agrawal Ltd for a merger, which was
beneficial for both the companies. It was decided that this merger would be kept confidential
for some time, CEOs of both the company and Shivani knew of this merger as of now. After
a week, Manas Ltd issued a prospectus issuing shares next month and it was also decided that
information about the merger would be made public after the allotment of the shares.
Meanwhile, Shivani accidentally tells about the merger to her friend Rachna and how this
will make buying of shares more beneficial, who rely on this buy shares worth Rs 10,00,000
of Manas Ltd. Later, the merger does not happen and therefore, Rachna does not benefit
anything from buying the share. Can this be termed as insider trading? Should Shivani be
held liable for this? Choose the option based on the based-

(a) Yes because Shivani fulfils all the criteria to be an insider and provided with sensitive
non- public information to Rachna.

(b) No because Shivani gave the information accidentally which shows there was no intention
on her side and thus not liable.

(c) No because Rachna did not benefit from buying the shares post receiving the information.

(d) Yes because non- public information was released which would have benefitted Rachna
had the merger happened. Thus, the actual benefit is irrelevant and Shivani is liable.

. Now assume that the merger did happen and Rachna gained on her investment made relying
on his. Is this insider trading- Will your answer change from above and why?
(a) Yes, now it is insider trading because Rachna has benefitted from the information. 

(b) No because in both the cases it was insider trading, the benefit is irrelevant the
information should be sensitive and non-public.

(c) No, in both the cases the information was released accidentally and lacks the requisite
intention.

(d) Yes because earlier the merger did not happen and thus the information was incorrect.
Now the merger makes the information correct and sensitive. Thus, amounting to inside
trading.

. There was a company named Sarthak Wadhwa, primarily involved in the Banking sector.
Recently, Sarthak Wadhwa decided to start the business of manufacturing and selling cement.
This was a profitable business which would boost the profits of the company, thus increasing
the demand for its share. It was decided thatthis would be kept confidential for the time
being. All the preparations of new business were carried out by Mr. Aarohi, the general
manager of Sarthak Wadhwa. However, Mr. Aarohi was the least trusted employee and was
fired fearing he would reveal this to others, but he had no intention to do so. On being fired,
Mr. Aarohi to take revenge told this to the rival bank Sanyukta Fazudar, who took advantage
of this information and themselves started the business of manufacturing and selling cement
before Sarthak Wadhwa and earned huge profits. Can Mr Aarohi be termed as an insider?
Answer based on the three elements-

(a) Yes, because he fulfils all the three elements.

(b) No, because he does not fulfil the three elements.

(c) Yes because he provided with a third party who benefits from his investment activities
post receiving such information.

(d) No because Aarohi has left the job and is no longer connected to the company.

. In the above question, can this be termed as insider trading. Which among the following is
correct based on the above passage-

(a) No because Aarohi is not an insider.

(b) Yes because Sanyukta Fazudar benefitted from the sensitive information provided by Mr
Aarohi.

(c) No because insider trading is limited to securities market only.

(d) No because for insider trading investment must be made in the same company whose
information is being traded and that results in an unfair benefit.

 
. Suppose that Aarohi is not fired and while working there informed her friend Meghana at
Sanyukta Fazudar Company about Sarthak Wadhwa‘s new business. Meghana immediately
invests in Sarthak Wadhwa. After one day, Sarthak Wadhwa makes the information public.
Now, when the cement manufacturing business starts, the company earns lots of profit and its
share price rises. Meghana also earns a huge return on this. Will this amount to insider
trading-

(a) Yes because Meghana unfairly benefitted from investing based on the information.

(b) No because the information was no longer sensitive and was public knowledge.

(c) No because Aarohi is not an insider.

(d) Yes but in this situation insider trading is legal.

. In the above factual matrix, who is liable for insider trading based on the passage-

(a) No one because no insider trading occurred.

(b) Aarohi

(c) Meghana

(d) Both (b) and (c).

Passage (Q.-Q.): The men's rights activists claim that the anti-dowry laws are being
frequently misused to harass and extort husbands. The high rate of suicide among married
men in India is also attributed to harassment under these laws by the activists. The practice of
giving dowry was first criminalised in 19 under the Dowry Prohibition Act, 19 and later the
Section 4A of the Indian Penal Code was introduced in 19. The Section 4A of the Indian
Penal Code which deals with cruelty to a wife states that whoever, being the husband or the
relative of the husband of a woman, subjects such woman to cruelty shall be punished with
imprisonment for a term which may extend to three years and shall also be liable to fine. For
the purposes of this section, "cruelty" means:

(a) Any wilful conduct which is of such a nature as is likely to drive the woman to commit
suicide or to cause grave injury or danger to life, limb or health (whether mental or physical)
of the woman; or (b) harassment of the woman where such harassment is with a view to
coercing her or any person related to her to meet any unlawful demand for any property or
valuable security or is on account of failure by her or any person related to her to meet such
demand. The Section 113-B of the Indian Evidence Act, 18 says that if a married woman
committed suicide within seven years of marriage, it must be assumed by the court that her
husband and his family abetted the suicide, especially if there was evidence of prior dowry
demands. Until July 2014, the Section 4A of the Indian Penal Code allowed the police to
arrest the persons mentioned in the complaint without a warrant or any investigation. The
crime was non-bailable, so chances of getting bail are low and husbands usually lost their
jobs while in custody. On 2 July 2014, the Supreme Court of India in an order stopped
automatic arrests under Section 4A. The Court directed the police to use Section 41 of the
Code of Criminal Procedure, 19, which contains a checklist, to decide whether an arrest is
necessary. The Court also stated that in all arrests the magistrate must examine whether
further detention was necessary. There is also no provision of withdrawing a complaint in
case of a reconciliation. However, an amendment to rectify this has been proposed. Former
justice of Delhi High Court Shiv Narayan Dhingra has admitted that this law is being used by
women to harass their husbands and in-laws. He has claimed that these laws assume that
women are always truthful, and don't place much importance on evidence. He has termed
misuse of these laws, legal terrorism.

. Sachin married Roma six years back and they have a 4-year-old son. One day, out of the
blue, Roma leaves her husband‘s home and files a dowry harassment case against her
husband and his relatives including the mother-in-law. Sachin and his mother get arrested.
Sachin‘s mother is years old and is an asthma patient. He is trying hard to get her out on bail.
Based on the information provided in the passage, would the police consider Sachin‘s plea
following Section 4A?

(a) No. Section 4A of the Indian Penal Code is a non-bailable offence.

(b) Yes. Under severe health problems, some rebates can be provided.

(c) No. Arrests couldn't have happened without any evidence.

(d) Yes. Under the law, senior citizens are compensated for barring until proven guilty.

. Sushma filed a police complaint against her husband, sister-in-law, and father-in-law under
the Dowry Prohibition Act. Sushma‘s husband, Pradeep, is a clerk with a private bank.
Pradeep‘s sister is to appear in class 12 board exams, and his father is mostly ill and
bedridden. According to the information provided in the passage, how the law allows the
police to act on the complaint?

(a) Police should go for an immediate arrest.

(b) Police should launch an investigation to verify if the complaint is true.

(c) Police should not arrest them if not necessary.

(d) This is a family matter. Police should refer it to the family court.

. Aman married Roma six years back. One day, out of a misunderstanding, Roma leaves her
husband‘s home and files a dowry harassment case against her husband and his relatives
including the mother-in[1]law. A couple of days later, Roma returns to the police and
requests them to dismiss her complaint. She says that it was all a misunderstanding. Based on
the information provided in the passage, how does the law allowpolice to act on Roma‘s
request?

(a) Police should release the accused.

(b) Police should arrest Roma for filing a false complaint.


(c) Police should investigate the matter to check if Roma is being pressurised.

(d) Police should report it to the magistrate and await orders.

. According to the information provided in the passage, in which of the following cases, a
case could not be registered under Section 113-B of the Indian Evidence Act?

(a) If a woman married for 5 years files a complaint of dowry harassment by her in-laws
without evidence.

(b) If a woman having a 5-year-old child files a complaint of dowry harassment by her in-
laws.

(c) If a woman married for 8 years commits suicide at her in-laws‘ place.

(d) If a pregnant woman commits suicide at her in-laws‘ place.

. Sushma filed a police complaint against her husband, sister-in-law, and father-in-law under
the Dowry Prohibition Act. Sushma‘s husband, Pradeep, is a clerk with a private bank.
Pradeep‘s sister is to appear in class 12 board exams, and his father is mostly ill and
bedridden. Police sprang into action and arrested the accused immediately. As a result of
which, Pradeep lost his job, his sister failed in exams and went into depression and his
father‘s health worsened. Pradeep took this matter to the Supreme Court saying that the
police did not follow the guidelines, hence, the responsible officers should be punished.
According to the information provided in the passage, what guidelines did the police fail to
follow?

(a) The police failed to follow section 41 of the Code of Criminal procedure as per court‘s
guidelines.

(b) The police failed to launch a primary investigation to check if the complaint held any
weight.

(c) Under the section 113b of the Evidence Act, the police failed to take a senior citizen‘s
health under consideration.

(d) As per the Supreme Court‘s ruling, the police failed to consult a magistrate first.

. Garima had a disagreement with her husband, Sanjay, and filed a police complaint against
her husband and sister-in-law under the Dowry Prohibition Act. Sanjay claims that it is a
false complaint meant to harass and extort money from him. He wants to challenge his wife‘s
testimony in the court. According to the information provided in the passage, what is
Sanjay‘s option to make his argument considerable in the court?

(a) He can file a petition for bail depending on the turn of events.

(b) He can ask the cops to report it to the magistrate.


(c) He can file a petition to dismiss the case.

(d) He cannot do anything.

Passage (Q.-Q.): Medical negligence also known as medical malpractice is improper,


unskilled, or negligent treatment of a patient by a physician, dentist, nurse, pharmacist, or
other health care professional. The liability of the doctor cannot be invoked now and then and
he can‘t be held liable just because something has gone wrong. For fastening the liability,
very high degree of such negligence was required to be proved. A doctor or a medical
practitioner when attends to his patients, owes him the following duties of care: · A duty of
care in deciding whether to undertake the case · A duty of care in deciding what treatment to
give · A duty of care in the administration of the treatment In some situations the complainant
can invoke the principle of res ispa loquitur or ―the thing speaks for itself‖. In certain
circumstances no proof of negligence is required beyond the accident itself. The principle of
res ipsa loquitur comes into operation only when there is proof that the occurrence was
unexpected, that the accident could not have happened without negligence and lapses on the
part of the doctor, and that the circumstances conclusively show that the doctor and not any
other person was negligent. There is a lot of uncertainty in medical profession, therefore in
situations where there is risk involved in surgery, it is the responsibility of the surgeon/
doctor to inform the patient about the same. Once the patient gives consent to that risk, then
he cannot claim for negligence unless any accident has happened because of any negligent or
irresponsible act committed by the doctor. A doctor is not an insurer for the patient, inability
to cure the patient would not amount to negligence but carelessness resulting in adverse
condition of the patient would. Doctors in India may be held liable for their services
individually or vicariously unless they come within the exceptions specified in the case of
Indian Medical Association vs V P Santha i.e. they treat patients for free. In medical
profession there may arise a situation where the doctor needs to make a complicated yet
important decision in order to save the life of the patient. In such situation the doctor cannot
be accused of negligence for any incident which occurs despite of taking all the necessary
precaution and care. Even after adopting all medical procedures as prescribed, a qualified
doctor may commit an error. The National Consumer Disputes Redressal Commission and
the Supreme Court have held, in several decisions, that a doctor is not liable for negligence or
medical deficiency if some wrong is caused in her/ his treatment or in her/ his diagnosis if
she/ he has acted in accordance with the practice accepted as proper by a reasonable body of
medical professionals skilled in that particular art, though the result may be wrong. In various
kinds of medical and surgical treatment, the likelihood of an accident leading to death cannot
be ruled out. It is implied that a patient willingly takes such a risk as part of the
doctor[1]patient relationship and the attendant mutual trust.

. Suresh went to a doctor for his treatment of asthma. The doctor prescribed him appropriate
medicines and started his treatment accordingly. Despite the full treatment, there was very
little improvement in his condition. He wanted to sue the doctor for medical negligence.
Decide.

(a) Yes because is the duty of the doctor to cure the patients.

(b) No because the doctor had adopted the appropriate course of treatment.
(c) Yes because the doctor did not give proper treatment to the patient.

(d) No because the patient did not take reasonable care

. Ram went to doctor for a liver surgery. The doctor informed ram of the danger and ram gave
his consent to it. During the operation the doctor by mistake cut his pancreas which resulted
in lot of blood loss. The pancreas was damaged and required another operation. Ram wanted
to sue doctor for medical negligence. Decide.

(a) Yes because the consent given was not for doctor to commit mistake.

(b) No because ram gave consent to the danger that existed in the operation.

(c) No because it was during the course of operation and was unavoidable.

(d) No because the doctor took all the precautions and care.

. Ajay was admitted in the city hospital because of Ill health. The senior doctors without
testing gave him medicines of Flu and fever. But his condition got worse. He was shifted to
Max hospital where he immediately died. They told his parents that he was suffering from
typhoid. Can the parents sue the doctors of city hospital for negligence?

(a) Yes, because they did not try to save Ajay.

(b) Yes, because they gave medicines without testing, this was their first duty.

(c) Yes, because they were senior doctor and should know about the symptoms of typhoid.

(d) No, because the symptoms of flu and typhoid are very similar and the doctor gave him
proper treatment in their part.

. Amir met with an accident and was taken to National Hospital. He was in a very severe
condition and was unconscious. During surgery the doctors found that his one kidney is fully
damaged and is causing severe damage to his other organs. Therefore they removed his one
kidney and saved his life. Can Amir sue the doctors for removing his kidney without his
consent?

(a) Yes because a person has right over his body and the decision was taken without his
consent.

(b) Yes, because the doctor should have tried to save his kidney which is his first duty. 

(c) No, because doctor had to make a tough choice in order to save the patient's life 

(d) No, because a person whose life is saved by the doctor cannot sue him

. Priya was admitted in a private hospital. She was treated by a junior doctor who by mistake
gave her wrong medicines which affected Priya's Health. Priya wanted to sue the hospital for
negligence. Will she succeed?
(a) Yes, because it was a private hospital and can be sued.

(b) Yes, because the doctor was the employee or the staff of the hospital and the hospital can
be sued for his actions

(c) No, because it was an individual act of negligence, therefore the doctor alone can be sued.

(d) No, because it was not a mistake that any doctor could have made.

. Can a doctor be sued if one of his patient dies despite of giving proper treatment and
medicines?

(a) Yes because it is the duty of the doctor to save the life of his patients.

(b) No because the doctor followed his duty without any mistake.

(c) Yes because the doctor might have made some mistake in the treatment.

(d) No because everyone has to die one day.

Passage (Q.-Q.): Article 15 occurs in the Constitution of India under Part III that with
fundamental rights of the citizens of India. Further article 14 to18 deal with various issues
related to Right to Equality. Fundamental rights are applicable to all the citizens of the
country universally is respective of race, gender, place of birth, caste, religion. In case of
citizens of India comes across a case of violation of fundamental rights, he or she can
approach the Supreme Court of India for ultimate justice. The fundamental rights mentioned
in the Indian Constitution have their origin from different sources like England's Bill of
rights, the United States Bill of Rights and France‘s Declaration of the Rights of Man. Article
15 (1) and (2) proscribe the nation from discriminating any Citizen on the basis of any one or
many of the aspects such as religion, race, caste, sex, place of birth and others. These articles
make room that there will not be any restriction whatsoever to any person on the above
ground in order to access and make use of public facilities and amenities like shops,
restaurants, places of entertainment, and others. No person in India can be denied the use of
tanks, wells, bathing ghats, roads and sides of public resort that are created and managed fully
or in part from the state funds or have been dedicated for the use of the general public.
Starting from article 15 (3), the constitution of India attempts to deal with protective
discrimination. Article 15 (3) makes it possible for the state to create special provisions for
protecting the interest of women and children. Article 15 (4) capacitates the state to create
special arrangements for promoting the interests and welfare of socially and educationally
backward classes of the society such as SC and STs. 25 of 35 Article 15 (5) moves a step
ahead and empowers the country to make reservations with regard to admissions in the
educational Institutes both privately run and those that are aided or not aided by the
government. From this rule, only the minority-run institutions such as the Madarsas are
exempted. Therefore, we can say that article 15 is the foundation stone for making
reservations in the country.

. Alex wants her son to study in a school run by a Hindu organization that focuses on Vedic
studies. The school denies admission on the grounds of her religion, which is Christianity.
Alex thinks this is unfair and decides to stand against this bias. According to the information
provided in the passage, where should she file her complain?

(a) Alex should report this to the magistrate of the town.

(b) Alex should file a petition in the Supreme Court of India.

(c) Alex should raise a complaint with the Minority Welfare Department.

(d) The school is run by private funds. Therefore, Alex has no right to challenge the decision.

. Mr and Mrs Singh are celebrating their 25th wedding anniversary. They went to an upscale
restaurant in South Delhi but were denied entry because the husband and wife were wearing
Kurta-pajama and sari respectively. The restaurant says that they have a strict dress code
policy - a blazer or tux for males and a long gown or single piece dress for women. But Mr.
Singh took believed it to be a violation of article 15 and dragged the restaurant in the court of
law. However, his petition was outright rejected. Now, based on the information given in the
passage, can you tell why this happened?

(a) Because there was no violation of article 15 or any other law.

(b) Because this was a case of violation of article 14. Mr Singh made a wrong judgement.

(c) Because Mr Singh reported it in the high court. He should have moved to the supreme
court directly.

(d) None of the above.

. For better English education, Amit Kumar, who falls under a reserved category, wants his
son to study in a school run by the Indo-Roman Catholic Organization, a minority-run school.
The school, however, denies admission on the grounds of caste. Amit Kumar files a petition
in the Supreme Court against the act of the school, but it gets denied. According to the
information provided in the passage, can you tell why his petition was denied by the apex
court?

(a) Because he should have filed a police complaint first.

(b) Because he should have reported this to the education ministry first.

(c) Because the Supreme Court has more important issues to address.

(d) Because, by the law, the school had the right to do so.

. According to the information provided in the above passage, can you tell what does article
15 (3) allow?

(a) Caste-based reservation in higher education.


(b) Reservation of seats for women in the local bodies.

(c) Access to shops and hotels for all.

(d) Access to religious places regardless of gender.

. Rajan filed a petition in the Supreme Court claiming that by law, the allocation of seats for
OBC in higher education should not be more than 9%, but the government of Rajasthan
passed a bill to raise it to 12%. This is unconstitutional and should be scrapped. But the
Supreme Court rules the petition to be non[1]judicial. Can you tell why the Supreme Court
did this?

(a) Because by the law, the state government can do that.

(b) Because parliamentary bills cannot be challenged in the court of law.

(c) Because the bill should have been challenged in the high court.

(d) Correct reason is not mentioned in the list.

. According to the information given in the passage, which of the following does not qualify
as a violation of any of the provisions of article 15?

(a) Laxmi denied entry in Swami temple because the priest suspected she is on her periods.

(b) Ramesh was told to keep out of an eatery because he works as a scavenger.

(c) Aparna was forced to be a housewife even after being highly qualified.

(d) Roshni being denied a promotion even after being the best and deserving candidate as she
was woman.

4
Passage (Q.-Q.): Mistake may operate upon a contract in two ways. It may, firstly, defeat the
consent altogether that the parties are supposed to have given, that is to say, the consent is
unreal. Secondly, the mistake may mislead the parties as to the purpose which they
contemplated. Where both the parties to an agreement are under a mistake as to a matter of
fact essential to the agreement, the agreement is void. However an erroneous opinion as to
the value of the things which forms the subject –matter of the agreement, is not deemed a
mistake as to a matter of fact. An agreement upon the same thing in the same sense is known
as true consent or consensus ad idem, and it is the root of every contract. Two or more person
are said to consent when they agree upon the same thing in the same sense. A contract is said
to be void because of mistake when: · Both the parties to an agreement are mistaken, · Their
mistake is as to a matter of fact, and · The fact about which they are mistaken is essential to
the agreement. A contract is not void because it was caused by a mistake as to any law in
force in India; but a mistake as to a law not in force in India has the same effect as a mistake
of fact. Mistake as to identity occurs when one of the parties represents himself to be some
person other than he really is. There can be a mistake of identity only when a person bearing
a particular identity exists within the knowledge of the plaintiff, and the plaintiff intends to
deal with him only. If the name assumed by the accused is fictitious there will be no mistake
of identity.

. A, being entitled to an estate for the life of B, agrees to sell it to C. B was dead at the time of
the agreement, but both parties were ignorant of the fact. Decide the validity of the contract?

(a) The agreement is not void since B was alive when the contract was formed.

(b) The agreement is not void since the essential condition of the agreement was an estate and
not the presence of B.

(c) The agreement is valid and enforceable.

(d) The agreement is void since both the parties were ignorant of the death of B, which was
an essential fact of the contract.

. A person called Hillary, brother of Ron, represented himself as Ron, and thereby induced a
Government‘s agent to contract with him. The government agent intended to contract only
with Ron and not with Hillary and Ron knew this. The government‘s offer was meant for Ron
and Hillary posing as Ron accepted it. Decide whether the consent of government‘s agent is
valid or not?

(a) The consent is valid since the government agent could have easily figure out that it was
Hillary and not Ron.

(b) The consent given by the government‘s agent is not valid since he was deceived by the
brothers

(c) This is a valid contract since the party to a contract is not an essential fact.

(d) The contract is not void since the brother belongs to same family and contracting with
either of the brothers does not affect the conditions of the contract.

. Jethalal had taken over the business of one Brocklehurst. Heeralal used to deal with
Brocklehurst and not knowing of the change sent him an order for certain goods. The order
was received by Jethalal who sent the goods. Heeralal came to know of the change only when
he received an invoice and by that time he had already consumed the goods. Heeralal had a
set-off against Brocklehurst and, therefore refused to pay the price. Jethalal sued him:

(a) Heeralal is liable to pay since he consumed the goods and hence bound to pay.

(b) Heeralal is not liable to pay since he never contracted with Jethalal.

(c) Heeralal is not liable since his consent is not valid because he never consented to a
contract with Jethalal.

(d) Heeralal is liable to pay since the party to the contract is not an essential condition.
. A man called North, entered the Jeevan‘s shop and selected some pearls and some rings
worth Rs. 3000. He produced a cheque book and wrote out a cheque for the amount. In
signing it he said: ‗You see who I am, I am Sir George Bullough‘ and finding on the
reference to a dictionary that Sir George Bullough lived at the address mentioned, Jeevan let
him have a ring. But before the fraud was discovered he pledged the ring to Shyam, who
advanced money bona fide, and without notice. Jeevan sued Shyam for the ring or its value.
Decide whether Shyam is liable or not?

(a) Jeevan will neither get the ring nor its price since he sold it with the greedy intention to
make excessive profit.

(b) North is liable to pay the price of the ring or return it to Jeevan since it was he who
committed the fraud.

(c) Since North himself does not have good title over the ring he cannot sell it to Shyam and
hence Shyam is liable.

(d) Shyam is not liable to return the ring or pay its equivalent price to Jeevan since was
innocent on his part while purchasing it.

. Thomas received orders in writing from a fraudulent man, called Blenkarn. The order paper
had a printed heading: ‗Blenkarn & Co.‖. There was a well know and respectable firm,
named ‗Blenkiron & Co.‖ in the same street. Thomas believing that the orders had come from
this firm, sent a large quantity of handkerchiefs. Blenkarn received the goods and disposed
them off to John, who acted in good faith and purchased them. Decide the right of Blenkarn
over the goods:

(a) Blenkarn has title over the goods even though Thomas never intended to deal with him. 

(b) Blenkarn has no title over the goods since Thomas never intended to deal with him and
hence there was no contract between them.

(c) Blenkarn has title over the goods since he had paid for them.

(d) There is contract between the parties since the essential condition of contract, i.e. delivery
and payment of good is met.

Passage (Q.-Q.): The relevant sections of the IPC are Section 2 covering negligent act likely
to spread infection of disease dangerous to life, Section 2 covering malignant act likely to
spread infection of disease dangerous to life, Section 2 covering disobedience of quarantine.
The penalties for the above offences are simple or rigorous imprisonment extendable up to 6
months or fine or both; simple or rigorous imprisonment extendable up to 2 years or fine or
both; and simple or rigorous imprisonment extendable up to 6 months or fine or both,
respectively. While the offences under Section 2 and 2 are cognisable and bailable, offence
under Section 2 is non-cognisable and non-bailable. This would mean that unlike in bailable
offences where the accused has a right to bail, they would be at the discretion of the court in
seeking a bail. Relating these general provisions to the core theme of the article, it provides a
resounding affirmation that the law does not in any manner authorise the police to use
corporal force. Disaster Management Act, 2005 The first relevant provision of the act is
Section 51. The provision has twin aspects to it. Firstly, it is attracted by the persons who
leave their homes to pursue non-essential work. In the words of Section 51, the conditions
and punishment prescribed are – Condition: Whoever, without reasonable cause (a) obstructs
any officer or employer (b) refuses to comply with any direction. Punishment: Imprisonment
of a term upto 1 year or fine (not prescribed under the provision) or both. Secondly, the latter
part of Section 51, in present conditions, is attracted by persons who are tested positive for
the Coronavirus but run away from quarantine. In the words of Section 51, the conditions and
punishment prescribed are – Condition: Whoever, without reasonable cause (a) obstructs any
officer or employer (b) refuses to comply with any direction causing loss of lives or imminent
danger thereof. Punishment: Imprisonment of a term upto 2 years.  

. Kunjbihari returned from Dubai in February, 2020 and was diagnosed with COVID-19 after
he showed symptoms of the disease. He was admitted to the hospital and recovered within 15
days after which he was sent back home and was required to self-quarantine for the next 14
days. Kunjbihari however, was craving street food and ventured out without knowing that the
disease had recurred. He infected around 250 people with the disease. Decide

(a) Kunjbihari is liable under Section 2 and 2 of the Indian Penal Code

(b) Kunjbihari is liable under Section 2 and 2 of the Indian Penal Code

(c) Kunjbihari is not liable for any offence as he had recovered from the disease

(d) None of the above

. Please refer to the facts mentioned above. Kunjbihari upon his return from Dubai bribed the
police personnel and evaded the mandatory screening for COVID-19 for he suspected a
positive report. He roamed around the entire city of Delhi spreading the infection to about
200 people.

(a) Kunjbihari is liable under Section 2 of the Indian Penal Code

(b) Kunjbihari is liable under Section 2 of the Indian Penal Code

(c) Kunjbihari is not liable for any offence as he would recover from the disease

(d) None of the above

. Apurv returned from the United States. On his way out of the airport, he tried to escape
screening but was caught. When he was screened, he was found positive. However, he broke
his quarantine and ran away. He was booked by the police for offending provisions of the
Indian Penal code. In addition, he was also booked for breaking self-quarantine. Apurv‘s
lawyers approached the court seeking bail and asserted that it was their right to get bail.

(a) Apurv can seek bail as a right for it was his right to get bail under Section 2/2 of IPC

(b) Apurv cannot seek bail as a right for he is a heinous public offender who has caused
significant problems

(c) Apurv can seek bail as a right for he needs medical care at this point in time

(d) Apurv cannot seek bail as a right for he doesn‘t have a right under Section 2 of IPC
. In the wake of the COVID19 outbreak, there has been a lockdown imposed all over the
country. There have been instances of police personnel using innovative methods to enforce
such lockdowns. While some police personnel were spotted getting citizens to do push up,
some others have been spotted getting citizens to clean roads. Decide

(a) The police personnel are justified to impose such punishments for they do not involve the
use of corporal force.

(b) The police personnel are not justified to impose such punishments for they involve the use
of corporal force

(c) The police personnel are justified to impose such punishments for they are required in the
interest of the nation

(d) None of the above

. Please refer to the fact scenario in 6.4. The police personnel were spotted hitting a person
with a lathi after an argument happened when the person refused to head back home during
the lock down.

(a) The police personnel are justified to impose such punishments for they do not involve the
use of corporal force.

(b) The police personnel are not justified to impose such punishments for they involve the
use of corporal force

(c) The police personnel are justified to impose such punishments for they are required in the
interest of the nation

(d) None of the above

. Ajinkya wanted to go out of his home during the COVID-19 lock down to see the fresh
Bangalore Cherry Blossoms everyone was talking about. It happens to be that this was a once
in a 12 year opportunity and missing this chance would mean waiting for another 12 years.
He put on his suit, mask and gloves and set out to leave. The police personnel outside
Cubbon Park stopped Ajinkya. Ajinkya refused to comply. Decide

(a) Ajinkya violated Section 51 of the Disaster Management Act, 2005 for this cause is
unreasonable

(b) Ajinkya did not violate Section 51 of the Disaster Management Act, 2005 for the cause is
reasonable

(c) Ajinkya needs to understand the gravity of the situation

(d) Ajinkya had taken all necessary precautions. Cause cannot be stated to be unreasonable in
that case
 

Passage (Q.-Q.): The term ‗Force Majeure‘ appeared in the common law world in the 10s
and was borrowed from the Napoleonic Code (see Lebeaupin v Crispin [10] 2KB 4),
although its origins can be traced back to Roman law. Under Indian law, one of the first
decisions to deal with the concept of force majeure was the Madras High Court decision in
Edmund Bendit And Anr. vs Edgar Raphael Prudhomme. In this case, the Court cited with
approval the passage from Matsoukis v. Priestman and Co, wherein the definition given by an
eminent Belgian lawyer of force majeure as meaning "causes you cannot prevent and for
which you are not responsible", was adopted. The test for seeking to rely on a force majeure
clause is that the event and the non-performance were due to circumstances beyond a party's
control. Thus, force majeure will not include economic problems like insufficient funds.
Another condition that needs to be fulfilled is that there were no reasonable steps that could
have been taken to avoid or mitigate the event or its consequences. The party seeking to rely
on the clause may also need to show it was not aware, at the time of entering the contract,
that the circumstances giving rise to the event of force majeure was likely to occur. If the
clause refers only to performance of obligations being prevented by the relevant event, then a
party may not be able to rely on the clause if its performance has been made more difficult or
delayed, but not completely prevented (in other words, it can still perform, but it is more
difficult to do so and/or it cannot perform as expected). The fact that it is more expensive to
perform will also not normally entitle reliance on the force majeure clause (so a rise in
underlying costs or expense will not normally be treated as a force majeure event). The fact
that it is more inconvenient and/or more difficult to perform will also not normally entitle
reliance on the clause. If the difficulty was due to circumstances outside a party's control and
was so great that no reasonable person in similar circumstances would be likely to overcome
it, then that might be an exception.

. JBM Environment Ltd. has been asked to construct a solid waste management plant in New
Delhi for power generation in the year 2019. As they seek to move forward with the
procurement of raw material for construction of the plant, the NGT bans all construction
activity in the area due to increasing pollution. JBM stops all work. Decide

(a) JBM can seek the benefit of a force majeure clause for construction was stopped for
circumstances beyond their control i.e. pollution in Delih NCR

(b) JBM cannot seek the benefit of a force majeure clause for they could still have procured
all raw material

(c) JBM can seek the benefit of a force majeure clause for construction was stopped for
circumstances beyond their control i.e. NGT order.

(d) None of the above

. Please refer to facts abovementioned. The construction ban was lifted after 2 months and
JBM sought an extension citing force majeure. They sought for 4 months extension i.e. 2
months for procurement of raw material and 2 months for construction. However, the same
was refused to them by the contracting party. Decide

(a) JBM is entitled to 4 months of extension for force majeure reasons


(b) JBM is entitled to two months of extension for they could have taken steps to mitigate the
delay.

(c) JBM is entitled to no extension for they could have still carried out with the construction.

(d) JBM should drag the party to court.

. SuperNova is an engineering company and were constructing a dam on Teesta river when a
ban was placed on the dam construction due to tribal protest. During the construction ban,
there was a mass exodus of labourers for they had no livelihood left in the area. After the
construction ban was lifted, it took around 1 month for the labourers to return to the area.
Since there were only a limited number of labourers in the city, engaging their service
became quite expensive. SuperNova preferred to wait it out for the labourers to return. When
they couldn‘t meet the deadline, they cited force majeure and sought an extension of 1 month
for the time lost.

(a) SupreNOVA should be given the extension as the delay was caused due to circumstances
beyond their control

(b) SUPERNOVA should not be given extension as the construction ban had now been lifted

(c) SUPERNOVA should be given the extension as cost overruns were a major concern

(d) SUPERNOVA should not be given extension for mere difficulty to perform is not force
majeure

. Please refer to the facts above. During the construction ban, there was mass exodus of
labourers. The mass exodus of labourers meant that there were no labourers left in area and
thus no construction work could be carried out whatsoever for the next 1 month till the
labourers came back. SUPERNOVA upon not being able to fulfill the deadline asked for
extension of 1 month citing force majeure.

(a) SUPERNOVA should be given the extension as the delay was caused due to
circumstances beyond their control

(b) SUPERNOVA should not be given extension as the construction ban had now been lifted

(c) SUPERNOVA should be given the extension as cost overruns were a major concern

(d) SUPERNOVA should not be given extension for mere difficulty to perform is not force
majeure

. BB is an aircraft manufacturing company. They often depend on cheaper services of


mechanical engineers from third world countries for their job. However, in 2020 due to the
suspension of H1B1 Visas, a significant shortage was seen in the influx of such engineers.
The cost due to the shortage had driven upto 10 times. The manufacturing work being
engineer heavy was severely hit due to such high prices and the Aircrafts Manufacturing
Association of New Jersey (of which BB was a member) collectively decided to ban
manufacturing for 2 months feraing cost significant overruns. BB complied too. When they
couldn‘t meet the deadline, they cited force majeure and sought an extension of 2 months for
the time lost.
(a) BB should not be given extension for mere difficulty to perform is not force majeure

(b) BB should be given an extension for everyone else situated in the same position as them
stopped construction.

(c) BB should not be given extension as the construction ban had now been lifted

(d) BB should be given the extension as cost overruns were a major concern

. Please refer to the facts above. Cost of engineers due to the shortage had driven upto 10
times. Multiple sectors were severely hit due to such high prices. The Local Dam Builders
Association of New Jersey passed a resolution collectively decided to ban all manufacturing
for 2 more months fearing cost significant overruns. BB found this as an opportunity to halt
construction and save money. When they couldn‘t meet the deadline, they cited force majeure
and sought an extension of 2 months for the time lost.

(a) BB should not be given extension for mere difficulty to perform is not force majeure

(b) BB should be given an extension for everyone else situated in the same position as them
stopped construction.

(c) BB should not be given extension as the construction ban had now been lifted

(d) BB should not be given extension as they are not situated similarly as the Local Dam
Builders Association of New Jersey.  

(Q.-Q.): Right to Privacy has always been a contentious issue. The COVID-19 pandemic has
brought new considerations to the table. In the coordinated efforts of authorities across the
globe to tackle this pandemic we see a sacrifice of an individual‘s right to privacy. Instances
of a heightened tracking of an individual‘s location, mass surveillance, applications to keep
track of your personal movement, use of travel history have raised pertinent privacy
implications. Various states have tapped phone records, CCTV footages, phone GPS and
published detailed timelines and quarantine lists of the patients and officials. A patient‘s right
to privacy with regard to their personal details was recognized in the case of Mr. X v.
Hospital Z. The court opined that the patient had the right to confidentiality and privacy as
regards their personal details; however, this right was not absolute. Further, the Apex Court
has both in Puttaswamy I and Puttaswamy II crystallized the importance of informational
privacy i.e. data protection. The Court held that any infringement of a right to privacy by the
government must be reasonable and proportionate, and must satisfy the following (i) the
restriction must be effected through a law which pursues a legitimate state aim, (ii) has a
reasonable nexus between the objects and means to achieve them, and (iii) is the least
intrusive means to achieve the state aim. Further, the Personal Data Protection Bill provides
that the consent of the person whose personal data is collected (Data Principal) is necessarily
to be obtained by any person or the government (Data Fiduciary). The personal data collected
must be used only for a specific, clear and lawful purpose for which the consent has been
obtained. The personal data must also be used in a fair and reasonable manner. The PDP Bill
also provides that under emergent circumstances the Data Fiduciary is exempt from taking
the consent of the Data Principal, provided that the collection and processing of personal data
is authorized under law. Though just a Bill, this can be a guiding factor in deciding the use of
a person‘s information. The state at no point in time can act disproportionately even in a
pandemic.

. Mr. Bakliwala is a doctor of immense repute. He is known for his experience and service in
the field of sexually transmitted diseases such as HIV. Abraham approaches Mr. Bakliwala
and gets tested for HIV and the result turns out to be positive. Mr. Bakliwala informs
Abraham that with the advancements in medical science the situation can be sustained if not
completely cured. A few days later a group of researchers come to work under Dr. Bakliwala
and he shares the instances of his patients with the team, which also includes Abraham. In the
end result published, Abraham‘s name is included in the report. Decide:

(a) Abraham’s right to privacy has been violated by Mr. Bakliwala.

(b) Abrahms’s right to privacy has not been violated by Mr. Bakliwala as the same was for
research purposes and shall in turn help the majority of public at large and thus was justified.

(c) Abraham’sright to privacy in such a situation can be curtailed as HIV is a disease which
can have a bearing on the society at large and thus it was important to release the same.

(d) Abraham’sright to privacy has been violated as the information of such a nature is a part
of informational privacy.

. Findcaller is a mobile based application which helps individuals to intercept who is calling
them. It requires individuals to register their details and personal information and processes
the same to intercept calls. On 4th May, 2020 Findcaller accidentally revealed the details of 4
lakh customers on various websites. A probe into this event later revealed that Findcaller was
selling such data to companies who created their databases from such information. Assuming
that the PDP Bill is now an Act, decide whether there was a breach of privacy.

(a) No, there is no breach of privacy as this right is only enforceable against the state.

(b) No, as the selling of data was for commercial purposes and the same is justified on
business grounds.

(c) Yes, as there was a public release of such information. However, only those consumers
whose information has been revealed on websites can contest a breach.

(d) Yes, there has been a breach of privacy on part of Findcaller.  

. The Karnataka State Government has been ramping up its activities to tackle the COVID-19
pandemic. Several steps have been taken to this effect and one of these steps include
preparing a list of people who have been asked to self-quarantine and uploading the same on
the website of the government. This had been duly informed to the individuals as a
precondition of returning to the state from abroad or elsewhere. As soon as the quarantine
period for an individual will end his or her name would be removed from the list. But as a
result of this the individuals whose name was included in the list were ostracized. Will such a
situation amount to violation of right to privacy?

(a) No, such a situation shall not be a violation of the right to privacy as it is in the larger
public interest.
(b) No, such a situation will not be a violation of the right to privacy as the restriction has
been effected through a legitimate law and is in consonance with the goal it seeks to achieve.

(c) Yes, it is a breach of privacy as it was not effected through any legislation passed by the
government, rather it was an arbitrary act of the authorities.

(d) Yes, it is a breach of the right as state cannot restrict the right in such an arbitrary manner.

. Ram is a student who duly fills out the CLET (Common Law Entrance Test Form). He fills
in information of sensitive nature such as his contact details, address, caste etc. into the form
and successfully registers for the same. This is the only form he has filled for colleges. After
a few days Ram starts getting messages from other private universities about their law
program and other coaching institutes about their different courses. He is quite baffled as he
never shared this information with anyone on any platform. He approaches the High Court
contending the violation of Right to Privacy and contends that in all probability his data has
been compromised by CLET. Decide:

(a) His contention is correct as all factors reasonably indicate that the information has been
accessed via the CLET database.

(b) His contention is correct as his personal data was for a specific purpose and such use of
this data by other universities and coaching‘s is not in consonance with law.

(c) His contention is wrong as CLET can give access to other universities in the legal domain
to take information and develop their database. This however should have been duly
communicated to Ram.

(d) His contention is correct because the principles of proportionality have been violated by
the CLET.

. Mr. Bakliwala also finds out that Abrahamis getting married in a few days. He also knows
that Abrahamha not informed anyone about his HIV status and neither does he intend to
inform anyone. Mr. Bakliwala goes on to find Abraham’s fiancée and reveals this
information to her. Abraham says that this is a violation of his right to privacy. Mr. Bakilwala
argued that this was in furtherance of the fiancée‘s right to healthy life under Artcile 21.
Decide:

(a) This is not a violation of right to privacy as this would have had severe impacts on the life
of the wife hence that right i.e. right to health shall outweigh Abraham’sright to privacy.

(b) This is a violation of right to privacy as personal autonomy must be given to Abraham.

(c) This is a violation of right to privacy as this information can have immense impact on
Abraham’s life and thus Abraham should have the right over such information.

(d) This is not a violation as right to a healthy life will outweigh the right to privacy of
Abraham.

Passage (Q.-Q.): Indian hospitals have been held liable for their services- rather lack of
services[1]individually or vicariously. They can be sued for negligence either in Criminal
Courts, Civil Courts or Consumer Forums. The Supreme Court has held that every doctor
―has a duty to act with a reasonable degree of care and skill‖ in the case of State of Haryana
v. Smt. Santra. The liability of a medical practitioner does not merely arise when the patient
has suffered an injury; it arises when the injury is a direct consequence of the conduct of the
doctor when he failed to exercise reasonable care. In other words, a doctor is not liable for
every injury suffered by the patient. First, existence of a duty of care by the doctor towards
the patient has to be established and then the patient must prove breach of such a duty. In
case there was no breach or the injury was not a direct consequence of the breach, the doctor
will not be liable. It was held in Calcutta Medical Research Institute v. Bimalesh Chatterjee
that the onus of proving negligence and the resultant deficiency in service was clearly on the
complainant. The liability of a hospital in cases of medical negligence could be direct or
vicarious. Direct liability in this sense would mean a deficiency in the services provided by
the hospital thus making it unsafe and not suitable for treatment. Vicarious liability, on the
other hand, would refer to the liability of the hospital as an employer for the negligent acts of
its employees. The employer is responsible not only for his acts and omissions but also for
those of his employees, as long as such acts occur within the course and scope of
employment. This liability is based upon the maxims “respondent superior” which means
―let the master answer‖ and “qui facit per alium facit per se” which means ―He who acts
through another does the act himself.‖ An exception to the above principle is seen in the
―borrowed servant doctrine‖ according to which the employer shall not be liable for acts of
an employee when that employee is working under the direct supervision of another
employer.

. Professor and Nairobi conceived a baby through IVF. In her 26th week of pregnancy,
Nairobi developed gestational diabetes. She was prescribed with metformin. After 39 weeks,
Nairobi went into tough labour and gave a birth to baby boy. The baby was born with
permanent physical disability and had to be resuscitated. It was later found that the disability
is a side effect of metformin (a drug that should not be consumed by pregnant women).
Professor and Nairobi decided to sue the doctor. Decide.

(a) The doctor will be held liable as this is a clear case of negligence. It was the duty of the
doctor to inform about the side effects of the drug.

(b) The breach of duty of care by doctor while prescribing a wrong medicine makes him
accountable for medical negligence.

(c) The hospital, along with doctor will be liable for medical negligence.

(d) The couple will not be able to proceed as it was their duty to consult an endocrinologist
for diabetes as it cannot be expected for a gynaecologist to prescribe correct medicine for
diabetes.

. Mr. Batra after getting his heart surgery done from KEM Hospital, alleged that he was
treated badly by the staff of the hospital during his stay at the hospital. He also alleged that
Dr. Mahesh did not possess the required skills for conducting a surgery and when the first
procedure failed, to rectify his mistake he performed a corrective surgery. Hospital and Dr.
Mahesh denied all the allegations against them. As per the given passage, with whom does
the onus of proof lies?
(a) The onus of proof lies with defendant. Therefore, in the present case the doctor and the
hospital management should prove their innocence.

(b) The onus of proof lies with the accused. Therefore, in the present case Mr Batra should
prove negligence.

(c) The onus of proof lies with the plaintiff. Therefore it‘s the duty of Mr. Batra to provide
evidence.

(d) The onus of proof lies with the deponent. Therefore it will be the duty of the Hospital and
Dr. Mahesh to provide evidence.

. When Mrs. Menon went for her spinal cord surgery, Dr. Knight failed to convey her the risk
involved in the procedure nor did he discuss the outcomes of failure of the surgery. After the
surgery, Mrs Menon was left disabled with her lower part of the body. Mrs. Menon wishes to
take your advise. Kindly advise her.

(a) Mrs. Menon will succeed against Dr. Knight as he lacked possession of required set of
skills for a surgeon.

(b) Mrs. Menon will not succeed as she never asked the doctor about the possible ill
outcomes of the surgery.

(c) Mrs. Menon will succeed as Dr. Knight owed a duty of care towards his patients to keep
them well informed and he breached his duty of care resulting in negligence.

(d) Mrs. Menon will not succeed as every surgery has a possibility of failed outcome and this
should not be regarded as a breach of duty at the hands of the doctor.  

. Dr Umang and Dr Samara are the resident doctors at DAIIMS, Bangalore. Both of them are
appointed with the neo natal care unit in the hospital and are assigned the duty to monitor the
babies in the unit who are under critical care. Once on duty, Umang and Samara indulged in
playing ludo and forgot to switch on the oxygen supply, resulting in death of four children. A
case was brought against the hospital decide.

(a) Hospital will not be held liable as the case doesn‘t qualify under vicarious liability.

(b) The hospital will be vicariously held liable as the medical negligence took place during
the course of employment of two doctors.

(c) Dr Umang and Dr Samara will be held liable and not the hospital.

(d) The hospital will be held liable and compensation should be paid by both the hospital as
well as the two doctors.

. Which among the following cases are not justiciable?

i. Mistake done by anesthesiologists resulting in permanent brain damage or death.

ii. A botched procedure involving a large ethanol injection causing a child‘s face and nose to
deteriorate, requiring skin from his forehead and ear to repair it.
iii. Discharging Covid 19 suspect without conducting proper tests at the hospital quarantine
centre.

iv. Person injuring himself after falling in the hospital premise due to wet floor. v. Patient
forgetting that he has already taken his medicine, consumes another dose resulting in high
blood pressure due to over dose of medicine.

vi. An obstetrician in Mumbai fails to perform a C-section in a timely manner, resulting in


serious injuries to the baby.

(a) Only I, II and III (b) Except V, VI, and IV (c) Except IV and V (d) Only I, II, IV and V

. Somu is suffering from kidney failure and requires dialysis every week. When he went to
get his dialysis done at AIIMS hospital, he caught corona virus infection and was later tested
positive for Covid 19. He decided to sue AIIIMS hospital for infecting him with Covid 19.
Decide.

(a) Somu will succeed as AIIMS hospital breached its duty of care by failing to curtail the
infection.

(b) Somu will succeed as AIIMS hospital failed to have separate quarantine centre for Covid
19 patience.

(c) Somu will not succeed as hospitals are breeding grounds of infections and one cannot do
much about it.

(d) Somu will not succeed as him catching corona infection cannot be regarded as breach of
duty on the part of the hospital.

Passage (Q.-Q.): To use insanity as a legal excuse, the defendant has to show that he/she
lacked the capacity to understand that the act was wrong, or the capacity to understand the
nature of the act. The logic of the insanity as a defence goes back to the idea of mensrea and
culpability. The foundation of this law was first laid in the M'Naughton case by the House of
Lords in 13. The basis of the M'Naughtoncase is the inability to distinguish right from wrong.
The basic idea is that some people, under the duress of a mental disorder, cannot control their
actions despite understanding that the action is wrong. Section : Act of a person of unsound
mind.—Nothing is an offence which is done by a person who, at the time of doing it, by
reason of unsoundness of mind, is incapable of knowing the nature of the act, or that he is
doing what is either wrong or contrary to law. So it falls upon the accused to prove his
insanity at the time of offence. It needs to be proved by the accused that because of the
accused's unsoundness of mind, he was incapable of knowing the nature of the act or that the
act was contrary to provisions of law, or was wrong. Infancy: Infancy is a legal incapacity to
be held responsible for a crime due to the age of the perpetrator. There is a legal incapacity
for the crime under seven years of age. DoliIncapaxis a presumption of law which provides
that child has no discretion to distinguish right from wrong, thus criminal intention does not
arise. Section : Act of a child under seven years of age.—Nothing is an offence which is done
by a child under seven years of age. Section : Act of a child above seven and under twelve of
immature understanding.—Nothing is an offence which is done by a child above seven years
of age and under twelve, who has not attained sufficient maturity of understanding to judge of
the nature and consequences of his conduct on that occasion. The essential ingredients of
Sections and are that children under seven years of age is doliincapax, i.e. he is incapable of
committing a crime and cannot be guilty of any offence.

. On her 7th birthday, Riddhima had costume party in which dress like a princess while the
others came for her class dressed as different Disney characters. One of the kids who was
dressed like a pirate got into an argument with a kid dress like the prince. In a pretend sword
fight the pirate impaled the shoulder of the prince with his costume‘s sword which had a
pointy end. Would the child be convicted?

(a) Yes, as they were above the age of 7 years.

(b) No, as they were under the age of 12 and did not understand the nature of their activity.

(c) Can‘t say, as Riddhima was 7 but the kids could have been older or younger at the party.

(d) No, as they were merely playing and did not intend to cause any harm to each other, they
also wouldn‘t have known the risk of a costume.

. A person Sarthak was prone to epileptic fits. During one of these situations he managed to
knock over a vase on the head of a passerby from his balcony. Being grievously injured, the
spouse of this passerby filed suit for bodily harm injury again Sarthak. Decide.

(a) The suit will not sustain as the spouse cannot file a complaint on behalf of the injured.

(b) The suit will not sustain as it was an accident.

(c) The suit will not sustain as he was not sane in that moment and can plead insanity.

(d) The suit will not sustain as the passerby should have been more cautious before walking
underneath someone‘s balcony.

. A Six-year-old boy, Rohan after watching a series of criminal drama on television became
extremely intrigued with the concept of crime. One day when he was really upset after an
argument with his mother he decided to kill her in her sleep completely aware of the
consequence. Can Rohan be convicted for Murder?

(a) Yes, as he had complete understanding and maturity of the consequences.

(b) No, as he was under less than 7 years of age.

(c) Yes, as he acted out of provocation and anger and plotted it with intention.

(d) No, as a minor cannot be blamed for any offence, it falls under doli incapax.

. After finding out that his wife was having an affair with their neighbor Mr. Sharma went
into depression. One day after reconsidering the situation in a fit of rage, decided to
physically harm either one of them. In his attempt for vengeance, he ended up causing the
death of his neighbor. He later claimed before the court that he had lost his sanity and that
hour and hence should not be found guilty. Decide.

(a) Yes, as he was not thinking straight and had lost his sanity to make prudent decisions.
(b) No, as anger is not a ground of claiming insanity. It has to be a medically proven one.

(c) No, as he had already plotted revenge before got a fit of rage.

(d) Yes, as he have plotted something in his sanity, but he only acted on it in moments of
insanity.

. A child of 14 years of age found a lost ring in a park and decided to keep it as he found it
really pretty. Thinking that it belongs to nobody else he decided to sell the same and keep the
money for himself. Has a child committed an offense?

(a) Yes, as he knew the consequence of his action.

(b) Yes, as he was above the age of 14 years and hence can be held liable for any an offence
regardless of his maturity.

(c) Yes, as he may not have had any motive when he picked the ring, but he later sold it for
wrongful gain.

(d) Yes, as he should have handed it to any adult or the police as he is old enough to know is
duties and responsibilities.

. A school took their students on a field trip to a museum. Among the student of class 9th was
Tina, who was a kleptomaniac. Unable to contain herself she managed to swipe what are the
pieces of jewelry from under the display. She was called later by the authorities and charged
for theft. Should she be convicted?

(a) Yes, as was older than the age of 12 years clearly.

(b) No, as he was a kleptomaniac and can use the defense of insanity to get an acquittal.

(c) Yes, as even though she was older than 7 years of age, she did not have the maturity or
understanding of an adult.

(d) Can‘t say as the facts do not mention her age or her mental status.

Passage (Q.-Q.): Religion is a matter of belief or faith. The constitution of India recognizes
the fact, how important religion is in the life of people of India and hence, provides for the
right to freedom of religion under Articles 25 to Article 28. The Constitution of India
envisages a secular model and provides that every person has the right and freedom to choose
and practice his or her religion. In a number of cases, the Apex Court has held that secularism
is the basic structure of the Constitution, the most important being the Kesavananda Bharati
case. Secularism means developing, understanding and respect for different religions. It is
believed that the word ‗Secularism‘ has its origin in late medieval Europe. In 18, during the
constituent assembly debate, a demand was made by the KT Shah to include the word
‗Secular‘ in the Preamble to the Constitution. The members of the assembly though agreed to
the secular nature of the constitution but it was not incorporated in the Preamble. Later, in 19
the Indira Gandhi government enacted the 42nd Amendment Act and the word ‗Secular‘ was
added to the Preamble. The 42nd Amendment Act also known as the ‗Mini Constitution‘, is
the most comprehensive amendment to the Constitution. Article 25 of the Constitution
guarantees freedom of religion to all persons in India. It provides that all persons in India,
subject to public order, morality, health, and other provisions: · Are equally entitled to
freedom of conscience, and · Have the right to freely profess, practice and propagate religion.
It further provides that this article shall not affect any existing law and shall not prevent the
state from making any law relating to: · Regulation or restriction of any economic, financial,
political, or any secular activity associated with religious practice. · Providing social welfare
and reform. · Opening of Hindu religious institutions of public character for all the classes
and sections of the Hindus.

. India has been land of various customs and usages. One of them includes the marriage of
women with a tree on account of them being Mangliks. Mangliks is an astrological concept
wherein any woman having this in their chart would lead to an early death of their husband.
Therefore, the curse is broken down by undertaking the aforementioned ceremony. This tree
marriage ceremony is still practiced by few people in northern India. The Rajasthan
government decides to pass a law banning this ceremony. Decide.

(a) The law poses a reasonable restriction as such superstitions should not sustain.

(b) The law is reasonable as it affects a very smaller section of the society.

(c) The law is unreasonable as it does not instigate anything to fall under the purview of
being against public interests.

(d) The law is not reasonable as it impacts the right of women to marry.

. St. Joseph International School is administered by a Christian family. To thank the


Almighty, the school used to have certain lines from Bible recited everyday at the school
assembly. Three students refused to be a part of the process and therefore expelled. The
students filed a petition that this violates their right to religion as they belong to a religion
sect which does not allow them to take part in thanking god through different means. Decide.

(a) The petition will succeed as their right of religion was getting affected.

(b) The petition will not succeed as it was not a matter of religion but just a way of
expressing gratitude.

(c) The petition will not succeed as the religion of the students puts forth unreasonable
circumstances.

(d) Both (b) and (c)

. The Government of Himachal Pradesh passes a law which makes the religious conversions
or attempt of conversions on account of force or allurement as a penal offence. Some Hindu
leaders challenge the law claiming it to be in violation if Article 25 on the ground that the
term propagate in the aforementioned article gives them the right to convert people into their
belief system and faith. Decide

(a) The law is constitutional as the right to convert cannot be part of right to religion.

(b) The law is not constitutional as it violates the right of the leaders to propagate the religion.

(c) The law is constitutional as it talks about conversion on account of fraud and allurement.
(d) The law is constitutional as reasonable restrictions can be imposed based on the
requirements of the situation.

. A religious sect has a practice of offering prayers five times in a day. While offering prayers
their temple used loudspeakers which was creating disturbance for the residents of the nearby
area. A lot of residents started complaining especially for the early and late hours. One such
resident named Revathi filed a complaint against the priest to the concerned administrative
authorities. The priest justified by taking the argument of freedom of religious practice.
Meanwhile, the state passes a law restricting the use of loudspeakers during prayer offerings
in early and late hours. The priest moves to the court challenging the law to be
unconstitutional. Decide.

(a)The priest will succeed as it is violation of the right granted under Article 25.

(b) The priest will fail as it is a reasonable restriction on the method of the practice.

(c) The priest will fail as the state has to take care of unreasonable religious practices.

(d) The priest will succeed right of religion cannot be restricted through such means.

. Momina is a Muslim girl who wears a headscarf as a religious symbol. She studies in a
government school and keeps it on even during class hours. The school prescribes for a
school uniform and the scarf is not a part of the said uniform. She is being asked to remove
the scarf during class hours but she denied. On such refusal she is punished by the authorities
for not complying with the uniform. Decide.

(a) The fundamental right of religion has not been violated as it is just a matter of school
uniform.

(b) The fundamental right of religion has not been violated as same uniform is mark of
equality.

(c) The fundamental right has been violated as the scarf was a religious symbol.

(d) Both (a) and (b)

5
Passage (Q.-Q.):The meaning of offer and acceptance is the basis of a contract. To form a
contract, there must be an offer made by one party which is, in turn, accepted by another
party, and then, in most cases goods and/or services must be exchanged between the two. An
offer contains 2 parts. (1) The Expression: this is where the parties articulate in some form an
inclination to enter into a contract and to make that contract legally binding upon the
acceptance by both parties. The expression may take a wide variety of forms, from a personal
discussion to a letter that lays out the basics of the terms. (2) The Intention: This is a slightly
more vague concept, and it is often left up to the courts to determine whether or not intention
was breached by a party. Essentially, intention involves a presumption by both parties that the
agreement will be legally binding and comes down to the belief that neither party would
begin discussions without the intent to live up to their side of the bargain. Offers can really
cover anything, from a verbal agreement to provide a service, such as housesitting, to a
detailed contract with legal terminology that one may find in an agreement to transfer real
estate. It’s more than a promise, because it must be made with the understanding that what is
being agreed to will be legally binding. It can be for the sale of goods, a pledge to perform a
service, or even a promise not to engage in an activity. The more complex the agreement, the
greater the likelihood will be that each party would engage legal counsel to negotiate the
contract. Acceptance is the final agreement of both parties to consent to the terms of the offer.
While it is common for the terms of the offer to be negotiated before acceptance, if it can be
shown that through conduct and communications that the parties did in fact intend to agree to
the final terms of the contract, then formal acceptance of an offer is not required for it to be
legally binding. It is also not always necessary that acceptance be in the form of a signature
on a piece of paper, although this is the most commonly accepted agreement between parties.
For instance, if a party performs an act that would not otherwise happen, such as a painting
contractor painting a house or a professional moving company moving furniture from one
location to another, it would be interpreted as acceptance and agreement to the terms of the
offer of payment for these services.

. Grant offered to buy shares from Carriage Accident Insurance Co. The insurance company
sent a letter of acceptance to Grant but the letter was lost in the post and never arrived. The
liquidator of the insurance company sued Grant for the money owing for the shares. Was
there a valid contract?

(a) No, the acceptance was not communicated in a manner accessible to Grant.

(b) No, as the letter and the acceptance never reached Grant.

(c) Yes, as the acceptance was communicated and the contract was formed.

(d) Yes, the contract was formed the moment the offer was accepted.

. Ravi is extremely thirsty. He says "I would probably give 5000 rupees to anyone who
brought me water at this time." Akash hears this and expecting 5000 rupees, gets Ravi a
bottle of water. Upon demanding the money, Ravi refuses. Can Akash go to court over this?
(a) Yes. Ravi had made a general offer which was fulfilled by Akash.

(b) Yes, Ravi’s offer constituted an expression of interest to enter into a binding contract.

(c) Ravi could not have made a general offer. A specific offer to Akash is necessary for a
contract to be formed.

(d) No, there was no offer, merely a declaration of interest.

. Neha telegraphed to Aditi, writing: “Will you sell me your Rolls Royce CAR? Telegram the
lowest cash price.” Aditi also replied by telegram: “Lowest price for CAR is Rs. 20 lakh.”
Neha immediately sent his consent through telegram stating: “I agree to buy the CAR for Rs.
20 lakh asked by you.” Aditi refused to sell the car. Decide

(a) Aditi’s letter was the offer and Neha’s reply was the acceptance. There is a valid contract. 

(b) Aditi did not intend to make an offer. She can refuse to sell the CAR as there is no valid
contract.
(c) Aditi did not show any willingness to enter into a contract. In the absence of a legal
intention, there can be no contract.

(d) Neha’s letter itself did not constitute an offer. There is no valid contract.

. Arun and Ravi are supposed to be participating in a race. The night before the race, Ravi
offers Arun that if he will not participate in the race Ravi will give him 2000 Rupees. Arun
agrees. Decide

(a) The contract is not valid for an offer asking to abstain is not a valid offer.

(b) An offer asking to abstain as consideration for the contract is a valid offer.

(c) A contract opposed to public policy cannot be a valid contract. Thus, this contract is
invalid as it is unethical.

(d) This is not a contract opposed to public policy. It is a contractual transaction and not
unethical.

. Ashrut is a software engineer and works at InfoSys. When his employer asked him to resign
from the job, and in return he would be paid 3 Lakh rupees, Ashrut did not say anything.
However, 2 days later, Ashrut quit his job and then demanded his boss to pay him 3 Lakh
rupees. Decide

(a) Ashrut’s employer has entered into a binding contract with Ashrut. It was accepted by
conduct.

(b) Ashrut’s employer has not entered into a binding contract with Ashrut. There was no
acceptance which is a fundamental requirement of a contract.

(c) Ashrut’s employer has made an offer which is contrary to public policy. This cannot be a
valid contract.

(d) Ashrut’s employer has sought a consideration which is illegal. There cannot be a valid
contract with an illegal consideration.

Passage (Q.-Q.):Volentinon fit injuria is a defence of limited application in tort law. A direct
translation of the latin phrase volentinon fit injuria is, 'to one who volunteers, no harm is
done'. Where the defence of volenti applies it operates as a complete defence absolving the
Defendant of all liability. It is often stated that the Claimant consents to the the risk of harm,
however, the defence of volenti is much more limited in its application and should not be
confused with the defence of consent in relation to trespass. The defence of volentinon fit
injuria requires a freely entered and voluntary agreement by the Claimant, in full knowledge
of the circumstances, to absolve the Defendant of all legal consequences of their actions.
There is a considerable overlap with contributory negligence and since the introduction of the
Law Reform (Contributory Negligence) Act 15, the courts have been less willing to make a
finding of volenti preferring to apportion loss between the parties rather than taking an all or
nothing approach. The requirements of the defence are that there must be (1) a voluntary (2)
agreement (3) made in full knowledge of the nature and extent of the risk. The agreement
must be voluntary and freely entered for the defence of volentinon fit injuria to succeed. If
the Claimant is not in a position to exercise free choice, the defence will not succeed. This
element is most commonly seen in relation to employment relationships, rescuers and
suicide.The second requirement for the defence of volentinon fit injuria is agreement. The
agreement may be express or implied. An example of an express agreement would be where
there exists a contractual term or notice. However, this would be subject to the controls of s.2
of the Unfair Contract Terms Act 19. An implied agreement may exist where the Claimant's
action in the circumstances demonstrates a willingness to accept not only the physical risks
but also the legal risks. For knowledge, the test is subjective and not objective and in the
context of an intoxicated claimant, the question is whether the claimant was so intoxicated
that he was incapable of appreciating the nature of the risk.

. Two brothers Sanjay and Sunil had been working in Dheeraj’s quarry. They tried to test
some detonators without taking requisite precautions and their act was in contravention of
statutory provisions and also the orders of Dheeraj to both of them. One day, this act resulted
in an explosion causing injury to Sunil. He brought an action against Dheeraj saying that his
brother was equally responsible for the accident and that Dheeraj being the employer was
held vicariously liable for his brother’s conduct. Decide.

(a) Dheeraj is not liable because he had warned them not to do the dangerous act.

(b) Dheeraj is not liable because he is protected by the defense of volentinon fit injuria.

(c) Dheeraj is liable by the principle of vicarious liability.

(d) Dheeraj is liable because there is a breach of statutory obligation.

. Ashay had to catch a flight to Mumbai. He was already late to the airport. He hailed ataxi on
the road and asked the taxi-driver Agarwal to hurry to the airport at the maximum
speedpossible. Agarwal, who was smelling strongly of liquor, started driving rashly. On the
way to the airport, the taxi collided with a truck near the airport and Ashay was severely
injured. He sued Agarwal for negligence.

(a) Ashay would be successful because Agarwal was driving recklessly.

(b) Ashay would not be successful because it was Ashay who asked Agarwal to drive fast and
therefore, he voluntarily agreed to the risk.

(c) Ashay would not be successful because he knew that Agarwal was drunk and therefore, he
knew that the chances of accident were thereby increased.

(d) Both (b) and (c)

. Ratnesh is a pilot who was supposed to fly the Joint Secretary of the Karnataka Health
Ministry to Delhi for a meeting with the Prime Minister. Ratnesh instructed the Secretary
about flight safety and asked to not use his phone as it could disturb flight navigation.The
Secretary however used his phone and the navigation was disturbed which led to the flight
crashing.It turns out that the system was not of a good quality but would not have failed if not
for the disturbance. The Secretary sued the airlines.

(a) The Secretary would be successful because the airlines had a bad navigation system.
(b) The Secretary would not be successful because it was him who used his mobile phone and
therefore, he voluntarily agreed to the risk.

(c) The Secretary would not be successful because he knew that the system was not in a good
condition and therefore, he knew that the chances of accident were thereby increased.

(d) Both (b) and (c)

. In which of the following circumstances, is the exercise of a free choice most likely?

(a) A politician agreeing to favour a certain party on their request and award them a tender
thus risking corruption charges.

(b) A teacher agreeing to award more marks to the principal’s son risking outrage from
parents

(c) A smuggler agreeing to smuggle guns instead of a banned species of flower thus risking
significantly higher punishment

(d) A doctor agreeing to turn off the ventilator of a patient upon being threatened by goons to
do so thus risking disciplinary action.

. Venkata was intoxicated and in a state of panic when he asked the helicopter pilot to land on
ice for he feared that there was storm ahead. This was not a practice usually done by
pilots.The helicopter crashed. When the accident occurred, Venkata sued the pilot for
negligence. The pilot claimed volentinon fit injuria

(a) Venkata consented to the risk for he was in a state of panic

(b) Venkata did not consent to the risk for he was intoxicated

(c) Venkata consented to the risk for he asked the pilot to land on ice

(d) The liability shall depend on the level of Venkata’s intoxication

. Please refer to the facts above. Venkata did not ask the pilot to land on ice but was
constantly looking at the weather and discussing with his co passengers about the safest
option being the possibility of landing on ice. The driver anticipating that Venkata might be
panicking, starting descending on ice.Venkata did not say anything. The accident occurred.

(a) Venkata consented to landing on ice by not objecting

(b) Venkata did not consent expressly to the landing on ice which was required

(c) Venkata should not have panicked in the first place

(d) Venkata did not consent. Mere silence cannot be construed as active consent19 of 36

Passage (Q.-Q.):Under common law, there are seven types of intentional torts: assault,
battery, false imprisonment, trespass to land, trespass to chattels, conversion, and intentional
infliction of emotional distress. Trespass to chattels refers to the use of property without
permission of the owner. Trespass to chattels can be easily confused with the tort of
conversion because they both deal wrongful interference of personal property. A trespass to
chattels is an intentional interference with another person's lawful possession of a personal
property. A "chattel" refers to any personal property, moving or unmoving. Trespass to
chattels does not apply to real property or any interest in land. In order to prove trespass to
chattels, it must be shown that (1) there was an intent to trespass. Merely intending to do the
act is enough to show this element of trespass. You don't necessarily need to show intent to
harm a specific person. (2) The second element is lack of the owner’s consent. There must be
an unauthorized, unlawful interference, which means the person interfered with or
dispossessed the chattel without the owner's permission. Such consent must be free. The third
element is that there must be interference of chattel. A person commits a trespass to chattel
by (1) dispossessing another of the chattel, (2) using or intermeddling with a chattel in the
possession of another, or (3) damaging the chattel. Interference does include dispossession of
a chattel, but it must be something short of conversion. A mistake of ownership is not a valid
defense to a trespass to chattels. In other words, it doesn't matter if the person didn't know the
property belonged to you. Possessing or damaging the property itself is enough to show
interference. In general, a person will be held liable for a trespass to chattels in any of the
following situations: 1. The person dispossesses the other of the chattel. 2. The chattel is
impaired as to its condition, quality, or value. 3. The possessor is deprived of the use of the
chattel for a substantial time. 4. Bodily harm is caused to the possessor, or harm is caused to
some person or thing in which the possessor has a legally protected interest. In a trespass to
chattels claim, you can only recover actual damages (as opposed to nominal damages).
Actual damages are measured by the diminished value of the chattel that resulted from the
defendant's actions.

. Arjun stole Anand’s phone in the Delhi Metro. Arjun registered an FIR. When the police
finally traced Arjun’s phone at Anand’s place, Anand could not find the phone in his
possession. It was found that Ahuja had taken the phone from Anand’s house and was using
it. Anand filed a suit.

(a) Ahuja is liable for trespass to chattel as there was intentional interference with Anand’s
property

(b) Ahuja is not liable for trespass to chattel as he did not have the intent to commit the tort

(c) Ahuja is liable for trespass to chattel as Anand was in possession of property

(d) Ahuja is not liable for trespass to chattel as Anand was not in lawful possession of the
property

. Please refer to the facts above. Infuriated, Anand along with his 5 friends went in to Ahuja’s
house and threw away his property from his house on the road. Decide

(a) Anand is liable for trespass to chattel as he interfered with Ahuja’s property by throwing it
on the road.

(b) Anand is not liable for trespass to chattel as he did not use Ahuja’s property

(c) Anand is liable for trespass to chattel as he intentionally committed the tort

(d) Anand is not liable for trespass to chattel


. Munna Bhai is the goon of the locality and gets houses emptied for builders. One such
house belonged to Dr. Anand. Munna Bhai threw away Dr. Anand’s property and occupied
his house.

(a) MunnaBhai is liable for trespass to chattel as he interfered with Dr. Anand’s property by
occupying it.

(b) MunnaBhai is liable for trespass to chattel as he intentionally committed the tort.

(c) MunnaBhai is not liable for trespass to chattel as he interfered with his land and house.

(d) MunnaBhai is a goon and should be punished.

. Apurv bets with his friends that since his wallet is old, he will pick someone’s pocket and
use their wallet. Apurv acts on this bet and picks Baikunth’s pocket. Decide

(a) Apurv is liable for trespass to chattel as he intentionally picked Baikunth’s pocket

(b) Apurv is not liable for trespass to chattel as he did not specifically announce evil intent
against Baikunth

(c) Apurv is liable for trespass to chattel. Intent against specific person is not required.

(d) Apurv is not liable for trespass to chattel as he only acted in pursuit of a bet

. A is B’s friend. A wants to take his book lying in B’s cupboard and asks for the lock code to
his cupboard. B refused. A was curious about what was lying in B’s cupboard so he spiked
A’s water with a medicine that made him feel drowsy. A asked B for his lock code and B told
him. A accessed B’s cupboard and took a valuable book away.

(a) A is liable for trespass to chattel as he intentionally deprived B of his property.

(b) A is not liable for trespass to chattel for he took B’s consent before accessing his
cupboard

(c) A is liable for trespass to chattel as B’s consent was not free

(d) A is not liable for trespass to chattel

. Please refer to the facts above. A’s book had been kept by B for a long while now. Despite
repeated reminders, B did not return the same. A sneaked into B’s cupboard and took his own
book. However, it turns out that A did not take his book but B’s book by mistake.

(a) A is liable for trespass to chattel as he intentionally deprived B of his property.

(b) A is not liable for trespass to chattel for he took B’s consent before accessing his
cupboard

(c) A is liable for trespass to chattel as mistake of ownership is not a valid defence

(d) A is not liable for trespass to chattel as B was not giving his book
Passage (Q.-Q.):Extortion is a criminal offense, which occurs when a person either obtains
money or property from another through coercion or intimidation or threatens one with
physical harm unless they are paid money or property. Euphemistically, refraining from
doing harm is sometimes called protection. Extortion is commonly practiced by organized
crime groups. The actual obtainment of money or property is not required to commit the
offense. Making a threat of violence or a lawsuit which refers to a requirement of a payment
of money or property to halt future violence or lawsuit is sufficient to commit the offense.
Section 3. Extortion.—Whoever intentionally puts any person in fear of any injury to that
person, or to any other, and thereby dishonestly induces the person so put in fear to deliver to
any person any property or valuable security, or anything signed or sealed which may be
converted into a valuable security, commits “extortion”. The element of dishonesty is the
essence of this offence. The chief element in the offence of extortion is that the inducement
must be dishonest. It is not sufficient that there should be wrongful loss caused to an
individual but the person putting that individual in fear of injury must have the intention that
wrongful loss should be caused. Where the accused honestly believed that the complainant
had taken the money belonging to him (the accused), an attempt to get it back cannot be said
to be with the intention of causing wrongful loss to him. The essential elements of the crime
of extortion can be enumerated as:

1. It should be shown that there was a force to create threat or fear in the victim of the
wrongful act. 2. The wrongdoer must be having some interest in putting the victim under fear
of any sort of threat or injury. 3. There should be the presence of a dishonest intention. 4.
Such threat or fear of injury in is caused on to the other person in order to force him to
transfer any of his property or any other valuable item.

. K on finding out that Z has been having an affair outside of his marriage, threatens to
publish a defamatory libel concerning Z unless Z gives him money. Z refuses to do so. Has K
has committed extortion?

(a) No, as K has not threatened any harm or injury to his person.

(b) No, as Z has not handed him any money, hence there was no loss.

(c) Yes, as K has caused a fear of injury to reputation in turn of causing him wrongful loss.

(d) Yes, as K has dishonest intention to get a wrongful gain out of Z’s predicament.

. Z threatens D to sit inside his vehicle without causing any ruckus or he would harm him
physically. Later he called the family members of D to ask for ransom for the release of D or
he would cause him harm. Can Z have been said to cause extortion?

(a) No as Z did not threaten D to cause himself any wrongful gain or loss of valuable property
to D.

(b) Yes, as Z caused a fear of injury to D for non-compliance to his measures.

(c) No, as Z caused extortion against D’s family, but to D it is not extortion but could be a
case of abduction.
(d) Yes, as Z has extorted both D and his family by threatening injury in turn for unlawful
gain of valuable property.

. ‘A’ threatens Z that he will keep Z’s child in wrongful confinement, unless Z will sign and
deliver to ‘A’ a promissory note binding Z to pay certain monies to A. Z signs and delivers
the note. Decide?

(a) Yes, as there was the presence of a dishonest intention.

(b) Yes, as it was in order to force him to transfer any of his property or any other valuable
item.

(c) No, as there was no threat of injury to his person, it was to his son.

(d) No, as there was no dishonest intention as promissory note is a future contract and does
not require immediate transfer of valuable property.

. A, by putting Z in fear of grievous hurt, dishonestly induces Z to sign or affix his seal to a
blank paper and deliver it to A. Z signs and delivers the paper to A. Has A committed
extortion?

(a) Yes, it is as the blank paper can be turned to valuable security later.

(b) Yes, as he has threatened injury to Z for unlawful purposes.

(c) No, as a blank paper is not a valuable property, hence he couldn’t have extorted him for
any wrongful loss.

(d) No, as all the elements of extortion are not fulfilled in the above instance.

. Mr. Y, unhappy with his daughter’s choice of groom, threated to harm himself if she did not
break of such engagement and marry a boy of his choice. Scared for her father’s life, she
agreed and broke of the bond. Has Mr. Y committed extortion?

(a) Yes, as he induced harm and injury to force her to commit an act against her will.

(b) No, as did not aim to have any gain of property through this act.

(c) Yes, as he had dishonest intention to achieve his will in her marriage.

(d) No, as family matters are not in the subject matter of extortion.

. Bank of Hindustan, used to give out small loans against collateral to their customers. Mr. Z
took a loan against his Car and had to return the same with interest in certain installments.
When a few installments failed, the bank warned him against seizing his car in case of next
default. Mr. Z filed a suit of extortion. Decide.

(a) Yes, as the bank aims to harm the property of Z for gain of money.

(b) No, as the bank is duly owed that sum along with the collateral, and it’s not a dishonest
intention.
(c) No, as the bank has a right to make any threats in case of defaulting customer.

(d) Yes, as inducing fear to gain is enough to establish extortion regardless of the fact that the
loss was caused or not.

Passage (Q.-Q.):In the wake of the migrant crisis, several states have amended existing labour
laws, either suspending them altogether or increasing working hours. It is axiomatic that
these amendments must be in consonance with the fundamental rights guaranteed to labour,
as also the Directive Principles of State Policy which enjoin the state to further the interests
of labour in its policies. Any legal changes will inevitably be challenged in court and must
meet constitutional standards to survive such a challenge. To be compelled to work violates
Article 23 of the Constitution of India, which provides a “right against exploitation”
prohibiting human trafficking, begar, and forced labour. In PUDR vs Union of India (19), the
Supreme Court held that laws protecting contract labour and inter-state migrant workmen
were intended to ensure basic human dignity; violating these laws would violate the right to
life under Article 21. In PUDR, the Court also noted that no one would willingly work for
less than the minimum wage without some force or compunction. Yet, rather than
encouraging workers to return by securing wages and improving working conditions, the
amendments introduced by the states are removing basic labour law protections. The
Directive Principles of State Policy (DPSP), though unenforceable, are constitutionally-
mandated goals for the State to work towards when making laws. The DPSP enjoin the State
to ensure that the “operation of the economic system does not result in the concentration of
wealth and means of production to the common detriment” (Article 39), to “make effective
provision for securing the right to work” (Article 41), “to secure to all workers a living wage,
conditions of work ensuring a decent standard of life.” (Article 43). It is tempting to see the
pandemic as a once-in-a-lifetime opportunity to push through labor reforms. Even so, it is
unclear that the whole scale abrogation of the labor law system benefits either labor or
industry. If anything, this is an apt moment for the governments, at both the Centre and in the
states, to renew their commitment to the DPSP.

. A competent court passed a sentence to a convict providing for a rigorous imprisonment for
a period of seven years. In furtherance to the same, the jailor assigns the prisoner for breaking
stones and puts him as a labourer for an ongoing construction operation inside the jail. The
prisoner has not consented for doing such manual work. Further, he challenges the work to be
forced and thus violative of Article 23. Decide.

(a) The jailor has committed an offence putting him for work against his will.

(b) The jailor has committed no offence as the circumstances would alter in case of a
prisoner.

(c) The jailor has committed an offence due to absence of remuneration.

(d) The jailor has committed no offence but the court provided for rigorous imprisonment.
. Meethesh has a family of 6 members. He is sole bread earner of the family which puts an
excessive financial burden on him. Due to such circumstances, he decides to work in the
factory of Mr Bajaj at a rate less than the minimum wage. Whether this would amount to
forced labour? Decide.

(a) This cannot be considered forced labour as it is out of willingness of Meethesh.

(b) This cannot be considered forced labour as he is paid for the services from by him.

(c) This cannot be considered as forced labour but would be a violation of DPSP.

(d) This can be considered as forced labour as due to the external factors the consent is not
free.

. Aman is a diligent and laborious employee. He works for Namit who runs his own business.
With an intention of further growth, Aman decides to work at a different place. When Namit
finds out, he threatens him of false prosecution and imprisonment on account of a false
charge of stealing from the business. This was done to retain him in the business. Would this
amount to forced labor. Decide.

(a) No, it would be covered under malicious prosecution.

(b) No, as reasonable compensation is being paid to Aman by Namit.

(c) Yes, as the willingness is induced by threat of prosecution and imprisonment.

(d) Yes, as the remuneration was not enough so he intended to shift to a different place.

. Mannu Singh is a daily wage worker who took a loan of Rs. 35,000/- from the moneylender,
Maniram. The loan was to be utilised for the purposes of marriage of the daughter of Mannu
Singh. In exchange of the loan, Maniram asked Mannu to work on his farm for a period of 15
years where he shall be provided with food, shelter and a remuneration of Rs. 500 every
month. Constrained by the circumstances, Mannu Singh accepts the offer. A welfare
organisation finds out about the arrangement and files a petition on the ground thatMannu
Singh is being forced to work. Decide.

(a) The petition will not succeed as the Mannu is being provided with food and shelter along
with the remuneration.

(b) The petition will not succeed as the arrangement is entered into by the consent of Mannu
Singh.

(c) The petition will succeed as this amount to exploitation by Maniram.

(d) The petition will succeed due to lack of free consent.

Passage (Q.-Q.):Rape is a crime, which has a devastating effect on the survivors; it has been
described as a “beginning of a nightmare”. The aftershocks include depression, fear, guilt-
complex, suicidal-action, diminished sexual interest. etc., “one becomes afraid
of’……..writes a victim, “half the human race”. Referring to the pitiable condition of women
in society Mr. Justice S. Ahmad observed that “unfortunately, a woman in our country,
belongs to a class or group of society who are in a disadvantaged position on account of
several social barriers and impediments and have therefore, been victims of tyranny at the
hands of men with whom they, unfortunately, under the Constitution “enjoy, equal status”.
“Women also have the right to life and liberty; they also have the right to be respected and
treated as equal citizens. Their honour and dignity cannot be touched or violated. They also
have the right to lead an honorable and peaceful life”. Rape is a stigma which exists in the
society from a long time. The dictionary meaning of word rape is “the ravishing or violation
of a woman.” She is traumatized after the event; it is very difficult for a woman to come out
of this trauma. Rape in India is a cognizable offence. There are many provisions in various
Acts. The word rape is legally defined u/s 3 of Indian Penal Code, 18. It defines the rape and
also prescribes its punishment. Whenever a man penetrates or does sexual intercourse with a
woman without her consent or will it amounts to rape. Penetration here means that only a
slightest of the touch of penis to vagina amounts to rape, unruptured hymen of woman does
not prove that rape was not committed. The Supreme Court has expressed strong disapproval
of courts casting a stigma on the victim’s character, but stigmatization continues, leading to
acquittals. Setting aside one acquittal by a Ludhiana court that labeled the victim with ‘loose
character’ while interpreting her consent to sex, the Supreme Court in a 19 judgment said,
“The trial court interpreted that the victim was habituated to sexual intercourse just because
the speculum the doctor used entered her vagina easily and hence she was of loose character.
These observations lack sobriety expected of a judge. No stigma should be cast against a
victim of sex crime who is on trial”.

. Madhur meets Anisha at a party where both of them exchange pleasantries. After a while,
when both of them are alone Madhur imposes himself on Anisha and forcefully engages into
a sexual intercourse even when she protests for the same. Decide.

(a) This shall amount to rape due to absence of consent by Anisha.

(b) This shall not amount to rape due to an exchange of pleasantries.

(c) This shall amount to rape due to a sexual intercourse between them

(d) This shall not amount to rape but shall fall under assault.

. Disha and Harsh have known each other for a long time and have engaged in sexual
intercourse at various instances. All such sexual intercourses involved the consent of the
individuals. However, at one instance, both the individuals had a sexual intercourse for which
Disha did not consent to. Whether this would amount to the offence of Rape. Decide.

(a) This shall not amount to rape as they had sexual intercourse at earlier occasions for which
she consented to.

(b) This shall not amount to rape as consent is vague in the last case.

(c) This shall amount to rape as there was no consent and previous consent is not relevant.

(d) Both (a) and (b)

. Apoorva went to get together of her college friends. Ramesh was also there and both of then
exchanged a pleasant conversation. They decided to engage in a sexual activity. Apoorva
consented to one particular type of sexual activity. However, when they got to the point of
intercourse, she withdrew her consent. Would this amount to the offence of Rape? Decide.

(a) No, as she consented for the sexual activity before the beginning of the intercourse.

(b) No, as such withdrawal at such a later stage would not amount to withdrawal.

(c) Yes, as she withdrew her consent for that particular sexual activity.

(d) Yes, as she consented for only one type of sexual activity.

. Rahul engaged in a sexual intercourse with Simran who was 15 years old. The parents of
Simran decide to file a case against Rahul for the offence of rape. However, he contends that
the intercourse had the consent of Simran therefore cannot be categorized as rape. Decide.

(a) Rahul is not liable as the act was consented by Simran.

(b) Rahul is not liable as the parents cannot file the case bur only Simran

(c) Rahul is liable as the consent of Simran cannot be considered a valid consent.

(d) Rahul is liable as the parents of Simran can decide for their daughter.

. Choose an appropriate reason of the following for the lack of gender neutral laws:

(a) Women are the ones who are the victims of such heinous crime. (b) Women cannot
commit rape due to biological reasons.

(c) Men are the aggressors in the majority of the cases.

(d) Women are the disadvantaged group due to stunted societal structure so they need more
protection.

. Robin is a police officer who met Malaika at a party. After the party, he told her that if she
did not engage into a sexual intercourse with him then he would put her in jail and harass the
family members on account of the powers he possess through his profession. They both enter
into a sexual activity wherein she does not protest or explicitly resist. Decide.

(a) Robin shall be liable as the consent was obtained by threat.

(b) Robin shall not be liable as there was no explicit resistance.

(c) Robin shall not liable due the existence of inherent consent.

(d) Robin shall be liable for coercion but not the offence of rape.

Passage (Q.-Q.):The Indian Penal Code provides some general defences under chapter four
that exonerate criminal liability which based on the premise that though the person
committed the offence, he cannot be held liable. This is because at the time of commission of
offence, person was justified of his/her acts, or there was absence of mensrea. However, it is
not all acts that are to be punished. There are certain defences provided under the ambit of
Indian Penal Code (IPC), 18 from Sections to . Mistake of fact arises when accused
misunderstood some fact that negates an element of crime. This legal weapon can be used,
where accused succeeds to prove that he/she was mistaken to the existence of some facts or
ignorant of the existence of such facts. It is a condition that such mistake must pertain to fact
not law. Section and of IPC contains the provision of mistake of fact. Such mistake must be
reasonable and must be of fact and not of law. The legal maxim, "ignorantia facti excusati
ignorantia juris non excusat" which means ignorance of fact is an excuse, but ignorance of
law is no excuse. So it is a basic requirement to be get protected under the sphere of this
defence that mistake must be of fact. Section : Act done by a person bound, or by mistake of
fact believing himself bound, by law.Nothing is an offence which is done by a person who is,
or who by reason of a mistake of fact and not by reason of a mistake of law in good faith
believes himself to be, bound by law to do it. Section : Act done by a person justified, or by
mistake of fact believing himself justified, by law.Nothing is an offence which is done by any
person who is justified by law, or who by reason of a mistake of fact and not by reason of a
mistake of law in good faith, believes himself to be justified by law, in doing it. Thus, the
general defences enshrined under IPC are of paramount importance in establishing the
parameters of criminal offences. Criminal liability makes a person liable for the acts which
are prohibited by law. Indian Penal Code took cognizance of fact that all acts are not to be
punished. The acts devoid of mensrea are exempted from criminal liability.

. Before going to the Church, Zeeshan fired the gun and left it empty. However, during his
absence, Chester visits the house and takes the gun for shooting and while keeping it back
leaves it loaded. Later on the same day, Zeeshan takes the gun and touches the trigger. The
gun goes off and kills his wife who was sitting in front of him. Decide.

(a) Zeeshan is guilty because he was negligent and that killed the wife.

(b) Zeeshan is not guilty as he was under a mistake of fact.

(c) Zeeshan is guilty and liability should be absolute.

(d) Zeeshan is guilty as such a weapon should be handled with great care.

. Chulbul is a renowned police officer who is ordered to catch a dangerous criminal named
Hari Pandey. After due enquiry in good faith, Chulbul, believing Prasoon to be Hari Pandey,
arrests him. Decide the liability of arresting a wrong individual and curtailing his
fundamental rights.

(a) Chulbul will be liable as this impacted the fundamental rights of Prasoon.

(b) Chulbul will be liable as his enquiry was deficient making an innocent suffer.

(c) Chulbul will not be liable as he was acting under the orders.

(d) Chulbul will not be liable since he acted in good faith.

 
. A complaint was made by a nurse engaged with a civil dispensary against the accused who
was the President of the Municipal office at that moment. He wrote a letter to the surgeon on
the basis of the allegations made by the renowned men of the community that the nurse had a
bad reputation for having illicit relations with a lot of men. The letter indicated that the
accused approved of such allegations on the nurse. However, there existed no proof that there
was an enquiry made on such facts. Decide.

(a) The President of the Municipal office is not liable as it could be mistake of fact.

(b) The President of the Municipal office is not liable as he did not say anything about the
nurse.

(c) The President of the Municipal office is not liable as he just the information further.

(d) The President of the Municipal office is liable as he failed to take due care.

. Seeta and Geeta had a strained relationship. At one instance, certain expensive utensils were
stolen from the house of Seeta so she thought of a plan to catch hold of the thief. She thought
that the thief would come to the only village pond to fetch water and at that moment she will
get hold of it. In pursuance to the same, she hid herself behind a tree and when she saw Geeta
filing water in the utensils gave a blow with a lathi on her head. Geeta sustained injuries and
filed a case against Seeta. Decide.

(a) Seeta is not liable as this would be considered a mistake of fact.

(b) Seeta is not liable as she was just trying to recover her stolen property.

(c) Seeta is liable as there was a complete lack of good faith.

(d) Seeta is liable as Geeta sustained injuries because of her blow.

. Jadegas & Sons is a renowned firm in the business of import and export. They have been
dealing with edible and non-edible oil since the last fifty years. On one day, they were
charged with the offence of importing non- edible coconuts oil under an open general license.
They pleaded that they believed it to be a non-canalized item and therefore acted in good
faith. Decide.

(a) They are not liable as the factual information suggest that they acted in good faith.

(b) They are liable as such act cannot be considered bona fide when they have been
functioning for such a long period.

(c) They are liable as the act was under mistake of law which is not excusable.

(d) They are not liable as they did not possess any dishonest intention.

6
Passage (Q.-Q.):The Indian Penal Code under Section 120-A defines conspiracy as “When
two or more persons agree to do or cause to be done, an illegal act, or an act which is not
illegal by illegal means, such an agreement is designated a criminal conspiracy, provided that
no agreement except an agreement to commit an offence shall amount to a criminal
conspiracy unless some act besides the agreement is done by one or more parties to such
agreement in pursuance thereof.” The very agreement between two or more persons to carry
out a crime gives origin to the offence of conspiracy irrespective of the fact whether such act
has been brought into motion or not.For establishing a conviction under Section 120A, one is
required to prove the existence of an ‗agreement‘ as it is the sine qua non for the composing
an offence of conspiracy. Such a scheme regarding the commission of a crime may be
implied or express. What is required is the unity of objects and not necessarily physical unity.
A mere entertainment of an evil wish or a coincidence of adverse intentions among certain
individuals would not amount to conspiracy.Neitherit is necessary that all the conspirators
must be known to one another nor all the details of the conspiracy. Meeting of minds is to
commit an illegal act or application of illegal means is requisite. After the individuals have
gained knowledge of the scheme regarding the commission of the crime, there may be
individuals possessing the intention to carry on with the same or drop out of the scheme. The
individuals who do not carry forward the plan due to the absence of intention must not be
counted in as conspirators unless they act in such a way that the existence of intention can be
inferred. In case certain individuals commit a crime or perform certain legal acts through
illegal means which is related to the object of the scheme of conspiracy, all of them will be
roped in for the commission of conspiracy even if they had not participated actively in
perpetuating the offence.Further, conspirators may cease to exist in every phase of the
carrying out of the conspiratory

. The State of Rusasa has brought a legislation banning the import of camphor. However, the
law only applies prospectively. Raza and Murad together smuggle camphor out of the
warehouses in Rusasa and sell it in the markets. Decide

(a) Raza and Murad committed an offence under Section 120 A for they committed an illegal
act

(b) Raza and Murad have not committed any offence for there is no illegality involved

(c) Raza and Murad have committed an offence under Section 120 A for the act is harmful
towards another person

(d) Raza and Murad have not committed any offence under Section 120A. The harmfulness
of the act against another person cannot be proven.

. Arnab and Rajput own factories that manufacture masks. After the imposition of lockdown
due to COVID[1]19, the government restricted prices of masks on all new produced stocks.
Arnab and Rajput were still selling at higher prices in the markets. When caught, they
claimed that this was not newly produced stock.

(a) Arnab and Rajput committed an offence under Section 120 A for they committed an
illegal act

(b) Arnab and Rajput have not committed any offence for there is no illegality involved
(c) Arnab and Rajput have committed an offence under Section 120 A for the act is harmful
towards another person

(d) Arnab and Rajput have not committed any offence under Section 120 A for the harmful
nature of the act cannot be proven.

. Vikash and Avinash run their own imported silk garment stores. Vikash imported
Strawberry, an illegal variety of silk on an aircraft. Avinash imported Strawberry via ships.
They sold it at their own shops which are located side by side in the same market. Decide

(a) Vikash and Avinash committed an offence under Section 120 A for they committed an
illegal act

(b) Vikash and Avinash are not liable for the act was committed out of necessity

(c) Vikash and Avinash are liable for there existed an agreement in their acts

(d) Vikash and Avinash are not liable for their acts are separate thus no two persons are
involved

. A group of girls went to a club for fun and frolic. One of them suddenly fired atthe bar mate
for her refusal to serve drinks and due to this bar mate got killed. The others were unaware
that the accused(P) wascarrying a loaded pistol. They had stayed at the club for about 2
hours.

(a) All of them are guilty of murder for they all constituted part of the group that murdered
the bar mate.

(b) Only P is guilty of murder. There is no evidence of the group having agreed to the firing.

(c) P is guilty of murder and the others are guilty of criminal conspiracy for they acted in
concert with P.

(d) The entire group is liable for criminal conspiracy to murder as there was an agreement to
fire at the bar mate.

. A and B enter into a conspiracy to kill C. They hired goons to kill him. The goons
demanded a service fee of Rs. 20,000. They asked their servant D to deliver the packet to the
goons. Decide.

(a) D knew of the purpose of delivering the money. Yet he complied. He is liable for
conspiracy.

(b) D is liable for conspiracy to kill as he did not know about the contents of the package

(c) D‘s information about the contents of the package does not matter. D participated in an
act that led to illegality. Thus, D is liable.

(d) D‘s information solely determines his liability. Without any information, D cannot be said
to be partaking in the conspiracy.
 

Passage (Q.-Q.):Consumer rights are the rights given to a "consumer" to protect him/her from
being cheated by salesman/manufacturer. Consumer protection laws are designed to ensure
fair trade competition and the free flow of truthful information in the marketplace. The laws
are designed to prevent businesses that engage in fraud or specified unfair practices from
gaining an advantage over competitors and may provide additional protection for the weak
and those unable to take care of themselves. Consumer Protection laws are a form of
government regulation which aim to protect the rights of consumers. Who is a consumer
according to the Consumer Protection Act, 19? A consumer is one that buys goods for
consumption and not for the resale or commercial purpose. The consumer also hires service
for consideration. The following are some practices that must be followed by a business
under the Consumer Protection Act. If any defect found the seller should remove the
mentioned defects from the whole batch or the goods affected. For example, there have been
cases where car manufacturing unit found a defect in parts of the vehicle usually they remove
the defect from every unit or they call off the unit. They should replace the defective product
with a non-defective product and that product should be of similar configuration or should be
the same as the product purchased. To provide simple, speedy and inexpensive redressal of
consumer disputes, the CPA envisages 3-tier quasi-judicial machinery at the National, State
and District levels. National Consumer Dispute Redressal Commission, known as National
Commission, deals with complaints involving costs and compensation higher than Rs. One
Crore. State Consumer Dispute Redressal Commission, known as State Commission, deals
with complaints involving costs and compensation higher than Rs. Twenty Lakh and less than
Rs. OneCrore.District Consumer Dispute Redressal Forumknown as District Forum, deals
with complaints involving costs and compensation less than Rs. Twenty Lakh.In this, the
complaining party should not value the complaint more than 20 Lacs and once the complaint
is filed the goods are sent for testing and if they found defective the accused party should
compensate and if the party is dissatisfied can make an appeal with state commission within
30 days. State Commission is set up by each state.

. Savita buys 12 KGs of TATA Slat fearing that a lockdown may be imposed due to COVID-
19. She could only consumer 3KGs however and the lockdown was now lifted. She doesn‘t
want to store a lot of salt in her house and sells some of the packets to her neighbours at cost.
Decide

(a) Savita is a consumer as she bought the salt packets for her domestic consumption

(b) Savita is not a consumer for she resold the salt packets for commercial purpose.

(c) Savita is a consumer as she did not earn profit out of the sale of salt packets

(d) Savita is not a consumer. The resale of salt packets is enough to prove the same. 19 of 36

. Please refer to the facts above. What would be your answer if Savita resold the salt packets
at a markup of Rs. 5 by stating that the same was her fuel cost for the run up to the shop. Her
fuel cost was Rs. 3 however.

(a) Savita is a consumer as she bought the salt packets for her domestic consumption

(b) Savita is not a consumer for she resold the salt packets for commercial purpose.
(c) Savita is a consumer as she did not intend sale of salt packets

(d) Savita is not a consumer. The resale of salt packets is enough to prove the same.

. Anuj procured empty CDs as he was fond of listening to music. He requested a professional
to record some good music for him in those CDs for free. The professional obliged. However,
since he did not like the music he decided to sell those off at double the market price. Decide

(a) Anuj is a consumer for he brought the CDs and hired the service for his own consumption

(b) Anuj is not a consumer. The fact that he earned profit makes him a trader.

(c) Anuj is a consumer. The involvement of commercial purpose is only applicable to


purchase of goods and not hiring of services.

(d) Anuj is not a consumer. There was no hiring of service for a consideration.

. ABC is a car manufacturing company. Sumit purchased a brand-new model called Vegeta
from ABC Company. However, it turned out that the engine of this unit (which was named
Hit) of Vegeta was not very sturdy and used to break. Sumit took the car to the company and
they replaced it with a newer model called the Zen-oh engine, which has different
configuration. Decide

(a) ABC‘s replacement is justified as Sumit got better technology

(b) ABC‘s replacement is not justified as they must replace with the same technology

(c) ABC must pay compensation to Sumit

(d) Sumit must pay more money to ABC

. Please refer to the facts above. Sumit is not happy with the replacement of the newer
technology as the new engine Zen-oh also keeps on faltering. Sumit wants the same engine
again. He decides to drag ABC to the district forum. The value of the car is 25 lakh and the
value of the engine is 12 lakh. Decide

(a) The district forum shall not have jurisdiction as the value of the car is 25 lakh

(b) The district forum shall have jurisdiction as the value of the engine is 12 lakh

(c) The district forums shall not have jurisdiction as Sumit should try and negotiate the matter

(d) None of the above

 
. Rathore buys mustard oil extracting machine. The new model however did not work and he
complained at the District Consumer Forum. Which of the following is true if the machine is
sent for testing and is found to be working normally?

(a) The complaint will be rejected. Rathore cannot do anything now

(b) The complaint will be rejected. Rathore can still appeal to the court withing 40 days

(c) The complaint will be rejected. Rathore cannot ask for a product of similar configuration

(d) The complaint will be rejected. Rathorecan approach the state commission within 20
days.

Passage (Q.-Q.):In a democratic society even the rights of the accused are sacrosanct, though
accused of an offence, he does not become a non-person. In the leading case of Kishore Singh
Ravinder Dev v. State of Rajasthan, it was said that the laws of India i.e. Constitutional,
Evidentiary and procedural have made elaborate provisions for safeguarding the rights of
accused with the view to protect his (accused) dignity as a human being and giving him
benefits of a just, fair and impartial trail. Articles 10 of the UDHR declares that everyone
entitle in full equality to a fair and public hearing by an independent and impartial tribunal, in
the determination of his legal rights and obligation and of any criminal charges against him.
Articles 14(1) of the international covenants on civil and political rights provide that all
people shall be equal before the court and tribunals. There are various facets to the right to a
fair trial. The Hon‘ble Supreme Court in the case of Zahira Habibullah Sheikh &Anrvs State
of Gujarat has held that, ―the principle of fair trial now informs and energizes many areas of
the law. It is reflected in numerous rules and practices.... fair trial obviously would mean a
trial before an impartial Judge, a fair prosecutor and atmosphere of judicial calm. Fair trial 20
of 36 means a trial in which bias or prejudice for or against the accused, the witnesses, or the
cause which is being tried is eliminated.‖ The concept of fair trial entails familiar
triangulation of interests of the accused, the victim and the society and it is the community
that acts through the State and prosecuting agencies. Most of these safeguards to ensure a fair
trial are contained under the Code of Criminal Procedure, 19 which contains and defines the
procedure which has to be followed in criminal cases. Important features that indicate such
fair trial rules are pertaining to an accused person, at least where the charge is an offence
punishable with imprisonment, being entitled to be offered free legal aid, if he is too poor to
afford counsel; The counsel for the accused being given sufficient time and facility for
preparing his defence. The law also provides that any breach of the above-mentioned
safeguards of fair trial would invalidate the trial and conviction, even if the accused did not
ask for legal aid.

. Farid is arrested on charges of theft. A trial is held but Farid has no counsel to represent
him. Not having much luck defending himself, Farid is convicted for the offence of theft.
Choose the correct statement:

(a) Farid‘s conviction is valid because there was proper evidence against him.

(b) Farid‘s conviction is invalid since he was not given legal aid.

(c) Farid‘s conviction is valid because no demand for legal aid was made.

(d) An analysis of Farid‘s socio economic condition needs to be made to arrive at a decision.
 

. Vidisha, an IT consultant, is arrested for cybercrime, punishable with imprisonment. She


had gone bankrupt a few months ago, so much so, that it was hard for her to fend for herself.
She is not provided with a counsel and the trial proceeds as is, after which she is convicted of
theft. Which of the following statements, if true, would make her trial valid?

(a) Vidisha‘s friends were of good socio economic background. Non-engagement of a


counsel is reflective of her intention.

(b) Vidisha‘s parents were of good socio economic background. Non-engagement of a


counsel is reflective of her intention.

(c) Vidisha was well–read in the law but did not ask for legal aid. Her conduct implies that
she did not intend to engage a counsel.

(d) Vidisha‘s own socio economic conditions may be good enough to hire a lawyer.

. Ishika is arrested on charges of house–trespass. She does not have the means to sustain
herself. The State forgets to assign her a legal counsel. A day before the trial, a legal counsel
is assigned to Ishika. She briefs the counsel about the case and the hearing is held the next
day. Ishika is convicted of house– trespass. Have the rules of fair trial been violated in
Ishika‘s case?

(a) The very fact that hearing was held only after 1 day of preparation for the case means that
the rules of fair trial have been violated.

(b) Ishika was assigned a counsel which is enough to not vitiate a trial on grounds of
violation of fair trial.

(c) It depends on the expertise of the counsel and the fact whether he could prepare in one
day or not.

(d) The fact that the counsel could have asked for an adjournment is enough to show that fair
trial was given.

. Please refer to the facts above. Ishika is in jail and wishes to file an appeal. The police rules
allow for a lawyer to only meet their client for 15 minutes once in a day. The lawyer based on
those meetings prepared a defence. Decide

(a) There is a violation of rules of fair trial as the lawyer has not been given enough time to
prepare a defence

(b) There is no violation of the rules of fair trial for the rules stipulate such duration of 15
minutes

(c) There is a violation of rules of fair trial as the counsel isn‘t learned enough
(d) There is no violation of rules of fair trial. The lawyer could have come on the next day.

. A small child was fighting a legal battle for a sexual assault committed onto her by a man.
She wishes for the matter to be heard by the judges in private. The judges want to hear it in
open court.

(a) The child is not entitled to ask for a private hearing as there is no provision to do so.

(b) The child is entitled to ask for a private hearing as a public hearing is only an entitlement
and not a mandatory requirement.

(c) The child is not entitled to ask for a private hearing as the judges must do justice

(d) The child is entitled to a private hearing as a public hearing may be against morality and
decency.

. Please refer to the facts above. The accused‘s lawyer asks for a trial in open court. Decide

(a) The accused is not entitled to a public hearing as the matter is sensitive

(b) The accused is entitled to a public hearing in the interest of justice

(c) The accused is not entitled to a public hearing as he is condemnable

(d) The accused is entitled to a public hearing as it is within his rights

Passage (Q.-Q.):False Imprisonment may be defined as an act of the defendant which causes
the unlawful confinement of the plaintiff. To constitute false imprisonment certain factors
such as probable cause for imprisonment, knowledge of the plaintiff for imprisonment,
intention of the defendant while causing imprisonment and period of confinement matters.A
private individual, a police officer or any public authority can falsely imprison a person as
well. For purposes of this law, the meaning of the police officer includes every person who is
responsible for the security of a certain place. For imprisonment it is not necessary that the
person should be put behind bars, but he should be confined in such an area from where there
are no possible ways of escape except the will of the person who is confining the person
within that area.It is not the degree of the imprisonment that matters but it is the absence of
lawful authority to justify unlawful confinement which is of relevance.As almost every tort
has a defence, so does this tort. The defences of false imprisonment are consent of the
plaintiff or voluntary assumption of the risk, probable cause and contributory negligence. The
defence of consent of the plaintiff and probable cause are complete defences while the
defence of contributory negligence is used only for mitigation of damages. To invoke the
defence of consent however, courts would always look for whether free consent is present or
not. False arrest is the arrest of the individual by the police officer or private person without
lawful authority. Although we call it false arrest, it is a part of false imprisonment only. False
arrest and false imprisonment are virtually indistinguishable except in their terminology and
have been held by the courts as a single tort. There are three remedies for false imprisonment.
They are damages, habeas corpus and self-help. Being a tort, the basic remedy for false
imprisonment is an action for damages which can be due to physical or mental suffering, loss
of reputation or even malicious intent on behalf of the defendant. If a person is unlawfully
confined then he can be released from such confinement by the writ of habeas corpus. A
person can also use reasonable force in order to escape from the confinement.

. Rajshekhar is a lawyer who has a flourishing practice in Bangalore. One day while
preparing for a case, he was in office till late night along with his juniors. One of his juniors
desperately wanted to go back to his home but was not allowed by Rajshekhar. The junior
told Rajshekhar about the risk of theft in Nagarbhavi. Rajshekhar asked him to resign if he
wanted to go. The junior did not leave and theft occurred in his house at night. Decide

(a) Rajshekhar is liable for false imprisonment for he did not have lawful authority to confine

(b) Rajshekhar is not liable for false imprisonment for there was consent of the junior

(c) Rajshekhar is liable for false imprisonment for he is a private individual

(d) Rajshekhar is not liable for false imprisonment for there was voluntary assumption of risk
on part of the junior.

. Rohit is a friend of Suryonshu. Suryonshu is employed as a security guard in a private


university. Rohit often visits his friend in his workplace. One day when Rohit went and met
Suryonshu, he seemed to be in a hurry to go to the gym with his mother. He asked Rohit to
help him out watch his place as security guard for some time. Rohit agreed. While Rohit was
doing Suryonshu‘s duties, he happened to catch a student smoking and confined him for 1
hour in his office till Suryonshu returned. The penalty for smoking was onlya fine of Rs. 0.
Decide

(a) Rohit is liable for false imprisonment as he did not have any lawful authority to confine

(b) Rohit is liable as police officers can also falsely imprison a person without having lawful
authority

(c) Rohit is not liable as the degree of imprisonment is not high and he was only confined for
1 hour

(d) Rohit is not liable for false imprisonment as he is a police officer at the relevant time.

. Rohit is a friend of Suryonshu. Suryonshu is employed as a security guard in a private


university. Rohit often visits his friend in his workplace. One day when Rohit went and met
Suryonshu, he seemed to be in a hurry to go to the gym with his mother. He asked Rohit to
help him out watch his place as security guard for some time. Rohit agreed. While Rohit was
doing Suryonshu‘s duties, he happened to catch a student smoking and confined him for 1
hour in his office till Suryonshu returned. The penalty for smoking was grounding for 1 hour.
Decide

(a) Rohit is liable for false imprisonment as he did not have any lawful authority to confine
(b) Rohit is liable as police officers can also falsely imprison a person without having lawful
authority.

(c) Rohit is not liable as the degree of imprisonment is not high and he was only confined for
1 hour.

(d) Rohit is not liable for false imprisonment for he had lawful authority to confine the person
as per college smoking penalty rules.

. Asur is a man of his word and never indulges in corrupt practices. Dev one day asked him
for a favour which Asur deemed to be unjust and refused. Dev held Asur‘s hands till he
agreed to do it. Asur agreed. Decide

(a) Dev is liable for false imprisonment as he did not have authority to lawfully confine Asur

(b) Dev is not liable for false imprisonment as Asur had consented to the same

(c) Dev is liable for false imprisonment as his demands were unjust

(d) Dev is not liable for false imprisonment as here merely held Asur‘s hands.

. Please refer to the facts above. Since Asur was not agreeing to Dev‘s demands, Dev locked
himself in a room till Asur agreed to Dev‘s demands. Dev later suedAsur for false
imprisonment.

(a) Asur is liable for false imprisonment he was the cause for Dev‘s confinement in a room.

(b) Asur is not liable for false imprisonment as Dev has consented to himself being confined

(c) Asur is liable for false imprisonment as he held Dev captive

(d) Asur is not liable for false imprisonment as he is morally upright

. Kansa had confined his sister and her husband in prison. Whenever his sister gave birth to a
child, Kansa vowed to come in and kill the child. When Kansa came in to kill the 1st child of
his sister, his sister, with energy from her eyes killed Kansa and ran away.

(a) His sister is liable for causing Kansa grievous hurt (b) His sister is not liable for causing
Kansa grievous hurt

(c) His sister was acting in self defence for Kansa had falsely imprisoned her. She used a
reasonable force

(d) His sister was not acting in self defence for Kansa had falsely imprisoned her. She used
unreasonable force
Passage (Q.-Q.):The crimes of assault, assault and battery, and aggravated assault all involve
intentional harm inflicted on one person by another. Any crime involving a physical attack is
usually classified as an assault, a battery, or both. Depending on the seriousness of the attack
(or the dangerousness of the weapon used), these acts can rise to the level of aggravated
assault. And more than one-sided attacks can constitute assault. Fighting can lead to an
assault charge, even when two people have mutually agreed to fight.Assault is sometimes
defined as any intentional act that causes another person to fear that she is about to suffer
physical harm. This definition recognizes that placing another person in fear of imminent
bodily harm is itself an act deserving of punishment, even if the victim of the assault is not
physically harmed. This definition also allows police officers to intervene and make an arrest
without waiting for the assaulter to actually strike the victim. Historically, battery and assault
were considered separate crimes, with battery requiring that the aggressor physically strike or
offensively touch the victim. In that way, a battery was a ―completed‖ assault. The criminal
laws of many states classify assaults as either simple or aggravated, according to the gravity
of the harm that occurs—or is likely to occur if the assaulter follows through and strikes the
victim. Aggravated assault is a felony that may involve an assault committed with a weapon
or with the intent to commit a serious crime, such as rape.An assault may also be defined as
aggravated if it occurs in the course of a relationship that the legal system regards as worthy
of special protection. It turns out that words alone won't usually give rise to assault charges
either. The general policy against punishing people for naked threats recognizes that in the
heat of the moment, people often make threats they have no intention of carrying out.

. Hitesh and Tejas were flying in an aircraft. Hitesh did not notice that the seatbelt he was
going to remove was actually Tejas‘. After he removed the seat belt, there was sudden
turbulence in the aircraft and Tejas fell down and hit his head against the seat in the front.

(a) Hitesh is liable for battery because his action was not justified

(b) Hitesh is liable. Intention is irrelevant.

(c) Hitesh is not liable as he did not intend to commit battery

(d) Hitesh is not liable as he did not hurt Tejas majorly.

. Munib and Shaqib are best friends. They were playing pranks onto each other
meanwhileMunib fell on the ground. After Munib fell on the ground, he felt that Shaqib had
humiliated him and got up to punch Shaqib on the face. Shaqib punched him back too and a
scuffle happened. Decide

(a) Shaqib is liable for assault as he started it first

(b) Munib is liable for assault as he started the scuffle first

(c) Both are liable for assault

(d) None is liable for assault as they mutually agreed to fight.

 
. Please refer to the facts above. After Munib felt on the ground, he felt insulted and
threatened Shaqib that he would beat him to death. The moment Munib said this, the police
arrested him. Munib questioned the police‘s powers

(a) The police can arrest Munib the moment a threat of assault has been made

(b) The police cannot arrest Munib. Assault has not been committed yet

(c) The police must arrest Munib as he is morally culpable

(d) The police cannot arrest Munib as he is a victim.

. Mehta was playing cricket when the leather ball hit Harshil. Infuriated, Harshil threatened
Mehta saying that he would call the police. Mehta question police‘s powers of arresting him.
Decide

(a) The police cannot arrest Mehta since naked threats are not taken cognizance of.

(b) The police can arrest Mehta since it constitutes assault

(c) The police should not arrest Mehta since naked threats are not taken cognizance of

(d) The police can arrest Mehta since he violated peace.

. Please refer to the facts above. When Harshil got hit by Mehta‘s ball, he fell on the ground
and found a knife lying on the ground. He attacked Mehta with the knife injuring him.

(a) Harshil is liable for assault

(b) Harshil is liable for aggravated assault

(c) Harshil is liable for a felony

(d) Harshil is not liable

Passage (Q.-Q.):Privacy includes at its core the preservation of personal intimacies, the
sanctity of family life, marriage, procreation, the home and sexual orientation. Privacy also
connotes a right to be left alone. Privacy safeguards individual autonomy and recognises the
ability of the individual to control vital aspects of his or her life. Personal choices governing a
way of life are intrinsic to privacy. Privacy protects heterogeneity and recognises the plurality
and diversity of our culture. While the legitimate expectation of privacy may vary from the
intimate zone to the private zone and from the private to the public arenas, it is important to
underscore that privacy is not lost or surrendered merely because the individual is in a public
place. Privacy attaches to the person since it is an essential facet of the dignity of the human
being. -KS Puttaswamy v UoI
. The government in the wake of the 2011 bomb plantings, issued a directive mandating the
set-up of CC TV cameras inside the bathrooms of public malls. The cameras were to installed
outside the toilet stalls  but within the bathroom compound to keep a check on who enters and
monitor the activities of the janitors.

(a) This shall amount to an invasion of the right to privacy of the janitors.

(b) This is an invasion to the privacy of individuals who enter the restroom

(c) This is an accepted practice; as long as the cameras are installed outside the cubicles there
is no privacy concern

(d) None of the above

. The parliament proposed a Bill which sought to create a provision in the IPC that in cases of
murder suspects the police will have unrestricted access to search a person's body, house, his
personal belongings including his electronic equipment and any communication he has had
with anyone without a warrant of any sort. Strictly based on the above passage, would such a
bill be constitutional?

(a) The bill is unconstitutional

(b) The bill is constitutional as it is based on the reasonable classification of a ‗murder


suspect‘

(c) The bill is unconstitutional inasmuch it talks about an ‗unrestricted‘ access.

(d) None of the above.

. The parliament proposed a new law which stated that when a person is sent to prison for the
offence of being involved in terrorist activities or sedition then after the person has completed
his prison sentence, once released, that person would have a GPS chip implanted near his
ankle (permanently) which if tried to tamper with would alert the authorities immediately.
This process would take place under expert medical supervision and safety. Decide the
constitutionality of the law.

(a) This law is constitutional

(b) This law is unconstitutional and likely to be struck down

(c) This law is constitutional inasmuch it makes a classification between ordinary convicts
and those involved in terrorists‘ activities.

(d) None of the above.

. Based on the understanding of the passage choose the best statement that follows:
(a) An individual‘s privacy is limited to the individual‘s private auspices. The moment the
individual becomes a part of a public gathering, his privacy is naturally compromised.

(b) An individual‘s reproductive choices are covered under his right to privacy.

(c) Privacy pertains to the individual‘s choices that conform to the societal standards. As long
as the conformity is maintained, the individual choices are protected under privacy.

(d) All of the above.

. A new order proposed after an attack on the Delhi High Court by the members of Jaesh was
to make it mandatory for any individual to reveal his name, nationality, age and gender
before entering any Court in India. Decide the constitutionality of the law proposed:-

(a) Constitutional

(b) Unconstitutional

(c) partially Constitutional

(d) none of the above

Passage (Q.-Q.):Force Majeure (FM) event means extraordinary events or circumstances


beyond human control such as an event described as an act of God like a natural calamity or
events such as war, strike, roits, crimes but not including negligence or wrong-doing,
predictable/seasonal rain and any other event specifically excluded in the clause. FM clauses
are typically used in commercial contracts in case such specified conditions prohibit
contractual obligations from being fulfilled. A contract to do an act which, after the contract
is made, becomes impossible, or, by reason of some event which the promisor could not
prevent, unlawful, becomes void when the act becomes impossible or unlawful. Where one
person has promised to do something which he knew, or with reasonable diligence, might
have known, and which the promisee did not know, to be impossible or unlawful, such
promisor must make compensation to such promisee for any loss which such promisee
sustains through the non- 25 of 36 performance of the promise. A party to a contract will only
be able to consider Force Majeure to excuse non-performance if there is a Force Majeure
clause in the contract. For an event to be covered under FM, a party will have to show that: ·
An FM event has occurred which is beyond its control; and · It has prevented, hindered or
delayed its performance of the contact; and · It has taken all reasonable steps to avoid or
mitigate the event or its consequences. A FM clause in the contract frees both the parties
from contractual liability or obligation when prevented by such events from fulfilling their
obligations under the contract. A FM clause does not excuse a party‘s non-performance
entirely, but only suspends it for the duration of the FM. The firm has to give notice of FM as
soon as it occurs and it cannot be claimed ex-post facto. Section 56 of Indian Contract Act
lays down that a contract to do an act which, after the contract is made, becomes impossible,
or, by reason of some event which the promisor could not prevent, unlawful, becomes void
when the act becomes impossible or unlawful. And this is known a doctrine of frustration.
This doctrine comes into play in two types of situation, first, where the performance is
physically cut off, and, second, where the object has failed, If there are more than one ways
of performing a contract and the war cuts off only one of them, the party is still bound to
perform by the other way, however inconvenient or expensive.

. John agreed to sell to George three hundred tons of Sudan Groundnuts. The usual and
normal route at the date of the contract was via Suez Canal. Shipment was to be in
November/December. 16, but on November 2, 16, the Canal was closed to traffic and it was
not reopened until the following April. John had an option to transport the goods via the Cape
of Good Hope but he refuses to ship via the cape. Decide whether by reason of the closing of
the Suez route, the contract has been ended by frustration?

(a) The contract is frustrated since it was an implied term of the contract that shipment should
be via Suez.

(b) The contract is frustrated since the closure of Suez Canal renders the performance of
contract frustrated.

 (c) The contract is frustrated even though there is another option available but it will add on
to the cost John.

(d) The contract is not frustrated because there is no impossibility in performing the condition
of the contract since there are different options available to deliver the groundnuts.

. There was a contract between X and Y, who was an eminent pianist, that she should play the
piano at a concert to be given by X on a specific day. On the morning of the day of concert, Y
informed X that he was too ill to attend the concert. The concert had to be postponed and X
lost a huge sum of money. X brought an action for breach of contract against Y in the court of
law which was opposed by Y contending the frustration of contract. Decide the liability of Y.

(a) Y should not be held liable since her condition did not allow her to perform and her
performance was the foundation of the contract, failing which the whole contact rendered
frustrated.

(b) Y should be held liable for breach of contract since she did not perform on the day of
concert.

(c) X‘s claim of breach of contract should succeed since he incurred financial loss while
arranging of the essentials of the contract.

(d) Y should not be held liable since her health should be given priority over her
performance.

. There was contract between Zoya and Akhatar, they both belong to Kaziabad, for the supply
of ―Krupps‖ steel rails, which is manufactured in Germany. But the contract deed does not
specifically ask for Krupps from Germany only. Zoya failed to supply the Krupps to Akhatar
due to the outbreak of World War 2 which declared Germany as an alien nation of Kaziabad.
And any kind of trade with alien enemy is not possible. Zoya claimed that his performance of
the contract became impossible and illegal. Decide whether contract between Zoya and
Akhatar become frustrated because of the World War 2?

(a) Any supervening act which is beyond the control of supplier renders the contract
frustrated.

(b) The contract required Zoya to supply the Krupps steel rail which is manufactured in
Germany but after the war Germany became an alien enemy which makes any trade with it
impossible.

(c) The contract is rendered frustrated since the outbreak of World War 2 is a supervening act
which make the performance of contract impossible.

(d) The contract is not frustrated since there are still other sources available with Zoya to
supply the goods and the performance of the contract is still possible.

. Suppose A and B entered into a contract where A was required to supply wall paint from his
warehouse which is located 500 kms away from B‘s shop. A used to supply the stock to B on
weekly basis. Unexpectedly there was an outbreak of Corona virus in the country. The
government asked people for complete self-isolation for 21 days and any kind of public
gathering was strictly prohibited. Under these circumstances A was unable to supply stock for
the next three weeks. B sued A for breach of contract. Decided whether A is liable or not?

(a) A is liable for breach of contract since he has an obligation to provide stock on weekly
basis which he did not supplied.

(b) A is not liable since the circumstances were out of his control and it was impossible for
him to supply goods in those three weeks since that will violate government order.

(c) A is not liable since there is complete police guard and escaping them to supply the stock
is very difficult task.

(d) A is liable since because of him the business of B is hampered.

. By the contract in writing Jai brought to Vijay a number of dhotis to be manufactured by


specified mills and to be delivered as when the same may be received from the mills. The
seller delivered only the part of the goods owing to the mills failing to perform their contract
with Vijay as they were engaged in fulfilling certain government contracts. Jai brought an
action for breach of contract against Vijay but later pleaded frustration in the court of law.
Decide

(a) This is a breach of contract and not the frustration of contract since the closing or even the
destruction of the mills would not affect the contract between third parties, which is in term
absolute.

(b) This is a case of frustration since the delivery of goods is not in the hands of Vijay.
(c) This is not a case of frustration since Vijay should have engaged more with the mills and
should have ensured the delivery.

(d) This is not a breach of contract since Vijay had tried his best to procure the goods.

7
Passage (Q.-Q.):Mrs Donoghue's friend ordered a drink. Then, her friend was alleged to have
committed an act of generosity that has haunted our law of tort ever since. She purchased for
Mrs Donoghue an ice cream and a ginger beer. Had Mrs Donoghue purchased the ginger beer
herself she would have had a perfectly good action for breach of contract against the retailer.
The ginger beer came in an opaque glass with the name of the manufacturer, David
Stevenson, on the side. Her friend poured part of her drink and Mrs Donoghue drank it. As
her friend was proceeding to pour the remainder of the contents of the bottle into the tumbler,
a snail, which was in a state of decomposition, floated out of the bottle. Mrs Donoghue
subsequently contracted what her doctors described as “severe gastro-enteritis”. Mrs
Donoghue sued the manufacturer, Stevenson and Co. The manufacturer brought a plea of
relevancy (similar to a demurrer) all the way up through the Scottish courts to the House of
Lords. A manufacturer of products, which he sells in such a form as to show that he intends
them to reach the ultimate consumer in the form in which they left him with no reasonable
possibility of intermediate examination, and with the knowledge that the absence of
reasonable care in the preparation or putting up of the products will result in an injury to the
consumer's life or property, owes a duty to the consumer to take that reasonable care. The
liability for negligence, whether you style it such or treat it as in other systems as a species of
"culpa," is no doubt based upon a general public sentiment of moral wrongdoing for which
the offender must pay. But acts or omissions which any moral code would censure cannot in
a practical world be treated so as to give a right to every person injured by them to demand
relief. In this way rules of law arise which limit the range of complainants and the extent of
their remedy. The rule that you are to love your neighbour becomes in law, you must not
injure your neighbour; and the lawyer's question, who is my neighbour? receives a restricted
reply. You must take reasonable care to avoid acts or omissions which you can reasonably
foresee would be likely to injure your neighbour. Who, then, in law is my neighbour? The
answer seems to be - persons who are so closely and directly affected by my act that I ought
reasonably to have them in contemplation as being so affected when I am directing my mind
to the acts or omissions which are called in question. . Neha runs a small tailoring business
where she procures fabric from the local market and stitches frocks for toddlers. Neha was
sued by three of her clients for selling them frocks which resulted in allergic reaction to the
toddlers. When investigated, it was found that the fabric manufacturer added chemicals in
more than the required quantity resulting in bad quality of fabric causing allergy to tender
skin of toddlers. Neha argues that it should be the liability of the local manufacturer of fabric
as she was only responsible for the stitching part. Decide. (a) Neha is not liable as in the
current fashionable world, one should not expect high quality product. (b) Neha is not liable
as her clients are not her neighbours and she doesn’t owe any duty towards them. (c) Neha is
not liable as the damage was not forceable at her end and she did not act negligently with
respect to stitching of garments. (d) Both (b) and (c) . Referring to the above question, local
manufacturer denies liability stating that Neha’s clients do not qualify as manufacturer’s
neighbours. Applying the rules stated in the paragraph decide whether manufacturer will be
liable or not? (a) Manufacturer will be liable as acted negligently while adding more than
required quantity of chemicals in the fabric. (b) Manufacturer will be liable as the clients of
Neha are directly affected by the act of the manufacturer. (c) Manufacturer is liable as he
breached his duty of care resulting in damage. (d) Manufacturer will not be liable as his duty
is only restricted till he sells fabric to the tailors and not beyond that. 18 of 36 . Aman runs
Amul Milk Parlour where he also sells daily essentials products including bread from Modern
Bread Co and Biscuits from Sunfeast Co. Mr. Duggal bought a packet of bread from Aman’s
shop and after eating it he broke his teeth due to a stone present in the bread. Mr. Duggal
sued Aman for his injuries. Decide. (a) Mr. Aman will be held liable as Mr. Duggal suffered
injuries due to his direct act of selling him a packet of bread. (b) Mr. Aman is not liable as he
is a mere seller and is not responsible for negligence in production. (c) Mr. Aman is not liable
as he took reasonable care and such injuries were unforeseeable. (d) Mr. Aman is liable for
all the issues resulting from the product he sold in his shop, including Modern bread. . In the
above scenario, had Mr. Duggal found the bread to be stale and tempering with the expiration
date on the packet of bread. How will Aman’s liability change from the above question. (a)
Liability will remain same. Aman will not be held liable. (b) Liability will be mitigated.
Aman can bargain for relief in liability. (c) Liability will be reversed. Aman will be held
liable. (d) Liability will intensify. Aman will pay heavy damages. . KEM Hospital purchased
PPE kit from Mahesh Babu, who is a medical equipment supplier of the town. Hospital
authority realised that the kit supplied to them were defective and were of no use while
treating covid 19 patients. When a case was brought against Mahesh Babu, he argued that it is
beyond his duty of care as he is just a supplier. Decide. (a) Mahesh Babu will be held liable
as it is duty of the supplier to check for defects in the supplies. (b) Mahesh Babu will not be
held liable as his duty of care is limited to the supply related issues and not manufacturing
defects. (c) Mahesh Babu will not be liable as he did not breach his duty of care. (d) Both (b)
and (c) . Shalini ordered as expensive piece of glass crockery from an e-commerce website
called, Maalkart. When she received the delivery after a week, it was found the two bowls in
the set of six were broken. When she raised a complaint with Maalkart, they shrug off their
responsibility by citing that it’s the manufacturing defect. On investigation it was found that
the glass crockery manufacturer shipped the prefect piece and the damage occurred while the
products were in transit. Advise Shalini. (a) Shalini should file a case against the
manufacturer as it is manufacturer’s responsibility to make sure there is no damage done
during transit. (b) Shalini should file a case against both the manufacturer as well as the
MaalKartcompany. (c) Shalini should file a case against the Maalkartcompany as the damage
was done by the seller while the products were in transit and not by the manufacturer. (d)
Shalini should not indulge in court proceeding and should ask for a refund.

Passage (Q.-Q.):Clause (3) of Article 20 of the Indian Constitution, 10 provides that “No
person accused of any offence shall be compelled to be a witness against himself.” This
principle is espoused on the maxim “nemo teneteurprodre accussarese ipsum”, which
essentially means “NO MAN IS BOUND TO ACCUSE HIMSELF.” The Fundamental Right
guaranteed under Article 20(3) is a protective umbrella against testimonial compulsion for
people who are accused of an offence and are compelled to be a witness against themselves.
The provision borrows from the Fifth Amendment of the American Constitution which lays
down that, “No person shall be compelled in any criminal case to be a witness against
himself”, same as mentioned in the Constitution of India embodying the principles of both
English and American Jurisprudence. This libertarian provision can be connected to an
essential feature of the Indian Penal Code based on the lines of Common Law that, “an
accused is innocent until proven guilty” and the burden is on the prosecution to establish the
guilt of the accused; and that the accused has a right to remain silent which is subject to his
much broader right, against self-incrimination. This protection by the Indian Constitution is
also extended to suspects. Article 20 clause 3, has been carefully crafted to protect 19 of 36
the accused from further self-incriminating himself only if any statement of his might result
in prosecution. This article also stretches its privileges to a person who is compulsorily being
made a witness and also covers searches and seizures wherein, an accused or the person being
searched is under no obligation to be a part of the search. The tendency of Indian legal system
manifests skepticism of the police system. This is the reason confessions of an accused is
only admissible if recorded by a Magistrate in accordance with an elaborate procedure to
ensure that they are made voluntarily. Protection is also accorded by the provisions of The
Indian Evidence Act. This protection is available to every person including not only
individuals but also companies and incorporated bodies.

. Bhanu has been alleged to have to have committed the murder Mr. Pratap. The police have
been undertaking its investigation to gather evidence to substantiate the case. They decided to
take the fingerprints of Bhanu to see whether it matches with the weapon found by them.
Bhanu claims the violation of Article 20(3). Decide.

(a) The claim is justified as this reveals his identity in a forceful manner.

(b) The claim is justified as this compulsion would result in prosecution.

(c) The claim is not justified as only objective information is being brought out without
forcing him to reveal anything.

(d) The claim is justified along with a violation and invasion of privacy.

. Based on the factual information provided in the aforementioned question, whether DNA
profiling of Bhanu would be violative of Article 20(3). Decide.

(a) Yes, as brings out information in a without control of individual which can be considered
forceful.

(b) No, as there is no element of compulsion involved under this.

(c) Yes, as it seeks for information through an invasion of privacy.

(d) No, as it is not a testimonial act through personal knowledge.

. The police came to Dholu’s house to conduct a search for a particular case. He was not
asked to be a part of the search process. In furtherance to the same, certain documents were
found by the police inviting prosecution for Dholu. Dholu alleged for self-incrimination.
Decide
(a) This amounts to self-incrimination as the documents were forcefully taken.

(b) This amounts to self-incrimination as it invited prosecution for Dholu.

(c) This does not amount to self-incrimination as there were no statements made by Dholu.

(d) This does not amount to self-incrimination as he was not forced to be a part of the search.

. Rohan is taken under the custody for the charges of murder of Jeevan. The police conducted
an interrogation and told him that they found the weapon. They told him to confess the same.
Rohan confesses the crime in the court but later finds out that the weapon was not found by
the police. He claims protection under Article 20(3) saying he was forced to make the
statement and they cannot be used against him. Decide.

(a) The claim is justified as it was forceful as it was brought out through false information.

(b) The claim is justified as it was self- incriminating in nature.

(c) The claim is not justified as it was a confession made under police custody.

(d) The claim is not justified as it was not through compulsive means.

. Ram went to the residence of the Deputy Superintendent of Police. He handed him an
envelope. When the DSP opened the envelope, he found a certain amount of cash. This meant
Ram had come to bribe the officer. The DSP refused it. However, he asked Ram to place the
envelope and the currency notes on the table. As soon as Ram did that he said that the cash
was seized by the Police. Would this amount to violation of Article 20(3)? Decide.

(a) It does not amount to violation of article 20(3) as he bribed the officer.

(b) It does not amount to violation of article 20(3) as Ram was under no duress.

(c) It amounts to violation of article 20(3) as Ram was compelled for the amount.

(d) It amounts to violation of article 20(3) as he was a part of the seizure.

Passage (Q.-Q.):Ostensible literally means apparent or seeming. An Ostensible owner is a


person who apparently or seemingly appears to be the owner, though in reality he is not. He
is person having alindicas of ownership having being real ownership. He is different from a
mere trespasser or a person in unlawful occupation of the property. His behaviour and
conduct appear to be that of the owner of the property with the consent or conduct of the real
owner. Transfer by ostensible owner (Section 41 of Transfer of Property Act) - Where, with
the consent, express or implied, of the persons interested in immoveable property (within
territorial jurisdiction), a person is the ostensible owner of such property and transfers the
same for consideration, the transfer shall not be voidable on the ground that the transferor
was not authorized to make it: Provided that the transferee, after taking reasonable care to
ascertain that the transferor had power to make the transfer, has acted in good faith. 1) Free
Consent- By his conduct or consent or otherwise makes the other believe that a particular
person has all the powers over the property as that of the owner over the,including that of
alienation, the transferor should be shown to have been the ostensible owner with the express
or implied consent of the true owner but the transfer itself need not be with the consent of the
true owner. The consent includes express or implied as well. Negligence may amount to
implied consent. 2) Alienation - Such person in fact is not authorized to alienate the property.
He alienates itas an ostensible owner. 3) The transfer is for value or consideration, 4)
Transferee must take reasonable care- The transferee acts bona fide and has taken reasonable
care to ascertain that he is competent to transfer, i.e., the transferee does not have actual or
constructive notice of the real facts, The transferee must show that he had made such enquires
as a reasonable prudent man would have taken to safeguard his interests.The burden of
proving that the transferee was an ostensible owner is on the transferee who seeks the
protection of this section. 5) Estoppel- The real owner would be prevented on disputing the
validity of the transfer onthe ground that the transferor was not, in fact, competent to do so.
Benami transactions are one of the most notorious sources of circulation and investment of
black money. The Government has recently been facing a great deal of heat on the issue of
black for not doing enough to deal with this menace. Sadly, such transactions are rampant in
the real estate sector of our country.

. A gift deed was executed by father, who was the ostensible owner of the property in favor of
his daughter and the consideration for the transfer was contended to be love and affection in
favor of his daughter. Decide the validity.

(a) Invalid as ostensible property cannot be transferred to a family member.

(b) Invalid as there is not alienation of property as the daughter will become the owner and
hence it remains with them.

(c) Invalid as it was done with free consent and without any duress, with the consent of the
owner.

(d) Invalid as ostensible property cannot be transferred without any consideration.

. Alexander bought the property in the name of BunooBebee. In June 13, she sold the
property to RamdhoniKundoo (father of RamcoomarKundoo) by a sale deed. After which he
and after his death, his heir enjoyed undisputed ownership for 24 years. Alexander made a
will and the will be stated that after Alexander’s death, the property would go to Macqueen,
who was the daughter of Alexander and BunooBebee. When the will came to the knowledge
of BunooBebee, a case was filed contending that BunooBebee didn’t have the right to sell the
property as she was merely an ostensible owner. Alexander had no intention to transfer the
property to BunooBebee. Decide.

(a) The will would be valid as during the sale, there was no way for knowing that alexander
had other intentions.

(b) Invalid as Bunoo did not have the consent of the owner to alienate this property.
(c) It would be a valid deed, as the transfree to the best of this knowledge had the expressed
consent that bunoo could make this deed.

(d) It would be invalid as alexander left the property to his daughter and as the true owner his
say would trump Bunoo’s say in the deed.

. The owner of a property died and thereafter the possession of his property was with his
illegitimate sons who were not entitled to the legal title of the property. The legal heirs of the
owner filed a suit against the illegitimate sons to recover the property. However, the
illegitimate sons sold the property to a third party, claiming themselves to be ostensible
owners. The court held that section 41 cannot be invoked. Why?

(a) The court could not as they would be barred by the principle of estoppel.

(b) The court did not invoke 41 stating that the illegitimate heirs were not ostensible owners
in their rights.

(c) The court did not invoke 41 as it’s not a matter of Transfer of property act, it is criminal
case of fraud.

(d) The court did not invoke section 41 as the transferee should have taken due care properly
before it.

. A husband while leaving for pilgrimage entered his land in the revenue records under his
wife’s name. He subsequently allowed her to mortgage the land. When the husband left, the
wife sold the land to a third party and the purchaser paid off the mortgage. The husband filed
a suit to recover or redeem the land from the purchaser, decide.

(a) The purchaser would have to return the land as he did not do a proper check of the land
and its ownership.

(b) The purchaser would not have to return as the wife had the consent of the husband to be
ostensible property.

(c) The purchaser would win the case as he would invoke the principle of estoppel.

(d) Both b and c are valid arguments in favor of the purchaser.

. A Burmese husband from the very outset allowed his wife and children to hold themselves
out as sole owners of the property and to deal with it as if they were its owners. They
mortgaged the property to the mortgagee who acted in good faith and without notice of the
title of the husband. Decide.

(a) The buyer would be entitled to the ownership of the land as he would have the right based
on estoppel.
(b) As the wife and children are the ostensible owner they do not have to notify the husband.

(c) The husband had given them free consent to alienate the property hence there is no
conflict.

(d) Cannot be decided as this is an Indian law, and not applicable outside the territory.

. On the death of their widowed mother, the daughters succeeded to the property of their
father. But, thereafter they allowed a parental agnate who was treated as a member of the
family to take possession of the property and they allowed him to enter into an agreement of
sale of a portion of the property and they allowed him to do so despite the fact that the eldest
of them was living with him, others were far away but had relations to watch their
interest.Decide.

(a) Even though he was appointed as ostensible owner he cannot sell the land without the
consent of all the heirs.

(b) Since the elder daughter was living with him, and had given the consent it was on behalf
of everyone.

(c) The parental agnate had implied consent as he was already the ostensible owner.

(d) The relatives had a share and eye on the acts and hence they would have given or
withdrawn consent if they felt like it.

Passage (Q.-Q.):In January 2020, while reviewing all orders of restrictions imposed in
Jammu and Kashmir in the aftermath of the scrapping of the state’s special status, the
Supreme Court declared that the indefinite suspension of the internet was not permissible and
repeated shutdowns amounted to an abuse of power. More importantly, the court held that
“the freedom of speech and expression and the freedom to practice any profession or carry on
any trade, business or occupation over the medium of internet enjoys constitutional protection
under Article 19(1) (a) and Article 19(1)”. The court also held that the restrictions on the
internet have to follow the principles of proportionality under Article 19(2). The judgment by
the court follows the Kerala high court’s judgment in FaheemaShirin v. State of Kerala,
where the right to internet access was recognized as a fundamental right forming a part of the
right to privacy and the right to education under Article 21 of the constitution. Both judgment
establish that the right to internet access has been given limited recognition as an enabler for
other rights within Article 19 and 21. Most academic and non-academic literature, especially
in India, 22 of 36 seeks to locate the right to internet within Article 19. However, an
argument can be made that internet access should be recognized as an independent
fundamental right, conceivable as an aspect of the right to life under Article 21. Internet
access means access to actionable information, innovation best practices, access to finance
and credit facilities, facilitation of entrepreneurship and an enhanced employment market. In
terms of social benefits, internet access helps by facilitating and enabling an enhanced utility
value of primary necessities, internet access can prove a useful accelerator in all social
development objectives and targets of the Sustainable Development Goals.

 
. Because of the pandemic that broke into the country, a lot messages on a messaging apps
and social media were being circulated, giving supposed guidelines for the prevention of the
same. However,considering the large amount of panic the false information was causing, the
government brought out guidelines to curb such circulation over the internet. Is this a
violation of Article 19?

(a) Yes, as right to actionable information is part of this right to internet. 

(b) Yes, it is as curbing of information access, or right to communication and best innovative
practices in private conversation is a right under Article 21 and 19.

(c) No, it’s not as proportional restrictions of fundamental right to internet is a valid act.

(d) No as it only refers to right to occupation, livelihood and education and not right to share
false messages.

. A riot broke down in the district of XYZ and the Magistrate announced a curfew under
section 133 of CrPC. Under the same, he declared the suspension of the all movements and
even internet to curb any false information. Decide the validity of the Act.

(a) It’s invalid as it suspends the access of all to take any action for themselves through any
medium.

(b) It would be valid as long as the suspension is for a limited time period.

(c) It’s invalid as at the time of such unrest people have a right to access information and
news.

(d) It’s valid as it saves people from false information and being traced in their homes.

. A law was passed by the valid authority that any messages related to hate speech or crimes
(anything that might instigate violence against a particular community) cannot be circulated
or forwarded. Any and all people involved in such chain of messages would be held guilty of
this offence. A mob lynching of a deemed lower caste man took place and the news broke of
like fire. Mr. M is a journalist who shared the same over his blog and to his contacts on
messages. Can he be held guilty?

(a) No, he cannot as a journalist he was just passing on information within his rights.

(b) Yes he can as the law disallowed any discussion of such fact and he breached it by
sharing such information.

(c) No, he cannot as the law is invalid and in breach of Article 19 of speech and right to
access information over the internet.

(d) No he cannot as they law may be valid but his message does not amount to hate speech.

 
. Because of the pandemic, a lot of schools decided to switch from offline mode of education.
In light of the same, Ramesh’s school also did the same and requested all their students to
enroll themselves over the online mode of classes. Ramesh was unable to get any device to be
able conduct such classes. Can he claim one under the fundamental right?

(a) Yes as right to internet is in alignment with the right to education under article 21.

(b) No, as right to internet does not give one right to devices to access internet.

(c) Yes, as right to education is also a fundamental right derivative from article 21 and 19.

(d) No, he cannot claim it his rights have not been breached under the law.

. A state emergency has been declared. The government has decided to press censoring
motions on the internet and the information circulating through all forms for media platforms.
It has hampered a lot of activities of the people and the restrictions have been imposed
indefinitely. Is this a constitutional act?

(a) Yes, it is as in the state of emergency and hence this act is proportionate.

(b) No, as the emergency does not qualify for omitting the rights of citizens to access
information.

(c) Yes, it’s constitutionally valid as right can have restrictions, as long as the time period is
limited and qualified.

(d) No, it’s not as emergency does not qualify as constitutionally valid act.  

Passage (Q.-Q.):A Tax is a compulsory contribution to the government, imposed in the


common interest of all, for the purpose of defraying the expenses incurred in carrying out
public functions or imposed for the purpose of regulation, without reference to the special
benefits conferred on making the payment. Necessary corollary that follows is that its
quantum need not commensurate with costs incurred by such public authority. Further, tax is
devoid of any quid pro quo. There exists no connection, whether direct and immediate or
broad and casual between the contributor of tax and benefits. Fee, is defined to be a charge
for a special service rendered to individuals by some governmental agency. Fee may either be
regulatory or compensatory. Where a fee commensurate with the cost of rendering the service
though not in ‘exact arithmetical equivalence’ it is compensatory fee. A fee charged to
regulate or control, is regulatory fee, provided it is not excessive or dominantly intending to
raise revenues for the public authority. Article 301 of the constitution provides for freedom of
trade, commerce and intercourse throughout India. Under Article 302, Parliament, in public
interest, may impose restrictions on this freedom. Article 303(1) asserts that neither
Parliament nor state legislatures can make laws discriminating between states with respect to
trade and commerce. Article 303(2) made an exception in favour of Parliament alone to
enable enactment of discriminating laws in case of scarcity. Article 304(a) enables state
legislations to impose tax on goods, provided these are non-discriminatory. There existed a
controversy whether taxing statutes came under the purview of Article 301. Atibaricase held
that such taxes that directly and immediately restrict trade would fall under the purview of
Article 301. Automobile case carved out an exception to Article 304. Accordingly,
'regulatory measures and measures imposing compensatory taxes for use of trading facilities
do not come within the purview of the restriction contemplated by article 301. Compensatory
taxes were held to be the ones that do not hinder freedom of trade, commerce and intercourse,
instead facilitated the same. Working test for deciding whether a tax is a compensatory

(a) whether the trade is having the use of facilities for better conduct of its business,

(B) paying not more than what is required for providing the facilities.

. What does the phrase “quid pro quo” mean in this article?

(a) It means that tax is not imposed in return of a specific benefit.

(b) It means that tax imposed is not commensurate with cost incurred by the public authority.

(c) It means that tax is not imposed in return of common of interest of all.

(d) None of the above.

. Shirur Math is a religious institution managed by a government appointed manager named


Mr. Ashok Verma. He looked into the management of the funds and invested funds provided
by the government in the maintenance of the math. A law governing religious institutions was
passed by the government which levied an amount of Rs. 500 on the religious institutions to
be paid annually. Such an amount went into the Consolidated Fund of India which was later
utilised for various purposes. Whether the levy imposed by the government through a law
governing religious institution is tax or fee?

(a) Such a levy is not tax because there isquid pro quo involved.

(b) Such a levy is fee because the levy is in exchange of the services provided by government
in maintenance of shirur math.

(c) Such a levy is tax because it is having been levied for common interest of public and is
not for the benefits provided to the math by the government.

(d) Such a levy is fee because the amount levied is proportional to the expenses incurred by
government in maintaining the math.

. The Uttar Pradesh Government in an initiative to promote greenery and protect environment
had built several parks and gardens with many trees and plants in the city of Lucknow. It had
installed swings and see-saws for children and walking paths for adults to take walks. It had
sanctioned Rupees 2 crores for the building the gardens and maintaining them for the public.
It levied a very minimal charge of Rupee 1 for children and Rupees. 3 for adults to access the
public parks and gardens. Is such a charge a tax or a fee?

 
(a) Such a levy is tax because the cost incurred in building and maintaining the parks and
gardens is not proportional to the charge levied.  

(b) Such a levy is fee because it is being charged in return of providing the service of access
to parks to the public.

(c) Such a levy is tax because it has been levied in the common interest of public and not
specifically for giving access to parks and gardens.

(d) Such a levy is fee because it has been levied to recoup the costs of building and
maintaining parks and gardens.

. Aman carried a trade of supply marijuana and other drugs in the city of Gurgaon to the
residents of Gurgaon. Indian law considers marijuana as an illegal drug and prevents its
circulation. The State of Haryana had passed a law banning the sale of marijuana. Aman
contested such law to be violative of Article 301 of Indian Constitution. Would Aman
succeed in his claim?

(a) Yes, because it restricts his freedom of trade and commerce.

(b) No, because unlawful trade does not come within the ambit of Article 301.

(c) Yes, because it is not a reasonable restriction.

(d) No, marijuana created addiction which is harmful for a person.

. Mihika is a seller of bidi leaves residing in Madhya Pradesh. Aleem is a manufacturer of


bidi who lives in Uttar Pradesh. Mihika has a contract with Aleem to sell bidi leaves to
Aleem. It is the responsibility of Mihika to deliver the bidi leaves to Aleem. Mihika has
employed certain road transport services and uses it to deliver the bidi leaves to her
customers. Akhil is one of the truck drivers who has been employed by Mihika to deliver bidi
leaves. Akhil was delivering bidi leaves to Aleem and while crossing the border of Madhya
Pradesh to Uttar Pradesh, he had to pay a toll tax of Rs. 1. Akhil also stayed in a hotel of
Uttar Pradesh at night and had to pay some tax in addition to the room rent. When Akhil
asked for compensation of the taxes paid by him in delivering the bidi leaves from Mihika,
Mihika challenged such taxes to be violative of Article 301. Will Mihika succeed?

(a) No, because the toll tax and tax charged at the hotel are in the form of compensatory tax
to facilitate the trade.

(b) Yes, because the toll tax and hotel tax in no way help in better conduct of the business.

(c) No, because the toll tax and hotel taxes are ordinary charges to be paid in the course of
business.

(d) Yes, to the extent of hotel tax because while toll tax may facilitate the trade by
maintaining the roads but hotel taxes are extra burden on the traders.
 

. State of Kerala recently was flooded and majority of the state was affected because of the
floods. The State of Kerala utilised all of its funds to restore the condition and help the public
with food and other necessary items. Many other states came to help the state of Kerala in
providing necessary goods to the public and to restore the places affected by floods. One such
state was State of Andhra Pradesh which donated funds to Kerala. State of Kerala was able to
restore the condition of the state and started its other economic activities as usual. It passed a
law which imposed a tax on goods like sanitizers, sanitary napkins, cleaning products, etc.
being imported from other states so as to make good the losses which led to scarcity.
Whereas, similar products being produced and sold in Kerala itself was free of any tax. State
of Andhra Pradesh challenged the constitutionally validity of such law. Would State of
Andhra Pradesh succeed?

(a) Yes, because such a tax is discriminatory in nature and is thereby violative of Article 303.

(b) No, because State of Kerala was in scarcity of funds and therefore such a tax falls within
the exception.

(c) Yes, because State of Kerala cannot make discriminatory laws even in the situation of
scarcity.

(d) No, because such a tax is non-discriminatory in nature and thereby not violative of Article
303.

Passage (Q.-Q.):Demonetization was a big shock for the whole nation in 2016 as Rs.500 and
0 currency notes ceased to be a legal tender as a prevailing currency in India. Just like the
currency notes, an act that was legal at one time is suddenly no longer legal and instead is
illegal and prohibited by law. These changing circumstances have led to wrongful
punishment of many individuals who did not actually commit any offense that was
punishable by law and according to their knowledge, that act was actually legal. These laws
which criminalize any act or increase the punishment of any offense are called ex post facto
laws and it has been mentioned in Article 20(1) of the Indian Constitution (1). Ex post facto
25 of 36 law is derived from the Latin word “ex post facto” which means ‘out of the
aftermath’, it is a law that has a retrospective consequence on any act committed, which is not
prohibited by law, before the enactment of a preceding law. Ex post facto laws can either
label an act as an offense with retrospective effect; or increase the punishment that is
prescribed for an act committed in the past. It is important to protect the citizens of India
from being punished for an act done, which was legal when it was done but was criminalized
or the punishment for that act has been enhanced by any act that was formulated later on. The
prohibition under Article 20(1) is only for a conviction or sentence, but not trial procedure.
The objection does not apply to a change of procedure or of court. A literal interpretation of
Article – 20(1) of the Indian Constitution would mean that the safeguards provided under this
article are given against conviction for an act or omission which was not an offense under the
law that existed at the time of the commission and against any increased punishment for the
same act for which the punishment was different at the time of the commission of the act. It is
usually claimed that Article – 20(1) invalidates ex post facto law.

 
. Chitmann Pandey got married to Mahima in 19. They were happy in their marriage until
Chitmann fell in love with Shanaya in 19 and got married to her in 19. The Bigamy act was
passed in 19 laying down that any person who had two spouses living at the time of
enactment of the law would be punishable with an imprisonment of 7 years. Chitmann was
charged under the aforementioned act for having two wives. Chitmann filed a petition
challenging the validity of the Act on the ground that the act was committed in past and
cannot be criminalized by subsequent law. Decide.

(a) Chitmann will not succeed as the Act does not use the term retrospectively and only
criminalizes the current act,

(b) Chitmann will not succeed as the Act prohibits Bigamy in the interest of the public.

(c) Chitmann will succeed as the past acts cannot be criminalized with retrospective effect.

(d) Chitmann will succeed as the punishment of 7 years is not appropriate to the act
committed by him.

. Badri was arrested for the possession of an AK-47 rifle and was booked under Terrorist and
Disruptive Activities (Prevention) Act (TADA) which provided for an imprisonment of 10
years. After his arrest the Government passed a new law Prevention of Terrorist Activities
Act (POTA) that made the possession of a rifle punishable for 20 years. Badri was convicted
and was sentenced to 20 years in prison. He challenged his conviction before the State High
Court on the ground that punishment for the act cannot be enhanced with retrospective effect.
Decide.

(a) Badri will succeed as the law which retrospectively increases the punishment is
unconstitutional.

(b) Badri will succeed as the punishment of 20 years is not reasonable.

(c) Badri will not succeed as the law does not use the term retrospectively.

(d) Both (a) and (b)

. A new taxation law was introduced by the government of India. This law provided that all
the individuals who evaded tax liability in the past six months would pay a higher penalty to
the concerned tax authorities. Kishan decided to challenge the law to be violative of the
fundamental right enshrined in Article 20 of the Constitution of India. Decide.

(a) The challenge is justified as it has increased the punishment through payment of higher
penalty.

(b) The challenge is not justified as it cannot be extended to imposition of fines.

(c) The challenge is justified as it operates through a retrospective effect.

(d) The challenge is not justified because the State has to punish such tax evaders.
 

. Mukesh is a sixteen years old boy who is convicted of committing an offence of house
trespass along with outraging the modesty of a girl aged seven years. In pursuance to the
same, he has been sentenced for a rigorous punishment of six months along with a fine of Rs.
50,000. After the judgment was passed, a new legislation came into existence. This provided
that an individual below the age of 21 years shall not be sentenced to imprisonment.
Therefore, Mukesh claims the benefit of the same. Decide.

(a) Mukesh cannot claim the benefit as the law was not in existence when he committed the
act.

(b) Mukesh cannot claim the benefit as the judgment has been passed.

(c) Mukesh can claim the benefit since rigorous imprisonment is not a proportionate
punishment to his act.

(d) Mukesh can claim ex-post facto law could be applied to reduce the punishment.  

. In order to control the corrupt practices, Prevention of Corrupt Act (PCA), 10 was
introduced. This provided that if any public servant is found in possession of disproportionate
assets without an explanation they shall be found guilty of criminal misconduct. Sachin
challenged the law on the ground that while calculating the disproportionate assets only the
assets acquired after the date of PCA can be taken into account. If the asset acquired prior to
PCA is taken into account then it shall amount criminalizing a past act thus violative of
Article 20. Decide.

(a) The offence of disproportionate assets is different from its calculation therefore no
violation.

(b) It would be unjust to take past acts into consideration thus violative of Article 20.

(c) The data so available is not sufficient to reach a conclusive end.

(d) The assets acquired from more than ones known sources were made a crime only in 10 so
the property prior to it was irrelevant for PCA.

Passage (Q.-Q.): In common parlance, the word ‘abet‘ signifies help, co-activity and support
and incorporates within its ambit, illegitimate reason to commit the crime. So as to bring an
individual abetting the doing of a thing under any of the conditions specified under Section
of the Indian Penal Code, it isn’t just important to demonstrate that the individual who has
abetted has participated in the means of the transactions yet additionally has been associated
with those means of the transaction which are criminal. The offense of abetment by
instigation relies on the intention of the individual who abets and not upon the act which is
finished by the individual who has abetted. The abetment might be by instigation, connivance
or purposeful aid as given under Section of the Indian Penal Code. Abetment under the Penal
Code involves active complicity on the part of the abettor at a point of time prior to the actual
commission of that offence, and it is of the essence of the crime of abetment that the abettor
should substantially assist the principal culprit towards the commission of the offence.
Nowhere, concurrence in the criminal acts of another without such participation therein as
helps to give effect to the criminal act or purpose, is punishable under the Code. For an
individual to be called liable for Abetment, and so as to proceed against an individual for a
criminal offense under Section , prosecution must claim the component of mensrea.
Negligence or carelessness can’t be named to be abetment in order to punish the liable,
according to the arrangement of penal laws. So as to establish abetment, the abettor must
have appeared to “deliberately” support the commission of the wrongdoing. In such a case we
need to just prove that the wrongdoing charged couldn’t have been done without the
association as well as intervention of the supposed abettor isn’t sufficient with the
prerequisites of Section .

. Rajat and Ambika were married. However, Rajat’s mother Sashi is not fond of Ambika
because of the dowry given by her family. The conduct, language and expression of Sashi are
derogatory towards Ambika suggesting that she comes from a broke family. Ambika is
mentally broken by all this and gets depressed. One sudden day she decided to commit
suicide by hanging herself. Rajat and Sashi are prosecuted for abetting the suicide of Ambika.
Decide.

(a) Rajat is liable as he did not intervene in the cruel treatment of the mother to his wife.

(b) Rajat and Sashi are both liable. The former is liable indirectly while the latter is directly
responsible for the outcome.

(c) Only Sashi is liable as her conduct stimulated the outcome.

(d) Both of them are not guilty as no active steps were taken so as to be held liable under
abetment.

. Amit and Akhila are good friends. At one instance, where they were together Akhila told
Amit about the tussle that is going on between Akhila and her business partner Abhijeet.
Amit suggests the murder of Abhijeet. However, Akhila did not murder Abhijeet and ignored
the instigation. Decide.

(a) Amit is not guilty of abetment as the instigation was ignored.

(b) Amit is guilty of abetment even when it was not acted upon.

(c) Amit is not guilty as the Abhijeet was not murdered.

(d) Amit is not guilty as the words were devoid of mensrea.

. Based on the factual information provided in the aforementioned question, if Akhila stabbed
Abhijeet but he ended up recovering in the hospital, would that be considered abetment?

(a) No, it would not amount to abetment as the desired result was not achieved.

(b) No, it would not amount to abetment as Abhijeet has recovered.


(c) Yes, it would amount to abetment as the requisite effect is not an essential condition.

(d) Yes, it would amount to abetment and the liability would be shared by Amit and Akhila.

. Aditya instigated Bheem to instigate Palash to commit the offence of theft. Palash has
committed the theft and has been arrested for the same. Bheem is also arrested and charged
for the offence of abetment. Decide the liability of Aditya.

(a) Aditya will not be liable as he was not the instigator of Palash.

(b) Aditya will not be liable as he did not intend for the outcome.

(c) Aditya will be liable for abetting Bheem.

(d) Aditya will be liable but not under the offence of abetment.

. Based on the information provided in the passage, decide whether the words uttered in an
angry state would amount to abetment?

(a) Yes, it would amount to abetment but it has to assessed based on the result.

(b) Yes, it would amount to abetment if such words incite the other person.

(c) No, it would not amount to abetment as it is devoid of intention.

(d) No, it would not amount to abetment but negligence.

8
Passage (Q.-Q.): Under National legislative enactments of Indian Penal Code, 10, Section
330 and 348 laid down the punitive provisions for the offence of torment. But, when the same
offence of torment is committed by the police officials on duty, the provision does not apply,
thus, giving a gate-way to the authorities in uniform. Rudul Shah v. State of Bihar marked the
first instance which prompted the rise of a compensatory statute for the infringement of rights
of the prisoner under the purview of the Constitution. In the case, the liability was imposed
upon the state for the wrongful detention of the petitioner. In Nandani Sathpathy v. P.L. Dani,
the Supreme Court held that “A person while being examined under Section 1 of the CrPC is
not required to answer those questions that have a tendency to self-incriminate.” This
principle can be extended as even the police authority while interrogating an accused cannot
force an individual to reveal any information, which can be self-incriminating or pose them to
any physical and mental harm. In Nilabati Behera v. State of Orissa, the Apex Court
reiterated that the safety of the inmate must be maintained by the police authority, in custody.
The Supreme Court in D.K. Basu v. State of Bengal recognized the right of the prisoner
against custodial torture and deaths in police custody. In addition, the Supreme Court also
extends to pay the acknowledgement and recognizes the Right against cruel and unusual
punishment, Right to free legal Aid and fair trial, Right to Speedy Trial and Rights of Inmates
of protective homes. This implies that the Apex Court had already in receipt of the threat of
custodial torture while the executive always fails to adopt a path to curb it. Thus, it can be
drawn that a specific statute is required to abolish police brutality.

. Deepak, a farmer was arrested by police for alleged rape of a girl in her village. He was in
remand till the whole trial and also during the appellate stages. Till the appeal reached the
Supreme Court he had already been in judicial custody for 8 years. He was finally held to be
not guilty and acquitted by the Supreme Court. Even after the judgment of the Supreme
Court, Deepak was not released from custody for four more years. Deepak filed an SLP
before the Supreme Court for his release. The Supreme Court regretted the incarceration of
Deepak for so long, ordered his immediate release and ordered the concerned state
government to pay a compensation of 2 Lacs to Deepak. Can the Supreme Court order
compensation to be paid by the State?

(a) Yes, the payment of compensation can be ordered as Deepak was wrongfully detained for
around 13 years despite being acquitted of the crime by the Supreme Court.

(b) No, despite the detention of Deepak after the acquittal order by the Supreme Court and
that must be classified as defence of exercise of statutory duty and be overlooked.

(c) Yes, the payment of compensation can be ordered as due to the casual attitude of our
judiciary Deepak had to be detained for over 13 years despite being innocent.

(d) No, as the State cannot be made to pay any compensation unless any mistake on its part is
an outcome of gross negligence by the employees while exercising their official duties.

. Pranav married Pooja, and both shifted to Mumbai after their marriage. Both lived happily
together until one day when Pooja was found dead in their apartment. Pranav being the only
one who had access to the house was the prime accused in the case. Police arrested Pranav,
who was sent to police remand of 10 days by the magistrate. He made a formal complaint
before a magistrate that he is being tortured by the officers in the station. He stated that he
has been told to confess to the crime else he will be given more severe punishment if the
court finds him guilty in the court. He grieved that the same has inflicted mental torture on
him and therefore makes the concerned officers liable for the same. Is the complaint valid?

(a) The complaint is invalid as any action to be taken against custodial violence has to be
taken by a court exercising its power to take suo motu cognizance of a matter.

(b) The complaint is valid as the Supreme Court recognizes right against custodial violence as
a valid right and a victim can enforce it when needed.

(c) The complaint is invalid as threatening a victim with the penalizing provisions is not any
form of mental torture of violence against him.

 (d) The complaint is valid as being the prime accused of the offence; Pranav is more likely to
be compelled by the Police to confess to reduce their burden of investigating the case.

. Rajmohan, the CM of Orissa, was in highlights during his term for allegations of gross
misconduct and corruption. After his term ended, he no longer was in the office as he lost the
election because he was booked for corruption during his term. He was brought to the police
station and was interrogated. During the time of interrogation, he refused to utter a word from
his mouth. He was informed that answering questions, asked during police examination is
mandatory in law. He put forth that he is exercising his fundamental right against self-
incrimination. Police lodged another complaint against Rajmohan for not answering the
question, asked by a public officer, on the ground that right against self-incrimination is only
available to an accused in trial. Is the further FIR legal in law?

(a) FIR is legal, as Rajmohan has refused to answer the questions asked by a public officer in
official capacity, which is not only an offence but also obstructs the legal procedure of
investigation. (b) FIR is illegal, as no person can be forced to give a confession against
himself and the same has to be done with free will and consent.

(c) FIR is legal, as right against self-incrimination is a fundamental right that extends to an
accused during the trial, so that he cannot be forced to confess against himself.

(d) FIR is illegal, as a person even during examination by the police can refrain from
answering questions that would be self-incriminatory in nature.

. Rizwaan, was arrested from his house, without giving appropriate reasons to her mother for
the arrest, for aiding a politician with her illegal practices. He was kept in custody, after
producing before a magistrate. Her mother went to the police station where she was neither
allowed to meet Rizwaan and nor was she told about his well-being. After few days, a dead
body was found near a railway track, which was identified as Rizwaan’s. His mother made
representations to authorities but was given no reply. She made a petition before the Supreme
Court, where State contended that death of Rizwaan was not an outcome of any torture
commited by the police officers and may be it was the attempt by the politician to eliminate a
witness against her. The Supreme Court allowed the petition and held the State liable for the
death and imposed a compensation of 5 lacs to be paid to the mother by the State. Choose the
correct option?

(a) The State shall be liable as they should be prepared to deal with such a situation when
they have arrested someone who could be a potential witness against someone else.

(b) The State shall not be liable as death of Rizwan was not an outcome of any brutality or
cruelty he was subjected to in the police custody and State shall only be liable for torture that
has been committed by the officers only.

(c) The State shall be liable as the safety of any accused in the police custody is the
responsibility of Police Officers only, so any harm caused to Rizwan during custody makes
the State liable for the same.

(d) The State shall not be liable as Rizwan was murdered by the Politician herself because she
was under the threat of being arrested herself if Rizwan gave any statement against her.

. A merciless group of terrorists entered Mumbai, the gateway of India. They killed over a
thousand people in their attacks that lasted for as long as 4 days. Azeer, one of the terrorists,
was caught shooting openly on the crowd at one of the most crowded stations in the city. He
was finally arrested and put in police custody. He was tortured by police to make him say the
name of the mastermind of their plan and their local link in Mumbai. Azeer, did not utter a
word and was admitted to hospital due to severe injuries. The Supreme Court took the suo
motu cognizance of the case and ordered police to act civil to all the criminals, including
Azeer. Can police use torture as a measure to ensure efficiency of the trial?

(a) Police can use torture, to some extent, to reach truth in complex cases where investigation
has not reached a conclusion despite months of hardwork at the end of Police.

(b) Police cannot use torture to reach to truth as the same is a punishable in law and can be
enforced even against public servants exercising their official duty.

(c) Police can use torture in cases involving special laws such as terrorist acts wherein such
practices are permitted to establish links in the case and reach a conclusive end.

(d) Police cannot use torture as the same is impermissible as per the rulings of the Supreme
Court, and victim has right against any form of such torture.

Passage (Q.-Q.): The Bench of Justices R Banumathi, AS Bopanna, and Hrishikesh Roy held
that only a prima facie case is needed to be considered at this point, and that the law does not
require an elaborate inquiry under Section 228 of Code of Criminal Procedure. The judgment
states, for framing the charges under Section 228 of the Code, the judge is not required to
record detailed reasons. As pointed out earlier, at the stage of framing the charge, the court is
not required to hold an elaborate enquiry; only prima facie cases are to be seen. This
judgment was delivered in an appeal against a decision of the Madhya Pradesh High Court,
which had quashed the charges against accused persons on the ground that the trial court
failed to apply its judicial mind while framing the charges. The High Court had held that the
trial court ought to apply its mind while framing charges and must give concise reasons for
the charges framed. This order was challenged by the complainant-appellant before the
Supreme Court. The Supreme Court, while dealing with the issue at hand, delved into the
scope of Sections 226, 227, and 228 of the Code of Criminal Procedure. While Section 226 of
the Code provides for opening of the criminal case by the public prosecutor, Section 227
provides for discharging of the accused persons if there are insufficient grounds to proceed
against them. The judge may proceed with framing the charges against an accused under
Section 228 if, after perusal of the records and documents, there is prima facie reason to
presume that a triable offence may have been committed by the accused. The judge is not
required to record his reasons for framing of charges, the Supreme Court observed, placing
reliance on the precedents laid down in this regard.

. Mr. Sethmalani was a very cunning lawyer, who was known for his defence tactics and for
taking up difficult criminal cases. He has been undefeated in his entire career, and has no
plans of altering this. He took the case of Manu Sharma, who was alleged of assault of his
wife Tanu. The prosecution produced evidence showing grave wounds and burn marks, along
with the certified medical report and statements of Tanu, admitting that her husband was
usually cruel and brutal to her. Mr. Sethmalani asked permission for producing their
evidence, to prove that his client Manu Sharma is innocent. Magistrate, however, declined
analyzing the evidence of Mr. Sethmalani and proceeded with the framing of charges against
Mr. Manu Sharma. Mr. Sethmalani, filed an appeal on the grounds that Manu Sharma cannot
be convicted before considering the evidence presented by defence and no proper trial was
conducted. Decide:
(a) The appeal will sustain, as even if the court is not required to conduct a thorough inquiry
at the stage of charge framing but there is need to listen to both the sides before passing the
order of conviction.

(b) The appeal will not sustain, as a magistrate is not bound to record reasons at the stage of
charge framing and thus neither is accountable to superior courts nor his decision at this stage
is subject to appeal.

(c) The appeal will sustain, as rule of law commands that principles of natural justice shall be
followed, which allows accused to produce evidence in his favour so as to ensure his
acquittal.

(d) The appeal will not sustain, as to proceed with framing of charges the court just needs to
make sure that there is prima facie case against the accused that he has committed a triable
offence, and charge framing is not as same as conviction. .

. Rihan, whose father served in jail despite being acquitted due to an overburdened judiciary,
completes his law and clears M.P Judiciary. Zealous about his new role, he decides that he
will be as quick as possible in disposing of the cases so that somebody else need not suffer as
his own father did. In a murder case before him, he asked both the parties to produce all the
evidence and witnesses at the stage of charge framing so as to reduce the delay in decision
making that arose due to a full trial. He considered the evidence and held the accused to
convict of the rape case and thus sentence him with an imprisonment of 7 years. His steps are
appreciated by the common people due to instant justice and he is now an internet sensation.
What is the legal flaw in the process employed by Rihan?

(a) Rihan cannot ask the prosecution to present their side of the case at the stage of framing of
charges as there is no need to conduct an elaborate trial at this stage.

(b) Rihan cannot have convicted accused at the stage of framing of charges by having a
summary trial at this stage, and by enquiring the case elaborately to give instant justice.

(c) Rihan cannot decide the evidence produced by the prosecution to have a prima facie
impression of the accused having committed the offence as the same would create biases in
his mind.

(d) Rihan cannot let his personal emotional reasons to blind him while adjudicating cases in
court of law as the same impair the entire process of administration of justice.

. Abhik, a businessman, using his power and dominant position compelled his secretary to
indulge into a sexual relationship with him for keeping her job. Secretary, in need of the job
and money, agreed to the same reluctantly. After some time, she filed an FIR against Abhik
for her rape. Abhik was arrested but later released on bail. He hired the best lawyers in the
country for his case. At the stage of framing of charges, prosecution could not make out a
prima facie case that the accused had committed a triable offence and thus was discharged by
the court of his offence. The prosecution rose the question of how could an accused be
acquitted for an offence unless it is an outcome of a trial. The court rejected the contention
and did not proceed with the framing of charges. Choose the correct option.
(a) The question raised is invalid, as when the court is not convinced by the case, presented
by the prosecution that the accused has committed a triable offence, it can discharge the
accused from the offence.

(b) The question raised is valid, as the accused cannot be permanently released from the case
unless the charges are framed and the accused is properly tried providing both the sides to
present their evidence and witnesses as well as substantiate them.

(c) The question raised is invalid, as the magistrate holds ultimate discretion while making
decisions regarding conviction or acquittal of an accused and the same cannot be confined by
the statutory laws as in the judicial hierarchy; a judge holds the paramount pedestal.

(d) The question raised is valid, as the principles of natural justice suggest that a proper trial
be conducted to decide on an offence, especially that are a threat to the society in large.

. Motu and Patlu were two smart thieves, who always managed to escape after a theft.
Always two steps ahead of Police, Motu and Patlu plan every robbery so that they leave no
sign of their presence at the place. A new police officer, Jai, made a plan to nab them the next
time they robbed a place. The plan went successful and they were both nabbed by Police red-
handed. Prosecution presented the case with concrete evidence to approve that both the
accused have committed offences triable by the court, and so the court proceeded with the
framing of charges. Motu and Patlu, to comprehend what made them caught this time, studied
the copy of the order in which they found no reasons for framing charges against them. They
filed an appeal before a superior court against the framing of charges. Which of the following
is correct?

(a) Appeal has no merit, as the court is not required to record reasons at the stage of framing
of charges, because it only has to check that there is a prima facie case against the accused.

(b) Appeal has merit, as the court is not required to record reasons as long as neither of the
party has requested for the same, but when either of the party makes an application for
recording of the reasons the court is bound to comply with it.

(c) Appeal has no merit, as whenever an accused is not discharged at the stage of framing of
charges, he is entitled to know the reasons for initiating the trial against him.

(d) Appeal has no merit, as in the present case neither of the parties made an application to
the court for recording reasons, so nothing binds to record the reasons for proceedings with
the framing of charge.

. Balu Yadav was the chief minister of the State of Dihar. Balu began as a local leader and his
popularity took him to making him the head of the government. He was loved by people and
thus remained in power for multiple terms. Once he was alleged for being involved in a
mammoth scam that involved a whopping sum of money. He was arrested and after the
submission of the final report the magistrate called both the parties for framing of charges.
Prosecution presented its case and made out that Balu is accused of a triable case. The court
discharged Balu, holding that presented evidences are not enough to convict Balu of the
offence. Which of the following is corrected?

(a) Balu can be discharged by the court because if it does not find that there is prima facie
evidence for his conviction then it may not proceed with the framing of charges and the trial.

(b) Balu cannot be discharged because the same is the result of his popularity and influence,
and not because the prosecution failed to make out a prima facie case against him.

(c) Balu can be discharged as the magistrate has given the proper reasoning of his discharge
and thus it is neither arbitrary nor biased in any manner.

(d) Balu cannot be discharged by the court on the ground that there is not enough evidence to
convict him, as there is no need to prove conviction at this stage.

Passage (Q.-Q.): Protection of rights of the religious and ethnic minorities is the backbone of
India's secular values. With a legacy of bringing all religions under its fold, India has always
advocated the principle of equality. The Article 30 of the Indian constitution is one of the
many provisions that ensure preservation of minority rights. Article 30 is classified under Part
III of the Indian Constitution that throws light on the fundamental rights provided to the
citizens of India irrespective of their caste, religion and sex. Article 30 promises the rights of
minorities "to establish and administer educational institutions". Article 30 (1) promises to all
linguistic and religious minorities the ‘right to establish’ and the ‘right to administer’
educational institutions of their own choice. The right is provided by this clause on two types
of minorities, namely, religious and linguistic minorities. The right vested in the above
minorities is to establish and administer educational institutions of their choice. The basic
ground for a community to be nominated as a religious minority/linguistic minority is the
numerical strength of the community. The word "establish" indicates the right to bring into
existence, while the right to administer an institution means the right to effectively manage
and conduct the affairs of the institution. The administration implies management and affairs
of the institution. The management must be free of control and restrictions so that the
founders of their community can frame the institution as they think fit in correspondence with
their views and ideas of how the interest of the community in general and the institution will
be delivered. Thus it gives choice to the minority community to establish such educational
institutions as it will serve both purposes, that is, the purpose of protecting their religion,
language or culture, and also the purpose of giving through general education to their
children in their own language.Article 30 (2) restricts the State from making discrimination in
the matter of providing help to any educational institution on the ground that it is managed by
a religious minority or linguistic minority. Article 29 of the Indian Constitution uses the word
‘minorities’ in its marginal heading but it speaks about "any section of the citizens inhabiting
the territory of India or any part of the country should have the right to protect their language
or script or culture which is different and varied. It also says that citizens should be allowed
to take admission in any educational institution which is maintained by the State or getting
help from State funds whether they vary in religion, race, caste, language or any of them.

 
. Charminder decides to set up a dance institute for the dying art of Pranammiya, a dance
form originating in South India, aeons ago. Which of the following is true?

(a) Charminder, under the constitution of India has the right to establish such an institution

(b) Article 29 does not give Charminder, the right to establish such an institution

(c) Charminder must belong to a religious/linguistic minority for him to have this right to
establish an institution.

(d) Charminder’s right exists, as long as he can prove that Pranammiya is an ancient dance
form.

. The State refuses to provide aid to Charminder’s dance institute, calling it a waste of
resources. Which of the following statements is correct?

(a) The State has discriminated against Charminder for educational institutes like his cannot
be discriminated

(b) The State has not discriminated against Charminder since he does not belong to a
minority.

(c) The State has discriminated against Charminder on the ground that the institute belongs to
a nonstate actor.

(d) It cannot be determined.

. Please refer to the facts above. Assume that Charminder is a Muslim and the community has
been classified as a religious minority. Would Charminder’s school of Pranammiya, fall
within Article 30?

(a) Charminder’s school will fall under Article 30 since Muslims being a religious minority
have the right to establish institutes based on their choice

(b) Charminder’s school will not fall under Article 30 since Charminder is not a religious
group.

(c) Charminder’s school will fall under Article 30 since Muslims are a religious minority

(d) Charminder’s school will not fall under Article 30 for a school on Pranammiya does not
protect religion and culture of the Muslim community.

. Rajat is a Muslim and the community has been classified as a religious minority. Would
Rajat’s school of a north east Indian tribal dance, fall within Article 30?

(a) Rajat’s school will fall under Article 30 since Muslims being a religious minority have the
right to establish institutes based on their choice

(b) Rajat’s school will not fall under Article 30 since Rajat is not a religious group.

(c) Rajat’s school will fall under Article 30 since Muslims are a religious minority
(d) Rajat’s school will not fall under Article 30 for a school on North Eastern Tribal Dance
does not protect religion and culture of the Muslim community.

. Please refer to the facts above. Assume that instead of North Eastern Tribal Dance, Rajat
wishes to open a community run school to teach English language. Which of the following is
true?

(a) Rajat’s school will fall under Article 30 since Muslims being a religious minority have the
right to establish institutes based on their choice

(b) Rajat’s school will not fall under Article 30 since Rajat is not a religious group.

(c) Rajat’s school will fall under Article 30 since Muslims are a religious minority and the
school fulfills a purpose of general education of Muslim children

(d) Rajat’s school will not fall under Article 30 for an English language school does not fulfill
the purpose of Article 30

. Lajpat is a Hindu which is the majority community. He decides to open a school on vedic
scriptures and seeks government aid under Article 30 for the same. Decide

(a) Lajpat is not a religious/linguistic minority and thus should not be allowed such funding

(b) Lajpat’s case falls within “any section” phrase used in Article 29 of the Constitution

(c) Lajpat does not run a in a community and thus cannot be given the benefit

(d) None of the above

Passage (Q.-Q.): In India, the statutory provisions which come closest to encapsulating the
principles of force majeure are Sections 32 and 56 of the Indian Contract Act, 18. “32.
Enforcement of contracts contingent on an event happening.—Contingent contracts to do or
not to do anything if an uncertain future event happens cannot be enforced by law unless and
until that event has happened. If the event becomes impossible, such contracts become void.”
“56. Agreement to do impossible act.—An agreement to do an act impossible in itself is void.
Contract to do an act afterwards becoming impossible or unlawful.—A contract to do an act
which, after the contract is made, becomes impossible, or, by reason of some event which the
promisor could not prevent, unlawful, becomes void when the act becomes impossible or
unlawful. Section 56 encapsulates the doctrine of frustration of contract. The parties seeking
to invoke Section 56 will face consequences under Section , wherein any advantage that has
been received by any party to a contract shall have to be restored. The law in India has been
laid down in the seminal decision of Satyabrata Ghose v. Mugneeram Bangur & Co...What
was held was that the word “impossible” has not been used in the Section in the sense of
physical or literal impossibility. The performance of an act may not be literally impossible
but it may be impracticable and useless from the point of view of the object and purpose of
the parties. If an untoward event or change of circumstance totally upsets the very foundation
upon which the parties entered their agreement, it can be said that the promisor finds it
impossible to do the act which he had promised to do. The key factor that has to be borne in
mind is that there is a difference between impossibility vis-a-vis commercial difficulty.
Commercial difficulty is not excused.

. What is the major difference between a Contingent contract stipulated under Section 32 and
Doctrine of Frustration under Section 56?

(a) Contingent contracts depend on a future event but Doctrine of frustration need not be
dependent on a future event.

(b) Contingent contracts do not refer to impossible acts whereas Doctrine of Frustration is
based on acts becoming impossible.

(c) Contingent contract governs express force majeure clauses in contracts whereas Doctrine
of frustration operates in absence of force majeure clauses in the contract.

(d) None of the above.

. Amy and Jack entered into a lease agreement under which Jack leased his property to Amy
for 4 years. When Amy started moving her belongings in the house, the local authority closed
down the only access road to the leased premises. The road was closed because it passed a
derelict and dangerous building, which could endanger a passer-by’s life. The local authority
closed down the road for 12 months for revamping the building and making the road safer for
everyone. Amy therefore argued that the tenancy has been frustrated because of the change in
circumstances. Has the contract been frustrated under Indian law?

(a) Yes, because it is no longer possible for Amy to access the premises.

(b) No, because the closure of access was temporary and does not frustrate the lease for
remaining 3 years.

(c) Yes, because the very foundation of lease agreement is that the tenant has access to leased
premises.

(d) No, because the closure of the only access road was done in order to ensure of safety of
passer-by like Amy.

. Ocean Trawlers Ltd. owned several fishing trawlers. Maritime Fish Co. hired one of the
fishing trawlers from Ocean Trawlers named “Rosemary” which was to be employed in the
fishing industry. Both the parties were aware of the law which provided that such trawlers
could be only used after obtaining a license form the government. Maritime Fish Co. was
using five trawlers and therefore sought five licences from the government. But the
government only allowed 3 licenses and asked Maritime Fish Co. to name the three trawlers
for the license. The Maritime Fish Co. did not name the “Rosemary” fishing trawler for the
license. When Ocean Trawlers asked for the payment for the “Rosemary” Trawler, Maritime
Fish Co repudiated the charter of hire and claimed frustration. Would Maritime Fish Co.
succeed in claiming frustration?
(a) Yes, because the government had not given the license to use the “Rosemary” trawler due
to which the contract has become impossible to perform.

(b) No, the impossibility of performing the contract is due to Maritime Fish Co.’s own
election to exclude the “Rosemary” trawler form the licenses granted.

(c) Yes, because both the parties had knowledge that the trawler could be used only if the
license is granted by the government and the contract was dependent on the license.

(d) No, because the contract has to be performed irrespective of whether the license has been
granted or not as it is independent of the license granted from the government.

. Prakash let certain premises to Prabhu for a restaurant. The premises were near a ground
where a concert was going to be held from 31st March to 30th April and would have visitors
from all over the country and also from foreign countries. The agreement was to remain in
force for the period of the concert and then the terms had to be renewed. The amount of rent
fixed was higher as compared to the general rent in the area. Prabhu had agreed to the higher
rent because he was expecting higher profits during the time of the concert. Due to Covid-19
outbreak, the concert was cancelled. Prabhu claimed frustration of contract because of the
covid-19 outbreak. Will Prakash succeed?

(a) Yes, the contract has become impossible to perform because the concert has been
cancelled.

(b) No, because the cancellation of concert does not affect the contract between Prakash and
Prabhu.

(c) Yes, because the Prabhu would not have agreed to higher rent apart from expectation of
deriving higher profit from concert visitors and therefore is disadvantaged.

(d) No, because the situation has become commercially difficult for Prabhu which has made
the performance unprofitable and expensive but such cannot be excused.

. Ramakrishanan and Monika entered into a contract where Ramakrishnan agreed to pay Rs.
10,000 to Monika if it becomes really cold in December in Delhi. Is this a contingent
contract?

(a) Yes, because the contract is dependent on a future event.

(b) No, because the contract is not dependent on a future event.

(c) Yes, because the contract is dependent on an uncertain event.

(d) No, because the contract is not dependent on an uncertain event.

 
. The Batala Municipal Committee is in charge providing taxi stands to the taxi drivers so as
to maintain road traffic. Hakim Singh entered into a lease agreement with the Municipal
Committee. As per the agreement, the Municipal Committee leased out certain taxi stands to
Hakim Singh for a period of 1 year in consideration of Rs. 00. Hakim Singh paid the amount
at the beginning of the lease agreement period and aimed to profit from the fees levied on the
taxi drivers for using the taxi stands. During the period of the lease agreement, due to
lockdown no taxi driver used the stands and Hakim Singh failed to realize anything.
Therefore, Hakim Singh claimed the contract has become void and asked for refund of Rs.
00. Will Hakim Singh succeed?

(a) Yes, because the lease was granted under the assumption that taxi stands would be used
by the drivers and Hakim Singh would recover fees from them.

(b) No, because the lease was merely for providing the land to Hakim Singh without any
intention of it being used as a taxi stand for recovering fees,

(c) Yes, because the non-use of taxi stands by the taxi drivers was not under the control of
any of the parties to the contract.

(d) No, because the non-use of taxi stands by the taxi driver was not contemplated by the
Municipal Committee.

Passage (Q.-Q.): Section 509 of the Indian Penal Code reads “Whoever, intending to insult
the modesty of any woman, utters any word, makes any sound or gesture, or exhibits any
object, intending that such word or sound shall be heard, or that such gesture or object shall
be seen, by such woman, or intrudes upon the privacy of such woman, shall be punished. The
court noted the dictionary meaning of the word `gesture' and said: " It is not necessary that a
person uses any body signs to make a gesture. He can convey or express his intentions by
sending a doctor, writing a letter or by making a phone call etc. Modesty of Woman The
modesty of an adult female is writ large on her body. Young or old, intelligent or imbecile,
awake or sleeping, the woman possesses modesty. Modesty is a virtue which attaches to a
female owing to her sex. If the word uttered or the gesture made could be perceived as one
which is capable of shocking the sense of decency of a woman, then it is an act of insult to
the modesty of the woman. Section 354D which now reads: Stalking.—(1) Any man who—
(i) follows a woman and contacts, or attempts to contact such woman to foster personal
interaction repeatedly despite a clear indication of disinterest by such woman; or (ii) monitors
the use by a woman of the internet, email or any other form of electronic communication,
commits the offence of stalking. Mischief Rule Since there is no Section which directly deals
with an act of outraging modesty of a woman by word, gesture or act intended to insult the
modesty of a woman in Information Technology Act, it can be safely said that Section 509
can be charged against those who does the crime on virtual world as well. By applying the
mischief rule, one has to essentially ask what part of the law did the law not cover, but was
meant to be rectified by the legislature. Section 509 was enacted at a time when there was not
even a concept of internet. The mischief that was sought to be suppressed was the act of
outraging modesty of a woman. Now, such mischief has to be remedied by applying Section
509 to crimes done in virtual world as well.

. Maria lived alone in a flat in Verghese Colony. She was often ridiculed by her neighbours
and others in the colony for living alone, interacting with boys and for wearing short clothes.
Therefore, she could not have a good relationship with people in her colony. She filed
complaint against two boys who used to take bath from the tap fitted on the rear side of her
building in full nudity under Section 509 for insulting her modesty. She claimed that the tap
is clearly visible from her rear window and the boys have a bathroom in their house. Will
Maria succeed under Section 509 of IPC?

(a) Yes, because the act of bathing in full nudity which is visible to Maria amounts to
insulting modesty of a woman.

(b) No, because one has the right to choose to bath in open or in closed space.

(c) Yes, because she is often ridiculed by the people in the colony and insulted for her choices
amounting to insult to modesty of a woman.

(d) No, because it cannot be established that the two boys had intention to insult her modesty.

. The Street Offences Act was enacted to prohibit solicitation in public space. Therefore, no
one was allowed to solicit in public spaces like roads, footpaths, etc. Ragini, Razia and
Rachel were prostitutes. They stood on the balconies and near the windows of the private
premises near the road such that they were easily seen by public and soliciting from there.
They were charged under the Street Offences Act for soliciting in public space. Would they
be liable under the Street Offences Act?

(a) Yes, as per the Mischief Rule, the act of soliciting from windows and balconies comes
under the mischief of solicitation in public place, meant to be rectified by the Act.

(b) No, because the solicitation was technically done from the private premises which is not
covered under the objective of the Street Offences Act. 

(c) Yes, because the prostitutes could be seen soliciting from road which is a public place and
therefore would amount to solicitation in public space.

(d) No, because the act of soliciting from windows and balconies does not come under the
mischief of solicitation in public place as per the Act. 

. Madhurima lives in a high class society with proper security. Therefore, she does not always
lock the door of her flat during daytime. Pankaj is Madhurima’s neighbour and both of them
share good friendship. On a Sunday afternoon, Madhurima was taking a nap without being
fully dressed in her house. Pankaj wanted to borrow a book from Madhurima and therefore
knocked on her door. When she did not open, he found the door to be unlocked. He entered
the house to take the book without any hesitation because both of them were good friends.
While he was going through Madhurima’s bookshelf, he saw Mahurima lying on her bed. He
got tempted by seeing her and tried to touch her legs. Will Pankaj’s act amount to insult to
modesty of a woman under Section 509 of IPC?

(a) Yes, because he intruded upon the privacy of a woman when he entered Madhurima’s
house without permission.
(b) No, he entered Madhurima’s house without permission because of the friendship he
shared with her.

(c) Yes, because he intentionally intruded upon Madhurima’s privacy by touching her legs
when she was asleep.

(d) No, because Madhurima was sleeping and therefore incapable of possessing modesty
which could be insulted by Pankaj’s touch.

. Priyansh has invited job applications for the post of his private secretary. He received
several applications. He asked another employee to shortlist the applications as per the
requirement of the post and bring 10 applications to him. He also mentioned that all
applicants should be female commenting that “women are better at this job.” When he
received the shortlisted applications, he went through them. He shortlisted further five
applications from the ten. Then, he checked the linkedin and other social media profiles of all
the five women and noted down the relevant information. Thereby, he invited all five of them
for the interview and questioned them about their qualifications and other things including
the things they have mentioned on their social media accounts. Madhuri, one of the applicants
who was rejected and got offended by Priyansh’s questions filed complaint of stalking
against Priyansh. Will Madhuri succeed?

(a) Yes, because Priyansh has monitored her social media profile and noted down personal
information which amounts to stalking.

(b) No, because Priyansh’s action of merely accessing their social media profiles does not
amount to monitoring of their use of internet.

(c) Yes, because Priyansh had specifically asked for all female applicants and also noted
down their information on social media.

(d) No, because Madhuri had only filed the complaint as she was upset for being rejected.

. The Licensing Act makes it an offence to be ‘drunk while in charge on any highway…of
any carriage, horse, cattle, or steam engine’. The Act was passed to prevent danger to anyone
on road and highway. In this case Rose was riding her bicycle under the influence of alcohol.
Can Rose be charged under the Licensing Act?

(a) Yes, as per the mischief rule, a carriage would include a bicycle as Rose can cause danger
to herself and others while riding bicycle under influence of alcohol.

(b) No, the Act sought to prevent danger from riding heavy vehicles under influence of
alcohol as they are capable of causing danger to others on road.

(c) Yes, a bicycle is a carriage as per the dictionary meaning as it is capable of carrying
people and goods.

(d) No, because a bicycle would not cause the same danger as any other vehicle if being
driven while being drunk.
. A boy of age 21 years found a baby lying in her cradle. He made obscene gestures by
touching his penis and made lewd comments at the baby. Can he be charged for insulting the
modesty of the baby?

(a) Yes, because the gesture could be perceived as one which is capable of shocking the sense
of decency of a woman

(b) No, because an infant is not capable of possessing modesty.

(c) Yes, because the boy was an adult and capable of understanding the meaning of his
actions.

(d) No, because an infant cannot perceive such gestures as insulting.

Passage (Q.-Q.): Many see the ending of the Supreme Court’s case of criminal contempt
against Mr Bhushan as a moral victory for him, for all he had to do was pay a fine of one
rupee. Yet the Supreme Court convicted Prashant Bhushan of criminal contempt of court,
which includes scandalizing the institution and its officers, thus lowering its dignity in the
eyes of the people. This definition of criminal contempt is usually linked to words or acts that
are intended to obstruct the delivery of justice or the administration of the judicial system.
The court decides whether lowering its dignity is also affecting its functioning. There was
widespread response on mainstream and social media, where his point of view received
publicity. This special focus would be lacking in the case of a citizen not so well-known. The
court’s conviction for criminal contempt stands; it has further said that the right to freedom of
speech is not absolute — it cannot be abused in order to scandalize a public institution and its
officers who cannot defend themselves. Anyone who is not a known figure convicted of
criminal contempt would be unlikely to receive the benefit of publicly expressed
professional. Mr Bhushan’s case underlines the need for a review of the law. Why is criticism
of the court or of judges, even if baseless, a crime? Britain abolished this law after 10; the
United States of America, Canada and Australia no longer apply it. Free public expression of
opinion can only strengthen an institution, the court is not so fragile as to wither away at
criticism. Not fear, but fairness, should be the foundation of an institution’s superiority.

. Which of the following would be a strong argument opposing the author’s opinion?

(a) India being a democratic country, judiciary plays a very important role. Therefore, it is
essential to respect such an institution and its order.

(b) India’s judicial system is different from that of Britain, United States of America, Canada
and Australia and therefore the criminal contempt of court cannot be done away with. 

(c) Free criticism of the court would instill fear in the mind of courts and affects its
independence in judicial decisions.

(d) Contempt of court is a reasonable restriction of freedom of speech and expression.

  

. Raksha and Darshika had a long-standing property dispute over a piece of land. Both
claimed it to be theirs. Their case had finally reached the High Court which was supposed to
settle the matter on the next date of the hearing. Darshika realized that Raksha’s counsel has
come up with evidence and arguments that could favour the case of Raksha. Therefore,
Raksha wrote a letter to Raksha’s counsel telling him that if he does not back out of the case
and miss the next scheduled hearing before the court, his daughter’s life would be in danger.
Would Darshika’s actions amount to criminal contempt of court?

(a) Yes, because it amounts to obstruction of administration of justice.

(b) Yes, because it amounts to blackmailing.

(c) No, because it does not affect the court in any manner.

(d) No, because Darshika did not threaten the judges.

. Several lawyers of the Supreme Court created a union and went on a strike in front of the
court. The strike was conducted to highlight and improve lawyers’ plight in current times.
They argued that since independence, lawyers’ condition in India has not improved. Their
work is undervalued and the courts have a major role in worsening lawyers’ conditions in
India. These lawyers blocked the entrances of the court and did not let other lawyers, judges
and litigating parties to enter the court. Indian law allows lawful strikes. Would such an
action of the lawyers’ union amount to criminal contempt of court?

(a) Yes, since it amounts to interference in the administration of justice.

(b) Yes, since lawyers cannot strike in front of the court.

(c) No, since strikes are legal in India and lawyers have the right to strike.

(d) No, since a strike by few lawyers does not affect the overall functioning of the court.

. A senior lawyer of the Supreme Court, Mr. Bhakti Bhoshan posted a tweet about the Chief
Justice of India. The tweet consisted of a photo of the Chief Justice of India riding a Harley
Davidson bike amidst the lockdown imposed due to the coronavirus pandemic. Along with
the picture, Mr. Bhakti criticized the Chief Justice of India for not wearing a mask and
roaming around in the public during a situation of lockdown when the entire country is
struggling with the situation of lockdown. He emphasized how such an action would create a
bad example for the members of public. Would Mr. Bhakti Bhoshan’s tweet amount to
criminal contempt of court?

(a) Yes, since it lowers the dignity of the Chief Justice of India in the eyes of people.

(b) Yes, since it obstructs the delivery of justice by the court.

(c) No, since the Chief Justice was wrong in not wearing a mask and going out in public
during a lockdown.

(d) No, since it is a personal remark on the Chief Justice and does not scandalizes the court in
any manner.
 

. Ms. Medha is the editor of “India Speak” magazine. In one of her article, she criticized the
Supreme Court for its inability to tolerate criticism. She also accused three Supreme Court
judges of carrying out their personal vendetta against some people while deciding the cases.
The Supreme Court initiated suo moto case against Ms. Medha for criminal contempt of
court. Ms. Medha argued that Supreme Court is impinging upon her fundamental right of
speech and expression. Which of the following is correct according to the passage?

(a) Ms. Medha cannot be held liable for criminal contempt of court because it is a fair
criticism by a newspaper editor.

(b) Ms. Medha can be held liable for criminal contempt of court because her freedom of
speech and expression is limited in cases where it scandalizes court or its officers.

(c) Ms. Medha cannot be held liable for criminal contempt of court because she has the
fundamental right of freedom of speech and expression under Article 19(1)(a).

(d) Ms. Medha can be held liable for criminal contempt of court as her article interferes with
delivery of justice by the Supreme Court and the three judges in particular.  

Passage (Q.-Q.): A curative petition is a way to ask the court to review and revise their own
decision even after a review petition is dismissed or used. In a recent judgement regarding the
Nirbhaya case, AP Singh, the lawyer of convict Vinay Sharma, has placed the argument that
young age and the socio-economic background should be considered as mitigating factors.
But, the court has been very cautious in the use of such a petition. The court clearly stated
that such kind of petitions must be rare instead of regular and to ensure it, the court in its
guidelines to file the petition stated that to file such a petition a gross violation of a principle
of natural justicethe court needs to be proved by the contending party, unlike other petitions.
Also, to prove such violations, a senior advocate needs to certify and point out substantial
grounds for the petition to be entertained. The same would be reviewed by the three senior-
most judges of the court alongside the judges who passed the judgement, and if the majority
feels that there was a violation, the curative petition would be heard by the same bench.
Adding to the specialty, a curative petition is not governed by the provisions of the
Limitations Act but the court made it clear that itto be filed within a reasonable time.

. Shyam was part of a land dispute regarding a certain piece of ancestral property.
Disheartened by the judgement of the High Court, he decided to file an appeal before the
Supreme Court. The bench upheld the order by the High Court. What remedy does Shyam
have?

(a) Shyam should file a curative petition against the said order.

(b) Shyam has no remedy as the Apex court is the highest court of Appeal.

(c) Shyam has a remedy to file for a review petition of the said verdict.
(d) Shyam has no remedy as this is a personal matter without public interest.

. Shyam and Sundar were senior advocates who had jointly filed a case of corruption against
one of the judges of the Supreme Court. However, the judge was the part of the bench when
the hearing was held, and they decided against the petition of Shyam and Sundar. If principle
of natural justice requires removal of bias, can they file a curative petition?

(a) Yes, as there is gross violation of principles of natural justice in this case.

(b) Yes, as they are also both senior advocates with a substantial question about the decision-
making process.

(c) No, as they must first file a review petition for the same and then claim a curative one.

(d) No, as the accused judge would have to be the part of the curative petition bench and it
would defeat the purpose.

. Smriti want to file a PIL for the rights of children working in factories. But due to
insufficient proof the High Court rejected the plea. Later she filed a petition before the
Supreme Court and they held verdict in favor of Smriti. The bench consisted of 5 judges from
the Supreme Court roster. Decide.

(a) If the state wants to argue the matter further, they would have to file an appeal to a seven
judge bench.

(b) If the state wants to file for appeal they would have to review it by the same bench first.

(c) If the state would have to file for appeal, then they would have to file a curative petition
with the bench of 8 judges.

(d) If the state wants to pursue the matter further, they would have to file a review before the
high court bench first then appeal to the Supreme Court.

. Ramesh filed a case before the Supreme Court and it was rejected by a single judge bench.
He filed for a review before the three-judge bench which also rejected his plea. He filed for a
curative petition before the court. Decide the quorum.

(a) The quorum would consist of the single judge, the three judges from the review bench and
3 of the senior most judges.

(b) The quorum would be three judges from the review bench and three senior most judges.

(c) The quorum would be 3 senior most judges and 3 other judges as was the bench in review
petition.

(d) The quorum would be 3 senior most judges, the review petition bench, the single judge
bench and a senior advocate.
 

. Raghav has finished his graduation in law from a prestigious college in the country. He
wants to file a petition for the unconstitutionality of a law which curbs the freedom of speech
and expression before the Apex Court. Can he do so without a senior advocate’s substantial
arguments before a court?

(a) No, as a case before the Supreme Court requires substantial argument from a senior
advocate.

(b) Yes, if he proves gross violation of principles of natural justice.

(c) Yes, if he can prove social and economic backwardness as a mitigating consideration to
his petition.

(d) Yes, as this is merely a petition, anybody can raise such a case.

9
(Passage Q.68-Q.): Over the years, new consumer protection laws were needed owing to the
drastic change in the manner market functions. The New Act has brought in some major
changes and provides for more protection to the consumers in parimateria to the earlier 19
Act. A consumer is defined as a person who "buys any goods" and hires or avails of any
service" for consideration but does not include a person who obtains goods for resale or
goods or service for commercial purpose. It shall now include both online and offline
transactions through electronic means. The New Act introduces, for the first time, a definition
of misleading advertisement. Under the New Act "misleading advertisement " in relation to
any product or service, means an advertisement, which— (i) falsely describes such product or
service; or (ii) gives a false guarantee to, or is likely to mislead the consumers as to nature,
substance, quantity or quality of such product or service; or (iii) conveys an express or
implied representation which, if made by the manufacturer or seller or service provider
thereof would constitute an unfair trade practice; or (iv) deliberately conceals important
information". Under the New Act, Consumers can challenge contracts that are unreasonable.
"Unfair Contract" means a contract between the manufacturer/trader/service provider and the
consumer and includes, (i) demand for huge payments to fulfil the contractual obligations; (ii)
imposing penalty on consumer for breach of contract when the consumer has already suffered
a greater loss due to the breach; (iii) unilateral termination of contracts without proper
reasons; (iv) imposing inequitable conditions that affect consumers. Under the new act,
“product liability” means the responsibility of product manufacturer or product seller, of any
product or service, related to the product or service to compensate for any the harm caused to
a consumer by such defective product manufactured or sold or by deficiency in services
relating to a product. No liability will be fastened on the product seller if at the time of harm,
the product was misused, altered or modified.

68. The company, Motorola Pvt. Ltd. purchased a computer from Zenith Computers. The
computer was merely used by the employees of Motorola Pvt Ltd. to deal with its customers
and their issues with the products of Motorola Pvt. Ltd. The computer ran into constant
trouble from the date of its first use and such was not attended by Zenith computers properly.
Therefore Motorola Pvt. Ltd. filed a consumer complaint against Zenith computers. Zenith
computer argued that Motorola Pvt. Ltd. is not a consumer under the definition of
“consumer” under Consumer Protection Act because the computer was used for commercial
purposes. Will Motorola Pvt. Ltd. succeed?

(a) Yes, because the computer system was not directly used for commercial purposes but only
to facilitate the work of the company.

(b) No, because the computer was being used by the employees of the company.

(c) Yes, because the computer was used for providing customer service which is not the
actual business of Motorola Pvt. Ltd.

(d) No, because the computer system was a part of assets of the company which contributed
to its business and was bought out of business income.

. Nidea Cellular is a network service provider. While advertising the Nidea Internet Network
(NIN), Nidea cellular came up with series of advertisements which showed several characters
who use NIN services to achieve their dreams of learning engineering, algebra, cooking,
foreign languages, etc. It had shown female characters from remote locations of Haryana,
Rajasthan and Uttar Pradesh who could not step out of the house to gain higher education.
These characters had been shown to use NIN services to fulfil their dreams of higher
education. Some ads had also shown celebrities using NIN services to study while shooting
their films at various locations. Several complaints were filed against Nidea Cellular to be
misleading the public by showing NIN service as an alternative institution which teaches
different things. Will such complaints succeed?

(a) Yes, because such ads mislead public by giving false impression of the nature of the NIN
service.

(b) No, because such ads describe internet services which can be used to learn different things
as claimed by the ads.

 (c) Yes, because such ads mislead public by falsely describing the NIN service.

(d) No, because such ads do not mislead the public as it does not falsely describe the service.

. Totstar is a subscription based streaming service that allows its members to watch TV
shows, documentaries and movies without commercials on an internet connected device.
Totstar has a provision where if the customer opts for “automatic renewal”, then at the end of
a member’s subscription, it is automatically renewed if the customer does not cancel it within
the two days of the ending of subscription. One has to untick the option of “automatic
renewal” and then click on “Agree” button to opt out of automatic renewal. Zoya had
subscribed to Totstar for a month on 1st December 2019 for her vacation and in a hurry
clicked on “Agree” button without removing the tick from automatic renewal option. She had
given her account details to Totstar which they had saved for further subscriptions. Zoya
went on a trip to Jammu and Kashmir on 30th December and had difficulty in accessing the
internet throughout her trip. On 8th January 2020, after Zoya returned from her trip, she got
the notification that her subscription to Totstar has been renewed and it had deducted the
money for the same. Zoya did not intend to continue the subscription because her vacation
had ended. Zoya sued Totstar for the terms of contract being unfair. Would Zoya succeed?
(a) Yes, because the “automatic renewal” provision imposes an inequitable condition on the
consumers by burdening them to cancel it within limited time of 2 days.

(b) No, because Zoya had agreed to the terms and failed to cancel the subscription in the
required time due to her own negligence.

(c) Yes, because the “automatic renewal” option is already ticked and one has to actively
remove it, thereby imposing inequitable condition affecting customers.

(d) No, because the contact provides the option to the consumer to opt out of the “automatic
renewal” option by removing the tick and then agreeing to it.

. Yuba Power Products Inc. manufactured a gadget called “Shopsmith” which was a
combination power tool that could be used as a saw, drill and a wood lathe used to make
wood rods into cylindrical rods. Greenman was passing the shop of mechanical tools and
stopped to see the retailer demonstrating the Shopsmith gadget. He went on to read the
brochure provided by the company and then left the shop. His wife later gifted him the
Shopsmith gadget because she knew her husband was fascinated by it. He bought necessary
attachments as prescribed in the brochure to use it as a drill. He followed every information
provided in the brochure and used it to drill various object twice without any difficulty. He
plugged it in the socket without switching the switch off before plugging in. and started
drilling a small wooden piece. The gadget suddenly threw a piece of wood striking him in the
head, inflicting serious injuries. Therefore, he sued Yuba Power Products Inc. for
compensation for the injury inflicted due to some defect in the under “product liability”
provision of the new Consumer Protection Act. Will Greenman succeed?

(a) Yes, because he has read the brochure and acted as per the instructions.

(b) No, because he plugged in the gadget without switching off the switch first, implying the
misuse on his part.

(c) Yes, because he was injured due to the manufacturing defect in the gadget Shopsmith.

(d) No, because the gadget worked properly in its first two uses, implying no defect in the
gadget.

. What does the term “parimateria” mean in the article?

(a) The new act repeals the earlier 19 Act.

(b) The new Act overrides the earlier 19 Act in case of conflict between the provisions of two
laws.

(c) The new act and the 19 act have to be analysed in reference to each other as they deal with
same subject matter.

(d) The new Act is an addition to the earlier 19 act and the provisions of both laws do not
conflict.

 
. A man buys a jeep from the Jeep Company and uses it to run as a taxi. Within a month, the
jeep faced several issues. One day, while he was driving, the brake of the jeep failed causing
him leg injury. Therefore, the man sued the Jeep Company. Will the man succeed?

(a) Yes, because he can claim compensation for harm caused to him due to defect in the
goods under the provision of product liability.

(b) No, because he does not fall within the definition of consumer under the new Act.

(c) Yes, because he is a consumer under the new act as he has purchased the jeep in return of
an adequate consideration.

(d) No, because the jeep was not meant to be used as a taxi.

(Passage Q.-Q.): Relevant laws that are applicable to a contract involving an arbitration
agreement, are as follows: 1. Applicable law — Contract The law governing the performance
of the obligations under the contract may be distinct from the law that is applicable to the
arbitration clause. Applicable law or the proper law of the contract are terms which refer to a
legal system in which a contract may be executed. 2. Applicable law — Arbitration
agreement A clause referring parties to arbitration is treated as a separate agreement.
Therefore, any dispute pertaining to the validity of the contract itself, does not render the
arbitration agreement inoperative. Juridical seat or lex arbitri The law that is applicable to the
arbitration proceeding is the law of the juridical seat of the arbitration or more commonly
referred to as lex arbitri. The lex arbitri would determine the courts which can exercise
supervisory jurisdiction over the arbitration proceedings. Parties may choose as the seat of
arbitration proceeding, a legal system which has no nexus with the parties simply to obviate
the possibilities of bias or unfamiliarity. The provisions relevant for understanding concept of
seat and venue under the 19 Act are Sections 2(2) reads as “This Part shall apply where place
of arbitration is in India” and Section 20 reads as “Place of arbitration.—(1) The parties are
free to agree on the place of arbitration.” Analysis of Judgements: In Bhatia, the court
interpreted Section 2(2) in a manner because of which the distinction between “seat” and
“venue” (in this case the word “place” means venue) was obliterated. According to it, an
international commercial arbitration where an Indian party is involved, being proceeded with
in any part of the world, would confer jurisdiction on Indian courts to exercise powers under
Part 1 of the 19 Act. The court in BALCO expressly overruling Bhatia, observed that the
section has to be interpreted to mean that only when the seat/place of arbitration is in India,
will Part I apply, restoring the distinction between seat and venue. Enercon clarified the
position of law in case the parties have failed to mention the law applicable to the arbitration
agreement. The court observed that the legal system in which the arbitration is taking place or
the system which has the closest and most intimate connection with the arbitration
proceedings is relevant.

. Mr. James and the Indian Classics company entered into a contract wherein Mr. James
agreed to supply raw material for the manufacture of saris to the Indian Classics Company.
The contract had an arbitration clause where they agreed that if any dispute arises, it would
be arbitrated by an arbitrator chosen by the parties. Later, the Indian Classics Company
claimed that the contract has been entered based on a misrepresentation by Mr. James where
he misrepresented to supply a distinct quality of the raw material. The Indian Classics
Company went to the court against Mr. James. Mr. James said that the dispute should be
arbitrated as per the contract. The Indian Classics company claimed that the contract is
invalid and therefore, the court has appropriate jurisdiction. Which of the following is true?

(a) The court has the appropriate jurisdiction because the question is of the validity of the
contract.

(b) The dispute can be resolved by arbitration because the arbitration clause is a separate
agreement, unaffected by the invalidity of the contract.

(c) The whole contract has been rendered invalid and therefore, the arbitration clause is
vitiated.

(d) The dispute has to be resolved by the arbitration agreement because the applicable law on
contract is distinct from the applicable law on arbitration agreement.

. Kabab Ji is a Lebanese company. It entered into a franchise development agreement with


Khan Kabab, an Indian company as licensee. Article 3 of the Franchisee development
agreement stated that “All the clauses of this Agreement would be governed by and construed
in accordance with Indian laws.” Whereas Article 14 of the Agreement stated that “The
arbitration shall be conducted in Paris, France in English language.” In 2018, Khan Kabab
became a subsidiary of Kabir Food Company and a dispute arose under the Franchisee
Development Agreement. Kabab Ji referred the dispute to arbitration before the 16 of 36
International Code Council (ICC) in Paris which was opposed by Khan Kebab. Which of the
following is true?

(a) The dispute will be rightly adjudicated in Paris because the law governing the arbitration
agreement is decided by the seat.

(b) The dispute will be adjudicated by the Indian courts because the agreement clearly stated
that all its provisions would be governed by the Indian laws.

(c) The dispute will be adjudicated in Paris so as to avoid any bias in favour of the Khan
Kabab as it is an Indian company.

(d) The dispute will be adjudicated by Indian courts because the governing law of the
arbitration agreement in this case is Indian law.

. Rinz & Minz Co. is car manufacturing and selling company in India. It gets outer body of
the car made by a German company called Vladoisir Wagen Co. Thus, Rinz & Minz had
entered into an agreement with Vladoisir Wagen Co. to exclusively supply their outer body
which is of distinct design to Rinz & Minz Co. Article 10.1 of the Agreement stipulated that
“if a dispute arises, it will be resolved by Arbitration.” Article 10.2 stated that “The
arbitration shall be held in International Chamber of Commerce (ICC), Paris, France.”
Vladoisir Wagen Co. sold some of its outer body parts to a Chinese Company called MiMi
Qiche Co. Therefore, a dispute of breach of contract arose between Rinz & Minz and the
Vladoisit Wagen Co. Rinz & Minz initiated arbitration proceedings in ICC and
simultaneously filed an application under Part I of Indian Arbitration Act of 19 seeking
injunction against Vladoisir Wagen Co. from further selling the car bodies to any other
company. Will an Indian court have jurisdiction when the seat of the arbitration has been
chosen as ICC according to the BALCO judgement?

(a) Yes, because the seat of the arbitration is India and inly when the seat of the arbitration is
India, would the Part I of the 19 be applicable.

(b) No, because the law governing the contract is not Indian law but law of Paris.

(c) Yes, because when one of the parties to the contract is Indian, Part I of the 6 Act would be
applicable.

(d) No, because the seat of arbitration is ICC, Paris and only when the seat of the arbitration
is India, would the Part I of the 19 be applicable.

. Which of the following if held by a court would be contrary to the Bhatia judgement?

(a) In case both of the parties to the contract is Indian, Part I of the 19 Act would apply.

(b) In case none of the parties to the contract are Indian, Part I of the 19 would not apply.

(c) Expression “Place” used in Section 2(2) connotes seat and not the venue.

(d) Expression “place” used in Section 2(2) depends on the parties to the contract.

. A Sale and Purchase Agreement was signed between Mr. Allhuwalia and Ms. Parineeti
Chopra. Both the parties were Indian and the contract was drafted according to the Indian
provisions. The parties included an arbitration clause which provided that the disputes arising
between the parties have to be resolved by arbitration. When the dispute arose, the party
chose England to conduct the arbitration proceedings. Which of the following is true as per
the laws clarified by the judgements?

(a) England law will be applicable to the arbitration agreement because it is the seat of
arbitration.

(b) Indian law will be applicable to the arbitration agreement because both the parties to the
contract are Indian.

(c) The applicable law to the arbitration agreement is unclear and therefore, Indian courts will
have jurisdiction.

(d) The applicable law to the arbitration agreement is unclear and therefore England courts,
where the arbitration proceedings are conducted will have jurisdiction.

. What is the final ruling after the three judgements discussed in the article?

(a) The seat of arbitration determines the courts which would exercise jurisdiction over the
arbitration proceeding.
(b) The seat of arbitration decides the governing law of the contract including the arbitration
agreement.

(c) The venue where the arbitration is agreed to be held decides the courts which would have
supervisory jurisdiction over arbitration proceeding.

(d) The law governing the contract decides the courts which would exercise jurisdiction over
the arbitration proceeding.

(Passage Q.-Q.): According to English law, if one of the several joint promisors dies, the
rights and liabilities under the contract develops upon the surviving joint promisors. The
representatives of the deceased promisor neither obtain any rights nor assume any liability,
unless they are the representatives of the last surviving promisor. This rule sometimes causes
injustice, inasmuch as the creditor loses the security of the solvency of a promisor, Section 42
of the Contract Act therefore lays down a different rule: When two or more persons have
made a joint promise, then, unless a contrary intention appears by the contract, all such
persons, during their joint lives, and, after the death of any of them, his representatives jointly
with the survivor or survivors, and, after the death of the last survivor, the representatives of
all jointly, must fulfill the promise. According to Section 45 of Indian Contract Act,
devolution of joint right is governed by almost the same kind of principles as the devolution
of joint liabilities. When a promise is made to more than one person jointly, the right to claim
performance rests with all of them jointly. If anyone of them dies, it rests with his legal
representatives jointly, with the survivors and after the death of the last survivor, with the
representatives of all jointly. When two or more persons make a joint promise, the promise
may, in the absence of express agreement to the contrary, compel one or more of such joint
promisors to perform the whole of the promise. Each of two or more joint promisors may
compel every other joint promisor to contribute equally with himself to the performance of
the promise, unless a contrary intention appears from the contract. If any of two or more joint
promisors make default in such contribution, the remaining joint promisors must bear the loss
arising from such default in equal shares. A joint promisor who has been compelled to
perform the whole of the promise, may require the other joint promisors to make an equal
contribution to the performance of the promise. Nothing shall prevent a surety from
recovering, from his principal, payments made by the surety on behalf of the principal, or
entitle the principal to recover anything from the surety on account of payments made by the
principal. When a promise is performed on a certain day, and the promisor has undertaken to
perform it without application by the promise, the promisor may perform it at any time during
the usual hours of the business on such day and at the place at which the promise ought to be
performed.

. A, B and C borrowed Rs. 3000 from D and they jointly promise to pay the amount to him.
But on the day of making the payment A and B fled away from the country. D compelled C
to make the payment whole payment of Rs. 3000. Decide the liability of C.

(a) C can be liable to pay the amount of even A and B, i.e whole Rs. 3000.

(b) C is not liable to pay the arrears of A and B, and should only pay his share of money.

(c) C cannot be made liable to pay the whole amount since the money was borrowed on
individual capacity.
(d) C cannot be compelled to pay for A and B because it will be very unfair for C to pay the
borrowed money of other two.

. In the above question suppose C, under compulsion paid the arrears of even A and B to D.
But he now demands the equal contribution to the performance of the promise. Can A and B
be made liable to pay their equal share?

(a) A and B are not liable to pay their share of money to C.

(b) A and B are liable to pay their share of money to C.

(c) A and B cannot be compelled by C to pay the remaining amount to him since C paid in his
individual capacity.

(d) A and B are not liable to pay their share to C since there was no express agreement
between the three to pay on behalf of each other.

. X promises to deliver goods at Y’s warehouse on the 1st January. On that particular day X
brings the good to Y’s warehouse, but only after the usual hour for closing it because he was
struck in traffic. And because the warehouse was closed till the time goods reached, Y could
not receive them. Y sued X for the nonperformance of the contract. Decide the liability of X.

(a) X is not liable for non-performance since he was only a little late when the usual hour of
business closed.

(b) X is not liable since usual hour of business has subjective inferences and he not expected
to have knowledge of usual hour of every business.

(c) X is liable for non-performance of contract since he did not deliver the goods on during
the usual hour of the business.

(d) X is not liable since being struck in traffic is beyond his control and hence he should be
pardoned for being late.

. A, in consideration of Rs. 5000, lent to him by B and C, promises B and C jointly to repay
them that sum with interest on a day specified. Unfortunately, B died in an accident. When
the day of payment came, A paid C his share but did not pay the share of B to his
representatives. B’s representatives sued A for the B’s share to be paid to them. Decide
whether their claim would succeed or not?

(a) Their claim would not succeed since A did not enter into contract with them and hence
not liable to pay to them.

(b) Their claim would succeed since non-performance of promise results in the loss of
monetary income to B’s family.

(c) Their claim would succeed since the right to claim performance of the promise rests with
them after the death of B.
(d) Their claim would not succeed because when A entered into contract with B, he only had
a legal intention to form a contract with B and not with his representatives and hence A is not
liable to pay them.

(Passage Q.-Q.): From its establishment as an amendment to the original Information


Technology Act in 2008, Section A attracted controversy over its unconstitutional nature. It
says that any person who sends by any means of a computer resource any information that is
grossly offensive or has a menacing character; or any information which he knows to be
false, but to cause annoyance, inconvenience, danger, obstruction, insult shall be punishable
with imprisonment for a term which may extend to three years and with fine. In December
2012, P Rajeev, a Rajya Sabha member from Kerala, tried to pass a resolution seeking to
amend Section A. P Rajeev pointed out that cartoons and editorials allowed in traditional
media were being censored in the new media. He also said that law was barely debated before
being passed in December 2008. In November 2012, IPS officer Amitabh Thakur and his
wife social activist Nutan Thakur filed a petition in the Lucknow bench of the Allahabad
High Court claiming that the Section A violated a fundamental right guaranteed in the Article
19(1)(a) of the Constitution of India. They said that the section was vague and frequently
misused. The petitioners have said that section A IT Act information encompasses any
message or information that is offensive or that has a menacing character or causes
annoyance, inconvenience, danger, obstruction, insult, hatred, injury, criminal intimidation,
enmity or ill will. These words have not been defined in the IT Act and are vague, wide, all-
pervasive, confusing, slack, and ambiguous which has a possibility of being misused and
abused. Also, in November 2012, a Delhi-based law student, Shreya Singhal, filed a Public
Interest Litigation (PIL) in the Supreme Court of India. She argued that the Section A was
vaguely phrased, as result, it violated Article 14, 19 (1)(a) and Article 21 of the Constitution.
On 24 March 2015, the Supreme Court of India gave the verdict that Section A is
unconstitutional in entirety. The court said that Section A of IT Act 2000 is "arbitrarily,
excessively and disproportionately invades the right of free speech". But the Court turned
down a plea to strike down sections A and of the Act, which deals with the procedure and
safeguards for blocking certain websites.

. Based on my permission provided in the passage, as per the ruling of the court of law, in
which of the following cases original section A of the IT Act is not being misused?

(a) A woman was arrested for questioning why Mumbai was shut down on Facebook?
Another woman was arrested for just liking this post.

(b) A businessman in Pondicherry who was arrested for commenting on a politician in a


tweet.

(c) Arresting a cartoonist for publishing cartoons in a magazine, depicting widespread


corruption in India.  

(d) A chemistry teacher arrested for sharing a cartoon of a politician on Quora.

 
. Raju is a right-wing columnist. He was arrested in 2009 for writing and publishing articles
against the leftist government on his website - which was also blocked. However, a year later,
when section A was amended, he was released from the captivation, but his website is
blocked till date. Raju filed a petition in the court of law to lift restrictions from his website
too, but it was turned down. Based on the information provided in the passage, why did the
court not consider Raju’s argument?

(a) Because the contents of the website were considered a threat to national integrity.

(b) Because even after the amendment, certain websites can still be blocked.

(c) Because the website had some obscene images on it.

(d) Because the website still had wrongful allegations against some important politicians.

. According to the petition filed in the High Court, section A of the IT Act violated a
fundamental right guaranteed in Article 19(1)(a) of the Constitution of India. Based on the
information provided in the passage, which fundamental right it did section A of the IT Act
invade?

(a) Freedom of speech

(b) Freedom of worship

(c) Right to equality before the law

(d) Freedom to travel anywhere in the country

. Munna shared a tweet accusing a politician and his son of corruption, religious bigotry, and
being involved in frauds. On the complaint of the politician, Munna was arrested under
section A of the original IT Act, and the charges were later expended to sedition under IPC.
Later, the charges under section A were removed after the amendment of the Act, but Munna
was not released from jail. Why did the law still hold Munna captive even after original
charges no more existed?

(a) Because an amendment does not mean nullification of the arrests made earlier.

(b) Because an amendment does not mean nullification of the other charges as well.

(c) Because an amendment does not hinder the ongoing proceedings of a case.

(d) All of the above.

. Ramesh, an investigative journalist, somehow managed to break in his colleague Barkha’s


phone and found that large sums of money have been deposited into her bank account from
some offshore accounts. Ramesh took it to Twitter and Facebook, and accused Barkha of
corruption and being dishonest to her profession. In turn, Barkha had Ramesh booked under
the IT Act. A couple of months later, section A was slashed down but Ramesh wasn’t
released. Ramesh filed a plea in the court to dismiss the case against him, but it got rejected.
Why did the court uphold accusation against Ramesh?

(a) Because Ramesh was facing a defamation suit for falsely accusing Barkha.

(b) Because breaking into someone’s phone is still an offence.

(c) Because an amendment does not mean nullification of the arrests made earlier.

(d) Because an amendment does not hinder the ongoing proceedings of a case.

. In his latest movie, Anurag Kashyap portrayed the atrocities committed by the upper caste
leading to the victimization of a woman belonging to a lower caste and her pitiful living
conditions. In the movie, the woman was forced to take up the route of crime to escape the
clutches of poverty. This story enraged a lot of people, belonging to the upper castes, of
course, including many social media trolls and right-wing bloggers, who began harassing,
insulting, and intimidating Anurag to the extent where it started hurting his work and
professional image. Anurag Kashyap believed it to be a conspiracy and called his lawyer
intending to sue the initiators for orchestrating a social media war against him. According to
the information provided in the passage, what all options do Anurag’s has under the law to
have the belligerents punished?

(a) Anurag can have the belligerents booked under section A of the IT act.

(b) Anurag can sue the belligerents for thrashing his image in public.

(c) Anurag can have the bloggers booked under section A.

(d) Anurag cannot do anything.

(Passage Q.-Q.): The Trafficking of Persons (Prevention, Protection and Rehabilitation) Bill,
2018, provides for the prevention, rescue, and rehabilitation of trafficked persons. The district
and state anti-trafficking committees will undertake measures to protect and prevent
vulnerable persons from being trafficked. These measures include: (i) facilitating the
implementation of livelihood and educational programmes for vulnerable communities, (ii)
facilitating the implementation of various government programmes and schemes for
prevention of trafficking, and (iii) developing law and order framework to ensure prevention
of trafficking. The Bill specifies various penalties. All offences are cognizable (i.e. police
officers can arrest without a warrant) and non-bailable. Note that if a person is found guilty
under the Bill and also under any other law, the higher punishment will apply. Trafficking of
one person can fetch imprisonment of 7-10 years, and fine, whereas trafficking of more than
one person is punishable with imprisonment of 10 years to life, and fine. Trafficking of a
minor attracts a penalty of 10 years to life imprisonment, and fine. Trafficking of more than
one minor brings life imprisonment, and fine. If a public servant or public official is found
involved in trafficking, the penalty could be life imprisonment, and fine. Aggravated Forms
of Trafficking may attract imprisonment of 10 years to life imprisonment, and fine of at least
Rs 1,00,000. Repeat Trafficker of Aggravated Offences may get Life Imprisonment, and fine
of at least Rs 2,00,000. Buying or selling persons brings Imprisonment of 7-10 years, and a
fine of at least Rs 1,00,000. Trafficking with the aid of the media brings imprisonment of 7-
10 years, and fine of at least Rs 1,00,000. If caught managing the premises of trafficking for
the first time: Imprisonment of up to 5 years with a fine of up to Rs 1,00,000; Subsequent
conviction brings at least 7 years with a fine of up to Rs 2,00,000. However, the
owner/occupier of premises of trafficking may get up to 3 years imprisonment with fine of up
to Rs 1,00,000 for the first-time conviction; Subsequent conviction: at least 5 years with fine
of up to Rs 2,00,000. Publication or distribution of obscene material which may lead to
trafficking: Imprisonment of 5 - 10 years, and fine of Rs 50,000 – 1,00,000. The omission of
Duty by an authority: First-time conviction: fine of minimum Rs 50,000; Subsequent
conviction: up to one-year imprisonment with fine of at least Rs 1,00,000. The Bill permits
attachment of property if there is an apprehension of the commission of an offence. Upon
conviction, such properties shall be forfeited to the government. The government may sell the
properties and remit the sale proceeds to the Rehabilitation Fund.

. When Sonu reached puberty, her uncle tried to trade her into commercial prostitution when
a joint team of NGO and police raided and released Sonu. in their statements, Sonu’s uncle
pleaded about his being poor and holds the circumstances and government policies guilty,
and the buyer claims that it was his first time and since the trade has not been realised, he
shall not be held guilty. According to the information provided in the passage, who should
face the penalty under the new anti-trafficking law?

(a) Sonu’s uncle. Since he was trying to trade Sonu for his selfish gains.

(b) The buyer. Even if it’s his first time as a pimp, it’s a crime.

(c) Both the parties. Sonu was being trafficked into prostitution business.

(d) No one. The arguments of both the parties are valid.

. The Maharashtra government appointed a nodal officer in the anti-trafficking department


and instructed him to be present during the raids and rescue operations. But during one of the
operations, the officer was absent from the duty without any prior information. Nor could he
explain his absence. According to the information provided in the passage, what punishment
is the officer entitled to receive under the law?

(a) Considering that it happened for the first time, the officer should be let go with a warning.

(b) The officer should be fined with Rs 50000 for deserting his responsibilities.

(c) The officer should face imprisonment for a minimum of 6 months.

(d) The officer should be suspended.

. A group of five young girls were being escorted to Hyderabad by a man via train. The girls
appeared scared and hungry. An onlooker informed Hyderabad police of susceptible case of
human trafficking. The anti- 21 of 36 trafficking department sprang into action and arrested
the man at Hyderabad railway station. During the interrogation, the man revealed that a
police officer from Madhya Pradesh was also involved in the trafficking syndicate. He was
arrested too. According to the information provided in the passage, what penalty is the
policeman entitled to under the new law?

(a) The policeman should be fined with Rs 50000.

(b) The policeman should be removed from the force and should be tried as a civilian in the
court.

(c) The policeman should be jailed for 5 years with a fine.

(d) The policeman should face imprisonment for life and fine.

. Rocky was arrested for trafficking a minor boy. In another case, he was found guilty of
murder. The act of murder and trafficking won him imprisonment of 5 years and 3 years
consecutively. According to the information provided in the passage, what length of the
penalty in total would Rocky have to bear?

(a) 5 years of imprisonment.

(b) 3 years of imprisonment and appropriate fine compensating the rest of the penalty.

(c) 8 years of imprisonment.

(d) 5 years of imprisonment and appropriate fine compensating the rest of the penalty.

. Raju owned a building in the outskirts of the city which he let to a transport company to use
as a warehouse. The company was later found to be involved in trafficking people from
different villages. The police arrested Raju too and presented him before the court, where he
pled not guilty as he was not aware of the company's unlawful activities. According to the
information provided in the passage, does the argument of the landlord, Raju, hold validity
before the law?

(a) Yes. Raju cannot be held guilty for unawareness.

(b) No. Raju should have made sure about the tenant's background before letting his property.

(c) Yes. Raju was not himself involved in the crime.

(d) No. Aware or not, Raju has to face the penalty.

. Sara recruited and duped many girls into trafficking masquerading as a modelling agent on
social media. When an organ syndicate was busted by the police, Sara was also arrested.
According to the information provided in the passage, what penalty would Sara be entitled to
under the new anti-trafficking law?

(a) Minimum 7 years of imprisonment and a fine of Rs 000.

(b) Rigorous imprisonment for 3 years.


(c) A fine amounting to Rs 500000.

(d) Imprisonment for life.

(Passage Q.-Q.): Unlike the prosecution which needs to prove its case beyond reasonable
doubt, the accused merely needs to create reasonable doubt or prove their alternate version by
mere preponderance of probabilities. Thus, once a plausible version has been put forth in
defence at the Section 313 (power to the court to examine an accused), Code of Criminal
Procedure examination stage, then it is for the prosecution to negate such defense plea, stated
the judgment authored by Justice Surya Kant. Allegedly the appellant called the prosecutrix
to her house and tried to entice her to indulge in illicit intercourse with the rich tenant boy in
return for clothes and trips from him. The appellant pushed the prosecutrix into the room with
the tenant boy and bolted it from outside. Later, the appellant caught hold of the prosecutrix
and threatened to kill her and her brother if anyone was informed of the matter. It was after
this incident that the prosecutrix went to lodge a police complaint with her father about the
two incidents. During the trial, the appellant denied all allegations and offered an alternate
version in her statement under Section 313 of the Code of Criminal Procedure, claiming that
there was no tenant at all in her home and that the complaint was nothing but motivated
revenge at the instance of one Bhola Singh against whom she had leveled allegations of rape
a few months ago. The Supreme Court noted that, under the Code of Criminal Procedure, 19
after the prosecution closes its evidence and examines all its witnesses, the accused is given
an opportunity of explanation through Section 313(1) (b). Any alternate version of events or
interpretation proffered by the accused must be carefully analyzed and considered by the trial
Court in compliance with the mandate of Section 313(4). Such opportunity is a valuable right
of the accused to seek justice and defend oneself. Failure of the trial 22 of 36 Court to fairly
apply its mind and consider the defence renders the entire proceedings void if the decision
violates the rights of the accused.

. Prem is a famous and loved Indian actor, who is known for his action movies. A Toyota
Landcruiser, which is personally used by Prem, hit some people sleeping by the side of the
road one night. The car did not stop or took the victims to the hospital. A FIR was lodged
against Prem and the trial initiated at the court. Prem in his explanation told that he was not
present in the car during the accident and the same was driven by his driver Shankar and
produced valid documents to support his statements. The prosecution just submitted that
Prem’s alibi should not be relied on and he should be awarded strict punishment. What
should be the course of action by the court?

(a) Prem should be held guilty as he was personally driving the car because the car was
always used by him personally and so the alibi presented by him before the court is false.

(b) Prem should not be held guilty as he produced a reasonable doubt before the court which
was not properly negated by the advocate for prosecution, thus failing to prove the guilt of
Prem.

(c) Prem should be held guilty as he failed to back his defence of alibi with enough evidence
and did not conclusively prove that he was not present in the car at the time of accident.
(d) Prem should not be held guilty as prosecution failed in establishing the guilt of Prem
before the court and therefore the court could not rely on prosecution to determine the guilt in
the case.

. Rehan and Vivek started a business of exporting tea together. Soon a dispute arose between
them and they decided to split their business. Vivek accused Rehan of misappropriation of
profits and also threatened to ruin him and his family. Rehan’s son was 8 years old and was
found dead in unknown circumstances after a week of their fight. Rehan told police that all of
this is Vivek’s revenge as he said that he will ruin his family. Vivek was arrested and
produced before the court. Rehan asked justice for his innocent kid. A servant of Rehan also
said that he saw his son with Vivek before he got missing. Just after hearing this, the court
announced that Vivek is guilty as the incident happened in lieu of his words after a week.
Was the decision of the court justified?

(a) Yes, the court is right in holding him guilty as he threatened to ruin Rehan’s family and
the incident happened just after a week.

(b) Yes, the court is right in holding him guilty as he was the one with whom Rehan’s kid
was seen just before he was gone missing.

(c) No, the court is not right in holding him guilty just after hearing Rehan and his servant
because he has the right to be given an opportunity of hearing and explanation.

(d) No, the court is not right in holding him guilty as he just threatened Rehan because he
misappropriated the profits of their business and there is no proof that he actually killed
Rehan’s son.

. Billu and Manu were cousins sharing the same house for a long time. Soon they started
having disputes regarding the division of their ancestral property. Due to this feud, Billu
started throwing Manu’s stuff from the land and challenged him to claim the property if he
can. This led to a fight between the two, where Manu threatened Billu to kill him if he acts
like this again. A few days later Billu went to a police station informing that Manu fought
with him and broke his leg to warn him to refrain from claiming his own share of land in
future. In the trial, Manu just submitted before court that he was out of town for some work
when the incident is alleged to have happened. The court, on the basis of facts found Manu
guilty of the crime and awarded him punishment. Manu filed an appeal to the superior court
against the order on grounds that his defence was not properly negated by the prosecution.
Which of the following is correct?

(a) There is no need for negation of Manu’s contention as no reasonable doubt was created by
him with the defence of his alibi as it was a mere statement and not supported by proper
documents.

(b) There is no need for negation of Manu’s contention as it was only after threat that Billu
approached the police station to register a complaint against Manu.
(c) There is a need for negation of Manu’s contention as every reasonable doubt created by
the accused is required to be negated by the prosecution, which did not happen in the present
case.

(d) There is a need for negation of Manu’s contention as mere threat does not eliminate the
legal necessity of negating a reasonable doubt created by the accused in a case.

. Sneha always wanted to be a dancer but her family always objected because of their
orthodox beliefs that dancing is not a profession for rich and big family’s girls. She was very
dejected by her family's response but she otherwise chose to pursue dancing as her career.
Meanwhile her uncle was always supportive of her decision and they soon grew a strong
bond. One day, her parents got the news and were very angry with her and even threw her out
of the house. Her uncle took her into his house and seeing that she was vulnerable, he raped
her. She filed a complaint of rape against him. Which of the following is true?

(a) The burden to prove that she was raped by her uncle is on Sneha as the prosecution has to
prove her side of the case beyond reasonable doubt and not by mere preponderance of
probabilities.

(b) The burden to prove that she was not raped is on her uncle as in the case of rape offences,
the burden to prove the innocence beyond reasonable doubts lies on the accused.

(c) The burden of proof lies with the both parties as both of them have to prove their sides of
the case beyond reasonable doubts to convince the court of their innocence.

(d) The burden of proof is on Sneha but she only has to show some preponderance of
probabilities that she was raped by her uncle.

. Rana was an infamous gangster, with multiple charges and trials pending against him. Once
in a gang war he killed four people before some police constables. They arrested Rana from
the site and registered a complaint against him for murder. In the court the prosecution very
well presented the case against Rana and backed their arguments with evidence and
witnesses. Rana explained that his act was merely an act of private defence and that the
members of other gang, he fought with, were his arch nemesis and had threatened him with
life multiple times. The magistrate, well aware of the reputation of Rana, convicted him
without paying heed to his explanations. Was the magistrate right in holding Rana guilty?

(a) Yes, as he murdered four people right before the police officers who witnessed the entire
incident and corroborated with the submissions of the prosecution.

(b) No, as his explanations were not given due consideration by the magistrate.

(c) Yes, as the case against him was well presented by the prosecution and the magistrate
relying on the evidence and witness has rightly held Rana to be guilty of murders.

(d) No, as Rana opted for violence as a resort to protect himself which has been submitted by
him, a person is permitted by law to use force as measure for private defence.
(Passage Q.-Q.): The Jharkhand High Court has reiterated that non-bailable warrant of arrest
and processes and order of attachment under the Code of Criminal Procedure (CrPC) cannot
be issued in a mechanical manner, and the court has to record its satisfaction with regard to
the pre requisites before passing such an order. In terms of Section of CrPC, the court opined
that a Magistrate has jurisdiction and power to issue warrant of arrest, which can be directed
against any escaped convict, proclaimed offender, or against any person who is an accused of
a non-bailable offence and is evading arrest. A person, against whom warrant of arrest can be
issued, must fall in any of the aforesaid three categories. It observed that only being an
accused of a non-bailable offence is not a ground to issue warrant of arrest, as per the
provisions of Section of the Code. The said accused, who is wanted in a case involving a
non-bailable offence, must also be evading his arrest. The word ‘and’ used in Section (1) of
the Code is a conjunctive clause. Thus, both the conditions should simultaneously exist to
enable the Court to issue a warrant of arrest. The Court has to record his satisfaction that the
conditions laid down in the law for issuing warrant of arrest has been fulfilled and the
procedure has been complied with. The Court has to prima-facie be satisfied that the person
accused of committing a non-bailable offence is also evading his arrest. There has to be
material before the Court to reach at the aforesaid conclusion. Without recording such
subjective satisfaction to the effect that the accused is also evading his arrest, which should
be on the basis of the materials placed before the Court, warrant of arrest cannot be issued.
This satisfaction can be derived from the police paper/ case diary. Section of CrPC
prescribes the procedure for declaring a person as a proclaimed offender and prescribes that
only after the Court is satisfied that the person is absconding, or is concealing, and it is not
possible to arrest him, the Court should issue proclamation requiring the accused to appear on
a specified date on specified time not less than 30 days from the date of publication of such
proclamation.

. A nationwide protest broke after people gathered on the roads against a law that not only
propagates discrimination but also does it explicitly. The protest was led by Jasmine Qureshi,
a practicing advocate. She gave a speech on how the law manifests propaganda against the
minorities of the nation, giving fundamentalist majority a supreme pedestal. Many FIRs were
lodged against her for inciting violence, hate speech and disturbance of public order. The
police, upon a warrant by the local magistrate, arrested Jasmine and kept her in station for all
night. The next day she filed an application for unlawful arrest, on the ground that the
magistrate stated no reasons for his satisfaction to allow arrest and this is gross
noncompliance of law. Decide:

(a) The arrest was unlawful, as any magistrate issuing a warrant for arrest is required to state
reasons as to explain why he is satisfied that all the requisites in the law have been complied
with and there is a reason to allow the arrest.

(b) The arrest is lawful, as in haste, when a person is trying to escape arrest, some mere
procedural infirmity can be overlooked for the sake of preventing the accused from avoiding
the administration of justice.

(c) The arrest is lawful, as opposing a law promulgated by the state is cognizable offence
called sedition, therefore warrant is not a mandatory requirement and an infirmity in the same
is irrelevant.

(d) The arrest is unlawful, as the act of consolidating the minority by opposing the tyrannical
policies of the government flows through the very fabric of our Constitution and cannot be
punished by law.
 

. Sunil was a big fan of Bollywood movies, and always copied the dialogues and actions of
the leads. Following one such movie, he proposed to a girl before the entire college. Sunil
thought the girl's behaviour to be an initial reluctance and shyness on the part of the girl and
kept being persistent like a hero in a movie. Sunil abducted the girl to marry her when she
was on her way home from college, which is a nonbailable offence. He was arrested upon a
warrant by the magistrate, and kept in custody for several hours. The defence lawyer made an
application stating the arrest to be unlawful as the permission was given by the local
magistrate, even if it is a case of such a grave offence. The application was rejected and the
lawyer filed the appeal. The appeal states that in severe cases of cognizable offence, the same
should have been given by the Sessions judge. Should the appeal be allowed?

(a) The appeal shall be allowed, as a warrant being issued by a magistrate is gross non-
compliance of law that renders the entire exercise of arrest futile, and vitiates the trial if
initiated by the state.

(b) The appeal shall not be allowed, as magistrates have jurisdiction to issue a warrant for
arrest in their jurisdictions, irrespective of the nature or gravity of the offence committed.

(c) The appeal shall be allowed as Sunil wanted to marry the girl and meant her no harm, the
case did not reflect urgency and thus all procedural requirements should be satisfied.

(d) The appeal shall not be allowed, as in the offence was a non-bailable one and Sunil had
escaped from the arrest so the warrant in such grave cases shall be issued by the nearest
magistrate.

. Megha Hatkar was a renowned social activist leading the fight for tribal rights in India for
over two decades. Known for her fierce nature, Megha called for a nationwide strike of all the
tribals working in industry or offices. The industrialists facing loss filed a complaint against
Megha for disruption of public order and riots, both non-bailable offences. She submitted
herself in the police station. When presented before a magistrate she was released on bail, on
her assurance that she would be available whenever needed regarding the case. She was
again, later, arrested on a non-bailable arrest warrant, which also declared her a proclaimed
offender, despite her giving regular attendance in the local police station and not leaving the
city without prior approval. The warrant is challenged in the Sessions court. Is the arrest
warrant valid?

(a) Arrest warrant is unlawful, as strike is a peaceful way of demonstration and it neither
causes violence nor disturbs the public order, it was only due to the loss being borne by
industrialists that they lodged an FIR against Megha.

(b) Arrest warrant is lawful, as she could have easily evaded the arrest in the guise of her
assurance to show up whenever needed in the police station or the court.

(c) Arrest warrant is unlawful, as the reason for the same is her being a proclaimed offender.
Megha is neither absconding nor is she impossible to arrest, as she gives regular attendance
as required and has not acted beyond the confines of law.
(d) Arrest is lawful, as she is dictating an entire group of people across the nation to come on
streets disrupt the public order and to not cooperate with the government and its policies
which is a serious offence against the state, thus requiring her immediate arrest.

. Kancha, a drug mafia, controlled the entire drug network of Mandwa. Vijay, the new
Superintendent of Mandwa, started preparation of nabbing all the criminals of the island.
After getting all the criminals convicted Kancha was last on his list. He raided his properties
but could not find any evidence against him, and the people in Mandwa were too scared to be
witnesses. He received a complaint for a person being assaulted, and the complainant
identified Kancha as the principal accused. This being a non-bailable offence magistrate
listed no other reasons for Kancha’s arrest. Kancha was sent to custody by the magistrate.
Decide, whether the arrest of Kancha complied with all the prerequisites or not?

(a) Arrest of Kancha satisfies all requisites, as the offence committed is serious and non-
bailable and thus there is no requirement for any other requisites to be met.

(b) Arrest of Kancha does not satisfy all requisites, as issuing of warrant is not a magistrate’s
discretion, for every non-bailable offence a warrant shall be issued and not recording the
reasons in such a case would be gross non-compliance of legal provisions.

(c) Arrest of Kancha satisfies all requisites, as to arrest a hardened criminal like him failure to
record reasons for satisfaction that arrest is compulsorily required is just a mere irregularity
that can be complied with even after the arrest.

(d) Arrest of Kancha does not satisfy all requisites, as when a magistrate issues an arrest
warrant recording the reasons for his satisfaction that all requisites have been complied with,
is a necessary criterion that cannot be dispensed with.

. Rick and Morty were known inter-galactic thugs and the security forces of many galaxies
were trying to chase them. They decided to rob a new planet this time and landed on Biggum.
They looted a shop which was a bailable offence on the planet. They fled and were arrested
only after 5 years when they came again to rob the planet. On questioning the reason for their
arrest, they were told that they have been arrested upon an arrest warrant for looting a shop,
and evading their arrest. Only after thorough perusal of the law of Biggum they realized that
they cannot be arrested for a bailable offence and thus made an application stating the arrest
to be unlawful. Presuming all the above stated law in the comprehension applies on the planet
of Biggum. Decide:

(a) Rick and Morty could be arrested upon the warrant, as they are wanted inter-galactic
thugs committing offences all over the universe and so they could be nabbed without even a
warrant.

(b) Rick and Morty could not be arrested upon the warrant, as they committed loot, a trivial
offence, in Biggum, which does not warrant arrest and thus issuing a warrant is unlawful.

(c) Rick and Morty could be arrested upon the warrant, as when they came to loot the planet,
for the second time, their previous offence became a non-bailable one and thus they can be
arrested upon a warrant by a magistrate.
(d) Rick and Morty could not be arrested upon the warrant, as an arrest warrant can be issued
by a magistrate for committing a non-bailable offence by the accused, and cannot be issued
for bailable offences like in the present case.

10
Passage (Q.-Q.): Privacy includes at its core the preservation of personal intimacies, the
sanctity of family life, marriage, procreation, the home and sexual orientation. Privacy also
connotes a right to be left alone. Privacy safeguards individual autonomy and recognises the
ability of the individual to control vital aspects of his or her life. Personal choices governing a
way of life are intrinsic to privacy. Privacy protects heterogeneity and recognises the plurality
and diversity of our culture. While the legitimate expectation of privacy may vary from the
intimate zone to the private zone and from the private to the public arenas, it is important to
underscore that privacy is not lost or surrendered merely because the individual is in a public
place. Privacy attaches to the person since it is an essential facet of the dignity of the human
being. -KS Puttaswamy v UoI

. The government in the wake of the 2011 bomb plantings, issued a directive mandating the
set-up of CC TV cameras inside the bathrooms of public malls. The cameras were to
installed outside the toilet stalls but within the bathroom compound to keep a check on who
enters and monitor the activities of the janitors.

(a) This shall amount to an invasion of the right to privacy of the janitors.

(b) This is an invasion to the privacy of individuals who enter the restroom

(c) This is an accepted practice; as long as the cameras are installed outside the cubicles
there is no privacy concern

(d) None of the above

. The parliament proposed a Bill which sought to create a provision in the IPC that in cases
of murder suspects the police will have unrestricted access to search a person's body,
house, his personal belongings including his electronic equipment and any communication
he has had with anyone without a warrant of any sort. Strictly based on the above passage,
would such a bill be constitutional?

(a) The bill is unconstitutional

(b) The bill is constitutional as it is based on the reasonable classification of a ‘murder


suspect’

(c) The bill is unconstitutional inasmuch it talks about an ‘unrestricted’ access.

(d) None of the above.

. The parliament proposed a new law which stated that when a person is sent to prison for
the offence of being involved in terrorist activities or sedition then after the person has
completed his prison sentence, once released, that person would have a GPS chip
implanted near his ankle (permanently) which if tried to tamper with would alert the
authorities immediately. This process would take place under expert medical supervision
and safety. Decide the constitutionality of the law.

(a) This law is constitutional

(b) This law is unconstitutional and likely to be struck down

(c) This law is constitutional inasmuch it makes a classification between ordinary convicts
and those involved in terrorists’ activities.

(d) None of the above.

. Based on the understanding of the passage choose the best statement that follows:

(a) An individual’s privacy is limited to the individual’s private auspices. The moment the
individual becomes a part of a public gathering, his privacy is naturally compromised.

(b) An individual’s reproductive choices are covered under his right to privacy.

(c) Privacy pertains to the individual’s choices that conform to the societal standards. As long
as the conformity is maintained, the individual choices are protected under privacy.

(d) All of the above.

. A new order proposed after an attack on the Delhi High Court by the members of Jaesh
was to make it mandatory for any individual to reveal his name, nationality, age and gender
before entering any Court in India Decide the constitutionality of the law proposed: -

 (a) Constitutional

(b) Unconstitutional

(c) partially Constitutional

(d) none of the above

Passage (Q.-Q.): A cheque is defined as a bill of exchange drawn on a specified banker and
not expressed to be payable otherwise than on demand There are three essential parties to
a cheque: The Drawer of a cheque, the drawee of a cheque and the payee of a cheque. On
Section 138 negotiable instruments act Supreme Court judgments have time and again
highlighted various important provisions. Every year holiday under negotiable instruments
act is declared and a cheque cannot be presented on a holiday. Section 139 of Negotiable
Instruments Act, 18 talks about the liability of a person who is issuing a cheque which has
been dishonoured Such a person is presumed to be guilty until and unless he proves his
innocence. When an offense under section 138 is constituted, there is a set of admitted facts
and situations where it is presumed that the person is guilty. In Krishi Vikas Kendra v
Mukund, the amount due was paid partly by the accuse(d) This amounts to a transaction
made and the court held that the burden of proof is on the accused to prove his innocence
with respect to dishonoured cheque. Under section 139 there is a presumption that a cheque
presented is for a discharge of the liability of debt either partially or wholly. And until and
unless the respondent purposes any purpose other than the discharge of liability, it shall be
presumed under section 138. Such purpose may be through conduct or express words. The
accused person cannot merely escape by saying it was only given as security and the day
cheque was issued there was no liability towards the person. Once the presumption of
liability is rebutted, the burden of proof shifts to the complainant to prove that the same
cheque was issued for discharge of liability. In the case of Rangappa v. Sri Mohan, it was
held that if to any fact the accused has not replied in the statutory notice (notice under
section 138) proves to be a merit for the complainant side. This presumption is governed by
the rule of evidence which is dealt by in section 118(a) of chapter XIII. Section 140 talks
about the possible grounds which may not be allowed as a ground of defence for
prosecution under section 138. It is a presumption only with regard to existing debts. So, if
the amount on the cheque exceeds the amount which is due to the person, then section 138
and section 139 shall not be attracted, this was held in the case of Angu Parameswara
Textiles P Lt(d) v. Sri Rajan and co. [Source: https://legodesk.com/legopedia/section-139-ni-
act/]

. A and B enter into a contract for supply of cotton. When A delivers the cotton to B, B gave
him a cheque dated 20/2/2019. On 21/2/2019, when B went to the bank to present it, the
cheque could not be processed for the cheque was dishonoured. Which of the following is
not a correct statement to be made?

(a) B is entitled to a legally enforceable debt against A in pursuance of the contract

(b) There is a presumption of liability against B under Section 138 of the Negotiable
Instruments Act

(c) The burden of proof to prove that the cheque was issued to discharge the liability of
cotton is on B.

(d) The presumption of liability as per Kisan Vikas Kendra case comes after proving liability.

. When the cheque was dishonoured, A filed a complaint in the court of law. Which of the
following is a correct statement regarding the next step to be taken?

(a) In line with the burden of proof in such cases, A has to show that he innocent.

(b) In line with the burden of proof in such cases, B must show that A is guilty.

(c) A must break the link between the cheque and the debt in question.

(d) B must link the cheque to the debt in question.

. B issued the cheque as payment for the contract. However, at the same time, B had also to
pay off for the transportation charges which were supposed to be paid by him as per the
contract however, were paid in cash by A. B claims that he intended to pay off for the
transport and not for the contract.

(a) B is liable for payment for the transport is also part of the contract

(b) B is not liable for the contract is only for cotton and transport payment is separate
(c) B must show that he intended to only pay for the transport

(d) B may not show anything as his conduct reflects his claim

. Ashish is a dealer in alcohol. He often gets Razzak to sell him cheap alcohol which is
smuggled from Indonesia Ashish then uses the liquor in his stores making 200% profit on
every bottle sold Razzak 16 of 36 however, asks for the money in advance before he goes to
Indonesia When Ashish issued a cheque to Razzak to get his alcohol from Indonesia, the
cheque bounced Decide

(a) Ashish is not liable for the bouncing of the cheque. The cheque was given as security.

(b) Ashish is liable for the bouncing of the cheque. The cheque may have been given prior to
the delivery, but forms part of the same transaction.

(c) Ashish is not liable for the bouncing of the cheque as there is illegal supply of alcohol in
question. There did not exist a legally enforceable debt.

(d) Ashish is liable for the bouncing of the cheque as the moment he placed an order, the
terms of the order placed a legally enforceable onto him.

. Aftab is an oil trader. Vishnu purchased a few barrels from him which Aftab was supposed
to deliver at his door. Aftab was to get paid extra for delivery of the oil. Vishnu issued him a
cheque which bounced Which of the following is the best defence that Vishnu can possibly
take?

(a) Vishnu issued the cheque for discharging the liability for transport expenses

(b) The contract amount was less than the amount in the cheque.

(c) The cheque was issued as security to Aftab for his further transactions.

(d) Vishnu had asked Aftab to wait for a couple days before putting the cheque in the bank
so that he could organize enough balance.

Passage (Q.-Q.): Consideration is an essential element to make a contract. It must be


provided for a contract to be legally binding. In contract law, it is said that "consideration
must move from the promisee". Drawing out the subtlety of this statement, it seems that if
there is "a promisee", there must already be a promisor; The promisor has already made a
promise to the promisee, which is sufficient to form a contract (but it's not formed at this
point); The promisee must give something back to the promisor - a promise; When the
promisee promises to do something - gives consideration (and it doesn't have to be given to
the promisor - a legally binding contract is formed, provided the other elements have been
satisfied The first element is (i) Consideration must move at the desire of the promisor:
Consideration can be offered by the promisee or a third-party only at the request or desire of
the promisor. It is important to note that there can be a stranger to consideration but not a
stranger to the contract. Thus, consideration may move to a person who is not the other
party to the contract. When contracting parties are already contracted with one another, a
promise to do something that they have already contracted to do can't be "fresh"
consideration. It's consideration which has been provided in the past, and not at the time of
formation of the contract. It is important to note that past consideration is not considered for
a new promise since it is not been given in lieu of the promise. According to Indian law, ‘past
considerations’ is ‘good consideration’ if it was given at the desire of the promisor.
Consideration is able to be minimal. Contracts supported by small consideration are
enforceable, because Courts will look for consideration for a promise, rather than examine
the commercial merits of the contract. Courts are reluctant to interfere with contracts which
are freely made between contracting parties. If the promises to be performed at a later date
or at the time the contract is made, it is sufficient. The law looks for some value in an
economic sense - even minuscule. Further, the same should be over and above the
promisor’s existing obligations. If the promisor is already obligated either by his promise or
law to perform or abstain from a certain act, then it is not a good consideration for a promise.
Finally, a consideration cannot be unlawful. [Source: https://hallellis.co.uk/contractual-
consideration/]

. A, due to his admiration for the U.P. Chief Minister, builds a stage on a ground owned by
him so that the CM can hold his rally. After the elections, the stage is used by a drama
company for staging plays. They promise to pay him. Later, the drama company doesn't pay
him.

(a) There is no contract between A and the drama company due to invalid consideration.

(b) There is no contract between A and the drama company.

(c) There is a contract between A and the drama company not for using the stage but for
using the ground on which the stage exists.

(d) There is a contract between A and the drama company for using the stage.

. Abhishek was chased by a group of thugs near the police station. The inspector, Mr.
Navedia happened to be Abhishek’s neighbour. went to Mr. Navedia’s house later in the
evening and narrated his misery. He requested Mr. Navedia to look into the matter and said
he would help him in the investigation. Mr. Navedia promised to nab the culprits at the
earliest and make the area more secure.

(a) The promise between Abhishek and Mr. Navedia is a promise with consideration

(b) The promise between Abhishek and Mr. Navedia is a promise without consideration

(c) The promise between Abhishek and Mr. Navedia is a contract

(d) It cannot be determined from the facts given

. Pawan was asked by Rohan to buy her some goods as he returned from office. She
persuaded him by saying she'd keep hot tea and halwa ready for him when he got home.
Pawan, while buying the goods slapped the shopkeeper. The shopkeeper seeks to hold
Rohan liable. Can he do so? 

(a) No, since Pawan had not been authorised to slap the shopkeeper. 

(b) No, since there is no contract of agency. 


(c) Yes, since Pawan was authorised to buy the goods. 

(d) No, since Pawan acted without authority. 

. Please refer to the facts above. What would the answer be if Rohan had asked Pawan that
he would withhold Pawan’s share of monthly profits if he did not oblige?

(a) Yes, since there is a valid contract of agency.

(b) No, since there is no contract of agency.

(c) Yes, since Pawan was authorised to buy the goods.

(d) No, since Pawan acted without authority.

. Asur and Rohan are partners in a firm. Asur asked Rohan to enter into a contract of agency
with him. Asur said that he would give his share of monthly profits in advance if he obliged. 

(a) Yes, there is a valid contract of agency.

(b) No, there is no contract of agency.

(c) Yes, Rohan is authorised to buy goods.

(d) No, since Rohan is stupid.

. A husband promises his wife a monthly maintenance if she agrees to live separately, since
their marriage is on the rocks. He also signs a document to the affect. Is the contract valid?

(a) Yes, since the agreement is a written one.

(b) No, since there is no natural love and affection.

(c) No, since there is no consideration.

(d) No, since any agreement in restraint of marriage is void ab initio.

Passage (Q.-Q.): After the sub-prime crisis, speedy recovery of debts became the prime
concern of banks and financial institutions world over. To this end, they incorporated split or
hybrid arbitration agreements in their facility agreements. “Split” or “hybrid” arbitration
agreements allow one or both parties the right to elect either litigation (by way of civil suits)
or arbitration once the dispute has arisen. Split arbitration agreements are of two types: “sole
option,” where one party has the right of election, and “mutual option,” where both parties
have the right of election. Since the Indian courts have not specifically adjudicated on this
issue, the above arbitration agreement is susceptible to two views: VIEW 1: The arbitration
agreement is not enforceable. At the time of signing the arbitration agreement, it gave the
option to either party or a sole party to, “in its discretion”, trigger the arbitration agreement or
pursue litigation. Thus, since the words used in the arbitration agreement are “The Lender
(either party) may”, it does not reflect the absolute intention of the parties to adjudicate
disputes by way of arbitration alone, especially at the time of signing the arbitration
agreement. Thus, the principle of interpretation in good faith could be applied, i.e. the
parties' intention at the time of signing the agreement, to state that the absolute intention of
the parties was not to solely arbitrate. Arbitration agreement being uncertain, causes
hopeless confusion by giving an option to the parties to either pursue litigation or arbitration,
thus the arbitration agreement is not enforceable. View II: Such an arbitration agreement is
valid and enforceable. As per the principle of effective interpretation, where a clause can be
interpreted in two different ways, the interpretation enabling the clause to be effective should
be adopted in preference to that which prevents the clause from being effective. It could
therefore be argued that the presumption always lies in favour of the validity of the arbitration
agreement and the choice of the parties rather than against it.

. Palak wanted to buy a house with modern interior and modern furniture. Palak really liked
the house owned by Mehak situated in Neeti Bagh, Delhi. Mehak had put the house on sale.
On visiting the house, Palak did not like the interior of the house which was unlike her
expectations. The price of the house in the market was Rs. 4 crores. Palak agreed to buy
Mehak’s house situated in Neeti Bagh, Delhi for Rs. 6 crores provided Mehak renovates the
house with modern interior decoration and modern furniture. Is this contract enforceable?

(a) Yes, because the contract has been entered in good faith.

(b) No, because the contract has not been entered into good faith.

(c) Yes, because the terms of the contract are clear and certain without any ambiguity.

(d) No, because the terms of the contract are uncertain.

. Rohit and Prachi are friends from their school time. Rohit ran a business of selling different
types of materials and fabrics to various designers and people directly. Prachi planned to
start her own business of designer saris and suits. She asked for help from Rohit to arrange
for fabric for her designer clothes and Rohit agreed to sell fabrics to Prachi at cheaper rates.
Rohit and Prachi entered into an agreement where Rohit agreed to sell meters of silk fabric
to Prachi every month. A clause in the contract stated that “in case of any dispute, parties
may refer to arbitration. Is such an arbitration agreement enforceable?

(a) Yes, because the contract was performed with the intention to sell and but the fabric at
cheaper rate.

(b) No, because the parties do not have absolute intention to refer the dispute to arbitration.

(c) Yes, because the parties have discretion to decide if they want to refer the dispute to
arbitration.

(d) No, because Prachi is taking advantage of friendship with Rohit and such a contract is
not in good faith.

. Willows owns a poultry processing plant from which considerable amount of waste organs,
blood, feathers and other residues were produces as a by-product. Therefore, Willows
entered into an agreement with Marquest, according to which Marquest at its own expense
would erect a system at Willows plant, for the disposal of the by-product. Such system would
become the property of Willows in return of which Willow agreed to sell all of its by-products
to Marquest at Rs. 0 per month for five years. Clause 10 of the agreement stated that “In the
event Willows becomes insolvent within 2 years or change management or ownership in any
way, new management or owner must honour this agreement until expiration or reimburse
Marquest to the full amount.” The management and ownership were transferred to Can
poultry in a year and Can poultry send notice of termination of the agreement to Marquest.
Marquest claimed that Clause 10 is uncertain as the term “full amount” and “reimburse” are
unclear as to what amount. Thus, Marquest claimed that Can poultry have to oblige by the
agreement. Which of the following is true as per the principle of effectiveness?

(a) Clause 10 is enforceable as it can be made certain by interpreting “reimburse the full
amount” to mean reimburse the expenditures already incurred in erecting the system.

(b) Clause 10 is unenforceable because it is impossible to render it effective by any way of


interpretation.

(c) Clause 10 is enforceable because the sole purpose of the clause was to allow the new
management or owners to be empowered to terminate the contract with Marquest.

(d) Clause 10 is unenforceable because it is uncertain whether “full amount” means mere
expenditure incurred by Marquest or the by-products it was to receive for 5 years.

. Amrita agreed to sell tons of oils to Neelam for a consideration of Rs. 10,000. When Amrita
delivered the oil, Neelam found that it is mustard oil, whereas she wanted to buy Groundnut
oil. Neelam refused to pay and claimed that the agreement is uncertain and therefore
unenforceable. Which of the following is true as per the principle of effective interpretation?

(a) The contract is enforceable because it can be interpreted to mean that Neelam agreed to
but any kind of oil, to make it effective.

(b) The contract is unenforceable because the contract is uncertain as to what kind of oil.

(c) The contract is enforceable because Neelam cannot at the end moment refuse to pay.

(d) The contract is unenforceable because the parties did not agree to the same thing in
same sense.

. The basic principle of an arbitration agreement is party autonomy and flexibility. Does
hybrid or split agreement adhere to this principle?

(a) Yes, it allows the party to decide if they want to refer the dispute to arbitration or not.

(b) No, as it gives more autonomy to one of the parties to the agreement.

(c) Yes, it allows the lender to recover debt as per a procedure which would make it easier.

(d) No, as hybrid or split arbitration agreement cannot be called an agreement to refer the
dispute to arbitration.

 
Passage (Q.-Q.): Criminal liability stands on the basic rule i.e. an act is wrongful only when it
is done with a wrongful state of mind However, a corporation neither has a mind which can
possess knowledge or intention, nor does it have hands to carry out its intentions. To
address this issue, the original application of criminal liability to corporate entities were
derived from the common law tort doctrine which says masters had “vicarious” liability for the
wrongful actions of their servants. As a result, the courts were soon willing to hold
corporations criminally liable for almost all wrongs except rape, murder, bigamy, and other
crimes of malicious intent. In the United Kingdom, in case of Tesco v. Nattrass held that an
employee was not a part of the ‘directing mind’ of the corporation and, therefore, his conduct
was not attributable to the act of corporation. However, in Meridian Global case the Privy
Council ruled that a company can be held liable for the crimes of its senior personnel,
committed without the knowledge of the company. The identification theory imposes
vicarious liability of an organisation for the acts committed by agents of the organisation and
was identified in the above two judgments. Both vicarious liability and Respondent Superior
employ the ‘identification’ approach pioneered in England Respondent Superior provides
that a corporation may be held criminally liable for the acts of any of its agents [who] (1)
commits a crime, (2) within the scope of employment, (3) with the intent to benefit the
corporation. This standard is quite broad, permitting organisational liability for the act of any
agent, even the lowest level employee. The special vicarious liability doctrine adopted in
India emanates from the common law principle which enables the courts to hold the directing
minds responsible for the actions and affairs of the company. In the landmark case of Iridium
the Supreme Court observed that corporate houses can no longer claim immunity from
criminal prosecution on the ground that they are not capable of possessing mens rea.
[Source (edited): Cyril Amarchandas Mangaldas, Corporations-Legal Fiction or an Unborn
Predator, SCC Online, (July 13, 2020), available at
https://ezproxy.nujs.a(c)in:2053/blog/post/2020/07/13/corporationslegal-fiction-or-an-unborn-
predator/.

. Bigamy is a criminal offence in India Mr. and Mrs. Mukherjee got married in 2015. Mrs.
Mukherjee wanted to go on a one-month cruise that would start in January 2016. Mr.
Mukherjee clearly refused for the cruise because he did not have time. In December 2015,
Mrs Mukherjee deserted Mr. Mukherjee. Mr. Mukherjee made all possible enquiries about
his wife but could not find her. He got the information that Mrs. Mukherjee was purchasing
the ticket for the cruise which was set to sail in January 2016. On 20th January, he received
the information that the cruise has been destroyed and majority of the people on the cruise
have died He still could not find Mrs. Mukherjee and assumed that might have died with
cruise destruction. In March 2017, he remarried another woman. In December 2017, Mrs.
Mukherjee returned after being stranded in a different country for several years. She was
shocked to find Mukherjee being married with another woman and filed the case of Bigamy.
Will she succeed?

(a) Yes, because Mr. Mukherjee intentionally married another woman while being in a
marriage with Mrs. Mukherjee.

(b) No, because Mr. Mukherjee believed that Mrs. Mukherjee is dead and thereafter married
another woman.

(c) Yes, because Mr. Mukherjee was wrong in assuming that Mrs. Mukherjee would have
been on the cruise that was destroyed.

(d) No, because Mr. Mukherjee made all possible enquiries to find her but remarried on
being unable to find Mrs. Mukherjee.

 
. Prashant works as a pump attendant at Morrisson Petrol Pump. Prashant had personal
enmity with Mihir from their college times. Mihir once arrived at Morrissons Petrol Pump and
bought fuel for his car. After getting the fuel, Mihir stopped at the forecourt of the petrol
pump to buy some snacks when Prashant saw him. Prashant was angry from his last
interaction with Mihir where Mihir had physically assaulted him. Therefore, when Prashant
saw Mihir, he went and started punching and kicking Mihir, resulting into severe back injury
to Mihir. Mihir brought a claim against Prashant and Morrisson Petrol Pump for the injuries
he suffered as a result of assault by Prashant. Will he succeed against Morrisson Petrol
Pump?

(a) Yes, because Prashant wasa acting as the servant of Morrisson Petrol pump and
therefore Morrisson is vicariously liable for Prashant’s actions.

(b) No, because when Prashant assaulted Mihir, he was not acting as Morrisson’s servant
but acting out of personal enmity between Prashant and Mihir.

(c) Yes, because the assault had been committed on Morrisson’s premises and the
supervisors at Morrisson didn’t do anything to stop Prashant.

(d) No, because the assault had not been committed at any of the pumps but at the forecourt
and therefore Prashant cannot be said to be acting as Morrisson’s servant.

. Motorala Inc floated a private placement memorandum (PPM) to obtain funds/investments


to finance the Iridium project. The project was “represented as being the world’s first
commercial system designed to provide global digital hand-held telephone data and it was
intended to be a wireless communication system through a constellation of satellites in low
orbit to provide digital service to mobile phones and other subscriber equipment locally.”
Such representations were drafted by Sakshi, the director and the manager of Motorola Inc
Several financial institutions invested in the project based on the information contained in the
PPM including Gonsola Corporation. However, Gonsola found that the representations made
by Motorola Inc were false and that the project turned out to the commercially unviable
resulting in significant loss to Gonsola Corporation. Therefore, Gonsola filed a criminal
complaint for cheating under Section 420 of IPC read with conspiracy under Section 120B of
IPC. Motoral Inc claimed that the corporation is incapable of cheating as it has no mind of its
own and therefore a criminal complaint against it would not succeed Will Gonsola
Corporation succeed against Motorola Inc under Indian law?

(a) Yes, because a company is capable of having mens rea for the offence of cheating as it
has mind of its own.

(b) No, because a company is incapable of having mens rea for the offence of cheating as it
has no mind of its own.

(c) Yes, because Sakshi is the directing mind of the Motorola Inc who can be held
responsible for the actions of company.

(d) No, because Sakshi is merely an employee of the company whose actions cannot hold a
company criminally liable.

. Michael is employed as a security guard at Artolia Antique Items Shop. It is a large, and
very busy shop. Michael has been employed to walk around the shop so as to creating a
presence to reduce theft. Jack is a customer who was looking around and touching every
item. Michael grew suspicious of him and politely asked him to refrain from touching every
item. Jack took it as an insult and started an argument with Steve. Jack as then ushered out
the door by another security guard Michael, however, became very angry and followed Jack
out into the parking lot, and punched him so as to establish that nobody can cause mischief
in Artolia Antique Items Shop. Jack therefore filed a small claims lawsuit seeking payment
for medical bills, as well as for pain and suffering against the owner of Artolia Antique Items
Shop. Will the owner be liable as per the principle of Respondent Superior?

(a) Yes, because Michael committed the crime within the scope of employment so as to
benefit the Shop owner.

(b) No, because Michael did not commit any crime when he was acting under self-defence to
protect the items of the shop.

(c) Yes, because though Michael chased Jack out of the shop and punched him, was acting
for the benefit of the shopowner so as to establish that no harm can be done.

(d) No, because when Michael got angry and went out to punch Jack, he cannot be said to
act in the course of employment.

. What is the Identification theory recognised in the two cases of United Kingdom?

(a) It holds a corporation criminally liable for the acts committed by its employees.

(b) It is a vicarious liability theory which hold a corporation liable for the acts of its servants.

(c) It is a principle which identifies the directing mind of the corporation to hold the
corporation criminally liable for their actions.

(d) It is a principle to impose the criminal liability on the employees of a corporation instead
of the corporation.

Passage (Q.-Q.): A victim is defined u/s.2(wa) of the CrPC as a person who has suffered
any loss or injury caused by reason of the act or omission for which the accused person has
been charged and the expression 'victim' includes his or her guardian or legal heir. Despite
introduction of a victim in the CrPC and his/her rights to prefer an appeal u/s.3 Proviso,
these rights suffer from vital deficiency in case of Appeal against the inadequate sentence/s.
This has been demonstrated in case of Parvinder Kansal where court held that Appeal filed
by Victim seeking enhancement of sentence is not maintainable. The proviso u/s.3 of the
CrPC is restricted only to three eventualities: i. acquittal of the accused; ii. Conviction of the
accused for lesser offence; iii. for imposing inadequate compensation. The 2nd eventuality
'convicting for a lesser offence' does not give any right to victim to prefer an appeal for
enhancement of sentence. The simple reading of these words means, conviction is imposed,
but it is in respect of lesser offence. It does not include 'less quantum of punishment - either
in a form of sentence for imprisonment or fine'. There are cases, in which sentences were
not awarded properly i.e. in proportionate to crime or as per the victim's expectation, but
then, 'victim' has no statutory remedy under the law to prefer an appeal to the higher forum
against it. Criminal Law clearly differentiates between the conviction and sentence.
Conviction is 'the act or process of judicially finding someone guilty of a crime; the state of
having been proved guilty, whereas the Sentence is the (actual) punishment imposed on a
criminal wrongdoer. The difficulty is: the proviso of S.3 CrPC does not specify the word
'sentence'. Undoubtedly, u/s.3 of the CrPC an Appeal by the State Government against the
quantum of sentence on the ground of its inadequacy is maintainable, but then, only State
can challenge the quantum of sentence and not the victim. [Source (edited): Kashyap Joshi,
“Why Victim cannot seek enhancement of sentence U/s. 3 of The CrPC”, LiveLaw, available
at https://www.livelaw.in/columns/why-victim-cannot-seekenhancement-of-sentence-us3-of-
the-crpc-1242].

. Rashi was raped by Amit. An FIR was filed against him and court proceedings also began.
At the end of the court proceedings, court imposed a sentence of 5 years imprisonment and
directed him to pay Rs. 0 every month to Rashi. Rashi’s mother sought to file an appeal
under Section 3 of CrPC against the sentence given by the court. Which of the following is
most appropriate?

(a) Only state can file appeal against the sentence because a crime is an offence against
humanity and not an individual.

(b) Rashi’s mother has rights of a victim because she is the legal guardian of her daughter.

(c) Only Rashi can file appeal against the sentence because she has suffered injuries due to
the act committed by Amit.

(d) None of the above.

. Mahesh has been charged with the offence of murder under Section 302 of IPC for
committing the murder of Ramesh. The trial court conducted its proceeding and at the end
found Mahesh guilty of culpable homicide not amounting to murder under Section 304 of
IPC. Ramesh’s father filed appeal against the trial court’s order under the power granted
under Section 3 proviso of CrPC. Will he succeed?

(a) Yes, since S. 3 provides the power to file appeal against conviction for lesser offence.

(b) Yes, since S. 3 provides the power to victim to appeal for enhancing the punishment.

(c) No, since S. 3 does not provide the power to file against trial court’s conviction.

(d) No, since Ramesh’s father doesn’t have locus standi.

. Rakesh Sharma is an IPS officer. His daughter is getting married the next month. Rakesh
Sharma had kept around Rs. 10 lakhs cash in his cupboard for his daughter’s wedding.
Rakshit worked as the peon of Rakesh Sharma. He closely assisted Rakesh Sharma in his
work and had knowledge about Rs. 10 lakhs. One night, he entered Rakesh Sharma’s house
and stole Rs. 7 lakh and some jewelleries that were kept in Rakesh’s cupboard. While
leaving, Rakesh Sharma caught Rakshit and threatened him with his gun. Rakshit had no
option and therefore used a glass vase and smashed it on Rakesh Sharma’s head, leaving
him unconscious. Rakshit ran with the money and jewelleries. Raskhit was charged with the
offence of theft. The trial court on analysing the facts of the case held him guilty of robbery
and awarded him 3 years of imprisonment instead of 10 years which is prescribed by the
law. Can Rakesh Sharma file an appeal to enhance the punishment to 10 years?
(a) Yes, Rakesh can appeal under Section 3.

(b) Yes, since the law prescribes 10 years of imprisonment if held guilty for the offence of
robbery.

(c) No, since Section 3 does not envisage the right to appeal for enhancement of sentence.

(d) No, since the trial court has already convicted him for the offence of robbery instead of
theft when he had no intention to commit robbery.

  

. The proviso to Section 3 had been added with the objective that “A victim-oriented
approach to certain aspects of criminal procedure was advocated in the Law Commission of
India's 154th Report, 19, which noted that "increasingly, the attention of criminologists,
penologists and reformers of criminal justice system has been directed to victimology,
control of victimization and protection of the victims of crimes…victims are the worst
sufferers in a crime and they don’t have much role in the court proceedings. They need to be
given certain rights and compensation, so that there is no distortion of criminal justice
system.” Is the judgement of Parvinder Kansal in consonance with the objective of the
addition of the proviso as per the author?

(a) Yes, the Parvinder Kansal case categorized the right of victim to appeal against the
acquittal of accused, conviction for lessor offence and for inadequate compensation.

(b) Yes, the judgement has tried to provide the rights to the victim so as to maintain the
criminal justice system.

(c) No, the judgement fails to provide the very basic right to the victims to appeal for
enhancement of a sentence.

(d) No, the judgement goes against the statutory remedy provided by the proviso under
Section 3 of CrPC.

. Radhika was returning from a party thrown by her friend She was intoxicated but was in
conscious state of mind She was driving back to her home when she met an accident with
Rahul’s car. Rahul was driving rashly and negligently at a very fast speed Rahul suffered
injuries on head whereas Radhika suffered minor cracks. Rahul filed case against Radhika
in court alleging negligent driving. The court held that Radhika is guilty of “drunk driving” and
imposed a fine of Rs. 5000. Radhika challenged the conviction before the court. Which of the
following is true?

(a) Radhika can challenge appeal against the conviction under Section 3 of CrPC.

(b) Radhika can challenge the imposition of fine of Rs. 5000 to reduce it.

(c) Rahul can challenge the conviction under Section 3 of CrPC.

(d) Rahul can challenge the imposition of fine of Rs. 5000 to increase it.

Passage (Q.-Q.): Section 112 of the Indian Evidence Act provides: “the fact that any person
was born during the continuance of a valid marriage between his mother and any man, or
within two hundred and eighty days after its dissolution, the mother remaining unmarried
shall be conclusive proof that he is the legitimate son of that man unless it can be shown that
the parties to the marriage had no access to each other at any time when he could have
been begotten”. For refuting the presumption of the legitimacy of such a child, it would have
to be proved that during the subsistence of a valid marriage, the parents of the child did not
have access to each other. In Morris v. Davies it was held that the burden to prove ‘non-
access’, or lack of an opportunity of sexual intercourse between the parties, lies on the party
contesting so. However, of late, the exponential growth in science and technology has
questioned this provision. As per studies, comparing genetic profiles of a man and a child
can ascertain the paternity of that child with laserlike precision. This makes the DNA test
scientifically accurate and irrefutable. Although DNA tests are not explicitly mentioned as an
exception to the conclusive proof of Section 112. Indian courts have, in some instances,
allowed parties to conduct the same to ascertain paternity. However, this practice is not
uniform or regulated In Nandlal Wasudev case the Supreme Court stated, “when there is a
conflict between a conclusive proof envisaged under the law and a proof based on scientific
advancement accepted by the world community to be correct, the latter must prevail over the
former.” [Source (edited): Ansh Singh Luthra, “Indian Evidence Act and Society Dynamics: A
Call for a Review”, Bar and Bench, available at
https://www.barandbench.com/columns/indian-evidence-actand-society-dynamics-a-call-for-
a-review].

. Ritika was married to Amit for five years. Recently Amit had started drinking and beating
Ritika Therefore, Ritika divorced Amit by law. After the end of their marriage, Amit visited
Ritika several times to accept him as her husband and remarry but Ritika refused After 200
days of the dissolution of their marriage, Ritika married Soham, her childhood best friend
After a month of their marriage, Ritika gave birth to a daughter Riya Whose legitimate
daughter is Riya as per Section 112 of the Evidence Act?

(a) Riya is the legitimate daughter of Soham because Ritika is married to Soham now.

(b) Riya is the legitimate daughter of Soham because Ritika and Amit did not have access to
each other.

(c) Riya is the legitimate daughter of Amit because Ritika had only been married for a month
to Soham.

(d) Riya is the legitimate daughter of Amit because Riya was begotten within 2 days of
dissolution of Amit and Ritika’s marriage.

. Naina and Aman have been married but got divorced in January because Naina was
cheating on Aman. Naina got into relationship with Ritwik after the divorce. Naina conceived
a baby on 15th June. Naina remained unmarried after the divorce. Naina claimed that her
baby is the legitimate child of Aman. Aman refuted being the father of the baby. He proved
that he didn’t have access to Naina from 13th June till 20th June because he was in
Singapore for a business trip. Is it true that the Aman is successful in proving that the baby is
not the legitimate child of Aman?

(a) Yes, since he did not have access to Naina on 15th June, i.e. when the baby was
conceived.

(b) Yes, since Naina was in a relationship with Ritwik at the time when the baby was
conceived.
(c) No, since Aman should be able to prove that he did not have access to Naina at any time
the baby could have been begotten.

(d) No, since the relationship with Ritwik is of no importance in deciding the legitimacy of the
baby.

. Rahul and Tina have been validly married for ten years. They had been trying for a baby for
several years but could not have one. Later, Rahul got distant from Tina and started an
extra-marital affair with Anjali. He left his house to live with Anjali. He had been living with
Anjali for almost a year. He was planning to divorce Tina when he heard the news that Tina
conceived a baby. Tina claims that Rahul is the father of her baby. But Rahul wants to
divorce Tina and marry Anjali and therefore refuted the fact that he is the father of the baby.
Rahul proved that he had been living with Anjali for almost a year and did not have access to
Tina Which of the following is correct?

(a) The baby will be considered the legitimate child of Rahul because Rahul and Tina have
been validly married throughout.

(b) The baby will be considered the legitimate child of Rahul because Tina was not in any
extra marital relationship at the time of conceiving the baby.

(c) The baby will not be considered the legitimate child of Rahul because Rahul was involved
in an extra marital affair with Anjali.

(d) The baby will not be considered the legitimate child of Rahul because Rahul had been
living separately from Tina and did not have access to her at the time.  

. Veer and Zaara are validly married Zaara gave birth to a baby girl named Viza Veer alleged
that Viza is not his legitimate daughter but he could not prove that he and Zaara did not have
access to each other during the time when Viza could have been begotten. Veer got a DNA
test done. The results of DNA test said that the DNA of Viza and Veer does not match.
Which of the following is correct as per the case of Nandlal Wasudev?

(a) Viza is not the legitimate child of Veer because Veer did not have knowledge of her birth.

(b) Viza is not the legitimate child of Veer because the DNA of both of them did not match as
per the result of the DNA test.

(c) Viza is the legitimate child of Veer because he failed to prove that he did not have access
to Zaara at the time when Viza could have been begotten.

(d) Viza is the legitimate child of Veer because Veer and Zaara are in a valid marriage.

. Which of the following arguments weakens the decision of the Nandlal Wasudev case?

(a) The practice of conducting DNA test to assess paternity is not uniform and unregulated

(b) The conclusive proof as envisaged by law is based on child welfare so as to prevent child
from being bastardised and face social stigma

(c) DNA tests are .9% accurate and rarely used to assess paternity test.
(d) Scientific innovation would lead to changes in the law and allow reading of scientific
advancement under laws.

11
Passage (Q.-Q.68): The police have power under law to compel an accused to give blood
samples against his wish for the purposes of investigation. This is clear from Section 53 of
the Code of Criminal Procedure, which expressly says that ‘reasonable force’ can be used for
the medical examination of an accused by a police officer. The explanation to that provision
states that examination will include examination of blood, blood stains, semen, swabs in case
of sexual offences, sputum and sweat, hair samples and fingernail clippings etc. The
compulsory extraction of bodily substances for criminal investigation will not amount to
violation of Article 20(3) i.e. protection against self-incrimination. State of Bombay v. Kathi
Kalu Oghad held that taking thumb impressions, finger prints, and specimen handwritings of
an accused will not violate Article 20(3). The protection of Article 20(3) is available to
matters within the subjective consciousness of a person and not to his physical features, the
Court reasoned. Such physical objects did not amount to testimony and one cannot be said to
have been forced to be a witness against oneself in those cases. The giving of finger
impression or of specimen signature or of handwriting, strictly speaking, is not to be a
witness. To be a witness means imparting knowledge in respect of relevant fact, by means of
oral statements or statements in writing, by a person who has personal knowledge of the facts
to be communicated to a court or to a person holding an enquiry or investigation. The
Supreme Court in Selvi v. State drew a distinction between investigative techniques such as
narcoanalysis, brain mapping, polygraph tests etc, and compulsory taking of bodily
substances. The Court held that compelling a person to undergo narco-analysis, brain
mapping and polygraph resulted in violation of Article 20(3). However, the compulsory
drawing of bodily substances will not be hit by Article 20(3), added the Court. There is also
an endorsement of the view that the use of 'force as may be reasonably necessary is mandated
by law and hence it meets the threshold of procedure established by law.

. Shruti and her husband Hassan constantly kept fighting over various issues. One day, Shruti
was found dead in the kitchen of her house while Hassan was not in the house. Police kept
looking for him for several days and then he was found in a friend’s house after 2 months. He
claimed that he was scared after seeing his wife’s body and therefore ran away from the
house but the police were certain that Shruti was murdered by Hassan. Due to the lack of
evidence, the police requested the District Magistrate to allow brain mapping of Hassan. The
permission was granted but Hassan was very reluctant to go through the same. He was finally
compelled by the police to go through brain mapping and it was then found that he was the
one who killed Shruti and then ran away to hide from the police. Is the brain mapping test of
Hassan legal?

(a) The brain mapping test of Hassan is legal as the Police are empowered to use force on
anyone to take any information which might be useful in finding out the real culprit.

(b) The brain mapping test of Hassan is not legal as compelling a person to go through a
brain mapping test which could provide information that might be a personal knowledge of
facts to the accused is self-incriminating in nature.
(c) The brain mapping test of Hassan is legal as all the evidence related to Shruti’s murder
were against Hassan and it is permissible under law to take any information that is not a
subjective consciousness of a person.

(d) The brain mapping test of Hassan is not legal as the police cannot force a person to go
through any test that could provide them the name of the real culprit without any evidence.

. Meena was in love with a boy named Mahesh. Even after many efforts made by Meena to
impress Mahesh, he was very reluctant to talk with her. Meena was infuriated with this
rejection and therefore she kidnapped Mahesh’s sister and blackmailed him to talk to her if he
wants his sister to be safe. Mahesh filed a complaint against Meena and she was later caught
by the police. Her lawyer claimed that neither Mahesh nor the police have any evidence
against Meena to arrest her. The police then forced Meena to tell them where she kept
Mahesh’s sister. Her lawyer then filed a complaint in the Session Court contending that her
right under Article 20(3) of the Indian Constitution is violated by the police. Decide.

(a) Meena’s right protected under Article 20(3) of the Indian Constitution is not violated by
the police as she blackmailed Mahesh to talk with her if he wanted his sister safe which is
evident enough to show that she was the real culprit.

(b) Meena’s right protected under Article 20(3) of the Indian Constitution is not violated by
the police as the Police are empowered to use force on anyone to take any information which
might be useful in finding out the real culprit.

 (c) Meena’s right protected under Article 20(3) of the Indian Constitution is violated by the
police as they had no evidence against her and still they forced her to give statements that
might be selfincriminating.

(d) Meena’s right protected under Article 20(3) of the Indian Constitution is violated by the
police as she was forced to be a witness against herself by orally stating where she had kept
Mahesh’s sister which is a personal knowledge to her.

. Jignesh was a very creative person and he can copy any handwriting very efficiently. The
Chief Minister of Jal Pradesh was receiving so many letters under various names, all
threatening to kill him. The Police began their search for the writers of the letters and after a
continuous search of 3 months, they caught Jignesh. The police asked Jignesh to give the
specimen of his handwriting and to their surprise his handwriting did not match any of the
letters’ handwriting. He was then asked to give his thumb impressions and fingerprints to
which Jignesh denied by claiming that they have no evidence against him and cannot ask him
to do anything that could be self-incriminating. Can the police force him to give his thumb
impressions and fingerprints? Decide.

(a) Yes, the police can force Jignesh to give his thumb impressions and fingerprints as under
Section 53 of the Code of Criminal Procedure reasonable force can be used for the medical
examination of an accused by a police officer.
(b) Yes, the police can force Jignesh to give his thumb impressions and fingerprints as the
Police are empowered to use force on anyone to take any information which might be useful
in finding out the real culprit.

(c) No, the police cannot force Jignesh to give his thumb impressions and fingerprints as his
handwriting sample was already taken which did not match with any of the letters’
handwriting.

(d) No, the police cannot force Jignesh to give his thumb impressions and fingerprints as he
has already given his handwriting sample to prove that he is not the one who wrote letters to
the Chief Minister threatening to kill him.

. The body of a girl was found in a club in a very critical condition with blood all around her.
It was later found in the medical examination that she was raped and brutally beaten. It also
showed that apart from the blood of the girl, there is blood of three more types. The police
examined every member of the club present on that day and recorded their statements. They
were also asked to give their blood samples. Mihika denied to give her blood samples as it
would violate her right under Article 20(3) of the Indian Constitution. She also claimed that
the police cannot compel her to give any information that is personal to her. Nevertheless, she
was forced by the police to give the blood sample which was later not matched with the other
three blood specimens found near the body. Mihika filed a complaint in the Sessions Court
against the police. Choose the correct answer.

(a) The action of the Police forcing Mihika to give her blood sample was illegal because
randomly asking any person present in the club that day to give their blood sample is not
logical at all.

(b) The action of the Police forcing Mihika to give her blood sample was not illegal as taking
blood samples of an accused is not something that is subjective consciousness of a person.

(c) The action of the Police forcing Mihika to give her blood sample was not illegal as the
Police is empowered to use force on anyone to take any information which might be useful in
finding out the real culprit.

(d) The action of the Police forcing Mihika to give her blood sample was illegal because
taking blood samples of an accused is something that is subjective consciousness of a person
andnot just a physical feature.  

68. Decide which of the following are protected under Article 20(3) of the Indian
Constitution.

(a) The police officer compelled a girl aged 20 to give her hair samples in a case where a
body was found cut in small pieces and stored in the refrigerator of her house.

(b) The police officer forced a boy aged 17 to give his semen samples in a case where he was
accused of a gangrape of a girl aged 24.
(c) The police officer compelled a boy aged 20 to go through a blood test in a case where he
was unable to recall anything that he did to his friend under the influence of alcohol, with his
consent.

(d) The police officer forced a girl aged 17 to go through a polygraph test in a case where she
was unable to recall anything that she did to her friend, without her consent.

Passage (Q.-Q.): The doctrine of privity of contract in the common law of contract provides
that a contract cannot confer rights or impose obligations arising under it on any person or
agent except the parties to the contract. The premise is that only parties to contracts should be
able to sue to enforce their rights or claim damages in case of breach. The term “parties” may
seem simple enough but there are situations where it may become doubtful as to exactly who
the parties are and resultantly, who, in the eyes of the law should be liable or should be
compensated in event of inevitable breaches that may occur from time to time. Doctrine of
Privity” is one of the most controversial doctrines under law of contracts, including that in the
country of India. The debates are not just due to the lack of clarity in the statutes or dissenting
judicial pronouncements but much of these owe to the academic and judicial debates linked
with the ground roots of this doctrine. The debates and discussions on the Doctrine of Privity
are relevant not only in daily life commercial contracts but also in the less frequent and
comprehensive transactional contracts. It can be seen that practices such as imposing
obligations on other party’s affiliates, relatives and agents with respect to terms like
restrictive covenants, non-compete and confidentiality obligations are quite common for the
parties under a contract these days. Interest of such third parties secured by the contracting
parties through which they have been benefited or burdened by the contract. No doubt there
are volumes of cases in the books and journals in which such related third parties who are not
parties to a contract have been allowed to sue upon it and their interest is secured against any
breach by the counter party. But those cases are based on the view that such related third
parties are claiming through a party to the contract, that it is in the position of a “cestuique
trust” or trust or admission or of a principal suing through an agent, that under the old
procedure he/it could have filed a suit in equity, even if he/it could not have sued at common
law.

. Anita and Vijay were going to get married. Vijay’s father and his prospective father-in-law,
mutually agreed to pay sums of money to Vijay on the event of marriage. Vijay got married
but the father-in-law died before making the payment of his portion of money. Vijay wanted
to sue the representatives for claiming the amount. Decide.

(a) The claim will succeed as the obligation has not been performed.

(b) The claim will not succeed as Vijay is a stranger to the agreement.

(c) The claim will not succeed as the representatives cannot be made liable in the existing
situation.

(d) The claim will succeed as the agreement was for the benefit of Vijay.
 

. The wife of Y left him because of his cruel tendencies towards her. Later, Y executes an
agreement with his father stating to treat the wife properly. On failure of fulfillment of the
obligation, he would be entitled to pay monthly maintenance along with a dwelling to her.
However, she was mistreated again and decided to file a suit for the same. Decide.

(a) The suit is maintainable as she has a substantial interest in the agreement.

(b) The suit is not maintainable as she is not the party to the agreement.

(c) The suit is maintainable as it is purported out of trust.

(d) The suit is not maintainable as it would defy the privity of arrangement.

. Yashwardhan was appointed as the successor by his father and was provided with
possession of his entire estate. In consideration of the same, Yashwardhan agreed to pay a
certain sum of money and give a village to Johan, the illegitimate son of his father. This was
to be undertaken on his attaining of majority. After few years Johan came to know about it
and decided to claim as per the agreement. Decide.

(a) The claim is not justified as he was not a part of the agreement.

(b) The claim is justified as it was made for his benefit.

(c) The claim is justified as there existed an element of trust.

(d) The claim is not justified as he did not provide any consideration.

. Narayan makes a contract with Devi to provide her with a monthly allowance of Rs. 10,000
during her lifetime and after that to her son Vinod. The transaction has been admitted in the
presence of Vinod. However, after the death of Devi, Narayan fails to pay the amount. Vinod
decides to file a suit to claim the predetermined amount. Decide.

(a) Vinod will not succeed due to him not being a party to the transaction

(b) Vinod will succeed as acknowledgement was made in his presence.

(c) Vinod will not succeed as he did not provide for consideration.

(d) Both (a) and (c).

. X acts an agent on behalf of Z against a third-party Y in the sale of tulip farm. Y fails to
make the payment for aforementioned sale. Aggrieved by the same, Z decides to file a suit
against Y to claim the amount. Decide.

(a) The suit is maintainable as X acts on behalf of Z.


(b) The suit is not maintainable as the arrangement was between X and Y.

(c) The suit is maintainable on account of trust and equity.

(d) The suit is not maintainable as only X has the right to sue Y.

Passage (Q.-Q.):The Convention (Convention on the Elimination of all forms of


Discrimination Against Women) relates not only to ILO action on sexual harassment and
gender-based discrimination at work, but even to forced labour and trafficking, child labour,
migrant workers, pregnant workers and workers with family responsibilities, domestic
workers as well as indigenous and tribal women. The new Convention on harassment is thus
based on the two traditional tracks followed by ILO.The twotrack approach to violence and
harassment is evident in the definitions adopted. Under Article 1, ‘violence and harassment’
refers to ‘a range of unacceptable behaviours and practices…that aim at, result in, or are
likely to result in physical, psychological, sexual or economic harm, and include genderbased
violence and harassment’. Gender-based violence and harassment is defined as violence and
harassment ‘directed at persons because of their sex and gender, or affecting persons of a
particular sex or gender, and includes harassment’. The ‘harm’ as defined by the new
Convention is thus explicated by tangible effects (physical, psychological, sexual or
economic) of an occupational health hazard nature, rather than violation of human rights
(dignity and self-determination) per se (however, the definition also reflects the CEDAW
definition of discrimination). The scope of the Convention covers ‘workers and other persons
in the world of work…irrespective of their contractual status’ (Article 2). The Convention
applies, for example, to persons in training, and to volunteers. The new Convention applies to
‘violence and harassment in the world of work occurring in the course of, linked with or
arising out of work’, and even applies to work in the informal economy. The scope of the
Convention covers violence and harassment ‘occurring in the course of, linked with or arising
out of work’, for example when commuting to work, or using work-related communication
technologies (Article 3). The Labour Office commentary reminds us, however, that where the
employer has no or little control of a situation, the Convention should not place an undue
burden upon it. The core principles (Article 4) include promoting and realizing the right to
work free from harassment and violence. The parties are to adopt a set of measures to that
end: legal prohibitions, policies, a  comprehensive prevention strategy, monitoring, remedies,
sanctions, awareness raising and investigation by competent bodies such as labour
inspectorates. Implementation shall involve employers, workers, and governments in their
complementary roles. Each Member shall adopt ‘an inclusive, integrated and gender-
responsive approach’ (Article 4 (2). Legislative means are required (Article 7). The
requirements concerning non-standard forms of employment (such as crowd work and gig
economy) are not very clear. Members are to recognize the important role of authorities in
case of informal economy workers (Article 8 (a)).

. Which of the following situations is not covered under the purview of the sexual harassment
provision as mentioned in the convention?

(a) a female domestic help subject to sexual abuse by her employer.

(b) a newlywed wife being sexually harassed by her husband.


(c) a female intern in the journalism department of a media house being subject of sexual
harassment by the strangers while covering a news story.

(d) All of the above.

. Which of the following situations fall out of the jurisdiction of the convention completely?

(a) a woman facing harsh and violent backlash in the firm she’s been employed in for
wearing prohibited dress code.

(b) an auto rickshaw driver harassing a female legal intern on her way to home from office.

(c) a female director not being paid equally by the producer as against to male co director.

(d) none of the above.

. Which of the following measures shall not be a welcomed move in accordance to Article 4
of the convention?

(a) HR department of a law firm is set to constitute an all-female member board to deal with
the issues of sexual harassment arising out of their workplaces.

(b) Central government is set to devise a policy mandating all the state governments to
increase the salary of the female employees during the maternity leave period so as to ensure
no drop-outs.

(c) Both a & b

(d) None of the above

. According to the ILO’s convention which of the following does not constitute as ‘harm’?

(a) a female candidate upon joining the army faces discriminatory behaviour with respect of
food items she gets as compared to the ones served to the males.

(b) State government mandating a particular territory to be evacuated by a tribal community.


Many female members of this community profess handicrafts business on this territory.

(c) A trans women facing discrimination while filling out the recruitment application form in
a firm which does not allow for a column of her gender identity.

(d) None of the above.

. The commentary of the labour office shall be sought for, in which of the following
situations?
(a) not placing the burden upon the employer for the acts of sexual harassment carried out by
the watchmen of the head office of the firm.

(b) not placing the burden upon the employer for the acts of sexual harassment carried out by
the HR of the firm.

(c) Both a & b

(d) None of the above.  

Passage (Q.-Q.): The Supreme Court’s ruling excluding the “creamy layer” from reservation
in promotions for the Scheduled Castes and the Scheduled Tribes has predictably sparked a
debate. AlokPrasanna Kumar argues that the court erred by holding that the Scheduled Castes
and the Scheduled Tribes can avail of reservation in promotions only if they are backward,
and challenges its presumption that backwardness and untouchability will disappear if a
generation or two of Dalits and Adivasis get access to education and jobs. This presumption
is also rebutted by PS Krishnan, who points to instances of Dalits having been treated as
untouchable even after occupying high office. Christodas Gandhi argues that the creamy
layer rule may reduce reservation in promotions, although the exclusion criteria he fears may
be applied are radically different from those currently used for the Other Backward Classes.
Across the aisle, Devika Agarwal supports the filtering out of the creamy layer from all
castebased reservation. Rajeev Dhavan, one of the senior advocates involved with the matter,
reiterates in print his argument in the court that the creamy layer principle is a necessary facet
of equality and, thus, a part of the Constitution’s basic structure. There’s one premise of the
creamy layer principle, though, that has not been properly anaylsed. The court set it out thus:
“The whole object of reservation is to see that backward classes of citizens move forward so
that they may march hand in hand with other citizens of India on an equal basis. This will not
be possible if only the creamy layer within that class bag all the coveted jobs in the public
sector and perpetuate themselves, leaving the rest of the class as backward as they always
were.” The observation is predicated on a 19 judgement where the Supreme Court held: “In
the light of experience, here and elsewhere, the danger of ‘reservation’, it seems to me, is
three-fold. Its benefits, by and large, are snatched away by the top creamy layer of the
‘backward’ caste or class, thus keeping the weakest among the weak always weak and
leaving the fortunate layers to consume the whole cake.” This line of reasoning, that the
poorer among Dalits and Adivasis are losing out on reservation benefits because the creamy
layer corners all opportunities, is not backed by evidence. Consider the central civil services:
senior officials from Dalit and Adivasi communities are too few to even fill all reserved
positions. In fact, posts set aside for the Scheduled Castes and the Scheduled Tribes often go
unfilled and the government has to undertake special drives to fill backlog vacancies.The
“weakest among the weak” are not being deprived by the creamy layer, but by the same
obstacles that made reservation necessary in the first place. To exclude, through the creamy
layer test, the people who have overcome such hindrances to the extent they are able to avail
of reservation goes against the very purpose of reservation. [Extracted from ‘Why the creamy
layer test for SC/ST promotions may be more harmful than using it for recruitment’.
Published by Scroll.in]

 
. The passage starts with the notion that the Supreme Court’s ruling excluding the “creamy
layer” from reservation in promotions for the Scheduled Castes and the Scheduled Tribes has
predictably sparked a debate. What are the two arguments? I. The court erred by holding that
the Scheduled Castes and the Scheduled Tribes can avail of reservation in promotions only if
they are backward. II. It is based on the presumption that backwardness and untouchability
will disappear if a generation or two of Dalits and Adivasis get access to education and jobs.
III. There are instances of Dalits having been treated as untouchable even after occupying
high office.

(a) Both II and III

(b) Both I and II

(c) Only II

(d) All I, II and III

. As per the judgement of the court, excerpted in the above passage, what is the ultimate
objective of introducing reservation in the country? I. That the backward classes of citizens
move forward so that they may march hand in hand with other citizens of India on an equal
basis. II. That the creamy layer within any class bag all the coveted jobs in the public sector
and perpetuate themselves, leaving the rest of the class as backward as they always were

(a) Only I

(b) Only II

(c) Both I and II

(d) Neither I nor II  

. As per the judgement of the court, excerpted in the above passage, what is the danger of the
ongoing practice of reservation to backward classes in the country? I. The benefits of
reservation policy are snatched away by the top creamy layer of the ‘backward’ caste or class.
II. The weakest from amongst a class remains always weak and leaving the fortunate layers to
consume the whole cake.

(a) Only I

(b) Only II

(c) Both I and II

(d) Neither I nor II

. Which of the following reasoning does the author of the above passage not use to rebut the
line of reasoning, that the poorer among Dalits and Adivasis are losing out on reservation
benefits because the creamy layer corners all opportunities?
(a) The author mentions that the claim is not supported by evidence.

(b) He argues that the reserved sears central civil services for Dalit and Adivasi communities
are not even filled completely, let alone competition. In fact, posts set aside

(c) Posts set aside for the Scheduled Castes and the Scheduled Tribes often go unfilled and
the government has to undertake special drives to fill backlog vacancies.

(d) The “weakest among the weak” are not deprived by the creamy layer, but this problem
dates back to before the country has a reservation policy

. As per the above passage, how does the exclusion through the creamy layer test goes against
the very purpose of reservation?

I. By excluding the creamy layer, the people who have overcome such hindrances to the
extent they are able to avail of reservation are disallowed benefits.

II. Reservation was meant to uplift the backward community, but what constitutes the
backward community cannot be decided by the creamy layer test.

III. Creation of classes within classes was not the intended purpose of offering reservation.

(a) Both I and II (b) Both II and III (c) Only I (d) Only III

Passage (Q.-Q.): Whether the obligation on the part of the Judicial Magistrate to record the
statement of the victim of any of the offences specified in sub-section (5A) of Section 1
Cr.PC., is limited only to those cases where the victim is mentally or physically disabled? A
rape victim can be considered as an example. Clause (a) of sub-section (5A) of Section 1 of
the Code of Criminal Procedure (CrPC) enjoins that the Judicial Magistrate shall record the
statement of the person against whom any of the offences specified therein has been
committed. This includes a rape victim also. Such victims may or may not include persons
who are mentally or physically disabled. The Supreme Court of India in State of Karnataka
by Nonavinakere Police v. Shivanna had issued directions to all police stations in the country
in the case of rape victims. Upon receipt of information relating to the commission of offence
of rape, the Investigating Officer shall make immediate steps to take the victim to any
Metropolitan / preferably Judicial Magistrate for the purpose of recording her statement under
Section 1 CrPC; a copy of the statement should be handed over to the Investigating Officer
immediately with a specific direction that the contents of such statement should not be
disclosed to any person till chargesheet/report under Section 1 CrPC is filed; if there is any
delay exceeding 24 hours in taking the victim to the Magistrate, the Investigating Officer
should record the reasons for the same in the case diary and hand over a copy of the same to
the Magistrate; Section 1 A CrPC imposes an obligation on the part of Investigating Officer
to get the victim of the rape immediately medically examined. Thus, regardless of the
question as to whether the rape victim is under a disability or not, recording of her statement
by a Judicial Magistrate is a must and such Magistrates cannot refuse to record the statement
of a rape victim or a victim of the other offences specified in sub-section (5A) of Section 1
CrPC, for the reason that she is not under any disability.  
. Neena, a blind girl, went to the market with her sister Seema. On their way back to home,
they boarded a city bus which had only a few passengers. After a few stops, they realized that
the driver did not stop at any bus stop. Seema asked the driver to stop the bus but he paid no
attention and the other male passengers in the bus grabbed her and then raped both of them.
They then threw them out of the bus. After a few days when Seema regained consciousness,
she went to file a complaint. The police officer immediately took her to the Magistrate but
she refused to record Seema’s statement. The Magistrate recorded Neena's statement and
asked Seema to be present at the court to give her statement on oath. Seema filed a petition in
district court requesting the judge to take action as she was denied the protection provided
under section 1 of CrPC. What should the judge decide?

(a) The judge should take action as the Magistrate refused to record her statement and denied
her the protection provided under section 1 of CrPC.

(b) The judge should not take any action as the Magistrate’s refusal to record the statement
under section 1 of CrPC is correct because Seema was not under any disability.

(c) The judge should take action as the Magistrate should have recorded the statement of both
Neena and Seema or should have refused to record either statement because both were the
victims of rape.

(d) The judge should not take any action as it is the discretion of the Magistrate to record the
statements on the merits of the incident.

. Priya was working in a call centre and due to her late-night shifts, she was always late in
returning to home. Once when she was on her way back to home, two drunk boys started
following her. After that, the same thing happened for 3 more days and when she shouted at
them, one of the boys grabbed her and then they both raped her and left her there. She went to
the police station and the police officer immediately recorded her statement. She was then
produced before the Judicial Magistrate within the next 24 hours. The Magistrate told her that
her statement is already recorded by the police under section 1 of CrPC, and therefore she is
not required under the Code of Criminal Procedure to record the statement again. Choose the
correct option.

(a) The Magistrate is correct as Priya’s statement was already recorded by the police officer
under section 1 of CrPC and the same is not necessary to be recorded again.

(b) The Magistrate is not correct as it mandatory that the statement of the rape victim must be
recorded by the Judicial Magistrate under section 1 of CrPC.

(c) The Magistrate is correct as Priya was not under any mental or physical disability and
therefore recording of her statement is not mandatory under section 1 of CrPC.

(d) The Magistrate is not correct as Priya’s statement recorded by the police officer holds no
ground under CrPC and the same is required to be recorded by any Magistrate.

. Mani, a 12-year-old girl, went to a picnic with her school friends. An old man lured her to
visit his house for chocolates and ice creams. She went with the man and he then raped her
every day for a month. After choking her with a belt and thinking that she was dead, he threw
her in front of the park where she met him. She was admitted in the hospital by the park
owner and after a week she regained consciousness. An FIR was already registered by the
park owner and the medical examination reports clearly suggested that she was raped
continuously for a long time. After regaining consciousness, she was to be produced before
the Magistrate but while talking with the police officer she again lost her control after
remembering everything. She was then produced before the magistrate after two days.
Decide.

(a) The police officer should have taken Mani directly to the magistrate after she regained
consciousness as her medical examination clearly suggested that she was raped continuously
for a long time.

(b) The police officer should have recorded Mani’s statement himself as she was not in a
good condition and then he should have provided the copy of the statement to the Magistrate.

(c) The police officer should have recorded in the case diary the reasons for delay beyond 24
hours in producing Mani before the Magistrate and should also have provided the copy of the
same to the Magistrate.

(d) The police officer did no wrong in taking Mani to the Magistrate after 2 days when her
condition was completely stable as her medical examination was already conducted for the
evidence.  

. Kirti had to travel long distances for her work. One day while she was out for her work in
Bhopal, some boys tried to molest her. She went to the police station to report the incident.
The investigating officer registered the FIR but did not take any action. After a month, when
she was travelling from Indore to Jhansi, she was raped in the train by three boys. She went to
the same police station to again file a complaint of the rape against the boys. The police
officer immediately recorded her statement and took her to the nearest Judicial Magistrate for
the purpose of recording her statement under Section 1 of CrPC. Kirti later alleged that the
police officer had violated the directions of the Supreme Court given in State of Karnataka v.
Shivanna as he took no action on the FIR filed by her earlier. Decide.

(a) The police officer has violated the directions of the Supreme Court as he should have
taken action when the FIR regarding the molestation has been filed by Kirti.

(b) The police officer has violated the directions of the Supreme Court as he has no right to
record Kirti’s statement under section 1 of CrPC.

(c) The police officer has not violated the directions of the Supreme Court as producing Kirti
before the magistrate in case of her molestation is not mandatory and the incident of her rape
was not related to her molestation.

(d) The police officer has not violated the directions of the Supreme Court as he immediately
produced Kirti before the nearest Judicial Magistrate after she filed a complaint informing
about her rape in the train.

. With the above context, the Judicial Magistrate after recording Kirti’s statement under
Section 1 of CrPC handed a copy of the statement to the investigating officer. The Magistrate
directed him to keep the contents of the statement confidential until the closure report or
chargesheet is filed. After the Investigation was completed, a chargesheet was filed by the
officer under Section 1 of CrPC against the boys. Raju’s mother, one of the three alleged
rapist's mothers, approached the police officer to know about the allegations made by Kirti in
her statement against Raju. The police officer informed her about the same. Kirti’s lawyer
contended in the court that the officer has disclosed the content of Kirti’s statement against
the direction of the Magistrate. Choose the correct answer.

(a) The police officer has not disclosed the content of the statement against the direction of
the Magistrate as the chargesheet under section 1 of CrPC has already been filed by him
against the three boys.

(b) The police officer has disclosed the content of the statement against the direction of the
Magistrate as he informed Raju’s mother about the allegations made by Kirti in her statement
against Raju.

(c) The police officer has disclosed the content of the statement against the direction of the
Magistrate as he was asked by the Magistrate to keep the information or content of the
statement confidential.

(d) The police officer has not disclosed the content of the statement against the direction of
the Magistrate as informing an accused or his family about the allegations or charges against
him does not amount to disclosing the information.

Passage (Q.-Q.): The ordinance punishes any 'act of violence' against 'healthcare service
personnel', ie any person who has epidemic-related responsibilities, and thereby comes in
contact with affected patients – including doctors, nurses, etc at private and public hospitals.
The definition of an act of violence includes not just direct harm, injury, obstruction or
intimidation of the healthcare worker, but also covers harassment impacting their living or
working conditions, and damage to property (equipment, ambulances, etc) or documents
connected to a healthcare worker. Investigations into such cases are to be completed within
30 days of the registration of an FIR regarding the incident, and trials are to be fast-tracked to
be completed within 1 year. The punishment for anyone convicted of committing an act of
violence against healthcare service personnel or damaging any property relating to them such
as their equipment is three months to five years of imprisonment, and a fine of Rs 50,000 to
Rs 2 lakh. Cases, where the violence was not severe, can be compounded, ie settled, with the
consent of the victim. If the person committing an act of violence also causes grievous hurt to
the healthcare worker, things are different. Firstly, the punishment is more severe: a
minimum of six months’ imprisonment and aRs 1 lakh fine, and a maximum of seven years’
imprisonment and a Rs 5 lakh fine. Secondly, in cases involving grievous hurt, the ordinance
also reverses the burden of proof and creates a presumption of guilt. If a  24 of 40 person is
prosecuted for having committed such serious violence, it will be presumed they committed
the offence and had the intention of doing so, unless the accused person can prove otherwise.
Generally, in criminal law, the burden of proof is on the prosecution to prove beyond
reasonable doubt that a person has committed a criminal offence. By reversing this burden of
proof, the ordinance will require those tried in grievous hurt cases to prove, beyond
reasonable doubt, that they did not commit the offence, or did not have the required mental
state to do it. In all cases – whether those involving grievous hurt or otherwise – the person
convicted of committing an act of violence will also have to pay compensation for the
violence against a healthcare worker and/or damage to property. In property cases, the
compensation will be equal to twice the fair market value of the property or loss caused.
. Three doctor officials are sent to a slum area where they have to test a number of people for
COVID-19 symptoms. The area is an underprivileged and uneducated area. A rumour runs
amongst the slum people that the doctors have come to take their children away from them.
This causes chaos in the area. The doctors somehow escape from the area On coming back,
one of the doctor’s files an FIR against a person allegedly abusing him in the public and
threating to cause him grievous hurt. However, there is no proof with him to establish his
allegations. His co-doctors were not present at the moment and the slum people are not ready
to testify. Decide.

(a) The court will not accept the allegations made by the doctor because they are evidence
less and cannot be proven in the court of law.

(b) The court will accept the allegations as there is no need for the doctor to show evidence as
the burden of proof is on the defence.

(c) The court will not accept the allegations as this is a trivial case which should not be tried
and court’s time must be saved in these desperate times.

(d) The court will accept the allegations as doctors are sacrificing their everything for the
nation and these types of incidents should not be entertained.

. Tabrez is a resident of Bareilly district of Uttar Pradesh. He opens his grocery shop daily
even amid this pandemic. He is really respected by his colony residents as without him, it
would have been highly difficult to get daily products. He is a big help for them as he risks
his life to make others’ easy. One fine day, while giving the products, he gets into an
argument with a person over some money. He files an FIR against him claiming that he has
hurt him grievously on the head. The accused said that he did not hit him and it was Tabrez
who hit himself to file a case against him. Due to lockdown, there was no witness to the
incident. Decide.

(a) The court will accept the allegations as Tabrez is important for the society and he should
be safeguarded.

(b) The court will not accept the allegations made by Tabrez because they are evidence less
and cannot be proven in the court of law.

(c) The court will not accept the allegations as this is a trivial case which should not be tried
and court’s time must be saved in these desperate times

(d) The court will accept the allegations as burden of proof is on the accused to prove as this
is a case of grievous hurt.

. A medical nurse is sent to a locality to check on recovered COVID-19 patients. While being
there, she enters in an argument with one of the family members of a recovered patient about
something and amid the argument, she gets hit grievously by them. This happens on 17th
May 2020 and she files an FIR on 20th May 2020. Decide, as to, by when investigation is to
be completed as per the new rules.
(a) 20th June 2020

(b) 19th June 2020

(c) 17th June 2020

(d) 16th June 2020

. In the cases of healthcare service personals being grievously hurt, which of the following,
the accused can prove in the court of law to be acquitted from the case?

(a) That they have not committed the offence.

(b) That they were not in a requisite mental state to commit the offence.

(c) They have to prove nothing as he will be presumed to be innocent.

(d) Both (a) & (b).  

. A doctor is attacked in his office by angry members of one of his deceased patients on 16th
April 2020. He was stuck inside his clinic for the next 3 days, as the attackers wouldn’t let
him out. On 19th April, somehow, he gets out and reaches home. On 21st April, he goes to
file an FIR. The police refuse to file an FIR as the attackers had political connections. Finally,
amid media pressure, FIR is filed on 26th April 2020. Decide, by when, the trial will be
completed respectively.

(a) 21st April 2021

(b) 16th May 2020

(c) 26th April 2021

(d) 26th May 2020

Passage (Q.-Q.): Section 125 of Maharashtra Regional and Town Planning Act, 19 read with
Section 126 enables the state/Planning authority to acquire land. Section read with Section ,
on the other hand, protects the interests of the owners. Considering all the relevant
provisions, the Court held that on a proper construction of Section , when land is acquired for
the purposes of a Development Scheme, the same vests in the State free from encumbrances.
No third party can claim any right of easement to the land, or claim any right as an occupier,
licensee, tenant, lessee, mortgagee or under any sale agreement. However, “Section of the
Regional and Town Planning Act cannot be read in isolation. It has to be read with Section
125 to 129 relating to compulsory acquisition as also Section 59, and .” Article 300A of the
Constitution of India embodies the doctrine of eminent domain which comprises two parts;
possession of property in the public interest; and payment of reasonable compensation. “In
case of dispossession except under the authority of law, the owner might obtain restoration of
possession by a proceeding for Mandamus against the Government.” The Court was hearing
the case pertaining to a private road in Pune being declared as being owned by Pune
Municipal Corporation whilst in the property records, there was no private road. In 19, by an
order of the Pune Municipal Corporation, a Plot was divided into 4 plots and a private road
admeasuring 414.14 square meters. On perusal of the documents, the Court noticed that there
can be no doubt at all that the road in question measuring 444.14 sqm. never belonged to the
Pune Municipal Corporation. In the property records, there was no private road. The
Municipal Corporation was never shown as owner of the vacant plot or of any private road.
Even assuming that there was any policy decision to have an approach road to every plot, it
was incumbent upon the authorities concerned to acquire the land. “There is no whisper as to
how the road came to be shown as in possession of Pune Municipal Commissioner nor of the
procedure adopted for effecting changes, if any, in the property records.” The Court, hence,
held that the Pune Municipal Corporation had a public duty under Section to appropriately
modify the scheme and to show the private road as property of its legitimate owners, as per
the property records in existence, and or in the award of the Arbitrator. It, hence, directed the
Municipal Corporation to “delete the name of the Pune Municipal Corporation as owner of
the private road in the records pertaining to the Scheme and carry out such other
consequential alterations as may be necessary under Section of the Maharashtra Regional
and Town Planning Act, 19.”

. In the city of Bhopal, a 16,000 sqmetres land in the backyard of the Bhopal Municipal
Corporation is possessed by one Mr. Rashid Khan. The land is often used for parking the
vehicles and there is barely any part of the land in use by Mr. Khan. The BMC takes
possession of the land and pays the respected amount to Mr. Khan. Decide.

(a) The possession was legal as it was done in a good faith and proper amount was paid to the
owner.

(b) The possession was illegal as the BMC is not competent enough to do so.

(c) The possession was illegal as the owner was not asked before the sale.

(d) The possession was legal as the possession of land was necessary to be taken for the
welfare of the city.  

. In the above case, Mr. Khan files a complaint with the collector office and the collector,
after hearing the case in a quasi-judicial capacity, gives the order against the complainant.
Decide.

(a) The possession will be legal as the BMC is not doing this job in isolation but is backed up
by proper judicial standings.

(b) The possession will be illegal as the owner’s consent is something which can be done
away.

(c) The possession will be legal as there is no merit left in the case of the complainant i.e. Mr.
Khan.

(d) The possession will be illegal as the BMC is not competent enough to do so.

. RakeshSikarwar has a land in the outskirts of Delhi. DMC ‘s director Mr. Santosh More
wishes the land to be converted into a petrol pump in a similar way it has a petrol pump
across the road so that the vehicles coming from the other way do not have to turn and go to
the old petrol pump on the opposite side or travel 500 m to another petrol pump on the same
side. Under Article 300A, the land is possessed by the DMC and proper amount is paid.
Decide.

(a) The possession is legal as all the act is done is public interest and proper amount is paid.

(b) The possession is illegal as the act doesn’t amount to be that of public interest.

(c) The possession is legal as there exists no fundamental right to property and property, thus
can be acquired by state at any time in any way.

(d) The possession is illegal as the fundamental right of Mr. Sikarwar is violated.

. A person’s land is wrongfully acquired by the state administration. There is a clear non –
compliance of Article 300A of the Indian constitution. He moves to the Supreme Court under
Article 32 asking for a remedy of a violation of his fundamental right and gross negligence
committed towards him. Decide.

(a) The court will give him remedies as it’s a clear case on merits and will issue a writ of
mandamus.

(b) The court will not give him remedies as it’s not his fundamental right to property and
hence the state is allowed to take possession of his land.

(c) The court will not give him remedies as it’s not its jurisdiction to decide upon.

(d) The court will give him remedies as not doing so will set an authoritarian precedence and
might give rise to anarchy.

. In the above case, the person instead of going to Supreme Court, goes to the High Court
under Article 226. Decide.

(a) The court will give him remedies as it’s a clear case on merits and will issue a writ of
mandamus.

(b) The court will not give him remedies as it’s not his fundamental right to property and
hence the state is allowed to take possession of his land.

(c) The court will not give him remedies as it’s not its jurisdiction to decide upon.

(d) The court will give him remedies as not doing so will set a authoritarian precedence and
might give rise to anarchy. Passage

(Q.-Q.): Article 131 deals with the original and exclusive jurisdiction of the Supreme Court,
under which the suit challenging the validity of CAA has been filed. Jurisdiction of a court is
its power or authority to hear and determine a matter. Jurisdiction could be pecuniary (power
of court based on the amount of money involved in the matter), territorial (power of court
based on the geographical limit where the cause of action arises or offence is committed),
subject-wise (power of court based on a particular subject matter), original (power of court to
hear a case from its beginning), appellate (power of court to hear a case on appeal) A
judgment passed by a court without jurisdiction is a corum non judice (in presence of a
person, not a judge). It is null and void and its validity can be challenged at any stage.
Original jurisdiction of a court refers to its power to hear a case first.The constitution of
India, under Article 131, confers the Supreme Court original jurisdiction in certain special
matters. The exclusivity factor of the jurisdiction provides an exceptional and particular
power to the Court to hear and decide the matter than any other Courts. What are the matters
where jurisdiction of the Supreme Court under Article 131 is provisioned? Any dispute
involving a question of law or fact on which existence of legal right depends comes under
original jurisdiction of the SC. The disputes may be: (a) between the government of India and
one or more states; (b) between the government of India and any state/states on one side and
one or more states on the other; (c) between two or more states Is there any bar on
jurisdiction of the Supreme Court under Article 131? Yes, the original jurisdiction of the
Supreme Court does not extend to: (a) A dispute arising out of any treaty, agreement,
covenant, engagement or other similar instrument executed before the commencement of the
constitution and continues to be in operation or which provides that the jurisdiction of the
Supreme Court shall not extend to such a dispute; (b) The parliament may exclude the
jurisdiction of the Supreme Court in disputes relating to the use, distribution, or control of the
water of any inter-state river; (c) Suits brought by private individuals against the government
of Indi(a) The Supreme Court has held in a previous case that Article 131 is attracted only
when a dispute arises between or amongst the states and the union in the context of
constitutional relationship that exists between them and the powers, rights, duties,
immunities, liabilities, disabilities and the like flowing therefrom. [Extracted, with edits and
additions, from ‘What Is Article 131, Under Which Kerala Has Challenged CAA?’, The
Wire, 16th January, 2020]

. BrijeshwarDalmia is involved in a property dispute. The worth of the disputed property was
Rs. lakhs and it was located in Kollam. He filed a case in Kollam district court as any
property dispute under Rs. 2 crores are to be filed in district court. The court ruled in favour
of Mr. Bharath, the other party. Mr. Dalmia then approached the High Court of Kerala as he
thought that justice had not been done. Which of the following is/ are true in the mentioned
case? I. Kollam district court heard the matter as it had pecuniary jurisdiction over it. II.
Kollam district court heard the matter as it had original jurisdiction over it. III. Kollam
district court heard the matter as it had territorial jurisdiction over it. IV. High Court of
Kerala heard the matter as it had original jurisdiction over it. V. High Court of Kerala heard
the matter as it had appellate jurisdiction over it.

(a) Only I and V.

(b) Only I, III and IV.

(c) Only I, II, III and V.

(d) Only I, II and V.


 

. Determine in which of the following cases shows exclusive jurisdiction of the Supreme
Court.

(a) Entry of a particular sect of people was barred in a temple in Andhra Pradesh. Some
people of that sect felt their right to life and right to equality was violated. They could
approach High court under article 226 but anticipating that the verdict of high court will be
challenged in front of the apex court, they directly approached Supreme Court for remedy.

(b) The Constitution of India says that any dispute arising out of the elections of President
will be heard and decided only by the Supreme Court.

(c) The Delhi High Court decriminalized homosexuality. Some people were not happy with
the judgment and Supreme Court was the only court they could go to. So, they approached
Supreme Court which heard the matter and gave its decision.

(d) A Voluntary Consumer Organisation (VCO) had filed a case on behalf of a consumer.
NCDRC refused VCO appearance before the commission saying it cannot voluntarily
represent the consumer. However, VCO did not think that the ruling served the ends of
justice for the consumer. They had no alternative but the Supreme Court to represent their
case.  

. Indo Sikkim treaty was signed in 10 between the Maharaja of Sikkim and the Indian
government. Sikkim finds the treaty to be against the principles of federal autonomy which
are recognized by Indian as well as international laws. The government of Sikkim wants
changes in some sections of the treaty. It has tried to negotiate with the union government but
it did not work. Can the government of Sikkim approach the Supreme Court under Article
131?

(a) This is a matter concerning constitutional relationship of the state with the union and
hence Sikkim government is free to approach Supreme Court under Article 131.

(b) Sikkim government cannot approach Supreme Court as it would encourage other states to
demand for autonomy which might lead to dismemberment of states.

(c) Sikkim government can approach the Supreme Court under Article 131 only if the Indo
Sikkim treaty was signed after 26th January.

(d) Sikkim government should try to settle the dispute out of court as it will lead to a
mutually acceptable solution in a fairly quick time.

. Consumer Protection Act, 2019 provides for jurisdiction of various commissions. Cases
where amount claimed is less than one crore will be heard by district commission, those from
one to ten crore by state commission and those over ten crores by national commission. It
also provides that the case has to be instituted at a place where the respondent carries his/ her
business. Mr. Gautam, a resident of Bengaluru purchased a fairness cream after watching the
advertisement on a television channel. He applied the cream as per instructions but his skin
color remained same. He felt cheated and approached Karnataka State Consumer Dispute
Redressal Commission claiming amount of one crore as compensation. The company did not
carry on its business from its sole office in Noida, Uttar Pradesh. The state commission
decided in favour of Mr. Gautam and awarded him the desired compensation. Which of the
following is most appropriate?

(a) The decision of the commission is valid as it had both pecuniary and territorial
jurisdiction over the given matter.

(b) The decision of the commission is null and void because it did not have territorial
jurisdiction over the given matter.

(c) The decision of the commission is null and void because it did not have pecuniary
jurisdiction over the given matter.

(d) The decision of the commission is null and void because it did not have both territorial
and pecuniary jurisdiction over the given matter.

12
(Passage Q.-Q.): The tort of intentional infliction of emotional distress (IIED) is defined as
the plaintiff acting abominably or outrageously with the intention of causing the defendant to
suffer severe emotional distress. This is typically done by a defendant vocally issuing the
threat of future harm to a plaintiff. The elements of establishing a prima facie case in this
regard are (1) the defendant acts (2) the defendant‘s conduct is outrageous (3) the defendant
acts for the purpose of causing the victim emotional distress so severe that it could be
expected to adversely affect mental health (4) the defendant's conduct causes such distress.
Certain intentional actions which may meet the prima facie case for an IIED (particularly as
related to the outrageous conduct components) may not qualify for tort liability as an IIED,
depending on the person at whom the conduct is directed or who commits the action.
Typically, a court will not assign IIED tort liability to a defendant who speaks harmfully
about public figures. Further, as per Texas v. Johnson (19), ―Government may not prohibit
the expression of an idea simply because society finds the idea itself offensive or
disagreeable.‖ There are certain possible defences to the tort too. If the plaintiff gives consent
to the defendant to engage in the outrageous conduct, then courts will likely consider the
conduct to be non-outrageous, thus negating the prima facie claim. Further, context matters as
well. If the conduct is done in a situation in which it may be deemed normal or appropriate,
then the prima facie claim is likely negated. Some jurisdictions will expand IIED liability by
modifying the prima facie case. Rather than requiring that the defendant's action causes
emotional distress in an intended plaintiff, some jurisdictions will allow that even if the
defendant directs conduct at plaintiff A, but someone close to Plaintiff A (Plaintiff B) suffers
severe emotional distress, then Plaintiff B is allowed to bring an IIED claim against the
defendant.In Snyder v Phelps (2010), the Supreme Court signaled a move away from
imposing IIED liability. The Court set aside the trial court's jury verdict that found IIED
liability: "[Applying the IIED tort] would pose too great a danger that the jury would punish
[the defendant] for its views on matters of public concern." [Source:
https://www.law.cornell.edu/wex/intentional_infliction_of_emotional_distress]
. Sugriv and Bali are brothers but bitter enemies. There have been multiple instances where
Bali has beaten up Sugriv badly and Sugriv now hates and fears Bali badly. On April 1, Bali
passes by Sugriv‘s home. Sugriv is scared and distressed. Decide

(a) Bali is liable for IIED for he has caused severe emotional distress to Sugriv

(b) Bali is not liable for IIED for he did not act thus the elements of IIED are not fulfilled.

(c) Bali is liable for IIED for his act was walking in front of Sugriv‘s house.

(d) Bali is not liable for IIED for Sugriv is super sensitive

. Rajesh has quite a few enemies in his neighborhood. One such enemy hires a goon who is
now standing outside Rajesh‘s house with a sunflower. Rajesh has allergy to sunflower pollen
but no one except his family knows about the same. Decide

(a) The goon is liable for IIED for he has caused severe emotional distress to Rajesh

(b) The goon did not act in an outrageous manner thus is not liable for IIED

(c) The goon is liable for IIED for Rajesh was scared suspecting that the goon might know
about his allergy (d) The goon is not liable for he did not act.

. Anand‘s father does not like his son for he did not make life choices as per his wishes. One
day, Anand‘s father stood outside his son‘s house with a flower bouquet. However, he knew
that Anand was allergic to a few flowers in the bouquet. Decide.

(a) Anand‘sfatheris liable for IIED for he has caused severe emotional distress to his son, as
he knew about his allergy

(b) Anand‘s father did not act in an outrageous manner thus is not liable for IIED (c) Anand‘s
father is liable for IIED.

(d) Anand‘s father is not liable for it was only a flower that would have made him sneeze

. Vishal and his brother Anil were former members of a band and lovers of Meera, the
famous beer brand. The band broke up and it did not end well with Vishal and Anil hitting
each other with beer bottles. Vishal wished to end the rivalry between the two. He went to
meet Anil with a bottle of beer to recount their old memories. Decide

(a) Vishal is liable for IIED for he has caused severe emotional distress to Anil as he knew
about his allergy

(b) Vishal did not act in an outrageous manner thus is not liable for IIED

(c) Vishal is liable for the context does demonstrates an outrageous act (d) Vishal is not liable
for the context does not demonstrate an outrageous act 15 of 36
. A case occurred in Kerala wherein a Malyalam magazine printed a picture of a woman
breastfeeding her child. There was objection raised on the decency and obscenity of the same
and that the picture hurt the sentiments of the society causing them distress. Decide (a) The
magazine is liable for IIED for it has caused severe emotional distress to Bali (b) The
magazine is not liable for IIED for they have not acted in an outrageous manner (c) The
magazine is liable for IIED for it threatened the fundamental values of Indian society (d) The
magazine is not liable for IIED for only the society finding it offensive is not a good ground
to establish IIED.

(Passage Q.-Q.):The Indian Penal Code under Section 300 defines murder. The code lays
down circumstances in which an act of culpable homicide may become an act of murder. The
first instance is where an act by which death is caused was done with intention of causing
death. Here action also includes intentional omission. Where a child was very ill and his
family members refused to take him to the hospital for providing necessary treatment, as a
result the child dies. Such a case shall be called as murder. The second instance is where
there is an intention to cause bodily injury which is likely to cause death. Here the offender
has the intention to cause bodily harm and knowledge that by his act death will be caused.
This clause may include those cases where the victim was suffering from a trauma or disease
that the offender knew about and used it as an advantage by inflicting bodily injury which
resulted in death of that person, what may not have caused death of a normal person. The
third is when injury is sufficient to cause death in the ordinary course of nature. For this
clause the offender must have intention and would‘ve caused a bodily injury that is sufficient
enough to cause death to any other person in ordinary course of nature. The fourth is having
knowledge of the dangerousness of the act. Where the offender doing the act knows the
gravity of his actions that it will, in all probability, will cause death of the person or the
bodily injury that is inflicted is likely to cause death, and still commits such act without any
prominent excuse for causing death or such injury. This is applicable on cases of dangerous
action without any intention to cause specific injury. For e.g., rash driving. [Source:
http://lawtimesjournal.in/murder/]

. Rathore is a scientist. He administered a new variety of glucose to a human subject


intending to test the substance. The subject seemed to have the intended reaction at first,
however, died later. Rathore however, never intended for the subject to die. When an enquiry
was conducted it was found out that the glucose was of a dangerous nature and significantly
harmful to human health. Which of the following is a defence that Rathore can take?

(a) Rathore is not liable for murder as there was no intention to kill the subject.

(b) Rathore is not liable for murder as the glucose would not in the ordinary course, lead to
death. 

(c) Rathore is not liable for murder as he did not know that the glucose was so dangerous to
human health.

(d) The results of the experiment were beyond Rathore‘s control and he cannot be liable for
such actions.

. After Bittu hit Amish on his head with an iron rod, he was taken to the hospital. The doctor
refused to take care of Amish, who could have been saved. Amish died

(a) The doctor is liable for murder of Amish for they had an intention of causing death
(b) The doctor is not liable for murder of Amish as he did not intend to cause A‘s death

(c) The doctor is liable for murder of Amish for his inaction is read as an ―act‖ within
Section 300, IPC.

(d) The doctor is not liable for murder of Amish as he was busy attending other patients.

. After Bittu hit Amish on his head, he was taken to the hospital. The doctor could not attend
to Amish‘s condition as he was busy examining other patients. Decide

(a) The doctor is liable for murder of Amish for they had an intention of causing death

(b) The doctor is not liable for murder of Amish as he did not intend to cause A‘s death

(c) The doctor is liable for murder of Amish for his inaction is read as an ―act‖ within
Section 300, IPC.

(d) The doctor is not liable for murder of Amish as he was busy attending other patients.

. Aftab likes to play PUBG. He learns how to make Molotov cocktails online, makes them
and starts throwing the same in his neighbourhood. Luckily for Aftab, they do not explode
but Betty trips on one of the bottles, hits his head on the ground and dies. Decide

(a) Aftab is liable for murder for he should have realised the dangerousness of his act

(b) Aftab is not liable for murder for there was no intention of killing Betty

(c) Aftab is liable for murder for he knew that someone may trip on the bottle

(d) Aftab is not liable for murder. He started playing COD instead.

. Please refer to the facts above. Aftab makes better molotovs now. One of them explodes
near a car and the car explodes. As a result of the car explosion, a neighbor die. Decide

(a) Aftab is liable for murder for he should have realised the dangerousness of his act

(b) Aftab is not liable for murder for there was no intention of killing Betty

(c) Aftab is liable for murder for he knew that someone may trip on the bottle

(d) Aftab is not liable for murder. The neighbor was killed in car explosion.

(Passage Q.-Q.): Quite recently, the World Health Organisation (WHO) declared the novel
Coronavirus (COVID-19) a pandemic. According to the WHO, a pandemic is defined as the
"worldwide spread" of a new disease. WHO, however, also remarked that it is a pandemic
which can be controlled if governments around the world undertake effective measures. In
response to the novel virus which has claimed more than 5000 lives worldwide and affected
more than 1,45,000 people, governments around the world have imposed, prohibiting any
mass gathering. Furthermore, leading multinational corporations have asked their employees
to work from home and all major sporting events have been suspended. However, a piece of
news recently garnered a storm over social media when it was reported that a COVID-19
victim who had been quarantined in Bengaluru escaped from Bengaluru on March 8, took a
flight to New Delhi and further travelled to Agra to be with her parents. In addition to this,
they faced resistance on making attempts to quarantine the said person. The questions which
now arise for consideration from a legal perspective are: Which law provides the state the
power to use appropriate force against such persons in order to place them into quarantine?
What is the legal framework in relation to the control of epidemics in India? What are the
gaps in the existing framework and what is the framework in some other nations? There are
two legislations which have been invoked by the Union and State Governments to effectively
tackle the outbreak and spread of COVID-19. The first is the Epidemic Diseases Act, 17. As
the year of enactment suggests, the Epidemic Diseases Act is a colonial era legislation which
was primarily enacted to tackle the epidemic of bubonic plague that had spread in the
erstwhile Bombay Presidency in the 10s. Using powers conferred by the Act, colonial
authorities would search suspected plague cases in homes and among passengers, with
forcible segregations, evacuations, and demolitions of infected places. Section 2 of the
Impugned Legislation is a sweeping section that empowers the state governments/UTs to
prescribe special measures and formulate regulations for containing the outbreak and spread
of an epidemic if it thinks that the ordinary provisions of the law for the time being in force
are insufficient for the purpose. These regulations can range from prescribing such temporary
regulations to be observed by the public or by any person or class of persons as it shall deem
necessary to prevent the outbreak of such disease or the spread thereof to the inspection of
persons travelling from railways or otherwise, segregation in hospital, creation of special
wards etc. Section 2A provides similar powers to the Central Governments in reference to
ports, vessels etc. Section 3 penalises a person for disobedience of any order given by any
public servant. For such a violation to occur, there must be 1) an order promulgated by a
public servant 2) such public servant must have been lawfully empowered to promulgate such
order 3) a person must have knowledge of the existence of such an order 4) such
disobedience must cause or tend to cause obstruction, annoyance, or injury. Section 4 of the
act provides immunity to any person acting under the act. [Source:
https://www.livelaw.in/columns/covid-19-decoding-the-legal-framework-for-pandemic-
control-1538]

 . A virus named PAX-12 spread from a village in Western Africa in 2007. This virus showed
symptoms of infectivity through coughing and could also travel on dust particles present in
the air and had a long life in water causing transportation by sea to be another means of
transmission. The virus gradually spread from person to person and through air to multiple
countries in Western Africa killing more than a whopping 16 people. Concerned, the WHO
decides to label the disease as a pandemic.

(a) PAX-12 cannot be labeled as a pandemic for it has not yet claimed enough lives

(b) PAX-12 can be labeled as a pandemic for it has affected multiple countries.

(c) PAX-12 cannot be labeled as a pandemic for the disease has not spread worldwide.

(d) PAX-12 can be labeled as a pandemic for the disease may otherwise spread worldwide.
. A case of 6 members of a family was discovered in Kerala who had travelled from Italy.
These family members failed to inform the government of their travel history and were
staying at their home for about 10 days before the emergency medical teams arrived at their
homes. The medical team demanded that they be allowed to search the home of the family.
The family resisted. Decide

(a) The emergency medical team is justified in searching the place of the family under the
Epidemics Act

(b) The emergency medical team is unjustified in searching the place of the family for they
have a right to privacy.

(c) The emergency medical team is justified in searching the place of the family only if state
has formulated rules expressly allowing such search. (d) The emergency medical team is
unjustified in searching the place of the family since the police exist.

. The state of Punjab has prescribed sweeping norms that require mandatory testing of any
person who deplanes in India and is coming from abroad especially Canada. Gurpreet comes
from Kuwait on board a ship to Mumbai. He then takes a flight from Mumbai to Amritsar. He
is subjected to testing by the authorities.

(a) Gurpreet should be tested for he has come from abroad.

(b) Gurpreet should not be tested for he has deplaned from a flight that took off from
Mumbai

(c) Gurpreet should be tested for he might be a danger to the community.

(d) Gurpreet drinks lots of cow milk. He can never catch coronavirus.

. The State of Tripura has imposed a curfew due to coronavirus. The existing laws impose a
penalty of Rs. 0 and imprisonment for violation of such curfew by any individual. However,
the state assembly, in light of the pandemic of COVID-19 has declared an additional penalty
of 2 years imprisonment upon the violation of the curfew. Dheer, a resident challenge such
enhanced penalty.

(a) The state of Tripura is justified in imposing the ban due to the urgency and necessity of
the same.

(b) The state of Tripura is unjustified in imposing the ban for it is unreasonable punishment.

(c) The state of Tripura is justified in imposing the ban for the Epidemic Act allows it to do
so.

(d) The state of Tripura is unjustified in imposing the ban for existing laws are sufficient to
prevent harm.

. The State of Karnataka has banned L&L from conducting any classes in light of the
impending COVID19 threat. L&L challenges such decision of the government before the
High Court of Karnataka.
(a) L&L shall be given relief for such ban cannot be imposed on their right to trade.

(b) L&L shall not be given relied for Epidemic Act allows for such a ban to be placed on
them.

(c) L&L shall be given relief for Epidemic Act allows to place such a ban only on a class of
persons.

(d) The information given about the nature of ban is insufficient for a decision to be made.

(Passage Q.-Q.):Section was part of the Indian Penal Code as enacted in 18. Explaining
Section , Supreme Court speaking through Justice Vivian Bose in Amjad Khan Vs the State
(12) held as follows: "Under section of the Indian Penal Code the right extends not only to
the defence of one's own body against any offence affecting the human body but also to
defending the body of any other person. The right also embraces the protection of property,
whether one's own or another person's, against certain specified offences, namely theft,
robbery, mischief and criminal trespass". The right of private defence, however, cannot be
claimed by an accused, if the facts suggest that the parties had involved in a free fight
between them, and it could not be determined as to which party was the first aggressor, thus
forcing the other party to use force against the aggressor, so as to defend. The Supreme
Court, in Suresh Singhal vs. State, has held that a mere reasonable apprehension of danger to
life is enough to put the right of self-defence into operation and it is not necessary that there
should be an actual commission of the offence in order to give rise to the right of private
defence. This court has held that a person who is in imminent and reasonable danger of losing
his life or limb may in exercise of self-defence inflict any harm even extending to death on
his assailant either when the assault is attempted or upon being directly threatened. We are
inclined to think that the appellant had been put in such a position,‖ the bench said, referring
to Darshan Singh vs State Of Punjab. The court said: ―In the instant case, Suresh Singhal
had exceeded the power given to him by law in order to defend himself, but we are of the
view that the exercise of the right was in good faith, in his own defence and without
premeditation.‖ ―In such a situation, it would be unrealistic to expect the appellant to calmly
assess who would have the upper hand before exercising his right of private defence,‖ the
bench said. [Ashok KM ‗Right To Private Defence Is Available Even When There Is
Reasonable Apprehension Of Danger To Life: SC‘ Live Law as accessed on 17 September
2020]

. Sandhus and Randhwas were two most powerful families of Sikha village, with a long
history of feud between them. When one of Randhawa‘s married Sandhu girl, infuriated
Sandhus began slaying every Randhwa they could find in the village. In the process, they
killed every man, woman and even elderlies of the family and buried them all in the ground.
At last when they found a small boy crying in a room, they slit his throat and left him there to
die. Their justification to the crime was their right to private defence, as the boy would have
sought vengeance in future had they left him alive. Decide whether the act of killing the boy,
in isolation, was an act of private defence.
(a) It was not an act of private defence, Sandhus had no imminent threat from the little boy as
he could have not caused any harm to any person or property, thus no right to private defence
invoked at the end of Sandhus, to exercise.

(b) It was an act of private defence, as the boy would have sought vengeance against Sandhus
after he would grow up, this causes a threat of life from the boy and therefore gives Sandhus
right to private defence against the boy.

(c) It was not an act of private defence, as the threat from the boy was very far-fetched and
the force used against the boy was too disproportionate as compared to the threat he radiates.

(d) It was an act of private defence, as leaving the boy alive would have furthered the feud
between the two families, causing threat to life of members of both families. Therefore,
killing the boy was an exercise of lawful right to private defence.

. Kabir was gifted a Bullet bike by her grandmother. After her demise, the bike reminded her
of her grandmother and therefore he looks after the bike really well. Once in a football
competition, he was embroiled in a fight with the players of the opposing team and beat one
of them, Karan, brutally. The next day, when he parked his bike in front of the college, Karan
started denting his bike with a stick. Kabir saw it happening, while coming out of the college,
and he threw a stone at him. The stone hit Karan and injured him. Karan filed a police
complaint, and argued in the trial that if no harm was caused to Kabir himself he has no right
to cause harm to Karan. Kabir is seeking the right under private defence. Decide:

(a) There is no valid right to private defence, as the fight between the two was a free fight
agitated by Kabir and thus he gets no right to exercise private defence against Karan, even if
the same is to defend a person or any property.

(b) There is valid right to private defence, as the right to private defence also extends to
property, along with the human body, thus an act to protect his bike by throwing a stone at
Karan was a valid act of self defence.

(c) There is no valid right to private defence, as there was no threat to the life of Kabir when
he threw stone at Karan, mere denting of a bike does not invoke any right to private defence
at the end of Karan.

(d) There is valid right to private defence, as Karan became a threat to Kabir, when Kabir
beat him up after the football match, so when he dented Kabir‘s bike, it was a threat not only
to his property but also a potential threat to his life.

. Bablu and Guddu, were two youngsters who stepped in the underworld from a very early
age. They became trusted men of Kaleen Bhaiya, the don of Mirzapur. Where Bablu was a
smart person, with great strategies, Guddu was the one who expanded the fear of Kaleen
Bhaiya in nearby territories. Their meticulous involvement in the business was not
appreciated by Munna, son of Kaleen Bhaiya. After Kaleen Bhaiya‘s death, the throne of
Mirzapur was lying vacated. All of them decided to find the successor by a gang war. They
all met at SudarshanChowk in Mirzapur to fight. During the fight, Guddu shot Munna, to
save Bablu from being thrashed on his head. Was shooting Munna would be an act of self
defence on the part of Guddu?

(a) The act of shooting Munna is not an act of private defence, as their fight was a free fight
in which no one gets a right to private defence even if the action was in furtherance of
protecting a person‘s body or any property.

(b) The act of shooting Munna is an act of private defence, as Munna approaching Bablu to
thrash him with a stick, created a reasonable apprehension of threat to Bablu‘s life,
consequently invoking the right to private defence.

(c) The act of shooting Munna is not an act of private defence, as Munna claimed the throne
to Kaleen Bhaiya‘s business, and became an aggressor by forcing Guddu and Bablu to
indulge in a fight, and the act of Guddu was, thus, to defend Bablu.

(d) The act of shooting Munna is not an act of private defence, as Munna had not yet hit
Bablu when Guddu shot him and thus there was apprehension of threat to Bablu‘s life.
Therefore, there arose no right to private defence.

. Dimpy woke up at night after hearing some noise in their kitchen. She slowly stood up and
walked towards the kitchen, and saw somebody stealing food from their kitchen. Dimpy
shouted to wake everybody, scared thief slowly moved towards Dimpy and waived knife,
who till then picked up a gun in her hands. Seeing the gun, the thief ran to save his life and
jumped out of the window. After he escaped the house, Dimpy shot at him to prevent him
from getting away. The bullet hit the thief‘s leg and injured him badly. The thief has now
filed an FIR against Dimpy, for causing hurt and attempt to murder. During the trial Dimpy
claims the defence of private defence, for protecting her own body and property. Which of
the following is correct?

(a) Dimpy can take the right to private defence, as she fired the shot to prevent the thief from
running, who was waving a knife at her which caused her imminent threat and apprehension
of danger to life.

(b) Dimpy cannot take the right to private defence, as when the thief was shot he had already
escaped the house of Dimpy and thus there was neither any threat to anyone‘s body or
property. Therefore, the act of shooting the thief was not an act of private defence.

(c) Dimpy can take the right to private defence, as if the thief had escaped, there would have
been a reasonable apprehension of his returning back at Dimpy‘s home. Thus, the act was
reasonable in the view of apprehension to life.

(d) Dimpy cannot take the right to private defence, as her action was highly disproportionate
and grave as compared to the actions of the thief, she could have used the right to private
defence against the actions of the thief but could not shoot her for waiving a knife.

. Seema was about to get to bed after finishing all her daily chores, at around 1:00 in the
night. She heard a distant gunshot and came out of her house to check what happened. She
saw a silhouette of a man coming towards her with a gun on his shoulders, he shouted to ask
who it was but got no reply. As an immediate reflex, she brought a knife from her house and
killed the man when he came close to him. It was found in the morning, the person who died
was Seema‘sneighbour, who was coming back from hunting in the forest. Police filed the FIR
and made Seema the accused for culpable homicide, not amounting to murder. Both the
subordinate courts, i.e. trial court and High Court, have convicted Seema. The appeal against
the judgment is now pending before the Supreme Court of India. Decide:

(a) Seema is not guilty of culpable homicide, as there was no malafide intention, i.e. mensrea,
on the part of Seema, which is a determinant factor while classifying a case as culpable
homicide.

(b) Seema is guilty of culpable homicide, as there was no reasonable or imminent


apprehension of danger or threat on her part and therefore killing a person in such a case is an
evident case of culpable homicide.

(c) Seema is not guilty of culpable homicide, as her act of killing the person approaching her
with a knife was in furtherance of private defence, the need for which arose from the
imminent threat when a man was approaching her with a gun right after she heard a gunshot.

(d) Seema is guilty of culpable homicide, as she intentionally killed the man, which is clear
from her action of harming him before she inflicted any harm on her. Intentional act of killing
cannot be shrouded with the exception of private defence.

(Passage Q.-Q.):The recent decision of the Supreme Court in the case of P Chidambaram v.
Directorate of Enforcement yet again gives rise to the eternal debate between custodial
interrogation and anticipatory bail. Section 438 of the Criminal Procedure Code deals with
‗grant of bail to a person apprehending arrest‘ and empowers the High Court or the Court of
Sessions to grant anticipatory bail upon exercising discretion. Anticipatory bail can be
granted subject to conditions that the accused shall make himself available for investigation
as and when required and not threaten or influence witnesses or tamper with evidence. The
judgment in Gurbaksh Singh Sibbia v. State of Punjab sends out a word of caution that
Courts should be particularly mindful of the stigma attached to arrest of an individual as well
its ramifications while deciding an application for grant of anticipatory bail. For this purpose,
Courts should look into the authenticity of the allegations to rule out ulterior motives and
potential objectives to injure and humiliate an applicant by having him arrested. The Supreme
Court, in its decision in the case of SiddharamMhetre v. State of Maharashtra (2011), added a
new dimension to the interpretation of Section 438 by specifying that there is no requirement
to make out a ‗special case‘ for exercise of power under Section 438. It held that custodial
interrogation should be avoided in cases where the accused has joined investigation and is
fully cooperating with the investigating agency and is not likely to abscond. NandiniSatpathy
v. PL Dani broke new ground by its opinion that Article 20 (3) becomes operative from the
stage of police interrogation. The judgment took a step further and pronounced that in terms
of Article 20 (3) read with Article 22 (1) of the Constitution, the police is required to permit
the advocate of the accused to be present at the time he is examined. It is common knowledge
that custodial interrogation is the weapon wielded by investigating agencies to secure
clinching evidence against an accused. On the other hand, anticipatory bail is the shield
deployed by an accused to avoid the inclemency of arrest and custody. Divergent opinions
and views emanate from the Supreme Court in cases involving anticipatory bail. Subjectivity
has become the hallmark in bail matters because peculiar facts play a pivotal role in
influencing the Court‘s decision to grant or refuse bail. [ChinmoyPradip Sharma
‗Anticipatory Bail versus Custodial Interrogation: Striking a balance‘ Bar and Bench as
accessed on 18 September 2020]

. Bala was a dacoit, who lives in the forests of Sunderbans. He knows the forest so well, that
the Police have failed to nab him several times, as he manages to escape. Now that he knows
that Police comes with more force every time to nab him, he decided to avoid arrest by filing
for an anticipatory bail application for all the prosecutions pending against him. He made the
application before a local magistrate, promising to fully cooperate with the investigation. The
magistrate, however, declined his request as well as the application. Bala was arrested by the
Police from the magistrate‘s court. He feels cheated and has challenged the order of the
magistrate by which he did not grant the anticipatory bail. Which of the following is the
correct option?

(a) The court could not have declined the bail application, as Bala assured his full cooperation
in the investigation, and the court could have reasonably made out that he would also not try
to tamper with the evidence and the witnesses, if he had himself submitted to the jurisdiction
of the court.

(b) The court could have declined the bail application, as the court has to be extra cautious
while deciding cases of hardened and notorious criminals, as they would abscond or tamper
with the evidence.

(c) The court could not have declined the bail application, as the court shall look beyond the
ulterior motives, and potential objectives to injure and humiliate an applicant by having him
arrested.

(d) The court could have declined the bail application, as the court can exercise discretion
while deciding an application for anticipatory bail, on the grounds that the person would
neither abscond or nor tamper with the witnesses.

. Sushil returned to his village after completing his post-graduation in law from Cambridge
University. While resolving a property dispute between his and his uncle‘s family, he had to
throw stone at his cousin to defend his father from being hurt. The stone inflicted a serious
wound and the cousin is admitted in a hospital. Sushil knows he will easily get acquitted as
his actions were covered under the right to self defence, however, he applied for an
anticipatory bail, assuring that he is a law student and won‘t abuse the criminal procedure.
Sushil was already cooperating with the investigating agency, while the application was
pending before a magistrate. His application was denied, and he was arrested. In the light of
the judgment in SiddharamMhetre v. State of Maharashtra, decide whether the magistrate's
order was correct or not.

(a) The decision of the magistrate was incorrect, as Sushil had committed the offence in
exercise of his right to private defence and has already been cooperating with the
investigation; in such a case his arrest shall be avoided.
(b) The decision of the magistrate was correct, as the magistrate has the discretion while
allowing any application for anticipatory bail and he can decline such an application if he
feels that the requisites are not met.

(c) The decision of the magistrate was incorrect, as Sushil was a meritorious law graduate and
was well aware of the law and would never take any step that would result in harming the
law's very sanctity.

(d) The decision of the court was correct, as arrest gives a more beneficial outcome in the
investigation of an offence, so the bail shall be granted in only special cases and be declined
in the rest.

. Abu Mallick was being searched by Police due to the violence that he created due to his hate
speech against the government and its ideology about the minorities of the nation. He was
reported to have been seen in Allahabad, so a team of six officers travelled to Allahabad to
bring him to Bangalore. The orders were to arrest him and bring him to Bangalore, where he
would be examined. He was arrested, and after obtaining a transit remand from the local
magistrate was being taken to Bangalore. He reiterated that he wants to see his lawyer, which
is also his constitutional right. However, for being in transit his request was denied. Decide
whether the denial of right to a lawyer in the present case is a breach of fundamental right. 

(a) There is a denial of fundamental rights, as the constitution of India gives a right to a
lawyer to every arrested person and non-compliance of the same is a breach of the
fundamental right by the state.

(b) There is no denial of fundamental rights, as the right to lawyer to an arrested person only
accrues at the stage of examination of the arrested person by the police and not before.

(c) There denial of fundamental rights, as the police failed to give the right to lawyer to Abu
in Allahabad before transiting him to Bangalore for examination.

(d) There is no denial of fundamental rights, as the same can be exercised with reasonable
restrictions, and a lawyer cannot be provided to Abu during the transit, however if the same is
not done after reaching Bangalore then the same would be a breach of fundamental rights.

. Lathika is a law student, who has already started extending help to others in terms of legal
advice. When a friend of hers eloped with a minor, she suggested him to apply for an
anticipatory bail, to avoid arrest in case he is charged with abduction. The friend hired a
lawyer and applied for an anticipatory bail, which was denied and he was arrested when FIR
was filed for abduction of the girl. Lathika has now suggested the friend to challenge the
order of denial of the anticipatory bail and to file another application for anticipatory bail
alongside. She further suggested the friend to give assurance of cooperating in the
investigation, as the same is the ultimate criteria for allowing anticipatory bail application,
and it could not be denied this time. According to you, were suggestions of Lathika legally
accurate?

(a) Lathika‘s advice is legally inaccurate, as the court is not bound to allow the anticipatory
bail application, even upon an assurance of the applicant to neither abscond nor tamper with
the evidence and witness.
(b) Lathika‘s advice is legally accurate, as the court, when assured of the criteria as listed by
the Supreme Court in the aforementioned cases, like cooperation in investigation, shall allow
the application, to save the applicant from the humiliation that the arrest entails.

(c) Lathika‘s advice is legally inaccurate, as an application for anticipatory bail can be filed
before a court when there is apprehension of arrest and not when the same is already made.

(d) Lathika‘s advice is legally accurate, as the application for cooperation in the
investigations is the only relevant criteria, while deciding an application for the anticipatory
bail.

. Mrs. Lizzy was mid-aged woman who runs a flower shop in Prism County. The county
witnessed several cases of missing children, who were never to be found. The county sherif
was conducting regular patrolling but still the kids kept going missing. A neighbour of Mrs.
Lizzy‘s, saw blood dripping out of her drain and made out it to be a human‘s. The news
spread like a wildfire, that Mrs. Lizzy is a cannibal who has eaten all the missing children.
Apprehended of arrest, Mrs. Lizzy filed for an anticipatory bail application, the judge
scrutinizing the same, asked her for the reasons of applying for the bail. Concluding that there
is no medical illness or no dependants of Mrs. Lizzy declined the application. Mrs. Lizzy was
arrested for the crime of cannibalism and is being tried in a court. Decide:

(a) The bail application was not rightly declined, as the court could have shown consideration
taking into account that an arrest would be humiliating to her reputation, putting her in a
more miserable position than she is already in, after the rumors against her.

(b) The bail application was rightly declined, as for crimes as serious as cannibalism the
person shall not be left in the open, because the same would be a threat to the society, putting
more life in danger.

(c) The bail application was not rightly declined, as there is no need for special reasons to
grant anticipatory bail by a judge, only if requisites in the law are satisfied the application
could be allowed.

(d) The bail application was rightly declined, as there is no need for an anticipatory bail if
Mrs. Lizzy is unable to furnish valid reasons for the same to be granted by the judge.

(Passage Q.-Q.): In a bizarre case Bombay High Court recently allowed an application filed
to quash an order of the Trial Court by way of which Section 4A, IPC proceedings were
initiated for inflicting cruelty on a woman. In doing so, Justice AM Badar opined that Section
4A of the Indian Penal Code (IPC) would not be attracted for every instance of cruelty
against a married woman. After extracting the provision in the judgment, the Court said,
"Section 4A of the Indian Penal Code does not come into play in every case of harassment
and/or cruelty to a married woman. What is required to be shown is willful conduct of such a
nature as is likely to drive or propel or compel a married woman to commit suicide or to
cause grave injury or danger to her life, limb or health. It must be shown that acts were of
such a nature, as were sufficient for causing a married woman to lose her normal frame of
mind." The Court explained that, "the term ―cruelty‖ implies harsh and harmful conduct
with certain intensity and persistence. It covers acts causing both physical and mental agony
and torture. In order to hold that the acts amount to cruelty, it must be shown that such acts
amount to unbearable, continuous, repeated acts of brutality." On the other hand, Section 4A
says - Husband or relative of husband of a woman subjecting her to cruelty — Whoever,
being the husband or the relative of the husband of a woman, subjects such woman to cruelty
shall be punished with imprisonment for a term which may extend to three years and shall
also be liable to fine. Explanation. — For the purposes of this section, ―cruelty‖ means— (a)
any wilful conduct which is of such a nature as is likely to drive the woman to commit
suicide or to cause grave injury or danger to life, limb or health (whether mental or physical)
of the woman; or (b) harassment of the woman where such harassment is with a view to
coercing her or any person related to her to meet any unlawful demand for any property or
valuable security or is on account of failure by her or any person related to her to meet such
demand. Source- https://www.barandbench.com/news/litigation/it-must-be-shown-that-the-
acts-were-sufficient-to-cause-a[1]married-woman-to-lose-her-frame-of-mind-bombay-hc-
while-interpreting-section-4a

. Praful and Hansa are married for 5 years and have a son named Bittu. Praful was not happy
with his Hansa as she was devoting all her time in learning English rather than taking care of
the household. He once shouted at her and asked her to pay attention to household chores and
their son. Hansa is upset after this incident. Decide

(a) Praful has committed cruelty for he has induced his wife Hansa to commit suicide.

(b) Praful has not committed cruelty for mere shouting is not enough.

(c) Praful is liable under domestic violence act for he coerced her to meet an illegal demand

(d) Praful is trying to dominate her and as a resultant have caused Hansa mental torture.

. Bhaiyaji‘s wife Bhabhiji is a famous drug peddler in Mumbai. She sources drugs from
African countries through her links in customs department. Once Bhaiyaji asked Bhabhiji to
get him an expensive stolen laptop delivered from US. Getting a stolen laptop through
customs was difficult and therefore Bhabhiji denied Bhaiyaji‘s request. As a resultant,
Bhaiyaji beat her up black and blue and Bhabhiji broke her wrist. Decide

(a) Bhaiyaji is liable to be punished under Section 4A of IPC as he harassed his wife for an
illegal demand of property.

(b) Bhaiyaji is not liable for the property was legal to be imported.

(c) Bhaiyaji is liable for causing mental harassment to Bhabhiji for refusing his demands.

(d) Bhaiyaji's liability will depend on the statement given by Bhabhiji in the court of law.

. Heera and Panna are childhood buddies and considers each other as family. Heera is married
to Sona who is an income tax officer. Panna who runs a grocery business was caught by
authorities for committing tax fraud. Panna has been harassing Sona to get his case settled
and when Sona couldn‘t get it settled, he defamed her calling her to be a bad character
person. Sona brought as case against Panna under Section 4A of IPC. Decide.

(a) Panna is liable for he is as good as Heera‘s family and harrassedHeera‘s wife.

(b) Panna is not liable for he is not a relative and will not be charged under Section 4A.

(c) Panna is not liable for he is not a relative though he can be charged under some other law.

(d) Panna is liable for he not only harassed Sona but also defamed her.

. Mr. Brainless was a staunch believer in Chanakya's philosophy which stated that from 0-5
years, a child must always be forgiven, from 5-18 years he must be beaten/rebuked for his
mistakes and after 18, he becomes a friend of his parents. Mr. Brainless used to beat up his
children often when they committed mistakes. Their mother, Mrs. Sensible however did not
like this beating at all and often complained. Decide

(a) Mr. Brainless is liable under Section 4A of IPC as his beating of children forms part of
cruelty.

(b) Mr. Brainless is not liable under Section 4A of IPC for his acts are not enough to
constitute cruelty.

(c) Mr. Brainless is liable under Section 4A of IPC for the mental health of Mrs. Sensible was
affected by such beating.

(d) Mr. Brainless is not liable. He is only following Chanakya's philosophy.

. Pooja file as case a case against her husband Dooja seeking divorce on the grounds of
cruelty, for not taking her out on vacations and mostly cancelling their movie dates. Dooja in
his response pleaded heavy work load in his office as a reason for cancellation of plans. Pooja
is adamant in taking her divorce petition ahead on grounds of cruelty. Decide. 

(a) Pooja will succeed as by cancelling plans every now and then Dooja caused mental
trauma to Pooja.

(b) Pooja will not succeed as Dooja‘s act doesn‘t qualify as harmful conduct done
intentionally.

(c) Pooja will succeed as she is denied her conjugal right of having company of her husband.

(d) Pooja will not succeed as cancelling plans is not enough and there should be more act to
constitute harassment.

  

(Passage Q.-Q.): The transgender person‘s bill 2019 defines a transgender person as one
whose gender does not match the gender assigned at birth. It includes trans-men and trans-
women, persons with intersex variations, gender-queers, and persons with socio-cultural
identities, such as kinnar and hijra. Intersex variations is defined to mean a person who at
birth shows variation in his or her primary sexual characteristics, external genitalia,
chromosomes, or hormones from the normative standard of male or female body. The bill
states that transgender persons will have the right to ―self-perceived‖ gender identity. But in
the same section, it adds that a transgender person must make an application to the district
magistrate to receive an identity certificate. The certificate ―shall confer rights and be a
proof of recognition of his [their] identity as a transgender person.‖ In case a person
undergoes a sex reassignment surgery, then they must apply for another certificate to change
their gender identity. The district magistrate will analyse the ―correctness‖ of the medical
certificate issued by the medical superintendent or chief medical officer, according to the bill.
Causing harm of any kind, including physical or sexual abuse, to a transgender person, could
attract punishment from six months and extend up to two years. The bill provides for the
formation of a National Council for Transgender Persons that will advise the government on
formulating policies for the community, and monitor the implementation, and address
grievances, among others. Besides this, the bill does not make any mention of reservation in
education and employment for the transgender community. The legislation also states that the
court can decide to place a transgender person in a rehabilitation centre if their families are
unable to take care of them. It also adds that the Centre will provide for healthcare services,
including sex reassignment surgery and hormonal therapy. The bill is, however, not specific
on whether these services would be free or subsidised by the government. It adds though that
there will be ―provision for coverage of medical expenses by a comprehensive insurance
scheme for sex reassignment surgery, hormonal therapy, laser therapy or any other health
issues of transgender persons.‖ (Extracted and edited from prsindia.org & Quartz India)

. The state of Avarumal rolled out a policy decision granting all transgender 25% reservation
in the state services examination. Avanti who was born as a girl always found it difficult to
subscribe to the standard gender roles. She soon realized that she was more of a man than a
woman. However, she was too afraid to disclose it to her parents and continued with her
hidden identity. She would dress up in her father‘s clothes when no one was home. Once, she
moved to her college she found the environment to be more accepting and eventually came
out. However, her identity remained hidden from her friends and family at home and she did
not get an identity certificate either. After her graduation she took the State service exams and
cleared the cut-off set for transgenders. However, the state refused to consider her eligible.

(a) The state has wronged the rights of Avanti

(b) Avanti is entitled to reservation

(c) Avanti is not entitled to reservation

(d) Both a and b

. Ligma was a 37-year-old transgender. He was actively employed as a labourer under a govt.
scheme. He used to face a lot of backlash and harassment in his village by his relatives and
others for his biological differences. This made him fall into severe depression over the
course of a few years. He was having suicidal thoughts and even tried to commit suicide once
by jumping off an under-construction building, however he survived. An NGO for mental
health wrote a letter to the government to provide him with therapy and medication if needed
for free. Decide whether: -
(a) The government is liable to provide him with all said services free of cost.

(b) He will first need an identity certificate issued by the district magistrate.

(c) The government is not liable to provide said services free of cost.

(d) Insufficient information in the passage.

. Nayantara was born as a female. At 19 years of age she got herself enrolled into a medical
university availing the reserved quota for females. The university provided for a separate
female quota both for entrance as well as placements. In the third year of her college, she
came out to the public as a queer person and underwent a sex[1]reassignment surgery, post
which she surgically got her genitals changed to that of a male‘s. She later took up the
pronouns they/them. Their certificate for identification as a male was however rejected by the
district magistrate on unspecified grounds. Nayantara nevertheless secured a job through the
reserved quota for females. Her colleagues challenged her appointment.

(a) Nayantara cannot avail the reserved quota for placement as she does not fall into the
category of a woman anymore.

(b) Her admission to the university happened as a female therefore regardless of her gender
transformation she can avail the services of reserved quota for female.

(c) Nayantara is entitled to the reservation in placements as a female.

(d) Both b and c

. Nayantara was born as a female. At 19 years of age she got herself enrolled into a medical
university availing the reserved quota for females. The university provided for a separate
female quota both for entrance as well as placements. In the third year of her college, she
came out to the public as a queer person and underwent a sex[1]reassignment surgery, post
which she surgically got her genitals changed to that of a male‘s. She later took up the
pronouns they/them. Their certificate for identification as a male was accepted by the District
magistrate. Their batchmates challenged her continuation as a university student as she was
now legally a male and had been admitted on the grounds of the women‘s quota.

(a) Nayantara‘s continuation with the university will be terminated.

(b) Nayantara is no longer a female as her identity certificate now calls her a male. Therefore,
she has to forego the benefit that she availed when she was a female.

(c) Her admission to the university cannot be challenged on the given grounds.

(d) Both a and b.

 
. Varamullupu was born as a trans-woman and she lived her life as a female and also
identified as one. One day in college she was subject to physical abuse for being a trans-
woman. She filed an offence against the perpetrators under the bill and requested for them to
be imprisoned for 2 years i.e. the maximum punishment under the said bill. Decide: -

(a) The perpetrators will be punished according to the provision under the bill.

(b) The perpetrators will be punished if Varamallupu presents an identity certificate issued by
the district magistrate.

(c) The perpetrators will not be punished.

(d) Varamallupu has to go under IPC for relief

13
Passage (Q.-Q.): Section 16 of ICA states that ‘a contract is said to be induced by undue
influence where the will of the party consenting is able to be dominated by the other one due
to the existence of the relation subsisting between them’. One party influences the other while
the contract is formed to get an unfair advantage over the other. It further qualifies that a
person is able to dominate the will of the other if he (1) has a real or apparent authority
arising out of fiduciary relations (relationship of trust) between them, or (2) forms the
contract with a person whose mental capacity, due to illness or age or due to mental distress
is temporarily or permanently affected. The burden of proof to prove that the contract was not
affected from undue influence lies with the defendant i.e. the person who was in a position to
dominate the will of the other. In addition, this section mentions that it will not affect Section
111 of the Evidence Act, 18 which talks about good faith in the transaction between the
parties. Every contract which involves undue influence comes under the ambit of the
principle of equity. The following elements are required to vitiate a contract for reasons of
undue influence. (1) Dominating Position: The existence of dominating position along with
its use is mandatory to invoke an action. If once dominance is established, unless any
contrary object appears, it is presumed that there was a use in the particular instance. (2)
Unfair Advantage: Unfair advantage is the advantage or enrichment which is obtained
through unrighteous or unjust means. It comes into existence when the bargain favours the
person who enjoys influence and which proves unfair to others. (3) Real and Apparent
Authority: In this type of influence, there is a real authority like a police officer or an
employer who uses his dominance for his enrichment. Apparent authority is pretending real
authority without its existence. (4) Fiduciary Relationship: This type of relationship is solely
based on the existence of trust between the parties for each other. It is such that one of the
parties naturally reposes its confidence in the other one and with an increase in that
confidence gradually, one party starts influencing the other. Under Section 19A of the
Contract Act, an agreement induced by undue influence is voidable at the option of that party
whose consent was taken by influencing him/her. Performance of such agreements may be
avoided absolutely or on prescribing certain terms and conditions.

. Rajesh is a school student. He often sneaks in and out his exam sheet to edit it and score
better marks. He has befriended a school peon for this task and pays him some money now
and then. The peon and Rajesh enter into a contract. Decide

(a) The contract is valid for the peon and Rajesh are not in a fiduciary relationship
(b) The contract is voidable for the peon has a dominating position over Rajesh

(c) The contract is valid for the peon and Rajesh are mere friends and undue influence cannot
occur

(d) The contract is voidable between Rajesh and the peon.

. Please refer to the facts above. It is stated that Rajesh is 15 years old and studies in the 4th
standard. The peon enters into a contract with Rajesh. Decide

(a) The contract is valid for the peon and Rajesh are not in a fiduciary relationship

(b) The contract is voidable for the peon commands influence over Rajesh

(c) The contract is valid for there is no presumption of the peon misusing his dominance over
Rajesh

(d) The contract is voidable for Rajesh’s mental capacity is distressed

. Patil is the owner of an engineering firm which often takes government contracts. He has a
few good connections with people in the department who give him information on the tender
amount. Patil now enters into a contract with one such employee. On whom does the burden
of proof of proving undue influence lie?

(a) The burden of proof lies on Patil since he still needs to continue his privilege of changing
grades in the marks sheet.

(b) The burden of proof lies on the employee since he is in a fiduciary relationship with Patil

(c) The burden of proof lies on Patil since the contract is voidable at his option.

(d) The burden of proof lies on the employee since he is the dominating party in this contract

. Please refer to the facts above. Assume that the employee has a dominating position over
Patil. Which of the following facts would rebut the presumption of the use of such
dominating position?

(a) The employee is poor and Patil only helped him out with the money he paid for the
employee’s services

(b) The employee during the time of signing the contract was not in school

(c) The employee had quit his position at the department prior to signing the contract
(d) The employee during the time of signing the contract was travelling to the north pole and
there was no communication whatsoever

. Rashomon had entered into an unfavourable contract and now does not want to abide by. He
speaks to his family lawyer. The lawyer tells him that filing a legal challenge would be
tedious and he should refer the same to mediation instead. The lawyer charged more for
mediation proceedings. However, mediation proceedings were also more effective. Decide

(a) Rashomon and the lawyer are in a fiduciary relationship. The lawyer abused his dominant
position for unjust enrichment

(b) Rashomon and the lawyer are in a fiduciary relationship. The lawyer did not abuse his
dominant position for unjust enrichment.

(c) Rashomon and the lawyer are not in a fiduciary relationship. The lawyer abused his
dominant position for unjust enrichment

(d) Rashomon and the lawyer are not in a fiduciary relationship. The lawyer did not abuse his
dominant position for unjust enrichment.

. Please refer to the facts above. The lawyer instead of a mediation settlement, suggested
filing a writ petition. The lawyer charged the most for arguing a writ petition. However, a
writ petition would be the most ineffective in this case.

(a) Rashomon and the lawyer are in a fiduciary relationship. The lawyer abused his dominant
position for unjust enrichment

(b) Rashomon and the lawyer are in a fiduciary relationship. The lawyer did not abuse his
dominant position for unjust enrichment.

(c) Rashomon and the lawyer are not in a fiduciary relationship. The lawyer abused his
dominant position for unjust enrichment

(d) Rashomon and the lawyer are not in a fiduciary relationship. The lawyer did not abuse his
dominant position for unjust enrichment.

Passage (Q.-Q.): In a reference dealing with the interpretation of Section B of the Evidence
Act, 18 that deals with admissibility of electronic records, the 3-judge bench of Supreme
Court has held that the certificate required under Section B(4) is a condition precedent to the
admissibility of evidence by way of electronic record, as correctly held in by the 3-judge
bench in Anvar P.V. v. P.K. Basheer, and incorrectly “clarified” by a division bench in
Shafhi Mohammad v. State of Himachal Pradesh. The Court further clarified that the required
certificate under Section B (4) is unnecessary if the original document itself is produced. The
Court after quoting Anvar P.V. v. P.K. Basheer, it was found that a Division Bench judgment
in Shafhi Mohammad v. State of Himachal Pradesh may need reconsideration by a Bench of
a larger strength. The Division bench, in the Shafhi Mohammad judgment, had “clarified”
that the requirement of a certificate under Section B(4), being procedural, can be relaxed by
the Court wherever the interest of justice so justifies, and one circumstance in which the
interest of justice so justifies would be where the electronic device is produced by a party
who is not in possession of such device, as a result of which such party would not be in a
position to secure the requisite certificate. The 3-judge bench in the present case, holding the
Shafhi Mohammad judgment to be incorrect said, “the major premise of Shafhi Mohammad
(supra) that such certificate cannot be secured by persons who are not in possession of an
electronic device is wholly incorrect. An application can always be made to a Judge for
production of such a certificate from the requisite person under Section B(4) in cases in
which such person refuses to give it.” In Anvar P.V. case it was held that: 19 of 36 The Court
also clarified the confusion in the Anvar P.V. Case and held that the last sentence in Anvar
P.V. case which reads as “…if an electronic record as such is used as primary evidence under
Section of the Evidence Act…” is to be read without the words “under Section of the
Evidence Act,…” It said: “The clarification referred to above is that the required certificate
under Section B(4) is unnecessary if the original document itself is produced. This can be
done by the owner of a laptop computer, computer tablet or even a mobile phone, by stepping
into the witness box and proving that the concerned device, on which the original information
is first stored, is owned and/or operated by him. In cases where the “computer” happens to be
a part of a “computer system” or “computer network” and it becomes impossible to
physically bring such system or network to the Court, then the only means of providing
information contained in such electronic record can be in accordance with Section B(1),
together with the requisite certificate under Section B(4).” [SOURCE (edited): Prachi
Bhardwaj, SC clarifies law on admissibility of electronic evidence without certificate under
Section B of Evidence Act, 18,
https://www.scconline.com/blog/post/2020/07/14/sc[1]clarifies-law-on-admissibility-of-
electronic-evidence-without-certificate-under-section-b-of-evidence-act[1]18/ ]

. In a trial producing certificate for secondary evidence becomes onerous. The device so
produced as evidence is in possession of a third-party. Assuming that the case of Shafhi
Mohammad v. State of Himachal Pradesh holds superior authority, decide the outcome.

(a) The evidence becomes inadmissible as a certificate is mandatory for secondary evidences
according to Section B(4).

(b) The evidence may or may not be admissible taking into consideration other facts of the
case.

(c) The judge must compel the third-party to produce certificate for the secondary evidence.

(d) The evidence is admissible according to the interpretation provided under the judgement
of Shafhi Mohammad v. State of Himachal Pradesh.

. In a case the issue considered by the court was whether the produced evidence is either
primary or secondary. Decide the conditions for admissibility of the evidence.
(a) If the evidence produced is clarified to be primary, producing a certificate for the
admissibility of the same is unnecessary.

(b) The evidence is admissible, irrespective of it being primary or secondary, as producing a


certificate is not mandatory as per the Shafhi Mohammad case.

(c) If the evidence produced is clarified to be secondary, producing a certificate for the
admissibility of the same is discretionary.

(d) The evidence is admissible only if a certificate is produced as per the latest clarification
by the Supreme Court

. The prosecution failed to obtain the certificate for the secondary evidence from the third-
party possessor. Discover alternate methods to ensure admissibility of the evidence.

(a) Investigate and produce wholly alternate evidence.

(b) File a case in the Supreme Court to amend the existing Indian Evidence Act, 18.

(c) Request the presiding judge to order production of certificate by the third-party.

(d) Produce the primary source of the secondary evidence as evidence.

. In a trial a USB containing videography of the crime scene was produced as evidence.
Determine the conditions necessary for admissibility of the evidence.

(A) It is primary evidence as the video directly contains the events of the crime.

(B) It is secondary evidence.

(i) A certificate must be provided for the evidence to be admissible.

(ii) Production of certificate is not necessary.

(a) (A) and (ii)

(b) (B) and (i)

(c) (B) and (ii)

(d) The evidence is inadmissible.

 
Passage (Q.-Q.): According to section 2(b) of the Contempt of Courts Act, 19 civil contempt
means willful disobedience to any judgement, decree, direction, order, writ or other process
of a court or wilful breach of an undertaking given to a court. Thus, from the abovementioned
definition it can be ascertained that there are two important essentials to constitute civil
contempt: Disobedience of any judgement, decree, direction, order, writ or other process of a
court or an undertaking given to the court. There should be disobedience of a valid order to
constitute contempt of court. An order includes all kinds of judgements, final, preliminary,
ex-parte and contempt order. Disobedience of a decree, direction, writ or other process of a
court, or an undertaking given to the court, will also amount to contempt. The basis behind
the breach of undertaking as contempt of court is that the contemnor obtains a beneficial
order for himself from the court, by giving an undertaking and if he fails to honor the
undertaking at a later stage, he plays a serious fraud on the court and thereby interferes with
the administration of justice. A willful breach of an undertaking would amount to contempt
of court. An undertaking can be given to the court in two ways: · By moving an application
or filing an affidavit before the court clearly stating the terms of the undertaking. · By giving
clear and express oral undertaking which is incorporated by the court in the order. The
Disobedience or breach must be willful, deliberate and intentional. Mere disobedience or
breach of the court’s order by the person is not sufficient to constitute civil contempt. Such a
disobedience or breach must be willful, deliberate and intentional. In order to exercise its
power to punish the contemnor the court has to be satisfied beyond reasonable doubt that
he/she has willfully, deliberately and intentionally violated the court’s order. No court
including contempt court is entitled to take trivialities and technicalities into account while
finding fault with the conduct of the person against whom contempt proceeding is taken.
Where the order has been substantially complied with and a reasonable explanation has been
provided for the delay in compliance with the order, the contempt will not lie as the violation
is not willful and deliberate.

. The High Court of Maharashtra summoned the defendant to attend the next trial. The
defendant failed to appear before the court, hence, the HC of Maharashtra passed an Ex-parte
judgement and notified the same to the defendant. The defendant failed to honor the
judgement as he claimed that he was not heard before the court. Decide.

(a) The defendant failing to appear before the court constitutes civil contempt of court

(b) The defendant failing to honor the ex-parte judgement constitutes civil contempt of court

(c) Both (a) and (b)

(d) Audi alterampartem principle is not followed by the court hence the judgement is invalid

. Rani school in the state of Haryana is a government school. It denies admission to girls
below the age of 12, also no such restriction is observed in the case of boys. A PIL is filed by
an NGO in the Supreme Court claiming that the school has violated the fundamental right.
The Supreme Court issues writ of mandamus to enforce the fundamental right. The school
does not follow the orders of the Supreme Court.
(a) The act of the school constitutes civil contempt of court

(b) Fundamental rights are not violated since only girls below the age of 12 are denied
admissions to the school, hence, the writ issued by SC is invalid

(c) NGO cannot file a PIL as it is not in any way involved in the issue

(d) As the school is a govt. school it has the discretion to choose to whether to follow or not
follow the decisions of the SC

. The High Court of West Bengal passes a judgement in a case ordering community service
along with a fine payment of Rs.00. The defendant due to health emergency could not
perform the community service for a period of time. Later, he complies with the judgement
and serves accordingly. Decide.

(a) Civil contempt of court can be initiated against the defendant as the community service
was delayed

(b) Contempt of court is not observed as the defendant did not willfully disobey the court
order

(c) Community service is a social service and no one is obligated to perform it

(d) Health emergency cannot be considered as a reasonable explanation

. Raja is accused of damaging the public property, i.e., cutting down of trees in an area
without prior permission from the govt. The court orders him to pay a compensation of
Rs.000. Raja is from a poor household and is unable to pay the fine, hence, he agrees to plant
trees in a year along with a fine of Rs.00. The court incorporates the oral undertaking given
by Raja in the order and thereby reduces the fine payment. Raja plants trees by the end of the
year, but fails to pay the fine. Decide.

(a) Raja is liable to pay Rs.000 as he failed to honor the undertaking

(b) The undertaking cannot be accepted as the defendant is benefitted by it and justice is not
served

(c) Planting trees and a fine payment of Rs.00 is not equivalent to a fine payment of Rs.000,
hence, the judgment is invalid

(d) Raja can be charged under civil contempt of court as he did not honor the undertaking as
well as the order passed based on the undertaking

Passage (Q.-Q.): Duress is a situation whereby a person performs an act as a result of


violence, threat or other pressure against the person. According to the Black’s Law
Dictionary, duress may be “any unlawful threat or coercion used… to induce another to act
[or not act] in a manner [they] otherwise would not [or would]”. Duress is the weapon with
which the common law protects the victim of improper pressure. The court intervenes where
a party enters into a contract as a result of pressure which the law regards as unacceptable.
There is a thin between acceptable and unacceptable pressure, which has been shifting over
time. The boundaries of what is considered unacceptable pressure have been pushed outwards
to encompass many more forms of pressure, including economic pressure. In the case of
economic duress, some judges are already adopting a restrictive approach, which makes it
more difficult for relief to be available on this ground. The conceptual framework for
allowing a duress defense generally stems from the laudable notion that one should not be
forced into contracting with another, but should come to the bargain voluntarily. Volition is
the touchstone of the freedom to contract. At common law, when an agreement is the product
of coercion and not entered into voluntarily, it was considered void ab initio. The nature of
the coercion that would support a defense was limited historically to threatened or actual
imprisonment or fear of loss of life and limb. As the law developed in the early part of the
last century, the threats that could qualify under the duress doctrine broadened in scope to
include threats to detain goods. In addition, courts began to find that threatened breaches of
contract resulting in irreparable harm constituted duress. Originally, the parameters of the
doctrine were very narrow in that an agreement could be avoided for duress only where the
duress was in the form of a threat to the person. Subsequently, it was accepted that duress of
goods can also vitiate consent to an agreement, and recent developments in respect of
economic duress show that the categories of duress should not be regarded as closed.

. X and Y are neighbors. X comes to know that Y had stolen the goods of Z. X tells Y to sell
him certain books at a price Rs.500 wherein the actual price of the books is Rs.5000. Y
refuses to this arrangement but X threatens to file a complaint for the theft due to which Y
agrees. Decide the validity of the contract.

(a) The contract is void ab initio as it was not voluntarily entered.

(b) The contract is not valid as the consideration has to be reasonable.

(c) The contract is valid as it was based initiating a lawful prosecution against the person.

(d) The contract is not valid and it amounted to economic duress.

. There was an agreement between Ghanshyam, a young song writer and FITZI Music
Company. The arrangement was for the period of five years and would automatically be
extended for another 5 years if the worth of Ghanshyam reaches to an amount of Rs 10,
00,000. However, the company could terminate the contract at any time with a month’s
notice. Ghanshyam did have that and therefore, he opted out of it. Decide.

(a) There was no pressure exercised but the terms were conditional.

(b) There was no pressure as Ghanshyam had the liberty to not enter into the agreement.

(c) There existed some pressure evident from the terms of the contract.
(d) There was no pressure as a reasonable time duration was offered before the termination of
the contract.

. Zen is a renowned doctor undertaking the treatment of Mona, an -year-old woman for a
period of 15 years. They develop a healthy relationship wherein Zen takes care of her as a
mother. Mona also considers Zen like a son. At an instance, Mona decides to gift a house to
Zen out of her property. Decide.

(a) There existed an element of duress as Zen is taking care of her for a long time puts him
into a dominant position.

(b) There existed no element of duress as this arrangement was out of will by Mona.

(c) There existed no element of duress as they developed a healthy relationship.

(d) There existed an element of duress as because Mona felt obligated to do this in exchange
of the care she received.

. Monica is an unemployed person and Zoto is the Chief Operating Officer in an organization.
Monica meets Zoto and gets her some expensive gifts to secure a position in the organization.
Decide whether there exists an element of duress in the existing situation.

(a) Yes, as it puts an obligation on Zoto to reciprocate.

(b) Yes, as it was intended to secure employment in the organization.

(c) No, as this gift has not impacted the arrangement.

(d) No, because Monica is not in a dominant position but Zoto.

. Harman is resident of New Delhi. Sharman lives in New York. Sharman threatens Harman
to enter into a contract with him otherwise he would kidnap his son and kill his wife. Due to
this, he enters into the contract. After some time, Harman denies fulfilling the obligations and
alleges that he was coerced. Decide.

(a) The claim is justified as he was threatened through an act against his family.

(b) The claim is not justified as it would have not been possible for Sharman to commit the
act.

(c) The claim is not justified as it gave enough access and time to Harman to approach for
protection before entering the contract.

(d) Both (b) and (c)


 

Passage (Q.-Q.): The Indian Penal Code provides some general defences under chapter four
that exonerate criminal liability which based on the premise that though the person
committed the offence, he cannot be held liable. This is because at the time of commission of
offence, person was justified of his/her acts, or there was absence of mensrea. However, it is
not all acts that are to be punished. There are certain defences provided under the ambit of
Indian Penal Code (IPC), 18 from Sections to . Mistake of fact arises when accused
misunderstood some fact that negates an element of crime. This legal weapon can be used,
where accused succeeds to prove that he/she was mistaken to the existence of some facts or
ignorant of the existence of such facts. It is a condition that such mistake must pertain to fact
not law. Section and of IPC contains the provision of mistake of fact. Such mistake must be
reasonable and must be of fact and not of law. The legal maxim,
"ignorantiafactiexcusatiignorantiajuris non excusat" which means ignorance of fact is an
excuse, but ignorance of law is no excuse. So it is a basic requirement to be get protected
under the sphere of this defence that mistake must be of fact. Section : Act done by a person
bound, or by mistake of fact believing himself bound, by law. Nothing is an offence which is
done by a person who is, or who by reason of a mistake of fact and not by reason of a mistake
of law in good faith believes himself to be, bound by law to do it. Section : Act done by a
person justified, or by mistake of fact believing himself justified, by law. Nothing is an
offence which is done by any person who is justified by law, or who by reason of a mistake
of fact and not by reason of a mistake of law in good faith, believes himself to be justified by
law, in doing it. Thus, the general defences enshrined under IPC are of paramount importance
in establishing the parameters of criminal offences. Criminal liability makes a person liable
for the acts which are prohibited by law. Indian Penal Code took cognizance of fact that all
acts are not to be punished. The acts devoid of mensrea are exempted from criminal liability.

. Before going to the Church, Zeeshan fired the gun and left it empty. However, during his
absence, Chester visits the house and takes the gun for shooting and while keeping it back
leaves it loaded. Later on, the same day, Zeeshan takes the gun and touches the trigger. The
gun goes off and kills his wife who was sitting in front of him. Decide.

(a) Zeeshan is guilty because he was negligent and that killed the wife.

(b) Zeeshan is not guilty as he was under a mistake of fact.

(c) Zeeshan is guilty and liability should be absolute.

(d) Zeeshan is guilty as such a weapon should be handled with great care.

. Chulbul is a renowned police officer who is ordered to catch a dangerous criminal named
Hari Pandey. After due enquiry in good faith, Chulbul, believing Prasoon to be Hari Pandey,
arrests him. Decide the liability of arresting a wrong individual and curtailing his
fundamental rights.

(a) Chulbul will be liable as this impacted the fundamental rights of Prasoon.
(b) Chulbul will be liable as his enquiry was deficient making an innocent suffer.

(c) Chulbul will not be liable as he was acting under the orders.

(d) Chulbul will not be liable since he acted in good faith.

. A complaint was made by a nurse engaged with a civil dispensary against the accused who
was the President of the Municipal office at that moment. He wrote a letter to the surgeon on
the basis of the allegations made by the renowned men of the community that the nurse had a
bad reputation for having illicit relations with a lot of men. The letter indicated that the
accused approved of such allegations on the nurse. However, there existed no proof that there
was an enquiry made on such facts. Decide.

(a) The President of the Municipal office is not liable as it could be mistake of fact.

(b) The President of the Municipal office is not liable as he did not say anything about the
nurse.

(c) The President of the Municipal office is not liable as he just the information further.

(d) The President of the Municipal office is liable as he failed to take due care.

. Seeta and Geeta had a strained relationship. At one instance, certain expensive utensils were
stolen from the house of Seeta so she thought of a plan to catch hold of the thief. She thought
that the thief would come to the only village pond to fetch water and at that moment she will
get hold of it. In pursuance to the same, she hid herself behind a tree and when she saw Geeta
filing water in the utensils gave a blow with a lathi on her head. Geeta sustained injuries and
filed a case against Seeta. Decide.

(a) Seeta is not liable as this would be considered a mistake of fact.

(b) Seeta is not liable as she was just trying to recover her stolen property.

(c) Seeta is liable as there was a complete lack of good faith.

(d) Seeta is liable as Geeta sustained injuries because of her blow.

. Jadegas& Sons is a renowned firm in the business of import and export. They have been
dealing with edible and non-edible oil since the last fifty years. On one day, they were
charged with the offence of importing non- edible coconuts oil under an open general license.
They pleaded that they believed it to be a non-canalized item and therefore acted in good
faith. Decide.

(a) They are not liable as the factual information suggest that they acted in good faith.
(b) They are liable as such act cannot be considered bona fide when they have been
functioning for such a long period.

(c) They are liable as the act was under mistake of law which is not excusable.

(d) They are not liable as they did not possess any dishonest intention.

Passage (Q.-Q.): Consumer rights are the rights given to a "consumer" to protect him/her
from being cheated by the salesman/manufacturer. Consumer protection laws are designed to
ensure fair trade competition and the free flow of truthful information in the marketplace. The
consumer is defined as someone who acquires goods or services for consideration for direct
use or ownership rather than for resale or use in production and manufacturing. To provide
simple, speedy and inexpensive redressal of consumer disputes, the CPA envisages 3-tier
quasi-judicial machinery at the National, State and District levels. National Consumer
Dispute Redressal Commission, known as National Commission, deals with complaints
involving costs and compensation higher than Rs. One Crore. State Consumer Dispute
Redressal Commission, known as State Commission, deals with complaints involving costs
and compensation higher than Rs. Twenty Lakh and less than Rs. One Crore. Section 11 of
the Consumer Protection Act states: "(2) A complaint shall be instituted in a District Forum
within the local limits of whose jurisdiction,"

(a) the opposite party or each of the opposite parties, where there are more than one, at the
time of the institution of the complaint, actually and voluntarily resides or 2 [carries on
business or has a branch office or] personally works for gain, or

(b) any of the opposite parties, where there are more than one, at the time of the institution of
the complaint, actually and voluntarily resides, or 3 [carries on business or has a branch
office], or personally works for gain, provided that in such case either the permission of the
District Forum is given, or the opposite parties who do not reside, or 4 [carry on business or
have a branch office], or personally work for gain, as the case may be, acquiesce in such
institution; or

(c) the cause of action, wholly or in part, arises.

. Mr A purchased Kgs of cotton as raw material to produce cloth in his factory. Later, he
finds out that the cotton was not the one he ordered and also was not fit for further
production. He files a complaint against the seller under consumer protection law. Is the
seller liable under the consumer protection act-?

(a) Yes, the seller was negligent in delivering the wrong order, not fit for production.

(b) No, the buyer should be aware and careful while making the purchase.

(c) Yes, the seller should be aware and protect the interest of the consumer by following fair
trade practices.

(d) No, Mr A is not a consumer and thus, the seller is not liable under the Consumer
Protection Act.
 

. In the above factual matrix, Mr A instead of using it for further production purchases it for
self[1]consumption. However, the cotton was of substandard quality and not fit for
consumption. He files a complaint against the seller under consumer protection law. Is the
seller liable under the consumer protection act-?

(a) Yes, the seller was negligent in delivering the wrong order, not fit for production.

(b) No, the buyer should beware and careful while making the purchase.

(c) Yes, the seller should beware and protect the interest of the consumer by following fair
trade practices.

(d) No, Mr A is not a consumer and thus, the seller is not liable under the Consumer
Protection Act.

. Mr B purchases a premium limited edition Roli watch for a price of Rs. 18 Lakhs from Mr
C for his wedding. Later he finds out that the watch was a copy of the original watch. He fills
a complaint against C for selling duplicate products and using unfair trade practices. He
claims compensation for Rs 22 lakhs which includes the price of the watch and compensation
for mental agony and all the costs incurred. Which among the following is correct-?

(a) Mr B should file the complaint in the State Commission as the claim is Rs 22 lakhs.

(b) Mr B should file the complaint in the District Forum as the cost of the watch is Rs 18
lakhs.

(c) Mr B is not a consumer and thus, can’t file a complaint.

(d) Mr B can claim only Rs. 18 lakhs.

. Rhea was a shopkeeper and was selling a sample face cream GLOW UP for free on her
shop. In her shop, she used to sell goods manufactured at her own small factory. Meghana
went to buy some products from Rhea’s shop and also takes that face cream. After applying it
for one week, she has a skin reaction and boils all over her faces. She sees a doctor and finds
out that the ingredients used to make the face cream her of substandard quality. Also, two
ingredients were of opposing nature and when used together produce a harmful effect. Is
Rhea liable under the Consumer Protection Act-

(a) Yes, Rhea was careless and negligent to use such ingredients and thus, should be made
liable.

(b) No, Meghana should have known the contents of the product and then used it.

(c) Yes, Rhea was using substandard and harmful ingredients has violated the consumer
rights of Meghana and thus, should be made liable.
(d) No, Meghana is not a consumer and thus, Rhea is not liable under COPRA.

. Shantanu lives in Bhopal and during his visit to Delhi buys a music system from Prajjawal.
Prajjawal lives in Kolkata but has a branch office in Delhi and gives with a warranty of 5
years on the product. On returning to Bhopal, the Speakers becomes dysfunctional and he
asks Prajjawal to repair them. He refuses to repair or exchange them citing his place of
business in Mumbai and only goods sold in Mumbai have a warranty. Now, Shantanu files a
complaint against him in the Consumer Forum. In which city can Shantanu file a complaint
as per COPRA[1]

I. Bhopal II. Delhi III. Mumbai IV. Kolkata

(a) All of the above

(b) I, II, III

(c) II, III, IV

(d) I, IV, II

. Who among the following is a consumer-?

(a) Patient getting free treatment at a Doctor’s camp

(b) A person buys a bike and resells it.

(c) A person buying a cinema ticket of the movie and makes the payment online

(d) None of the above

Passage (Q.-Q.): Malicious prosecution is the malicious institution of unsuccessful criminal


or civil proceedings against another without reasonable or probable cause. This tort balances
competing principles, namely freedom that every person should have in bringing criminals to
justice and the need for restraining false accusations against innocent persons. Malicious
prosecution is an abuse of the process of the court by wrongfully setting the law in motion on
a criminal or civil charge. The foundation lies in the abuse of the court process by wrongfully
setting the law in motion and it is designed to encourage the perversion of the machinery of
justice for a proper cause. The tort of malicious position provides redress for those who are
prosecuted without cause and with malice. In order to succeed the plaintiff must prove that
there was a prosecution without reasonable and just cause, initiated by malice and the case
was resolved in the plaintiff’s favour. It is necessary to prove that damage was suffered as a
result of the prosecution. In an action of malicious prosecution, the plaintiff must prove: ·
That he was prosecuted by the defendant. · That the proceeding complained was terminated
in favour of the present plaintiff. The claimant must show that the prosecution ended in his
favour, but so long as it did, it is of no importance how this came about, · That the
prosecution was instituted against without any just or reasonable cause. · That the prosecution
was instituted with a malicious intention, that is, not with the mere intention of getting the
law into effect, but with an intention, which was wrongful. · That he suffered damage to his
reputation or to the safety of the person, or to security of his property. The action for
malicious prosecution being an action on the case it is essential for the claimant to prove
damage.

. Polla files a case of defamation against dots. The Court issues a notice against dots and
proceedings for the case starts against him. Later, the case was dismissed due to lack of
proper evidence. Can Dots sue Polla for malicious prosecution-

(a) Yes, as all the elements for the institution of malicious prosecution have been satisfied.

(b) No, issuing of notice does not amount to prosecution

(c) No, there was no damage caused to Dots.

(d) Data insufficient.

. Taimur committed theft at the house of Saif stealing all the precious jewellery from his
home. Later, Saif files a case against Kareena, his neighbour for theft. He knows she is
innocent but nevertheless files a case against her to take revenge. The Court finds Kareena
guilty of committing theft and sentences her to 5 years of prison. Taimur, who is Kareena’s
son, feels miserable and after 5 months confesses his crime and Kareena is released. Can
Kareen file a case for Malicious Prosecution against Saif-

(a) Yes, all the essentials have been satisfied.

(b) No, the suit for malicious prosecution cannot be filed after the person is convicted.

(c) Yes, a mother should not be held liable and pay for her child’s crime.

(d) No, the prosecution did not result in favour of Kareena as now her son will be convicted.

. A murder of a boy was committed at a Supermarket and Abhirup the father and also the sole
witness. He saw the crime and the murderer’s face, who was Aarohi. Later, during
investigation he tells that Sarthak committed the murder just to take revenge from Sarthak,
his business revival and teach him a lesson. The case reached court and during the
investigation it was found that both Aaarohi and Sarthak conspired to commit the murder.
They both were convicted of committing the murder. Can Sarthak file a suit of malicious
prosecution against Abhirup-

(a) Yes, all the elements are satisfied.

(b) No, all the elements are not satisfied.

(c) Yes, Abhirup held Sarthak liable with malice and had no reasonable cause.

(d) No, the essentials are not satisfied.


 

. Jisoo filed a FIR against YG for fraud. Later during the investigation, due to lack of
evidence, the FIR was dismissed. Can YG file a case for malicious prosecution against Jisoo-

(a) Yes, all the essentials are satisfied.

(b) No, filing of FIR does not amount to prosecution.

(c) Yes, it is important to punish people for filing frivolous complaints.

(d) Data insufficient.

. Henry is a petty thief who commits theft on a small scale and never gets caught due to lack
of evidence. Once he committed a thief of Rs 1 crore from a bank, now he had enough case
and decided not to steal ever in life. The next day another thief was committed and Henry
was spotted near the crime scene 5 minutes before the thief. Thinking that he has committed
the crime he was arrested and a case was filed against him. In the court proceeding, it was
discovered that this time it was someone else and Henry was acquitted. Can Henry sue for
Malicious Prosecution-

(a) Yes, all the elements are satisfied.

(b) No, he was not convicted was this crime but will be convicted for the theft of Rs 1 crore.

(c) Yes, if he is not allowed to sue now, he may return to being a petty thief again.

(d) No, all the elements are not satisfied.

. Which among the following is not essential for malicious prosecution-

(a) The case must result in favour of the plaintiff.

(b) Some monetary loss must be caused to the plaintiff.

(c) The prosecution was instituted with malice

(d) The prosecution was instituted without a reasonable cause.

14
Passage (Q.-Q.): A contract is an agreement enforceable by law. [Section 2(h) of The Indian
Contract Act, 18]. For every contract, there should be an agreement that is made by the free
consent of parties competent to contract, for a lawful consideration and with a lawful object.
The agreement should not be declared void hereby to form a contract. This definition of
contracts as per Indian Contract Act, 18 is based on Sir Pollock’s definition which states that
every agreement and promise enforceable at law is a contract. Thus for the formation of a
contract, there must be an agreement and something in addition to that, i.e., an agreement,
and its enforceability at law. The word contingent ordinarily means ‘subject to chance’. In the
Indian Contract Act, 18, this word has been used to mean conditional, just the way we use it
generally. Uncertainty is the hall-mark of the future. Estimating the chances of an uncertainty
becoming certain, calculating the results if the event doesn’t happen and then measuring the
potentiality to deal with its consequences are all about contingent contracts. Parties may
stipulate that performance of obligations under a contract is dependent on a contingency,
even though the contract is validly formed. The parties agreeing to the conditions agree that
the rights will be enforced and the obligations will be due on the happening of the
contingency on the contracting of a valid contract. Section 31 to 36 of The Indian Contract
Act, 18 deal with this type of contract. Section 31 of the Act defines ‘contingent contract’
thus: A contingent contract is a contract to do or not to do something, if some event,
collateral to such contract does or does not happen. Every contingent contract is thus a
contract primarily. Like any other contract, it is also a contract to do or not to do something. It
is not, however, an absolute and unconditional one, without any reservations or conditions,
which is to be performed under any event. Its performance is dependent on some event’s
happening or not happening- the contingency. [Yashu Bansal, Contingent Contracts
https://www.lawctopus.com/academike/contingent-contracts/]

. ABC is a renowned insurance company. Y approaches the company for fire insurance
policy for his godown. The policy states that an amount of Rs 50,000 shall be provided in
case the godown is burnt. Decide whether this would be a contingent contract.

(a) The contract is contingent in nature as there is an element of uncertainty of the event.

(b) The contract is not contingent but compensatory in nature.

(c) The contract is not contingent but holds a guarantee element.

(d) The contract is not contingent as the terms are uncertain to be called valid.

. Dhara is a bookmaker and promoter who takes interest in the new entrants in the Derby.
She comes to know about the bank loan of Geeti and her default. She suggests her to put
some amount on the racing business in the adjacent turfs. Further, Dhara tells Geeti that if
her horse wins then she will buy her horse and pay out the loan. However, in case she loses
then Dhara will forfeit the horse. Geeti’s horse wins and now she is claiming the amount.
Decide.

(a) The claim is justified as it was a contingent contract.

(b) The claim is not justified as Dhara did not intend to create a legal obligation.

(c) The claim is not justified as it is not legally enforceable.

(d) The claim is justified as the agreement was entered into by the free consent of the
parties.

. Badri is a renowned businessman who promises to give a huge donation of Rs 20lakh to


Sumit for the construction of a library. Based on this, Sumit took a loan and initiated the
construction work. Badri was unaware of the progress so undertaken. Now, Badri is reluctant
to make the payment. Decide the validity of the contract.

(a) The contract is not valid as it lacked consideration.


(b) The contract is not valid as the promise has no value in the eyes of law.

(c) The contract is valid as the promise was based on consideration.

(d) The contract is valid as it was for a lawful object.

. Free consent cannot be obtained when it is obtained by means of:

(a) Mistake of law

(b) Unilateral Mistake in an agreement

(c) Misrepresentation

(d) Both (a) and (b)

. EasyPay Bank lends an amount of Rs. 50,000 to Balwant to be awarded as a prize to


Nisha for winning the horse race. Later, Bhanu refuses to pay claiming it to be based on
wager and betting therefore void in its very nature. EasyPay Bank decides to claim the
amount. Decide.

(a) The claim will not succeed as it is not a contingent contract but wager.

(b) The claim will not succeed as the interest of associated parties is limited to monetary
aspects.

(c) The claim will succeed as it is a collateral contract.

(d) The claim will succeed as the arrangement is between EasyPay and Balwant.

Passage (Q.-Q.): A petition has been filed in the Supreme Court challenging the
constitutionality of several provisions of the Special Marriage Act, 14. The provisions of the
Act which call for making public the personal details of the couple intending to marry
seriously damage one’s right to have control over personal information, the plea contends.
Section 5 of the Act requires that a notice of intended marriage is given by the parties to the
Marriage Officer of the district where at least one of the parties to the marriage has resided
for a period of not less than thirty days immediately preceding the date on which such notice
is given. Section 6 mandates that all such notices received shall be entered in the marriage
notice book and the Marriage Officer shall publish a notice by affixing a copy thereof. The
provisions also allow anyone to submit objections to the marriage and empower the
Marriage Officer to inquire into such objections. The requirement of notice before marriage is
absent in the Hindu Marriage Act, 15, and Customary laws in Islam. Therefore, the said
provision is also discriminatory and violative of Article 14 of the Constitution of India. It states
that the impugned provisions, by throwing the personal information of the individuals open to
public scrutiny, seriously damage one’s right to have control over her or his personal
information and its accessibility. [Source: Chilling effect on the Right to Marry: Petition filed in
SC challenging provisions of Special Marriage Act on disclosure of details, Debayan Roy,
Sept 3, 2020, https://www.barandbench.com/news/litigation/petition-supreme-court-
challenging-provisions-of[1]special-marriage-act]
 

. Praveen and Shilpa want to elope from their houses to another city and decide to get
married by the registrar as they both belong to different religions. They reached the
registrar’s office only to be denied certification. Decide.

(a) They could not because there is objection against their marriage.

(b) They could not as neither of them is a residents of the city.

(c) They could not as Special Marriage Act, is just court marriage, not for eloping couples.

(d) They can as they fulfill all requirements about the Special Marriage Act.

. Mary and John are practicing Christians and want to get married by a court official.
However when informed for a period to file a notice seeking any objection they found it to
disagreeable. Christianity however has a provision for asking for objections before
confirming the matrimony. Can they file a petition against it?

(a) No they cannot as it would be violative of SMA.

(b) No they cannot as their religion permits the validity of seeking objection.

(c) Yes they cannot as their religion is unlike Hinduism and Islam allows for such rituals.

(d) Yes they can as it’s a violation of fundamental rights.

. Parveez and Meena’s union as couple was frowned upon in their respective communities
because they belonged to the different religions. From the above para select the argument
that dissuades their publication of information under section 6.

(a) It is against both their religious belief to have such publication.

(b) It is against their privacy to have such public scrutiny of their relationship.

(c) It is unlawful to publish such statements as under other acts there is no such
requirement.

(d) None of the above can be used to dissuade such publication.

. Article 21 enshrines the fundamental right to life and personal liberty, the above petition
states that these provisions are in violation of article 14. Select the arguments that could
help them favor Article 21 from the given options.

(a) The right to live with dignity as without public scrutiny of your choices.

(b) The right to exercise, freedom of religious belief and faith.

(c) The right to secure your privacy and information from public eye.
(d) All of the above.

. Rahul and Meena are in a relationship and they are both Hindus, but want to register their
marriage under Special Marriage Act, hence they submit a notice, fulfill the perquisite of
Section 5. However under the public notice, certain objections are bought forth to their union,
based on the law who has the adjudication power to confirm the same?

(a) The parties of the union, to justify their stance.

(b) The marriage officer after through inquiry into the matter.

(c) The courts, after the verdict of the petition filed.

(d) The parties who raised such objection.

Passage (Q.-Q.): Plea Bargaining, in the most traditional and general sense, refers to a pre-
trial negotiation between the defendant, who is represented by a counsel and the
prosecution, during which the defendant agrees to plead guilty in exchange for certain
concessions by the prosecutor. Plea Bargaining is further divided into two categories –
"charge bargaining", where the negotiation is to reduce or dismiss some of the charges
brought against the defendant in exchange for a guilty plea and "sentence bargaining",
where the prosecutor promises to recommend a specific sentence or refrain from making
any sentence recommendation defendant in exchange for a guilty plea. Despite its
resounding success in the countries like the United States of America, there were divided
opinions with respect to introducing Plea Bargaining in the Indian criminal justice system. In
this article, we will examine all the concerns raised at various public institutions in India with
respect to introducing Plea Bargaining in India so that we can proceed, very cautiously to
dovetail interests of the society and met demands of the justice system. The Indian
Parliament on Plea Bargaining Plea Bargaining was included in the Code of Criminal
Procedure (Cr.P.C) under chapter 21A from Section 2A to Section 2L through the Criminal
Law (Amendment) Act, 2005 (2 of 2006) on with effect from 5th July, 2006 as a prescription
to the problem of overcrowded jails, overburdened courts and abnormal delays. Now, more
than ever, India is at crossroads. With innovative investigative tools and the use of
technology in pinning down criminal involvement, plea bargaining has a very important role
to play in the dispensing of justice- both to the victim and the accused. Its benefits are
certainly many. Nonetheless, we need a set of clear and decisive guidelines taking into
account the concerns vis-à-vis plea bargaining. It has to be a fair deal- for all. Following are
the silent features of Plea Bargaining; 1. It is applicable in respect of those offences for
which punishment is up to a period of 7 years. 2. It does not apply to cases where offence is
committed against a woman or a child below the age of 14 years 3. When court passes an
order in the case of plea bargaining no appeal shall lie to any court against that order. 4. It
reduces the charge. 5. It drops multiple counts and press only one charge. 6. It makes
recommendation to the courts about punishment or sentence. 20 of 36 Plea Bargaining can
be described as a process whereby the accused may bargain with the prosecution for a
lesser punishment. In simple words, Plea Bargaining is an agreement (contract) between the
accused and the prosecution regarding disposition of the criminal charge leveled by the
prosecution against the accused. In layman’s language, it is bargaining done by the accused
of a serious and severe offence, with the authority for a lighter punishment in lieu of a full-
fledged trial. [Source: (edited): https://www.livelaw.in/know-the-law/plea-bargaining-in-india-
15]
. A man Mr. X is accused of killing his wife. While there is no concrete evidence put up
against him yet, the counsel of Mr. X asks him to admit to the guilt and get a lesser sentence
in case of a trial, as the punishment for it could lead to life imprisonment. Should Mr. X
agree?

(a) No, as there is no concrete evidence Mr. X would not be penalized in the first place.

(b) Yes, as this could lead to serve a shorter sentence.

(c) No, as this deal would not be valid in the first place.

(d) Yes, as Mr. X should heed to his counsel’s advice as he has better legal knowledge than
him.

. A man Mr. Z agrees to plea bargain with the prosecution and lessened his sentence from 7
to 4 years. Later he realized that the police did not garner enough evidence against him. Can
he revoke his deal? (a) No, as a plea bargained for cannot be revoked.

(b) No, as a plea bargained for cannot be appealed.

(c) Yes, as he was never going to be convicted in the first instance.

(d) Yes, as he was not going to be released due to the lack of evidence.

. Raghav a 17 year old boy committed an offence which had a sentence for 6 years as
punishment under the Indian Penal Code. Can he plead guilty and ask for a plea bargaining
deal?

(a) No, he cannot as he is a minor.

(b) Yes, he can as minors and children under 14 cannot be tired for offences.

(c) Yes, he can, as it’s not an offence under the Indian Penal Code.

(d) Yes he can as the offence he committed was for under 6 years of punishment.

. Shikha committed an offence under Narcotics and Drugs Act of India and was charged for
a probable punishment of 14 years. Can she ask for a Plea Bargaining Deal?

(a) No she cannot as offences for under 14 years cannot fall under the law.

(b) No, as her offence is not criminalized under Indian Penal Code.

(c) Yes, as her offence is not criminalized under Indian Penal Code.

(d) No, as the offence has punishment for over 7 years.

 
. In a situation where a person is charged with more than one offence, wherein some have a
punishment less than 7 years but some more than 7 will he be eligible for Plea Bargaining?

(a) No, as to get the effect of plea bargaining the offences have to be less than 7 years.

(b) Yes, as plea bargaining can help him drop multiple charges for just one charge

(c) Yes, as it could reduce the charges for his other offences that could lead to imprisonment
over 7 years.

(d) No, as this is a provision only for the case of a singular offence, not multiple offences.

. Person H committed an offence with a punishment of no less than 3 years of imprisonment.


He was put on trial without being told of the option of plea bargaining. Can he invoke this
right mid trial?

(a) Yes, as plea bargaining is tool to replace the necessity of going through a full-fledged
trial.

(b) No, as once the order is given you cannot plea or revoke the plea bargaining agreement.

(c) Yes, as he was not aware of his rights hence his counsel should be blamed for it not the
accused.

(d) Can’t say based on the above information.

Passage (Q.-Q.): “A new law ought to be prospective, not retrospective in its operation.”
Ordinarily, a legislature has power to make prospective laws, but Art.20 of the Indian
Constitution, 10 provides certain safeguards to the persons accused of crime and so Art.
20(1) of the Indian constitution imposes a limitation on the law making power of the
constitution. It prohibits the legislature to make retrospective criminal laws however it does
not prohibit a civil liability retrospectively i.e. with effect from a past date. So a tax can be
imposed retrospectively. Clause (1) of the Article 20 of the Indian Constitution guarantees
rights against ex-post facto laws. It provides that “ no person shall be convicted of any
offence except for violation of a law in force at the time of the commission of the act charged
as an offence, nor be subjected to a penalty greater than that which might have been
inflicted under the law in force at the time of the commission of the offence.” The
retrospective operation of an enactment may mean one thing and its affecting the rights of
parties another. Normally, an enactment is prospective in nature. It does not affect that
which has gone, or completed and closed up already. Ordinarily, the presumption with
respect to an enactment is that, unless there is something in it to show that it means
otherwise, it deals with future contingencies, and does not annul or affect existing rights and
liabilities or vested rights, or obligations already acquired under some provisions of law
although its effect is that it does not affect an existing right as well. If an enactment expressly
provides that it should be deemed to have come into effect from a past date, it is
retrospective in nature. It then operates to affect existing rights and obligations, and is
construed to take away, impair or curtail, a vested right which had been acquired under
some existing law. If an enactment is intended to be retrospective in operation, and also in
effect, the legislature must expressly, and in clear and unequivocal language, say so, in the
enactment itself. A retrospective operation is not given to a statute, so as to impair an
existing right or obligation, otherwise than as regards matters of procedure unless that effect
cannot be avoided without doing violence to the language of the enactment. If the enactment
is expressed in a language which is capable of either interpretation, it ought to be construed
prospectively. [Source: http://www.legalservicesindia.com/article/517/Prospective-Vs.-
Retrospective.html]

. A law was passed criminalizing begging on the signals. A lot of people previously known to
have been doing this were rounded by the police and put under custody. Were they right in
doing so?

(a) No, as this is against the criminal procedure code.

(b) No as this is against the Article 20 of the Indian Constitution.

(c) No, as this is against the rules of retrospective laws.

(d) No, as this is against the rules of arrest and evidence.

. Section 234D of the Act was introduced by the Finance Act, 2003 with effect from June 1,
2003, wherein an assessed getting a refund under the provisional assessment and liable to
pay up the same on the regular assessment is liable to return the same with interest at 18
per cent per annum from the date of the refund to the date of the regular assessment. Will
this law be applied prospectively? (a) No, as taxation laws can be read and applied
retrospectively. (b) Yes, as it does not mention any explicit application of retrospective
application. (c) Yes, as it mentions the date from which it would have effect. (d) No, as since
it talks about all refunds hence it could be operated even retrospectively. . A law was brought
into force claiming that any person engaged in the activity of importing coffee beans from
certain nations listed by the government would be penalized under the Custom Duty Act with
hefty fines. Would this law have a retrospective or prospective effect? (a) It would be
retrospective as it’s a civil law which imposes fines. (b) It would be prospective as its Article
20 state all laws have to be prospective in nature. (c) It would be retrospective as the act
does not explicitly mention if it would have a prospective operation. (d) It would be
prospective as it’s a penal offence and it cannot have retrospective operation unless
explicitly mentioned.

. Supreme Court of India Decriminalized Sec 3 of the IPC which made anyone participating
in homosexuality an offence as it was against the fundamental rights of the land. The
legislation passed a law, explicitly stating that 3 would still be intact and functioning,
retrospectively to the judgment of the Supreme Court. Is this a valid law?

(a) It would be valid as the law mentions that it has a retrospective operation till the time the
judgment was declared.

(b) It would be unconstitutional as they cannot bring a law retrospectively to overhaul a


fundamental right.
(c) It would be invalid as criminal laws cannot be retrospective in nature and Art 20 states
that in case of penalty it has to be according to the current interpretation only. (d) It would be
valid as the legislature has the power to make anything an offence and penalize it.

. An Emergency was declared in the State of Madhya Pradesh, which suspended the law
securing right of publishers to be void of censorship regime. A journalist was stripped of his
license to print his articles during this time as it was against this fundamental right to speech
and expression. Will his suit sustain?

(a) No, it wouldn’t as a person can be penalized for his action with any law in force at the
time

(b) Yes, as it was against his fundamental right to speech and expression.

(c) No, as there is no retrospective saving clause for prior to emergency period and he would
be penalized according to the current law.

(d) Yes, as even during emergency the media and newspapers run full time eg. COVID
crisis.

. In the new budget, the tax slab of payment was altered to benefit the tax payer, it increased
the limit of exemption from payment of tax to 5,00,000 p.a. instead of 2,50,000 p.a. The
taxation authorities deemed it to be those taxpayers who started earning post this
amendment, and taxpayers deemed it to everyone post this amendment. Whose
interpretation would you follow?

(a) The taxpayers as there is no explicit mention of retrospective operation.

(b) The taxpayers as there is no explicit mention of excluding any earners.

(c) The tax authorities as taxation statutes have retrospective operation.

(d) The tax authorities as they are the final assessors of the taxation amounts.

Passage (Q.-Q.): There is an inflamed controversy as to when judges should recuse


themselves from hearing a case. By now this should be reasonably obvious. But the
controversy has been brought to the fore by unhappiness that Justice Arun Mishra refused to
recuse himself in a case on which he had admittedly strong views stated in a Supreme Court
judgment on the subject of the manner of payment of compensation to be awarded in certain
classes of cases under the old Land Acquisition Act 14. It would be difficult to sustain a
broad proposition that judges with strong views, or for that matter strong views on a
particular area, should never hear cases on which they have strong views. Can lawyers
object to judges hearing a matter? Usually not. But lawyers do tell judges to consider
whether there is a conflict of interest or because there are other reasons for the judge or
bench not to hear the matter. Indeed, some judges recuse themselves if they feel there is a
case for doing so. In the Babri first appeal case, Justice U.U. Lalit was reminded that he had
appeared in an earlier criminal case concerning Babri as evidenced by the orders showing
his appearance in the criminal case. On bringing this to the judge’s notice without any
allegation of bias, Justice Lalit decided to recuse himself as a matter of propriety and within
what I call the 1% rule. I think that it is the duty of lawyers to give voice to their bonafide
apprehensions. The emphasis is on the word ‘bonafide’ not on unscrupulous forum
shopping. There is one other matter that needs discussion. Sometimes, judges take the
unusual stance which goes something like this: “I know that I should not hear this case, but if
you lawyers agree, I will proceed to hear it and pass judgment”. With this, the onus passes
on the lawyer to agree to an impropriety. At this juncture, what is the lawyer to say? For
many reasons, lawyers agree to the judge hearing the case because they do not want to
attack the judge for unfairness, or prejudice their client’s case, or anger the judge in respect
of future appearances before him or her. For the present, it needs to be said that the lawyers
were right to express their apprehensions about Justice Mishra’s inclusion in the bench
hearing the land acquisition case. Then-Chief Justice Gogoi could have resolved the
problem but did not. Justice Arun Mishra’s long judgment in his own case begs precisely the
question of propriety which lawyers placed before him in a bench that was not fairly
constituted. Eventually, even his colleagues’ defence of his stance was half-hearted,
guarded and contradictory. Justice was ill served by the judges closing ranks to defend a
colleague. [Extracted from ‘Judges and Recusal: The Issue Is Propriety, Not Bias’, Editorial
at The Wire.]

. Which of the following is the correct argument of the author as per the above passage?

(a) The question of propriety or impropriety is for the lawyers to decide who appear before
an apparently unfairly constituted bench.

(b) The lawyers did not want Justice Arun Mishra to preside over the matter as they
apprehended unfairness, or prejudice towards their client’s case, or anger the judge in
respect of future appearances before him or her.

(c) The decision of Justice Arun Mishra to preside over his own judgment and not recuse
from the matter places a question mark on the accepted practice in the judiciary.

(d) That is because judges with strong views, or for that matter strong views on a particular
area, should never hear cases on which they have strong views.

. What is the major reason behind the demand for Justice Arun Mishra’s recusal from
hearing the case?

(a) His prior involvement in the case gives rise to the issue of conflict of interest and an
element on bias.

(b) His unhappiness and unwillingness to recuse himself from hearing the Land Acquisition
matter.

(c) His strong views on the matter, and thus should never hear cases on which they have
strong views.

(d) The provisions of the 1% rule due to which Justice Lalit recused himself from the Babri
Masjid Case.

. Why did Justice U.U. Lalit recuse himself from the BabriMashid case appeal?
(a) Because of the 1% rule relating to the recusal of judges established by commonly
accepted practice.

(b) Because of his strong views on the matter which were expressed in the earlier judgment.

(c) Because he did not want the lawyers to point out the chances of want unfairness, or
prejudice their client’s case.

(d) Because he was made aware of him involvement in an earlier criminal case concerning
Babri matter.

. In the case where there is an apparent conflict of interest or reason of bias due to the
involvement of a judge, what is the duty of the lawyers?

(a) To clearly allege the said judge for unfairness and prejudice in his client’s case
irrespective of his future appearances before him or her.

(b) To not say anything in this regard and let the judges decide the fate of their involvement
in the said matter.

(c) To assess the propriety in the judge’s involvement and accordingly allow him to proceed
with the matter and pass the judgment.

(d) To give voice to their bonafide apprehensions relating to the judge’s involvement and not
unscrupulously.

. What does the author feels the lawyer does when the onus of impropriety in shifted on
them by the judges.

(a) The lawyers will surely raise their voice if they have bonafide apprehensions regarding
the involvement of the judge in the given case.

(b) The lawyers allow the judge to hear the matter as they do not want to attack the judge for
unfairness, or prejudice their client’s case, or anger the judge in respect of future
appearances before him or her.

(c) The lawyers do not allow the judge to hear the matter upholding the commonly
established practice in the judiciary.

(d) The lawyers should precisely beg the question of propriety before the bench that was not

(Q.-Q.): A decree of restitution of conjugal rights implies that the guilty party is ordered to live
with the aggrieved party. Restitution of conjugal rights is the only remedy which could be
used by the deserted spouse against the other. A husband or wife can file a petition for
restoration of their rights to cohabit with the other spouse. But the execution of the decree of
restitution of conjugal rights is very difficult. The court though is competent to pass a decree
of restitution of conjugal rights, but it is powerless to have its specific performance by any
law. The non-compliance of the issued decree results to constructive destruction on the part
of the erring spouse. At present as per the provisions available under the Indian personal
laws, the aggrieved party move a petition for a decree of divorce after one year from the date
of the passing of the decree and the competent court can pass a decree of divorce in favour
of the aggrieved party. The decree of restitution of conjugal rights can be enforced by the
attachment of property, and if the party complained against still does not comply, the Court
may also punish him or her for contempt of court. But under no circumstances the court can
force the erring spouse to consummate marriage. Decree of restitution of conjugal rights
could be passed in case of valid marriages only. Under all personal law, the requirements of
the provision of restitution of conjugal rights are the following: a. The withdrawal by the
respondent from the society of the petitioner. b. The withdrawal is without any reasonable
cause or excuse or lawful ground. c. There should be no other legal ground for refusal of the
relief. d. The court should be satisfied about the truth of the statement made in the petition
by the petitioner (Source: Indian National Bar association) Based on the understanding of
the passage and assuming that everything mentioned in the questions is true, answer the
following: (The first three questions are in continuation of the same factual matrix)

. Atul was an abusive husband. He would come late to home at nights, heavily drunk and
force himself on his wife Mridula. Mridula had been facing the atrocities of her marital home
for quite some time now. However, she would endure everything without any complaints as
she believed that her husband loved her nevertheless. On March 2019, Mridula caught her
husband cheating on her. There was a violent altercation between that ended in Atul
slapping Mridula and storming out of the house. Months passed and there was no
communication from Atul. He had completely cut off his ties from Mridula and had started
living with his mistress. Mridula tried contacting him but every time she would be rebuffed.
She filed for a petition of restitution of conjugal rights.

(a) Her petition would fail as it a case of domestic violence

(b) The withdrawal by her husband is justified as there was clearly a matrimonial discord
between them.

(c) Her petition will sustain as her spouse has withdrawn from the society without any
reasonable cause or lawful excuse.

(d) Both (a) and (b).

. In the above question, Mridula and Atul mutually decided to part their ways. However, both
of them were reluctant to file for divorce. Atul nevertheless started living with his mistress.
After months of living with his mistress, he realized that this is not something that makes him
happy and he would rather go back to resume his matrimonial home with Mridula. When he
discussed the same with his mistress she got angry and threatened to file for a decree of
RCR.

(a) Her petition is not maintainable as Atul has reasonable grounds to withdraw from her
society

(b) Her petition is not maintainable

(c) The withdrawal from her society has not yet taken place. Atul is only at the stage of
contemplation. Therefore, she can’t file for RCR at this stage.

(d) All of the above

 
. Atul decides to move back with his wife however she blatantly refused to resume
cohabitation with him. After, a lot of unsuccessful attempts at convincing his wife,Atul finally
moved the court with the petition of Restitution of Conjugal rights.

(a) His petition will fail as it was a mutual decision of both the spouses to part their ways

(b) There is a valid and reasonable excuse for Mridula to not resume cohabitation with her
husband

(c) Both (a) and (b)

(d) None of the above.

. Based, on the understanding of the passage what is the recourse available to the
aggrieved party under the petition of RCR?

(a) The aggrieved party gets a portion from the property of the party complaint against

(b) The court may order the specific performance of the decree

(c) The aggrieved party, on the refusal of the other party to oblige by the decree, gets a
ground to divorce the party complaint against

(d) None of the above

. Based on the inferences drawn from the paragraph the non-compliance of the decree leads
to:

(a) Annulment of marriage if the non-compliance persists for more than six months

(b) Lapse of the decree of RCR

(c) Attachment of property of the complying party

(d) None of the above Passage

(Q.-Q.): In Sodan Singh v. New Delhi Municipal Committee, the five-judge bench of the
Supreme Court distinguished the concept of life and liberty within Art.21 from the right to
carry on any trade or business, a fundamental right conferred by Art. 19(1)(g) and held the
right to carry on trade or business is not included in the concept of life and personal liberty.
Article 21 is not attracted in the case of trade and business. The petitioners, hawkers doing
business off the paved roads in Delhi, had claimed that the refusal by the Municipal
authorities to them to carry on the business of their livelihood amounted to the violation of
their right under Article 21 of the Constitution. The court opined that while hawkers have a
fundamental right under Article 19(1) (g) to carry on trade or business of their choice; they
have no right to do so in a particular place. They cannot be permitted to carry on their trade
on every road in the city. If the road is not wide enough to be conveniently accommodating
the traffic on it, no hawking may be permitted at all or may be permitted once a week.
Footpaths, streets or roads are public property and are intended to several general public
and are not meant for private use. However, the court said that the affected persons could
apply for relocation and the concerned authorities were to consider the representation and
pass orders thereon. The two rights were too remote to be connected together. Article 19 (1)
(g) of Constitution of India provides Right to practice any profession or to carry on any
occupation, trade or business to all citizens subject to Art.19 (6) which enumerates the
nature of restriction that can be imposed by the state upon the above right of the citizens.
Sub clause (g) of Article 19 (1) confers a general and vast right available to all persons to do
any particular type of business of their choice. But this does not confer the right to do
anything consider illegal in eyes of law or to hold a particular job or to occupy a particular
post of the choice of any particular person. Further Art 19(1) (g) does not mean that
conditions be created by the state or any statutory body to make any trade lucrative or to
procure customers to the business/businessman. Moreover, a citizen whose occupation of a
place is unlawful cannot claim this fundamental right. The right to carry on a business also
includes the right to close it, at any time depending upon the desire of the owner. Sources:
https://www.lawctopus.com/academike/article-21-of-the-constitution-of-india-right-to-life-and-
personal[1]liberty/

. Jaggu owns a small café near the main road cross-section. His business is legal and does
exceptionally well. But the road is congested and suffers jams almost every day. One day, a
state authority shuts down the entire road and began construction of a fly-over. The work got
delayed by 2 years and Jaggu’s business gets hit adversely. Jaggu took this matter to the
court of law stating that his livelihood is in danger. He also seeks compensation from the
state for mental and financial harassment. The court, however, denies Jaggu’s plea. Based
on the information given in the passage, can you tell why?

(a) because it is an individual vs. the state. Law always favours the state.

(b) because the fly-over was made for public convenience. The state could not be held
responsible for the delay due to heavy traffic load.

(c) because Jaggu’s café was one of the reasons for traffic jams. Therefore, he is not eligible
for any compensation.

(d) because Jaggu had the right to move into some other occupation or place to carry his
business all the time, but he did not.

. Raju, a hawker who sells munchies outside an illegal pub, was beaten by the pub owner
and removed forcibly. His place was then given to some kin of the pub owner. Raju took this
matter to the court of law claiming that this is a violation of his rights under Art. 19 (1) (g),
and pleaded for help. His petition, however, was denied outright. Based on the information
given in the passage, can you tell why?

(a) because Raju’s business was not registered with the Municipal Authority.

(b) because Raju’s claim should fall under Art. 21. Mention of Art. 19 (1)(g) was wrong here.

(c) because Raju might be carrying out his business at the pub owner’s personal property.

(d) because the pub was already an illegal establishment, his claim was unacceptable.

. Mr Shah owns and operates a sugar refinery on a big scale. He employees over 300
workers and labours in his factory and is known for his generosity. He is also seen as a local
beneficiary as he procures sugarcane from the local farmers at a reasonable rate. One day
he decides to sell the sugar refinery to a sugar baron at a good price. The prospective new
management is infamous for exploiting its employees and farmers. The employees and
farmers come together to form a union and take Mr Shah to the court of law calming that he
is ignoring their collective good for his personal benefit. Therefore, he should not be allowed
to sell the business. But the court rules in favour of Mr Shah. Based on the information given
in the passage, can you tell why?

(a) because Art. 21 gives Mr Shah right to exercise personal liberty.

(b) because Mr Shah’s trade of his business will provide the Government with a hefty
amount as tax.

(c) because Art. 19(1)(g) allows Mr Shah to exit any business at his will.

(d) because Mr Shah’s generosity will eventually doom the business and force him to sell.

. Karan is the owner of an electronics store. However, his store is registered and records an
enormous sale every year, he rarely pays any sales tax. He avoids giving bills to the
customers promising after[1]sales service verbally. Once, on a large order paid in advance,
his supplies dupe him by swapping the original technology of the articles with counterfeit
ones. Karan threatens the supplier to take this matter to the court of law but the supplier is
indifferent. Based on the information given in the passage, can you tell why?

(a) because Karan’s mode of operation is illegal.

(b) because Art. 19 does not guarantee any protection to his business.

(c) because Art. 19 (6) provides immunity to the supplier.

(d) all of the above.

. Based on the information given in the passage, which of the following cases is not
exercisable by the law?

(a) Parents forcing a boy to join the family business.

(b) Municipality officers forcing hawkers to shut down their business.

(c) Revenue department revoking the licences of liquor shops.

(d) A lawyer not practising his/her profession.

. Rajan is a graduate from IIT Mumbai and IIM Chennai. He got placed with a reputed
America bank operating in India with a yearly package of 2 crore rupees. While working with
the bank, he gets married to a beautiful girl in a big fat Indian wedding. After a few months of
getting married, Rajan resigns from his job to open a small tea shop near railway station.
Irked from his actions, Rajan’s wife sues him for fraud and demands a settlement. She says
he lied about his carrier plans before marrying her. Based on the information given in the
passage, does this case hold any weight?

(a) Yes. Rajan should go back to his job.


(b) No. Rajan can do what he wants to.

(c) Yes. But both of them need a family counselling.

(d) No. unless there is any written proof, Rajan’s wife cannot claim any settlement

15
Passage (Q.-Q.): The judgment first discussed the evolution of the principle of promissory
estoppel in the English law and the development of the doctrine of legitimate expectation as
its offshoot. Promissory estoppel developed as an equitable remedy to grant relief to a
person who has acted on a promise even in the absence of a contractual relationship or
consideration. It can be used as a "shield" in a legal proceeding and not as a "sword". In
other words, promissory estoppel will not by itself constitute a cause of action and was
allowed to be used as a defence in a legal proceeding. The doctrine of legitimate
expectations developed in parallel to the doctrine of promissory estoppel. It is founded on
the principles of fairness in government dealings. It comes into play if a public body leads an
individual to believe that they will be a recipient of a substantive benefit. Under English Law,
the doctrine of legitimate expectation initially developed in the context of public law as an
analogy to the doctrine of promissory estoppel found in private law. However, since then,
English Law has distinguished between the doctrines of promissory estoppel and legitimate
expectation as distinct remedies under private law and public law, respectively. Another
difference between the doctrines of promissory estoppel and legitimate expectation under
English Law is that the latter can constitute a cause of action. In other words, doctrine of
legitimate expectation can be used as a 'sword'. [Extracted, with edits and revisions,
https://livelaw.in/know-the-law/doctrine-of-legitimate-expectation promissory-estoppel-15] 

. Raju promised to make maintenance payments to his separated wife Sheetal in 2019.
However, due to the advent of coronavirus, he was unable to do so. Sheeteal brings a
lawsuit against Raju in December 2020. Which of the following is true? 

(a) Promissory Estoppel is applicable as a valid consideration is not needed in promissory


estoppel 

(b) Promissory Estoppel is not applicable as Raju and Sheetal do not have a contractual
relationship 

(c) Promissory Estoppel is applicable due to the principle of fairness. 

(d) Promissory Estoppel is not applicable as it cannot be a cause of action. 

. A vendor makes an oral promise to the customer to replace the goods if they do not fit the
customer size requirements. The customer purchases the clothes and takes them home.
However, the customer proposes to return the clothes to the vendor relying on the vendor
oral promise. Which of the following is true? 

(a) Promissory Estoppel is not applicable due to lack of consideration 

(b) Promissory Estoppel is not applicable as contractual relationship is over. 


(c) Promissory Estoppel is not applicable as it can only be used as a shield and not a sword. 

(d) None of the above 

. Ramesh was an employee in a company owned by Suresh. One fine day, Suresh, being
extremely happy with Ramesh‘s work, promised lifetime retirement pensions if he resigns.
Ramesh retired the next day. Suresh refused to pay him. Which of the following is true?

(a) Promissory Estoppel is inapplicable as Suresh did not make the decision in his right
mind. 

(b) Promissory Estoppel is inapplicable as the doctrine cannot be used as a sole cause of
action. 

(c) Promissory Estoppel is applicable as Ramesh acted upon Suresh's promise, making it a
valid defence. 

(d) Promissory Estoppel is applicable as the act of retiring was a valid consideration for
retirement pensions. 

. Rishabh and Mansi wanted to settle a tenancy dispute. They appointed a private arbitrator
to resolve the dispute in a fair and equitable manner. However, while hearing the dispute,
the arbitrator failed to follow due process. Which of these is true? 

(a) The doctrine of legitimate expectation is applicable as principle of fairness has been
violated. (b) The doctrine of legitimate expectation is applicable and it can be used as a
cause of action to challenge the arbitrator's decision. 

(c) The doctrine of promissory estoppel is applicable due to a private arbitrator. 

(d) The doctrine of legitimate expectation is inapplicable. 

. After winning the elections with a landslide victory, Prime Minister Rodie, as the
representative of the government promised concessions in cost of electricity in his
hometown Splitsvilla for the coming five years. However, no such concession was provided
by the government in those 5 years. A resident of Splitsvilla, Ranvijay, sought the
enforcement of the policy with retrospective effect. Which of the following is likely? 

(a) The retrospective enforcement would be allowed due to legitimate expectation 

(b) The retrospective enforcement would be allowed due to promissory estoppel. 

(c) The retrospective enforcement would not be allowed as the government usually makes
such hollow promises. 

(d) The retrospective enforcement would be allowed as the doctrine of legitimate expectation
can be used as a cause of action. 
. A severely disabled woman was promised in clear and unqualified terms a home for life by
the municipal corporation if she agreed to give up her existing accommodation. The woman,
on the promise made to her, left her existing accommodation. However, no such home was
provided to her. Which of the following is true? 

(a) The doctrine of promissory estoppel is applicable 

(b) The doctrine of legitimate expectation is applicable. 

(c) The Municipal Corporation can be estopped through both the doctrines. 

(d) The doctrine of promissory estoppel is not applicable due to lack of contractual
relationship. 

Passage (Q.-Q.): The Supreme Court has held that the offence of dowry death under
Section 304B of the Indian Penal Code cannot be made out if the cause of death has not
been established as unnatural. The Court also held that it has to be shown that the
deceased wife was subjected to cruelty or harassment in connection with demand for dowry
soon before her death. The judgment authored by Justice K M Joseph explained the
ingredients of the offence under Section 304B IPC as follows: "The ingredients of the
offence are well-settled. A marriage performed within seven years before the death of the
wife. The death must be unnatural. Soon before the death, the deceased wife must have
been at the receiving end of cruelty or harassment, on account of demand for dowry. It is
described as dowry death. The relatives concerned, including husband, become liable.
Section 113B of the Evidence Act comes to the rescue of the prosecutor by providing for a
presumption that a person has caused dowry death if, it is shown that soon before her death,
she was subjected by such person for cruelty or harassment for or in connection with
demand for dowry" Extracted, with edits and revisions, https://livelaw.in/know-the-law/dowry-
death-section-304b-ipc supreme-court-unnatural-death-13 

. Ramesh married Seema on 31st December 2001. The pregnancy report taken on 20th
December 2006 showed positive result. However, taunts on carrying of an illegitimate child
by the society led the victim to take her life after a month. Which of the following is likely? 

(a) This is a case of dowry death as the taunts started within 7 years of the marriage.

(b) This is not a case of dowry death as the victim took her life after the period of 7 years. 

(c) This is not a case of dowry death as the taunts were not issued by relatives but the
society 

(d) This is not a case of dowry death as there is no case of Seema been harassed or
subjected to the demand for dowry. 

. Ramesh married Seema on 31st December 2001. The pregnancy report taken on 20th
December 2008 showed positive result. However, taunts on carrying of an illegitimate child
by the society led the victim to take her life after a month. Which of the following is true? 

(a) This is a case of dowry death as the taunts started within 7 years of the marriage. 

(b) This is not a case of dowry death as the victim took her life after the period of 7 years. 
(c) This is not a case of dowry death as the taunts were not issued by relatives but the
society. 

(d) Both (b) and (c). 

. Khusboo married Luthra in the year 2010. A year after the marriage, Khushboo suffered
from lung cancer. After knowing this, Luthra‘s parents started demanding dowry from
Khushboo to conduct marriage of Luthra with someone else in the coming years. Thus
continued for another year and Khushboo died from cancer. Khushboo‘s father filed a case
of dowry death. 

(a) This is not a case of dowry death. 

(b) This is a case of dowry death as Khushboo was subjected to cruelty 

(c) This is a case of dowry death as Khushboo was subjected to constant mental trauma. 

(d) This is a case of dowry death as clearly her in-laws demanded dowry from her. 

. Khusboo married Luthra in the year 2010. A year after the marriage, Khushboo suffered
from lung cancer. After knowing this, Luthra‘s parents started demanding dowry from
Khushboo to conduct marriage of Luthra with someone else in the coming years. Thus
continued for another year and Khushboo died from cancer. Khushboo‘s father filed a case
accusing Khushboo‘s in-laws of dowry death. Which of the following is true? 

(a) As a cardinal principle of criminal law, the presumption lies on the prosecution,
Khushboo‘s father to prove the case 

(b) The burden lies on Luthra to prove that Khushboo was not subjected to any cruelty
before her death 

(c) The burden lies on Khushboo‘s in-laws to prove that Khushboo was not subjected to any
cruelty before her death. 

(d) The initial presumption to move forward in this case would be in favour of Khushboo‘s
father 

. Mike married Rachel in the year 2001. Soon after their wedding Rachel started receiving
letters from Mike‘s parents demanding dowry. After much efforts Mike persuaded his parents
to not send such letters. On 25th December 2006, Rachel was fired from her firm. When she
was at home, she saw those letters sent to her by Mike‘s parents. Reading the letters added
fuel to fire and she concluded her life. Which of the following is likely? 

(a) This is a case of dowry death as the letters by Mike‘s parents led to her death 

(b) This is not a case of dowry death as the letters were sent way back in 2001 and not soon
before her death. 

(c) This is a case of dowry death as all the ingredients of the offence are fulfilled 
(d) The presumption lies against Mike of having committed the offence 

. Shanti was married to Nikhil. At the time of marriage Nikhil‘s family received a dowry of 40
thousand rupees. Soon after the marriage Nikhil demanded a buffalo as additional dowry.
Shanti was tortured constantly. One day when her brother visited the family of in-laws he
found out that Shanti had died. Which of the following is true? 

(a) Nikhil and his family would be liable under Section 304B IPC. 

(b) Nikhil and his family would not be liable under Section 304B IPC.

(c) The burden of proof rests upon Nikhil and his family to prove their innocence. 

(d) None of the above 

Passage (Q.-Q.): Singapore Court has adopted a ‗one-dimensional facet‘ of liberty. This
form of liberty exists in constitutional text but cannot be enforced in real world. The
aforementioned comment is in response to the Court‘s argument that a homosexual person
is not being prosecuted for his identity, but rather for the ‗acts‘ he does in furtherance of this
identity which are socially comprehended to be of gross indecency. Highlighting that it is the
‗act‘ which is penal, the Court remarked that even a heterosexual person committing such
an act can be prosecuted under Section 3A. Sexual identity is ‗personal‘ to an individual and
the ‗liberty‘ to express such identity by way of behaviour, sexual acts or choice of life partner
falls well within the ambit of Article 9. By penalising conduct which does not fall within the
binary, the Court is inevitably prosecuting homosexual people for their identity. The Court
referred to various scientific evidences and expert testimony to conclude that sexual
orientation of a person may vary over time, and therefore, it is not ―immutable‖. This
deduction can be rebutted from two different angles. On one hand, reliance can be laid on
the judgement rendered 18 of 40 in Navtej Singh Johar to argue that sexual orientation of a
person is innate, ingrained and inherent. On the other hand, it can be argued that with more
and more people identifying themselves as LGBTQIA+, people around the globe are in the
process of exploring their true unchanging or immutable self. Thus, their sexual orientation is
not changing, rather their understanding of themselves is. This argument is based on the
premise that sexuality is fluid in nature. Even if the reasoning of the Court is accepted, it
does not in any manner justify that heterosexuality is the only ‗normal‘ and just because the
orientation of a person can vary, they should be forced by penal measures to resort to
heterosexual ways of sexual expression. 

. FarrokhBulsaraan as acclaimed celebrity, artist and musician in Singapore has been


involved in a controversy, he has been accused of indulging in act of carnal intercourse, in
pursuant to which a case has been filed against him under Section 3A. You are a Lawyer
who is obsessed with winning a case. Which role, amongst the following, are you most likely
to choose in order to maintain your win-loss ratio 

(a) Prosecution. 

(b) FarrokhBulsaraan. 

(c) Amicus Curie. 


(d) Legal view is conflicting, cannot be determined. 

. After contemplating morality and significance of this case you agreed to take this high
profile case. WRT the passage above, provide legal advice to Farrokh. 

(a) Indulging in homosexual activities is not a crime and Section 3 has been decriminalized
by the Supreme Court of India, there is nothing to worry about any legal prosecution. 

(b) Sexual Orientation is a personal choice and a right to life and personal liberty of an
individual, as per the law no one can be prosecuted in India for indulging in homosexual
activities. 

(c) The act against the nature, i.e. the act of unnatural sex, is not only against the law but
also considered gross indecency. 

(d) Even if Navtej Singh Joharcase is considered the court has yet not decriminalized the act
of homosexuality, the only recourse is to challenge the judgement. 

. Mary Austin ex-girlfriend of farrokh gave testimony that farrokh forced her in committing
sodomy, thou Mary stopped it immediately after the first successful attempt. Farrokh claims,
it happened by mistake. Help farrokh understand the future of the case: 

(a) Nothing done by mistake, or without intention to commit a crime can be prosecuted under
penal law, mensrea is essential. 

(b) Farrokh indulged in homosexual activity, whether by mistake or by will, offence has been
committed. 

(c) There is no case against farrokh. 

(d) Farrokh will be prosecuted as the act is punishable per se. 

. After the news of Mary‘s testimony got out several people came out and accused farrokh
for the same, including Paul Painter former best friend of farrokh. Paul accused farrokh of
forcing him to indulge in oral sex and produced unverified evidence for the same. Decide the
recourse. 

(a) Oral sex is not considered a homosexual activity therefore farrokh cannot be prosecuted
for the same, charges are frivolous. 

(b) Farrokh will be prosecuted. 

(c) Farrokh will be held liable for indulging in homosexual activity under Section 3A. 

(d) Farrokh has committed an ultimate sin, which is not only against the law but also
considered gross indecency in the eyes of the society. 
Passage (Q.-Q.):A confession or an admission is evidence against the maker of it, unless its
admissibility is excluded by some provision of law. Section 24 excludes confessions caused
by certain inducements, threats and promises. Section 25 provides: "No confession made to
a police officer, shall be proved as against a person accused of an offence." The terms of s.
25 are imperative. A confession made to a police officer under any circumstances is not
admissible in evidence against the accused. It covers a confession made when he was free
and not in police custody, as also a confession made before any investigation has begun.
The expression "accused of any offence" covers a person accused of an offence at the trial
whether or not he was accused of the offence when he made the confession. Section 26
prohibits proof against any person of a confession made by him in the custody of a police
officer, unless it is made in the immediate presence of a Magistrate. The partial ban imposed
by S. 26 relates to a confession made to a person other than a police officer. Section 26
does not qualify the absolute ban imposed by s. 25 on a confession made to a police officer.
Section 27 is in the form of a proviso, and partially lifts the ban imposed by ss. 24, 25 and
26. It provides that when any fact is deposed to as discovered in consequence of information
received from a person accused of any offence, in the custody of a police officer, so much of
such information, whether it amounts to a confession or not, as relates distinctly to the fact
thereby discovered, may be proved. Source: AghnooNagesia v. State of Bihar [AIR 19 SC
119] 

. Bob lived in a village with his old aunt, her son, daughter-in-law and a four year old
grandson. One morning, following a fight, he kills his aunt, his cousin with a scythe, and
throws the scythe in the canal outside the village. He runs away without anybody noticing the
happening of the criminal act. Late in the afternoon, his conscience is shaken, and
consequently, he goes to the police station, and narrates the entire episode to the officer on
duty. The officer wrote down Bob‘s entire confession, took him into custody, and now wants
to prove Bob‘s confession in the trial for the murders. 

(a) Bob can be convicted based on the confession because he has accepted his guilt. 

(b) Bob cannot be convicted based on the confession because his confession was made to a
police officer. 

(c) Bob can be convicted because when he was making the confession, he was not yet
accused of murder, and hence the section 25 restriction would not apply. 

(d) Bob cannot be convicted because there are no witnesses to the commission of the act. 

. Based on the details told by Bob, police go to the canal, and find the scythe, wrapped in a
gunny bag. The police want to prove the murder weapon in the court. 

(a) The police cannot prove the murder weapon because the confession was made in police
custody. 

(b) The police cannot prove the weapon because the confession was not made in the
presence of a magistrate. 

(c) The police may prove the weapon, because it was discovered in pursuance of
information received from Bob, and it related distinctly to a fact discovered. 

(d) The police must prove the murder weapon, because otherwise the accused would have
to be acquitted. 
. David Abraham is accused of indulging in drugs trade, money laundering and terrorist
activities. After being at large for over a decade, he returns to India. A police team, in an
attempt to nab him, sets up a trap and asks a journalist to set up a telephonic conversation.
On the phone call, unaware of the trap, David brags about his drug empire and the wealth,
and how the police have been unable to get hold of him. The police record the conversation,
and arrest David. David pleads not guilty, and this recording is presented in the court as
evidence against him. 

(a) This recording of the phone call cannot be proved because the confession was being
overheard by the police, and hence is barred under section 25. 

(b) The recording of the phone call cannot be proved because it was not made to a
magistrate but to a journalist. 

(c) The recording of the phone call can be proved as evidence because a confession made
to a journalist is not prohibited.

(d) The recording of the phone call can be proved in court because this call was the basis of
his arrest. 

. Raju and Rashi had been married to each other for a decade. Raju had been an alcoholic,
and indulged in domestic violence against Rashi very frequently. Fed up with the constant
torture and violence, Rashi sought the help of the police. Mr. Charlie, the police inspector,
talked to Raju, and warned him of serious consequences if he does anything again. He
accepted that he used to hit Rashi under the influence of alcohol, and promised to never do
it again. He stopped this behaviour and started working. During the COVID lockdown, he lost
his job, and started drinking again. One day, after drinking a lot, he started to argue with
Rashi, and slapped her hard. Having given up on him, finally Rashi files a suit for divorce on
grounds of cruelty and presents Inspector Charlie as a witness. 

(a) Inspector Charlie cannot be a witness and Raju‘s statement cannot be proven in
evidence because it falls within ‗confession made to a police officer.‘ 

(b) Inspector Charlie cannot be a witness because this is a domestic affair and police are not
supposed to interfere in domestic matters. 

(c) Inspector Charlie can be a witness and testify in the trial because the section 25 bar
applies only to criminal trials, when the person making the confession is accused of an
offence, and this is a divorce proceeding. 

(d) Inspector Charlie can be a witness because he was aware of the nature of domestic
violence and harassment which Rashi was subjected to, and was the investigating officer
when Rashi had complained to the police. 

Passage (Q.-Q.):Just recently, it has come to light that the State of Arizona has put forward
a new death penalty plan given the lack of availability of anaesthetics and sedatives
previously used in the death chamber. The plan being put forward is a provision that puts the
onus on a death row inmates‘ lawyer to provide the drugs so the State can follow through
with the planned death penalty. To say that this plan throws some ethical issues in the face
of those lawyers would be a great understatement. One of the main duties of any lawyer is to
act in the clients‘ best interests and in accordance with their instructions. Here we have a
situation where Arizona is attempting to put a duty on a lawyer to act in the state‘s best
interests and as per their instructions rather than the client‘s. This flies in the face of the
duties of a lawyer to their client. This is before we even take into account the fact that it is
against Federal law in the US to import these drugs. It should be noted that a lawyer is
bound to not assist or take part in criminal conduct with their client but here, Arizona expects
a lawyer to commit a criminal act for the State so that they can execute their client? The fact
that committing a criminal act can be seen to indicate that the lawyer is not fit to practice and
can amount to professional misconduct. We could also turn our mind to the matter of
professional ethics which stipulates that a lawyer must act independently and impartially in
providing legal services but the plan being put forward is essentially asking the lawyer to be
a slave to the aims of government and hammer their client to the stake, so to speak. Further,
when acting with this independence the lawyer is given the right to refer to considerations
such as moral, social and political factors in addition to matters of law. However, it is
perfectly fine for a lawyer to buy end-of-life drugs for someone who is not choosing to die.
Even more ironic than any of the above would have to be the professional rule that Arizona
lawyers are bound by, which insists that a lawyer should reveal any information they receive
that could prevent their client from committing a criminal act that would result in death. But
here we are expecting a lawyer to commit a criminal act that results in the death of their
client? At the end of all that, we can also add the fact that the lawyer has also likely engaged
in conduct that could be seen to be prejudicial to the administration of justice. Now we could
potentially end up with a bunch of lawyers who will have been acting in the way they have so
been ordered by the State, yet are likely in prison themselves for importing an illegal
substance into the US. Is Arizona trying to ensure the end of the lawyers that advocate for
these people too? [Tasha Russell, ‗Arizona‘s Death Penalty Procedures & Professional
Ethics‘ Jurist accessed on 23 May 2020] 

. In light of the state of affairs as highlighted in the above passage, which of the followings is
not likely to be a measure which the state of Arizona is going to implement on its land?

(a) the lawyers to be given some financial help for buying the drugs which shall be used for
executing the death sentence. 

(b) the lawyers to be allowed the immunity to hold representations against the directions of
the state in a completely free manner. 

(c) to organise a medical workshop for the lawyers, seeking to train the candidates to
administer the drug in a right manner in order to execute death sentence. 

(d) Both a & c 

. In light of the professional ethics, the author suggests which of the following upholds the
author‘s view? 

(a) Compelling the lawyers to buy drugs can result in commencement of the trial on
themselves which amounts to professional misconduct. 

(b) Asking the Arizonian lawyers to reveal any information about their client to the state
before the institution of the death sentence amounts to professional misconduct. 

(c) Not following Federal US law and the directions of the government amounts to
professional misconduct on part of the lawyers. 
(d) Compelling the lawyers to act on the directive of the state which imply acting partiality in
favour of the state‘s amounts to professional misconduct on the part of lawyers. 

. Which of the following reasons does the author believe would be probable for the Arizonian
lawyers to land up behind the bars? 

(a) For importing life-threatening drugs to the US for the purpose of killing their clients. 

(b) For resisting to follow the state orders with respect of executing the death sentence of
their clients. 

(c) For not revealing enough information about their clients before the pronouncement of
death sentence.

(d) All of the above 

. Let‘s say, the state of Arizona, in addition to the above state of affairs, implements a
provision whereunder, all the lawyers practicing in the state of Arizona shall be exempted to
be held liable for importing the drugs required for the execution of their clients who have
been sentenced by the court. In such a case, which of the followings most likely still remains
as the author‘s concern? 

(a) Lawyers not receiving enough medical training to administer the drug on their client. 

(b) Lawyers not being able to fulfil their professional ethics and still functioning on the state
orders which fall contrary to their duties.

 (c) Lawyers not being able to practice again in the legal field due to violating law. 

(d) None of the above.

. The author ends the passage by posing a question that is - ‗Is Arizona trying to ensure the
end of the lawyers that advocate for these people too?‘ What does author imply here? 

(a) the author asks the readers to ponder over the recent changes that the Arizonian state
has imposed. And in addition to this, analyse as to how through these changes the lawyers
would lose to profess their professional duties at the cost of benefit of the state. 

(b) the author asks the reader to observe the implications of state sanction murders
committed by the lawyers and how it will further the apprehension of the Arizonian citizens
towards the lawyers of this state. 

(c) the author asks the reader to give consideration towards the possibility that the Arizonian
state is conspiring to put all the layers behind the bars. 

(d) Both a & b 


Passage (Q.-Q.):As the Australian government struggles to keep up with the rapid pace of
COVID 19 (coronavirus), societal freedoms are being restricted. At the same time, certain
constraints around criminal procedure are being loosened. In Australia‘s most populous
state, New South Wales (NSW), the government has introduced the COVID-19 Legislation
Amendment (Emergency Measures) Bill 2020 (Emergency Legislation) that amends laws
around pre-recorded evidence in criminal trials. NSW, along with most other states and
territories in Australia, already allows for pre-recorded evidence from witnesses who are
cognitively impaired or children who are victims of sexual assault. This Emergency
Legislation expands the categories of witnesses that can give pre-recorded evidence prior to
the trial taking place. Witnesses eligible to give pre-recorded evidence will now include
complainants of serious violence, including domestic and sexual offences, alongside
complainants or witnesses who may have a greater risk of harm from coronavirus. This
timely change self-evidently responds to concerns in relation to criminal procedures which
are far removed from social distancing. While change is therefore necessary in the current
health crisis, it is important to ensure that such change does not unduly impact the
accused‘s right to a fair trial. This fundamental right is inscribed internationally within Article
14 of the International Covenant on Civil and Political Rights, to which Australia is party. In a
process evaluation report of the NSW program for pre-recorded evidence from victims of
child sexual assault, the University of NSW Social Policy Research Centre outlines the key
concerns to the accused‘s right to a fair trial. These concerns are two-fold: (i) short
timeframes may impact the accused‘s ability ‗to know the full case before cross-examination
of the main witness‘, and (ii) new evidence ‗sought after the pre-recording‘ may impact the
accused‘s ability to know ‗the entirety of the evidence prior to the balance of the trial‘. In the
expanded context within which pre-recorded evidence will be allowed under the emergency
legislation, both of these concerns are also valid. While emergency legislation is important
considering the unprecedented times within which we are living, such times should not allow
for the health concerns of an individual witness to be unnecessarily placed above an
accused‘s right to a fair trial. This is critical in situations where an accused‘s liberty is at risk.
Alternative options are being explored within different states in Australia. In Victoria,
Australia‘s second most populous state, the expanded use of virtual courts is being actively
considered in the wake of coronavirus. Such alternative options should be explored
nationally. Yet this emergency legislation may also prove advantageous to sexual assault
victim advocates. In the context of child victims of sexual assault, pre-recorded evidence has
been found to reduce the trauma and abuse such victims may endure if required to give
evidence in court. In extending pre-recorded evidence to adult victims of sexual assault, this
legislation could create the impetus needed to extend this program permanently to all victims
of sexual assault. [Samantha O'Donnell, ‗Pre-recorded evidence in Australia in an era of
COVID-19‘ Oxford Human Rights Hub accessed on 23 May 2020] 

. Daniel, a NSW citizen recently tested positive for COVID – 19. He is scheduled to provide
his testimony in a case where his brother, George is accused of domestic violence. Amongst
which of the following, is the author of the passage most likely to agree with? 

(a) if in case after Daniel records his testimony there occurs some change of events, crucial
for the administration of justice, George might not have a fair chance to represent himself
before the court of law. 

(b) Daniel‘s health condition can be compromised against George‘s right to fair trial. 

(c) George‘s right to access a fair justice procedure cannot be prioritized over 

(d) Both a & b 


. The author of the passage does not agree on which of the followings? 

(a) virtual courts do not disadvantage the accused from his/her rights as enshrined under
International Covenant on Civil and Political Rights. 

(b) the emergency legislation implemented in view of the current health crisis has acted as a
long due step which was required to take the concerns of the adult witnesses of the sexual
assault into consideration. 

(c) both a & b 

(d) None of the above 

. In light of the situation prevailing before the implementation of the emergency legislation,
which of the following is most likely to be true?

(a) all the sexual assault victims in Australia used to provide their testimony on virtual
courts. 

(b) only the child sexual assault victims were allowed to give pre-recorded testimony. 

(c) the Article 14 of the International Covenant on Civil and Political Rights went
unimplemented as a national legislation of Australia. 

(d) both b & c 

. Leslie, a major witness to the child sexual assault survivor is supposed to provide her
testimony before the court but due to COVID -19, is asked to provide a pre-recorded
evidence. According to the author, by such a procedure, she shall be saved from which of
the followings?

(a) by contacting the COVID – 19. 

(b) she shall not have to face the potential abuse from the accused in the courtroom thus not
resulting in any form of additional trauma. 

(c) she shall not be held liable for going against the statute. 

(d) None of the above 

. NSW has implemented the current emergency legislation with which of the following
objectives? 

(a) to incorporate the international obligation mentioned under article 14 of the International
Covenant on Civil and Political Rights. 
(b) to devise an inclusive approach, taking into consideration all the aspects of witness
protection in a criminal case. 

(c) to adopt an approach which ensures social distancing 

(d) None of the above 

Passage (Q.-Q.):According to the Delhi High Court‘s decision in Myspace v. Super


Cassettes, intermediaries (which include online messaging platforms like Telegram) can
claim the status of intermediaries and can claim the ‗safe harbour‘ from liability for copyright
infringement, available under Section of the Information Technology Act (IT Act). According
to Section and the rules made thereunder, the intermediary can claim such safe harbour
from liability provided it does not have ‗actual knowledge‘ of the illegal (here, infringing)
content, and does not fail to expeditiously take down such content upon receiving such
knowledge. The more unsettled question is the liability for unlawful content on closed forums
or online groups. This is a question that‘s being hotly debated under Indian law – with the
Supreme Court adjudicating a matter concerning WhatsApp‘s responsibility for unlawful
content. Similarly, there are numerous examples of orders under Section 144 of the CrPC
being issued to hold administrators of WhatsApp groups liable for ‗fake news‘ and unlawful
content distributed on the group. The question of liability of an intermediary for online content
becomes significantly muddier if the intermediary itself has no knowledge or means of
examining the content in question, as is the case with certain encrypted messaging services.
The administrator of a closed forum would not be granting permission for the content to be
posted on the forum; therefore, the administrator will have no direct control for granting
permission for the content posted on the forum. Therefore, the role of the administrator will
have to be considered, to see if they have control over the infringing content shared on the
channels or groups, to see if they generally exercise editorial control (such as removing
posts or filtering messaged), or if they permit unlawful activity 24 of 40 despite knowledge of
the same. Liability of this nature may, in fact, be attributed to both the platform (intermediary)
as well as the administrators responsible for an online forum like Telegram channels. (The
extract has been taken with edits from https://spicyip.com/2020/06/delhi-hc-directs-telegram-
to-take down-channels-circulating-infringing-e-newspapers.html) 

. Ravish is the administrator of a WhatsApp group, where small scale business owners offer
their products for sale to potential customers. Ravish is only has the access to add or
remove members, he cannot regulate the content in any manner. Aman posted the pictures
of gown that he proclaimed to have created and offered it for sale to the users of the forum.
Mr. Goenka, a fashion designer discovered the design of the gown to be very similar to a
limited-edition gown manufactured under his label. Mr Goenka‘s Company has decided to
initiate action against Ravish for the plagiarism of their design. Would Ravish be liable for
infringement of Mr. Goenka‘s copyright? 

(a) Yes, Ravish was is the administrator of the group and the liability for the content posted
on the group lies with him. 

(b) No, Ravish only acted as the administrator of the group, he did not have the access to
moderate the content. 

(c) Yes, Ravish had a responsibility to check the authenticity of the content being posted on
the forum. 
(d) Yes, Ravish has been an accomplice to Aman in the act of infringing Mr. Goenka‘s
copyright over his design. 

. ABC Resident‘s welfare association created a WhatsApp group exclusively for its
members. Manish, the President of the association was the administrator of the group. Over
time it was found that Jatin a member of the association had been publishing defamatory
messages and insidous rumours against some members of the association in the members
exclusive WhatsApp group. Can the administrator Manish be held liable in the present
case? 

(a) No, Manish was only the administrator of the group and he is not responsible for the
content posted by the members of the group. 

(b) Yes, Manish was aware of the defamatory nature of the content and could see that
malicious content was being posted on the platform without any supporting evidence to
authenticate the same, but still failed to take any action. 

(c) No, this was a dispute between some members of the group and Manish is not a party to
the same. 

(d) Yes, posting derogatory material about a person without sufficient proof is an act of
defamation punishable by law and Manish has permitted the act to continue despite being
aware of the same. 

. ORM is an online video sharing application. Members registered on the application can
upload their videos and share it further on other Platforms. Kay a musician has sent a letter
to ORM informing them of a particular video uploaded on the Application comprising of a
musical note similar to his creation and the same has been used without his permission.
Receiving no response Kay proceeds to file a case of infringement of his copyright making
ORM a party to the same. Can ORM be held liable? 

(a) No, ORM is only an intermediary who host the videos on their application, they are not
responsible for the content uploaded in the videos. 

(b) Yes, ORM was informed about the unauthorised use of the copyrighted musical work of
Kay but failed to take down the same upon being informed. 

(c) Yes, ORM would be responsible as they permitted uploading of the content on the
platform without verification of the same. 

(d) No, the user responsible for uploading the content on the ORM Application alone would
be responsible as the right for the work vests with them alone. 

. Insta an instant messaging platform has been made a party to a copyright infringement
case by Nation Today, a monthly magazine for permitting circulation of the illegally
downloaded PDF version of the magazine on several of its user groups and failing to take
action even upon being reported about the same by Nation Today. To what extent is Insta
liable in the present case? 
(a) Insta does not bear any liability in the present case, it only hosts the groups and
facilitates messaging on the groups and is not responsible for the content shared. 

(b) Insta is liable for aiding in the unauthorised circulation of the magazines as they failed to
remove the groups or ensure removal of the infringing content. 

(c) Insta is liable in the present case as despite being informed of the illegal activity failed to
take reasonable measure to correct the same. 

(d) Insta would bear the liability as an intermediary who failed to take corrective action
despite breach of law by the users of the Platform, despite being aware of the same. 

Passage (Q.-Q.):Doctrine of proportionality finds its place in the Administrative law and is
used at the stage of judicial review. The doctrine asserts that there must be a reasonable
nexus between the desired result and the measures taken to reach that goal. The action
taken must not be shockingly disproportionate to the consciousness of the court and the said
action can then be challenged by way of judicial review. It can be better understood with the
help of an illustration. Let's say, if in a workplace some workers remain absent from their
duty then the punishment for it must be proportional, that is, the employer may treat it as
leave without pay and may warn them or may even levy a fine but to dismiss them from
service permanently would be disproportional. Sir John Laws has described 'proportionality'
as a principle where the court is concerned with the way in which the decision maker has
ordered his priority. Lord Diplock in R vs. Goldstein in a bit to explain 'proportionality' said,
"This would indeed be using a sledge-hammer to crack a nut" 'Proportionality' involves a
"Balancing test" which keeps a check on the excessive or arbitrary punishments or
encroachment upon the rights and "Necessity test" which takes into account other less
restrictive alternates. The principle of proportionality evaluates two aspects of a decision: (1)
Whether the relative merits of differing objectives or interests were appropriately weighed or
"fairly balanced"? (2) Whether the measure in question was in the circumstances
excessively restrictive or inflicted an unnecessary burden on affected persons? The court in
such a case will not be concerned with the correctness of the decision rather the method to
reach such decision. The decision making process involves attributing relative importance to
various aspects in the case and there the doctrine of proportionality enters. [Extracted, with
edits and revisions, from Doctrine of Proportionality - An Explainer by Saransh Sharma, Live
Law www.livelaw.in/know-the-law/doctrine-of-proportionality-an-explainer-1433] 

. An Army Officer disobeyed the lawful command of his superior officer by not eating food
offered to him. Court-martial proceedings were initiated and a sentence of one-year rigorous
punishment was imposed. He was also dismissed from service, with added disqualification
that he would be unfit for future employments. Which of these can be the possible reasoning
of the court? 

(a) The decision taken through the court-martial proceedings was right as the interests of the
army are far greater than the individual's interest.

(b) The decision taken through the court-martial proceedings are wrong as the punishment is
excessive in nature. 

(c) The court-martial proceedings considered the objective of deterring individuals in the
future from disobeying the commands of senior officers. In such circumstances, the court
was right to implement excessive restrictive harm to the officer. 
(d) None of the Above 

. Certain Employees went on an illegal strike. They also prevented others from discharging
their duty. The company held that the acts amounted to "Serious misconduct." The
employees were punished through the stoppage of increment in the coming months. This
was challenged by the court. Which of these can be a possible reasoning of the court? 

(a) The punishment imposed on the employees is wrong as they have a right to protest. 

(b)The punishment seeks to deter individuals in the future from going on illegal strikes
against the company. Therefore, it is not over-restrictive. 

(c) The punishment imposed upon the employees is right as it was an illegal strike against
the company. 

(d) Both (b) & (c) 

. The internet was shut down and the movement were restricted in the State of Kammu and
Jashmir due to mob protests throughout the State. The following order of the government
was challenged through judicial review. Which of these could be the reasoning of the court in
reaching a conclusion? 

(a) The order is invalid as it violates Article 19(1)(d) of the constitution. 

(b) The order is valid as it helps in maintaining peace in the State. 

(c) The government did not weigh the fundamental rights against maintaining peace of the
State appropriately and has put an unnecessary burden upon the people of Kammu and
Jashmir. 

(d) The order of the government was overly restrictive and should be struck down. 

. SedwardNowden, an Indian citizen, approached the Supreme Court when he came to know
that the government was tracking his every movement on the purported grounds that
Sedward was going to leak important government documents to an enemy country. Which of
these could be a possible reasoning of the Supreme Court for dealing with the issue? 

(a) Sedward should be provided with appropriate relief as his right to privacy has been
breached. 

(b) Government‘s breach of right to privacy of Sedward to secure national interest was over-
restrictive in nature. 

(c) National Security is far more critical in comparison to the right to privacy of a single
person. Therefore, the measure was not over-restrictive. 

(d) Both (b) & (c)


16
Passage (Q.-Q.): In this age of corporatisation, most jurisdictions recognise the concept of a
company as a separate juristic person, with an identity distinct and independent of its
shareholders, members or directors. This corporate existence separates a company’s identity
from that of its promoters or shareholders. It enables the company to contract in its own
name, with its shareholders and third parties, to acquire and hold property in its own name,
and to sue and be sued in its own name… This independence or distinction is not a new
concept. In the late 19th Century, the judgment in the classic case of Salomon v. Salomon
was passed, ruling that a company is a separate legal entity distinct from its members and so
insulating Mr. Salomon, the founder of A. Salomon and Company, Ltd., from personal
liability to the creditors of the company he founded… Each company upon incorporation, is
treated as a separate legal entity, unless two or more companies function as a single economic
entity, in which case the corporate veil may be lifted. So, for instance, where the subsidiary
does not in fact act autonomously and essentially only carries out instructions given to it by
its parent, it is possible to say that the subsidiary and the parent are really one and the same,
thus ignoring their separate legal status and lifting the veil. Among the factors a court would
consider in this regard, are whether the companies were “guided by the same head and brain”
and whether the parent controlled the actions of its subsidiary. Courts have also lifted the
corporate veil if it is found that a subsidiary company has been constituted with the sole
intention of concealing the true facts, to act as a façade and thereby perpetrate a fraud, or to
“look at the realities of the situation and to know the real state of affairs”.

. Crazy Foods Ltd. is a Gurugram based company which mainly deals in frozen foods. It has a
subsidiary, namely Gogo Foods Ltd., in the Southern part of the country to facilitate the
distribution in the said region. Crazy Foods has been a remarkable success in the Northern
region, which it oversees. However, Gogo Foods has not been able to fully establish itself in
its native market. To boost its revenue and capitalize more on its cold storage facilities, Gogo
Foods entered into a contract with a milk delivery start[1]up Daily Dairy. The start-up is in its
early stages, however, its partnership with Gogo Foods has given it an edge in the market.
However, unaware of this arrangement, Crazy Foods has unilaterally decided to exit the
Southern market. Therefore, it starts selling the assets of Gogo Foods Ltd. Now, decide
whether Crazy Foods decision will be upheld in a court of law.

(a) Yes, Gogo Foods being a subsidiary is a part of Crazy Foods. Therefore, it has the right to
strip off its subsidiary’s assets.

(b) Yes, Gogo Foods mainly was created with the sole purpose of handling the Southern
distribution of the parent company. Therefore, they are selling and marketing the same
product under different disguises.

(c) No, Gogo Foods Ltd. must be consulted before selling off its assets.

(d) No, Daily Dairy must be consulted before selling off Gogo Foods assets.

. Daily Dairy did well for a few months, however it was not able to raise the capital it needed
to sustain itself. Therefore, the business goes under and Gogo Foods sustains huge losses.
The parent company ultimately intervenes and bails out the business. It chides the Board and
the subsidiary’s CEO for their failings and removes them from their position. The changes
made were such that they are in tune with the Frozen Foods distribution in South and the
brand’s overall interest. To cut further losses the newly appointed CEO then takes initiative
to launch new frozen products in the market to match the demand of the Southern parts.
However, Gogo Foods was still unable to revive itself and the parent company has no choice
but to sell its business. Decide.

(a) Crazy Foods can sell the business as it is in a better position as a parent company.

(b) Daily Dairy has to compensate for the losses it has caused to Crazy Foods and Gogo
Foods.

(c) Gogo Foods has to be consulted before making this choice.

(d) New CEO’s efforts cannot be ignored as he has launched their own products in the market
making Gogo Foods a separate entity.

. New Horizon is a joint venture between three of the biggest publishers in the country. It is a
newly established entity with experienced staff members from the constituent organisations.
Department of Telecomm has invited tender for the publication telephone directories. New
Horizon offered the best bid (cost-wise), however they were rejected by the Tender
Committee for not fulfilling the minimum experience requirement. The company then
decides to pursue legal means. Decide.

(a) New Horizon has the best bid, it cannot be rejected in the name of equity, justice and good
conscience.

(b) New Horizon is a joint venture of different recognised publishers therefore it qualifies for
the bid.

(c) New Horizon is a joint venture and it is a legal entity in its own. It as a newly established
entity cannot bid unless it develops the relevant experience under its own name.

(d) New Horizon as a joint venture of various publishers is infact better suited for the bid.

. Jingalala Corp, a public company, finds itself unable to pay back a bank loan of a total of
Rs. 15,000 Crores. The Bank authorities want the CEO to cough up the money from his
assets since his reckless decisions caused huge losses to the company from which it was not
able to recover. Infact, both the assets of the CEO and the company are not sufficient to cover
the said amount. Aggrieved by this, the CEO approaches the court to direct the bank officials
from going after his assets. Decide.

(a) The CEO’s assets are his own and have nothing to do with the bank loan.

(b) The CEO’s property should be attached so that the loan can be recovered.

(c) It was the CEO who ultimately caused losses to the company therefore he has to pay for
his wrongs.
(d) Actions of the CEO directly reflect that of the company’s, therefore CEO is liable for the
losses his ill decisions made.

. Dodaphone is a multinational telecom company based out of UK. They have had presence
in India for a long time, however, under a different brand name Kutch. Kutch was later
renamed as Dodaphone India to make it more synonymous with the international brand, later
Dodaphone sold its majority share in the Indian company to another foreign entity. The tax
officials claim that this is a case of tax evasion and veil should be lifted to see that the parent
company is the one which has benefited from this transaction. Decide.

(a) This is a classic case of tax fraud and hence the veil can be lifted.

(b) The Indian entity is different. The veil cannot be lifted as the Indian entity or its Directors
did not benefit from the transaction.

(c) Dodaphone changed its name to make it synonymous with the original brand therefore it
is controlled for the benefit of the actual owners.

(d) Dodaphone UK controls and benefits the Indian arm and therefore the transaction is liable
to be taxed.

. Now, Dodaphone India is the largest Telecomm services provider in India, to obtain the 4G
spectrum the officials of Dodaphone India frequently visited the Minister for Telecomm and
Communication and it raised some suspicions regarding Insiders Trading. A case has been
filed against Dodaphone for insiders trading. Decide.

(a) Dodaphone cannot be prosecuted as a mere meeting, of officials with the minister,
suggests nothing and such arguments are baseless.

(b) While dealing with Tenders it is of utmost importance that the bidders maintain a safe
distance from the authorities concerned. Dodaphone will be held liable for insiders trading.

(c) Dodaphone is a company concerned with their business in telecom and has nothing to do
with insiders trading.

(d) The fact that the respondent managed to become the largest telecom service provider
within such a short period suggest some inappropriate conduct and the action of officials and
ministers suggest the same, Dodaphone must be prosecuted.  

(Q.-Q.): In Delhi’s Shaheen Bagh, women led peaceful protests were organised to oppose the
enactment of the Citizenship Amendment Act, 2019 (CAA) The Court highlighted that these
rights (a right to protest and express dissent) are subject to reasonable restrictions pertaining
to the sovereignty of the State and public order. Relying on its decision in Mazdoor v. UoI,
the Court said that the rights of the protestors have to be balanced with the rights of the
commuters… The Supreme Court had previously dealt with the clash between the right to
protest and convenience of the public in the case of Himat Lal v. Commissioner of Police.
While settling the issue, the Court said that a reasonable restriction in the form of seeking
prior permission of the authorities to hold the protest at a designated time and place can be
imposed. However, in doing so, the Court said that authorities should not have discretionary
powers in granting permission to the protestors. In the case at hand, the Court has imposed an
additional requirement on the protestors which is to not occupy public places, regardless of
seeking prior permission of the authorities. This decision not only limits the right to protest
but makes way for the authorities to clear any and every protest carried out at a public place.
Further, the Court also did not lay down any clear guidelines for the administrative
authorities to adhere to. This decision gives enormous power to the authorities to limit the
number of protestors, the duration of the protest and most importantly, the usage of public
spaces. Emphasizing on how the expression of dissent in a self-ruled democracy cannot be
the same as it was under colonial rule, the Court in this decision has taken away the entire
essence of the right to protest by giving administrative authorities the discretion to limit the
usage of public places, where most protests are carried out.

. The Parliament passed 3 laws recently. However, the laws have not been well received by
the businessman in certain states of the country. To voice their dissent, they plan to gather for
a peaceful protest around the national capital Delhi, without occupying public places.
However, their request for permission was rejected by the competent authority on the grounds
that protesting against an Act passed by the parliament is against the national interests of the
country. Decide.

(a) Permission cannot be rejected, it is the right of the people to protest and discretion cannot
take away a right bestowed upon by the constitution.

(b) Protest not only hamper the security of the state but also the economic condition, allowing
wide scale protests is against national interest and the permission was denied rightfully.

(c) In times of corona right to protest has to be suspended, in times of social distancing
allowing people to gather endangers them and are a liability to the state.

(d) Permission cannot be rejected as the businessman need to raise their voices, and protest is
the only way for them to have their voices heard when the government doesn’t

. Later the request for protest was accepted and businessman occupied a designated area, it
was observed that the road adjacent to the protest site met with severe traffic jam. On the
basis of disruption of commuter’s authorities cancelled the permission of protest and vacated
the protesters, is this decision correct?

(a) Yes, the protest resulted in a traffic jam and according to the court right of commuters
cannot be compromised because of a protest.

(b) No, the authorities cannot cancel the permission for protest on baseless reasons.

(c) No, the authorities do not want the protest to happen and are finding reason to disallow
the protest and snatching away a fundamental right of the citizens to protest, this decision
goes against the law.

(d) None of the above.

. If in the above facts, say the protesters move into public places and block them indefinitely
until the government listens to their demands. The government is also adamant in its stance
and wants them to clear the public places. According to the author does the right to protest in
the current situation fulfils the basic requirement of the right?

(a) This move is wrong. Permission to protest cannot be rejected because of standstill
between the government and protesters.

(b) I agree with the authorities. People should not occupy public places as this has been very
explicitly made clear by the court  

(c) The right to protest is being dominated and the essence has been destroyed by allowing
the authorities to decide the important aspect of the protest.

(d) The government and the authorities have a right to object to such demands when security
of the nation is at stake, rightfully rejected.

. Poppu, a politician, wants to hold a rally in favour of farmers’ right. His supporters will
brandish their personal traditional farming tools in the rally as a show of strength. The
authorities have rejected the request for grant of permission. Why do you think permission
was rejected?

(a) The protests which threaten the public order should be discouraged.

(b) The protests by politicians are not new and they can always voice their opinions in the
concerned state legislature, such protest tactics are used to create a political turmoil.

(c) Protest that is backed by political motive should be discouraged and allowing such protest
to happen threatens the very essence of democracy.

(d) Rallies are done on road and roads are a part of public place, allowing people to occupy
public place is against the law.

. The farmers are standing before the court and arguing to allow the protest to carry on. what
reason could they give for the court to accept such request?

(a) Farmers just want their voices heard and protesting publically is the only way to get in
sight of the government, it must be allowed.

(b) It is a fundamental right of a citizen to protest and brandishing is important to showcase


the tools, faux tools for safety, which put the food on everyone’s plate and are essential
symbolism for the protest and farmers will change the tactic and will now use fake tools to
remove the apprehension of danger, the authorities must allow them the permission.

(c) Tools will not be used for any purpose other than brandishing and the protest is peaceful.

(d) The rally will not occupy any public place and has no intention of threatening anyone;
therefore, the fundamental right to protest must be upheld.

 
Passage (Q.-Q.): Article 21 states that “No person shall be deprived of his life or personal
liberty except according to procedure established by law”. The term ‘life’ is interpreted in a
very wide sense; the inhibition against its deprivation extends to all those limbs and faculties
by which life is enjoyed and is available to both citizens and non-citizens. Hence, any form of
mutilation, amputation or the destruction of any other organ of the body is prohibited under
this Article. This is followed by Article 22, which immediately tells us exactly how the State
can take away this right to life and personal liberty. So, we find general protections in
Articles 22(1) to (3): every person arrested must be informed of the grounds of arrest and be
permitted to engage legal counsel of her choice [Article 22(1)], as well as be produced before
a magistrate within 24 hours of being detained [Article 22(2)]. This, interestingly, is followed
up by 22(3) which specifically takes away these rights in case of “enemy aliens” and
“persons arrested and detained under preventive detention laws”. The remainder of Article 22
– clauses (4) to (7): moves on to lay down the limits of preventive detention laws to prevent
their misuse and confer certain limited rights upon detenues i.e. detainees. Thus, ordinarily
preventive detention is capped at three months unless reviewed by an Advisory Board which
recommends otherwise [Article 22(4)], where the case is not of a category warranting special
treatment [Article 22(7)]. Detenues have a right to be informed of the grounds behind a
detention order as soon as may be [where disclosure is not contrary to public interest as per
Article 22(6), and also to make a representation against the order [Article 22(5)]. [Extracted
with minor revisions from ‘Preventive Justice Part 1 – The History behind Article 22’ by
Abhinav Sekhri, Indconlawphil.wordpress.com
https://indconlawphil.wordpress.com/2016/12/03/guest-post-preventive-justice-part-1-the-
history-behind[1]article-22/]  

. Johnny was arrested for robbery on 3rd June 2016 at 5pm. He was produced before a
magistrate on 4th June at 1pm who took cognizance of the matter and asked Johnny to be
presented before a judge for trial on 1st July. Johnny was poor and could not afford any
lawyer to fight his case so no one represented him in his case. Without a lawyer, Johnny
could not prove his innocence before the judge and was found guilty and imprisoned. Are his
fundamental rights being violated?

(a) Yes, Articles 22(1) and 22(2)

(b) Yes, Article 22(1)

(c) Yes, Article 22(5)

(d) Yes, Articles 22(2) and 22(5)

. Thomas Shelby is a student activist notorious for his speeches criticising the government.
He was taken into custody by the police without being informed of the grounds of his arrest
on 22nd March 2019 for his speech which was deemed violative of the ‘Public Safety Act’.
This Act permits arrest or detention of any person who uses hate speech against the
government in order to incite people, deeming it to be against public safety. He is produced
before a magistrate on 22nd May 2019. Which of the following fundamental rights are being
violated in this case?

(a) Article 22(2)


(b) Article 22(6)

(c) Both (a) and (b)

(d) None of the above

. Based on your reading of the passage, the protection against arrest and detention under
Article 22 is available to:

i. Indian Citizens

ii. Foreigners

iii. A person detained under a preventive detention law

iv. A Non-resident Indian

(a) i. and ii (b) ii. and iii (c) i., ii. and iv (d) iii. and iv

. Gian spat a chewing gum on the road and was immediately arrested by the police who told
him that he violated the ‘Cleanliness’ Act. He was not aware that it was illegal to chew gum
in the country and spitting it on the road was punishable. Is it a violation of Article 22?

(a) No, because he was informed about the grounds of his arrest

(b) Yes, because he was informed about the grounds of his arrest

(c) No, because it was his duty to be aware of the laws of the land

(d) Yes, because he was not aware that chewing gum was punishable in the country

. The countries of Liberalia and Leftistan share a border but have maintained peaceful
relations since their Partition. One day an unknowing villager from Leftistan crosses into
Liberalia by mistake. He was arrested by the Border Police that mistook him for a secret spy.
The Border Police kept torturing the villager so much so that his left ear became less
functional. Subsequently, it was found after an investigation that he was innocent. Can the
villager claim a violation of his right to life and personal liberty under Article 21?

(a) No, because the villager was an ‘enemy alien’

(b) No, because none of his organs were ‘mutilated’ or ‘amputated’ to constitute a violation
of Article 21

(c) Yes, because Article 21 is an inalienable right available to every person

(d) Yes, because Article 21 is an inalienable right available to every citizen


 

. Liberalia and Leftistan are at war. An enemy soldier from Liberalia is taken into custody by
the forces of Leftistan and kept in jail for 6 months. He is not given any opportunity to make
a representation. Is it a violation of his fundamental rights under under the laws of Leftistan?

(a) Yes, because Article 22 is available to both citizens and non-citizens

(b) No, because he does not have the protection of any fundamental rights

(c) No, because he is an ‘enemy alien’

(d) Both (b) and (c)

Passage (Q.-Q.): The distinction between kidnapping and abduction can often seem
perplexing at a first glance but the two are fundamentally different. In Kidnapping, the minor
is simply taken away. The means used may be innocent. In abduction, force, compulsion, or
deceitful means are used. Moreover, in the former, the consent of the person taken is
immaterial; while in the latter, the consent of the person moved can condone abduction.
Kidnapping from lawful guardianship under the Indian Penal Code can be understood as, a
person taking away or enticing a minor or a person of unsound mind, from his/her guardian
without their consent. The word ‘take’ doesn’t necessarily connote taking by force and is not
confined only to the use of force, actual or constructive. It merely means cause to go, to
escort, or to get possession. The mental attitude of the minor is not relevant in the course of
taking. On the other hand, the word ‘entice’ seems to involve the idea of inducement or
allurement by giving rise to hope or desire in the other. On the other hand, abduction,
includes the use of brute force or any deceitful means to take a person from one place to
another. It is defined under Section 3 of the Indian Penal Code, as, if a person compels
another person to go from one place, or induces some person to go from one place, then the
offence of abduction is committed. In a more restricted sense, it is also confined to the taking
of females for marriage, concubine, or prostitution. [Extracted with minor revisions from
‘Criminal Law: Kidnapping and Abduction’ by Kumar Writwick and Shivani Panda,
lexlife.in https://lexlife.in/2020/05/31/criminal-law-kidnapping-and-abduction/] . Eleanor was
born on 1st June 19. She decided to run away from her house and marry Chidi, her best
friend. She asked Chidi to pick her up from her house on 31st July 2013. She sat in his car
while he silently drove away.

(a) Chidi is liable for kidnapping because consent is immaterial in kidnapping

(b) Chidi is liable for abduction because he picked Eleanor from her house so that they could
get married

(c) Chidi is not liable for abduction

(d) Both Chidi and Eleanor are liable for abduction

. Jason was the 8yr old son of Michael. One day, when Michael was out for work, Jason was
playing alone in the park outside his house. A stranger came up to Jason and seeing that he
was alone, offered to play with him. This became a routine and the stranger started playing
with Jason every day when Michael was not at home. The two had almost become friends.
One of these days, he asked Jason to sit in his car and Jason agreed. When Michael came
back home, he was unable to find Jason.

(a) The stranger has not committed the offence of kidnapping

(b) The stranger has committed the offence of kidnapping

(c) The stranger has not committed the offence of kidnapping because he was friends with
Jason

(d) The stranger has committed the offence of kidnapping because Jason agreed to sit in his
car

. Blue Ivy was a 20yr old woman, suffering from a medical condition which caused her to
become unsound for three days after taking her medications. She was trying an
unconventional therapy, and this was a side effect of the medication. She took her medication
on Wednesday and went on a walk on Saturday with her mother. Her mother stopped to talk
to her friends, but Blue Ivy kept walking. A man came up to her, held her hand and walked
away with her.

(a) This is case of abduction because Blue Ivy was not a minor.

(b) This is a case of kidnapping because Blue Ivy was of sound mind

(c) This is a case of abduction because Blue Ivy was of unsound mind

(d) This is a case of kidnapping because Blue Ivy was of unsound mind.  

. Mala was coming back home from office when a van stopped in front of her. A group of 5
people pushed her into the van. She tried to resist but was unable to stop them. Mala
continued to scream to alert the people outside, but they drove the van in silence. Three hours
later, they dropped Mala in a locality two streets away from her neighbourhood.

(a) This is a case of abduction because they took Mala from one place to another

(b) This is not a case of abduction because they dropped her near her neighbourhood

(c) This is a case of abduction because they compelled Mala to come with them

(d) This is not a case of abduction because they did not use any force

. Shanaya and Ahaana were best friends. For Shanaya’s birthday, Ahaana surprised her with a
treasure hunt with clues hidden all around the city. If she found all the clues, Ahaana would
give her a special gift. Shanaya went from one place to another looking for clues for 5 hours
straight with Ahaana. Did Ahaana abduct Shanaya?

(a) Yes, because Ahaana induced Shanaya to look around for clues

(b) Yes, because Ahaana compelled Shanaya to look for clues

(c) No, because Ahaana did not take Shanaya from one place to another

(d) None of the above

. David was 16yrs old. One day, he was upset with his parents because they scolded him for
scoring bad marks. While his parents were away, his father’s friend Keith came to their
house. David said that he wanted to get away from his parents for some time and asked Keith
to help him so, Keith offered to take him to his house so he could play with their new dog.
Has Keith committed any offence?

(a) No, because he was a family friend

(b) No, because he was only trying to help David clear his head

(c) Yes, because he escorted David to his house

(d) Yes, because he agreed to help David

Passage (Q.-Q.): Tamil Nadu Gaming and Police Laws (Amendment) Ordinance 2020
amends the Tamil Nadu Gaming Act, 10 (TN Gaming Act) and extends its territorial scope to
cover the entire state of Tamil Nadu. The Ordinance comes in the wake of the observations of
the Madras High Court regarding the addictive nature of online games and its negative effect
on the masses. It has made the following key changes to the TN Gaming Act: Instruments of
gaming earlier only included any articles used or intended to be used as a subject or means of
gaming. The definition has now been expanded to specifically include items such as cards,
dice, gaming table or cloth, board, computers, computer system, computer network, computer
resource, any communication device, or any other article used or intended to be used as a
subject or means of gaming. It also includes electronic records that are used as registers or
records or evidence of any gaming. Further, instruments of gaming include proceeds of any
‘gaming’, including any online transfer of funds or transactions. Moreover, the definition of
gaming has been expanded to include wagering and betting in cyberspace. The Ordinance
introduces a new offence of wagering and betting in cyberspace. It criminalizes wagering or
betting in cyberspace using computers, computer system, computer network, computer
resource, any communication device or any other instrument of gaming, by playing ‘rummy’,
‘poker’ or ‘any other game’. For this purpose, wagering or betting is deemed to include
collection or soliciting of bets, and the receipt or distribution of winnings or prizes, including
through electronic transfer of funds. However, the Ordinance does not put a blanket ban on
offering online games. Online games involving skill continue to be exempted from the
application of the restrictions under the TN Gaming Act. [Extracted from ‘Tamil Nadu
promulgates ordinance to ban online gaming’ by Surbhi Kejriwal, Rishabh Bharadwaj and
Abhishek Rao in lexology.com https://www.lexology.com/library/detail.aspx?g=cf34354e-
d48f-4156-a9-dc]  

. Allie loved hosting gaming parties at her home. She invented a new game wherein every
player was supposed to throw coins in cups laid out on a table at different distances. The
players are also allowed to speculate and place bets on which cup will have the maximum
coins. The winner is given the entire prize money. Is this legal under the ordinance?

(a) Yes, because it is not wagering or betting

(b) Yes, because the prize money is not distributed through electronic transfer

(c) No, because wagering or betting is criminalized by the Ordinance

(d) No, because there is no evidence to prove that bets are being solicited

. Sujan plays a game called ‘Real Rummy Rambo’ online. It allows him to play rummy with
multiple players around the world. A unique feature of the game is that it allows a player to
enjoy the game without using any real money. The game allows players to bet using Rambo
coins which are allocated to the top performers of the game every day. IS Sujan committing
an offence by playing this game?

(a) Yes, because he is playing rummy

(b) Yes, because his data is being recorded in an online register

(c) No, because the game does not use any real money

(d) No, because he is not soliciting bets

. Prachi loves playing games online. She uses a website called Mega player which has
multiple games such as hidden object games, Russian roulette, poker, colouring games etc. Is
Mega player prohibited?

(a) Mega player is prohibited because it offers games like poker and Russian roulette

(b) Only some game from Mega player are prohibited

(c) Mega player is not prohibited because it does not involve any transfer of funds

(d) Mega prohibited is not prohibited because it is a free website.

. Based on your reading of the passage, which of the following could have prompted the
promulgation of such an ordinance?

(a) People end up incurring debts in connection with online gambling activities
(b) Online games involving betting and wagering can inculcate negative behaviours in the
youth

(c) There are instances of people dying by suicide when unable to repay these debts

(d) All of the above

. Although horse racing is a game of skill, it still involves people placing bets on different
horses and runners in the derby. Is it allowed under the given Ordinance?

(a) No, because it still involves betting

(b) Yes, because it is a game involving skill

(c) Horse Racing is allowed but betting on horses is not allowed

(d) None of the above

Passage (Q.-Q.): The right to protest peacefully is enshrined in the Indian Constitution under
Article 19(1)(b) which assures citizens the right to assemble peaceably and without arms. In
the case of Ramlila Maidan Incident vs. Home Secretary, Union of India, the Supreme Court
had stated, “Citizens have a fundamental right to assembly and peaceful protest which cannot
be taken away by an arbitrary executive or legislative action.” It was in Maneka Gandhi vs.
Union of India that Justice Bhagwati had said, “If democracy means government of the
people by the people, it is obvious that every citizen must be entitled to participate in the
democratic process and in order to enable him to intelligently exercise his rights of making a
choice, free & general discussion of public matters is absolutely essential.” At this point, it is
critical to note that all protests are legal only if they are non-violent and carried out with
appropriate permissions. Article 19(1)(3) says that this right is subject to “reasonable
restrictions” in the interest of public order. These reasonable restrictions include situations
wherein the security of the state is in jeopardy; the friendly relationship that we share with a
neighbouring country is at stake; public order is disturbed; there is contempt of court; or
India’s sovereignty and integrity are threatened.  22 of 35 In a recent matter on the scope of
right to protest, the Supreme Court held that public places cannot be occupied indefinitely.
The Court said that although Dissent and democracy go hand in hand, protests must be
carried out in a designated area. [Extracted from ‘Citizen Protests in India: Rights, Duties &
Permissions Needed’ by Vidya Raja, thebetterindia.com
https://www.thebetterindia.com/2061/how-to-protest-in-india-constitution-rights[1]right-
fundamental-duties-police-permission-nrc/]

. The Parliament of Schitt’s Creek passed a new law to support the fishermen of the town
with thumping majority in both the houses. This new law allowed the fishermen to directly
take up contract fishing with big corporates thus, eliminating the middlemen. Moreover, the
dispute resolution mechanism of the new law mandated any complainant to take up their
concerns with the Lok Adalat ‘without causing any ruckus to get their demands met in any
designated, public or government property’. Is this valid?
(a) No, because it was an arbitrary legislative action

(b) Yes, because the law is not arbitrary since it was passed with thumping majority in both
the houses

(c) No, because right to protest is fundamental in nature which cannot be taken away by the
Parliament arbitrarily

(d) Yes, because Parliament has the discretion to put reasonable restrictions on the right to
protest

. A majority of the fishermen were unhappy with this law because they wanted the
middlemen to stay for the additional support they provided. They immediately protested by
going on a strike and occupying the Parliament road till their voices were heard. Any
subsequent attempt at negotiating with the government also failed. Therefore, they decided to
continue to sit on a protest till the new law was repealed. It has now been 15 months, but they
continue to protest like it’s their first day. Can they do so?

(a) Yes, because they are not occupying a public space

(b) No, because they cannot occupy a public space indefinitely

(c) No, because right to protest in designated areas is a fundamental right

(d) Yes, because their protest is peaceful in nature

. The citizens of Atlantis were protesting a new government policy that increased the tax on
every citizen by 40% since December 2019. In January 2019, in lieu of a worldwide outbreak
of the SARS virus which spreads by contact, the government refused to allow more than 4
people at the protest site at a time. Is their right to protest being violated?

(a) No, because such a restriction is reasonable in nature

(b) Yes, because such a restriction is not reasonable in nature

(c) No, because such a restriction is not reasonable

(d) Yes, because such a restriction is reasonable

. The notorious terrorist group ‘SISI’ had released a video message on the internet claiming
that they had planted a bomb in a crowded space. The government immediately issued an
advisory preventing any assembly or gathering of people and forbade people from leaving
their houses till further notice. A citizen X claims that this a violation of Article 19(3).

(a) Yes, this is a violation of the fundamental right to assembly

(b) Yes, this is a violation of Article 19(3) because it is an arbitrary exercise of power

(c) No, it is not a violation of Article 19(3) because public order is disturbed
(d) No, it is not a violation of Article 19(3) because the security of the state is in danger

. A group of students were protesting against an arbitrary fee hike by a government


university. The matter was already being heard and the Court ordered the students to clear the
site of protest and, the university was directed to resolve the matter by holding consultations
with the students. Having lost no faith in the administration, some students continued to sit
for protest till the fee hike was revoked. Can they do so?

(a) No, because the matter was already being heard in the Court

(b) No, because this would amount to contempt of Court

(c) Yes, because no restriction was imposed on their right to protest

(d) Yes, because Article 19 is a fundamental right  

. In light of the government’s failure to curb the rising number of crimes against women and
holding the perpetrators of such crimes accountable for their actions, a large mass of outraged
people went on protest at India Gate. In order to boost their morale, some protestors started
singing songs on unity and fraternity of the people. Fearing that a prolonged protest would
turn violent, the government ordered the police to fire rubber bullets to displace the
protestors. Is this a violation of their fundamental rights?

(a) No, because singing songs of unity and fraternity could have incited the protestors

(b) No, because the protest was a peaceful assembly of people

(c) Yes, because the protest was a peaceful assembly of people

(d) Yes, because right to protest is a fundamental right without any restrictions

Passage (Q.-Q.): The Uttar Pradesh Prohibition of Unlawful Conversion of Religion


Ordinance, 2020, was promulgated on November 27. The ordinance makes religious
conversion a cognisable and non-bailable offence, inviting penalties of up to 10 years in
prison if found to be affected for marriage or through misrepresentation, force, undue
influence, coercion, allurement or other allegedly fraudulent means. A first information report
(FIR) in this regard can be lodged by the “aggrieved person, his/her parents, siblings, or any
other person who is related to him/her by blood, marriage or adoption”, as per the Ordinance.
It says any marriage done with the sole purpose of “unlawful conversion or vice-versa” by a
man of one religion with a woman of another religion, “either by converting himself/herself
before or after marriage, or by converting the woman before or after shall be declared void”.
Further, the burden to prove that a conversion was not done through misrepresentation, force,
undue influence, coercion, allurement, fraudulent means or for marriage would be on the
person who caused the conversion or the person who facilitated it. [Extracted with revisions
from ‘U.P. Governor promulgates ordinance on unlawful conversion’ by Omar Rashid, The
Hindu https://www.thehindu.com/news/national/other-states/up-governor-gives-assent-
to[1]ordinance-on-unlawful-conversion/article3319.ece]

. Farida, a follower of Islam fell in love with Hemant, a devout Hindu. Although, their
families were not happy with their union, the two decided to marry each other. After
marriage, Hemant while in all his senses, decided to convert to Islam. Subsequently,
Hemant’s parents lodged an FIR under this ordinance. Is Hemant’s act punishable?

(a) No, because only the conversion of women after marriage is criminalized

(b) No, because he converted after marriage not before

(c) No, because the religious conversion was not the sole purpose of their marriage

(d) No, because Farida and Hemant were genuinely in love

. Murad married Sybil, who was a Christian. After marriage, Murad told Sybil that if she does
not convert to his religion of Maradonianism and change her name to Safeena, then he will
hurt her. He offered to facilitate the conversion process. Sybil converts to Maradonianism. Is
the conversion illegal?

(a) Yes, because Sybil converted out of her own will

(b) Yes, because Sybil converted due to coercion

(c) No, because Sybil did not convert due to coercion

(d) No, because Sybil converted due to undue influence since Murad was her husband

. Sybil’s father was very upset that he would now have to call his daughter Safeena because
he gave her the name ‘Sybil’. Her brother John lodged an FIR with the police contesting this
conversion. Murad disagrees. Who has to prove that the conversion was valid?

(a) Sybil, because she was the one who converted

(b) John, because he contested the conversion

(c) Police, because it is their duty to investigate in this regard

(d) Murad, because he made Sybil convert.  

 
. Soham told Moeena and her family that he will bear all the expenses of Moeena’s family for
the next 50 years if Moeena converts to his religion. Since her family was very poor and in
desperate need of financial help, she offered to convert her religion. Is this lawful?

(a) No, because Soham allured Moeena to convert

(b) No, because Soham convicted misrepresentation for conversion

(c) No, because Soham only married her to convert her

(d) No, because Soham coerced Moeena

17
Passage (Q.68 – Q.): Mid-August, the Consumer Protection Act, 2019 (2019 Act) received
Presidential assent and came into effect. Notably, the 2019 Act, repeals the previous
consumer protection legislation which had been in effect since 19 (19 Act). While using the
same phrase in its preamble, “to provide for better protection of the interests of consumers”;
the new act has substantially enhanced the scope of protection afforded to consumers, by
bringing within its purview advertising claims, endorsements and product liability. The
definition of “consumer” under the 2019 Act includes those who make purchases online.
Endorsement of goods and services, normally done by celebrities, are also covered within
the ambit of the 2019 Act. In fact, an additional onus has been placed on endorsers, apart
from manufacturers and service providers, to prevent false or misleading advertisements. In
contrast to the 19 Act, the definition of “goods” has been amended to include “food” as
defined in the Food Safety and Standards Act, 2006. A significant addition to the 2019 Act is
the introduction of “product liability” whereby manufacturers and sellers of products or
services have been made responsible to compensate for any harm caused to a consumer by
defective products, manufactured or sold, or for deficiency in services. The definition of
“unfair trade practices” has been enlarged to include electronic advertising which is
misleading, as well as refusing to take back or withdraw defective goods, or to withdraw or
discontinue deficient services, and to refund the consideration within the period stipulated or
in the absence of such stipulation, within a period of thirty days. All these changes signify an
attempt to create more transparency in the marketplace, through legislative protection, with a
view to ensure that consumer interests are above all else. [Extracted from ‘Consumer
Protection Act 2019: Enhancing Consumer Rights’ by SatvikVarma from Bar & Bench
https://www.barandbench.com/columns/consumer-protection-act-2019-enhancing-
consumer[1]rights#:~:text=Bar%20%26%20Bench&text=The%202019%20Consumer
%20Protection%20Act,without% 20proposing%20adequate%20administrative
%20safeguards.]

68. KiratVohli is a famous cricketer who endorses the energy drink ‘Bluebull’. The drink
claims to increase productivity, concentration and instantly uplift the consumer’s mood. X, an
avid fan of Kirat buys the drink and does not experience any of the claimed results. Does he
have a case under the given Act?

(a) No, because he did not consume the Bluebull for long enough to experience its benefits

(b) No, because he does not fit the definition of ‘consumer’ under the Act

(c) Yes, because he was lied to


(d) Yes, because the product was fake

. In the same case, if X bought Bluebull from Future Retail, a large wholesale shop after
watching Kirat’s advertisement on television, who is liable under the Act?

(a) Kirat and Bluebull Co.

(b) Kirat and Future Retail

(c) Bluebull Co.

(d) Kirat, Future Retail and Bluebull Co.

. Birbal orders a veg biryani from an online food platform called Trimato. When he received
the order, he found that the biryani had pieces of chicken in it. He asked Trimato for a refund
but they refused. Can Birbal sue Trimato?

(a) No, because Biryani is a food item which is not covered under the Act

(b) Yes, because online food platforms are liable to give him a refund for the deficient
services

(c) Yes, because it is Trimato’s fault that they gave him the wrong order

(d) No, because online food platforms do not fall in the purview of the Act

. To seek admission in a medical coaching institute, Farida was made to deposit a lump sum
fee for two years within the first six months. Farida decided to leave the course midway
because the quality of services provided at the institute was deficient and subpar. She asked
for a refund of the remaining amount. The institute refused. Based on your reading of the
passage, can the institute be sued?

(a) Farida cannot sue the institute because education is not a service

(b) Farida can sue the institute because they misled her

(c) Farida cannot sue the institute because a student is not a consumer

(d) Farida can sue the institute for a refund

. Donna and Steve were at a café in Glasgow. Steve ordered and paid for her drink. The
cafe purchased the product from the manufacturer. The ginger beer came in a Dark bottle,
and the contents were not visible from the outside. Donna drank some of the contents and
Steve lifted the bottle to pour the remainder of the ginger beer into the tumbler. The remains
of a decomposed snail dropped out of the bottle into the tumbler. Who is liable?

(a) No one
(b) The café

(c) The café and manufacturer

(d) The manufacturer

. A Builder ‘ELF’ launched a residential project in Gurunagar. H, a Flat Purchaser entered


into an Agreement with ELF to purchase an apartment in the said project. ELF was to make
all efforts to apply for the Occupancy Certificate within 39 months from the date of
agreement and offer possession of the flat to H. ELF failed to apply for the Occupancy
Certificate in the given time and also did not allow H a refund on his payment. Subsequently
H approached the National Consumer Forum. Based on your reading of the passage, what
do you think was held by the Court?

(a) ELF made deficiency of service by not applying for the Certificate in the requisite time

(b) H was entitled to seek refund of the money deposited along with compensation.

(c) ELF’s conduct constituted an ‘unfair trade practices’ under the Act

(d) All of the above

Passage (Q. – Q.): The anti-defection law seeks to provide a stable government by ensuring
that legislators do not switch sides. ‘Aaya Ram Gaya Ram’ was a phrase that became
popular in Indian politics after a Haryana MLA Gaya Lal changed his party thrice within the
same day in 19. The anti-defection law sought to prevent such political defections which may
be due to reward of office or other similar considerations. The Tenth Schedule was inserted
in the Constitution in 19. It lays down the process by which legislators may be disqualified on
grounds of defection by the Presiding Officer of a legislature based on a petition by any other
member of the House. A legislator is deemed to have defected if he either voluntarily gives
up the membership of his party or disobeys the directives of the party leadership on a vote.
This implies that a legislator defying the party whip on any issue can lose his membership of
the House. The law applies to both Parliament and state assemblies. However, the law is not
without exceptions. Legislators may change their party without the risk of disqualification in
certain circumstances. The law allows a party to merge with or into another party provided
that at least two-thirds of its legislators are in favour of the merger. In such a scenario,
neither the members who decide to merge, nor the ones who stay with the original party will
face disqualification. The phrase ‘Voluntarily gives up his membership’ has a wider
connotation than resignation. The law provides for a member to be disqualified if he
‘voluntarily gives up his membership’. However, the Supreme Court has interpreted that in
the absence of a formal resignation by the member, the giving up of membership can be
inferred by his conduct. [Extracted with minor revisions from ‘The Anti-Defection Law
Explained’ by Vibhor Relhan in prsindia.org https://www.prsindia.org/theprsblog/anti-
defection-law-explained]

. Jayanti Lal is a minister of the Dilli Janta Party (DJP). The DJP brings out a bill which
makes it difficult for certain minorities to acquire citizenship. When the bill is up for voting,
Jayanti Lal votes against the law. Next day, he doesn’t come to the Parliament. Has he
defected?

(a) Yes, because he defied the party by voting against the bill
(b) No, because as a legislator he has a choice in how to exercise his vote

(c) Yes, because he party policy is above his personal vote

(d) No, because he was never asked to vote in favour of the bill only

. In the same situation, the party whip has asked the members of DJP to support this historic
bill that would strengthen the national character of their country. At the time of casting vote,
Jayanti Lal abstains. Can he be disqualified?

(a) Yes, because abstention amounts to violation of party policy

(b) Yes, because abstention amounts to defiance of the party whip

(c) Yes, because abstention is not allowed in the house

(d) Yes, because abstention is illegal

. Sahul Sandhi is a member of the Indian National Party (INP). He asks Tony to leave DJP
and offers him a chance to contest elections from his constituency of Varanasi from INP.
Tony offers a resignation letter to the DJP head. DJP refuses to accept the resignation letter.
Tony starts attending INP meetings while still representing his old constituency in the
Parliament. Is he in violation of the anti-defection law?

(a) Yes, because he started attending INP meetings even though his resignation from DJP
was not accepted

(b) Yes, because he started attending INP meetings before the next elections

(c) Yes, because his conduct shows that he has voluntarily given up the membership of his
party

(d) Yes, because the act of sending a resignation shows that he likes INP better

. What could be the possible drawbacks of the anti-defection law?

(a) Legislators are encouraged to change their parties frequently

(b) Legislators cannot change their policy even if the party policy goes against the concerns
of the voters in their constituency

(c) It makes legislators a tool in the hands of the Speaker of the house

(d) Legislators are forced to keep quiet on important issues and cannot give an opinion on
any matter in party meets

 
 

Passage (Q. – Q.): In a recent matter on Workmen’s Compensation, an application for


compensation was filed by mother of the deceased, who worked as a lorry driver. The
applicant contended that when the lorry reached Surat, the deceased felt severe stomach
pain and he was rushed to the hospital for treatment, where he died on the same day. The
claim for compensation was made after seven years. A separate delay petition was also filed
along with claim which pleaded that the delay was because the applicants were illiterate,
sick, poor and had to depend on intermediaries, who did not give proper advice. The
Commissioner, while condoning the delay held that the applicant had a legal claim for
compensation and accordingly, passed an order for compensation of Rs 4,11,0 with interest
against the respondents. Subsequently, the respondents filed for an appeal. It was submitted
that according to the Employee’s Compensation Act, 13 the claim for compensation ought to
be filed within two years of the cause of action. In appeal, the Court observed that the
Commissioner was not justified in condoning of delay and there must be a sufficient cause
and evidence to do so. More importantly, on the question of cause of death, the Court
noticed that in the post-mortem report, the final cause of death was shown as food
poisoning. Hence, considering this evidence, it was held that such death could not be said to
be causally connected with the work in which the deceased was engaged. The Court stated
that, unless stomach pain had causal connection or association with the work in which the
deceased was employed, it cannot be reckoned to be an accident to fasten liability on the
employer for payment of compensation. Hence, the decision was ordered to be set aside. 16
of 36 [Extracted with revisions from ‘Unless injury complained of had connection with
employment in which deceased was engaged, he cannot make a legal claim for
compensation; HC reiterates legal position’ published 29 Dec 2020 on scconline.com,
https://www.scconline.com/blog/post/2020/12/29/ker-hc-unless-injury-complained-of-had-
connection-with[1]employment-in-which-deceased-was-engaged-he-cannot-make-a-legal-
claim-for-compensation-hc[1]reiterates-legal-position/]

. Soham had been employed with a mining company since 12 years. His job involved
spending more than 12 hours a day deep into the Earth’s surface, mining for precious
minerals. One day when he was on the job, he suddenly collapsed and died on the spot. His
co-workers revealed that he had been coughing a lot since the past two weeks. Can a legal
claim for compensation be made?

(a) No, because his death has no direct connection with his employment

(b) Yes, because mining is a harmful occupation which can cause long term health
consequences

(c) No, because he had been coughing for more than two weeks which indicates he may
have had some other illness

(d) Yes, because the coughing can be attributed to spending so much of time in mines which
may have caused his death

 
. Soham’s post mortem report revealed that his death was due to a terminal lung disease
whose symptoms include coughing. As per the doctor, the cause of such disease is usually
‘long-term exposure to harmful chemical fumes and mineral dust’. Is the Company liable for
payment of compensation?

(a) Yes, because his terminal lung diseases couldn’t be attributed to mining

(b) Yes, because his death could be attributed to long-term exposure to such compounds

(c) Yes, because mining is an occupation which involves ‘long-term exposure to harmful
chemical fumes and mineral dust’

(d) Yes, because his illness had no connection with the employment he was engaged in

. X was a daily wage labourer employed in a construction site in Delhi since 1st June 2014.
One day, while on the job, a large iron rod accidentally fell from an overhead lever from an
adjoining construction site and severely injured X’s leg. He was rushed to the hospital and in
order to save the remaining body from contracting infection, an amputation surgery was
performed on 9th June 2017 which was paid for by the Company. X did not turn up for work
from the next day. Is X eligible to be paid compensation?

(a) No, because his leg injury was not related to the construction work that he was employed
in

(b) Yes, because his leg injury no had connection with his employment

(c) Yes, because his injury was caused due to his nature of work

(d) Yes, because his injury was associated with the work he was engaged in

. X stayed in the hospital for 6 months after the surgery. Even after leaving the hospital, he
had to undergo physiotherapy and other therapies to get used to walking with one leg. He
retained all the reports and receipts of the same. X did not receive any money from
anywhere and had no friends or family who could take care of him. He decided to file a claim
for compensation against the company on 13th June 2019. Is the Company liable to pay?

(a) Yes, because the injury was causally connected to X’s work

(b) Yes, because there was a sufficient cause of delay

(c) No, because the delay in filing the claim cannot be condoned

(d) No, because there was no sufficient cause of delay

. Mohan is a traffic police officer. His occupation requires him to manage traffic while
standing in the centre of a busy road. The road is a hotspot for speeding and rash drivers so
Mohan has to be extra cautious and ensure public safety along with traffic management.
Mohan was walking back home along the road after his shift and met with an accident. Can
he make a legal claim for compensation?

(a) Yes, because he met with an accident at the road where he was stationed

(b) Yes, because his accident could be attributed to his employment as a traffic police officer

(c) No, because he met with an accident after his shift was over and not while working

(d) No, because the traffic police department is not liable for his accident

Passage (Q. – Q.):Article 21 which guarantees personal liberty and protects against
exploitation, also includes the Right to Shelter as an implied fundamental right. The debate
around 'Right to shelter' has always been tabled in context with the poor or the vulnerable
class, because they are mostly exploited and neglected by the stake holders. Recently, the
Karnataka High Court directed the State to reconstruct the huts, which were burnt due to
misinformation. Poor migrant labours were evicted from their native shelters and the
concerned huts were demolished. This direction is not only a positive ray for those dwellers,
but also another brick into the 'Shelter[1]jurisprudence'. The observations made by the
Supreme Court in Chameli Singh v. State of U.P, is pertinent to note, which was also
referred by the Karnataka High Court during the course of hearing. The Supreme Court had
rightly observed that "Shelter for a human being, therefore, is not a mere protection of his life
and limb. It is a home where he has opportunities to grow physically, mentally, intellectually
and spiritually. Right to shelter, therefore, includes adequate living space, safe and decent
structure, clean and decent surroundings, sufficient light, pure air and water, electricity,
sanitation and other civic amenities like roads etc. so as to have easy access to his daily
avocation. In Oliga Tellis v. Bombay Municipal Corporation, the court observed that "An
equally important facet of that right is the right to livelihood because, no person can live
without the means of living, that is, the means of livelihood. Deprive a person of his right to
livelihood and you shall have deprived him of his life.” [Extracted with revisions from
‘Reconstructing The “Right to Shelter”' by Areeb Uddin, published 14 Dec 2020 in
livelaw.com https://www.livelaw.in/columns/shelter-right-to-shelter-karnataka-high-court-
article-21-shelter[1]jurisprudence-supreme-court-jhuggies-1215]

. The government placed 20 women in a shelter who were rescued from a trafficking racket.
The owner of the shelter had to conduct some repairs in the main building so he moved the
women to the basement of the building. The basement was a dark place with no electricity,
windows or washroom. Is their right to life being violated?

(a) No, because they have already been provided with shelter

(b) No, because the basement is temporary and the house provides decent living conditions

(c) Yes, because they do not have adequate living conditions

(d) Yes, because they are being deprived of basic sanitation facilities

. Migrant labourers from a distant state had settled in the town of Singapura. Over the course
of 10 years, as the population grew, a slum colony developed in a public park complete with
a makeshift school and hospital. An eviction notice was passed and the labourers were
asked to vacate the park. They were not given any alternative housing. Is it a violation of
Article 21?

(a) Yes, because right to life includes right to shelter

(b) Yes, because they cannot be asked to vacate without being provided with alternative
shelter

(c) Yes, because they had been living there for more than 10 years so it was their legal
dwelling

(d) No, because they were encroaching on a public space

. In order to get the park vacated faster, the Municipality offered the slum dwellers housing in
a new colony being developed by the government of Singapura for below poverty line
families. The slum dwellers refused to move because most of them were employed in the
construction areas near their present dwellings, while other worked as house help in the
residential areas surrounding the park. The new site was in a remote, undeveloped part of
the town. Can the Municipality force them to vacate the park?

(a) Yes, because their right to life is not being violated since they have been offered
alternative housing

(b) Yes, because they are occupying a public space and creating nuisance

(c) No, because their right to shelter is being violated since they will have to move from their
established dwellings

(d) No, because their right to livelihood will be violated

. A government NGO provided safe homes for children in conflict with the law or those in
need of protection. Skill training was provided to make them self-dependent and fit to do
small tasks like stitching, cooking etc. However, over time the number of child residents in
the house increased. So, those residents who were above 18 years of age and had
completed basic skills training were asked to vacate to make space. Is it a violation of their
right to shelter?

(a) No, because they have been imparted with skills that enable them to earn and live
adequately

(b) No, because right to life exists to protect against exploitation

(c) No, because their personal liberty is being violated

(d) Yes, because if they are asked to vacate they will not have a roof over their heads

. A dam was being constructed in a village ABC to generate hydro-electricity. The


government ordered the farmers to vacate their lands near the river for the construction of
the dam. The farmers were offered compensation for their lands at market rates. The
farmers challenged this saying that their river lands are important for agriculture which is
their only source of income. Can they do so?

(a) Yes, because their right to livelihood is being violated

(b) Yes, because their right to shelter is being violated

(c) Yes, because proving compensation at market rates is not enough

(d) No, because their right to life is not being violated

. Joginder, a habitual petty offender was sentenced to spend time in a juvenile correction
home. The juvenile home had a triple room occupancy system and shared bathrooms. There
was also a spacious library which had computers. Joginder asked for a personal laptop to
attend a free online educational course but, the same was refused. He says that it is a
violation of his right to shelter

(a) No, because he has a home to protect his life and limb

(b) Yes, because a personal laptop is necessary for his right to education

(c) No, because he has adequate space to grow physically, mentally and spiritually

(d) Yes, because a personal laptop is necessary for growing mentally and intellectually .

Passage (Q. – Q.): The commercialization of traditional and modern sports has paved the
way for online platforms to increase engagement in these activities. There are a host of
online games that are available, from money games such as online poker and fantasy
football or cricket, to casual games such as Candy Crush and Temple Run, and then of
course, e-sports such as Counter-Strike or FIFA. The rise of fantasy sports raises questions
about the legality of such activity, that is, whether it would amount to gambling or betting.
Under the Constitution of India, gambling and betting are State subjects, with each state
forming its own laws. Some states, such as Assam, Orissa, and Telangana, prohibit any
gaming activity for money. Other states permit gambling with respect to games of skill. The
expressions “gambling” and “game of skill” have been dealt with by a Constitution Bench of
the Supreme Court in the case of Dr. KR Lakshmanan v. State of Tamil Nadu & Anr.
Following the earlier decisions in RMD Chamarbaugwala & Anr. v. Union of India & Anr. and
State of Andhra Pradesh v K Satyanarayana & Ors, the Court held that gambling is the
payment of consideration for a chance to win a prize. A game may be of chance or of skill, or
a combination of both elements. A game of chance is determined entirely or largely by luck,
whereas a game of skill depends on the players superior knowledge, experience and
adroitness. The Court concluded that though an element of chance exists in a skill game, the
element of skill predominates over the element of chance.The Court authoritatively held that
a ‘game of chance’ would fall within the vice of gambling and is prohibited as per state law,
whereas a ‘game of skill’ is distinguishable from gambling and enjoys protection under
Article 19(1) (g) of the Constitution. Source – [Extracted, with edits and revisions, from:
“Legal Matrix of Online Fantasy sports in India”, by Sachit Jolly & Priyanka MP, Bar and
Bench, dated 24.11.2020] https://www.barandbench.com/columns/legal-matrix-of-online-
fantasy-sports-in-india-2
. Rohit wants to develop a Gaming App based on the concept of Fantasy Football which
would involve picking players from a real-life match from both sides and forming a team, if
the team wins then the user wins cash prizes amounting to more than INR 1 crore. Is the
App that Rohit wants to develop legal?

(a) Yes, since the App being developed involves a Football game which is not a game of
chance.

(b) No, since this App is based on a concept which allows its users to Gamble huge amounts
of cash amounting upto 1 crore.

(c) Yes, since the App allows gives freedom to the user to make his own team.

(d) No, since the App is based on the concept of gaming for money which is prohibited in
several states.

. Preeti is a part of an exclusive club which places bets on Horse Racing. Rita also wants to
join the club but she suspects that the club is not legal. Is it legal to form such clubs in India
which places bets on Horse Racing?

(a) No, it is illegal since betting for money is strictly prohibited under Indian law.

(b) Yes, it is legal since betting is a state subject and some states allow betting activity.

(c) No, it is illegal since betting on Horses for money involves a high degree of luck.

(d) Yes, it is legal since betting on horses is not illegal.

. Riya and Rohan decide to play a friendly game of Rummy for INR 200, Riya is the
undefeated champion of Rummy since she uses certain memory tricks. Is the game of
Rummy for stakes between Riya and Rohan legal?

(a) No, since card games like Rummy depend highly on luck.

(b) Yes, since Riya and Rohan were part of a friendly game and the amount involved was
not high.

(c) No, since several states like Telangana, Assam, Orissa etc have prohibited playing
games for money which makes it illegal.

(d) Yes, since playing Rummy for stakes in not prohibited by law.

 . Raghav develops an App ‘Casino – 8’ through this App the users can play casino game
like Slots digitally and also bet for money. The user spins the reels having different type of
symbols and wins if he/she gets the combination of winning symbols on the reel. Is the app
being developed by Raghav legal in India?

(a) Yes, since the app being developed by Raghav is based on online gaming concept.
(b) No, since the users of the app are allowed to place bets for money.

(c) Yes, since the game of slots is not totally dependent of chance.

(d) No, since casino games are not regulated and not allowed on India.

Passage (Q. – Q.):For Information like money spent on advertisements, accounts blocked,
audience targeted and other stats, can the Twitter handle of let’s say the Prime Minster –
which is not independent of his prime ministry – be considered a ‘body’ under the definition
of ‘public authority’ in the RTI Act? Krishna Iyer, J. has defined ‘authority’ from the Law
Lexicon by P.RamnathIyer to say that an “Authority is a body having jurisdiction in certain
matters of a public nature”. A Governments Twitter account has the ability to affect the rights
and liabilities of a user which may have far-reaching consequences, and thus be considered
an authority. An authority can also be an ‘instrumentality or agency’ of the State, provided
there is governmental ownership and/or control. In this case, the authority in the form of the
Governmental Twitter account is owned and controlled by the Government. Thus, the
Governmental account can also be considered as an ‘instrumentality or agency/ of the State.
A public authority, as has been held by the Supreme Court as a body which has public or
statutory duties to perform and which performs those duties and carries out its transactions
for the benefits of the public and not for private profit. A public authority under the RTI Act,
2005 has been defined as follows – “Public authority” means any authority or body or
institution of self-government established or constituted a. by or under the Constitution; b. by
any other law made by Parliament; c. by any other law made by State Legislature; d. by
notification issued or order made by the appropriate Government, 20 of 36 And includes any
– - body owned, controlled or substantially financed ; - (ii) non – Government organisation
substantially financed, Directly or indirectly by funds provided by the appropriate
Government” Therefore the definition can be extended to include an official Twitter account
that is owned and controlled by the Government and is used for discharging public functions.
Source – [Extracted, with edits and revisions, from: Can a Twitter Account of the government
be a ‘Public Authority’ under the RTI Act? By RaghavAhooja, Live Law, dated 26.12.2020]
https://www.livelaw.in/columns/government-twitter-account-rti-public-authority-16

. Delhi Cricket Association (DCA) is an autonomous association formed by retired State and
National level Cricket players with the aim to support upcoming talented cricket players. The
role of Government of NCT of Delhi is limited to providing assistance in travel, transportation
of sportsmen and sports managers for matches across the country. Rajat files an RTI
against DCA enquiring into the source of funding of the Association and the salary of Sports
Managers and Coaches. The RTI filed by Rajat was rejected and the reason cited for
rejection was that DCA does not fall under the category of a Public Authority under the RTI
Act. Rajat decides to file an appeal against this decision, will he be successful in his Appeal?

(a) Yes, DCA holds a preeminent position and has control over the selection of players to
compete at National level which affects the rights of the players.

(b) No, since DCA is an autonomous association which makes it fall outside the purview of
RTI Act.

(c) No, since DCA was not formed by an act of the Legislature or of the State or Central
Government.

(d) Yes, since DCA is a Public Authority under RTI Act.


 

. ‘Ray of Hope’ is a NGO which helps and support the Senior Citizens in India and also runs
an autonomous Elder Care Home. The State Government, as part of its initiative to support
the Senior Citizens, provides subsidies in Electricity and Water bill to Elder Care Homes
including the Elder Care Home run by Ray of Hope. Will the NGO - Ray of Hope fall under
the purview of the RTI Act?

(a) Yes, as the Elder Care Home run by it receives funding from the State Government in
form of Subsidies.

(b) No, as the Elder Care Home is run autonomously and there is no control of the
Government.

(c) Yes, as Ray of Hope is a NGO receiving indirect funding from State Government which
makes it fall under the definition of Public Authority under the RTI Act.

(d) No, as Ray of Hope does not fulfil the essentials to be categorized as Public Authority
under RTI Act.

. The Central Government decided to undertake various initiatives in order to provide better
Healthcare facilities to its citizens. As part of this initiative the Government allotted 150 acres
of land at a prime location in Delhi to an autonomous trust for construction of a state of the
art hospital with the best medical amenities. The Trust is solely responsible for the
administrative and policy decisions relating to the Hospital. The salaries of the staff and the
running cost of the Hospital is funded solely by the Trust. The Central Government provides
zero funding for running the Hospital and has no control over its management. Will the
Hospital fall under the category of a Public Authority under the RTI Act?

(a) It is a Public Authority since the hospital is set up as a part of a government initiative.

(b) It is not a Public Authority since the hospital is run by an autonomous trust.

(c) It is not a Public Authority since the government does not provide funding and has no
control over the management of the Hospital.

(d) It is a Public Authority since the trust has received funding from the Government.

. Isha is part of a Credit Co-operative Society which provides loans to its members at low
interest rates. The Co-operative society is registered under the Maharashtra Cooperative
Societies Act, 19. As per the provisions of the act, the authority established under the act
has the power to conduct audits and enquire into the irregularities of a Co-operative society
registered under the act. The Co-operative Society does not receive any funding from the
government and there is no say of the government in its administration. Is the Co-operative
Society bound to disclose information under the RTI Act?

(a) Yes, since the Co-operative Society is bound to disclose information for audits and
enquiry by the authority.

(b) No, since the government does not provide any funding directly or indirectly.

(c) No, since Co-operative Societies do not form a part of the definition of Public Authority
under the RTI act.
(d) Yes, since the Co-operative Society is registered under the Maharashtra Cooperative
Societies Act, 19.

Passage (Q. – Q.): Bailment as defined in section 148 of the Indian contract act 18 is the
delivery of goods by one person to another for some specific purpose, upon a contract that
these goods are to be returned when the specific purpose is complete. The person delivering
the goods is known as bailor and the person to whom goods are delivered is known as
bailee. However, if the owner continues to maintain control over the goods, there is no
bailment. Essentials of Contract of Bailment: (1) The existence of a valid contract is a
foremost condition in bailment which implies that goods are to be returned when the purpose
is fulfilled. (2) The whole concept of bailment revolves around the fact that the goods are
delivered for a temporary period and bailee cannot have permanent possession. Delivery of
goods can be done through actual delivery or through constructive delivery which means
that doing something which has the effect of putting the goods in possession of bailee or any
other person authorized by him. (3) The bailee is bound to return the goods to bailor after the
purpose for which it was taken is over. If the person is not returning the goods then it will not
be bailment. Bailee’s Liabilities: (1) Care to be taken by the bailee (Section 151 and 152) -
The bailee is bound to take as much care of goods bailed to him as a man of ordinary
prudence would have under similar circumstances and therefore, he will not be liable for any
loss, destruction or deterioration of the thing bailed if he has taken care. (2) The duty of the
bailee to return the bailed goods (Section 1 and 1) - Bailee is under the duty to return or
deliver goods according to the bailor’s direction as soon as the time for which goods were
bailed has expired. (3) Bailee’s duty to deliver increase profit from the bailed goods to the
bailor (Section 1) - In the absence of any agreement, bailee is bound to deliver to the bailor
any increase in profit or any benefit which may have accrued from the goods bailed. [Source:
https://blog.ipleaders.in/what-is-the-contract-of-bailment/ ]

. Ram was running home for it was raining. He happened to meet Shyam on the way who
offered him an umbrella. Ram took the umbrella and kept it at his home. Decide

(a) This is a contract of bailment as goods have been given for a specific purpose.

(b) This is not a contract of bailment as goods have not been given for a specific purpose

(c) Shyam merely offered help to Ram. He did not intend to enter into a contract

(d) As essentials of a contract are not fulfilled, this cannot be construed to be a contract of
bailment.

. Please refer to the facts above. Ram upon returning the umbrella offered Shyam some
money as reward for his help. Shyam accepted the same. Decide

(a) This is a contract for bailment for all essentials of a contact are now fulfilled

(b) This is not a contract of bailment for both parties only intended to help each other

(c) This is a contract of bailment for goods were delivered for a specific purpose
(d) This is not a contract of bailment. Essentials of a valid contract have not been fulfilled.

. Radhey had given Raman a lighter when he needed to light a cigarette. When Raman went
to give the lighter back, Radhey asked Raman to get its handle repaired. Decide

(a) Radhey can do so since past consideration is valid consideration.

(b) Radhey cannot do so for a contract of bailment requires a bailee to supply goods in the
same condition as they were taken in

(c) Radhey can ask for the same. Raman got help and he needs to pay consideration for the
help.

(d) Radhey cannot ask for the same as both of them are friends.

. Upon giving a football to Jeevan to play, Raone asked him to get a puncture repaired and
give it back. Jeevan used to small repairs on his own usually. He also tried to fix the
puncture and damaged the same. Decide

(a) Jeevan is liable for the deterioration as he did not take reasonable care of the goods

(b) Jeevan is not liable for deterioration of the football. He took care of the football like he did
with all other things of his.

(c) Jeevan is liable for deterioration. He did not fulfill his bailee duties properly

(d) Jeevan is not a bailee as per the fact situation. Thus, he is not liable

. Please refer to the facts above. Jeevan went to the repair shop and asked the owner to
repair the puncture in the football. However, the cloth of the football was also torn and
Jeevan got that repaired too. Decide

(a) Raone must pay for the cloth repair too as Jeevan paid for it out of his own pocket

(b) Raone may not pay for the cloth repair as Jeevan unauthorisedly used the goods

(c) Raone must pay for the improved football as he only had instructed Jeevan to get it
repaired.

(d) Raone may not pay for the cloth for repairing the cloth was not part of the original
contract

Passage (Q. – Q.): The Delhi High Court refused to entertain a plea challenging the domicile
reservation in National Law School University of India University, Bangalore. The plea
comes after the Karnataka state government recently passed a state amendment allowing
for 25% horizontal reservation for Karnataka students in NLS. This plea was massively
protested against by the national media and the public in common. One of the students
giving CLAT in Delhi had filed a petition against the amendment at the Delhi High Court. In
refusing, the judges reasoned that the amendment act was passed by the State of
Karnataka, thus disallowing them to exercise any territorial jurisdiction over the matter. They
also noted that NLS is in Bangalore, Karnataka and that it is not appropriate for them to
decide in the matter. [Source: https://www.thehindu.com/news/cities/Delhi/hc-refuses-to-
entertain-plea-challenging-karnataka[1]govts-decision-on-quota/article31.ece]

. Asad owns a sweet shop in Hyderabad, Telangana. His sweets are famous for the
particular flavor throughout the country. With increasing popularity to his products, he
decided to open franchises in Chennai and other cities around the country. The sweets were
made in Hyderabad and brought to these stores for sale. Tamil Nadu state government had
recently passed a state legislation about sanitary measures in the manufacture of food
products. Among them was the mandatory use of gloves, shower caps and other protective
gear by the cooks while making the food products. A few days back, Shankar gets a viral
video of the sweet-making process in Asad’s sweet shop. Being a fan of Asad’s sweets, he
was aghast when he found out that none of the cooks wore gloves. Aggrieved by it, he goes
to the Madras High Court for compensation. Will he win?

(a) No, the sweets were manufactured in Hyderabad, so the State law cannot be applied.

(b) The only issue in this case is of jurisdiction. Jurisdiction resides with the court in
Hyderabad.

(c) Yes, the sweets being sold in Chennai would mean that the State law shall be applied.

(d) Compensation cannot be awarded in such an event. State law only provides for adoption
of hygienic measures but no penalty has been specified.

. Rohan is a college student living in Bangalore. Being an introvert, the only way he could
belong in his college atmosphere was if started doing drugs. Naturally he needed more
money as days went on. So to earn more pocket money he decided to start peddling
marijuana in his neighborhood. He claimed that his source was Modi from Kanpur. When he
placed his first order to Modi, his father, Rohan Sr., receives the package in Delhi accidently
and comes to know of Rohan’s drug racket. Peddling marijuana is a punishable offence
under the Narcotic Drugs and Psychotropic Substances which was passed by the Parliament
in 19. Charges are pressed against Rohan at the Delhi High Court.

(a) Yes, as the package was received in Delhi and therefore the concerned high court will
have territorial jurisdiction.

(b) Rohan who was in fact smuggling, so Modi should not be charged. Modi merely fulfilled
the order given to him. Thus, since the order was placed from Bangalore, Karnataka HC
should have jurisdiction.

(c) NDPS Act is a national act and the provisions will apply equally at all places. The petition
may be entertained at the Delhi HC.

(d) No, Modi hails from Kanpur, so he cannot be charged at the Delhi HC. The order was
dispatched from Kanpur so Lucknow HC should have jurisdiction.

. At the initial hearing at the High Court, the petition is dismissed. Rohan Sr decides to
appeal to the Supreme Court. Can he take the case to the Supreme Court?
(a) Yes, Supreme Court should hear it as it is the final court of appeal in the country.

(b) No, Supreme Court cannot hear it as it is a question of jurisdiction which must be solved
by the concerned High Court only.

(c) Yes, Supreme Court is eligible to hear cases concerning national legislations and since
NDPS Act is a national legislation, SC cannot refuse.

(d) No, Supreme Court cannot hear it as Rohan cannot appeal. He should have filed a
Special Leave Petition instead.

. The Parliament passes a bill to allow state governments to levy duties on use of agricultural
lands for industrial purposes. The state governments had to pass an additional act in their
state legislatures to make the duties legal. The state of Rajasthan, in dire need of agricultural
land, imposes a 50% duty on the value of the land. Industrialists based in Rajasthan file a
complaint at the Rajasthan High Court, claiming it is unjust to impose such amount of duties.
Will the Industrialists win?

(a) No, it is a central legislation and the Rajasthan high court cannot rule on the case.

(b) Yes, it is a state legislation and Rajasthan High Court is the legitimate authority to hear
the matter.

(c) The legislation was not passed by the Rajasthan legislature. Thus, the Rajasthan HC is
not the legitimate authority.

(d) Yes, the industrialists are based in Rajasthan, so they can claim at the Rajasthan High
Court even if it was a UP state legislation.

18
Directions (Q. – Q.): Read thecomprehension carefully and answer the questions based on it.
(Passage Q.-Q.):Custodial death is one of the worst crimes in a civilized society governed by
Rule of Law. Does a citizen shed off his fundamental right to life, the moment a policeman
arrests him? Can the right to life of a citizen be put in abeyance on his arrest? The answer,
indeed, has to be an emphatic ‘No’. In cases of police torture or custodial death, it is rare that
direct ocular evidence of the complicity of the police personnel would be available. To take
care of this problem, the law has envisaged a process of mandatory parallel Magisterial
Inquiry in addition to the inquiry or investigation held by the police, immediately after the
incident, as per S.1(1A) of the CrPC. It deals with cases of death, disappearance or rape in
police custody. This inquiry cannot be done by an Executive Magistrate and must be carried
out by a Judicial Magistrate who within 24 hours of the death of the person, forward the body
with a view to it being examined to the nearest Civil Surgeon. Registration of FIR in a case of
custodial death is mandatory but despite the mandatory nature of this provision, its
compliance is highly rare. Therefore S.154A of CrPC gives right to any person aggrieved by
a refusal on the part of an officer in charge of a police station to record the information in
cases relating to custodial offences, may file a petition before the Chief Judicial Magistrate,
in case of custodial offences other than those involving death of the victim, or before the
Sessions Judge, in cases of custodial offences involving death of the victim. The SC was of
the opinion that protection of S.1 CrPC which provides sanction to prosecute police officers
accused of custodial torture was not available for offences which have no connection with
official duties. The defence of S.1 CrPC was raised by accused officers in many cases of
custodial torture. Therefore the law commission recommended that the provisions of this
section do not apply to any offence committed by a judge or public servant, being an offence
against the human body committed in respect of a person in his custody, nor to any offence
constituting an abuse of authority. But this recommendation has not been acted upon yet.
[‘Custodial Deaths: What Is The Procedure For Inquiry?’ Live Law accessed on 5th July
2020]

. On 22nd January 2020, communal riots were incited in the NCR of Delhi. The police
officers of the concerned area arrested Shashi, a member of the riot. During investigation the
police officers asked him to reveal the names of all the leaders to save his life. He refused to
answer them and remained silent. The Inspector of the station in order to break his silence
kicked and punched him due to which Shashi suffered many injuries. Ritesh, another member
of the riot went to register FIR but the police officer refused to do so. He then went to the
Sessions judge to file a petition against the police officer for refusing to file the FIR. The
Sessions judge rejected the petition. Decide:

(a) The Sessions judge was wrong in doing so because it was his duty to initiate a mandatory
inquiry as per S.1(1A) of the CrPC.

(b) The Sessions judge was wrong in doing so because the registration of FIR is a mandatory
provision of CrPC in case of cognizable offences and the police officer in refusing to do so
violated the provisions of CrPC.

(c) The Sessions judge was right in doing so because the police officer has the protection of
s.1 of CrPC which provides sanction to prosecute police officers accused of custodial torture.

(d) The Sessions judge was right in doing so because in case of custodial offences other than
those involving death of the victim the petition should be filed before the Chief Judicial
Magistrate.

. Jeevan and Mayank are brothers. Due to some property dispute fight broke off between
them and they hit each other with lathis. Both suffered many injuries. Rajesh, their neighbour,
tried to stop them and in the meanwhile to avoid him Jeevan pushed his brother who suffered
serious head injuries and died. The police arrested Jeevan. Rajesh went to the police station to
give his statement where Jeevan was also present. He was outraged to see him and started
abusing and kicking him. The police officer in order to quiet him pushed him in the lock up
and he hit his head and suffered injury. He filed a petition to the CJM. The police officer took
the defence of s. 1 of CrPC. Decide

(a) The police officer will be absolved because the act was committed by him in the
connection of his official duties.

(b) The police officer will not be absolved because the sanction of s.1 does not apply to any
offence committed by a public servant, against the human body committed in respect of a
person in his custody.
(c) The police officer will be absolved because he was only discharging his duty and since
there were already injuries on the head of Jeevan, it cannot be concluded if he was saying
truth.

(d) The police officer will not be absolved because the act of pushing done by him was not in
connection of his official duty.

. Shree was convicted of murder of Alia, a famous Bollywood singer, and was sentenced for
life imprisonment. Some of the prisoners and officers were staunch fan of her and left no
chance to harm him and always threatened to kill him. He was scared and tried to escape
from prison twice to save his life. But he was unsuccessful. Suddenly he was disappeared and
police started searching for him. Later he was found dead on the terrace of the prison building
after 3 days. Shree’s wife wants justice for him. Which of the following is correct?

(a) She should register an FIR and it is mandatory for the police to register in a case of
custodial death.

(b) She should file a petition to the magistrate against the police officers complaining about
the threats and abuses they use to project towards Shree and accusing them for the
disappearing and then killing Shree.

(c) She should not go to the Executive magistrate since there is no substantive proof that
Shree was killed by the police officers during custodial violence and that Shree has many
enemies in the prison.

(d) She should go to the Judicial magistrate to file a complaint who is to initiate magisterial
inquiry in addition to the inquiry or investigation held by the police for the disappearance and
death of Shree.

. Riya and Naman fell in love during their college. They wanted to marry but their families
were against it. So they decided to run away and get married. But before they could do so
Naman got in a fight with Riya’s brother and was arrested for disturbing public peace. Riya’s
uncle was the officer in charge of the police station. He told Naman to go away from Riya’s
life otherwise he will charge him for various offences and he will have to spend the rest of his
life in prison. He tried to escape and so the officer shot him and he died. He took the defence
of s.1. Decide:

(a) He will get the defence as although he had a personal clash with Naman still the act done
by him is in furtherance of discharging his official duty. He shot at Naman because he was
trying to escape and it was his duty to stop him.

(b) He will not get the defence as the sanction of s.1 does not apply to any offence
constituting an abuse of authority. The force used by him was unreasonable and an abuse of
his authority.

(c) He will not get the defence as the act done by him was of malicious intent and there is no
reasonable connection between the act of shooting and the discharge of his official duty.

(d) He will get the defence as it was not custodial death and he is a public servant and can use
force to stop anything wrong happening.
. With the context of above question, NirmalJeeven, an NGO working for disabled people
went to the police station to register an FIR for the death of Naman but the police officers
refused to do so as Naman was killed while escaping from the jail. They went to the Sessions
Judge to file the complaint but the judge refused to do so. Was he right in doing so?

(a) No, because if a person is refused by officer in charge of a police station to record the
information, they may file a petition before Sessions Judge.

(b) Yes, because the NGO has no nexus with Naman and so they could not file any complaint
on behalf of him or his family.

(c) Yes, because the person filing the complaint should be Riya or any family member of
Naman and since the act was committed while he was running from jail so it is not custodial
death and so there can be no complain.

(d) No, because instead of rejecting the complaint he should have asked the chief judicial
magistrate to look into the matter as it was his power to do so.

(Passage Q.-Q.): Noting the frailties in Centre-State relations in our quasi-federal setup, the
Punchhi Commission in its famous report gave out guidelines delineating the role of
Governor in appointment of a Chief Minister in situations of hung assembly. The guidelines
were given out in wake of large uproar against the alleged misuse of office of Governor by
the Central Government against an opposite party in power in the State. A hung assembly is a
situation wherein no individual political party or an alliance thereof possesses a clear
majority (i.e. fails to cross the 50% seat mark) and hence unable to form any working
government. Giving the Governor guidelines to follow in such situations, the Commission
said: 1. The party or combination of parties which commands the widest support in the
Legislative Assembly should be called upon to form the Government 2. If there is a pre-poll
alliance or coalition, then it should be treated as one political party and leader of such alliance
should be called upon to form Government upon attaining majority 3. The following order of
preference should be followed: (a) Group of parties having pre-poll alliance commanding
largest number of seats (b) Single largest party staking claim to form government with
support of others (c) Group of parties forming post electoral alliance (d) Post electoral
alliance with some parties joining the government and remaining including independents
supporting the government from outside. Upon formation of alliance in post electoral
circumstances, the newly chosen Chief Minister is called upon to prove majority in the
Assembly by a vote of confidence.

. In the recently concluded elections in State A, the 2 major political parties (Conservatives
and Liberals) ended up winning 45% seats each in the legislative assembly. The remaining
10% seats got scattered amongst various independent candidates. All the independent
candidates having won the seats had a pre-poll alliance of forming government with the
position of Chief Minister being chosen by lottery after release of election results. However,
even when they miserably failed to obtain seats they stood with their alliance and refused to
support any political party. With other political parties not breaching the 50% mark, it was a
situation of hung assembly. The Liberals somehow managed to secure support of 6% of the
independent seats and wrote to the Governor claiming majority and request to allow
formation of government. The Governor allowed the move. However, the break of alliance of
independents was challenged by the remaining independent candidates and case was taken
before the Supreme Court. Would the action succeed?

(a) Yes, attempt to form alliance by Liberals does not amount to a pre-poll alliance and hence
not acceptable.

(b) Yes, breaking an already existing pre-poll alliance to secure support amounts to causing
defection and hence anti-defection law would be attracted

(c) No, post poll alliance did not include an alliance with other parties but only independent
candidates

(d) No, a post poll alliance is acceptable with newly formed government being subject to vote
of confidence, evidencing sufficient support

. In State B, the two major political parties (Democrats and Republicans) had agreed on a pre-
poll alliance for formation of government. The position of Chief Minister was agreed to be
held by a candidate from the Democrats party irrespective of the total seats secured by either
parties. In a unique turn out of events, Democrats (an otherwise political heavyweight in the
state) ended up securing only 30% seats with Republicans securing 48% seats. The remaining
22% seats got scattered amongst other political parties. The pre-poll alliance fell apart with
Republicans now changing mind to give away the Chief Minister’s position. The Governor
instead of inviting the pre-poll coalition of Democrats and Republicans to form government
imposed President’s rule citing insufficient majority with either party to form a government.
The Republicans secured support from several small parties and reaching 50% mark
approached the Governor to form government. The Governor did not allow. Action was
brought against the Governor. Will it succeed?

(a) No, since the Governor should give priority to only pre-poll alliances

(b) Yes, the Governor imposed President’s rule to support the sentiment of Democrats

(c) Yes, a post poll alliance can be invited to form government if a pre-poll alliance breaks

(d) No, since the Governor was acting neutrally by seeing the pre-poll coalition as one
political party

. State C has a legislative assembly with total 200 seats. Upon successful completion of
elections Party X with support of Party Y comfortably secured seats and Party Z secured
seats with remaining 9 seats being won by independents. The Governor instead of promptly
inviting Party X+Y to form government prioritized other official duties and which resulted in
delay in the formation of government. Meanwhile Party Z secured support of 5 candidates
from Party Y and who upon resigning from Party Y independently supported Party Z. Now
Party Z ended up securing support of all 14 independent candidates and claimed to have
majority to form government. The Governor meanwhile got free from other duties and invited
leader of Party Z to form government. Was the action correct?
(a) Yes, Governor could not control individual choices of candidates in Party X. Their choice
to support Party Z was an independent decision

(b) Yes, Governor has to allow a post-poll alliance securing sufficient majority. Also, there is
no specific time-frame to invite a party to form government.

(c) No, Governor’s action of delaying the invitation seem with an ulterior objective to prevent
Party X from forming any government

(d) Yes, Governor rightly foresaw a hung assembly in future and dutifully decided to wait for
Party Z to come up with a more stable coalition

. State D has a very fragmented political landscape with several political ideologies
flourishing in the State and each one enjoying considerable support. Parties P and Q agreed
upon a pre-poll alliance against Party R and contested the elections. In the results, P and Q
combined secured 47% seats and R secured 48% seats. Before R party could secure support
from remaining 5% seats, P and Q pounced upon the seats and secured their support with
threat and coercion, thus leading to a support of 52%. They approached the Governor to
allow forming government. The Governor allowed the coalition to form a government. In a
surprise turn out of events, the 5% candidates voted against the coalition during vote of
confidence. Party R then requested their support and formed a government with their support.
Government formed by R was challenged by P and Q before court. Will the action succeed?

(a) Yes, P and Q had secured a perfect post poll alliance which R dismantled to form
government on its own

(b) Yes, Governor should have ordered re-election since there was no clear majority with
either parties

(c) No, independent candidates’ choice to vote against P and Q cannot be questioned in any
court. Governor merely followed procedure as prescribed

(d) No, there could be multiple post poll alliances by and between different political parties
with various combinations possible. Independents now supporting R is also valid

(Passage Q.-Q.): The Indian Telegraph Act,18 defines ‘telegraph’ “any appliance, instrument,
material or apparatus used or capable of use for transmission or reception of signs, signals,
writing, images and sounds or intelligence of any –nature by wire, visual or other
electromagnetic emissions, Radio waves or Hertzian waves, galvanic, electric or magnetic
means. An explanation is added to section 3(1AA), which says that ‘radio waves’ or
‘Hertzian waves’ means electromagnetic waves that have frequencies lower than 3,000 giga-
cycles/second propagated in space without artificial guide. No private individual has the right
to incept the communication of another individual, only the government of India has the
power to take possession of any licensed telegraphs in case of a public emergency or in the
interest of public safety. Section 5 states, where the government can order interception of
communication and reasonable restrictions can be imposed on the right to privacy in the
interests of the “sovereignty and integrity of India, the security of the state, friendly relation
with foreign states or public order or for preventing incitement to the commission of an
offence”. Wiretapping is of two types first is Passive, which is completely legal because it
can only be done by the government by following proper procedure and the second one is
Active wiretapping/illegal interception where the attachment made to an unauthorized device
to gain every single access to data for instance by generating false messages, etc. The act of
telephone tapping affects the right to privacy of an individual as well as the right to freedom
of speech and expression, both are Fundamental Rights under the Constitution. There are two
statutory pre-conditions for the exercise of the power to intercept, either the “occurrence of
any public emergency” or “the interest of public safety”. Central Government or a State
Government or 19 of 40 any of its officers specially authorised by the Central Government or
the State Government, as the case may be to exercise powers of interception, under the IT act
2000. For obtaining orders in remote areas, Interception rules, 2009 state that it would be
permissible to carry out interception after obtaining the orders of the Head or Second Senior
most officer of security and law enforcement at the Central level, and an authorized officer
not below the rank of Inspector General of Police at the State or Union Territory Level.

. Abdul and Razia were a happily married couple. After few years of their marriage, things
between them started to change. Abdul started tumbling into bad practices, such as drinking
excessive alcohol, not coming home for days, unnecessary brawling with Razia and many
more. One day Razia found Abdul making love to Razia’s best friend ‘Hasina’, at Abdul’s
office. Slowly the toxicity in relationship, between the couple started to increase. Abdul
promised Razia, that he will be a changed man and such things won’t happen further. After
few weeks, Abdul gets promoted in his job and also got a chance to induct a secretary for his
office, which will be situated in Dubai. Without giving any thought, Abdul inducted Hasina
as his secretary. Razia after knowing, that his husband will be moving to Dubai, with her best
friend whom she caught making love red handedly, got angry and decided to divorce Abdul.
For having sufficient evidence in front of the court of law, she tapped Abdul’s office
telephone and got enough evidence how his husband was cheating on her, with Hasina. Can
Razia file a Divorce, with these evidence in hand in the court?

(a) Yes, since Husband and Wife are considered as a singular entity and nothing is a secret
between the both of them. The relationship they command, saves Razia from declaring this
evidence as legal.

(b) No, since no private individual has the right to incept the communication of another
individual, only the government of India has this power that too with restrictions and
conditions.

(c) Yes, since though the privacy of Abdul and Hasina was breeched, but for a better good. A
married individual, having relationship with another women is already a crime and this
cannot be proved, without proper evidence.

(d) No, since proper approval from the centre/state government should have been taken by
Razia before committing such actions. After following a proper protocol, any person can
legally tap any communicating device.

. Shubham and Priya are residents of Hastinapur, a rural area of Bihar. Hastinapur is a
backward place, where the people are very poor and the standard of living was below
average, as labours were the majority there. The major issue faced by them was the
connectivity issue as not sufficient network towers were installed in Hastinapur. Seeing a big
business opportunity, Shubham and Priya separately started a STD-PCO at a huge level.
Their business started to boom from the first day itself. However, after a couple of weeks,
Inspector general of the Hastinapur police department, with proper orders visited Shubham’s
office, demanded him to tap the telephone calls of ‘Sudheer’ who was a spy of the
neighbouring country. Shubham knew that, the act of telephone tapping affects the right to
privacy of an individual as well as the right to freedom of speech and expression, both are
Fundamental Rights under the Constitution. Additionally, his only business will be severely
affected as nobody will trust his STD[1]PCO since private and personal information of
people will be at sake whenever they will be using his services and also, they had another
option too. Finally, Shubham was forced to tap the calls and also had to incur high losses in
his business. Shubham pleads under the court of law, whether the acts committed by the
government officials were legal or not?

(a) Yes, since it is the job of the government agencies to work for the people and stop crime
in any situation. The officials were just performing their job as they are authorized to tap calls
at any time, if the situation demands so.

(b) No, since the fundamental rights of Shubham as well as Sudheer are at stake. It affects the
right to privacy of an individual as well as the right to freedom of speech and expression,
both are Fundamental Rights under the Constitution in every sense.

(c) Yes, since the act was committed after following a proper procedure as they had a valid
order with them. Additionally, the Indian Telegraph Act,18 and interception rules, 2009
protects them too.

(d) No, since the government cannot force Shubham to tap his customers telephone for their
benefit. Shubham can be requested to hand over the recordings to the government agencies as
it is his business. The fundamental right to practise business and the Indian Telegraph Act,18
legally protect Shubham in this situation.

. Which government has the authority to start an enquiry to conduct a Passive wiretapping on
any individual for the interest of public safety in the country?

(a) Only the Central government.

(b) Only the State government.

(c) Both the Centre and the State have the power.

(d) The power is completely vested in the centre as President or Governor provide the due
assent and both these posts are directly vested in the authority of the centre.

. Sanjay is a pseudo left wing activist, who under the fundamental right to free speech and
expression has been giving critical remarks on the government. He gained lime light recently,
for having some connections with the underworld as been alleged by some reputed news
journalists. When this news caught fire, government security agencies came into action. The
Chief of Research and Analysis Wing (RAW), ‘Vikram Singh’ to get a reality check of the
situation as well as to collect evidence by catfishing, first obtained his telephone number and
then called him for the enquiry. He did not took any permission from any senior authority nor
the centre/state government too. Adding to that, without the consent of Sanjay he recorded his
telephone call for show casing Sanjay’s motive. In the recorded conversation, Sanjay was
agreeing for his connections with the underworld. Taking this an evidence, Vikram demanded
to tape Sanjay’s telephone for security reasons. After knowing this Sanjay pleaded in the
court of law that his call was been illegally recorded and demanded protection under the
Indian Telegraph Act,18 and interception rules, 2009. Is the argument of Sanjay valid?

(a) Yes, since for conducting any recording or taping of any individual a due process has to
be followed by the government agencies, which were not at all followed here by Vikram
under the Indian Telegraph Act,18 and interception rules, 2009.

(b) No, since whenever sovereignty and integrity of India and the security of the state are in
trouble, certain exceptions are been provided under the Indian Telegraph Act,18. The acts
committed by Vikram fall under these exceptions and thus stand legal.

(c) Yes, since the fundamental right of free speech and expression under the Article 19 of the
Indian constitution is been violated here by the government agencies. Fundamental rights are
the backbone of the constitution and hence cannot be violated in any circumstance legally.

(d) No, since the argument laid by Sanjay do not fall in the scope of under the Indian
Telegraph Act,18 and interception rules, 2009.

. Ramesh is a student at the National Institute of Technology, Bhopal. He is a student of


computer science and electronics. With immense hard work and smart work, he was able to
make a device which was been considered way ahead of its generation. When asked about the
device, he stated that ‘he has developed a device which transmits images with the help of
electromagnetic waves which have a frequency of 20,000 gigacycles per second and the
information stored is on end-to-end encryption with the help of internet’. Does the device
developed by Ramesh in this situation falls under the definition of telegraph, of the Indian
Telegraph Act?

(a) Yes, since any device which can receive or transmit any information comes under the
definition of an ‘telegraph’ the Indian Telegraph Act.

(b) No, since the frequency are higher than as provided under the explanation added to
section 3(1AA), thus not under the scope of radio and Hertzian waves. Additionally, the
information is encrypted on the internet, which makes it fall out of the scope.

(c) Yes, since it does not depend upon the frequency of the waves to be added or not in the
definition of a ‘telegraph’. Every end-to-end encrypted device falls under the scope of the
definition of ‘telegraph’ such as Facebook, WhatsApp, etc.

(d) No, since it is not possible to put any device which receive or transmit any information
into the scope of ‘telegraph’. Only telegraph comes under the scope of this definition and all
other devices comes under the act not the definition.

(Passage Q.-Q.): Medical negligence or medical malpractice refers to the improper, unskilled,
or negligent treatment of a patient by a physician, dentist, nurse, pharmacist, or other health
care professional. Medical malpractice occurs when a health-care provider strays from the
recognized “standard of care” in the treatment of a patient. This “standard of care” is
understood to mean what a reasonably prudent medical provider would or would not have
done under the same or similar circumstances. Professionals are persons professing some
special skill or job, who are trained to profess in that area specially and bear the responsibility
of professing with due care. Such professionals include lawyers, doctors, architects etc. The
Supreme Court in Jacob Mathew v. State of Punjab explained: a professional entering into
certain profession is deemed to have knowledge regarding that profession and it is assured
impliedly by him that a reasonable amount of care shall be taken to profess his profession.
The person can be held liable under negligence if he did not possess the required skills to
profess or he failed to take essential amount of care to profess the said profession. In the
recent case of SunandabaiKisanji Dhole v. Dr VinodBele, the Complainant failed to produce
any cogent evidence or any expert opinion that the accused doctor treated the patient
negligently and caused post injection radial nerve palsy. It was found that the Complainant’s
disability was due to long standing arthiritis but not due to nerve palsy as alleged. Therefore,
it was held that “No cure is not negligence if there is no improvement despite reasonable
treatment”. [Extracted with revisions from ‘Medical Neglience’ by Aakarsh Shah,
lawctopus.com/academike https://www.lawctopus.com/academike/medical-negligence/]

. Derek Shepherd was a world-class neurosurgeon who had successful surgeries % of the
time. Having knowledge of the same, Alex asked Derek to remove his brother Dalex’s stage
IV brain tumour. Despite Derek’s best efforts, he could not save Dalex due to some
unforeseeable complications during the surgery. Alex sues Derek for medical negligence.

(a) Derek was not negligent because he took reasonable care

(b) Derek was not negligent because he did not take reasonable care

(c) Derek was negligent because he did not possess the requisite skill for the surgery and still
performed it

(d) Derek was negligent because he did not take reasonable care

. Sheetal was a nurse employed in Grey Sloan Hospital who loved to catch up with her other
nurse friends on phone while taking rounds to check up on the patients. One day, while she
was talking to her friend Meetal, she failed to notice that the urine bag of one of the patients
was full and needed to be replaced. Consequently, the bag burst causing an infection to the
patient.

(a) Sheetal is not liable for medical negligence

(b) Meetal is liable for medical negligence

(c) Grey Sloan is vicariously liable for the medical negligence of the nurse

(d) None of the above

. Mr Dharma went to the pharmacy to buy a prescribed medicine Y for his medical condition.
The Pharmacist told him that Y was out of stock but he could instead give him medicine Z
which had the same salts. The pharmacist was aware that Z could cause adverse reactions in
people with lactose intolerance. He did not ask Mr. Dharma if he was lactose intolerant. Mr.
Dharma consumed the medicine and had an adverse reaction. Is the Pharmacist liable for
medical negligence?

(a) No, because the Pharmacist was not a medical professional

(b) Yes, because the Pharmacist was a healthcare professional

(c) Yes, because the Pharmacist failed to take the reasonable standard of care

(d) No, because the Pharmacist was unware of Mr Dharma’s lactose intolerance

. A doctor while performing a surgery accidentally left her musical watch inside the patient’s
stomach. After three months, the watch automatically started playing music while inside the
patient’s stomach although he suffered no consequences and seemed fine. He sued the doctor
for failing to take reasonable care. Is the doctor liable for medical negligence?

(a) Yes, because any doctor is expected to remove any accessories before surgery

(b) Yes, because she failed to take the reasonable amount of care that was expected of her

(c) No, because the patient did not suffer from any consequences and was fine

(d) No, because there was no problem in the surgery or treatment of the doctor as such

(Passage Q.-Q.): In contracts, a mistake may operate in two ways. Firstly, it may defeat the
consent of the parties’ altogether and secondly, it may mislead the parties about the purpose
of the contract. Section 13 of the Act defines consent as, “Two or more persons are said to
consent when they agree upon the same thing in the same sense.” basically, for consent, there
should be consensus ad idem. When the mistake of the parties does not defeat the consent but
only misleads the parties then Section 20 of the Indian Contract Act shall apply. It reads as,
“Where both the parties to an agreement are under a mistake of fact essential to the
agreement, the agreement is void.” A contract is not voidable merely of the fact that it was
caused by the mistake of law in India, but a mistake of law, not in force in India has the same
effect as a mistake of fact. Further, the Act also provides that, “A contract is not voidable
merely because of the fact that it was caused by the mistake of the fact of one of the parties.”
There cannot be an exhaustive list of facts which are essential to contract and those which are
not. The importance of the facts depends upon the nature of the contract which varies in
every case. Broadly, certain facts which are essential to every agreement include the identity
of parties, identity and nature of the subject matter of the contract,and the nature and content
of the promise itself. [Extracted with minor revisions from ‘Free Consent of Parties to
Contract’ by SubodhAsthana in blog.ipleaders.in https://blog.ipleaders.in/free-consent-
parties-contract/amp/]
. AX and BY make a contract grounded on the erroneous belief that a particular debt is barred
by the Indian Law of Limitation. The contract is:

(a) Void

(b) Voidable

(c) Voidable at the option of A

(d) Valid

. AY and BX make a contract grounded on the erroneous belief that a particular debt is barred
by the French Law of Limitation. The contract is:

(a) Void

(b) Voidable

(c) Voidable at the option of AY

(d) Not voidable

. Sitaram is a jeweller. A man named AkashAmbani entered his store and told him his name.
Sitaram thought he was the son of MukeshAmbani and will therefore, pay him back on time.
So he agreed to contract for a sale of diamonds with him worth Rs. 30 lakhs. Later on, he
found out that he was a different AkashAmbani. He wishes to avoid the contract. Can he do
so?

(a) Yes, because the contract is voidable since there is a mistake of fact

(b) No, because the contract is already void since there is a mistake of fact

(c) Yes, because the contract is voidable at his option since there is a mistake of fact

(d) No, because the contract is not voidable at his option since there is no mistake of fact

. Jim had a ship named "Queen of the Blue Sea" and a big blue diamond of the same name.
Jim wanted to sell his ship. Pam was a connoisseur of diamonds. Jim contracted for the sale
of Queen of the Blue Sea with Pam. Later on it was found that Pam wanted to buy the
diamond and not the ship and thought that the contract was for the diamond.

(a) The contract is void at the option of Jim, since he wanted to sell the ship
(b) The contract is void at the option of Pam, since she wanted to buy the diamond

(c) The contract is void because there was consensus ad idem, leading to mistake of fact

(d) The contract is void because there was no consensus as idem, defeating the consent of the
parties

(Passage Q.-Q.): The Maharashtra government this week presented a draft bill making
changes to existing laws on violence against women and children. The Bill, proposed to be
enacted as Shakti Act, 2020, has provisions for enhancing punishment including death
penalty for rape, fine up to Rs 10 lakh on perpetrators of violence, faster investigations and
quicker disposal of cases. The draft Bill proposes to make changes to the Indian Penal Code,
the Code of Criminal Procedure and the Protection of Children from Sexual Offences Act.
The changes are proposed in existing sections of rape, sexual harassment, acid attack and
child sexual abuse. The draft Bill proposes an additional law to deal with abuse of women on
social media. Section 354E is added to include intentional acts creating “a sense of danger,
intimidation, fear to a woman” apart from insulting her modesty by any act, deed or words
including offensive communication will be an offence with a maximum punishment of two
years and a Rs 1 lakh fine. The Bill also proposes to add an explanation to Section 3 (rape) of
the IPC, “Rape committed in circumstance including but not limited to some form of
assurance including promise of marriage or understanding between the parties, where they
are consenting adults, and from conduct it appears that act has been committed with consent
or ‘implied consent’ may be presumed that valid consent is given.” The existing law does not
have a blanket assumption of consent being implied in such cases. The Bill also proposes
setting up of a “Women and Children Offenders Registry” linked to the National Registry of
Sexual Offenders and will be made available to law enforcement agencies with details of
persons convicted of specified offences of sexual violence against women and children. A
separate police team will also be set up in each district to probe such cases. Source –
[Extracted, with edits and revisions, from: Explained: Death Penalty, steep fines and other
changes in Maharashtra Bill on Sexual Crime, SadafModak, The Indian Express, dated
15.12.2020] https://indianexpress.com/article/explained/maharashtra-shakti-act-law-on-
violence-against-women-and[1]children-explained-005/

. Seema was on her way to School to attend Republic Day function. She was stopped by two
unknown men who enquired the way to her school and as she was explaining them the way,
they started pulling her arm trying to push her inside the van. Some people came to her rescue
after she raised an alarm. Will this act of the two men constitute as an offence under the draft
bill?

(a) The mere act of pulling by arm will not constitute an offence under the act.

(b) The act is an offence if Seema was not rescued timely and harm is done to her.

(c) The act of grabbing and pulling her hand threatened Seema.

(d) The act is not an offence since she was rescued timely before any harm was done to her.

 
. X is an accused in a Rape case however he was recently granted Bail by the court. He goes
to visit his girlfriend and proposes to her for marriage and expresses his love towards her. She
rejected his proposal and started walking away, in an attempt to persuade her and stop her he
held her hand. Threatened by this act she raises alarm and calls the police. The police take
Sunil under custody for outraging the modesty of a woman. Which of the following holds
true in context of the draft bill?

(a) X is guilty of the offence since the girl felt threatened by his act, outraging her modesty.

(b) X held her hand when she started walking away without her consent making it an offence
under the act.

(c) The act of holding hand without any sexual intention cannot be constituted as an offence.

(d) The act of holding hand of a woman is not an offence under the act.

. David and Emma were deeply in love. Emma always imagined herself getting married to
David and discussed about the same with David, he smiled and changed the topic whenever
Emma brought the topic of marriage. Emma under the pretext of marriage to David engages
in sexual intercourse with him. After sometime David breaks up with Emma and marries
another girl. She files a complaint against David accusing him of Rape. Will her claim be
successful under the existing law?

(a) He is guilty of Rape since Emma did not give her consent for sexual intercourse.

(b) He is not guilty of Rape since there was an implied consent on part of Emma.

(c) He is guilty of Rape since Emma did not give free consent to engage in sexual intercourse.

(d) He is not guilty since at the time of sexual intercourse Emma gave her free consent.

. Stuart and Amy have a disagreement over a telephonic conversation. During the
conversation he threatens to break into her flat tomorrow to Rape Amy and click intimate
pictures of her which he will upload on Facebook. Amy, scared by all this calls her friend to
stay with her. Due to the threats made by Stuart she stops going to College. Amy finally
decides to file a complaint against him a month after he made such threats. The police refused
to register her FIR stating the reason that Stuart has not contacted Amy since that incident. In
light of the draft bill, which of the following reasoning is correct?

(a) The threats made by Stuart were empty threats which do not constitute an offence.

(b) The threats made by Stuart will constitute an offence if he actually sexually assaults Amy.

(c) The threats will not constitute an offence since Stuart did not tried to contact or threaten
her since that incident.
(d) The threats will constitute an offence since Amy felt scared since that incident.

(Passage Q.-Q.): Section 41 of the CrPC commends: When police may arrest without warrant
(a document issued by a legal official authorizing the Police to make an arrest). Sub-section
(1)(a) says that whoever commits, in the presence of a police officer, a cognizable offence.
However, Section 41(2) specifies that no person concerned in a non-cognizable offence shall
be arrested except under a warrant or order of a Magistrate. In this section, three situations
have been indicated before making an arrest, namely, against whom a reasonable complaint
has been made, or credible information has been received, or a reasonable suspicion exists. If
the above circumstances are present, then also, to justify the arrest it is necessary to follow
any condition as mentioned below; (a) to prevent such person from committing any further
offence; (b) for proper investigation of the offence; or (c) to prevent such person from
causing the evidence of the offence to disappear or tampering with such evidence in any
manner; (d) to prevent such person from making any inducement, threat or promise to any
person acquainted with the facts of the case so as to dissuade him from disclosing such facts
to the court or to police officer (e) as unless such person is arrested, his presence in the Court
whenever required cannot ensured. Thus, any of the above conditions have been fulfilled,
Police can make an arrest, and this, of course, will be applicable for the offence punishable
up to 7 years. Besides, the police are required to record the reasons in writing for making
such arrest. However, in case of woman, the police officer shall not touch the woman for
making her arrest, unless the police officer is a female and the circumstances require.
Besides, to avoid arbitrary arrest, there is also a statutory provision u/s. 41-B that every police
officer while making an arrest shall bear an accurate, visible and clear identification of his
name, will prepare a memorandum of arrest, attested by a witness from arrestee’s family or a
respectable member of the locality, where the arrest is made. [Kashyap Joshi, ‘Arrest under
Criminal Law: Who, When, Why and How?’ (2020) Live Law as accessed on 27 November
2020]

. A notification was issued by the Central Government which clarified that aiding any convict
in hiding or escaping is a cognizable offence. A complaint was filed against Vishesh, by one
of his neighbours, Shashank, for helping a murder convict in hiding from the police and in
concealing the incriminating evidence. The police officer after filing the complaint
immediately rushed to find Vishesh and arrested him, as per the guidelines issued under the
notification. Vishesh challenged his arrest on the grounds that there is no proper evidence that
he had done anything to help any convict in hiding or escaping. Is Vishesh’s arrest legal?

(a) Yes, Vishesh’s arrest is legal as he was helping an offender who was accused of
murdering a person. This aid in itself is a crime for which he had to be arrested immediately;
otherwise he would have run away from the clutches of the police.

(b) No, Vishesh’s arrest was not legal as there is no proper evidence that he actually did
anything that he was accused of. He cannot be arrested on mere allegations of a neighbour.

(c) Yes, Vishesh’s arrest is legal as he was accused of helping a convict in hiding from the
police and it could be reasonably presumed that had he not been arrested immediately, he
would have tampered the incriminating evidence or disappeared the convict or any other
relevant information necessary to prove his guilt.
(d) No, Vishesh’s arrest was not legal as the police officer was not authorised by any
Magistrate to arrest Vishesh and neither the officer had any warrant issued against him to
make his arrest.

. Indumati was the MLA of Dholakpur region. Her opposition party made Chhutki, a local
leader, their campaign ambassador during the election period. She was very popular among
the local public and so it was easy to predict that if she continues to campaign for the
Indumati’s opposite party, she will definitely lose in the coming elections. Suddenly one day,
Chhutki was killed on her farm and it was alleged that Indumati murdered her. Police after
getting the information went to arrest Indumati but she refused to cooperate and deined the
allegations. The police officer forcefully took her to the station and locked her up in a cell.
She filed a petition against the police officer for forcefully making her arrest and touching her
despite being a male officer. Decide:

A. No police officer shall touch the woman while making her arrest, if she is denying the
same, as an accused is also entitled to the right of their body and personal liberty under the
Indian Constitution.

B. A woman police officer can touch the woman while making her arrest only if the accused
is not cooperating or is reluctant to go to the police station.

C. A male officer shall not touch the woman while making her arrest as it would be against
the dignity of a woman.

D. A police officer can touch the woman while making her arrest if the accused is not
cooperating or is reluctant to go to the police station.

Which of the following is/ are correct?

(a) Only D (b) Both B and C (c) Only B (d) None of the above.

. Which of the following circumstances are necessary to justify the arrest of a person without
any warrant or order from the Magistrate?

(a) If any credible information has been received against a person for committing a non-
cognizable offence, such person shall be arrested to prevent him from making any
inducement, threat or promise to any person acquainted with the facts of the case so as to
dissuade him from disclosing such facts to the court or to police officer.

(b) If any reasonable complaint has been filed against a person for committing a cognizable
offence, such person shall be arrested to prevent him from committing any further offence.

(c) If any reasonable suspicion exists against any person for committing any offence, such
person shall be arrested for proper investigation of the offence.

(d) If any credible information has been received against a person for committing any
cognizable offence, such person shall be arrested to prevent other person from committing the
same offence.
 

. Misha was a habitual offender. She was involved in many crimes including counterfeiting
currency, but she was able to escape from the police every time. Sarla, a police officer, while
she was on her honeymoon vacation in Goa, got an anonymous phone call informing about
Misha’s whereabouts in Goa. She immediately rushed to the place and found Misha. She tried
to arrest Misha and when she resisted, Sarla forcefully took her to the nearest police station
and locked her up. Sarla was later punished under Section 41B of CrPC. Decide Why?

(a) Because while making the arrest, Sarla was not working under the color of her office and
neither she was bearing a clear identification of her name.

(b) Because while making the arrest, Sarla forcefully arrested Misha and locked her in a
police station not under her jurisdiction.

 (c) Because while making the arrest, Sarla was not in the area which is under her
jurisdiction. Therefore she should have informed the local police instead of making the arrest
herself.

(d) Because while making the arrest, Sarla forcefully arrested Misha which is prohibited as
no police officer shall touch the woman while making her arrest, if she is denying the same.

. On 18th Feb, 2020, the Chief Minister of Lanka was lynched by an angry mob, due to his
discriminatory policies against the Suryavanshis. Rita, one of the leaders of the Suryavanshi
group, was arrested on the next day from her house. Although she resisted her arrest and
demanded memorandum of arrest from the police officer, she was not provided anything and
was locked up in a cell. She later challenged her arrest in the court as being illegal and
arbitrary. Decide:

(a) Rita’s arrest was not arbitrary as she was apprehended of leading a mob that lynched the
Chief Minister of Lanka, which is a cognizable offence and no document is required while
making arrest in such cases.

(b) Rita’s arrest was arbitrary as it is mandatory for a police officer to provide the accused of
all the necessary documents and inform them about the offence for which they are being
arrested.

(c) Rita’s arrest was arbitrary as she was forcefully arrested by a police officer even when she
resisted her arrest and demanded the memorandum of her arrest.

(d) Rita’s arrest was arbitrary as it is mandatory for a police officer to prepare a memorandum
of arrest, attested by a witness from arrestee’s family or a respectable member of the locality,
where the arrest is made.

(Passage Q.-Q.): Torture in custody represents a naked breach of human dignity and
deterioration that, in large part, personally destructs the individual. This is a predicted attack
on human dignity, and civilization takes a step backwards whenever human dignity is injured.
The Indian Constitution has fundamental rights that take the place of pride. Unless the
procedure established by the law, Article 21 stipulates that no person shall be deprived of his
life or his personal liberty. Personal freedom is, therefore, a constitutionally sacred and
valued right. The term “life or personal freedom” has been taken as a guarantee toward
torture, attacks, or assaults by the State or its officials, to include the right to live with human
dignity. Article 22 provides for safeguards in such cases against arrest and detention,
providing that, without being told of the reasons for such detainment, no person who is
apprehended shall be held in custody and that a lawyer of his choosing shall not refuse the
right of consultation and defense of himself. Consequently, in all cases of arrest and
detention, the Court considered it necessary to make the following requests as preventative
measures in that respect until legal provisions are made. · The police who carry out the
detention and treat the arrested person should have accurate, noticeable, and consistent
identity tags and names. All police staff involved in the questioning of the detained must be
recorded in a register. · That the police officer carrying out an arrest must make notice of
arrest before the arrest is made and that a notice of attestation must also be given by at least a
witness who may be either a member of the family of the arrested person or a reputable
individual from the location from which an arrest is performed. · The convicted party must be
informed of the opportunity to tell others of his detention or incarceration as soon as he or she
is placed in jail.

. Xiano Ronadhonalo, citizen of Catalonia, is in business of making low quality trophies and
certificates and is very well known internationally for his work. He visited India to deliver a
consignment of trophies. Quality Control Bureau of Customs Department of India detected
use of illegal material in the trophies, as and when this news reached Xiano he went into
hiding in India fearing prosecution. Police caught Xiano in an abandoned villa and took him
in custody. Maussi, a police officer, knew that Xiano does not have any fundamental rights
and he tortured Xiano to extract information of his buyer; now Xiano claims that the
treatment he received is against his Human Dignity. Decide.

(a) Foreigners do not have Fundamental Rights, especially when they are breaking the laws in
our country. Maussi did the needful, taking in account the status of law, and need of the hour
to find the buyers of illegal substances.

(b) Maussi is an honest cop doing his duty; Xiano on the other hand is a fugitive criminal
who does not have the rights he claims to have. Xiano was tortured because he wouldn’t co-
operate and to save time and ensure that justice is not delayed Maussi took the route of
torture.

(c) Xiano should contact his lawyer as soon as possible only way for him to get out of this
mess is to finally approach the problem with the laws of India.

(d) None of the Above.

. After two days in jail Xiano demands to be represented by Zudon, a very well-known
human rights activist in India. Zudon after listening to the merits of the case conveyed his
condolences to Xiano and denied to represent him citing reason of incapability to do so.
Based on the passage did Zudon do anything wrong by denying representation?

(a) Zudon knew that Xiano is beyond saving therefore it is a waste of time to defend him.

(b) ifZudon says he is incapable of representing Xiano then Zudon cannot be forced to fight
this case.
(c) Xiano ran away as a fugitive, tried to smuggle illegal substance in India against the law
and now demanding representation of his choosing is beyond acceptance, no such right is
bestowed on foreigners.

(d) Yes, by law Zudon is mandated to defend Xiano because Zudon is of Xiano’s choosing
and refusal to defend him could get Zudon in trouble with law.

. Currently Xiano’s case is being heard in the court of law, he claims mishandling of the case
as he was not provided with an opportunity to contact his family and inform them about his
situation. Police on the other hand claims that an opportunity to call his family was provided,
but his wife refused to talk because they are having a divorce. Were the court’s guidelines not
followed?

(a) An opportunity was given to Xiano, even though only informing of such opportunity is
mandatory, therefore guidelines were followed.

(b) Xiano being a foreigner has no such right, moreover getting an opportunity to place an
international call is more than enough and police in not obligated to serve such privileges.

(c) Police has no duty to allow such opportunity to Xiano therefore following the guidelines
isn’t even a matter of dispute.

(d) Xiano was unable to contact any of his peers or family about his situation and such
ignorance of whereabouts of Xiano is against law; his family must know where and how
Xiano is and that is why guidelines were not followed absolutely.

. Stan lee is a civil and human rights activist in India, and a patient of Parkinson’s disease. He
was arrested by the police under the charge of armed robbery. He has difficulty picking
things as his hands malfunction and is unable to even pick up a glass of water; he requested a
straw to ease his suffering. Police refused to provide him with any accessory and conveyed to
him that in jail no one is special. Such refusal resulted in Stan getting dehydrated and
admitted to hospital currently under coma. What could be the valid reason for such refusal?

(a) In jail no one is treated specially, precisely why Stan cannot be provided a stray, this will
result in influx of demand from the prisoners.

(b) Stan is under trial for armed robbery, providing anything apart from explicitly allowed
items in jail could hamper the security of jail and may aid Stan in escaping.

(c) Stan can drink water without having to hold the glass of water; several jail officials also
help Stan to drink water. There was no need or urgency to provide such unreasonable
demand.

(d) None of the above.

. Has Stan been tortured?

(a) Police did not even touch him least inflicting harm; therefore Stan has not been tortured.
(b) Stan was not been provided with bare necessities and later we was admitted to the
hospital, definitely constitute torture.

(c) Essential part of torture is to cause illegal harm to a person, it can be noted that Stan was
neither harmed legally nor illegally. No question of torture arises.

(d) Keeping a patient of Parkinson’s disease in a normal prison is a torture in itself; such
prisons aren’t compatible for people with special conditions.

19
Passage (Q.-Q.): In the absence of a doctrine of consideration, the category of intention to
create legal relations plays the role of filter on agreements that are to be enforced as
contracts. In Carlill v Carbolic Smoke Ball Co. [13] however, the “puff”, “was analysed in
terms of intention to create legal relations.” In Balfour v Balfour [19], intention to create
legal relations was pressed into service to deny enforceability to an agreement of a domestic
nature between a husband and wife. Then came, Rose & Frank v Crompton & Bros [15],
where the category was expanded to deny effect to an agreement of honour which two
businesses had mutually agreed to be not legally binding as a contract. It would be incorrect
to suppose that all these three cases are natural, organic instantiations or extensions of a
single doctrine or that they are all logically subsumed under it. The fact is that a
label[1]intention to create legal relations–came to be attached to various different legal
consequences unconnected by a common theme. 

. A husband and wife entered into an agreement that once the husband goes to England for
work, he would send money to his wife every month for her expenses. Gradually the relations
between them became sour and he stopped sending her money. The wife sued her husband
for breach of contract. Does she have a case?

(a) No, because matters between husband and wife are domestic in nature

(b) No, because there is no evident intention to create legal obligations

(c) Yes, because payment of monthly expenses amounted to consideration

(d) Yes, because there was an intention to create legal obligations

. Gita and Sita decide to meet at Starbucks at 5pm on 1st December 2020 to discuss their
notes. Gita arrives early waits for Sita. She waited for 3 hours but Sita does not turn up. Next
day Gita sues Sita for breach of contract. Is her claim valid?

(a) Yes, because there was an intention to create legal obligations

(b) Yes, because there was no intention to create legal obligations

(c) No, because there was no intention to create obligations

(d) No, because there was an intention to create legal obligations


. Ram contracts with Shyam to buy his old bike for Rs 15,000 so that he can go on a solo trip
to Ladakh. He books his hotel for 1st July 2020. He meets Shyam to buy the bike on 30th
June and starts talking about his trip. In their conversation they both forgot about the money
and Ram leaves with the bike. Next day Ram left for Ladakh so Shyam thought of asking for
the money once he's back. Once back, Ram returned the bike and refused to pay the money.

(a) Ram has to pay the money because there was an intention to create legal obligations

(b) Ram has to pay the money because it was the consideration for the bike

(c) Ram does not have to pay the money because there the money does not amount to
consideration

(d) Ram does not have to pay the money because there was no intention to create legal
relations.

. Mrs Jones was living alone in England so she called her daughter who was working in
Canada to live with her. She told her daughter that if she comes to England to live with her
then she will sponsor the funds required to complete her education in England. Is this a valid
contract?

(a) Yes, because there is an intention to create legal obligations

(b) Yes, because offering to pay for her daughter's education amounts to consideration

(c) No, because there is no intention to create legal obligations

(d) No, because offering to pay for her daughter's education does not amount to
consideration  

Passage (Q.68-Q.): The Karnataka Prevention of Slaughter and Preservation of Cattle Bill,
2020 seeks to ban slaughtering of all cattle. Section 1 (2) of the Bill reads, “‘Cattle’ means
cow, calf of a cow and llobull, buck and he or she buffalo below the age of thirteen years.”
The violation of the legislation, on conviction, would attract imprisonment ranging from three
years to seven years with a fine not less than ₹50,000 per cattle and may be extended up to
₹7 lakh or both. The Bill, makes even selling/disposing the cattle for slaughtering or
intentionally killing the cattle an offence. If the accused is convicted, then the court can
forfeit the confiscated cattle, vehicle, premises, and material on behalf of the State
government. The Bill has exempted cattle operated for vaccine lymph, serum, or any
experimental or research purpose at any government or recognised institute from its
provisions. It has also allowed slaughter if it is certified by a veterinary officer as being
necessary in the interest of public health and in the case where the cattle are suffering from
any disease which is certified by veterinary officer as contagious and dangerous to other
cattle. Similarly, the slaughter of terminally-ill cattle and buffalo which is above the age of 13
will be allowed upon certification by veterinary officer. Interestingly, the new Bill seeks to
protect persons acting in good faith to prevent cow slaughter. 
68. Y operates a slaughterhouse. He sells fresh cattle and poultry in his shop as required by
his customers. An enquiry revealed that all the cattle are above 12 years of age. Is Y liable
under the Bill?

(a) No, because all cattle are above 12 years of age

(b) Yes, because the Bill bans slaughter of cattle

(c) No, because he is not intentionally killing the cattle

(d) None of the above

. Y sources his meat fresh from a local farm owned by Z. Z raises young cows and buffaloes
using ethical methods for sale and consumption. After buying cattle from Z, Y keeps the
animals in his shed and takes them out one by one for butchering only when he receives an
order. Many restaurants purchase their meat from Y for serving to their customers. Who all
will be liable as per the passage?

(a) Y, Z, restaurant owners who buy fresh meat from Y

(b) Y

(c) Z

(d) Y and Z

. Dom was a lorry driver. One day he was transporting draught cattle by his lorry to a nearby
farm for tilling the field. He was stopped mid-way for checking by the police and when they
saw cattle in his lorry, they convicted him under the Prevention of Slaughter and Preservation
of Cattle Act. Is Dom’s conviction valid?

(a) Yes, because he was disposing off cattle

(b) Yes, because he was transporting cattle with the intention to kill them

(c) No, because merely transporting cattle is not prohibited

(d) No, because he would never have slaughtered the cattle

. John Snow managed a cow shed or ‘gaushaala’ where he used to provide food and shelter
for stray cattle, both young and old. He came across a young 1-year-old calf that had a severe
foot infection which was rapidly spreading to its other body parts. He did not have the funds
to arrange a veterinary doctor. He did not want to keep the calf in his gaushala because he
was worried that the infection might spread to other cows. So he fed the calf and added
poison to its food to end the animal’s suffering. Is he liable under the Act?

(a) No, because he did not intend to kill the calf


(b) No, because he was acting in good faith to protect the other cows from getting infected

(c) Yes, because he killed the calf without getting it certified from a veterinary doctor

(d) Yes, because he intended to kill the calf

. Tara had two buffaloes on her farm. The buffaloes although otherwise healthy, were well
over 13 years of age and had lost milk giving capacity. Due to their old age, they were also
unfit for draught purpose. Paying for their upkeep was becoming increasingly unprofitable
for Tara so she decided to sell them to a slaughterhouse. Will she face be liable under the
Act?

(a) No, because the buffaloes were over 13 years of age

(b) No, because their upkeep was becoming unprofitable

(c) Yes, because it was unnecessary

(d) Yes, because the buffaloes were not terminally ill and were healthy

. Tara decided to donate her buffaloes to a government research institute that tested ayurvedic
products on animals before certifying them for human trials. The buffaloes died at the
institute because of the harsh dietary and environmental changes they were exposed to at the
institute. Is the institute liable for cattle slaughter?

(a) Yes, because they exposed the cows to harsh living conditions leading to their death

(b) Yes, because it killed two otherwise healthy buffaloes

(c) No, because the buffaloes died due to research purposes

(d) No, because the institute never intended to kill the buffaloes

Passage (Q.-Q.): The Supreme Court agreed to look into whether it should examine the
constitutionality of the proclamation of national Emergency in 19. The issue came up before
the court as -year old Veera Sarin is seeking compensation for the loss she suffered due to the
proclamation of emergency. On June 25, 19 then President Fakhruddin Ali Ahmed relying on
Article 352 of the Constitution declared a national Emergency in the country. In 19, the
President could impose Emergency if satisfied with the “persistence of a grave threat to the
security of the whole of India or a territory of India, either by war, external aggression or
internal disturbances.” From media censorship, suspension of civil liberties and attempts to
fundamentally change the Constitution to suit the government, the Emergency is reckoned as
a dark period in India’s democracy. The 38th- 42nd Constitutional amendments were passed
during the Emergency which led to a tussle between the executive and the judiciary that
would have a lasting impact on the Parliament’s power to amend the Constitution. Mrs
Gandhi sought to expand the power of the President and barred judicial review of the
proclamation of Emergency by the President or any ordinance issued by the President even if
it infringed upon the fundamental rights guaranteed under the Constitution. In the 42nd
amendment, the Parliament expanded its powers to amend the Constitution, even its ‘basic
structure’ and curtail any fundamental rights. Changes were made to the Preamble, which
was subsequently considered to be a part of the basic structure. Much of these changes were
corrected in the 44th amendment. Right to Property was removed from the list of
fundamental rights and made a legal right under Article 300A. The Press was also given the
right to report freely and without censorship the proceedings in the Legislature. Judicial
interventions were also made which prohibited any changes to the ‘basic structure’ of the
Constitution. Now, the provisions of the Constitution forming a part of its basic structure
cannot be amended. [Extracted with revisions from ‘Plea in SC against Emergency: Why is
the petitioner seeking relief now?’ by Apurva Vishwanath, published 18 Dec 2020 in
theindianexpress https://indianexpress.com/article/explained/explained-plea-in-supreme-
court-against-emergency-and[1]why-is-the-petitioner-seeking-relief-now-0/]

. The government wants to acquire the lands of rich landowners and zamindars to distribute
them to poor tenants who work on the farms. It wants to do so to make land ownership more
equitable. Can it do so?

(a) No, because right to property is a fundamental right

(b) No, because right to property is a constitutional right

(c) No, because the government cannot infringe upon fundamental rights

(d) None of these

. Saif was a farmer in Punjab who owned 200 bighas of land near the highway. The
Legislature passed an order to take over the lands near the highway without any
compensation to expand the length of the highway. Can Saif take any recourse to the law to
get compensation?

(a) No, because Right to Property is not a fundamental right

(b) No, because he is not liable for compensation since it is a reasonable infringement

(c) Yes, because his Right to Property is being violated

(d) Yes, because the Executive can be made liable for its actions after the Emergency

. ‘Patrika’ a leading national daily published a detailed description of parliamentary


proceedings which turned violent due to opposing policies of different parties that
condemned a bill for discussion by creating a ruckus. The government asked Patrika to
censor the article and omit the violent outcome of the proceedings. Can it do so?

(a) No, because right to free press is a fundamental right in the Constitution

(b) No, because media has the right to report parliamentary proceedings

(c) Yes, because the Legislature can ask media to censor content during an Emergency
(d) Yes, because censorship of parliamentary proceedings is allowed

. The Legislature seeks to add the phrase ‘to ensure domestic tranquillity’ in the Constitution
to foster a spirit of peace and harmony in light of increasing domestic dissatisfaction amongst
different groups. Is this action valid?

(a) Yes, because the Preamble is a part of the basic structure

(b) Yes, because the Preamble can be amended

(c) No, because it is arbitrary

(d) No, because the Preamble is a part of the basic structure

. The Parliament seeks to make a new law which will prohibit reservation of seats for women
in Local Bodies. Currently, such reservation of seats is allowed under Article 15(3) of the
Constitution. Is this action valid? 

(a) Yes, because it can amend the constitution

(b) Yes, because reservation of seats for women is unnecessary

(c) Yes, because it is a part of the basic structure of the Constitution

(d) No, because it is a fundamental right in the Constitution

(Passage Q. – Q.): Genocide: - "any of the following acts committed with intent to destroy, in
whole or in part, a national, ethnical, racial or religious group, as such: killing members of the
group; causing serious bodily or mental harm to members of the group; deliberately inflicting
on the group conditions of life calculated to bring about its physical destruction in whole or in
part; imposing measures intended to prevent births within the group; [and] forcibly
transferring children of the group to another group." Universal jurisdiction is a unique
concept which, in its broadest sense, allows states to exercise criminal jurisdiction against a
person accused of grave international crimes regardless of the place in which the crime was
committed or the nationality of the accused or the victim. Certain countries have used this
principle to try individuals in the past and as per a report published by Amnesty International
in 2012, various states have incorporated different forms of universal jurisdiction for
genocide, crimes against humanity and torture. India has so far failed to criminalise grave
international crimes such as genocide and crimes against humanity. It ratified the Genocide
Convention, 11 in 19. However, to date, there exists no domestic law on genocide. Also,
India has not yet included the provision on crimes against humanity in its domestic criminal
code. Although the Rome Statute of the International Criminal Court defines genocide,
crimes against humanity and war crimes in 2002 (and the crime of aggression in 2010), India
is not a state party to the statute. [Extracted, with edits and revisions from “Argentina Is
Taking a Unique Route to Try Myanmar's Leaders for Crimes on Rohingya”
https://thewire.in/rights/argentina-universal-jurisdiction-myanmar[1]rohingyas
. Pakodastan is a country signatory to Geneva Convention. It has 2 religious’ sections DBZ
and Shippuden. % of the population follow Shippuden and the rest follow DBZ, both the
religious sections in 2002 had a fight over whose god is stronger and the evidences suggested
that God of DBZ is the strongest. This conclusion sends a wave of unrest amongst the
Shippudens. Military and the government have % of Shippudens, owing to the majority in the
parliament the government of Pakodastan made a law to uplift the minority and made it
mandatory for the DBZ to send their kids for military training in order to increase their
immunity, the law was met with harsh criticism and association of DBZ refused to follow this
harsh law. Is the new law Genocide?

(a) Yes, taking away the kids forcefully is a genocide.

(b) No, the kids aren’t taken forcefully but with law and good intentions, cannot be genocide.

(c) Yes, because the kids of minority are taken away forcefully, i.e. one group to the other
group.

(d) No, genocide requires killing of people of certain group at an exceptionally huge level.
Taking away kids does not qualify to be a genocide.

. The DBZ kids are forcefully taken to the military camps and they are trained extensively,
after a month 40% of the kids could not survive the training but the remaining became super
soldiers with immune system as strong as a vaccinated person. Though the kids were
traumatised and refused to come back to civilization and they now believe in the spirits of the
jungle and the rule of the jungle is their law. Decide.

(a) The DBZ kids ultimately lost their faith; therefore, Shippudens are a part of genocide.

(b) The death of 40% of the kids was the price paid by the DBZ community to make their
future generation stronger; the rest % has a very well-developed immune system. Therefore,
not a genocide.

(c) None of the above.

(d) This was not genocide as the intent of the Government was not to destroy religious group
DBZ but to make their kids stronger which ultimately happened.

. What is the most probable reason for including ‘forcibly transferring children of the group
to another group’ in the definition of genocide?

(a) Children represents the future of the whole generation, taking away the kids forcefully
could be used as a bargaining chip later on.

(b) Children are innocent they can be brain washed into believing anything, they must be
protected and therefore they have been included in definition of genocide to stop such
kidnapping.

(c) Children are the future, annihilating them or their beliefs annihilates the future of the
whole community, if they are allowed to be taken away or killed the whole new generation is
gone, therefore a genocide.
(d) All of the above.

. India, being a neighbour of Pakodastan, is concerned about the wellbeing of DBZ minority
and seeks to prosecute the officials of Pakodastan, is India capable of prosecuting them?

(a) Yes, under universal jurisdiction country has a right to prosecute people for international
crimes.

(b) No, because India is not a party to the convention that allows the country to prosecute
international criminals.

(c) Yes, India can prosecute them to safe guard the humanity and make the criminals pay for
genocide.

(d) No, India has not inserted any law related to genocide in their domestic law. Absence of
such provisions restricts the jurisdiction of India.

. Three weeks after DBZ lost their kids, they revolted against the government of Pakodastan
and the majoritarian regime was over thrown. The officials responsible for the law ran away.
Now the new government & military with equal representation made it mandatory for all
children to join the military training regardless of their religion. Shippudens didn’t protest
against the law but % of the Shippudens were against sending their children, regardless they
were sent to the military camp. This time results were worst, % of the Shippuden children
died and the rest were too traumatised to get back in the society. Is it genocide?

(a) Yes, because the children were taken forcefully and DBZ wanted to take revenge on the
Shippuden.

(b) No, this is not genocide as the children of all religion were asked to join the military, it
wasn’t selective.

(c) Yes, definitely genocide % of the new generation of Shippuden died.

(d) No, it is not genocide.

. Which amongst the following factors was also present in the factual matrix?

(a) Killing members of the group;

(b) Imposing measures intended to prevent births within the group;

(c) Deliberately inflicting on the group conditions of life calculated to bring about its physical
destruction in whole or in part.

(d) Both a & c.

 
Passage (Q. – Q.): The provision of sedition in India has been mentioned within Section
124A of the IPC, and has been subsequently dealt with in a number of cases. A person is said
to have committed sedition when he or she by way of words or by signs and visual
representations, brings or attempts to bring in hatred or contempt or incite disaffection
towards the lawful government. Sedition is punishable with life imprisonment along with fine
or with imprisonment which may extend up to a period of three years along with fine.
Sedition is considered by many as being opposed to the freedom of speech and expression
guaranteed under Article 19(1)(a) however the position regarding the constitutionality of
Section 124A was settled in Kedar Nath v. State of Bihar. This landmark judgment on
sedition upheld the constitutionality of Section 124A of the Indian Penal Code, 18, and ruled
that the offense of sedition shall be framed only when the alleged seditious speech or
expression made was intended at inciting violence, hampering public disorder or peace. The
HC did uphold Section 124A of the Indian Penal Code, 18 but narrowed down its scope so as
to maintain the sanctity of freedom of speech and expression which is a fundamental right
guaranteed to all the citizens of India. Sedition as a penal offense was included in the IPC
during the times of British Rule in India. It was then considered as a severe weapon deployed
by the British for suppressing the voices of the Indians seeking independence from the British
rule. Even after seven decades of existence, the viewpoint that Section 124A is a draconian
piece of law aimed at suppressing the voice of the masses is prevalent as it is a restriction on
the fundamental right to freedom of speech and expression as detailed under Article 19(2) of
the Constitution. [Extracted, with edits and revisions from “Freedom of speech and sedition:
the Amulya Leona case” https://blog.ipleaders.in/freedom-of-speech-and-sedition-the-
amulya-leona-case/amp/

. In a public gathering, Gajendar Singh, the leader of the opposition party stated that “The
current Government of foolish and scoundrels, is full of scamsters. They deserve to be
overthrown and we should do it, we should teach them a lesson by the time results of the next
election comes” Gajendar Singh is being prosecuted for sedition. Decide.

(a) Gajendar Singh is guilty of sedition for making such provocative and agitating statements
against the Government.

(b) Gajendar Singh is not guilty of sedition because his words cannot incite the people against
the government leading to violence and breakdown of law and order.

(c) Gajendar Singh is not guilty of sedition as he is merely exercising his freedom of speech
and expression.

(d) Gajendar Singh is guilty of sedition as his words are capable of instigating and provoking
violence amongst the people.  

. Which one of the following is punishable as sedition?

(a) Criticism of the government with the intention to overthrow it.

(b) Inducing people to cease to obey the law and lawful authority.
(c) A publicist and attack on the policies of the government.

(d) An attempt to remove the ministers from power.

. Based on complaints against the students from NLUI, for raising slogans like “Azaadi”
during a protest, the police arrested them. The students claim that they were protesting
against the delay in announcing the dates for the students’ union elections.

(a) The students are not guilty of sedition as they are only exercising their freedom of speech
in public.

(b) The students are guilty of sedition as their statement would incite students towards
violence leading to protest against the college authorities, which is State under Article 12 of
the Constitution of India.

(c) The students are not guilty of sedition as they were merely protesting against mal
administration and did not make any statement transpiring into contempt or attempt
disaffection towards the Government.

(d) None of these

. Zorawar Ahmed, the controversial stand-up comedian made the following two statements at
his stand-up comedy routine show – I. I was watching The Burning Train on TV. My father
came and told me not to watch such nonsense and turned off the channel. I was like, ‘why
so?’ He was like this “it is the video of Godhra kaand.” and this is a news channel. I said “I
thought it is a movie directed by Ommit Shah (home minister), produced by RSS. I don’t
know also if the 2002 Godhra ‘cartoon’ also was a piece of planned art”. II. Remove all these
corrupts and rascals, who are running this nation, whether they are leaders of the ruling party
or the government servants, come help me clean this nation and destroy these politicians.
DECISION:

(a) Zorawar is not guilty of sedition for having made the statement (i) and (ii) as he was
exercising his fundamental right of freedom of speech and expression.

(b) Zorawar is guilty of sedition for having made the statement (i) and (ii).

(c) Zorawar is guilty of sedition for making statement (i) only.

(d) None of the above.

Passage (Q. - Q.): In a marriage, the partners do not hand over their right to bodily autonomy
to the other. Each partner in the relationship has the right to their privacy and independence.
Article 14 affords every person equality in gender. In 17th Century England, Sir Matthew
Hale developed the contractual consent theory wherein he posited, “the husband cannot be
guilty of a rape committed by himself upon his lawful wife, for by their mutual consent and
contract, the wife has given up herself this kind unto her husband which she cannot retract.”
This is the harsh reality even today in the 21st century. Such a situation rips the wife of her
fundamental rights. The criminalization of marital rape is premised on the basic argument
that while in marriage the spouse has the right to have sex with his/her partner, that right
cannot be imposed by force, coercion or abuse of authority. The current law in India
concerning marital rape is in a sorry state. Section 3 of the Indian Penal code (IPC) states
(state amendment), ‘sexual intercourse by a man with his own wife, the wife not being under
fifteen years of age, is not rape’. While the legal age to marry for women in India is 18, the
amendment to Section 3 comes into question, ultimately rendering it redundant. In the eyes of
law, the relationship between the victim and the perpetrator should not matter, and if there is
coercion for sex it is rape and must be criminalized. In July 2018, it was stated by a two-
judge bench that in a relationship like marriage, both men and women have the right to say
“no” to physical relations. The central government in its reply before the bench submitted an
affidavit stating, ‘criminalizing marital rape may destabilize the institution of marriage’.
These remarks by the state reflect the ingrained sense of patriarchy which our systems have
internalised. [Extracted, with edits and revisions from, Rape in the Secrecy of Marriage: Need
for Criminalization, December 7, 2020, https://www.lawctopus.com/academike/rape-in-
marriage-need-for-criminalization/]  

. A and B, both adults, are in a live-in relationship and are sexually active with each other.
One day, A comes home from work, having had a rough day, he tries to engage in sexual
intercourse with B. B tries to reject his advances on that particular day, however A continues
to coerce her and even behaves roughly with her, and ultimately does engage in sexual
intercourse with B. According to the author of this passage, should such an act on part of A
come under the ambit of marital rape?

(a) No, this will not come under the ambit of marital rape because A and B were sexually
active, and that is considered a good reason to engage in sexual intercourse whenever they
want, even if one of the partners does not consent to it on a particular day.

(b) No, this will not come under the ambit of marital rape as the basic necessity of marital
rape is marriage, an institution which A and B had not entered into.

(c) Yes, this will be considered marital rape because in a relationship, the partners do have
the right to say “no” to sexual intercourse, and there can be no coercion to engage in the
same.

(d) No, this won’t come under the ambit of marital rape as both A and B are sexually-active
adults and should engage in sexual intercourse if one of them is going through a rough day.

. A and B have been married for three years, and have a child together. A is a worker in a
shoe-making factory. Recently, A has lost his job due to the global covid-19 pandemic, and in
the pressure of feeding his family without any source of income has been an alcoholic and
gambler. One night, upset over his loss in gambling he forcefully tries to have sex with his
wife who rejects his advances. However, A forces himself over B and threatens to harm their
child if she does not have sex with him. Having left with no other option, B does as she is
told. According to the passage, can this come under the ambit of marital rape?

(a) Yes, this can come under the ambit of marital rape as even in marriage the partners have
the right to say ‘no’ to sex and there can be no form of coercion in order to engage in sex.
(b) No, this does not come under the ambit of marital rape, as what goes on in the private
bedchambers of two people cannot be ascertained by the State.

(c) Yes, this can come under marital rape as the couple was married and using coercive
methods to engage in sex in a marriage is wrong.

(d) No, this cannot come under marital rape as the husband is the superior figure in a
marriage and has right over the body of his wife.

. According to the passage, does the non-criminalization of marital rape have to do with the
patriarchal notions of the Indian society?

(a) Yes, because the society believes that the husband is the supreme lord in a marriage and if
he refused something in the relationship, it might destabilize the marriage.

(b) No, marital sex has not been criminalized only because it is not possible for the State to
know for sure what went on in private between two individuals, thus it is difficult to trust one
party and declare the other guilty based on one person’s statement.

(c) No, because what goes on between two private individuals are not to be adjudged by the
society’s norms and regulations as it is their personal matter.

(d) No, men are aware of the fact that they don’t have authority over a woman’s body.

. Ramesh and Sheetal are having problems in their marital life and have become bitter
towards each other. One day, Ramesh has his friends Anuj and Raj over for drinks, and in a
state of intoxication and rage he allows his friends to rape Sheetal. Will this act of Anuj and
Raj constitute as marital rape?

(a) No, because marital rape can only be done by a partner or spouse.

(b) Yes, because Ramesh is the husband and rape done on his saying is considered as marital
rape.

(c) Yes, because rape done in the household of the victim is called marital rape.

(d) No, because the men were not in their right state of mind due to the intoxication and thus
should be excused.  

Passage (Q.-Q.): Conceding that the state had the right to impose Prohibition and confiscate
property, the Bombay High Court held that the legislature had the power to regulate only
“alcoholic liquors” and not all “intoxicating liquors,” as the Bombay PA sought to do.
Therefore, articles of everyday use that contained alcohol but were not ordinarily used as a
beverage, such as eau de cologne or medicines, could not be proscribed under the act.
Although the court recognized the possibility that such liquids could be used as a substitute
for alcohol, the state could not prohibit their legitimate use, and in fact such a ban would
violate Article 19, which included the right to acquire and hold property. Further restrictions
on medicines and cosmetics would hurt the objectives of public health and social welfare
promulgated by the state. Thus, although the court did not recognize a fundamental right to
drink, it argued that the right to property conferred on citizens the right to possess liquids
containing alcohol. The exemption granted to the army was held to be arbitrary and without
any reasonable basis. The judges rejected any attempt to show that the army should be treated
differently from Indian citizens. Since India was not a military state, the judges ruled, the
army was not entitled to special rights and privileges. The judges were particularly scathing
toward attempts to distinguish the army from other categories of public servants. The
maintenance of public order could not be said to be a higher order of responsibility, because a
similar responsibility was conferred on the police, and they received no such exceptions
[Source: Rohit De, A People’s Constitution: The Everyday Life of Law in the Indian
Republic, Chapter 1, Page 48] Answer the following questions, assuming the above decision
is valid law:

. Akash is a general store vendor in Maharashtra who sells a variety of items. Since he is the
only such vendor in his locality and due to the high population of women in that area, the
product “acetone” (propanone) sells like hot cakes. India’s largest supply of acetone comes
from China, but due to the pandemic and political tensions, the import has been significantly
disrupted. The Maharashtra government passed a legislation requiring vendors to limit the
amount of acetone they sell every month. Akash is aggrieved by this decision and seeks to
challenge it. Will he succeed?

(a) Yes, the State restricting any usage or sale of alcoholic liquids is violative of right to
property under Article 19.

(b) No, the context surrounding the legislation indicates that the restriction was reasonable,
thus it must stand.

(c) This a political act by the government to infuse nationalism and hatred against China. In
this context, it cannot be said that the restriction is a reasonable restriction.

(d) Yes, the right to use any form of alcohol is a fundamental right based on the above
decision.

. Suppose the above legislation restricted sale of Russian vodka due to supply chain
disruptions from the pandemic. What would be the most effective argument before a court?

(a) The restriction against all forms of alcohol consumption violates fundamental rights of an
individual.

(b) The right to consume intoxicating alcohol is not a fundamental right, and the State has
legitimate reasons to restrict consumption.

(c) The restriction cannot be validly invoked as there is no rational basis to discriminate
against Vodka.
(d) For the state to impose such a restriction validly, there must be shown a rational basis to
impose the restriction on individuals.

. The government in Maharashtra also passes a law where it establishes a sophisticated


system of ID cards, wherein those who have issues with intoxicants will not be allowed more
than a prescribed amount of alcohol per week. Kizzyanu, a drunkard, is extremely displeased
with this law, and saves money by not drinking for 10 days, and uses that to challenge this
law. Will he succeed?

(a) Yes, the law arbitrarily and unreasonably discriminates against Kizzyanu, like the
Bombay legislation did against non-army persons.

(b) Yes, in accordance with the reasoning above, every citizen has a fundamental right to
consume alcoholic beverages and the same cannot be restricted.

(c) No, the restriction is both proportionate and based on a rational distinction from other
categories, unlike the Bombay legislation.

(d) Yes, the restriction is not a proportionate restriction on Kizzyanu’s fundamental right to
consume alcohol.  . The Armed Forces Special Powers Act as imposed in Jammu & Kashmir
and several parts of North Eastern States provides special powers to the army. Under the act,
a military official is exempted from prosecution without consent from the central government
for any killings/offences done by them either of terrorists or of local people. Using his
powers, an army man kills a stone pelter in Kashmir and is not prosecuted for the same. The
stone pelter’s family seeks to challenge the AFSPA. What would the court hold according to
the passage above?

(a) The army is a special force installed in the state with AFSPA powers for a special
purpose. Thus, the restriction on their prosecution is rational and proportional.

(b) Since India is not a military state, there is no special immunity that must be granted to the
army and thus the law is invalid.

(c) A balance must be struck between the two actions. Even if the AFSPA is valid due to the
special nature of the army’s positioning in Kashmir, the action of killing of stone pelter must
be prosecuted.(d) India is a republic and the army’s powers must be comparable to that of the
police since both are public servants. Thus, the AFSPA cannot validly stand.

Passage (Q. - Q.): The offense of criminal breach of trust, as defined under section 405 of
IPC, is similar to the offense of ‘embezzlement’ under the English law. A reading of the
section suggests that the gist of the offense of criminal breach of trust is ‘dishonest
misappropriation’ or ‘conversion to own use’ another’s property, which is nothing but the
offense of criminal misappropriation defined u/s 403. The essential ingredients of the offense
of criminal breach of trust are (1) The accused must be entrusted with the property or with
dominion over it, (2) The person so entrusted must use that property, or; (3) The accused
must dishonestly use or dispose of that property or wilfully suffer any other person to do so in
violation, (a) of any direction of law prescribing the mode in which such trust is to be
discharged, or; (b) of any legal contract made touching the discharge of such trust. For
criminal breach of trust, the accused is entrusted with property or with dominion or control
over the property. As the title to the offense itself suggests, entrustment or property is an
essential requirement before any offense under this section takes place. The language of the
section is very wide. The words used are ‘in any manner entrusted with property’. So, it
extends to entrustments of all kinds-whether to clerks, servants, business partners or other
persons, provided they are holding a position of trust. “The term “entrusted” found in Section
405, IPC governs not only the words “with the property” immediately following it but also
the words “or with any dominion over the property.” There are two distinct parts involved in
the commission of the offense of criminal breach of trust. The first consists of the creation of
an obligation in relation to the property over which dominion or control is acquired by the
accused. The second is misappropriation or dealing with the property dishonestly and
contrary to the terms of the obligation created[vi]. The principal ingredients of Criminal
Breach of Trust are thus ‘entrustment’ and ‘dishonest misappropriation’. [Source:
https://www.lawctopus.com/academike/criminal-breach-trust/ ]

. The Crime Department is investigating a major bank fraud case. One of the employees X,
who is entrusted with handling the matter has lost an important file which has information on
the whereabouts of the accused. Decide

(a) X has committed criminal breach of trust as he dishonestly disposed of the property

(b) X has not committed criminal breach of trust as he was not dishonest in disposing of the
property

(c) X has committed criminal breach for he was bribed

(d) X’s liability cannot be determined

. Please refer to the facts above. An intel is received that there was a phonecall between the
accused and X before the file went in missing. Decide.

(a) X has committed criminal breach of trust as he dishonestly disposed of the property

(b) X has not committed criminal breach of trust as he was not dishonest in disposing of the
property

(c) X has committed criminal breach for he was bribed

(d) X’s liability cannot be determined

. Anil and Bhadu are working at the police department. A phone call was traced at Bhadu’s
place late at night. In addition to a phone call, a man wearing plain clothes was also seen
taking money to Bhadu’s place and documents concerning an important case went missing on
the next day. This is viewed as credible evidence of bribery. Decide

(a) Bhadu has committed criminal breach of trust as he dishonestly disposed of the property
(b) Bhadu has not committed criminal breach of trust as he was not dishonest in disposing of
the property

(c) The very allegation of bribery is enough to show that Bhadu has committed criminal
breach of trust.

(d) Bhadu is a public servant within the meaning of the IPC and thus, he cannot be held liable
for criminal breach of trust.

. Albert wanted to give some documents to his junior. The documents were entrusted to the
junior X over a phone call. X agreed. After hanging up the phone, Albert asked the peon to
deliver the documents to X’s office. The file never reached X’s office.

(a) The peon is liable for criminal breach of trust as he was entrusted with the documents

(b) The peon is not liable for criminal breach of trust as he was not entrusted with the
documents. He only delivered it.

(c) The peon is liable for criminal breach of trust and X is vicariously liable

(d) The peon is liable for criminal breach of trust and Albert is vicariously liable for he was
the one who entrusted the file to the peon.

. Please refer to the facts above. It turns out that the peon had spilt some water onto the file.
Scared, he put the file out in the sun and lied to X about the superior not giving the file.
Decide

(a) The peon is liable for criminal breach of trust as he misappropriated the property with a
dishonest intention

(b) The peon is not liable for criminal breach of trust as he did not have a dishonest intention.

(c) The peon is liable for criminal breach of trust as he lied to X about his superior and thus
has a dishonest intent.

(d) The mere fact that the peon lied about it, is enough to show the presence of dishonest
intention.

. Anil took a file along with him with the intent of taking it away from the police station. Till
this time, there is no evidence of Anil accepting a bribe.

(a) Anil has committed criminal breach of trust as he dishonestly disposed of the property

(b) Anil has not committed criminal breach of trust as he was not dishonest in disposing of
the property

(c) Anil has not committed criminal breach of trust for he has not misused the property
(d) Anil’s liability is subject to further evidence of causing the government wrongful loss
which is the accepted metric of a dishonest intention

20
Passage (Q.68-Q.): Attorney-client privilege or lawyer-client privilege is the common law
concept that makes all communications between a lawyer and their client confidential, and
such confidentiality is protected by law. "Privileged professional communication" is the
immunity accorded to the communication between the legal advisor and the client. According
to the Black's Law Dictionary, attorney-client privilege is "a client's right to refuse to disclose
and to prevent any other person from disclosing confidential communications between the
client and the attorney." In Upjohn Co. v. United States, the United States Supreme Court
stated that by assuring confidentiality, the privilege encourages clients to make "full and
frank" disclosures to their attorneys, who are then better able to provide candid advice and
effective representation. In Woolley v. North London Railway (18-18) LR 4 CP 2, the court
held that information called for by the client and provided by an employee or a third-party
agent, on the request of, and for the purpose of submission to, the attorney may also be
protected. The Indian Evidence Act, 18 provides protection to professional communications
and confidential communications with the legal advisors under Sections 126, 127, 128, and
129. The Evidence Act encompasses within its scope attorneys, barristers, pleaders or vakils
and not in-house lawyers. Section 126 of the Act defines the scope of privilege attached to
professional communications in an attorney-client setting. Section 126 expressly prohibits
legal representatives from disclosing any communications exchanged with the client. It also
prohibits stating the contents or conditions of documents in possession of the legal
representative during the course of the professional engagement. In Kalikumar Pal v.
Rajkumar Pal 11, the court ruled that the communications between an attorney and client are
privileged even if they contain information from third parties. [Extracted from ‘What Is
Attorney-Client Privilege?’ by Afreen Alam & Hamza Lakdawala published 26 Dec 2020 in
livelaw.in https://www.livelaw.in/know-the-law/attorney-client-privilege-indian-evidence-
act-bar-council-of-india[1]rules-17]

68. Raju is the owner of Laxmi Chit Fund. He was scammed by Anuradha Enterprises for a
certain amount of money. The money that he lost also included money that he had received
from Baburao under false pretences in the hopes of doubling it. Raju engages a lawyer and
shares that he has been a victim of a scam. However, he is hesitant to share that the money
lost also included money which he had taken from Baburao under false pretences. Should he
do so?

(a) No, because Raju is liable for misrepresentation and this disclosure does not fall within
‘privileged professional communication’

(b) No, because Raju is a victim of fraud and hence, this disclosure is privileged in nature

(c) Yes, because Raju is a victim of fraud and hence, he will not be penalized for sharing such
information with his lawyer

(d) None of the above


. Subsequently, Baburao finds out that something is wrong. Baburao is now asking about the
details of the same from Raju’s lawyer since he has also been defrauded. Should he disclose
the details revealed by Raju?

(a) Yes, because the money belonged to Baburao and the lawyer is liable to disclose this
information to the true owner of the money

(b) Yes, because Raju is the agent of Baburao and hence, information shared by Raju to his
lawyer is not confidential since it concerns the principle also

(c) No, because this information is privileged professional communication and disclosure is
prohibited

(d) No, because the lawyer was not engaged by Baburao himself to make him accountable to
Baburao

. In a bid to find out the truth of the matter and what Raju shared with his lawyer, Baburao
engages Raju’s lawyer as his own. In their first meeting, Baburao asks him to share the
details of his conversations with Raju. Can the lawyer do so?

(a) No, because that information is still not privileged as per Upjohn Co. v. United States

(b) No, because that information is still confidential as per attorney-client privilege

(c) Yes, because that information is still privileged as per Upjohn Co. v. United States

(d) Yes, because that information is not privileged or confidential now that there is a new
attorney-client relationship

. Anuradha Enterprises engages Louis Litt as their lawyer. They share with Louis documents,
which included a detailed list of all the transactions through which they scammed people like
Raju. Is Louis bound to disclose this evidence to the police?

(a) Yes, because Louis Litt is the in-house lawyer of Anuradha Enterprises and is exempt
from such disclosure under the Indian Evidence Act

(b) Yes, because the documents were part of a “full and frank” disclosure as per Upjohn Co.
v United States

(c) No, because the documents were not part of a “full and frank” during the course of the
professional engagement as per the Evidence Act

(d) No, because it falls within the scope of privilege under Section 126 of the Indian
Evidence Act

. Mike Ross was caught by the police in a case of identity theft. Mike’s lawyer was Harvey
with whom he had discussed his options earlier and who was aware about the entire matter.
The prison in-charge Shawn offered Mike a deal to reduce his sentence if Mike revealed
Harvey’s strategies. Mike refused. Can Shawn force him to disclose the information as the
prison in-charge?

(a) No, because that information is part of a “full and frank disclosure” under privileged
professional communication

(b) No, because it cannot be confidential as it is not privileged professional communication

(c) Yes, because prison-in-charges can claim exemption under the Evidence Act

(d) Yes, because the same was upheld in Woolley v. North London Railway

Passage (Q.-Q.): The Supreme Court has observed that before punishing a person for
non[1]compliance of the decision of the Court, the Court must not only be satisfied about the
disobedience of any judgment, decree, direction, writ or other process but should also be
satisfied that such disobedience was wilful and intentional. The bench observed that a person
does not commit contempt of court if during the pendency of certain proceedings, he takes
recourse to other judicial proceedings open to him, even though the latter proceedings put the
other party at a loss. It said: "such action of a person which he takes in pursuance of his right
to take legal action in a court of law, will not amount to interfering with the course of justice,
even though that may require some action on the part of the other party in connection with his
own judicial proceedings. The principle is, that a party is free to take action to enforce his
legal right.” The Court reaffirmed the view that a person does not commit contempt of court
if during the pendency of certain proceedings, he takes recourse to other judicial proceedings
open to him, even though the latter proceedings put the other party at a loss. The Court said:
"In a contempt proceeding before a contemnor is held guilty and punished, the Court has to
record a finding, that such disobedience was wilful and intentional. It has been held, that if
from the circumstances of a particular case, though the Court is satisfied that there has been a
disobedience but such disobedience is the result of some compelling circumstances, under
which it is not possible for the contemnor to comply with the same, the Court may not punish
the alleged contemnor." [Extracted with revisions from ‘Only Wilful And Deliberate
Disobedience To Court's Order Amounts To Contempt: Supreme Court’, published 21 Jan
2021 in livelaw https://www.google.com/amp/s/www.livelaw.in/amp/top-stories/wilful-
deliberate-disobedience-contempt[1]supreme-court-16]

. Mujrim was being tried in a case of theft. The trial was ongoing and Mujrim was out on bail.
The condition of granting bail was that Mujrim must appear in Court every time he is called
and he must not move out of the state without taking prior permission from the police.
Mujrim was asked to appear in Court at 4 pm but he got late because he was stuck in traffic
even though he started on time. Can he be charged with contempt of Court?

(a) Yes, because his disobedience was wilful and intentional

(b) Yes, because his disobedience was not the result of some compelling circumstances,
under which it is not possible for Mujrim to reach the court on time

(c) No, because Mujrim’s disobedience is the result of some compelling circumstances, under
which it is not possible for him to reach the court on time

(d) No, because his disobedience was not wilful and intentional
 

. Akshara was fighting a case against Funny Company in Consumer Court because she had
not received her refund for a defective product. Funny Company asked Akshara to settle
outside Court. Akshara agreed and took her case back. In the settlement, she received less
compensation than she would have in a Court case. Did she commit contempt of the process
of the Consumer Court?

(a) No, because Akshara is free to take action to enforce her legal right

(b) Yes, because by settling outside court she has been put at loss as compared to fighting the
case in court

(c) Yes, because she took recourse to other proceedings during the pendency of her case with
the consumer court

(d) No, because she may take recourse to other proceedings during the pendency of her case
with the consumer court

. Dholu and Bholu were fighting a property battle. The Court directed Dholu to not sell his
ancestral property till further orders from the Court. Two weeks after the Court order, Bholu
found out that Dholu was making a deal with someone to sell his ancestral property. Dholu
was discussing offers for sale because he urgently needed money to pay for his wife's
surgery. Can Dholu be held in contempt of the Court order?

(a) Yes, because Dholu’s disobedience was not wilful or intentional

(b) Yes, because Dholu’s disobedience was not the result of some compelling circumstances,
under which was not possible for him to comply with the Court order

(c) No, because his disobedience of the Court order was wilful and intentional

(d) No, because Dholu’s disobedience was the result of some compelling circumstances,
under which it was not possible for him to comply with the Court order

. Dholu was a rich businessman and Bholu was the poorer of the two brothers. After
considering the facts and circumstances given in the previous question, would a Court now
execute the decree of a contempt proceeding?

(a) Yes, because Dholu’s disobedience was wilful and intentional since he was rich enough to
afford a surgery without selling the property

(b) No, because Dholu’s wife’s surgery was a compelling circumstance for the Court to
consider irrespective of whether he was rich or not
(c) No, because the only reason he sold the property was to pay for his wife’s surgery so his
disobedience was not wilful or intentional

(d) Yes, because his wife’s surgery was a compelling circumstance for the Court to consider

. Ranjhana, who was in love with Sonam, was accused in a love jihad case by Sonam's
parents. Ranjhana was out on bail on the condition that he be present in every single stage of
the trial, without fail. While Ranjhana was at home, communal violence broke out in his
colony. He went outside to break the mobs but instead, he got caught up in the fight and was
badly injured. Next day, he did not appear in Court. Can he be punished for contempt in
wilful disobedience?

(a) Yes, because he committed disobedience of a Court order by not appearing in Court
which was his condition of bail

(b) Yes, because there were circumstances which made it impossible for him to comply with
the conditions of the bail granted to him

(c) No, because there were no compelling circumstances as he himself went out to go and
control the mob

(d) No, because there were circumstances which made it impossible for him to comply with
the conditions of the bail granted to him

Passage (Q.-Q.): The petition was filed by an ex-army officer who was dismissed by the
Lucknow bench of the Armed Forces Tribunal in 2018 after attacking his senior officer with
stones. The defense presented before the court was that the officer was depressed since 2006
and had even been taking treatment for the same. The defence of unsoundness of mind which
is given as a general exception under section of the Indian Penal Code was contended before
the Supreme Court. The bench of Justices R. Nariman, Navin Sinha and Krishna Murthy had
held that depression does not qualify as unsoundness of mind. The provision under Indian
Penal Code states that nothing is an offence which is done by a person who, at the time of
doing it, by reason of unsoundness of mind, is incapable of knowing the nature of the act, or
that he is doing what is either wrong or contrary to law. Section is to be used in those cases
when the accused has no sense of right or wrong or any differentiation. The appeal for
pension was also dismissed along with the petition to reverse the Armed Forces Tribunal’s
order. Source-https://www.livelaw.in/top-stories/depression-not-unsoundness-of-mind-for-s--
ipc-sc-junks[1]appeal-against-dismissal-by-ex-armyman-1257

. Facts: Mr. Raichand was a businessman by profession having a vast empire stretched across
the country. He was well known in the aluminium and cement industry. In the year 2005,
when market plunged Mr Raichand incurred huge losses. Due to this, he was disturbed and
regularly fought with his wife, Nandini. One night after an altercation with his wife, in a fit of
rage, he stabbed her with a knife. He stabbed her 5 times and his sons Rahul and Rohan
caught him with the weapon on hearing the cries of their mother. Mr. Raichand was charged
with murder by the police but he took the defence of insanity. Decide whether his defense
will sustain.
(a) No, as the act was done in a fit of rage and not due to unsoundness of mind.

(b) No, since Mr. Raichand deliberately killed his wife to get rid of financial burden he had
after incurring huge loss in business.

(c) Yes, because Mr. Raichand cannot adjudge his act. The fact that he inflicted his wife with
deadly blows evidently proves that he was unable to judge the casualties caused by him.

(d) Yes, since Mr. Raichand was mentally disturbed due to huge losses in business and may
have been under insanity while committing the act.  

. Gordon had undergone treatment over a period of one year for depression. One night his
neighbours were woken up at 3 a.m. in the morning on hearing his wife, Siera cries of pain.
When they opened the door, they found Siera lying on the floor in a pool of blood and with a
lot of injury marks on her body. On seeing the neighbours, Gordon jumped out through an
open window and started running away from the crowd, but he was shortly intercepted by
some of the people who ran after him. Siera died shortly thereafter. Gordon confessed to
having attacked her after the neighbours took him to the police station, initially denying any
role in the affair. In his trial, Gordon cited his previous illness and asked for exemption from
the charges on the ground that he was incapable of knowing the nature of the act or that it
was wrong or contrary to law due to his mental condition. Would Gordon be successful in his
defence? Outcome:

(a) Gordon’s mental condition, marked by depression, rendered him incapable of knowing the
nature of the act, therefore he will succeed in claiming the defense of insanity.

(b) That Gordon ran away from the crowd on seeing them shows he knew what he had done
was wrong and thus he was afraid of being caught. Therefore he will not succeed in claiming
the defense of insanity.

(c) Though Gordon knew the nature of the act, his mental condition prevented him from
knowing it was wrong or contrary.

(d) That he tried to deny his act in front of the police initially before confessing shows that he
knew it was an act contrary to law.

. Ali suffered from schizophrenia. He was under the delusion that his neighbour Jasmine
wanted him to kill her ex-husband Shahid. While under treatment, one day Ali saw a man
loitering around Jasmin’s house. Thinking it was Shahid he talked the man into following
him home and promptly knifed him to death. When the police come to arrest him he claims
that Jasmin asked him to commit the murder and was rejoicing after doing a good deed.

(a) Ali has committed no offence.

(b) Ali has committed an offence as he showed a great presence of mind to kill Shahid.

(c) Ali has committed no offence as Jasmin was the mastermind of the plan.

(d) Both (a) and (c)

 
. Anay is a mentally ill person undergoing treatment. He has been responding to the treatment
so well that he was lately allowed to move around in his home. When his younger brother,
Sonu, returns home after work, he usually brings a piece of chocolate for Anay. Today,
however, Sonu gets very late at night in returning home, and therefore is unable to bring the
usual piece of chocolate. Anay demands his chocolate, and on being denied one, all hell
breaks loose and he runs out, jumps into his brother’s car, finds the keys in place, drives away
at a very high speed into a dark street and runs over a pavement dweller sleeping right on the
road, who in the process got seriously injured. Is Anay liable?

(a) The pavement dweller was reckless enough to sleep on mid road. Thus, he has only
himself to blame.

(b) Sonu is liable as he had carelessly left the keys on the jeep to be driven away, knowing
fully well that his brother was still somewhat mentally deranged.

(c) Anay is liable, as he was completely out of his mind.

(d) Nobody is liable.

. Rao is suffering from mental illness. Doctors have forbidden his family members to not to
leave him alone in the house. One day Rao was with his sister and 1 year old nephew, he
wished to take bath near the well. His sister accompanied him. While Rao was taking bath
near the well and his nephew was playing in soil, Rao saw him covered with dust, threw him
in the well so that he could have a good bath and get cleaned. As a result the 1 year old child
died. Decide.

(a) Rao is liable for murder.

(b) Rao is not liable for murder because he was of unsound mind.

(c) Rao is not liable for murder but he is liable for culpable homicide not amounting for
murder.

(d) Rao is liable for culpable homicide amounting to murder.

Passage (Q.-Q.): Now, the court has repeatedly said in its oral comments that it views the
protests as completely legal and part of the exercise of citizens’ rights under Article 19 of the
Constitution. And it has also said that the police alone can take a call on the security aspect.
The court is not even framing these cases before it in legal terms. But it has still gone ahead
and intervened in this manner, noting that the government has not been particularly
successful in negotiating with the farmers’ groups. It has decided that it can do better than the
government and appointed its own committee. If you look at the January 12 order, what is
striking is that the court does not even set out clearly what the legal grounds of challenge are.
The petition filed by the Bharatiya Kisan Party argues that under our constitutional scheme,
agriculture and farm produce are matters reserved under entries 14, 18, 30, 46, 47 and 48 of
List II of the Seventh Schedule to the Constitution, which lays out the subjects on which State
Legislatures are competent to enact law. The argument is that the Centre simply could not
pass the farm bills as it did not have legislative competence. Courts are, of course, competent
to issue stay orders on parliamentary laws, but they need to set out legal reasons. When you
look at the reasoning given by the court, in paragraph 8, the court says, “We are also of the
view that a stay of implementation of all the three farm laws for the present may assuage the
hurt feelings of the farmers and encourage them to come to the negotiating table with
confidence and good faith.” Now, this is a strange reason and arguably not a sound legal
reason. To issue a stay, courts usually state the legal and constitutional arguments which
make them take the view that the law, on its face, raises a question of constitutional violation.
But when the court cites “assuaging hurt feelings” of a section of people who are protesting
against the laws as a reason to stay a parliamentary law, the order does not seem to have a
clear legal basis. That’s why some commentators have asked whether the court is looking at
administrative concerns rather than legal considerations [Extracted, with edits, from, ‘Are
courts encroaching on the powers of the executive?’ Anuj Bhuwania and Arun
Thiruvengadam discuss this question in a conversation moderated by Jayant Sriram, The
Hindu, 22 Jan 2021, https://www.thehindu.com/opinion/op-ed/are-courts-encroaching-on-the-
powers-of-the[1]executive/article33.ece]

. TYP Lt. is a cargo company owned by the government which operates through ships. The
government decided to raise the price of the shipment transportation by 5,000 per week. This
impacted the small company owners to a heavy threshold and they started protesting against
the government. The protesters also went to the Supreme Court and prayed that the
government not implement new plans. Should the SC, according to the passage, take
cognisance of such case?

(a) Yes because it is a matter of national importance and the Supreme Court is the responsible
for helping the government in resolving the same.

(b) Yes, as the Supreme Court is the highest court in the country and has the responsibility of
checking the actions taken by the executive.

(c) No, because taking the policy an action is the legislative action and not the judicial action.

(d) No, because there is no question of legal issues involved in this matter.

. The parliament, after having several debates, issued the notice that a new kind of tax will be
levied on the purchase of Wheat and rice over the country. Representatives of the poor, after
failed discussions with the executive, filed a petition in the Supreme Court that the poor will
not be able to afford this tax. According to the passage, should the Apex Court pass any order
to interfere with the notice?

(a) Yes because if the Supreme Court won’t here the poor people of the country then the poor
people won’t have a say in the policy making that affects them the most.

(b) No, as the legislation has the responsibility to make policy decisions of the country.

(c) No, because the policy made by the legislation can only be checked by the executive who
will implement it.
(d) Yes, because the judiciary has the power to check the workings of the Legislative as well
and is not inferior to it.

. Which of the following would strengthen the author’s arguments, evoked in the passage? (a)
The government has to follow the rule of law and because of that it has to abide by every
decision the Apex court give it even if the Apex Court is entering into the domain of making
policies.

(b) The Supreme Court does not have the authority to make legislation or implement any
policies.

(c) The legislative in India is weaker than the Legislative in England and hence cannot pass
decisions on social issues without the intervention of the judiciary.

(d) The Apex Court should adjudge the matters in Social or Social-economic matters
according to the constitutional matters.

. Which, among the following, the authors of the above passage may agree with?

(a) The courts should not procure stay on implementation of the policies put forward by the
legislative authority of the state.

(b) The court should not take into account the issues of minorities, but as democratic
institutions should follow the majority opinion.

(c) The court should adjudicate on reasons which are constitutional or at least legal in nature.

(d) All of the above

. The Bombay municipal office, according to the Bombay municipal act, ordered the street
vendors who were living on the street to vacate the place as they were causing difficulty to
the people owing residence at the place. The vendors filed a petition in the Supreme Court
and argued that the municipal office is violating their right to life as per article 21. The Court,
according to the passage, should or should not take cognisance of such matter?

(a) The court should take such matter into cognisance as it involves the issue of fundamental
rights of the citizen of the country.

(b) The court should not interfere in the regulation as it has been according to the procedure
established by law.
(c) The court should not enter into the domain of executive, which in this case is the Bombay
municipal office.

(d) None of the above reasons are adequate as per the question.

Passage (Q.-Q.): One of the oldest, most pernicious and widespread forms of abuse of state
power in India involves the police and enforcement agencies selectively targeting political
and ideological opponents of the ruling dispensation to interrogate, humiliate, harass, arrest,
torture and imprison them, ostensibly on grounds unrelated to their ideology or politics, while
sparing comparably placed supporters and friends of rulers of the day. The problem is that the
illegality involved in this type of prosecution is not self-evident. At first glance, the
prosecution appears legally kosher— acting on information about legal infractions, the police
pursue the accused as per law. The illegality becomes plain when two legal questions are
clearly distinguished and separated: first, the legality of the exercise of prosecutorial
discretion in the selection of the accused for being investigated and prosecuted; and second,
the merits of the criminal case filed against them. The two are independent legal issues and
should not be wrongly conflated. On the first question, the applicable legal standard is that
while the police and prosecutors in common law jurisdictions enjoy vast discretion in
deciding who they may pursue and who they may spare, the choice of accused must not be
based on grounds that violate Constitutional rights, including the Article 14 right to equal
protection of the law. In the words of then Chief Justice W. Rehnquist of the United States
Supreme Court, “A selective-prosecution claim is not a defence on the merits to the criminal
charge itself, but an independent assertion that the prosecutor has brought the charge for
reasons forbidden by the Constitution.” The theory is that the Constitution cannot be violated
to uphold the law — such an approach would spell doom for the Constitution. The selective
prosecution claim must be adjudicated as a threshold issue, with the prosecution being
quashed at the outset of the criminal case if the claim is justified. [Extracted, with edits, from,
‘Defending liberty against political prosecution’, G. Mohan Gopal, The Hindu, 22 Jan 2021,
https://www.thehindu.com/opinion/lead/defending-liberty-against-political[1]prosecution/
article33.ece]

. A is a student is an esteemed university in India. He is known to be a political leader of the


opposition party. He, with some of his friends, takes on protests in the streets against the
Citizenship act passed by the government and during the protest attacked the policemen. He
was arrested by the policemen but his friends escaped from the scene. Is this an example of
selective prosecution?

(a) Yes as no one other than A has been arrested even after committing the same crime.

(b) No because A has actually committed a crime by attacking the police and the fact that he
is belonging to any political party is immaterial.

(c) No because his friends escaped.

(d) Yes, because protesting against the democratically elected government is against the rule
of law.

 
. Which of the following concur with the opinion of Justice Rehnquist?

(a) The accused will be free if he/she has committed crime but has selectively chosen.

(b) The accused will be not tried if he/she has not committed a crime but has been selectively
chosen.

(c) The accused will be charged, irrespective of how he/she has been chosen because there
are legal grounds available for the same.

(d) The accused being guilty and him being selectively chosen are not coherent question of
law.

. A and B were friends from childhood. One fine day B and C were drunk and they abused A
in front of her wife’s family. A was defamed and hence filed a suit against C but not (b) Will
A be successful?

(a) Yes because he filed a suit against the party he wanted to claim damages from.

(b) No because he selectively filed suit against only C and not B because B was his friend.

(c) Yes, because A and B were friends and one friend cannot defame other and B was only
Joking.

(d) None of the above

. which of the following will the author of the passage agree with?

(a) The accused should not be tried before confirming the intention behind such prosecution
in the court of law.

(b) A criminal should not be abridged from his/her right to life as per article 21.

(c) An alleged person should be given proper chance to explain why he/she has done, what
he/she has been charged for and given punishment as per his/her explanation.

(d) All of the Above

. Abx Lt. is a company which operates a mall in the sundarnagri area in Rajasthan. It has been
alleged for the adulteration of some food products provided in the shop due to which three
persons have died. The company owner is known to be a rival of the prime minister from a
long time. A petition is filed by the government as Union of India against the company. Will
this come under selective prosecution as per the passage?

(a) Yes as the company owner was a rival of the leader of the ruling party.
(b) No because the company has indeed alleged for some crime and it should be tried for the
same

(c) No because the company and the owner are different legal entities. The owner is a rival of
PM and not the company.

(d) No because there is no one who has not been prosecuted for the same alleged crime at the
same time.

Passage (Q.-Q.): The conception that house makers do not "work" or that they do not add
economic value to the household is a problematic idea that has persisted for many years and
must be overcome, Justice NV Ramana observed in a judgment delivered today in an appeal
arising out of a motor accident compensation claim. The court was disposing an appeal
arising out of Motor Accident Compensation Claim filed by heirs of a deceased couple who
died in an accident. One of the deceased in this case was a house maker. The judge penned a
separate opinion while concurring with the judgment authored by Justice Surya Kant. In his
opinion, the Judge noted that there are two distinct categories of situations wherein the Court
determines notional income of a victim. The first category of cases relates to those wherein
the victim was employed, but the claimants are not able to prove her actual income, wherein
the Court "guesses" 21 of 36 the income of the victim on the basis of the evidence on record,
like the quality of life being led by the victim and her family, the general earning of an
individual employed in that field, the qualifications of the victim, and other considerations,
the judge said. The second category of cases relates to those situations wherein the Court is
called upon to determine the income of a non-earning victim, such as a child, a student or a
homemaker. The judge also made the following observations: "...It is a recognition of the
multitude of women who are engaged in this activity, whether by choice or as a result of
social/cultural norms (...) It is an acceptance of the idea that these activities contribute in a
very real way to the economic condition of the family, and the economy of the nation,
regardless of the fact that it may have been traditionally excluded from economic analyses."
[Extracted, with edits and revisions, from “Conception That House Makers Do Not "Work"
Or That They Do Not Add Economic Value To The Household Is A Problematic Idea: SC In
Motor Vehicle Compensation Case”, LiveLaw News Network, Live Law, 5th Jan 2021;
https://www.livelaw.in/top[1]stories/supreme-court-homemakers-house-hold-work-
economic-value-motor-vehicles-comeepensation[1]19]

 . Amy and Jake are high school sweethearts who were raised in a small village. To create a
good life for themselves, they decide to move to the city and open their own establishment.
They start small and open a tea stall. Soon, after accumulating some savings and taking a
small loan, they open a small food truck. Thanks to Jake’s culinary skills and Amy’s financial
management, they start to gain popularity for their food, which consisted of fusion street
food. Their newfound popularity was noticed by many people, including one Doug Judy, who
was the local thug of the area. One day, he arrived on their truck before its opening hours and
threatened them to pay him his hafta, or to face the consequences. Amy and Jake refused.
Angered at their refusal, he flashed them his knife and threatened them to think about it for a
day or worry about the dire consequences later, and then left. Scared yet still not daunted,
they go on with their everyday tasks and do not pay the hafta, despite numerous threats made
by Doug Judy and his goons. After a few months pass in similar fashion, one night while with
his friends, Doug Judy gets extremely angry and decides to kill the couple and be done with
it. His friends advise him against it but he goes to their place anyway, and finds their truck
and house empty. When he was almost about to leave, he finds Amy on the other side of the
road, picking apples from a nearby tree. When she was crossing the road, a car came out of
nowhere and ran her over. Doug Judy rushed her to the hospital, but she was declared to be
brought dead. Jake files a Motor Vehicle Compensation Case as the sole heir of his wife. The
counsel for the other side contends that using the formula provided in the Motor Vehicles
Act, 19, the income of a spouse could be calculated as one-third of the income of the earning
surviving spouse, if the other spouse is a homemaker. If the previous statement is true, what
should be the outcome of the case?

(a) Amy’s notional income should be calculated as one third of Jake’s total income.

(b) Amy’s notional income should be calculated using any method that would be employed to
calculate the income of a non-earning member.

(c) Amy’s notional income should be estimated depending upon the quality of life, the
general earning of an individual employed in that field, her qualifications, etc.

(d) Amy’s notional income should be equal to that of Jake since they ran a business together.

. Which of the following ideas are discussed in the paragraph given above?

(a) The bias and reason behind the exclusion of the work put in by home makers in economic
analysis.

(b) The challenges faced in calculating the notional income of homemakers, and the steps
taken to resolve them.

(c) The criterion for estimating the income of earning individuals in Court, if it is not known.

(d) Methods employed to calculate the income of home-makers, children, and other non-
earning members.

. According to the passage, the meaning of the term ‘notional income’ could be inferred to be:

(a) It is an estimated figure for a person’s income, when it is not known, or cannot be proved.

(b) Notional Income can be referred to as an additional source of income along with one’s
primary income.

(c) It is a calculated figure used in place of a person’s income in case of it being not known in
cases of Motor Accident Compensation Claims.

(d) None of the above.

 
. Saroj is a homely, middle-aged woman. Her life is based in and around her house, and she is
extremely devoted in taking care of her immediate and distant family members. She has two
sons, and once they came of age, decided to leave their country and settle abroad
permanently. Saroj and her husband decided to stay back in their ancestral home, despite their
requests. With the kids being gone and there being not much work in the household, Saroj
decided to upon up a small, informal business by selling some homemade pickles in her
neighbourhood. Her business did not accrue much earnings, neither did it require much
investment, as it was done majorly to pass her time. However, it did go on for a long time
well, even after her husband died. A few years after the death of her partner, Saroj was run
over by a bus while she was crossing the road at a red light. Her sons filed a Motor Accident
Compensation Claim, being the heirs of the deceased. Will Saroj’s sons be compensated
adequately?

(a) Yes, and the notional income of Saroj will be estimated on a variety of factors, such as
quality of life being led by the victim and her family, the general earning of an individual
employed in that field, the qualifications of the victim, and other considerations.

(b) Yes, and the notional income of Saroj will be estimated on a variety of factors, such as
quality of life being led by the victim and her family, the general earning of an individual
employed in that field, the qualifications of the victim, and other considerations, provided her
actual income cannot be proved.

(c) Yes, as Saroj’s status as that of a home-maker should not affect her estimated income that
will be used in calculating compensation, and the bias of social and cultural norms, along
with erring economic analysis should not affect the calculation.

(d) None of the above.

Passage (Q.-Q.): The Centre on Wednesday moved a plea seeking clarification on an earlier
Supreme Court order decriminalising adultery, with the intention of retaining adultery as a
crime in the armed forces. The plea said that when jawans and officers are posted in
inhospitable areas, their families are taken care of at base camp by field unit officers, and
should not harbour any misgiving about any promiscuous activity. Attorney General K.K.
Venugopal told the bench that the judgement decriminalising adultery among men does not
take into account the Armed Forces Act. In the Army Act, there’s a provision where an
officer can be court-martialled for “unbecoming conduct”. In September 2018, the apex court
had declared Section 4 of the Indian Penal Code (IPC) – that makes adultery a punishable
offence for men – unconstitutional and struck it down. According to Section 4 of the IPC, a
man who has sex with a woman “whom he knows or has reason to believe to be the wife of
another man, without the consent or connivance of that man, such sexual intercourse not
amounting to the offence of rape, is guilty of the offence of adultery”. “Adultery can be
ground for civil issues including dissolution of marriage but it cannot be a criminal offence,”
the apex court had said. The apex court had also declared Section 1(1) and 1(2) of the CrPC,
which allows a husband to bring charges against the man with whom his wife committed
adultery, unconstitutional. Section 4, which treated a woman as a property of her husband,
criminalised adultery through patriarchal control over their bodies. Though the law
criminalised only men, it’s equally anti-women because it further violated a woman’s dignity
by preventing her from exercising her freedom and making choices for herself. [Extracted,
with edits and revisions, from “Centre Moves SC to Keep Adultery as a Crime in the Armed
Forces”, The Wire Staff, The Wire, 13th January 2021; https://thewire.in/law/centre-sc-
adultery-crime[1]armed-forces-patriarchy-gender-rights]

. Pearl and Tarun are in a loving marital relationship for the past seven years. Theirs was an
arranged marriage, and the continued to discover new things about each other every single
day. However, this fairy tale ended when Pearl suffered from a miscarriage, as a result of
which their relationship got strained. When things started to get worse, they decided to live
separately without going through with a divorce. Avinash was a co-worker and close friend
of Tarun. He knew about Pearl and Tarun’s strained relationship, and when heard the news of
them living separately, assumed that they were now divorced. Tarun started to meet Pearl
frequently at her residence, and as they had been lovers in the past, they got into a casual
relationship as well. All of this unbeknownst to Tarun, who did not know about Avinash and
Pearl’s past. One day, while stopping by at Pearl’s residence to finally hand over divorce
papers, he catches the two of them in the act. Deeply angered by his friend’s betrayal, he files
charges of adultery against Avinash. Avinash argues that he was unaware of the fact that
Pearl and Tarun were not divorced. What should be the verdict?

(a) Avinash should not be liable for adultery.

(b) Avinash will be liable for adultery as according to Section 4 of the Indian Penal Code, he
had engaged in sexual intercourse with Pearl, without Tarun’s consent.

(c) Avinash will not be liable for adultery, as he had no reason to believe that Pearl was the
wife of another man, since they lived separately.

(d) Avinash will be liable for adultery as it was his responsibility to confirm his assumptions
of a divorce which he failed to do, and hence he is liable for his actions.

. What could best explain the reason behind the Section 4 being ‘anti-woman’?

(a) The Section only charged men for an act that both the man and the woman in the sexual
relationship are responsible for.

(b) It assumes that woman are docile and quiet, and cannot be commit adultery as it is
unbecoming of their ‘innate nature’.

(c) The section assumes that wife is a husband’s property, hence the person violating such
property should be charged. It shows patriarchal control.

(d) None of the above.

. Swayam and Sharon met in high school, and have been in love ever since. Swayam had
always dreamt of joining the Army, and had preparing for it from the very start. Sharon
supported his dreams, even though she was always scared of his safety and well-being and
would rather prefer him being employed in any other profession. After Swayam got done
with his training, they soon got married and became parents to a girl named Sasha as well.
Swayam eventually leaves for his posting in an inhospitable area, and Sasha and his daughter
live in their assigned quarters at the base camp. Shivam was posted as a field officer at the
same camp, and was an acquaintance of the family. As they were living alone, with Sharon
having recently given birth, Shivam occasionally visited them in case they needed any help.
A few officers noticed Shivam’s and Sharon’s harmless friendship. This news eventually
reached to Swayam as well, who came back home and confronted Sharon immediately. They
had a big argument, wherein Swayam decided to divorce Sharon, however she still wanted to
continue the marriage. During the proceedings, Swayam’s legal counsel pleads for divorce on
the basis of allegations of adultery. Sharon’s legal counsel replies in defence that such action
is not possible, as adultery is no longer an offense. Is this defence valid?

(a) Yes, as Adultery has been struck off by the Apex Court in the case of Joseph Shine v.
Union of India.

(b) No, as divorce requires the consent of both the parties to dissolve the marriage, which is
not the case here.

(c) No, as adultery can be used as a ground for dissolution of marriage.

(d) Yes, as adultery has not taken place in the circumstances provided, as there has been no
sexual intercourse.

. Meet is hardworking soldier in the Indian Army, and has recently married Riya. Since it was
an arranged marriage decided by their parents, they did not know each other. They decided to
get to know each other, but could not gel well together romantically and decided they were
better off as friends. However, for the sake of their parents’ happiness, they decided to not get
divorced. A few months after, he met Kiya, who was the wife of his superior officer. She was
younger than her husband and was stuck in a physically and emotionally abusive relationship.
Love blossomed as soon as they saw each other, and started spending more and more time
with each other, since her husband was away executing his duties. One day, her husband
caught them going to the movies together, and discovered the relationship. He locked up Kiya
in her house, and filed a case against Meet accusing him of adultery and conduct unbecoming
of an officer. Will Meet be liable?

(a) Meet will not be liable.

(b) Meet will be liable for both the charges.

(c) Meet can be liable for conduct unbecoming of an officer only.

(d) Meet can be liable for adultery only.

Passage (Q.-Q.): India’s Data Protection Bill recommends that the person to whom the data
relates be given rights over its control. Such data may only be processed “on the consent
given by the data principle” [Section 11(1)]. Given the ease with which we consent to give
away our data in exchange for free apps, marginal discounts, and online games, the law fails
to address the core problem. Indian courts may ultimately be called upon to intervene and
balance the unequal bargaining power embedded in these data transactions. The Data
Protection Bill suggests that personal data should include data “…relating to a natural person
who is directly or indirectly identifiable, having regard to any characteristic, trait, attribute or
any other feature of the identity… or any combination of such features, or any combination of
such features with any other information…” [Section 3(28)]. Verbiage apart, the Bill
essentially says that any data that identifies you in connection with any other information is
your personal data. All that said, how does India propose to regulate all this information?
Borrowing heavily from the European Union’s General Data Protection Regulation, the Bill
obligates organizations to (a) act transparently in its processing of data, (b) implement
appropriate security measures, and (c) promptly notify the authority about any data breaches.
Additional obligations include yearly audits and accurate record keeping. In turn, individuals
have the right to (a) determine what personal information is being processed in the
organization’s possession, (b) request correction of any inaccurate information, and (c) have
the data portable to any other organization rendering the same service. Chillingly, the
proposed law does not grant a person the right to secure deletion of all his data in another’s
possession. The Bill proposes that it apply to (1) organizations that collect or process data in
India, or (2) foreign companies profiling the data of individuals within India. [Extracted, with
edits and revisions, from “The Great Data Protection Debate: India’s new Data Protection
Bill”, Rohin Dubey, Bar & Bench, 28th December 2020;
https://www.barandbench.com/columns/the[1]great-data-protection-debate-indias-new-data-
protection-bill]

. Among the following list of persons whom shall the Bill be applicable to at present?

(a) Citizens only.

(b) Citizens and residents both.

(c) Residents only

(d) The bill will not apply to any one.

. Assuming that the bill becomes an act, which of the following will be considered as Data in
the Data Protection Bill?

(a) Monthly sales projections of a Pharmacy firm.

(b) Database of an anonymous survey conducted by a think tank targeting MBBS students.

(c) Precipitation data pertaining to the states of Rajasthan and Madhya Pradesh for the first
quarter of 2019 collecting by Indian Meteorological Department.

(d) CV database of a law firm named Bhayana Chand and Sons.

. Mrs. Rai has been a practicing lawyer in India for more than two decades and has recently
been made partner in the firm AXV, having left her previous firm ZXV. Her data has been
transferred from the databases of ZXV to AXV, however, one of her details pertaining to her
education has been entered incorrectly but since she was with ZXV for so long, she was
given assurances that this wouldn’t be an issue. However, AXV requires correctly filled out
details and thus asks her to get the same corrected. Assuming that the Bill is in force as an
Act, what would be the correct course of action for Mrs. Rai?
(a) The fact that Mrs. Rai has had incorrect credentials stored in the database of her previous
firm just because she had been working there for eso long is a grave misuse of her position.

(b) Mrs. Rai has the right to get her data corrected under the ambit of the Act.

(c) It is unclear that the Act would govern the firm or not.

(d) The bill will not govern this firm as the personal data in question is not directly/indirectly
identifiable.

. Foodpallate co. is an online culinary giant that stores mounds and mounds of data pertaining
to its subscribers and through its infinitely complex algorithms creates and suggests palatable
cuisines for them. The company all of a sudden faces a data leak by a disgruntled ex-
employee with access to such sensitive data regarding its subscribers and threatens the
company to leak the data should he be prosecuted for the breach. What should be the ideal
course of action for the firm, assuming that the bill becomes an act?

(a) The firm should report the breach at the earliest so that the sensitive data stays under the
protection of the concerned authorities.

(b) The firm should give in to his demands so that he keeps the information safe.

(c) The firm should do nothing as the data has been willing provided by the subscribers and
has been transparently stored by the firm.

(d) The employee does not command any credibility and would be ridiculed by the intelligent
society should he leak the information.

21
(Passage Q.-Q.): The Allahabad high court recently granted pre-arrest bail to Mohammad
Chand, one of the people who had offered namaz inside the premises of the Nand Baba
temple in Mathura. In its order, the bench of Justice Siddharth noted that “irrational and
indiscriminate arrests are gross violation of human rights”. As for the government counsel’s
contention that “in view of the seriousness of the allegations made against the applicant, he
is not entitled to grant of anticipatory bail,” the bench held that this apprehension was “not
founded on any material on record”. Justice Siddharth further held that “personal liberty is a
very precious fundamental rights and it should be curtailed only when it becomes imperative.
According to the peculiar facts and circumstances of the peculiar case the arrest of an
accused should be made.” He added that “the courts have repeatedly held that arrest should
be the last option for the police and it should be restricted to those exceptional cases where
arresting the accused is imperative or his custodial interrogation is required. Irrational and
indiscriminate arrests are gross violation of human rights.” Granting anticipatory bail to the
accused, he said: “Hence without expressing any opinion on the merits of the case and
considering the nature of accusations and antecedents of applicant, he is directed to be
enlarged on anticipatory bail as per the Constitution Bench judgment of the Apex Court in
the case of Sushila Aggarwal vs. State (NCT of Delhi) 2020.” [Extracted with revisions from
‘Irrational, Indiscriminate Arrests Are Gross Violations of Human Rights: Allahabad HC in
Mathura Namaz Case’ published 23 Jan 2021 in thewire https://thewire.in/rights/irrational-
indiscriminate-arrests-are-gross-violations-of-human-rights-allahabad[1]hc-in-mathura-
namaz-case]
 

. A, B, C, D and E were a group of five friends. They call their group “Hum Paanch”. They
were severely disappointed with the government’s functioning so they decided to plant a
harmless smoke bomb to scare the people and show their dissatisfaction with the regime.
Before they could execute their plan and while they were still in the planning phase, B and C
were arrested for “terrorist conspiracy of bomb blast”. Is this an indiscriminate arrest as per
the passage?

(a) No, because irrational and indiscriminate arrests are gross violations of human rights

(b) No, because arresting them was imperative when considering the peculiar facts of the
case at hand

(c) No, because of the seriousness of the allegations made against them which is not an
exceptional case

(d) None of the above

. John and Rimjhim fell in love. Even though their families were not happy with their union
due to their vastly different religious backgrounds, John and Rimjhim decided to get married.
John wanted to get married the traditional way so he Rimjhim to convert to his religion.
Rimjhim agreed without any protest because she did not mind changing her religion. A few
days later, John was arrested by the police based on an anonymous tip for forcefully
converting Rimjhim to his religion without giving him an opportunity to say anything. Is this a
valid arrest?

(a) No, because personal liberty is an unrestricted fundamental right and should be curtailed
only when it becomes imperative

(b) Yes, because arrest is a necessary option since custodial interrogation of John is
required to get to the bottom of the matter

(c) No, because arrest should be the last option for the police and it should be restricted to
those exceptional cases where arresting the accused is imperative

(d) Yes, because arrest cannot be the last option for the police as the case of love jihad falls
in the bracket of exceptional cases

. Funawar Maruqi was a budding stand-up comedian. He was getting ready to perform his
show and the audience was seated in the auditorium. His comedy set included a few jokes
on Babri Masjid. Before he could even go on the stage to perform his set, police came to the
venue and arrested Maruqi. When Maruqi enquired about the reason of his arrest, he was
told that he was arrested on the “apprehension of making a joke that may have hurt religious
sentiments”. Is this an indiscriminate arrest?

(a) Yes, because his personal liberty is being violated with a valid justification

(b) Yes, because the totality of the fact imply that Maruqi’s arrest was not exceptional

(c) No, because irrational and indiscriminate arrests are gross violations of human rights and
Maruqi’s arrest was on a rational basis only
(d) No, because arresting Maruqi was imperative as his custodial interrogation was required
to ascertain the contents of his comedy set

. HP was an enthusiastic political activist who often addressed large rallies. He was very
vocal about his alienation from the government and devoted his life to bring the
inconsistencies of the government before public eyes. He decided to form his own political
party and before the elections itself, everyone knew that he was a strong contender for
becoming the next Prime Minister. Politician from Opposing party. HP was addressing one
rally when he said that “the highest court of the country seems to be in with the government”.
He was arrested for contempt of court. Is this a valid arrest?

(a) Yes, because of the seriousness of the allegations made against them which is not an
exceptional case to arrest

(b) No, because the allegations against him were not very serious and could have been
investigated without arresting him

(c) No, because HP’s personal liberty was curtailed without any rational basis and not
necessary for the case at hand

(d) Yes, because he could have incited people and curtailing his personal liberty was
necessary for the case at hand

(Passage Q.-Q.): The MP Freedom of Religion Bill 2020, which came into force in the state
on January 9, provides for one to five years of imprisonment and a fine of Rs 25,000, for
forcing religious conversion. A public interest litigation was filed by a Bhopal-based student
which states that “the provisions contained in the ordinance are a grave violation of
constitutional provisions and a blatant attack on the religious autonomy of individuals”. Under
the provisions of the anti-conversion law, it is mandatory to relay information to the district
collector days about the conversion before doing it. The petitioner has alleged that it is in
violation of the fundamental rights. Under the MP Religious Freedom Act 18, it was stated
that if a person was forced to convert, they could file a complaint to the police and the
collector. The MP Freedom of Religion Ordinance 2020 states that if a person is converted
under pressure, the person concerned as well as his parents and family can also file a
complaint to the police and the collector. Governments in some other states have also
decided to introduce laws aimed at preventing inter-faith marriage. The Haryana government
has formed a three-member drafting committee to frame a law on the matter. Karnataka and
Assam governments have made similar announcements. These actions are despite the fact
that in February 2019, the Centre told the Lok Sabha that no “case of ‘love jihad’ has been
reported by any of the central agencies”. [Extracted from ‘Madhya Pradesh HC issues notice
to state government on plea against anti-conversion law’, published 30 JAN 2021 on
thescroll https://scroll.in/latest/27/madhya-pradesh-hc-issues-notice-to-state-government-on-
plea-against[1]anti-conversion-law]

. Nora Fatehi was a 25 yr old Muslim woman. She fell in love with Raju, a 22 yr old Hindu
man. Raju told her that his family would not accept their union since Nora belonged to a
different religious community. Nora offered to change her religion. Nora's father threatened
her against doing so and said that if she changed her religion, he would file a case against
her. Is he correct?

(a) Yes, because her father could file a case for forceful conversion under the MP Religious
Freedom Act
(b) Yes, because her father could file a case for forceful conversion under the MP Freedom
of Religion Ordinance

(c) Yes, because her father, as a person concerned as well as being her parent can file a
complaint to the police and the collector

(d) None of the above

. In the year 19, Daku Mangal Singh kidnapped a young girl Babli. Babli belonged to religion
X and Daku Mangal was a devout follower of religion Y. He was mesmerized by Babli's
beauty and was desperate to marry her. In order to do so, he forced Babli to marry him by
converting to religion Y. Babli's father liked Daku Mangal Singh but was angry that he made
his daughter change her religion. Can he file a case against Daku Mangal Singh under the
laws mentioned in the passage?

(a) Yes, because her father, as a person concerned as well as being her parent can file a
complaint to the police and the collector

(b) Yes, because Babli was forced to convert her religion by Daku Mangal Singh under the
MP Freedom of Religion Act

(c) No, because Babli was not converted under pressure and it is a case of kidnapping

(d) None of the above

. If the previous incident would have taken place in 2021, would Babli be eligible to file a
case for forceful conversion against Daku Mangal Singh?

(a) Yes, because Babli is eligible to file a complaint to the police and the collector under the
MP Religious Freedom Act 18

(b) Yes, because Babli was kidnapped and converted under pressure by Daku Mangal Singh

(c) Yes, because forceful conversions are a grave violation of constitutional provisions and a
blatant attack on the religious autonomy of individuals

(d) No, because Babli is not eligible to file a case under the provisions of the MP Freedom of
Religion Ordinance

. Rimjhim was a follower of Zoroastrianism. She wanted to marry Soda Bottleopener wala
who was also a Parsi. In Soda's community, Parsis could only marry other Parsis. If Rimjhim
and Soda get married according to Parsi customs, can a case be filed for conversion
according to the laws mentioned in the passage?

(a) No, because Soda Bottleopenerwala asked Rimjhim to convert in violation of her
religious autonomy

(b) No, because Rimjhim has not violated the provisions of the MP Freedom of Religion
Ordinance

(c) No, because Rimjhim has not violated the provisions of any law given in the passage
(d) Yes, because Rimjhim is adopting Parsi customs under force and pressure

(Passage Q.-Q.): In a Delhi court order on Umar Khalid’s application against the media trial
against him, the Magistrate said that ‘any news item should be published after verifying and
clarifying all the facts’. The order specifically mentions one news item as an example of
problematic reportage. The order reads: In one of the news items, the news starts with
words “Radical Islamist and Anti Hindu Delhi Riots accused Umar Khalid....”. The news
report portrays the entire Delhi riots as anti-Hindu, when that is not the case. “However, in
fact this does not appear to be the case, as all the communities have felt the consequences
of those riots,” the order reads. Explaining how media reported on accused Umar Khalid
admitting to his involvement in Delhi riots, based on confessional or disclosure statements,
the order states that ‘a confessional statement made to a police official is not admissible as
evidence in law’. The order states that news reports only highlighted that Khalid had
confessed to his involvement, however none of them clarified to its readers that such a
statement could not be used by the prosecution as evidence. “A reporter should have such a
basic knowledge of law as readers/viewers consider news item as true without verifying the
facts. Further, general public might not be aware of the law as above mentioned.” Speaking
about how problematic reportage can deprive the accused of his dignity, the order reads that
any news item should be published after verifying and clarifying all the facts. “The deliberate
and unfair targeting of the applicant/accused by selectively quoting and misquoting is aimed
at damaging the reputation of the applicant/accused and destroying the presumption of the
innocence of the applicant/accused unless proved guilty,” the order notes. [Extracted with
revisions from ‘Delhi Court on Umar Khalid’s Trial by Media: News Must be Verified’, by
Aishwarya S Iyer, published 23 Jan 2021 in the quint
https://www.thequint.com/news/india/delhi-riots-2020-court-order-on-umar-khalid-media-trial-
plea#read[1]more]

. "Jan Hit Mein Jaari" is a popular news channel in Macchipuram. On 3rd November 2020, a
bomb blast occurred in a crowded marketplace and several people, belonging to different
communities lost their lives in the unfortunate incident. It was later found that the bomb blast
was caused by the terrorism group Al Qaeda. Jan Hit Mein Jaari reported the incident with
the headline, "Several Hindus died in religiously motivated terrorist act by Al Qaeda”. Is this
in consonance with the principle laid down in the passage?

(a) No, because there is no deliberate and unfair targeting or misquoting of the terrorist
attack

(b) No, because the headline was based on the facts, circumstances and nature of the event
being reported

(c) Yes, because any news item should be published after verifying and clarifying all the
facts

(d) Yes, because the headline constitutes deliberate and unfair targeting of Hindus

. The terrorist accused for committing this atrocious act on the soil of Macchipuram was
caught. While in police custody, he was tortured till he revealed the names of his
accomplices. He also confessed that the bomb was purposely planted in the crowded area
where the majority of the traders were Hindus. Can this be used to convict him in court for a
hate crime?
(a) Yes, because he can be convicted based on his confessional and disclosure statements

(b) Yes, because a confessional statement made to a police official is admissible as


evidence in law

(c) No, because a confessional statement made to a police official can be admissible as
evidence in law

(d) No, because a confessional statement made to a police official is not admissible as
evidence in law

. Jan Hit Mein Jaari also publishes a weekly newsletter. The governments of several
countries released statements of solidarity with Macchipuram and condemned the terrorist
attack. Jan Hit Mein Jaari reported these statements with the headline "International support
for the Hindu majority of Macchipuram pours in from all over the world". Based on your
reading of the passage, is this headline legally sound?

(a) Yes, because there is no indiscriminate or unfair targeting of any particular community
through selective quotations

(b) Yes, because the headline based on the international support is factually correct
quotation (c) No, because the headline is selectively quoting and misquoting the statements
of solidarity (d) No, because the headline is aimed at damaging the reputation of the
accused terrorist and destroying his presumption of the innocence  

. Jamun was taken into police custody at 6 am in the morning due to charges of drug
trafficking. Since Jamun was a famous celebrity, the media started reporting on this status
starting at 6 am itself. As time progressed, the media kept highlighting how Jamun had been
in custody for more than 12 hours at 7pm in the evening. There was constant reportage on
how Jamun, a poor young boy had been kept in custody without being granted bail for so
long. It is legal to keep an accused in custody for less than 24 hours. Jamun was granted
bail at 12am and finally released at 5 am next morning. Can the media be held liable for
problematic reportage?

(a) Yes, because the media misrepresented the position of the law in their constant
reportage without verifying the facts of the case

(b) Yes, because the general public may not have been aware of the position of law
regarding bail and the reportage was not concerning

(c) No, because a reporter cannot have such a basic knowledge of law as readers/viewers
consider news item as true without verifying the facts

(d) No, because the media did not deprive Jamun of his dignity even if the reportage was
problematic in nature

(Passage Q.-Q.): The 500-page long Whatsapp conversations between Goswami and Das
Gupta found their way into social media after Mumbai Police annexed it in its supplementary
charge filed in the TRP rigging case. The first thing we have to keep in mind is that an
"electronic record" is also included in the definition of "evidence" under Section 3 of the
Indian Evidence Act. It is treated as 'documentary evidence'. According to Section 2(1) (t) of
the Information Technology Act, an electronic record is "data, record or data generated,
image or sound stored, received or sent in an electronic form or micro film or computer-
generated micro fiche". The Evidence Act allows giving of evidence in respect of facts in
issue and relevant facts. So, an electronic record can be evidence in a trial if it relates to a
fact in issue or relevant facts of a case. Section B says that any information contained in an
electronic record which is printed on a paper, stored, recorded or copied in optical or
magnetic data produced by a computer will be deemed to be a document. But for such
records to be admissible as evidence, the certain conditions have to be fulfilled. There are
certain decisions which have observed that Whatsapp chats can be admitted in evidence
provided they fulfil the conditions under Section B of the Evidence Act. In Ambalal Sarabhai
Enterprise Ltd vs. KS Infra space LLP Limited and Another, the SC, while hearing a petition
challenging an injunction order made a reference to the Whatsapp chats produced as
evidence in the case. Whatsapp Forward without original cannot be evidence. The Delhi
High Court in a case has held that a Whatsapp forward message, without an unknown
source, cannot be treated as evidence (National Lawyers Campaign for Judicial
Transparency and Reforms vs. UOI). The Court held that such a forwarded message,
without its original, cannot be regarded as 'document' under the Evidence Act. [Source- ‘Are
Whatsapp Chats Admissible in Evidence?’, Live Law, , as accessed on 09th Feb. 2021]

. Sahil was a student of class 12th and was indulged in petty business of selling
pornographic films and pictures to his classmates for a considerate amount of money. He
also sold them to his friend Sohail. He asked Sohail to return the pen drive in two days after
copying the pornographic films. While copying them in his computer, Sohail’s father caught
him and asked him for the source he is getting such films. Sohail, in fear of being beaten up
by his father, revealed Sahil’s name. Subsequently complaint was filed against Sahil for
selling pirated copied of pornographic films. In a trial against Sahil,

(a) The court will admit the pen drive as the documentary evidence because electronic
records are considered to be evidence under section 3 of Evidence Act.

(b) The court will not admit the pen drive as the documentary evidence because it is not an
electronic record and hence not covered under section 3 of Evidence Act.

(c) The court will admit the pen drive as the documentary evidence as contents of the pen
drive will be considered to be electronic record and hence document under section B of
Evidence Act.

(d) The court will not admit the pen drive as the documentary evidence because its contents
are not electronic records and not covered under section B of Evidence Act.

. Priyank was good friend of Raman. Priyank was in need of money and asked Raman for
sum of Rs. 20 lakhs and assured to return it back in a month. After a month, when Raman
asked Priyank for the amount, he requested for extra time of 15 days to manage the amount.
Raman after 20 days from such request visited Priyank’s farm house and met him there.
They get into discussions and after an hour arguments the matter was solved. Priyank asked
Raman to wait for him outside at the swimming pool. Raman did the same. Priyank attacked
Raman from back, pushed him into the pool and due to blood loss, Raman died. The police
reported the murder and find a copy of CCTV footage from the farm house. Decide –

(a) CCTV footage is not admissible as documentary evidence in court of law as it does not
fall within the definition of electronic records.

(b) CCTV footage is admissible as documentary evidence in the court of as it is in the form
of electronic record and deemed to be document under Evidence Act.
(c) CCTV footage is not admissible in the court of law as documentary evidence as it must
be original record to be deemed to be document under Evidence Act.

(d) CCTV footage is admissible as it is deemed to be electronic record under IT Act but not
evidence under section B of Evidence Act.

. A murder case was reported in district of Uttar Pradesh. Manu was arrested by the police
as a suspect. The deceased was his girlfriend whom he cheated and was caught by her. The
police officers investigated the case and find evidence against Manu. In the trial for murder,
the prosecuting lawyer present the evidence i.e Telegram chats of Manu and the deceased
shared over the mails. The court held Manu guilty of murder. The defence lawyer contended
that chats over email were not evidence and cannot be admissible in court. Decide-

(a) Telegram Chats over emails are admissible evidence under section B of Evidence Act
but not under IT Act.

(b) Telegram Chats over emails are not admissible as the source is unknown and hence not
documentary evidence under Evidence Act.

(c) Telegram Chats over emails are not admissible evidence as they are not electronic
records as per IT Act and hence not a document.

(d) Telegram Chats over emails are admissible evidence as they are electronic records as
per IT Act and hence deemed to be document.

. Piyush, a Hindu boy, was truly devoted to lord Ram and was working in large community to
spread and promote religious teachings. One day he was lynched by people of Muslim
community, as he gets indulged in a debate with them. 5 accused were arrested by the
police and joint trial was started against them on the charges of mob lynching and murder of
an innocent boy. One of the evidences presented before the court was call recordings of the
accused persons to meet at the place where Piyush was lynched. Defence lawyer
contended these recordings to be inadmissible by the court. The court can-

(a) Consider the evidence as admissible as call recordings are documentary evidence and
fall within the ambit of electronic records related to relevant facts of the case.

(b) Consider the evidence as inadmissible as call recordings are not documentary evidence,
though electronic records related to relevant facts of the case.

(c) Consider the evidence as inadmissible as call recordings are not documentary evidence
and are not related to relevant facts of the case.

(d) Consider the evidence to be admissible as call recordings are documentary evidence and
electronic records, but are not related to relevant facts of the case.

. Rahul, a young boy of 23 years, was indulged in illegal recordings of movies and selling
their pirated copies before their official release. One of his customers was not satisfied with
the quality of the film and asked him to return back his money or else he will complain
against him for doing such illegal work. Rahul denied returning the money. A complaint was
filed against Rahul for selling pirated movies. In a trial, the plaintiff presented the CD/DVD he
bought from Rahul as evidence. The court rejected the evidence as inadmissible. The court
was –
(a) Correct as CDs and DVDs are not electronic records within the definition under section 2
of IT Act and hence not deemed to be document.

(b) Incorrect as CDs and DVDs are valid electronic records within the definition under
section 2 of IT Act and hence deemed to be document.

(c) Correct as CDs and DVDs are inadmissible documentary evidence as their source is
unknown and not conclusive in nature.

(d) Incorrect as CDs and DVDs are not electronic records but deemed to be documentary
evidence under section 3 of Evidence Act.

(Passage Q.-Q.): The time spent in “unlawful custody” cannot be included while computing
the days period prescribed for grant of default bail under section 1(2) of the Criminal
Procedure Code, the Bombay High Court held while rejecting senior journalist-activist
Gautam Navlakha’s petition for bail. Navlakha was arrested in the Elgar Parishad- Maoist
Links case, twice. He was placed under house arrest and released in October after his arrest
was declared illegal. The division bench held that the 34 days Navlakha had spent under
house arrest can’t be used to calculate his total detention period, especially since his arrest,
“As the transit remand order was stayed, it can’t be said that the appellant was under
detention of police for investigation. And the section 1(2) assumes the detention is
authorised by a magistrate, and days upward from that day can be used to calculate the
period of custody for default bail.” However, once the authorisation by the Magistrate is
declared illegal consequently rendering the detention itself illegal, the said period cannot be
construed to be an authorised custody.” The court said. The bench relied on the decision of
Chaganti Satyanarayan and others vs. State of Andhra Pradesh where the SC held that the
period of or days, as the case may be, will commence running only from the date of remand
and not from any anterior date inspite of the fact that the accused may have been taken into
custody earlier by a police officer and deprived of his liberty. A Magistrate can authorise the
detention of the accused for a maximum period of 15 days from the date of remand and
place the accused either in police custody or in judicial custody during the period of 15 days’
remand. Beyond that period no Magistrate can authorise the detention of the accused in
police custody.” [Source- ‘Default Bail- Period Of / Days Will Commence From Date Of
Remand Only, Not From Any “Unlawful Custody” Prior To It: Bombay High Court’, Live
Law, , as accessed on 10th Feb. 2021]

. Rohan was charged with the serious charges of rape and murder of a 12 year old minor girl
in village of Uttar Pradesh. The UP police arrested the accused on 11-10-9 from a hideout
area near slum locality. He was taken to Police Station on 12-10-2019 and was locked up for
further investigation. He was dictated of his grounds for arrest and presented before the
magistrate on 13-10-2019. The Magistrate granted the remand under police custody.
Decide-

(a) Rohan can be under remand of police custody till 26-10-2019 as period of remand is
exclusive of date of arrest and after expiry of such period can demand default bail.

(b) Rohan can be under remand of police custody till 25-10-2019 as period of remand is
inclusive of date of arrest and after expiry of such period can be sent to judicial custody.

(c) Rohan can be under remand for police custody till 27-10-2019 as period from remand is
computed from the date when magistrate approves such period of remand under police
custody.
(d) Rohan can be under remand for police custody till 09-01-2020 as remand under police
custody can be extended maximum up to period of days from day of arrest.

. A murder of a renounced journalist, Sudhir Choudhary, was reported by Delhi Police on


13th July 2020. Simran, one of the suspects, was taken into custody on 15th July. Delhi
police has no evidence against her which infer her connection with the case. The period for
remand ended and the defence lawyer applied for the bail before the magistrate on 31st
July. The prosecuting lawyer contended for further extension for period of remand as
investigation officers get new evidence a day before the application of bail which proves the
allegations against the accused. As a Judge, you would-

(a) Extend the period of remand for days till 12th September 2020 and sent the accused into
judicial custody for further investigation.

(b) Extend the period of remand for days till 12th October 2020 and sent the accused into
judicial custody for further investigation.

(c) Extend the period of remand for days till 29th September 2020 and sent the accused into
police custody for further investigation.

(d) Extend the period of remand for days till 29th October 2020 and sent the accused into
police custody for further investigation.

. Arman was a reputed teacher in his locality. He was suspected of committing heinous
crime and was accused of sexual assault of minor girl in name of tuition classes. Police
arrested him under alleged charges of POCSO and was taken into custody on 10-03-2020.
He was tortured by the investigating official to accept his crime and ask him to confess.
Before the magistrate, Arman dictated the incident and was sent to judicial custody on 15-
03-2020. On 17-03-2020, prosecuting lawyer applied for the police custody of Arman for
further investigation in the case and undertakes that no such torture would be done again.
Decide –

(a) The court will send Arman back to police custody till 24-03-2020 to complete the further
investigation by the officers and cannot punish the innocent until proven guilty.

(b) The court will not send Arman back to police custody just on undertaking as he was
tortured and forced to confess by the officials.

(c) The court will send Arman back to police custody till 27-03-2020 as remand period for
police custody is for 15 days and will be calculated in total excluding the period of judicial
custody.

(d) The court will not send Arman back to police custody for the remaining time period of 10
days as police custody can be given once for maximum period of 15 days and not beyond
that.

. Preetam was interrogated about the recent crime incident happened in the outskirts of city
of Mumbai. The investigation officer arrested Mr. Khan on such statements of Preetam. He
was taken into custody on April 7th and kept in remand under police for period of 10 days
and was then sent to judicial custody due to lack of evidence against Mr. Khan. The
magistrate on request of prosecuting officer denied the default bail to the accused and
extended the remand for period of days due to gravity of offence alleged. No charge sheet
was filed by the Mumbai Police and court extended the remand for period of days. The
defence lawyer filed the bail application on July 6th. Decide-

(a) The bail application would not be allowed as period of remand was ended before 06th
July as it was further extended by the court for period of days afresh which ends on 20th
June.

(b) The bail application would not be allowed as period of remand was not ended on 06th
July as it was further extended by court for period of days afresh which ends on 20th July.

(c) The bail application would be allowed as period of remand ended on 06th July as days of
remand was in total inclusive of police remand and judicial remand.

(d) The bail application would not be allowed as charge sheet is being filed by the
investigating officer and no bail can be demanded after filing of charge sheet.

. A bomb blast took place in Mumbai on 10th January 2020. Sanjay was one of the suspects
behind the blast and was alleged of being terrorist. He was taken into custody on 15th
January. Magistrate court extended his remand under judicial custody after expiry of 15
days. In the mean while, CBI alleged Sanjay of committing money laundering and wanted to
interrogate him in this regard. They applied to court to sent Sanjay in police custody for
investigation purpose. Court rejected the application of CBI and directed them to proceed
with the interrogation in judicial custody only. The Court is-

(a) Correct in rejecting the application as Sanjay can be interrogated in the judicial custody
itself and no need for further new period of remand under police custody.

(b) Incorrect as offence of money laundering is different from previous alleged offence for
which Sanjay was sent to judicial custody and hence new period of remand under police
custody can be granted.

(c) Correct as period of 15 days of police custody for Sanjay has been ended and once sent
to judicial custody he cannot be sent back to police custody.

(d) None of the above.

(Passage Q.-Q.): Disha Ravi, a 22-year-old Bengaluru-based environmental activist, has


been accused of sedition, promoting religious enmity and engaging in criminal conspiracy,
among other offences. After the Delhi Police detained her in Bengaluru on Saturday, and
formally arrested her the next day, a Delhi magistrate has remanded her to a 5-day police
custody. The principal basis for the allegations against Ms Ravi is that she was involved in
framing and editing a “toolkit”, which outlined measures to protest against farm laws; that
she provided activist Greta Thunberg the “toolkit”; that the violence on January 26 copied the
“action plan” in the toolkit; and this also had the involvement of Khalistani groups. Ms Ravi’s
arrest throws up three issues. The first is the role of the State. The government has decided
to politically invest in a narrative of a global foreign conspiracy against India, of which the
“toolkit” is evidence. But the evidence to suggest such a conspiracy is limited. Despite its
objectionable characterisation of the Indian State in parts, did the “toolkit” — a common
technique in both political and civil society campaigns now. The second is the role of the
judiciary. The metropolitan magistrate’s decision to remand Ms Ravi to custody, without her
private counsel being present, without questions about the process of her arrest, and without
adequate scrutiny of the charges and evidence, does not reflect well. Courts must treat
cases involving personal liberty with more rigour, rather than casually send people to jail
when bail should be the norm. And finally, Dissent is indeed a right, and taking up positions
— even if they are logically inconsistent, is also a right. But often, activists end up getting
used in larger political games. [Source- ‘Disha Ravi’s arrest is wrong’, The Hindustan
Times, , as accessed on 18th Feb. 2021]

. Rishu, a student of JNU student, was engaged with international organisation and work for
them to share personal internal information regarding secret agencies and other government
actions. He conspires with them and had planned a bomb blast in the city of Delhi in the
Parliament of India. Suddenly, protests were started in India, few months back and Rishu
find an opportunity to execute their plan. He shared the relevant information with groups of
people and assigned them their roles to be performed on the day of bomb blast. Before the
plan can get executed, the information get leaked to police personals and Rishu was
arrested before execution of the plan. He was presented before the magistrate and he
claimed for bail as there is no evidence against him. Police official stated about the call
recordings and witnesses to testify against Rishu and bail was denied. Decide-

(a) The court is incorrect in denying the bail to Rishu as there are no strong evidences
against Rishu to prosecute him for planning a bomb blast in the Parliament.

(b) The court is correct in denying the bail to Rishu as there are strong and sufficient
evidences against him which proves his involvement in the case of bomb blast.

(c) The court is incorrect in denying bail to Rishu as though there are weak evidences, court
favoured the government as plan was to be against the government and thus no fulfil its role
to serve justice.

(d) The court is correct in denying bail to Rishu as they have lead accused in the case and
can give more time to police officials to investigate and collect more evidences against
Rishu.

. Ms. Vijaya Kala, a feminist, started a group called “The Leaders of Future” and works to
promote and upgrade the position of women in the society and awareness about their role in
bringing about the change. She was giving a public speech on International Women’s Day
where she stated about few Indian Women leaders who helped to achieve freedom and few
who were in the constitution drafters which gave India its sole, The Indian Constitution. She
continued with such examples and turned to the government where no such representation
is sought from women in the Parliament and called the present government the ill minded
patriarchs. She sharply pointed out how Indian Parliament discriminates against the women
representation and asked the people to revolt against them. She demanded for 50 %
reservation for women in each public department and asked everyone to move the idea
forward with large violent protests. Decide-

(a) Police can arrest Ms. Kala for charges of sedition as she had incited and instigated the
people to revolt against the government violently and thus is liable.
(b) Police cannot arrest Ms. Kala for sedition as her speech was not inciting but was only a
form of dissent towards the working of the present government.

(c) Police can arrest Ms. Kala for conspiring against the government and cannot claim her
speech to be exercising right to freedom of speech and expression under article 19.

(d) Police cannot arrest Ms. Kala for conspiring against the government and she has a right
to protest against her rights and instigate others for the same.

. Ms. Swara was a renounced journalist of Bangalore and she was totally anti-government.
She mainly works for downgrading the government for every action and criticising them to
the best of her skills. In all, she was a good and competent journalist of whom government
always has eyes upon. After the enactment of new Animal Laws which puts a ban of beef
eating and selling, she published an article in a national newspaper asking the government
to lift up beef ban and claiming it to be their political agenda. Violent protests were started in
state of Karnataka and Ms. Swara was found dead the next day in her hotel room. Many
alleged it to be the act of government and called them murderers. Soon the situation was
taken under control and no such investigation was done in regards to murder of Swara.
Decide-

(a) The state of Karnataka has to investigate into the murder of Ms. Swara as she was
renounced journalist and there can be found some political linkage behind her murder.

(b) The state of Karnataka need not investigate into the murder of Ms. Swara as she was
liable for her own acts and state has no role to play with regards to such investigation.

(c) The state of Karnataka has to investigate into the murder of Ms. Swara as it is the duty of
the state to look after cases where there is violation to human body and rights attached to it.

(d) The state of Karnataka need not investigate into the murder of Ms. Swara as it is implied
that it was the act of government behind her murder as she was anti-government.

. Aryan and Shayant were models and good friends too. But there arise some rift between
them due to some misunderstandings. They were not in talking terms for past two months.
Aryan was drunk one night and was partying with her friend model Priti. Priti find it a good
opportunity to kill Aryan took all his property with her. She seduced her and made him
engage with sexual intercourse and stabbed him with a sharp knife in his back. She left the
crime scene and informed the police of the murder. She also asked Shayant to clear his
misunderstandings with Aryan and continue their friendship. Shayant reached the place and
saw Aryan dead. He took the knife into his hand to observe the actual scenario. Police
reached the place and arrested Shayant for murder of Aryan. He was not informed about the
grounds of arrest and was taken into custody. He was put for remand for days and police
didn’t find any evidence against him. The court can-

(a) Held Police liable for illegal detention of Shayant and keeping him under custody for long
without having any strong evidence against him.

(b) Held Police not liable for illegal detention of Shayant as they were of the clear view of him
being the suspect and had right to keep any such suspect in remand for infinite period.
(c) Held Police liable for illegal detention of Shayant as they should have investigated first
into the case for a month and then go for arresting the accused or suspect as the case may
be.

(d) Held Police not liable for illegal detention of Shayant as they had sufficient evidences
against Shaynat being the murderer of Aryan and he can be convicted.

. There stands a massive difference between the definitions of ‘arrest’ and that of ‘detention’;
therefore, it is necessary for one to know how to differentiate between the two. ‘Arrest’ under
the Criminal Procedure Code has a different procedure altogether while detention is not as
grave as arrest; detention is of a shorter time period than arrest, therefore, needs less
burden of proof. A Police officer can detain an individual, if he has reasonable doubt or
suspicion that a crime has been or will be committed, or if he reasonably believes that an
individual may have information regarding the same, the Police officer may then have the
liberty of detaining the individual for a short span of time, in order to investigate into the
matter. What is the rationale behind the idea of not having illegal detention of the accused
person?

(a) The idea behind illegal detention is state protects the right of victim as well as of the
accused even if he is arrested and works on principle of innocent until proven guilty.

(b) The idea behind illegal detention is to provide liberty and freedom to the individual even
though he is accused and provide him right to life with dignity.

(c) The idea behind illegal detention is enshrined under Indian Constitution which doesn’t
allow executives or their servants to be infringing the fundamental rights of the individuals.

(d) The idea behind illegal detention is to adopt the procedure of fairness, justness and
reasonableness into the justice delivery system and support justice to be the main motive.

(Passage Q.-Q.): The question of choice has been extensively dealt with in Justice KS
Puttaswamy v. Union of India. The Court unanimously held that “the right to privacy is
protected as an intrinsic part of the right to life and personal liberty under Article 21” and also
held that the fundamental right to privacy would cover at least the following three aspects -
(i) privacy that involves the person, (ii) informational privacy and (iii) privacy of choice, which
protects an individual’s autonomy over fundamental personal choices. Shortly after the
Puttaswamy case, two judgments were delivered in quick succession by the Supreme Court
in which it held that the right of a person’s choice to marry whom she pleases in an inherent
part of individual dignity and intrinsic to Article 21. While the aforesaid judgments have in no
uncertain terms upheld that the right to marry is a fundamental right, none of them pertained
to same-sex marriages. This question can be answered in Navjet Singh Johar v. Union of
India. Though the Court only concerned itself with the constitutional validity of Section 3, the
majority opinion and the concurring opinion by Dr DY Chandrachud J did rely on cases for
the question of freedom of choosing a life partner of one’s own choice and recognized the
“sexual autonomy of an individual”. In the concluding portion of his concurring opinion, in
which he has specifically held that members of the LGBT community “are entitled, like all
other citizens, to the full range of constitutional rights including the liberties protected by the
Constitution” and are entitled to equal citizenship and “equal protection of law.” Therefore, it
is beyond doubt that the fundamental right to marry a person of one’s own choice has to be
conferred on same-sex couples intending to marry. And if such a right is denied, they can
certainly knock the doors of the Constitutional Courts to enforce these basic and inherent
rights. [Extracted, with edits and revisions, from “The fundamental right to marry in India and
its application to same-sex marriage”, Dormaan J Dalal, Bar & Bench, 3rd January 2021;
https://www.barandbench.com/columns/the-fundamental-right-to-marry-in-india-and-its-
application-to[1]same-sex-marriages]

. Tahir is a successful owner of a brand-new business that focuses on providing necessary


kitchen appliances and ingredients to restaurants through an application. Their business
idea has received great response, and is popular in the kitchens of many high-end hotels.
While attending a business meeting with an investor, he meets Riya, who works as the
investor’s personal secretary. They plan to meet outside of work, and eventually fall in love
with each other. Both of them belonged to different religions, with Tahir being a Buddhist and
Riya being a Christian, and wanted to marry each other despite the potential backlash they
might face by their communities and families. After 2 years of dating, Riya and Tahir decide
to marry each other, which causes an uproar in their communities, due to Tahir’s popularity
as an upcoming entrepreneur. Riya and Tahir start to get ominous messages by anonymous
people who threaten to kill them if the marriage takes place. Meanwhile, people start
boycotting Tahir’s application, as most of the restaurant owners of the city belong to the
Buddhist community. Riya and Tahir decide to file a writ petition in the Supreme Court
alleging violation of their fundamental rights, and seeking protection. What should be the
verdict?

(a) Yes, Riya and Tahir’s right to equality has been violated as they are being discriminated
against, and not being treated at par with other couples who want to marry each other.

(b) Yes, as the circumstances presented go against the idea of secularism, which is an
integral part of the Indian Constitution.

(c) Yes, as their fundamental right to life under Article 21 has been violated.

(d) Yes, as Tahir and Riya are entitled to the full range of constitutional rights including the
liberties protected by the Constitution.

. Kelly is a nurse in an established chain of hospitals. She is a paediatric nurse, and mostly
deals with infants and children, and occasionally teenagers. One night while she was on her
night shift, a teenage patient comes in with a severe pain in his chest, and after initial
examination, is recommended surgery. Kelly assists the doctors during the surgery, and is
entrusted to take care of the boy post-surgery as well. While taking care of him, she realises
that the patient is none other than Philip Halpert, son of the famous actors Jim Halpert and
Pam Beesly. Nonetheless, she takes care of him like any other patient of the hospital. A day
after Philip is discharged from the hospital, Ryan comes in the hospital for a common cold.
While Kelly was noting his symptoms down for the doctor, he kept asking her a lot of
questions about Philip Halpert and his health, and telling her that he was a ‘big fan’ of the
young star. Convinced, without any ill[1]intention, Kelly told him about Philip’s genetic heart
condition and his surgery, and how he had to be under observation for weeks. The next day,
this news becomes the front page of a reputed entertainment weekly, which touts Kelly as
the source. Philip’s legal counsel files a case against Kelly, accusing her of infringing his
right to privacy. Is Kelly liable?
(a) No, Kelly is not liable as there was no mal-intention behind her actions and she genuinely
believed it to be an innocent query rather than a media probe.

(b) Yes, as revealing such sensitive information violates Philip’s right to privacy as given in
the case of Navjet Singh Johar v. Union of India.

(c) No, Kelly is not liable, as a public personality such as Philip Halpert cannot claim the right
to privacy especially when Kelly and the other staff were made to sign no non-disclosure
agreements.

(d) Yes, as an intrinsic part of Philip’s life and personal liberty was violated by Kelly, and
hence she is liable.

. “And if such a right is denied, they can certainly knock the doors of the Constitutional
Courts to enforce these basic and inherent rights.” What ‘ basic and inherent rights’ does this
statement refer to?

(a) Fundamental Rights.

(b) All rights and liberties that are guaranteed by the Constitution.

(c) The freedom of choosing a life partner of one’s own choice.

(d) The right to privacy of all kinds.

. Elena is a dedicated and hardworking teacher, who teaches children studying in


kindergarten and nursery classes. She loves spending time with children, and this was a
primary reason behind choosing her profession. The parents of the children rarely have any
complaints about her, and hold her in high regard. Elena identified as a lesbian, and had a
female partner who she lived happily with it. However, nobody apart from her family and
close friends knew about this fact. After working for more than 3 years at her current place of
employment, she finally decided to reveal this part of her life to her school’s administration
and some parents of the children she taught, at a yearly event hosted by the school.
Contrary to her expectations, the attendees of the even didn’t react as positively as she
expected. The next few days after the event, the school administration started to receive
many complaints addressed to Elena, with multiple parents requesting the school
administration to either fire Elena or relocate their children to another class. In her work
place as well, her colleagues had started to behave strangely with her, and excluded her
from workplace activities and gatherings. Such behaviour started to make Elena feel
ostracised and secluded. Soon, the school administration informed Elena of their decision to
fire her, attributing it to the discomfort of parents being taught by a woman who identifies as
a lesbian, as according to them that would result in their children being impacted by such
‘immoral’ lifestyle and values. Disheartened and enraged, Elena files a writ petition in the
High Court alleging discrimination and violation of her rights. What would be the verdict?

(a) The behaviour exhibited is discriminatory, and is violative of Article 14 of the Constitution,
that talks about equality of persons.
(b) The behaviour exhibited by the school administration and parents is discriminatory, and
every individual has a right to choose a partner of their choice without being discriminated for
it.

(c) The parents are liable for discriminating against Elena on the basis of her identity and
sexuality. The school administration was merely acting on the complaints of the parents, and
hence should not be held liable.

(d) The school administration is liable for firing Elena, and making her feel ostracised just
because she identified as a lesbian. The parents were expressing concerns about their
children, but were not responsible for her dismissal, and hence should not be held liable.

. Within the context of the passage given, which of the following situations provided would
not be covered under the Right to Privacy?

(a) Farah has recently got constructed a house of her own. She holds a housewarming
ceremony and invites many close friends and family. During the party, she spots an
unknown man lurking about her house. She attempts to escort him out of her property, only
to later find out that he is a friend of one of her uncle’s, and he had brought him along. Later,
the man approaches her and asks her many questions about the house, such as its cost, its
legal owner, and security systems. Farah feels uncomfortable and escorts him out of her
property.

(b) Rishabh is an archaeologist, and historian. He was recently at one of his dug-outs where
the excavations were midway, when he stumbled upon some pieces of broken pottery. With
the pieces seeming he important, he analyses them in secret. Upon studying them for
weeks, he discovers that the script engraved on the pottery greatly helps in deciphering the
script of one of the first civilizations, the Meluhha Valley. He continues to keep this discovery
a secret, to make sure nobody else steals it. Jamal, his assistant, was extremely curious
about this study, and kept creeping upon his tent and stealing his notes to figure out the
topic of his research. One day, he steals the pottery pieces.

(c) Gargi was an air-hostess in an international airline, and was frequently out of her house,
and her city, for days at a stretch. On the few days that she stayed at home; she usually
invited some friends over to have a small party. However, she was always ensured that no
inconvenience was caused to the neighbours in any manner. Her landlord lived a floor below
hers, and constantly supervised the people who came and went. He had also instructed the
guards to keep a check on the people coming in Gargi’s house, and to inform them first if
any of them were male.

(d) Zoran had been trying to eat healthy and exercise, since his doctor had recommended
him to lose weight for his fitness. As a consequence, he had been watching his weight and
his nutrition according to a diet plan. However, since he was embarrassed about his weight,
he chose not to inform about his decision. His co-worker Matthew, spots him eating salad
one day at work, and gets suspicious. To satiate his curiosity, he rifles through Zoran’s office
and finds his diet plan, and doctor’s report. Soon, the news spreads from Matthew to other
employees in the office, who ridicule him behind his back for even trying to reduce weight,
and call him names. One day, Zoran overhears a similar conversation near the water cooler,
and goes back home disheartened. He soon stops his workout and diets, and confines
himself in his house. After seeking psychological help, he is diagnosed with depression.

 
(Passage Q.-Q.): In a detailed judgment rendered on Tuesday, the Kerala High Court
expounded the modalities of detention of vehicles and seizure of goods in transit under the
GST laws. It emphasized that detention and seizure of goods and vehicles is a reasonable
restriction on the exercise of free trade and movement, but it would nevertheless have to be
exercised strictly only in order to prevent possible tax evasion. The Court drew from GST
legislation to state that there are two essentials that should be present when goods are
being transported - an invoice presented by the supplier of goods specifying details about
the goods as well as the tax paid thereupon and an e-way bill .A tax official on suspicion of a
transaction involving possible evasion in tax, is empowered to seize the goods or detain the
vehicle upon an order to that effect, following which he can issue notice upon the consignee.
Only after a proper application of mind, based on the material available can an order of
detention be issued, the Court said. At the same time, if the situations envisaged in the
legislation for detention are met, the goods should be seized or detained without any laxity,
the Court said. The Court said, the consignee can choose to make the stipulated payments
as per the notice, after which the notice would be concretised in the form of another order
and the proceedings closed. If consignee chooses to challenge the detention, he can furnish
a security in the form of a bank guarantee covering the prescribed sum and get his
goods/vehicle released provisionally and then make his representation. If aggrieved, the
consignee can approach the relevant authority under the legislation in appeal, but cannot
approach the High Court at this stage, the Court said. Extracted with edits and revisions from
an article published by Bar and Bench dated 15th January 2021
(https://www.barandbench.com/news/litigation/kerala-high-court-detailed-judgment-
detention-of-goods[1]gst-law )

. Ryan is a company that manufactures slate pencils. The goods are manufactured in Dum-
Dum, West Bengal and transported to Thiruvananthapuram, Kerala. The goods are
transported by Motorbike. X is the person who rides the bike and transports the goods. Ryan
provided X with an invoice of the transported goods and an e way bill concerning the tax
paid. X didn’t had a haircut for 2 months, the tax official on seeing X suspected that she is a
criminal and seizes the vehicle. According to the given facts, which of the following is true?

(a) The tax official performed his duty by preventing a criminal from transporting the goods.

(b) The act done by tax official is in good faith and he is correct

(c) The tax official is not empowered by any act to seize in the current case

(d) The tax official is empowered to seize the vehicle at his discretion

. The GST act states that transporting laptops with any other electronic gadgets is prohibited,
even if there is invoice and an e-way bill. Zyan is a manufacturer of electronic devices. He
manufactures laptops, tablets, desktops, mobile phones, chargers and Bluetooth devices.
Zyan took 20 laptops and 5 desktops and began to transport them to sell them in a local
mandi. 8, a tax official detains the vehicle immediately. Which of the following is a valid
defense to Zyan?

(a) Zyan can sue 8 for misusing the powers conferred on him

(b) Zyan can state that he has both invoice and e-way bill

(c) Zyan can state that he is not aware of the law

(d) None of the above


 

. Uma was informed by the tax officials that she violated the GST act and the officials seized
her Vehicle and imposed a fine of 50 rupees. But Uma knows that she did not violate the law
and the officials had intentionally seized her vehicle. Frustrated with the act of tax officials,
she approaches an advocate and asks him to give a solution. If you are Uma’s advocate
what advice would you give to her?

(a) I will ask Uma to file a harassment complaint in the nearest police station

(b) I will ask Uma to produce the said amount in the form of a bank guarantee and get her
vehicle released provisionally

(c) I will ask Uma to pay 50 rupees to the official and continue transporting her goods.

(d) I will ask Uma to pay a sum of 50 rupees as bribe to the official to avoid unnecessary
legal process.

. Let’s presume that Uma is not satisfied with the conduct of officials in the process of getting
her vehicle back. Uma feels frustrated and wants to challenge the conduct of the officials in
the High court. Uma comes to you again for advice, what would you suggest her?

(a) I will suggest her to go ahead and file a complaint in the high court

(b) I will suggest her to file a complaint in the appellate office under the GST act

(c) I will suggest her to file a petition in the GST Tribunal

(d) I will suggest her not to file a complaint in the high court

. Most of the businessmen feels that the provisions of seizure/detention of the GST act is
imposing unnecessary restrictions on free trade and movement. Are the arguments of the
said businessmen in consonance with the principles laid down in the passage?

(a) Yes, the arguments of the businessmen are true

(b) No, the arguments of the businessmen are not true

(c) The arguments of the businessmen should not be considered

(d) The arguments of the businessmen should be taken seriously

. Statement 1: A tax official can seize transporting adulterated goods if there is possible tax
evasion Statement 2: A tax official can seize transporting adulterated goods if there is no
invoice Which of the following are incorrect?

(a) Only a

(b) Only b

(c) Both a and b


(d) Neither a nor b

22
Passage (Q.68-Q.): Admissibility of Whatsapp chats as evidence is under question post
conversations between Goswami and Das Gupta found their way into social media. The
nature of evidence in the case of online conversations will be secondary in nature. The
evidence produced in the court with respect to online chats will be printouts of the backup
documents or of the screen-shots of the chats, unless the device itself is produced. The
normal rule of evidence is that a document must be proved by primary evidence by proving
the document itself. Oral evidence about the contents of the documents is barred by the
Evidence Act by Section of the Indian Evidence Act. Section of the same says that
documents must be proved by primary evidence except in the circumstances mentioned in
Section . To admit the electronic record as evidence, it must be accompanied with a
certificate from a person who produced the copy certifying that the same fulfills the four
conditions of section B. The Supreme Court in Arjun Pandit Rao v. Kailash Kushanrao
settled conflicting decisions to authoritatively rule that a Certificate under Section B is
mandatory to the admissibility of evidence by way of electronic record. It further held that a
Certificate is not necessary if the 'original document' itself is produced as a primary evidence.
This can be done by the owner of the electronic device, and proving that the concerned
device, on which the original information is first stored, is owned and/or operated by him.
Section B differentiates between the original information contained in the “computer” itself
and copies made therefrom, the former being primary evidence, and the latter being
secondary evidence. In Ambalal Sarabhai Enterprise Ltd v. KS Infraspace LLP Ltd the
Supreme Court stated that the WhatsApp messages are matters of evidence with regard to
their meaning and its contents to be proved during trial. The e - mails and messages will have
to be read cumulatively to decipher whether there was a concluded contract or not. Further,
the Delhi High Court in National Lawyers Campaign for Judicial Transparency and Reforms
v. Union of India has held that a Whatsapp forward message, without an unknown source,
cannot be treated as evidence. The Court held that such a forwarded message, without its
original source, cannot be regarded as 'document' under the Evidence Act. Pasted and edited,
‘Are WhatsApp Chats Admissible in Evidence?’

68. Saurav and Neelima were a married couple who were living apart for quite some time.
After three months from their separation, Neelima committed suicide. Neelima’s family
accused Saurav of threatening Neelima with her life if her family did not pay 5 Lacs to
Saurav. During the trial, the family produced the whatsapp chats in the phone of Neelima,
which were evidence of the threatening messages from Saurav. Saurav’s lawyer has contested
the admissibility of the chats as they were not coupled with the certificate under section B.
Decide whether the chats could be admitted by the court or not.

(a) The chats can be admitted as the court can excuse the requirement of a certificate if a
person testifies that the chats are original and have been first stored in Neelima’s phone only.

(b) The chats cannot be admitted as electronic evidence cannot be considered by a court
unless it is coupled with the certificate proving its authenticity and undertaking that produced
evidence is first and originally stored in the same manner as produced before the court.
(c) The chats can be admitted as the chats in the phone are primary evidence and not
secondary electronic evidence, which requires a certificate to prove the originality.

(d) The chats cannot be admitted as electronic evidence are secondary evidence and shall be a
last resort in a criminal trial with severe penalty.

. Police have been trying to nab Jaggi, a notorious criminal, for long but could not find any
evidence against her. In the process, they got hold of Teerath who is a servant at Jaggi’s
house and Teerath agreed to testify before the court about Jaggi’s crime. Police on the basis
of Teerath’s statements filed a chargesheet holding Jaggi accused of extortion, abduction and
money laundering. Before the court, Teerath testified that he has seen the ledger wherein
Jaggi keeps account of all her crimes. Court, relying on the testimony, convicted Jaggi of all
the crimes she was charged with. Jaggi has filed an appeal against the ruling. Decide which of
the following could be the best possible ground for the appeal.

(a) Court could not have relied on the testimony as no person can give oral testimony
regarding the contents of a document either electronic or not, and thus the court could not
convict her on the basis of the same.

(b) Court could not have relied on the testimony as the original document has not been
produced before it and the court in no circumstance can rely on secondary evidence to
convict her.

(c) Court could not have relied on the testimony as the same is a secondary evidence which
could not be proven without a certificate under section B proving its originality.

(d) Court could not have relied on the testimony as the record of crimes committed is not
conclusive evidence unless it ties the accused directly with the crimes allegedly committed.

. Prajjawal was an activist who was fighting against the recent farm laws enacted by the
government. All the activists have been allowed to demonstrate in front of the Parliament but
they are prohibited from entering the campus. In fact, the government promulgated an
ordinance, criminalising entry of any activist in the Parliament premises until any further
orders. Prajjawal got a whatsapp forward that the government has now allowed
demonstration in the premises while the Parliament is in session. He was arrested and is now
being tried for the offence. In his defence, he has produced the whatsapp forward accusing
the government of propagating false information as their agenda to suppress the protest.
Choose the correct option?

(a) The message is admissible in the court as it has been produced in the original form in the
device itself and Prajjawal as the owner of the device is also vouching for its authenticity.

(b) The message is admissible in the court as electronic evidence are admissible as documents
in a court under the Indian Evidence Act.

(c) The message is inadmissible in the court as same is an electronic evidence and has to be
coupled with a certificate proving its originality and authenticity before being produced
before a court.

(d) The message is inadmissible in the court as the same is a forward from another source and
therefore cannot be treated as evidence by the court.
 

. Archana was an intelligence operative who was following an international criminal, Rouf
Lala. Archana’s mission was to gather evidence against Lala, so that he can be tried in India.
She managed to get hold of Lala’s computer, and copied contents pertaining to his criminal
empire in a pen drive. During the trial, the pen drive was submitted as primary evidence and
Archana stated before the court that the contents of the pen drive were original and were
herself stored by her. The Lala’s lawyers have challenged the admissibility of the evidence
due to the absence of the certificate under section B. Choose the correct option?

(a) Contents of the pen drive are admissible in the court as the same are a primary evidence
and the owner has proven that the information is original.

(b) Contents of the pen drive are inadmissible in the court as the pen drive is not the device in
which the information was first stored, thus, it was not a primary evidence. Therefore, it shall
be submitted with the certificate under section B.

(c) Contents of the pen drive are admissible in the court as Archana has proven its originality
before the court which excuses the requirement of the certificate.

(d) Contents of the pen drive are inadmissible in the court as Lala, being the original owner of
the device where the information was first stored, shall prove the originality.

. Thomas, an engineering student filed a complaint in the police station that he is ragged and
abused by his college seniors. As per the Indian Penal Code, inflicting physical or mental
agony by ragging is punishable by upto 6 years of punishment. Police are reluctant to file the
chargesheet because many such cases in the past have been found to be false. Thomas gave
police screenshots of the seniors which proves that Thomas was ragged by them. During the
trial, this evidence was produced before the court along with the certificate under section B of
the Evidence Act. Decide whether the screenshots can be admitted as evidence by the court or
not.

(a) Screenshots cannot be admitted as the same is a secondary evidence and the court cannot
convict a person relying entirely on a secondary evidence.

(b) Screenshots can be admitted despite it being the secondary evidence if coupled with the
certificate as per guidelines given in section B.

(c) Screenshots cannot be admitted as the screenshots are not original documents and
therefore cannot be relied upon to convict a person.

(d) Screenshots can be admitted as the screenshots would become primary evidence when
coupled with the certificate as per the guidelines given in section B.

 
Passage (Q.-Q.): Section 313(1)(b) castes an imperative duty upon the court to give the
accused an opportunity to explain any incriminating circumstances or inculpatory evidence
put by the prosecution against him. Further, it can be used to test the veracity of the
prosecution's case. However, by virtue of 313 (3), the accused can refuse to or provide false
answers during the examination. the accused an absolute 'right to silence'. This essential
safeguard in criminal procedure emanates from Article 20(3) of the Constitution as a right
against self-incrimination or 'testimonial compulsion'. The accused has an absolute right to
silence which emanates from Article 20(3) of the Constitution as a right against
self[1]incrimination or testimonial compulsion. In Phula Singh v. State of H.P., the court held
that the accused has a duty to furnish an explanation regarding any incriminating material that
has been produced against him. The accused has freedom to remain silent, however, the court
would be entitled to draw adverse inference against the accused as may be permissible in
accordance with law. The court in State of West Bengal v. Mir Mohammad, ruled that when
any fact is especially within the knowledge of any person, the burden of proving that fact is
upon him. This becomes an important standard for drawing adverse inference where the
accused especially withholds the knowledge that is impossible for the prosecution to prove
and only the accused knows the truth of the matter. The court in State of Maharashtra v.
Ashok Chotelal Shukla held that even though the respondent failed to explain incriminating
circumstances against him, the other circumstances do not coherently lead to the conclusion
that he had caused the death of his wife. By this inference, the Constitutional provision of
Article 20(3), is not violated as the right to remain silent envisages immunity to the accused
from being compelled to give testimony against himself. Secondly, it is mostly in
circumstantial evidence where furnishing such an explanation becomes imperative. Finally,
the accused is not compelled, and can refuse to answer. If adverse inference was equal to
deducing his guilt, the accused would've been forced to answer but silence only becomes a
factor in the larger mitigating circumstances. Pasted and edited, Sanskriti Gupta, 'Can
Adverse Inference Be Drawn From Silence Of Accused ?'

. Shree worked as a journalist who got invited to interview a politician at his farmhouse.
When she reached the farmhouse there was no one but the politician’s son, Manu, who tried
to commit rape on her. In her protection, she threw a vase at him, as result of this injury,
Manu dies then and there. She informs the police and reports that when she reached the
farmhouse he was already lying there dead. During the trial, the prosecution presented many
evidence incriminating Shree including her finger prints on the vase. The courts then imposed
the burden on Shree to explain the circumstances and on her silence convicted her of the
murder. Shree’s lawyers have filed an appeal against the order of conviction. Choose the
correct option.

(a) She cannot be convicted as she attacked Manu in self defence when he was trying to
commit rape on her, which is an absolute defence to the offence of murder.

(b) She can be convicted as the prosecution produced incriminating evidence against her and
unless she offers a reasonable explanation to the same, the court can pass an order of
conviction based on the evidence before it.

(c) She cannot be convicted as it is the burden of the prosecution to prove the guilt beyond
reasonable doubt and not the defendant to prove their innocence.

(d) She can be convicted as the court can draw adverse inferences from the silence on the
basis of which guilt of the accused can be proven.
. Taking advantage of the university protest, two students Guddu and Bablu broke into the
examination department to steal the question papers of mid-term examination. When this
news broke out university administration filed a police complaint against both of them. In the
trial, the court asked both the accused to offer explanations regarding copies of question
paper and other corroborating evidence found by the Police from them. Further, they were
strictly warned about the consequences of their silence and the evidence furnished against
them. On their conviction, Guddu and Bablu filed a petition before the Supreme Court for
protection of their right against self-incrimination under article 20(3). Which of the following
is the correct option?

(a) Petition shall not be allowed as the right against self-incrimination gives the accused the
right to not give evidence against themselves but does not prevent the court to rule their
silence as their guilt.

(b) Petition shall be allowed as if a person is bound to speak to prevent their conviction then
the right to remain silent that emanates from article 20 (3) would just be a dead letter with no
application.

(c) Petition shall not be allowed as the accused are just being asked to furnish explanation for
the incriminating evidence against them, in defiance of which the court can convict them
relying on the evidence produced.

(d) Petition shall be allowed as their conviction was a consequence of their silence which is
violation of their right against self-incrimination.

. Hari has been made the prime suspect in a murder case, because he was found near the
house of his dead batchmate whom he threatened to kill. The Police could not gather enough
evidence to incriminate Hari but during the trial the judge asked Hari to explain the
circumstances he refused to do so. Hari was convicted due to his silence, and made an appeal
to the high court. Choose the correct option?

(a) Hari has been rightly convicted as the court can draw adverse inference for his silence to
convict him of the offence of murder.

(b) Hari has been wrongly convicted as being found in the vicinity of the murder is not
enough evidence to hold him guilty of murder.

(c) Hari has been rightly convicted as it is burden of Hari to prove his innocence and when he
did not, he was found guilty of murder.

(d) Hari has been wrongly convicted as the court has found him guilty solely on the basis of
his silence.

. In which of the following circumstances the right against self-incrimination of a person is


not violated: I. An accused being forced by the Police and prosecution to confess to guilt in
exchange of lesser sentence. II. An accused being asked by the judge to explain their
presence at the crime of murder and their prints on the murder weapon. III. A witness being
forced to share only their true knowledge regarding an event else will be charged for perjury.
IV. An accused confessing to their crimes in exchange of life imprisonment instead of death
sentence. V. An accused being threatened to speak and prove their innocence else they will
be convicted on the grounds of their silence alone. Which of the following is the correct
answer?

(a) II, III and IV

(b) II, III, IV and V

(c) I, II and III

(d) II only

. Simran has been working as an employee at a jewellery shop for over 10 years now. She is a
trusted employee, who opens and closes the shop every day. One morning, when the owner
reached the shop Simran had not reached the shop. He opened the shop himself and found
multiple valuable pieces of diamonds and other jewelleries missing. The computer system
and CCTV cameras show that Simran was the last one to leave the shop and since then no
one has entered it. During the investigation, Simran was found in possession of a whopping
sum of money that was unlikely to be found with someone doing an averagely paying job.
Upon her silence, the court held her guilty of stealing the jewelleries and sentenced her with
imprisonment of 5 years. Choose the correct option.

(a) The accused has been wrongly convicted, as guilt cannot be deduced solely on the basis of
silence. That would amount to compulsion to speak which is in violation of right against self-
incrimination.

(b) The accused has been rightly convicted as the court ordered the conviction on the grounds
of circumstantial evidence and drawing adverse inference from the silence and failure to
explain the evidence against her.

(c) The accused has been wrongly convicted as circumstantial evidence is secondary evidence
and cannot be solely relied on to order conviction.

(d) The accused has been rightly convicted as all the produced evidence pointed in the
direction of her guilt and her being silent or not is irrelevant to the order of conviction made
by the court.

Passage (Q.-Q.): The Bench of Justices TV Nalawade and MG Sewlikar passed the order in a
plea filed by one Rajendra Ambhore seeking reliefs to prevent advertisements on TV
channels from promoting sale of articles like the Hanuman Chalisa Yantra. The Court
examined the case through the lens of provisions of the Maharashtra Prevention and
Eradication of Human Sacrifices and other Inhuman, Evil and Aghori Practices and Black
Magic Act, 2013 (‘Act’).On perusal of the Black Magic Act, the Court held, "The definition
of propagation, quoted already, shows that advertisement of present nature is covered by the
definition. Section 3 of the Black Magic Act prohibits not only commission of act of black
magic, evil practices etc., but also propagation, promotion of such practices and magic.
Section 3(2) of this Act shows that abetment of such propaganda is also an offence. Thus, TV
channels, which telecast such advertisement also become liable under Section 3 of the Black
Magic Act." The plea came to be filed in 2015 when Ambhore came across the advertisement
of Hanuman Chalisa Yantra on TV channels. The Court observed that despite having
reformists like Mahatma Phule and Dr. Babasaheb Ambedkar, who worked to remove evil
practices and spread awareness against superstition, a progressive and reformist state like
Maharashtra was required to enact the 2013 Act. It came to the conclusion that the
advertisement of the Yantra was aimed to exploit persons who feel that "they are in trouble
and they need some kind of help which may be of supernatural nature". Source- [Extracted,
with edits and revisions, from: “Advertisement/ sale of articles claiming they have
miraculous or supernatural properties is illegal: Bombay High Court”, Bar and Bench, dated
06.01.2021] https://www.barandbench.com/news/litigation/advertisement-sale-articles-
miraculous-supernatural-illegal[1]bombay-high-court

. Richa’s elder brother Samar is suffering from cancer. They consulted the best doctors and
tried various treatments but to no avail. A family friend suggest to consult Mr. X who had a
track record of miraculous recovery of all his patients suffering from cancer. Mr. X suggest
Samar that his condition is caused due to some evil forces and he will have to perform a
ceremony to get rid of the same. He asked Samar to pay INR 6 Lakhs to buy 4 pigeons from a
special place in the village. These pigeons will form part of the ceremony. Samar paid the
amount and within a week after the ceremony was conducted he started feeling better and had
a miraculous recovery. Will Mr. X be liable under the Act?

(a) He will not be liable since he merely suggested to perform the ceremony and not promised
to cure Samar.

(b) He will be liable if he had promised miraculous recovery to Samar.

(c) He will not be liable since Samar recovered from cancer after his ceremony.

(d) He will be liable since he performed the ceremony claiming to cure Samar.

. Mr. Y suffered huge losses in his business. He was approached by Ms. Z who suggested him
to visit a Guru in her village. Ms. Z promised Mr. Y that all his problems will be resolved by
the Guru and she herself vouched for him. Following Ms. Z’s advice he visited the Guru who
told him that he suffered losses due to his wife. According to the Guru, he should beat his
wife with a stick so that evil forces leave her body. Mr.Y followed the advice of the guru and
started beating his wife with the stick, due to this she suffered grave injuries and was
admitted to the hospital. Have they committed an offence under the Act?

(a) The Guru will be liable since he promoted black magic and evil practice.

(b) Mr Y’s will not be liable since he acted on the advice of the Guru.

(c) Mr. Y, Ms. Z and the Guru will be liable since they followed black magic.

(d) They will not be liable since there was no serious harm done to Mr. Y’s wife.

 
. Rick is the brand ambassador of a ‘Lucky Charm’ device. He recently shot an advertisement
where he was told to promote the device’s magical qualities. The advertisement was
broadcasted by a local news channel, seeing the advertisement several people ordered the
device believing in its magical qualities. Can Rick be made liable under the act?

(a) He can be made liable since he made false promises regarding the device.

(b) Only the advertising company and the company making the device will be made liable
since they promote black magic.

(c) He cannot be made liable since he is an actor and he was only doing his job.

(d) He can be made liable since he claimed the device has magical qualities.

. Mrs. Z visited a famous Baba in her village. He advised her to perform a yagna which will
guarantee that she will have a baby boy. As part of the Yagna she had to sacrifice two lambs
of a particular breed which can be found in a small south Indian village. Knowing about the
yagna and the need for two lambs, Sushma who was a friend of Mrs. Z used her contacts to
procure the lambs and gave them to Mrs. X. The baba then performed the yagna guaranteeing
the birth of a boy. Can they be made liable under the act?

(a) The Baba will be made liable under the act since he promoted black magic.

(b) They will not be made liable since performing a yagna is not black magic.

(c) Mrs. Z and the baba cannot be made liable since they only performed the yagna and no
harm was done to anyone.

(d) Mrs. Z, Baba and Sushma will be made liable since they performed act of black magic. 21
of 40

Passage (Q.-Q.): A consensual love affair is not a defence against the criminal charge of
kidnapping a minor girl under 18 years of age, the Supreme Court has held in a judgment.
“Section 3 of the Indian Penal Code (kidnapping from lawful guardianship) bestows the
ability to make crucial decisions regarding a minor’s physical safety upon his/her guardians.
Therefore, a minor girl’s infatuation with her alleged kidnapper cannot by itself be allowed as
a defence, for the same would amount to surreptitiously undermining the protective essence
of the offence of kidnapping,” Justice Surya Kant observed in the recent judgment authored
for the Bench, which included Justice S. Abdul Nazeer. Though, he was found guilty of rape
by the trial court, the Gujarat High Court, in 2009, acquitted him of the crime, but held him
guilty of kidnapping her by enticing her to flee with him. “Such ‘enticement’ need not be
direct or immediate in time and can also be through subtle actions like winning over the
affection of a minor girl. However, mere recovery of a missing minor from the custody of a
stranger would not ipso facto establish the offence of kidnapping,” the apex court upheld the
High Court verdict on January 12. Source - [Extracted, with edits and revisions, from:
“Consensual love affair not a defence against criminal charge of kidnapping minor girl: SC”,
The Hindu dated 15.01.2021] https://www.thehindu.com/news/national/consensual-love-
affair-not-a-defence-against-criminal-charge-of[1]kidnapping-minor-girl-sc/
article335012.ece
 

. Rajat and Reema were best friends. They both went to a birthday party together and while
coming back they decided to visit the local café. Reema’s parents got worried seeing that she
hasn’t returned from the birthday party and started searching for her in the neighbourhood
and enquired about her whereabouts. Shriya, a friend of Reema informed her parents that she
is at a local café with Rajat. Reema’s parents got furious at Rajat because he took Reema to
the café without their permission. They decide to report him to the police for committing the
offence of Kidnapping. Will Rajat be charged with kidnapping?

(a) He will be charged since he took Reema without her parents permission.

(b) He will not be charged since his act was not grave enough to be charged with kidnaping.

(c) He will be charged since he enticed Reema to go with him to the café.

(d) He will not be charged since he did not commit the offence of Kidnapping.

. Rohan, Lilly and Rita are in 10th standard and good friends. They decide to go on a trip to
Rishikesh. Rohan and Lilly take permission from their parents to go on the trip however Rita
did not take permission from her parents. Rita decided to run away and go on the trip with
Rohan and Lilly. Rita’s parents got worried seeing her missing and informed the police. The
police found Rita with Rohan and Lilly at a hotel in Rishikesh. They decide to charge Rohan
and Lilly along with the manager of the hotel, since he played the role of an accomplice, for
the offence of Kidnapping. Decide if they can be charged for kidnapping.

(a) Only Rohan and Lilly will be charged with kidnapping since they enticed Rita to go with
them.

(b) They will not be charged with Kidnapping since Rohan and Lilly are themselves minor.

(c) They will not be charged with Kidnapping since they did not entice Rita.

(d) They all will be charged since the Hotel manager also played a part as an accomplice.

. Vivek and Rose are very close friends and are in the same MBA college. They both fell in
love. Vivek asks Rose to go on a trip with him to Shimla. Rose refuses saying her parents
won’t allow but Vivek continues to compel her to go with him. She finally gives in and goes
on the trip without informing her parents. Rose’s parents are furious on finding out that she is
missing and decide to file a police complaint against Vivek for the offence of Kidnapping.
Will Vivek be charged for kidnapping?

(a) He will be charged for kidnapping since he took Rose without her parent’s permission.
(b) He will be charged for kidnapping if he took Rose against her will.

(c) He will not be charged since he did not commit the offence of kidnapping. (d) He will be
charged since he compelled her to go on a trip which amounts to enticing.

. Chole’s parents are in the process of getting a divorce. The court gives Chole’s custody to
her mother. Chole’s father has visitation rights. Her father visits her every Saturday. Chole
persists her father to take her for a concert happening in Goa on Sunday. On seeing her
willingness her father decides to take her to the concert on Sunday. Chole’s mother got
furious and decided to file a kidnapping case against Chole’s father. Will her mother succeed
in her claim?

(a) He will not be charged for kidnapping since he is her father.

(b) He will be charged for kidnapping since he took her to Goa for the concert.

(c) He will be charged for kidnapping since he enticed her and took her to Goa.

(d) He will not be charged for kidnapping since parents cannot be charged for kidnapping
their children.

Passage (Q.-Q.): In explaining the ambit of Article 25, the Supreme Court held that Clause 1
of the provision “does not grant the right to convert another person to one’s own religion but
to transmit or spread one’s religion by an exposition of its tenets.” The Court stated that
“Article 25(1) guarantees ‘freedom of conscience’ to every citizen,” and “if a person
purposely undertakes the conversion of another person to his religion, as distinguished from
his effort to transmit or spread the tenets of his religion, that would impinge on” such a
guarantee. The Supreme Court failed to discuss the definitions of inducement and allurement,
which was the primary bone of contention. It also did not revert to the legislative history of
Article 25—the term propagate was included in the Constitution as a compromise to assure
Christians that it would include freedom to convert. Moreover, if one takes the reductionist
understanding of propagation—given the court in this case—the inclusion of such a term in
the Indian Constitution would be rendered meaningless. The mere right to propagate for the
enlightenment of others would already be covered under the right to free speech and
expression under Article 19(1)(a) of the Indian Constitution. Thus, we submit that the right to
convert was actually included in Article 25, and, as such, the decision of the Supreme Court
in Stainislaus not only was erroneous but also led to instability in society, as Indian Christians
feel they have been cheated in this matter. The assurances given to them in the Constituent
Assembly on the inclusion of the word propagate have not been fulfilled, and the government
has done nothing to remedy the situation arising out of the highly restrictive interpretation of
the term propagation by the Supreme Court. (Extract with edits from
https://digitalcommons.law.byu.edu/cgi/viewcontent.cgi?article=3113&context=lawreview) .
Indiana is a country in the North of Europe. Jattism and Saintism are two major religion
followed there. Jattism followers have always propogated their religion and try to expand it.
The Jattis allow conversion from other religions to theirs only if its voluntarily. For such
conversions the rites and rituals have been increased recently and involves yagnas and
offerings to holy river, Vamuna which is the source of water and life for the majority of the
country. The environment activist lawyers have filed a PIL against the same on the ground
that such rites are causing pollution and contagious to the health of the people. Decide in the
light of Article 25.

(a) The propogation is protected as is not forceful conversion. Thus, withing the vires of
Article 25.

(b) The propogation is protected because there is only performance of rites and rituals not any
conversions

(c) The propogation is not protected as it is harmful to the environment. (d) The propogation
is not protected as Supreme Court is against the conversion.

. Indiana is a country in the North of Europe. Jattism and Saintism are two major religion
followed there. Jattism followers have always propogated their religion and try to expand it.
The Jattis allow conversion from other religions to theirs only if its voluntarily. For such
conversions, the rites and rituals have been increased recently and involves yagnas and
offerings to holy river, Vamuna which is the source of water and life for the majority of the
country. The environment activist lawyers have filed a PIL against the same on the ground
that such rites are causing pollution and contagious to the health of the people. The Saintism
preachers move to the court for the conversion discussed above. Will they succeed?

(a) Yes, since the conversion against Article 25 as it does not grant the right to convert
another person to one’s own religion but to transmit or spread one’s religion by an exposition
of its tenets.

(b) No, Since it is within the purview of Article 25 because it , if one takes the reductionist
understanding of propagation

(c) Yes, because it is involuntary

(d) None of above

. Talpade is a staunch follower of Vaishnism. He out of curiosity attends series of religious


events of Saintism and reads its scared books. Amid all this, he eventually gets influenced
and starts to follow Saintism leaving Vaishnism completely. Decide

(a) The conversion is protected as it is unfair to allow other religion person to attend such
events and get influenced.

(b) The conversion is not protected within Article 25 as it violates the right of Talpade

(c) Both A and B

(d) None of the above

. Decide, suppose Saintism holds an event on the occasion of birth of their Lord whom they
consider to be the divine power behind the creation of the world and in whose name most of
their rites and rituals are held. The event has various cultural and religious programs like -
reciting of holy books, sacrifices, holy bath, dances and bhajans etc. The event show cases
the limelight of the Saintism followers life and liberal beliefs. There are no restrictions on
visitors of other religion while allowing other religion followers the front seat. The organizers
also arranges for conversion rites for those who want to follow it, leaving Jattism or other
minor religions.

(a) Court can hold it constitutional under the reductionist idealogy of propagation

(b) Court can hold it unconstitutional under the reductionist idealogy of propagation

(c) Protection under Freedom of speech and expression as said that they were only holding an
event to propogate their religion under Article 25 and Article 19

(d) Both B and C

Passage (Q.-Q.): The Jharkhand High Court on Thursday made significant observations with
respect to payment of victim compensation under Section 357A of the Code of Criminal
Procedure. A Single Bench of Justice Ananda Sen has held that: The role of the Court is
recommendatory in nature and it cannot fix any quantum of victim compensation nor can
direct the authority to make payment of the same to the victim while exercising jurisdiction
under Section 357A of CrPC. When a person is an accused and the Court is not sure as to
whether he has committed the offence or not, in that situation, the accused cannot be saddled
with the liability to pay compensation. To direct an accused to pay any compensation or
victim compensation under Section 357A of the Code at the stage of bail by terming it as a
condition of bail also may amount to prejudging the guilt of an accused and such a course of
action runs completely contrary to the basic principle of criminal jurisprudence, i.e.,
presumption of innocence until proven guilty, the Bench held. It held that in terms of Section
357A of the Code, even if the accused is not identified or he is discharged or acquitted, the
victim is entitled to get compensation from the fund, which is created by the State. Even if
the accused is acquitted or discharged or even if the accused is not identified, it is the duty of
the State to protect its citizens and to rehabilitate them if they suffer loss and injury arising
out of a crime, the Court held. The Court is not vested with the power under Section 357A of
the Code to quantify the amount of compensation, rather that power is vested with the Legal
Services Authority. [Source- Akshita Saxena, ‘Section 357A- Court's Role Is Only
Recommendatory, Court Cannot Fix Any Quantum of Compensation to Victim: Jharkhand
High Court’, Live Law, , as accessed on 16th Feb. 2021]

. Pihu was a paramour of Salim. They want to get married but her family was against the inter
caste marriage. They decided to run away and get into wedding ties. But Pihu’s brother gets
to know about this plan and informed other family members of the same. He went to Salim
and asked him to stay away from her sister or else he will be beaten up to death soon. Salim’s
friend Asim was enraged with this unhealthy conversation and went to Pihu. He trapped her
into a conversation and suddenly throws a bottle full of acid on her face. She gets badly burnt
and was taken to hospital. Salim testified Aim to be the attacker and Asim was arrested by the
police. The trial court-

(a) Can provide the compensation to Pihu under section 357A of Cr.P.C to incur medical
expenses and other additional charges spend for curing acid attack.

(b) Cannot provide the compensation to Pihu under section 357A of Cr.P.C as court do not
have power to fix such compensation for victim and direct Asim to provide the same.
(c) Can provide the compensation to Pihu as court is only quantifying the amount of
compensation but only directing Asim to pay whatever the amount incurred for medical
expenses by Pihu.

(d) Cannot provide the compensation to Pihu as court has only recommendatory power under
section 357A of Cr.P.C and thus, has to direct the local authorities to pay the amount.

. Nitu was a medical student aged 21 years was with her friend Shalin. They went for dinner
one night and were returning back to home at about 12 in night. They were waiting for a bus
and saw a mini bus coming towards them with 3-4 passengers in it. They get into the bus and
saw that all 5 people in bus had conspired to hire Nitu and her friend. They ran to get down
from the bus but were forcefully taken to back seat of the bus and were tied separately. The
boy was beaten up badly and was thrown out of bus in mid[1]way. One by one other boy
including the driver went back seat to Nitu and attempted rape with her. She was brutally
assaulted and was left half dead in the bus. The police reported the crime and arrested 4 out
of 5 accused and trial begins. The magistrate denied the bail at prima facie case and asked for
compensation of Rs. 1,00,000 to be payable to victim suffering medical expenses in
Singapore. The defence lawyer contended it to be unlawful. Decide-

(a) The court cannot grant such victim compensation to Nitu in case of rape as there is no
sufficient proof prima facie that these 4 accused had committed the offence.

(b) The court can grant victim compensation to Nitu as 4 out of 5 accused were arrested and
can be asked to pay the medical expenses incurred by the victim to save her life.

(c) The court cannot grant such victim compensation to Nitu as it has to assess the injury
caused and ascertain the amount to be paid and can only direct state authorities for the same.

(d) The court can grant victim compensation to Nitu as crime committed by accused was
grave in nature and victim was brutally assaulted and raped.

. Munii, a minor girl of 13 year old, was sexually abused by a 30 year old man, Mr. Rajesh.
Mother of Munii filed a complaint in the District Legal Services Authority (DLSA) of state of
Odisha. DLSA took the case to trial court and charges were framed under POCSO against
Mr. Rajesh. Bail application was denied and court asked for the compensation to be paid to
the family members of Munii for mental and physical trauma suffered by her. Mr. Rajesh
consulted his lawyer and contended that court cannot ask for victim compensation unless his
guilt his decided and as such no bail is granted does not prove his guilt in the case.
Regardless of this, court asked the DLSA authorities to supervise the compensation to be paid
to victim for injuries done. Decide-

(a) The court was not correct in granting victim compensation to Munii and contention put
forth by Mr. Rajesh is valid. No compensation can be granted unless his guilt is proved.

(b) The court was correct in granting victim compensation to Munii and contention put forth
by Mr. Rajesh is invalid. Court has validly directed the DLSA to look after the compensation
to be paid.
(c) The court was incorrect in granting victim compensation to Munii as it has no power
under section 357A of Cr.P.C to quantify such compensation and force the accused to pay the
said amount.

(d) The court was correct in granting victim compensation to Munii and contention put forth
by Mr. Rajesh is invalid because principle under POCSO Act is guilty until proven innocent
and hence Mr. Rajesh is guilty of the said offence.

. A recent case was observed by the Supreme Court where accused was in live-in-relationship
with her paramour and was having sexual pleasures timely with each other consent. Later, she
asked the accused to marry her or she will file a frivolous case against him for committing
rape with her. Also, it comes to the notice of the accused that the girl was minor and she
deliberately had not disclosed her age to him. He refused for marrying the girl as he was with
him only to fulfill his sexual needs. The court dealt with the case and found accused to be
guilty of the offence of rape, prima facie, and that too of a minor girl. The court refused bail
to the accused and asked him to marry the girl if he wanted to be acquitted or pay her the
compensation for the same. The accused readily agreed to the same. Decide-

(a) The court cannot grant the accused bail based on the condition to marry the victim. The
court can only ask him for compensation to be paid.

(b) The court can neither grant conditional bail to marry the victim girl nor can ask accuse for
compensation. It can only convict the accused based on sufficient evidences.

(c) The court can grant the accused conditional bail to marry the victim girl but cannot ask
him for victim compensation as it has no power to direct such orders.

(d) The court can grant both conditional bails to the accused to marry the girl and ask him for
the compensation to be paid to the victim. It is under the power and jurisdiction of the court.

. The Legal Services Authorities Act, 19 was enacted to constitute legal services authorities
for providing free and competent legal services to the weaker sections of the society to ensure
that opportunities for securing justice were not denied to any citizen by reason of economic
or other disabilities and to organize Lok Adalats to ensure that the operation of the legal
system promoted justice on a basis of equal opportunity. The system of Lok Adalat, which is
an innovative mechanism for alternate dispute resolution, has proved effective for resolving
disputes in a spirit of conciliation outside the courts. What can be the reason behind
introducing Legal Services Authorities in each state?

(a) Legal Services Authorities in each state would ensure that cases are being decided fluently
and act as check and balance on judiciary.

(b) The State being the paramount protector of the life and liberty of each and every citizen
has some responsibility towards them. So victim compensation has to be paid by from state
funds.

(c) The State has the duty to safeguard the fundamental rights of the citizens and in any
offence committed against the victim is deemed to be against the state and thus has to
safeguard its interests.
(d) Legal Services Authorities are being introduced to interfere with the arbitrary rulings of
the court in order to provide victim compensation and legislature overlaps with the working
of judiciary.

Passage (Q.-Q.): “...we hold that once the parties agree to file a joint petition, pursuant to an
agreement/compromise in pending proceedings, then the parties are estopped from resiling
from the agreement. Therefore, the unilateral withdrawal of consent by the respondent,
especially after the appellant has performed his part of the terms in the memorandum of
agreement, is only a sharp practice which cannot be permitted or tolerated for a moment as it
would shatter the faith of the litigants in the justice delivery system and make a mockery of
alternative dispute resolution mechanism", observed a division bench comprising Justices A
M Muhammed Mustaque and CS Dias of the Kerala high court. The judgment authored by
Justice Dias observed:"Section 2 (e) of the Indian Contract Act, 18 states that every promise
and every set of promises, forming the consideration for each other, is an agreement. There
were reciprocal promises agreed by the parties, falling within the ambit of Section 51 of the
Indian Contract Act, 18, which was duly performed by the appellant. The respondent on
getting the custody of the children and receiving the compensation was obliged to perform
her part of the agreement, i.e., to give her consent for dissolution of the marriage". A
reference was made to the judgment of the Bombay High Court in Prakash Alumal Kalandari
v. Jahnavi Prakash Kalandari which held that when the parties agree to convert a pending
petition for divorce to a petition for divorce by mutual consent, on the basis of a compromise,
and on one of the parties fulfilling the terms of the compromise, the other party cannot
unilaterally withdraw consent. The Court held that the wife was precluded from withdrawing
her consent by the principles of promissory estoppels. "We are of the definite opinion that the
unilateral withdrawal of consent by the respondent is unsustainable in law and the Family
Court erred by allowing the applications filed by the respondent and dismissing the original
petition", the judgment held. 26 of 40 [Source- -‘Unilateral Withdrawal of Consent from Joint
Divorce Petition, After Other Party Has Performed Obligations under Agreement, Cannot Be
Permitted: Kerala’, , as accessed on 17th Feb. 2021]

. Sneha and Shaily entered into an agreement in which Sneha will be delivering bulks of
fashionable clothes to Shaily and she will be sharing the profits whatsoever earned from sale
of those clothes. Shaily sold the pieces on the decided price and shared those profits with
Sneha as decided among them. The last ten pieces she sold with extra margin of Rs. per
piece and kept that extra profit with her and shared the usual profit with Sneha. Sneha get to
know about this arrangement Shaily has done and asked her to share the profit earned from
extra margin. But Shaily refused to do so. Decide-

(a) Shaily has to share that part of extra margin with Sneha as she was party to the contract
and they both had entered into a valid agreement.

(b) Shaily need not have to share that part of extra margin with Sneha as she had done her
part in the agreement and Sneha cannot claim for something which is not agreed between
them.

(c) Shaily need not have to share the part of extra margin with Sneha as there has to be some
consideration from Sneha for claiming that extra profit.
(d) Shaily has to share the part of extra margin with Sneha as Sneha has validly performed
her part and she cannot withdraw from the agreement as decided.

. Dheeraj has two house properties and he decided to give one of them at rent. Since it was
double storey building, he thought to have two tenants instead on one to have more rental
income and shared an advertisement in the newspaper regarding the same. Mohit and Manoj
approached Dheeraj to have the rental apartment and subsequently decided their rental
amount of Rs. 5000 each per month. Manoj asked Dheeraj that he will be shifting to the
house after a period of month as he is busy with his prior commitments, to which Dheeraj
accepted. Soon after a week, Mrinal approached Dheeraj and asked him to give her the
apartment for rent to which she will pay Rs. 00 per month, more than Manoj. Dheeraj agreed
to the proposal and gave her the apartment. Mohit informed Manoj of the same and Mohit
sued Dheeraj for breach of contract. Decide-

(a) Dheeraj is liable for breach of contract and cannot give Mrinal apartment on rent though
for higher amount as he is bound by the contract with Manoj and estopped from doing so.

(b) Dheeraj is not liable for breach of contract and can give Mrinal apartment on rent as he is
getting a benefit of Rs. 3000 from her and there was no valid acceptance of proposal by
Manoj.

(c) Dheeraj is liable for breach of contract as he had validly accepted the proposal of Manoj
and can trace back from his promise if given much higher amount than Manoj.

(d) Dheeraj is not laible for breach of contract as there was no sufficient consideration from
Manoj in the agreement for rent and hence Dheeraj gave it to Mrinal for higher amount.

. Mr. Shahjad was a business man and was great lover of music. He visited Singapore for
business purpose and met Rahul Vaidya, one of the best singers of India and asked him for a
concert on his anniversary. Rahul agreed to the proposal and asked for amount of Rs. 5 lakh
to be given in advance. On day of anniversary of Mr. Shahjad, Rahul didn’t turn up as he was
suffering from cold and having the bad throat. Mr. Shahjad sued Rahul for specific
performance or refunds the amount with damages. Rahul contended it not to be breach of
contract and he is not liable to refund the amount of Rs.5 lakh. Decide-

(a) Rahul is correct in his contention and he is not liable for breach of contract as there is no
sufficient consideration paid by Mr. Shahjad for the agreement to be valid.

(b) Rahul is incorrect in his contention and he is liable for breach of contract as he cannot
withdraw his consent unilaterally from the agreement and has to perform in the concert as
promised.

(c) Rahul is correct in his contention and he is not liable for breach of contract as there was
no reciprocal promise on part of either party and thus, he can unilaterally withdraw his
consent.

(d) Rahul is incorrect in his contention and is liable for breach of contract as there is valid
acceptance and proposal from both sides and valid consideration given. Rahul has to go for
specific performance.
. Priya and Sushil were in love of each other and never want to be separated. Priya asked
Sushil to promise that he will never leave her alone in her lifetime. They started living in live-
in-relationship without actually being married to each other. Sushil got married to another girl
named Sukanya and they were living their happy life. Priya got to know about their marriage
and claimed the marriage to be invalid as she has already taken promise from Sushil. Sushil
cannot revoke his promise and he has to take divorce from Sukanya. Sushil refused Priya and
stated her that he still loves her but cannot marry her or divorce Sukanya. Priya sued him for
breach of contract. Decide-

(a) Sushil cannot withdraw his consent unilaterally from the promise given to Priya and he
has to keep his promise which he breached after marrying to Sukanya.

(b) Sushil can withdraw his consent unilaterally from the promise given to Priya as it was
given by mistake and there is no valid acceptance to such proposal.

(c) Sushil cannot withdraw his consent unilaterally from the promise given to Priya as there is
sufficient consideration in form of love from Priya and it’s a valid agreement.

(d) Sushil can withdraw his consent unilaterally from the promise given to Priya as it was not
a valid agreement between them and just mere promise which is not reciprocal.

. A divorce petition was filed in the family court of Patna where Arti and her husband
Shubham want to get divorced for not having mutual understanding as to their marriage. She
claimed that her marriage was done in forceful circumstances and she does not think this
marriage to be fit to be continued. Shubham also alleged that Arti is not a woman of good
character and he does not want to share his remaining life with her. The court assumes both
of them to be on mutual grounds and suggested for outside court settlement of any sum or
amount to be taken or returned back. Court will not interfere unless the decision is reached
within period of 2 months. In outside court settlement they decided to give child custody to
Arti and taking it as good opportunity to claim more monetary benefits, Arti withdraws her
consent to go by outside court settlement and want court to decide the matter. Decide-

(a) Arti cannot do so as she first mutually agreed for outside court settlement and cannot
withdraw her consent unilaterally from such agreement.

(b) Arti can do so as it was mere agreement to go for outside court settlement and she can
withdraw her consent unilaterally from such agreement and go by court orders.

(c) Arti cannot do so as Shubham as already agreed to the outside court settlement decision
and she cannot withdraw her consent once the decision is reached.

(d) Arti can do so as there was no agreement as to not withdrawing her unilateral consent
from outside court settlement procedure and can exercise it as her right.

Passage (Q.-Q.): Section 129 of the Criminal Procedure Code includes the power to dissolve
an unlawful assembly by civil force. The terms civil force, as opposed to armed force,
suggest the police’s use of force. In the case of Karam Singh v. Hardayal Singh, it was held
that there are three prerequisites to be fulfilled before any force can be used. Firstly, there
should be an unlawful assembly with the object of committing violence or an assembly of
five or more persons likely to cause a disturbance of the public peace. Secondly, such an
assembly should be ordered to be dissolved and, thirdly, the assembly should have refused to
disperse despite such orders to disperse. Thus the police can never have any justification from
their supervisors or the political executive for not acting or waiting for orders. It is a popular
misconception that the police need an order from a magistrate in order to act. Section 129
CrPC speaks of ‘Any executive magistrate or officer in charge of a police station or, in the
absence of such officer in charge, any police officer, not below the rank of a sub-inspector’.
This means that any one of them can order the use of force to disperse an unlawful assembly.
One often finds police officers taking a plea that while they wanted to use force, the
executive magistrate did not issue orders even though the law does not say that only an
executive magistrate can issue such an order. If there is an executive magistrate around, it is
generally considered good practice to obtain her or his orders, but it is not stated anywhere
that it is mandatory. On the contrary, police officers would be wrong in presuming that no
fault can be found with them if the order to use force or lethal force was duly given by an
executive magistrate. 28 of 40 [N.C. Asthana, ‘Explained: Why the Police Fails at Handling
Riots’ (2020) The Wire as accessed on 28 November 2020]

. Inspector Varun is in charge of a squad of five police constables tasked with arresting Richa,
a notorious gangster who has been running away from the police for some time. He receives
information that Richa is hiding in a remote Mumbai suburb and decides to take the local
train as soon as possible to catch her. In order to board the train, the Inspector and his team
arrive at Dadar station, but when they get there the platform is incredibly crowded and they
are unable to get to the train. Inspector Varun yells at the people on the platform angrily,
asking them to get out of the way, but no one hears him, since it’s the rush hour. To clear the
crowd, Inspector Varun opened fire. As he is called up for doing so by the police, he pleads
that he simply exerted his powers under Section 129 CrPC. Is his argument valid?

(a) No, Inspector Varun’s argument is not valid since it would have been better for him to
take a police van to capture Richa rather than boarding a local train.

(b) No, Inspector Varun’s argument is not valid since the crowd on the platform did not
constitute an unlawful assembly, and the Inspector did not have the authority to use force on
them under Section 129 of the CrPC.

(c) Yes, Inspector Varun’s argument is valid since by obstructing the police from catching the
train, the people on the platform constituted an unlawful assembly.

(d) Yes, Inspector Varun’s argument is valid since he had already warned the people to
disperse, and he only used force when they did not disperse and refused to follow his
commands.

. Inspector Varun and his team finally reached the location where Richa was hiding, a park in
a remote suburb. There, they found Richa talking to two of her associates, and overhear them
planning to rob a nearby bank. Inspector Varun shouts out to Richa and her associates, telling
them to “Stand still” and “Do not move!” Richa and her associates instead start running away
from the spot, and Inspector Varun and his team chase them down and start hitting them with
lathis (wooden sticks). Once again, Inspector Varun claims that this was a valid exercise of
his power under Section 129 of the CrPC. Which of the following are reasons why this was
not a valid exercise of the Inspector’s powers under Section 129 of the CrPC?

(a) Richa and her two associates together constitute three people which is less than five in
number, and therefore they did not constitute an unlawful assembly.

(b) Inspector Varun had ordered Richa and her associates to stand still, rather than disperse,
and therefore the requirements of Section 129 of the CrPC were not fulfilled.

(c) Neither (a) nor (b)

(d) Both (a) and (b)

. Richa and her associates ran away from the spot, and when Inspector Varun’s team finally
caught up with them, five more of Richa’s associates have joined her. The eight of them were
standing outside a local bank, armed with sticks and stones, and were breaking the windows
of the bank in an attempt to scare people and rob the bank. As Inspector Varun was tired of
chasing Richa and her group and had stopped somewhere to rest, sub-inspector Kumar was
leading the police team, and he ordered the team to start firing upon Richa’s group to stop
them. Did he have the authority to do so?

(a) No, Kumar has no authority to do so since a sub-inspector does not have the authority to
order the use of force under Section 129 of the CrPC.

(b) No, Kumar has no authority to use force against Richa and her group since they did not
constitute an unlawful assembly.

(c) Yes, Kumar has the authority to use force against Richa and her group since their actions
clearly showed that they constituted an unlawful assembly.

(d) Yes, Kumar has the authority to use force under Section 129 CrPC against Richa and her
group to disperse the unlawful assembly since he was leading the police team at the time.

. With the above context, Inspector Varun finally caught up with his team, and managed to
stop them just before they opened fire on Richa and her associates. He then shouted at Richa
and her group to disperse, and when they did not, he tried to call up the local executive
magistrate, to obtain her order to use force on Richa and her associates. Was it necessary for
him to obtain the executive magistrate’s orders in that situation?

(a) Yes, it was necessary for Inspector Varun to obtain the executive magistrate’s orders since
only an executive magistrate can order the use of force under Section 129 of the CrPC.

(b) No, it was not necessary for Inspector Varun to obtain the executive magistrate’s orders
since the sub-inspector Kumar had already issued the order to use force on Richa and her
associates, and there was no need for another order.
(c) No, it was not necessary for Inspector Varun to obtain the executive magistrate’s orders
since he himself had the authority under Section 129 of the CrPC to order the use of force to
disperse an unlawful assembly.

(d) Yes, it was necessary for Inspector Varun to obtain the executive magistrate’s orders
since obtaining the permission of the executive magistrate in such situations is always more
advisable.

. When the executive magistrate did not pick up Inspector Varun’s call, he decided to take
matters into his own hands. He shouted again at Richa’s group, telling them to disperse. This
time, Richa and her group got scared, and ran away in different directions. Inspector Varun
was, however, very keen to catch them, and he then ordered his team to start firing at them.
Did he have the authority to do so under Section 129 of the CrPC?

(a) Yes, Inspector Varun has the authority to do so since an inspector can order the use of
force to disperse an unlawful assembly under Section 129 of the CrPC.

(b) Yes, Inspector Varun has the authority to do so since Richa and her group did not listen to
his order to disperse when he shouted at them for the first time.

(c) No, Inspector Varun has no authority to do so since Richa and her group did not constitute
an unlawful assembly.

(d) No, Inspector Varun has no authority to do so since Richa and her group has already
dispersed in response to his orders.

23
Passage (Q.-Q.): Every person who enters into a profession assures of a skilled and efficient
work. He/she assures to work with reasonable care and skill and guarantees that the skills
possessed by him/her shall be exercised with due diligence. Similarly, a medical
professional does not guarantee the results of treatment to a patient. A surgeon may not
assure of a % affirmative result of the operation but guarantees that the skills possessed by
him in that branch of practice would be performed by him/her with reasonable competence.
Therefore, according to these standards, a professional would be negligent under two
instances- first, either the professional did not possess the requisite skills he/she was
required to possess; secondly, the professional did not exercise with reasonable
competence. Any person who provides medical treatment or advice to others impliedly
undertakes his possession of knowledge and skills. Therefore, such person has certain
duties to consider, such as decision regarding the undertaking of case by him/her and
decision regarding the kind of treatment to be administered. Any breach of such duties
indicates negligence on the person’s part and gives a right of action against him/her. As it is
said, to err is too human, but causing injury to others due to mere carelessness is
negligence. However, an error in judgement by a medical professional does not amount to
negligence as due to change of circumstances or even in the same circumstances, there can
be instances where the decision of the doctors goes wrong. But, considering the essential
factors related to treatment, if a wrong judgement is taken, then it would be negligence as
the medical professional had the efficient knowledge of the factors. The Supreme Court of
India in Kusum Sharma vs. Batra Hospital held that a doctor adopts procedure involving
higher element of risk, but he/she has an honest belief of greater chances of success in the
treatment. Therefore, if the doctor performs the treatment and it does not yield affirmative
results, then it does not amount to medical negligence. [Source- Priyanjali Priyadarshini,
‘Medical Negligence under law of Torts’, , as accessed on 14th Feb. 2021]

. Mr. Purohit was a reputed cardiologist in Gupta Hospital and runs a private clinic in other
hospital. Shushma, a year old diabetic woman, had breathing problem. She was not able to
find a good doctor to cure her disease. Her friend, Ms. Leela suggested her of Dr. Purohit
and asked her for consultation. Shushma took his appointment and Mr. Purohit asked her to
undergo typical tests to confirm whether she is suffering from asthama or not. Unaware of
Shushma being claustrophobic, she was exposed to typical tests and as a result of which
she died. Decide –

(a) Mr. Purohit cannot be held liable for this negligence as such facts regarding personal
information has to be conveyed by Shushma voluntarily before the tests.

(b) Mr. Purohit and the hospital can be held liable for the negligence as it is his duty to
collect relevant information of the patient before putting them to tests.

(c) Mr. Purohit cannot be held liable as he was not negligent. It is not his duty to ask
Shushma regarding her personal information.

(d) Mr. Purohit can be held liable because he should have applied his due diligence to collect
relevant information from Shushma before putting her to tests.

. Aryan was a special child as he was born with certain vitamin deficiencies and had
undeveloped bone marrow. His parents consulted various doctors and therapists and
approached Dr. Malti to cure their child. Doctor recommended several tests including blood
pressure and sugar checkups. After going by the reports, Dr. Malti prescribed certain heavy
dose medicines to the child and asked the parents to revisit after a month to observe the
effect and curing of the child. After 15 days of such appointment, Parents approached Dr.
Malti and alleged of her incompetency. Dr. Malti was unable to understand the situation and
came to know that she misread the count for white blood platelets as red blood platelets and
had given wrong tablets. She is –

(a) Not liable for medical negligence as she had only misread the reports and cannot be
alleged for not taking due care and diligence for prescribing medicines.

(b) Liable for gross medical negligence as she was professionally competent and had
efficient knowledge of effect of prescribing wrong medicines to the child.

(c) Not liable for medical negligence as she was only at fault to misread the reports and can
be said to take wrong decision but not negligent.

(d) Liable for medical negligence as she should take reasonable care and use her due
diligence before going by the reports as she was professionally competent doctor.

. Rahul was a 4th year student of M.B.B.S. Before his graduation, he went for a short trip
with his friends. On his trip, he met new experiences and explored different sides of his
studies. On one fine day of the trip, Rahul encountered an old lady, badly wounded, lying on
the path and was asking for help. Rahul analysed the problem and suggested her to apply
paste of wild plants to cure the wound. He himself made the paste and applied to the wound.
Due to much poisonous content in the wild plant, it get mixed with the blood and eventually
the lady died. Rahul can be-

(a) Held liable for medical negligence as he was not having proper knowledge regarding the
components of wild plants and has not applied reasonable care before applying it to the
wound.

(b) Held not liable for medical negligence as though he is professionally competent but does
not has requisite skills and knowledge regarding components of wild plants and was helping
the lady to cure the wound.

(c) Held liable for medical negligence as he was completing his graduation and can be
implied that he possess sufficient knowledge and skills of performing such medical help to
the injured persons.

(d) Held not liable for gross medical negligence as he is not professionally competent to
perform such medical help to injured persons and he do not possess such requisite skills or
knowledge for the same.

. Abhinav was a brilliant student and has secured a good rank in his college times. He
studied the other fields of biology and Vedic science and has researched much upon these
aspects. Soon he opened his own Hamdard Dawakhana and relied on homeopathic curing
of diseases. Nirmala listened much of this Dawakhana and visited for treatment of her
pancreatic pain. Due to overcrowd and rush at Dawakhana Abhinav gave Nirmala wrong
pills and as a result her internal reproductive parts get damaged. She claimed compensation
from Abhinav for her treatment. Abhinav refused and contended that it was mere mistake
and he is not liable for negligence. Decide-

(a) Abhinav is not liable for medical negligence as he rightly contended that it was mere
mistake on his part and not negligence in prescribing wrong medicines to Nirmala.

(b) Abhinav can be held liable for negligence as he should take reasonable care and apply
his due diligence before prescribing medicines to Nirmala.

(c) Abhinav cannot be held liable for medical negligence as he is not professionally
competent and does not has requisite skills or knowledge as to cure such diseases.

(d) Abhinav can be held liable for medical negligence as he has implied knowledge or skills
as to effect of such homeopathic medicines and should pay compensation to Nirmala.

. Shruti, a pregnant lady, was having her child after 20 years of her marriage. She was under
continuous guidance of gynaecologist, Dr. Suman. She analysed the critical circumstances
and stated that there can be complications in delivering the child of Shruti as she was
diabetic and hence to undergo surgery. Shruti insisted for normal delivery but Dr. Suman
refused to the proposal. Delivery was done perfectly but while doing the surgery, the Suman
forget to stitch an organ properly and it was left unclosed. Since no harm was caused to
Shruti, Dr. Suman asked her to perform another surgery to mend the mistake. Shruti can –
(a) Held Dr. Suman liable for medical negligence as she should have taken reasonable care
while performing the surgery and use her due diligence while stitching the parts.

(b) Held Dr. Suman not liable for medical negligence as she was not professionally
competent to perform the surgery and had done a mistake on her part.

(c) Held Dr. Suman liable for medical negligence as Shruti asked for normal delivery but she
has refused for it and is liable for her decision to undergo surgery for delivering the child.

(d) Held Dr. Suman not liable for medical negligence as she was correct with her decision to
perform surgery and not normal delivery of the child and had committed mere mistake while
performing surgery.

Passage (Q.-Q.): Negligence as a tort has evolved from the English law and accepted by the
Indian law as a substantially important tort. Negligence is of two types, civil and criminal and
each has various repercussions. In order to prove that an act was negligent, it is necessary
to prove all the essentials namely duty, breach of duty, damages and actual and proximate
cause. An important maxim regarding negligence i.e Res Ipsa Loquitur is used by the courts
when a negligent act cannot be explained. Also, the defences in a suit for negligence can be
used by the defendant to defend himself from a suit issued by the plaintiff. Criminal
negligence is said to take place when a person acts in a particular way which is an extreme
departure from which a reasonable person would act in a similar or same circumstance. The
difference in civil negligence is that the conduct may not be seen as a radical departure from
the way a reasonable person would have responded. Civil negligence occurs when a person
fails to exercise ordinary care or due diligence but criminal negligence relates to a conduct
that is considered so extreme and rash that it is a clear divergence from the way an
ordinarily prudent person would act and is considered to be more than just a mistake in
judgment or distraction. In civil negligence, there is a lesser burden of proof because the
plaintiff in such a case only has to prove that it is most likely that the defendant was
negligent. But in criminal negligence, the plaintiff has to prove “beyond a reasonable doubt”
that the defendant was negligent which is the highest standard of proof which means that the
evidence is so strong that there is no other logical explanation besides the fact that the
defendant acted with criminal negligence. The punishment for a person who was liable in a
civil negligence case only extends to the extent of damage caused to the plaintiff i.e
compensation for the damages. In criminal negligence cases, the punishment is much more
serious and can be convicted for a prison term, fine and probation supervision. Example the
punishment for criminal negligence amounting to death under section 304A of IPC can
extend to 2 years of jail and fine or both.

. Akash is a surgeon, who while operating a child of 12 years old unintentionally leaves in his
body a scalpels. It was a knee resection surgery. As a result, the child suffers internal
bleeding and infection causing him permanent impairment. Decide in light of civil wrong

(a) Yes, Akash is liable as he had duty of care towards his patient.

(b) No, Akash is not liable as it was done unintentionally.

(c) Yes, Akash is liable as he is a part of a noble profession which requires him to be more
vigilant and perform reasonable care.
(d) No, because the impairment was not the proximate cause of Akash’s act.

. Suppose, in the above question after the operation, the patient recovered without
complication noticed, but later after 1.5 months, the child falls while playing in the garden. As
a consequence, there started internal bleeding, near the knee. The family took him to the
hospital and found out about the scalpels and an dormant internal infection that it caused.
He underwent other surgery to remove the scalpel found and to stop bleeding. However, he
got permanently impaired. Now, decide, if Akash be liable for negligence.

(a) Yes, Akash is liable for negligence since the impairment was caused due to the
negligence act of his.

(b) No, Akash is not liable since the internal bleeding and impairment was not result of his
breach of duty of care.

(c) Yes, because if not internal bleeding but the infection found was the direct consequence
of the negligent act of Akash.

(d) None of the above

. A, was driving a car on a clear road. He drove safe within the permissible speed limit.
However, while his phone rang and he got distracted, a boy of 5 years from a nearby shop
ran to pick up his toy car in the middle of the street. Ms. B, the mother of the child, a young
aged woman, who was busy dealing with the shopkeeper. She saw him running but did not
come to stop her, carelessly. A, suddenly applies break, but hits the child. Decide.

(a) A is liable, though he was on a secluded street as he negligently drove the car and
breached the duty of care.

(b) A is not liable as he took the reasonable care of driving within the norms and applying the
brakes.

(c) A is liable as but Ms. B is also liable for contributory negligence as she failed to take care
of her child.

(d) A is not liable as the child ran and came in front of the car out of nowhere and A couldn’t
have foreseen such happening.

. Wilson was an event manager hired by Mr. Sharon for his daughter’s destination wedding.
While preparing for the reception, he took charge in his hand and helped his decorators to
do various tasks as time was running out and many things were clogged up. Wilson and two
of his men were fixing a chandelier at a height of about 7 feet above the ground. There were
people on the ground, when suddenly Wilson was distracted by a call on his phone and he
releases the rope tied to it. Consequently, the chandelier fell on Mr. Sharon and he died.
Decide

(a) Wilson is liable for criminal negligence.

(b) Wilson is liable for Tortious Liability of negligence.

(c) Wilson is not liable for tortious liability alone, but with his two men.
(d) Wilson is not liable for criminal liability as there is was no mens rea behind his act, which
is an essential requisite to constitute crime

Passage (Q.-Q.): The constitutional text also creates certain express defenses to crimes.
Consider the Bill of Attainder and Ex Post Facto Clause. The first half bars a legislature from
enacting a law that names and convicts someone of a crime without a trial. The second half
forbids a legislature from retroactively applying a statute defining a new crime or enhancing
the penalty for an old one. The First Amendment also takes away from Congress the
authority to make any law trespassing on certain civil liberties, which naturally includes any
law making it a crime to engage in the conduct that the provision safeguards. Defendants in
both federal and state criminal cases may defend against a charged federal offense on the
ground that the statute violates the First Amendment. The clauses have this in common:
they all address aspects of substantive criminal law. One defines a crime in the text of the
Constitution. Some describe the type of conduct that the government should outlaw. Others
place certain primary conduct entirely out of bounds. The Due Process Clause does none of
those things. Instead it ensures that no one can be criminally punished unless he has
committed a criminal offense defined by a different positive law and then only in compliance
with whatever procedural restraints the law elsewhere requires. The phrase “due process of
law” comes from a fourteenth century act of Parliament, stating that “no Man of what Estate
or Condition that he be, shall be put out of Land or Tenement, nor taken, nor imprisoned, nor
disinherited, nor put to Death, without being brought in Answer by due Process of the Law.
That provision, in turn, traces its lineage to Chapter 39 of Magna Carta of 1215, a document
that rivals our own Constitution in the protections it affords against arbitrary government
conduct. Chapter 39 provided that no free man is to be taken or imprisoned or disseized or
outlawed or exiled or in any way ruined, nor will we go or send against him, except by the
lawful judgement of his peers or by the law of the land. (Extract with edits from
https://www.harvard-jlpp.com/wp-content/uploads/sites/21/2020/01/Larkin[1]Canaparo-
FINAL.pd)

. Indiana is a highly developed country. It has become the first country in world whose
complete demographic area comes under CCTV surveillance since January 2014. In
December 2017, the Indiana parliament enacted a law which had provisions that if any
person is caught under the camera committing a crime, he will be assumed guilty during the
trial and he will have the burden to prove that he has not committed the crime. Decide (a)
The law deprives the accused of a trial (b) The law does not deprive the accused of a trial (c)
The motive behind enacting the law is good (d) The conduct of the government is arbitrary .
Based on the above question, In January 2020, the parliament of Indiana brought an
amendment for extending the new law to the acts committed after March 2019 and punish
convicts for the same. The amendment is:

(a) Against chapter 39 of Magna Carta

(b) Against second half of the Bill of Attainder and Ex Post Facto Clause

(c) Against first half of the Bill of Attainder and Ex Post Facto Clause
(d) None of the above

. In January 2020, Indiana successfully developed a technology “Lie Detect” which can
identify if a person is speaking a lie or not with % accuracy. If the government plans to use
the same for convicting a person found lying by it, how would the author response to it

(a) He will find it as conviction of the accused without a trial.

(b) He will allege retroactive application of a statute defining a new crime or enhancing the
penalty for an old one.

(c) Both a and b

(d) None of the above

. If “Lie Detect” is used to determine veracity of the statements of the accused for his past
acts and reducing his punishment if he is found to tell the truth, the author will

(a) Not find it problematic

(b) Find it problematic against the Bill of Attainder and Ex Post Facto Clause

(c) Find it problematic for not following due process

(d) Find it problematic with Chapter 39 of Magna Carta of 1215

Passage (Q.-Q.): Article 14 of the Indian constitution of India provides that the state shall not
deny to any person Equality before the law or the equal protection of the laws in the Territory
of India. Article 14 uses two expressions "Equality before law "which implies the absence of
any special privileges in favour of individuals and the subject of all classes to the ordinary
law and equal protection of the law which implies "Equal Treatment in Equal
Circumstances". Article 14 permits classification but prohibits class legislation. Class
legislation is that which makes an improper discrimination by conferring particular privileges
upon a class of person arbitrarily selected from a large number of persons. Article 14 does
not forbid reasonable classification of persons, Objects, transactions by the legislature for
the purpose of achieving specific ends but the classification should be reasonable.
Associated article is Article 15 of the Constitution. It states that no citizen shall, on grounds
only of religion, race, caste, sex, place of birth or any of them, be restricted with regard to the
use of wells, tanks, bathing ghats maintained wholly or partly out of State funds or dedicated
to the use of the general public. Article 15 of the Constitution also states that any provisions
made for the betterment of women and children shall not stand in violation of Article 14
standards. It also states that the State shall not deny to any person equality before law and
equal protection of the laws within the territory of India.

. Suraj Rathi is the 'sarpanch' of a small village Charanpur located in Rajasthan. Due to
scarcity of rainfall, the villagers faced a lot of problems arising from water shortage. In order
to tackle this problem, Suraj Rathi got 15 wells constructed at various locations in the village.
On the inaugration of these wells, he dedicated the wells for use by the inhabitants of the
village only. Certain students from Theatre Institute of Mumbai, born in Delhi, arrived in
Charanpur to shoot a documentary on the lives of Rajsthani women. They were to stay in the
village for a week. SurajRathi made it clear that the wells are meant to be used only by
villagers and hence the students cannot use it. One of the students, Nayantara, filed a writ
alleging violation of her fundamental rights under Article 15. Is Nayantara’s fundamental right
being violated?

(a) Yes, because she and her friends are being discriminated against on the basis of their
place of birth

(b) Yes, because Suraj Rathi is violating their right to life by denying them usage of water
from the wells

(c) No, because the wells have been especially constructed for usage by the villagers only

(d) None of the above

. Assume that Suraj Rathi had constructed a well in his own house for his private use but as
a matter of courtesy, he allows the inhabitants of village to draw water from it. Now if
Nayantara files a writ alleging violation of her fundamental rights under Article 15 then will
she succeed?

(a) Yes, because denial of a basic necessity like water violates right to life

(b) No, because the ambit of Article 15 does not include within itself the private wells

(c) No, because Nayantara cannot force Suraj Rathi to extend his courtesy to her as well

(d) None of the above

. Assume that in the given situation, instead of students, there would have been a group of
foreigners visiting Charanpur who wanted to stay there to experience the rural life of India
and they were denied usage of wells by SurajRathi. Would it amount to violation of Article 15
of the Constitution?

(a) Yes, because water is a basic necessity and its denial amounts to violation of right to life.

(b) Yes, because India follows the concept of 'vasudhaivakutumbakam' and it is not proper to
deny water to foreigners who are our guests

(c) No, because Article 15 is applicable only for citizens of the nation.

(d) None of the above

 
. Assume that seeing the plight of inhabitants of Charanpur, the government constructed
certain wells for usage by women. Menfolk of the village file a writ alleging violation of their
fundamental rights. Will they succeed?

(a) Yes, because the State is bound to provide equal treatment to all its citizens

(b) No, because the State has made a special provision for women.

(c) Yes, because their fundamental right to equality is being violated

(d) None of the above

. Sanjay Shukla is a medical professional who runs a laboratory specializing in performing


Coronavirus diagnostic tests. By way of a government notification, his laboratory was
disallowed from performing these tests. No similar ban was placed on any other laboratory.
Shukla is aggrieved and seeks to challenge this notification.

(a) Shukla shall get relief for he was unjustly denied his fundamental right to trade.

(b) Shukla shall not get relief as this was a class based legislation

(c) Shukla shall get relief only if he is able to show that he is in the same circumstances as
other laboratories.

(d) Shukla shall not get relief as the government is supreme under such circumstances of
health emergency.

. Shukla’s laboratory was banned but all other laboratories were open. The state gave a
reasonable explanation for the same. Decide

(a) The state needs to reasonably classify. Only a class of labs may be banned

(b) The state needs to reasonably classify. Shukla’s laboratory can form a class in itself.

(c) The state needs to ensure that Shukla’s circumstances are different

(d) None of the above

Passage (Q.-Q.): The Thane Government Railway Police (GRP) arrested four people on
Monday for allegedly running a fatka racket in Thane and are believed to be involved in five
incidents in January this year. Of the four who were arrested Hussain Shah (24) and Danish
Sheikh (25) stole mobile phones from train commuters by slapping and distracting them,
while Bilal Khan (25) and Mohammad Sayyed (32) sold them, the police said. “Shah and
Sheikh had a previous record of committing fatka crimes in the region. We checked our
records and located their current whereabouts,” said Smita Dhakne, senior police inspector,
Thane GRP station. They have been charged under Section 3 of the Indian Penal Code,
which pertains to committing theft by acts that may have resulted in death. Ms. Dhakne said
the two are under police custody and being interrogated. Theft under the Indian Penal Code
is said to occur whenever any person dishonestly takes any movable property out of the
possession of any other person without his consent. Courts always try to analyse whether
such consent is free consent or not. Such taking away may not be actual in nature but can
also be constructive i.e. taking away from the mental possession of someone. The presence
of a dishonest intention however is a very important element that needs to be proved under
every circumstance. In the event that such an act of theft leads to death of any individual, the
act in addition to being punishable under Section 3, is also punishable under Section 304 of
the Indian Penal Code that penalizes individuals for murder caused due to negligence. Such
provisions must be construed reasonably however and the intent of a person must be kept in
mind. [Source: https://www.thehindu.com/news/cities/mumbai/thane-grp-arrest-four-for-
operating-fatka[1]racket/article3067.ece]

. Abhinav Gangotri is a leading businessman. He is the CEO of Gangotri Enterprises which


is a leading publishing house. Gangotri is an ardent reader himself and often visits informal
discussion groups with his friends wherein his friends discuss the books they have recently
read. Gangotri comes to see a friend Yamuna Menon one day. He has always been
fascinated by Menon’s love for books. Generally looking through her book shelf, Gangotri
finds a book titled “The life of Rohit Patni” and is very fascinated about it. He keeps the book
with him for 15 days, reads it and returns the same to Menon; all without her knowing
anything about it.

(a) Gangotri must be punished under Section 3 of IPC for the book was taken away without
Menon’s consent.

(b) Gangotri must be punished under Section 3 of IPC for a moveable property was taken
away.

(c) Gangotri always intended to return the book. He cannot be punished under Section 3 for
there is no dishonest intention.

(d) Gangotri must not be punished under Section 3 of the IPC for the book was never moved
out of the constructive possession of Menon.

. After Gangotri took the book out of Menon’s cabinet without her consent, he accidentally
dropped the book on the main road right outside Menon’s bunglaw. Aakriti found the book
lying on the road. Being very interested in the life of Rohit Patni, Aakriti took the book to
keep it with her. (a) Aakriti has committed an offence within Section 3 of IPC for she failed to
return the book to the local police station. (b) Aakriti must be punished under Section 3 of
IPC for the book never belonged to her in the first place. (c) Aakriti must not be punished
under Section 3 of IPC for it would be against public policy to deny her, the right to
education. (d) Aakriti must not be punished under Section 3 of IPC for she did not take it out
of anyone’s possession. 21 of 36 . When Gangotri took the book out of Menon’s cabinet,
Gangotri kept the book in his pocket with the intent of taking it and making it a part of his
own collection.

(a) Gangotri commits an offence under 3, IPC the moment he picked up the book and
intended to dishonestly keep it.

(b) Gangotri commits an offence under Section 3 because he has moved the book from
Menon’s table without her consent.

(c) Gangotri does not commit an offence under Section 3 because Menon is careless about
her valuables.

(d) Gangotri did not commit an offence under Section 3 because he has still not left Menon’s
house and thus the book has not left Menon’s possession.
 

. Gangotri with the intention of taking the book away without Menon’s consent hid it
somewhere in her garden. He decided to come later and pick the book up so that he could
make it a part of his own collection as it was a very valuable book.

(a) Gangotri is guilty under Section 3 of the IPC as he took away the book from Menon’s
possession dishonestly.

(b) Gangotri is not guilty under Section 3 of the IPC as the book was never taken away from
Menon’s possession. It was still lying in her garden.

(c) Gangotri is guilty under Section 3 of IPC as he did not inform Menon about the
whereabouts of the book.

(d) None of the above.

. Gangotri was always a book thief and Menon knew this fact. She was always extra
cautious with him. When Gangotri entered Menon’s place, he looked at one book and
wanted to take it away. He put a red cover on it which was similar to another book in
Menon’s cabinet. He asked Menon if he could take the book with the red cover. Menon
agreed. Decide

(a) Gangotri is liable for theft as he dishonestly took Menon’s book without her consent.

(b) Gangotri is not liable for theft as Menon knew that Gangotri was a trickster and thus there
was implied consent for him taking the book

(c) Gangotri is liable because Menon’s consent was based on a mistake of fact

(d) Gangotri is not liable. He always intended to return the book.

. When Menon found out that her precious book had been stolen by Gangotri, she was
shocked. She hit her head with a stone and died for she could not tolerate being apart from
the book that Gangotri had stolen. Decide

(a) Gangotri is liable under Section 3 for he has caused the death of Menon

(b) Gangotri is liable under Section 304 for he negligently caused the death of Menon

(c) Both A and B

(d) Gangotri is not liable

Passage (Q.-Q.): The main criminal laws in India include the Indian Penal Code, Indian
Evidence Act and Criminal Procedure Code. People often refer to these three as general
laws. Apart from these three laws, we also have other laws that relate to specific kinds of
offences. The following are the most important elements of a crime. (1) Accused person: In
order to constitute a crime, it is important for somebody to commit it. The law should always
be able to pinpoint the person who is responsible for committing an offence. The term
accused “person” does not suggest that only a human being can commit offences. According
to Section 11 of IPC, the term “person” also includes a company and an association or body
of persons. (2) Mens rea: A mere person will never commit a crime unless he possesses
some intention to commit it. The element of mens rea itself comprises of certain inherent
elements. These include intention, motive or knowledge. Which of these elements must exist
in order to constitute an offence generally depends on the relevant provision. (3) Actus reus:
Merely possessing a guilty mind and thinking of committing a crime is not enough. The
accused person must also act on that intention and do something in its furtherance. Actus
reus basically refers to an act or omission which leads to the completion of an offence. Both
mens rea, as well as actus reus, together are important to create an offence. Actus reus can
be a positive act, such as stabbing a person to cause his death. It can also be an 22 of 36
omission (failure) to perform an action. (4) Injury: The last of the basic elements of crime is
an injury. There can be no crime if no person faces some kind of an injury. According to
Section 44 of IPC, “injury” means any harm caused to a person illegally either in mind, body,
reputation or property. However, there can be some crimes which might not require injuries
to anybody. [Source:
https://www.toppr.com/guides/legal-aptitude/indian-penal-code/elements-of-crime/]

. A has been found dead in his home. The police are trying to figure out who did it. The
neighbours stated that they had seen a UFO come and aliens from the UFO getting off and
into A’s home. Decide

(a) This is a crime for aliens come within the definition of persons within Section 11

(b) This is not a crime for aliens do not come within the definition of persons within Section
11

(c) This is a crime for a murder is always a crime irrespective of whatever happens

(d) None of the above

. ABC Ltd. is a partnership firm which indulges in chit funds. They robbed 7 persons of their
money through these chit funds. Decide

(a) The robbery is a crime for it was committed by a partnership firm which is a company.

(b) The robbery is not a crime for it was committed by a partnership firm which is not a
company

(c) The robbery is a crime for ABC Limited laundered money which is always a crime.

(d) None of the above

. C went out to hunt. When he thought that a deer was drinking water from a nearby stream,
he hot at the deer. The deer suddenly turned into a man. It turns out that the man was a
magician and had become a deer through magic. Assume that killing a deer is not a crime.
Decide

(a) C is liable for a crime for C had the mens rea to kill
(b) C is not liable for a crime for C only shot at the deer

(c) C is liable for a crime for he had the mens rea to shoot at the magician

(d) C is not liable for a crime for he did not have the intent to kill the magician.

. A with the knowledge that B has a weak liver forced him to consume a lot of alcohol. B
dies. Decide

(a) A is liable for a crime for he has caused the death of B as he knew about B’s disease.

(b) A is not liable for any crime for he did not have the required mens rea to commit a crime

(c) A's liability will depend on the circumstances of the death

(d) A's liability will depend on the provision he's being made liable under

. A is a doctor who handles patients in emergency. A comes across B who was badly injured
in an accident. B however requests A to not treat him. A seeing lack of resources and that B
has consented to his death, does not treat him and B dies. Decide

(a) A is liable for a crime as he is a doctor and he omitted from treating B

(b) A is not liable for a crime as he had received Bs consent for such omission

(c) A's liability will depend on the circumstances of the case

(d) A's liability will depend on the provision he is being sought to be brought under

. Brett Lee bowls a fast ball which hits Undertaker on the head and kills him. However
Undertaker is known for having 8 lives and fans know that he will resurrect again. Decide

(a) Brett Lee is not liable for a crime for no injury has been caused by killing Undertaker. He
will come back again

(b) Brett Lee is liable for a crime for injury has been caused by killing Undertaker

(c) Brett Lee's liability will depend on the facts and circumstances of the case

(d) Brett Lee's liability will depend on whether Undertaker actually returns to life or not.

Passage (Q.-Q.): The Bombay High Court on Friday granted permanent injunction against a
clothing manufacturer from using the trade name ISCKON and declared that the
International Society For Krishna Consciousness (ISCKON) is a well-known trade mark in
India within the meaning provided in Sections 2 (1)(zg) of the Trade Marks Act, 19. Justice
Colabawalla observed- "At the outset, it is important to note that the claim of the Plaintiff that
ISKCON is a well-known trade mark is not disputed by the Defendant. It is clear that
ISKCON is a coined trademark of the Plaintiff that is to say that the said term ISKCON did
not exist prior to the Plaintiff's adoption and use of the same. Since it is a coined trademark
which is associated exclusively with the Plaintiff, it undoubtedly deserves the highest degree
of protection. The documents / material evidently show that the Plaintiff's trade mark
ISKCON has acquired immense and long-standing reputation and goodwill throughout India
and abroad. There can be no doubt that the trade mark ISKCON is associated with the
Plaintiff and no one else." Furthermore, the Court said- "In view of the above, I have no
doubt in my mind that the Plaintiff's trademark ISKCON has come to enjoy a personality that
is beyond the mere products/services rendered thereunder and the recognition, reputation
and goodwill of the said trade mark ISKCON is today no longer restricted to any particular
class of goods or services. From the material placed on record, it is evident that - (a) the
Plaintiff's trademark ISKCON has wide acceptability; (b) the popularity of the Plaintiff' s
trademark ISKCON extends not only in India but in other countries as well; (c) the Plaintiff is
using its trade mark ISKCON openly, widely and continuously since the beginning; and (d)
the Plaintiff has taken several actions against various infringers in the past. I am therefore of
the opinion that Plaintiff's trade mark ISKCON satisfies the requirements and tests of a well-
known trade mark as contained in Sections 11(6), 11(7) and other provisions of the Trade
Marks Act, 19." Thus, Court held the Plaintiff's trademark ISKCON is a 'well-known' trade
mark in India within the meaning provided in Sections 2 (1)(zg) of the Trade Marks Act, 19
and decreed the suit. Source- https://www.livelaw.in/news-updates/isckon-a-well-known-
trademark-bombay-hc-restrains[1]clothing-manufacturer-from-using-trade-name-isckon-1548

. Plaintiff Apple Inc, a multinational technology company has filed a case against the
defendant for violating their well known, well established trademark by launching a music
company named, Apple Corps. Decide.

(a) Plaintiff will succeed in their suit as the defendant have violated their trademark.

(b) Plaintiff will succeed in their suit for trademark violation but will not be able to claim any
compensation.

(c) Plaintiff will not succeed in their suit for trademark violation as Plantiff’s is a technology
company whereas defendant has started a music company.

(d) Plaintiff will succeed in their suit for trademark violation only if they are able to prove that
they are well established brand.

. Kart Box is an e-commerce company selling apparels and footwear. Forever 21, a fast
fashion retailer filed a case against Kart Box for launching its in house clothing brand named
“Ever ” for being similar to a well-known brand of the plaintiff. Will Forever 21 succeed in its
claim for trademark violation?

(a) Forever 21 will not succeed as this is not a case of trademark violation.

(b) Forever 21 will not succeed as Ever is not deceptively similar and is not qualifying as
trademark violation.

(c) Forever 21 will succeed in its case of trademark violation as both the marks are similar.

(d) Both (a) and (b)

 
. Yipee noodles filed a case against Nestle Maggi for using the term “Magic Masala” and
filing an application with the registrar of Patents and trademark for grant of trademark for the
said term. Nestle Maggi argues that the term “Magic Masala” is associated with Maggi since
time inception and consumers have been association Maggi tastemaker with the name
Magic Masala. Which among the following would be the counter argument by Yipee noodles
in the present case? (a) Nestle Maggi was the first one to start using the term “Magic
Masala” for its tastemaker. (b) Nestle Maggi owns the trademark for “Magic Masala” and has
all rights reserved for the mark. (c) “Magic Masala” is a general term used by many other
brands such as Lays chips and therefore the trademark shouldn’t be granted to Nestle. (d)
“Magic Masala” is a general term used in every household of India and therefore should be
perceived as a generic term. . “Lal Balm” a pain relief ointment is a product by a local
manufacturer sold in retail shop in and around Amritsar. Manufacturers of Lal Balm filed a
case against Dabur for selling “Dabur Lal Teil” an ayurvedic baby massage oil. Dabur has
been in the business of selling “Dabur Lal Teil” for more than a decade and has been selling
their products not just in India but outside India as well. Decide.

(a) Dabur will not be liable for trademark infringement as they started using the disputed
term way before the plaintiff entered the business.

(b) Dabur will be liable for trademark infringement as the disputed term is resulting in
deception.

(c) Dabur will not be liable for trademark infringement as they are well known brand not just
in the country but also outside the country.

(d) Both (a) and (c)

. Bata, a well-recognized shoe making company filed a case against Bata Pvt. Ltd. for selling
shoe polish and other shoe accessories. Bata argued that Bata Pvt. Ltd has made
customers believe that the shoe polish too is from the Plantiff’s brand. Decide.

(a) Bata will succeed against Bata Pvt. Ltd. for trademark violation. (b) Bata will succeed
against Bata Pvt. Ltd. as it’s a clear case of copying one’s trademark to earn benefit out of it.

(c) Bata will not succeed against Bata Pvt. Ltd. as one is a shoe maker while other is into
shoe accessory business.

(d) Bata will not succeed against Bata Pvt. Ltd. as Bata haven’t suffered any loss yet.

. Which among the following statement is not a relevant argument in a suit filed for
trademark infringement?

(a) Party A’s trademark being deceptively similar to that of Party B.

(b) Party A’s trademark having same color and design scheme as that of party B.

(c) Party A selling its product at much lower price as that of party B.
(d) Party A having a well-established and well-known brand in the market and party B being
a new company.

24

Passage (Q.-Q.): The Calcutta High Court recently had occasion to rule that even a
trespasser, unless evicted by the due process of law, is entitled to electricity (Sukla Kar v.
The Calcutta Electric Supply Corporation Ltd. & Ors.) Justice Arindam Mukherjee, however,
clarified that even the electricity connection is granted to such a person, it would not create
any other right in her favour as regards the property. The petitioner in this case had moved
the High Court after her application for a new service connection was rejected by the
Calcutta Electric Supply Corporation (CESC). Mere possession doesn’t mean that the
possessor will have any rights, title and interest in respect of the said premises. The CESC
told the Court that it cannot provide a new service main in the premises occupied by the
petitioner as there already existed once. However, the petitioner can be provided a new
meter and a new connection from the existing main service, the Court was informed. Other
private respondents told the Court that the petitioner did not have rights over the property
where the electricity connection was sought. It was submitted that she had earlier been
enjoying electricity from an existing meter on the basis of an arrangement between the
petitioner and the private respondents. On account of failure to pay bills, the electricity was
disconnection, they contended. The High Court opined that the disputes over the premises,
whether it be ownership, title or interest, cannot stand in the way of the petitioner getting a
new electricity meter in her name, when the possession of the premises by her is admitted.
"Assuming without admitting that the petitioner does not have any right, title and interest in
respect of the said premises, the petitioner’s status then is that of a trespasser. Even a
trespasser, unless evicted by due process of law, is entitled to electricity. Electricity
connection, if granted to the petitioner, will not also create any right in her favour", reads the
order. Therefore, Justice Mukherjee proceeded to direct the petitioner to make a new
application for a meter and new electricity connection within 7 days. It further issued
directions for the CESC to process the application, inspect the premises, raise a quotation
and install a new meter to grant the petitioner a new electricity connection. The Court added
that it has not gone into the dispute between the petitioner and the private respondents, nor
will its order create or affect any right for the petitioner in respect of the premises. [Meera
Emmanuel, ‘Even a trespasser, unless evicted by law, is entitled to electricity: Calcutta High
Court’ < https://www.barandbench.com/news/litigation/even-a-trespasser-unless-evicted-by-
law-is-entitle[1]to-electricity-calcutta-high-court> as accessed 27 th December, 2020]

. A has been in possession of B’s house since long. C, his neighbour files a case against him
on the grounds that his cow ate leaves from a poisonous tree from his garden and died.
Decide.

(a) A is not liable because it was not A who planted the tree

(b) is not liable since he just had a mere possession of the house and not the garden

(c) is not liable because he did not do anything intentionally

(d) is not liable because having possession doesn’t mean he has rights and liabilities for the
same
 

68. A being the possessor of B’s house even after lawful eviction had stopped paying the
electricity bills after which he requested for a new connection in his name for which he was
rejected. He had therefore filed a case regarding the same. Decide

(a) He will lose since he has no rights over the property

(b) He will lose since he is just a mere possessor of the property

(c) He has to clear the previous bills; only then can he get a new one

(d) None of the above

. A being the possessor of B’s house had stopped paying the electricity bills after which he
requested for a new connection in his name for which he was rejected. He had therefore
filed a case regarding the same. Decide

(a) He will lose since he has no rights over the property

(b) He will lose since he is just a mere possessor of the property

(c) He has to clear the previous bills; only then can he get a new one

(d) None of the above

. A was staying at B’s property for past 3 years. The property had a huge garden in which
many beautiful shrubs including poisonous ones grew. Once, it so happened that A’s dog
chew on a such a poisonous flower and died. A then sues B for the same. Will he win?

(a) No, because B is no longer liable for his property.

(b) Yes, because it is B’s property.

(c) No, because A should have been careful while walking his dog.

(d) Yes, because B should’ve let A know of the types of plants present in the garden.

. A has possession of B’s property. A had stopped paying electricity bills and thus his
electricity was cut off. He therefore went on to file an application for a new connection on
July 24th, 2020 for which she was denied one. On July 25th, 2020, he got an eviction notice
from the court. She files a petition regarding the same. Decide

(a) She will get the connection since had asked for one before she was served an eviction
notice.

(b) She will not be given the connection since currently she is no more entitled to electricity.
(c) She will not get the electricity connection since has to first clear the previous bills

(d) She will not get the electricity connection since the whole country is under lockdown and
it is impossible to get a connection during the lockdown period

Passage (Q.-Q.): ‘Indians have nothing to fear,’ has been the refrain of the proponents of the
Citizenship Amendment Act (CAA). On the corollary move to implement a National Register
of Citizens (NRC), they keep changing their positions — sometimes they say NRC and CAA
have no link at all; other times they say the CAA will protect non-Muslims excluded from the
NRC exercise. At times they say it is an internal matter of India, but they also promise to
send back the “infiltrators” to places where they allegedly came from. The argument that
Indians have nothing to fear — made repeatedly by Prime Minister Narendra Modi and
Home Minister Amit Shah among others — is less a reassurance and more an assertion of
the intent of the NRC-CAA. Yes, Indians have nothing to fear. But to be assured of no fear,
she has to first PROVE that she is an Indian! The NRC puts onerous requirements that most
Indians will find unable to meet, Hindu or Muslim. But non-Muslims have, at least
theoretically, a route to citizenship under CAA. Muslims, if they cannot get into NRC — as a
large number of them like all other communities, will end up — will have no route to
citizenship. But the statement that ‘Indians have nothing to fear’ — while there are many
reasons why they are worried — also suggests that those who are worried, the protesters,
are not Indians or less Indians. The very act of protest itself undermines one’s credentials as
Indians. Considering the reflex response of the regime that labels every critic of it ‘anti-
national,’ this is a new twist. If you are Indian, you have nothing to worry. And inversely, if
you are worried, you may not be Indian! [Source:
https://www.thehindu.com/news/national/opinion-citizenship-amendment-act-the-
fear[1]factor/article.ece]

. Which of the following views can be correctly attributed to the author of the above
passage?

(a) The Indians should not take to the streets to protest against the CAA NRC as the newly
introduced law will not affect them in any manner.

(b) The intention of the CAA NRC law is to make the Indians feel less worried about their
citizenship

(c) Despite multiple assurances by Prime Minister Modi and Home Minister Amit Shah there
is speculation that the CAA NRC might strip off the citizenship of many people including
Hindus as well. Hindus might be able to use this to retain their citizenship but the Muslims
will have no way out.

(d) Indians should not be worried about CAA NRC exercise as it is being brought about to
deal with the Muslim infiltrators in the country

. As per the passage given above what is the reason given by the government for bringing
the CAA law?

(a) To send back the Muslim infiltrators where they belong

(b) To protect the non-Muslim excluded from the NRC exercise


(c) To strip the citizenship of the protesters who are less Indian

(d) To strip the citizenship of the government critic and Anti National

. Suppose if one Mr X is a Muslim and another Mr Y is a Hindu. Both X and Y feel to satisfy
the requirement under the NRC. As per as per the law explain above what effect will the
CAA have on their status of citizenship?

(a) Both Mr X and Mr will be denied citizenship as the field in the NRC exercise

(b) Both Mr X and Mr Y will be granted citizenship as even though they were excluded by the
NRC they will be observed as citizens under the CAA

(c) Mr X will continue to be a citizen of India as per the provisions of the CAA but Mr Y will
not be considered to be a citizen of the country

(d) Mr Y will continue to be a citizen of India as per the provisions of the CAA but Mr X will
not be considered to be a citizen of the country

. The author has expressed his views in support of the NRC CAA exercise. The above given
statement is true or false?

(a) True as the author constantly argues that the Indians have nothing to worry about the
NRC CAA

(b) False, as the author has argue that the concept of who is an Indian has itself come into
question after the government's response to criticism

(c) True as the author appreciates the reassurance given by Prime Minister Modi and Home
Minister Shah that Indians have nothing to worry about

(d) False as the author is worried about the non-secular nature of the law

. According to the above passage what is the response of the government to every critic?

(a) Critics will have no route to citizenship under the NRC CA

(b) Critics are labeled as Anti National and less Indian and thus they are raising their voice
against the law

(c) Critics should not worry as they are Indians and the law will not affect Indians

(d) Critics will not be able to meet the onerous requirements and the NRC

Passage (Q.-Q.): A contract of insurance is very similar to indemnity contracts. Here, the
insurer promises to compensate the insured for his losses. In return, he receives
consideration in the form of premium. However, the Contract Act does not strictly govern
these kinds of transactions. This is because the Insurance Act and other such laws contain
specific provisions for insurance contracts. Parties under Indemnity Contracts There are
generally two parties in indemnity contracts. The person who promises to indemnify for a
loss is the Indemnifier. On the other hand, the person whose losses the indemnifier promises
to make good is the Indemnified. We can also refer to the Indemnified party as the Indemnity
Holder. For example, in the earlier example, C is the Indemnifier and B is the Indemnity
Holder. Nature of Indemnity Contracts An indemnity contract may be either express or
implied. In other words, parties may expressly create such a contract as per their own terms.
The nature of circumstances may also create indemnity obligations impliedly. For example,
A does an act at the request of B. If B suffers some losses and A offers to compensate him,
they impliedly create an indemnity contract. Rights of an Indemnity Holder When parties
expressly make a contract of indemnity, they can determine their own terms and conditions.
However, sometimes they may not do so. In such a case, the indemnity holder can enforce
the following rights against the indemnifier: 1) The indemnifier will have to pay damages
which the indemnity holder will claim in a suit. 2) The indemnity holder can even compel the
indemnifier to pay the costs he incurs in litigating the suit. 20 of 40 3) If the parties agree to
legally compromise the suit, the indemnifier has to pay the compromise amount.

. A wheelchair manufacturer enters into an agreement with a large hospital to provide 500
wheelchairs at a discount price. The manufacturer asks that a clause be included in the
contract, in which the hospital agrees to protect the company from any losses or lawsuits
should patients be injured while using any of the wheelchairs. Is the following an indemnity
clause?

(a) Yes, it is, as the patients would be compensated incase they face any injury.

(b) No, injured patients would be indemnified, and since they are not party to the contract
this won’t be an indemnity clause.

(c) Yes, as the hospital indemnifies the wheelchair company, or the hospital guarantees for
any losses or injuries that may occur.

(d) No, as the manufacturer is not protected from any litigations or compensatory charges.

. Lola has a homeowner’s insurance policy on her home in Texas. The insurance company
has agreed to indemnify Lola against damages to her home and the personal property kept
there, from many types of damage, including fire, burglary, and liability if someone gets
injured on Lola’s property. A visiting neighbor trips on a crack in the walkway and falls,
breaking her arm. Will her contract protect her from liability?

(a) No, as the indemnity is only in terms of injury from fire, burglary or someone getting hurt
due to these.

(b) No, as the person tipped on the walkway which is not part of the home and outside it’s
periphery.

(c) Yes, the insurance company would protect Lola from the medical bills and other losses
claimed by the neighbor by paying the claim.

(d) Yes, as the contract of indemnity protects you from all the future losses incurred to you.

 
. Luke takes his car to the shop for repairs that will take a few days. The shop offers Luke a
loaner car so he can get back and forth to work. Luke signs the shop’s loaner car
agreement, which requires Luke have insurance, and includes an indemnifing clause. While
driving through town in in the loaner car, Luke rear-ends a car at a stoplight. Luke suffers
minor injuries, but the driver of the other car has several moderate injuries, and the damage
to the loaner car is substantial. A couple of weeks later, the other driver demands payment
from the repair shop, as owner of the car that hit her, for medical bills, repairs to her own car,
and pain and suffering. Who is responsible for this compensation?

(a) Luke as his written agreement to indemnify the repair shop protects the business from
liability.

(b) The lady will claim it from the repair shop and later they can make the dealing with Luke
based on compromise.

(c) Luke, who is solely responsible for any events surrounding the accident, as he was
driving the car.

(d) The repair shop as they are the owner of the car and thus responsible for who they let
drive their vehicle.

. Robert is remodelling his house to transform it into the home of his dreams. The front porch
and yard are being torn down and reconstructed, leaving Robert to worry about whether one
of the contractors may trip over debris and sue him for injuries. To protect himself, Robert
asks all of the contractors and laborers working on his home to sign an indemnity and hold
harmless agreement for the same. A worker while leaving the site for his house, falls and
injures himself over the pavement. Will he be indemnified by Robert?

(a) No, he won’t as the injury is not work related.

(b) Yes, he would, as he wouldn’t have been hurt if he wasn’t leaving from work itself.

(c) No, as the indemnity agreement protects him for paying for any liability caused due to
injury.

(d) Yes, as he is the indemnifier for all liabilities caused to his workers employed on the site.

. Mr. X is a surgeon and currently doing practice in India. Mr. Y offers him Job in Pakistan
and asks him to start his practice here. Mr. Y also promises him that he will save him from
loss incurred due to practice here. Identify the indemnifier?

(a) Mr X is the indemnifier because he benefits from the contract.

(b) Mr. Y is the indemnifier because he would take up the liability.

(c) Indemnity contracts cannot work in foreign nations, as it is part of Indian Contract Act.

(d) Third person would be indemnifier as, indemnity contracts require a third person to bear
the liability.

 
. A contracts with the Governments to return to India from abroad after completing his
studies and serve the Government for a fixed period. He fails to return to India. Is this a
contract of indemnity?

(a) This is a contract of indemnity and he is bound to reimburse the Government.

(b) This is a contract of indemnity but he is not the indemnifier.

(c) This is not a contract of indemnity but a contractual obligation.

(d) This is not a contract of indemnity as it does not fulfill the requisites for it.

Passage (Q.-Q.): Exceptions to Offence of Murder under Section 300 IPC, which could
mitigate it to culpable homicide not amounting to murder are:- Exception I – Grave and
Sudden Provocation as mitigation Culpable homicide is not murder if the offender, whilst
deprived of the power of self-control by grave and sudden provocation, causes the death of
the person who gave the provocation or causes the death of any other person by mistake or
accident. Essentials – The following conditions must be complied with in order to invoke the
benefits of this clause:- 1. The deceased must have given provocation to the accused. 2.
The provocation must be grave. 3. The provocation must be sudden. 4. The offender, by
reason of the said provocation, should have been deprived of his power of self[1]control. 5.
The accused killed the deceased during the continuance of the deprivation of the power of
self[1]control. 6. The offender must have caused the death of the person who gave the
provocation or that of any other person by mistake or accident. Exception II – Exceeding the
Right of Private Defence Culpable homicide is not murder if the offender, in the exercise in
good faith of the right of private defence of person or property, exceeds the power given to
him by law and causes the death of the person against whom he is exercising such right of
defence without premeditation, and without any intention of doing more harm than is
necessary for the purpose of such defence. Exception III – Public servants exceeding his
powers Culpable homicide is not murder if the offender, being a public servant or aiding a
public servant acting for the advancement of public justice, exceeds the powers given to him
by law, and causes death by doing an act which he, in good faith, believes to be lawful and
necessary for the due discharge of his duty as such public servant and without ill-will
towards the person whose death is caused. Exception IV — Sudden Fight This exception
applies to cases wherein death is caused in a sudden fight without premeditation in the heat
of passion in a sudden quarrel; so long as the fight is unpremeditated and sudden, the
accused, irrespective of his conduct before the quarrel, earns the mitigation provided for in
Exception 4 to Section 300, IPC. Subject to the condition that he did not in the course of fight
take undue advantage of or act in a cruel or unusual manner. Exception V – Consent
Culpable homicide is not murder when the person whose death is caused, being above the
age of eighteen years, suffers death or takes the risk of death with his own free and
voluntary consent. [Source: Indian Penal Code, 18, https://www.legalbites.in/law-notes-ipc-
exceptions-to-offence-of[1]murder/]

. Ram came home from work and found that his neighbor Sunil was molesting his 8 years
old daughter. He was so shocked and angry that he took axe and went towards Sunil to kill
him. Seeing him, Sunil pushed him and ran for his life. Ram ran behind Sunil with the axe
and found him 6 hours later hiding behind a bus. Ram caught him and killed him with his
axe. Decide whether the exception of Grave and sudden provocation will apply or not.

(a) Yes, because Sunil’s act could be considered as an heinous crime and was grave in
nature.
(b) Yes, because seeing his daughter getting molested, the father was filled with rage and
anger.

(c) No, because the act of killing Sunil was not sudden.

(d) Yes, because the act of Sunil deserves to be punished in this manner.

. Raj was sleeping in his house when a thief entered his house. Due to the noise, Raj woke
up and caught the thief. The thief tried to threaten raj with a baseball bat kept there and
started to escape from the house. Raj in order to prevent the thief from escaping, took out
his pistol from the shelf and fired at him, thus killing him. Will he be sued for murder or any
exception can be granted.

(a) No, because the thief was threatening raj and could have harmed it, therefore raj acted in
private defence

(b) Yes, because killing is not a justified punishment for stealing.

(c) Yes, because Raj used more power than was necessarily required for private defence.

(d) No because the thief committed mistake and should have been punished.

. X was a policeman who was trying to catch a dangerous armed culprit, who was hiding in a
factory. Unaware of this, the watchman of the factory who was taking round suddenly
appeared in front of the policeman holding his gun. As it was dark, “X” couldn’t see properly
and shot him dead. Will he be charged with murder?

(a) Yes, because it was because of his negligence that the watchman got killed.

(b) No, because he fired in discharge of his duty, thinking him to be the armed culprit.

(c) No, because it was an accident which took place because it was dark.

(d) Yes, because the policeman should have given warning before firing bullet.

. Mukesh and Nitin had a conflict due to which they had physically fought many times before.
Mukesh wanted to end this chapter so he called Nitin and asked to meet him in person. He
carried with him a knife. Soon their conversation got heated up and both got involved in a
physical fight. Finding the right time, Mukesh stabbed nitin with his knife many times. Will
any of the exception apply here?

(a) Yes, the exception of grave and sudden provocation will be applicable as Mukesh killed
Nitin in the heat of the moment.

(b) No, because the exception of sudden fight cannot be applied as it was planned by
Mukesh beforehand and was executed in a cruel manner.

(c) No, because he killed Nitin out of personal conflict and rage.

(d) Yes, because he acted in self defence against Nitin.


 

. A 19 year old blind person named Jai was living with his guardian who used to be very rude
to him. He used to torture Jai over his disability and taunted him everyday that he is a
burden to everyone. Finally, Jai got depressed and requested his guardian to kill him in order
to give him relief from all the pain. Will the guardian be liable of murder?

(a) Yes, because the consent given by Jai was not voluntary and free will, he was forced by
his guardian.

(b) Yes, because he killed jai because of his personal motive.

(c) No, because the guardian relieved Jai from all the pain and sufferings.

(d) No because the guardian had the consent of Jai who was over the age of 18 and had
given voluntary consent.

. “A” is a police officer who was investigating a matter related to terrorism in Delhi. He
arrested few people who were suspected to be involved in the crime. During investigation,
he found that none of them were involved in the crime and had to release them. One of
those suspect “X” met the police officer “A” while he was not on duty and engaged into a
physical conflict with him. “X” tried to stab “A” with a knife that he brought with him but “A”
quickly shot him dead with his personal licensed gun. Will “A” will be charged of murder.

(a) Yes, because he did not kill “X” in the discharge of his duty therefore the exception of
public servant exceeding his powers is not available.

(b) No, because he killed “X” out of private defence who was otherwise trying to stab him to
death.

(c) No, because he killed “X” out of mistake.

(d) Yes, because he went to meet “X” with a personal gun which shows that it was pre
planned.

Passage (Q.-Q.): In an interesting order, Justice Rajiv Narain Raina of the Punjab and
Haryana High Court adjourned a Civil Revision on Petitioner's counsel's pleading that the
court was in a bad mood and he would like to argue the case on some other day. The order
dated February 4 discloses that Justice Raina dismissed four urgent cases listed before him
that day, one after the other. This led to the Petitioner's counsel, Advocate KS Sidhu,
assuming that the court was not in the right mood to allow his case. "Counsel assuming that
mood of the Court is bad this morning dismissing the first four urgent cases one after the
other with orders dictated in Court, prays that time may be granted to him to argue the case
on some other day. I grant permission for an adjournment but not without saying that those
cases were not worth admission," the order read. The rules for adjournments are listed
under Order XVII of the CPC. Rule 1 thereunder clearly stipulates that the Court may grant
an adjournment to a party if "sufficient cause" is shown; provided that no such adjournment
shall be granted more than three times to a party during hearing. Sub-Rule (2) of Rule 1
further stipulates that adjournments are to be granted only if the circumstances are "beyond
the control" of the party who seeks it. However, these rules may be applicable only in cases
of civil courts and not any other authorities. [Extracted (edited) from: LiveLaw News Network,
https://www.livelaw.in/news[1]updates/lawyer-seeks-adjournment-as-the-mood-of-the-court-
is-bad-hc-allows-the-request-read-order[1]1529]
 

. Anahad Narain was arguing before the NCDRC which is a commission defending Piramal
Healthcare Limited in a tough case. As the judge started grilling him on the nitty-gritties of
the matter, Narain started to find it uncomfortable as there were too many things he did not
have answers to. Narain feigned unconsciousness. As he pretended to regain his
consciousness, he sought an adjournment citing ill[1]health from the Chairman of the
Commission. To his utter shock, the chairman denied such adjournment. Narain argued that
adjournment must be granted for the circumstances were beyond his control. Is the
chairman bound by Narain’s argument?

(a) Narain may be granted adjournment for he is not well in the mind of the judge.

(b) Narain may not be granted adjournment for he has feigned unconsciousness.

(c) Narain may be granted adjournment for the case is a tough one and he needs to prepare.

(d) Narain may not be granted adjournment for the CPC does not apply to Commissions and
thus they are not bound by the rule of circumstances being “beyond his control”.

. Narain always feigns unconsciousness before a judge whenever he is put in a tight spot. A
case was being argued before the Calcutta High Court. Since Narain is now very
experienced at falling unconscious and seeking adjournments, he keeps doing it whenever
he finds a case difficult to deal with. As expected, this time too, the judge believes him to be
genuinely unwell.

(a) Narain may be granted adjournment for he is not well.

(b) Narain may not be granted adjournment for he has feigned unconsciousness.

(c) Narain may be granted adjournment for the case is a tough one and he needs to prepare.

(d) Narain may not be granted adjournment for such acts are against the interest of justice.

. Narain is lazy and keeps procrastinating since the feigning adjournment business has been
working for him quite well now. Before the same judge, he has taken adjournments on
23/05/2016 and on 5/08/2016. The lawyer for the other side, learning from Narain’s tactics
also performs similar courtroom drama in the form of extreme headache and the judge
grants him adjournments on 29/08/2016 and 11/09/2016. Narain is again found to be in a
tough spot during the final arguments on 30/09/2016 and yet again seeks for adjournment.
This time however, Narain is genuinely feeling unwell because of too much partying last
night.

(a) Narain may not be granted an adjournment for he’s the reason for such immense delay.

(b) Narain may be granted an adjournment for the circumstances are beyond his control.

(c) Narain may not be granted an adjournment for the matter has been adjourned for more
than three times now.

(d) Narain may be granted an adjournment in the interest of justice.


 

. Which of the following scenarios would be the most likely to be granted an adjournment by
the court?

(a) Devgan was arguing before Justice Amish of the Trivandrum Civil Court. He sought an
adjournment on the ground that his junior was sick and there was no one to take care of the
junior. Such circumstances were beyond his control and thus fell under the relevant rule.

(b) On 29/04/2019, having argued for almost 2 hours, Goswami stated that his medical
condition did not allow him to argue for more than 2 hours before the court and thus sought
for an adjournment. He has already sought adjournments on 23/01/2019, 24/02/2019 and
15/03/2019.

(c) Rabish submitted before the court that he wanted to pick his daughter up from school and
was thus constrained to seek an adjournment before the court.

(d) None of these would be eligible for an adjournment.

. Laxmi, an advocate appears before the Karnataka High Court. While arguing, she seeks
her fourth adjournment on a frivolous ground. The judge however grants her an
adjournment. Decide

(a) The judge has acted illegally for he could only grant adjournments on reasons which are
beyond Laxmi’s control.

(b) The judge has acted illegally for he could only grant 3 adjournments to one person.

(c) The judge has acted legally for the rules above do not apply to his court.

(d) The judge has acted legally for adjournments are at his discretion.

. Laxmi appears before the civil court in Rampur. She argues that she had to see a friend
immediately in the hospital and thus cannot argue a matter. Upon being asked whether she
can send a junior to argue the matter, she stated that since the matter is close to her heart,
she would want to argue the same herself. Decide

(a) The judge can grant her an adjournment for the circumstances are beyond her control.

(b) The judge should not grant her an adjournment for the matter being close to her heart is
not a circumstance beyond her control.

(c) The judge must be sensitive to Laxmi’s friend being admitted in the hospital.

(d) The judge must order Laxmi’s junior to argue the case.

Passage (Q.-Q.): The definition of "private part" has to be interpreted as per the social
context, a Special Court dealing with cases pertaining to the Protection of Children from
Sexual Offences (POCSO) Act said convicting a person for committing sexual assault under
Sections 354, 354A of the Indian Penal Code and Section 10 of the POCSO Act. (State of
Maharashtra vs. Sahar Ali Shaikh). The designated POCSO judge MA Baraliya had to
consider whether the act of "touching the bums" of the victim by the accused would
constitute an offence under Section 7 of the POCSO Act. Clarifying that “the term private
part is to be interpreted into the context what is meant by it in our society,” the Court held
that Google’s definition of not including bum in private part may not be an acceptable
interpretation as far as Indians are concerned. Section 7- “whoever, with sexual intent
touches the vagina, penis, anus or breast of the child …or does any other act with sexual
intent which involves physical contact without penetration is said to commit sexual assault.”
Accused and other three were laughing at her, when she been to buy bread. Second time
when she was going still, they were laughing at her. Past conduct of accused laughing at her
and then touching her manifests that it was all with sexual intention, to grab the chance. The
Court noted that the accused had not touched the vagina, breast or anus of the victim.
However, it ruled that touching the posterior of the victim cannot be said to be without sexual
intention. Sexual intention is the state of mind, May not necessarily to be proved by direct
evidence, such intention is to be inferred from the attending circumstances of the case," the
order read. While holding so, the Court sentenced the accused to five years of rigorous
imprisonment and payment of fine of Rs. 10,000. [Source- Neha Joshi, ‘Google's definition of
not including bum as 'private part' may not be acceptable in Indian context: Mumbai Court
convicts man under POCSO’, Bar and Bench]

. Minakshi and Sheetal were best friends. One day they were playing in a park and Minakshi
saw a boy playing alone. She went to him and starts playing with him. In a meanwhile, Suraj,
father of the boy, saw Sheetal playing alone. He went to her and inquired her about her
family. Taking the opportunity in favour, Suraj asked Sheetal to touch upon his private body
parts and offered her chocolates for doing so. He went on to make her feel comfortable and
grab her from back and put his fingers across her private parts without unclothing her.
Minakshi saw the incident and went to Sheetal’s mother and stated her same. FIR was
lodged against Suraj under POCSO. Decide-

(a) Suraj is liable for sexual assault of Sheetal as he had touched upon her private parts and
had undue physical contact with her, though he had no such intent to do so.

(b) Suraj is not liable for sexual assault of Sheetal as though he had touched upon her
private parts, he had not unclothed her and there was no skin-to-skin contact.

(c) Suraj is liable for sexual assault of Sheetal as he had sufficient intention to have sexual
contact with Sheetal and had touched upon her private parts is material.

(d) Suraj is not liable for sexual assault of Sheetal as though he had touched upon her
private parts, it’s not clear whether its vagina, breasts or anus and definition is still unclear.

. Payal was a young girl and pretty looking. She was the most beautiful girl in her locality.
She went to shopping with her friend Saloni and was encountered with a gang of bad boys
who were stalking them from past 10 minutes. She asked her not to follow them or else she
will call the police. Boys asked her to remain silent and let them enjoy her beauty. They all
were giggling and one of them commented that he wants to sleep one night with Payal and
other with Saloni. Payal got enraged and called up police. The boy attacked Payal and
snatched her phone and ran away. Payal went to police station and complained of the gang.
Decide-
(a) Sexual intention is to be inferred from the statements made by the boy and not only by
the physical acts or conduct. This can amount to sexual assault.

(b) Sexual intention can only be concluded by direct evidence and has to be proved in court
of law. This cannot be said to be sexual assault.

(c) Sexual intention can be inferred from circumstantial evidences and not merely by the act
or conduct of the accused. But it alone cannot amount to sexual assault.

(d) Sexual intention has to be inferred from the conduct of the boys and even though they
have it, its a case of mere stalking and not sexual assault.

. Mehul and Tamana were in relationship from long time. But their families were against each
other. They fixed Tamana’s marriage with Rahul to which Tamana agreed readily. She
asked Mehul to forget their long relationship and she cannot go against her family decisions.
Mehul got angry with Tamana and planned to take her from home and get into sexual
intercourse after which her family cannot deny their marriage. He asked Tamana to meet her
for the last time and take her to a hotel for dinner. He takes her to the bedroom and made
her unconscious. He forcefully had sexual intercourse with her and asked her to marry him.
Tamana went to her family and stated the incident. They filed a case against Mehul for
sexual assault under IPC. The defence lawyer claimed that Mehul has not committed alleged
offence as he had used protection while having sexual intercourse and there was no
physical contact between them. Decide-

(a) The defence is correct as there was no skin to skin contact between Tamana and Mehul
and the act of sexual intercourse amount to sexual assault.

(b) The defence is incorrect as there was physical contact between Tamana and Mehul
irrespective of skin to skin contact and it amounts to sexual assault.

(c) The defence is correct as though there was sexual intention and forceful sexual
intercourse, the act has to be physical which not the case here is as Mehul used the
protection.

(d) The defence is not correct as the act of Mehul of forceful sexual intercourse with Tamana
amounts to much graver offence of rape along with sexual assault.

. Shabana, a Muslim girl of 10 years old, was made through typical practices of her religion
from time long. She asked her mother about the same to which her mother responded that
Female Genital Mutilation and Nikah Halala were long set religious practices followed by
their community and thus everyone has to follow it. She claimed it to be wrong and
approached court to file complaint against Maulana who helped her family in conducting
FGM successfully. She alleged him of sexual assault as he had touched upon her private
parts without her due consent and should be punished for the same. Decide-

(a) The court will held Maulana liable for sexual assault of Shabana as he had touched upon
her private parts and hence construed physical contact with her.

(b) The court will not held Maulana liable for sexual assault of Shabana as he was only
following the customary practices of their religion and had consented her family for the
same.
(c) The court will held Maulana liable for sexual assault of Shabana as there were enough
direct evidences to prove the alleged offence and religious practices is not an exception to
the offence.

(d) The court will not held Maulana liable for sexual assault of Shabana as there was no
sexual intention and physical contact with private parts were part of religious practice.

. Section 354 of IPC and other related sections covered the offences of Sexual Assault and
Sexual Harassment prescribing the punishment with gravity of offence committed. Then
what is the need for implementing same clauses with different or graver punishments under
POCSO Act.

(a) Sexual Assault under IPC is different from POCSO and hence there was need to redefine
the same under the POCSO Act with higher punishment.

(b) Sexual Assault and Sexual Harassment offences are graver and heinous in cases of
child and to grant much protection to them, POCSO was implemented.

(c) POCSO Act is much more comprehensive in nature and gives detailed essentials of
Sexual Assault and Harassment then IPC and thus has overriding effect over it.

(d) IPC covers only limited number of cases of sexual assault and is gender neutral but
POCSO is not gender neutral and thus was need to implement it.

Passage (Q.-Q.): The incidence of income tax under the Income-tax Act, 19 is based on the
residence of the taxpayer in a previous year, which is from 01st April to 31st March and the
source(s) of his or her income. While a person "resident" in India is taxed on his or her
worldwide income, non[1]residents are only taxed on their India-sourced income. Section 6
of the ITA treats an individual to be "resident" in India in any previous year if he or she is in
India for: (a) 1 days or more in that year; or (b) days or more in that year and has been in
India for 3 days or more in the four years preceding that year. A writ petition was filed before
the Supreme Court by a UAE-based NRI who, having come to India in March 2020, was
finally able to return to the UAE only after spending upwards of 1 days in India during FY
2020-21. The petitioner therefore sought a direction to the effect that he would be
considered "non[1]resident" for Indian income tax purposes for FY 2020-21, irrespective of
the duration of his stay in India, on account of the COVID-19 pandemic. Taking note of the
fact that certain relaxations had been granted by the Central Board of Direct Tax for FY
2019-20 through the Circular, and the fact that the pandemic had continued beyond March
31, 2020, because of which many people had remained stranded in the country, the
Supreme Court directed the petitioner to make a representation to the CBDT and further
directed the CBDT to consider the same within three weeks of the receipt thereof. English
courts have held since time immemorial that residence must be "adopted voluntarily and for
settled purposes", and Indian courts have followed suit. For instance, in CIT v. Suresh
Nanda, the Delhi High Court held that where a person was compelled to stay in India
because his passport was impounded had to be excluded in determining whether he was
"resident" in India for the relevant assessment year. Delhi high court held that if the record
discloses materials that the stay (to qualify as resident Indian) lacked volition and was
compelled by external circumstances beyond the individual's control, she or he cannot be
treated as a resident Indian. Anandapadmanabhan Unnikrishnan, Tax Residency In The
Time Of COVID - What Does The Supreme Court Ruling In Gaurav Baid v. Union Of India
Mean For NRIs?
 

. Dheer was an Indian citizen, who worked at TCS, Bengaluru as a consultant for a salary of
INR 55,000 per month. He worked there from 01 April 2019 to 27 September 2019. After he
resigned, he applied for jobs at multinational companies in foreign countries for better career
prospects. On 01 October 2019 he left for the USA, where he had been invited to work as
chief project engineer for SpaceX for a salary of INR 1,20,000. He worked there from
1.10.2019 to 31.3.2020. His salary for October to December 2019 was credited in his USA
bank account and the salary for January to March 2020 was credited in his Bombay account
directly. Decide which of his income would be taxable in India for the previous year 2019-20.

(a) Since he is a citizen of India, all the income that he earns anywhere around the world
would be taxable in India as per the Income Tax Act.

(b) Since he is a non-resident only the income that he earned from TCS in the concerned
previous year would be taxable in India as per the Income Tax Act.

(c) Since he is a resident, all the income that he earns anywhere around the world would be
taxable in India as per the Income Tax Act.

(d) Regardless of his residential status, the income earned from TCS and the income from
Google that was deposited in a Bank account in India would be taxable in India as per
Income Tax Act.

. Which of the following is/are not a conclusion of the article: A. Only Indian residents are
liable to pay tax on their worldwide income, whereas non-residents have to pay taxes on
only income earned in India. B. A person shall be liable to pay taxes if they satisfy the mere
letter of the provisions and voluntariness of their stay is irrelevant in the determination of
their residential status. C. Subject to factual circumstances, a person who was compelled to
stay for external reasons shall not be liable to pay taxes as a resident of the nation. D. The
Supreme Court upheld the ruling of Delhi High Court and allowed the petition by excluding
the period of involuntary stay of the petitioner in India for the purpose of determining
residential status. Choose the correct answer:

(a) II and IV

(b) I and III

(c) I, III and IV

(d) III and IV

. Azaan was a spice trader from Afghanistan, who imports spices from India. Once, when he
was in India to close a deal, Pakistan declared a war against India. As a result, all of the
Indian borders were sealed and no person was allowed to enter or leave the country. The
war lasted for 7 months and for all this time Azaan was compelled to stay in India. After the
war ended, he prepared to go back to Afghanistan, however, he received a notice to pay
income tax on all the income he has earned in the previous year as he qualifies as the
resident of India. He has contended that he was forced to stay in the country due to the
ongoing war and did not volunteer to do so. The case is now pending before a high court.
Which of the following is the correct answer?
(a) Azaan shall be liable to pay income tax on his entire income earned worldwide as he has
satisfied the criteria of being an Indian resident.

(b) Azaan shall be liable to pay income tax on his entire income earned worldwide as the
source of all his income is the spice that he imports from India

(c) Azaan shall be liable to pay tax on only income earned in India as he is an Afghan citizen
and the Income Tax applies on only Indian citizens.

(d) Azaan shall be liable to pay income tax on only income earned in India as he has been
involuntarily staying in India and could not return to Afghanistan due to the war.

. Mitchell was a lawyer from the UK, who was in India to represent a multinational
automobile company. When the Supreme Court reserved the judgment to be given after 3
months, the lawyer decided to stay to spare the cost of travelling. The judgment was given in
the favour of the multinational company and Mitchell prepared for going back to the UK.
When he filed returns to pay tax on income earned in India, he was notified that he had
stayed in India for a period of 1 days and thus was an Indian resident for the previous year,
and thus shall be liable to pay tax on all the income earned. He appealed against the
notification before the Income Tax Appellate Tribunal on the grounds that he was compelled
to stay in India because the judgment was reserved for a later date. Choose the correct
option.

(a) Appeal shall be allowed as Mitchell did not volunteer to stay in India and was under
compulsion to save the cost of travelling, thus the period of his stay shall not be counted to
determine his residential status.

(b) Appeal shall not be allowed as Mitchell was under no compulsion to stay in India and
volunteered to stay for a specific purpose and thus is a resident of India

(c) Appeal shall be allowed as Mitchell is just representing the company in India and
payment for that case would not be income earned in India Therefore, Mitchell shall not pay
any taxes on his income in India

(d) Appeal shall not be allowed as payment received to defend a client in India amounts to
income in India which is taxable regardless of the residential status of Mitchell.

. Anas was a student from Bangladesh, completing his bachelors and masters degree from
India in 2015. In November 2021, he decided to go back to Bangladesh to start up a tech
firm of his own. His firm did very well in the very first year and he earned enough to pay off
all his education loans. In April 2022, he received a notice from the Assessing Officer of
Income Tax Department, Delhi to pay tax on all the income he earned in the previous year
2021-22. He has made representation before the authority and contended that he did not
earn a single penny in India and all his income in the relevant year has been earned in
Bangladesh only. Which of the following is the correct option?

(a) Anas shall be liable to pay income tax on all the income he earned including that in
Bangladesh as in the relevant previous year, he was a resident of India

(b) Anas shall not be liable to pay any taxes in India for the relevant previous year as he did
not earn any income in India even if he was a resident in that previous year.
(c) Anas shall be liable to pay income tax on only the income that has been earned in India,
if any, in the relevant previous year.

(d) Anas shall not be liable to pay any taxes in India as he is a non-resident and he did not
earn any income in India in the concerned previous year

25
Passage (Q.-Q.): Res ipsa loquitur is a Latin phrase that means “the thing speaks for itself.” It
is considered to be a type of circumstantial evidence which permits the court to determine
that the negligence of the defendant led to an unusual event that subsequently caused injury
to the plaintiff. Although generally the duty to prove that the defendant acted negligently lies
upon the plaintiff but through res ipsa loquitur, if the plaintiff presents certain circumstantial
facts, it becomes the burden of the defendant to prove that he was not negligent. This doctrine
arose out of the case of Byrne vs Boadle (18). The plaintiff was walking by a warehouse on
the road and suffered injuries from a falling barrel of flour which rolled out of a window from
the second floor. At the trial, the plaintiff’s attorney argued that the facts spoke for
themselves and demonstrated the warehouse’s negligence since no other explanation could
account for the cause of the plaintiff’s injuries. Thus, the following are the three essential
requirements for the application of this maxim[1]1) The thing causing the damage must be
under the control of the defendant or his servants 2) The accident must be such as would not
have happened in the ordinary course of things without negligence. 3) There must be no
evidence of the actual cause of the accident. Source- Law Times Journal

. Shekhar, a hosteler in Sheerwood School was sleeping in his room when ceiling fan of his
room fell on him, causing minor injury to him. The reason for the falling of the fan was a
latent defect in the tensile strength of the suspension rod along with poor fixing of the fan. In
an action against the Sheerwood School, Sheerwood School is:

(a) Liable because it was a hostel maintained by Sheerwood School and he was rightfully
living there.

(b) Not liable because injury was not foreseeable. Rather the manufacturer should be held
liable.

(c) Liable because of res ipsa loquitur doctrine will be applicable in the present case.

(d) Liable because the injury was foreseeable, but liability will that be of the manufacturer.

. Sureeli is a housewife and in her free time, she prepares pickles. In monsoon, she thinks it
fit to keep the bottles filled with pickles on the iron railing of her home’s terrace so that they
get some sunlight and don’t get spoilt. Sureeli instructs her maid Bimla to remove the bottles
in an hour but Bimla forgets to do the same. While Chandragupt was passing by the road in
the evening, suddenly one of the bottles fell from the railing on his head severely injuring his
skull. Decide who will be responsible for the damage caused to Chandragupt.
(a) It is a case of contributory negligence by Chandragupt since he should have seen the
obvious danger beforehand.

(b) Since Bimla forgot to remove the bottles, she will be liable since her act of forgetting was
not under the course of employment.

(c) Sureeli shall be liable since she was negligent in not foreseeing that the bottles can fall
down and injure someone.

(d) Sureeli shall not be liable since she had taken due care in instructing Bimla to remove the
bottles.

. Ashok is living as a tenant on the ground floor of Manoj's house and Manoj himself lives on
the first floor. One day Manoj's brother, who is quite hefty, leans against a wall in Manoj's
house. The wall conies crashing down and falls on Ashok, who is standing on the ground
floor. Ashok sues Manoj.

(a) There is no negligence by Manoj. It was his brother who caused the wall to crash. (b)
Manoj is liable only if Ashok can show how Manoj has been negligent. (c) Manoj is liable.
Doctrine of Res ipsa loquitur applies.

(d) Manoj is not liable because it was an act of god.

. Manorama was a happily married women with two children at her home. After two children
she wanted to have tubectomy. One day she went to Doctor Patchy to get herself operated for
the removal of her uterus. Doctor Patchy performed the successful operation. After a couple
of days Manorama started to feel pain in her stomach. She took medicine, but it didn’t cure.
Later she went to another surgeon and got herself ultra sound. In the ultra sound, an
abdominal pack was located in her abdomen, which was a resultant of patchy work done by
Doctor Patchy during the operation. In this case-

(a) Doctor Patchy cannot be held responsible because it is merely a human error.

(b) Doctor Patchy cannot be held responsible because during operations, such risks are
involved.

(c) Doctor Patchy can be held responsible but Manorama will have to prove in the court of
law that the Doctor was grossly negligent.

(d) Doctor Patchy will be responsible and Manorama need not to prove doctor’s negligence
because presence of abdominal pack in her abdomen is sufficient proof therefor.

Passage (Q.-Q.): The state shall not deny to any person equality before the law or the equal
protection of the laws within the territory of India. Prof. Dicey, explaining the concept of
legal equality as it operated in England, said: “with us every official, from the Prime Minister
down to a constable or a collector of taxes, is under the same responsibility for every act done
without any legal justification as any other citizen.” The phase “equality to the law” find a
place in all written constitutions that guarantees fundamental rights. “All citizens irrespective
of birth, religion, sex, or race are equal before law; that is to say, there shall not be any
arbitrary discrimination between one citizen or class of citizens and another.” “All citizens
shall, as human persons be held equal before law.” “All inhabitants of the republic are
assured equality before the laws.” PantanjaliSastri, has expressed that the second expression
is corollary of the first and it is difficult to imaging a situation in which the violation of laws
will not be the violation of equality before laws thus, in substance the two expressions mean
one and same thing. According to Dr. Jennings said that: “Equality before the law means that
equality among equals, the law should be equal for all. And should be equally administered,
that like should treated alike. The right to sue and be sued, to prosecute and prosecuted for
the same kind of action should be same for all citizens of full age and understanding without
distinctions of race, religion, wealth, social status or political influence.” “Equal protection of
law” has been given in Article 14 of our Indian constitution which has been taken from
section 1 of the 14th amendment act of the constitution of the United States. Source: Legal
Services India

. The state government of Uttarakhand invites tender for wooden coffee mugs for official use
in its government offices and rest houses. Many manufacturers send in their bids. The terms
of the invitation for tenders was that the bidders quoting the lowest bid will get the contract.
Kumaon Cups Ltd. Who applied for the binding found that they had quoted the lowest rates
and ordinarily the bid would have been awarded to Kumaon Cups Ltd. but due lobbying it
was awarded to Hill Woods Ltd, (a company run by local politician belonging to Hindu
community).Kumaon Cups Ltd., a company run by Christian minority has challenged the
tender process in the High Court of Judicature of Uttarakhand. Decide the case.

(a) Kumaon Cups Ltd will succeed in their claim as their bid was an acceptance of the offer
of the State of Uttarakhand

(b) Kumaon Cups Ltd will succeed in their claim as theirs was the best quotation in the
bidding and they were discriminately rejected on the basis of religion.

(c) Kumaon Cups Ltd will not succeed in their claim as their bid was an invitation to offer for
supply of stationery items

(d) Kumaon Cups Ltd will not succeed in their claim as their bid was an offer for the supply
of stationery items  

. With 2020 annual budget, the Income Tax Act revised its rule providing that people with
annual income up to Rs. 5 lakhs per annum shall pay 10% of their income as tax and those
above Rs. 5 lakhs per annum to below Rs 10 lakhs per annum shall pay 20% of their income
as tax. Ramesh who got promoted last year and draws the salary of Rs. 5.2 lakhs per annum
challenge the legislation on the ground that it is in violation of the principle of equality before
law.

(a) Ramesh will succeed, because the law discriminates against the people who earn more
than Rs. 5 lakhs per annum.

(b) Ramesh will not succeed, because the people who earn more than Rs. 5 lakhs per annum
are not equal to the people who earn less than Rs. 5 lakhs per annum.
(c) Ramesh will not succeed, because this law enables the Government to equalise the
incomes of all the people in the country.

(d) Ramesh will succeed because as per the constitution everyone shall be treated equally.

. Jasmine is an employee of Extreme Fair Traders and is a part of marketing team along with
four more male employees. Jasmine along with others worked for same number of hours and
even achieved more customer count as compared to her male counterparts. At the end of the
month when remunerations were paid, Jasmine noticed that she received a salary of Rs.
50,000 whereas her male counterparts received Rs. ,000. When she brought up the matter to
the authorities, she was told that Extreme Fair Traders have believe that men run the family
therefore they are paid little more money as compared to the women employees. Jasmine
seek to bring action against the unfairness done by the Extreme Fair Traders. Help.

(a) Jasmine is discriminated on the basis of sex and is entitled to the same salary as her male
colleagues

(b) Jasmine cannot claim the same salary as her male colleagues as the salary has been fixed
as per the division of work in the marketing team

(c) Jasmine cannot claim the same salary as her male colleagues as she has just been recruited

(d) Jasmine can claim equal pay as her male colleagues for she is a woman and more
qualified than the others

. Anandi, a woman and Bismil, a man were Professors at a State-run University. There was a
stark difference between the salaries of Anandi and Bismil where Bismil earned more than
Anandi in spite of holding the same position in the University. A few years later, Anandi got
married. When the University administration came to know about the same, they terminated
Anandi’s services on the grounds that since Anandi is married now, she would move to a
different city to be with her husband. On the other hand, when Bismil got married, he
continued living on the campus and teaching at the University. Anandi filed a suit for
wrongful termination and gender discrimination. Was Anandi treated equal to her male
counterpart at the University?

(a) Yes. Anandi was treated equal to her male counterpart at the University as they were
holding the same post.

(b) No. Anandi was not treated equally as she was not getting the same remuneration as
Bismil at the University.

(c) Yes. Anandi was treated equally as she was getting the same facilities there as Bismil.

(d) No. Anandi was not treated equally as she was not given the same treatment as Bismil at
the University.

 
. Sunny applied for admission in a minority institution, which is governed by the government
fund. College administration denied admission to him on the ground that it is a minority
institution and only student from a particular religion is admitted in this institution. Sunny
filed writ petition in the Supreme Court challenging the decision of college administration as
violative of his fundamental right. Decide

(a) Sunny will not succeed as the institution is private.

(b) Sunny will succeed as it is discrimination on the basis of religion.

(c) Sunny will succeed as it is government institution run out of the government fund.

(d) Sunny will not succeed as the state has a margin of appreciation to frame laws.

. Anahita works in a government company. She works as executive engineer. Office timings
for female employees in the company are 10 am to 6 pm, for the male employees they are
from 10 am to 7 pm. The timings for the female employees is less because the company is
situated in the outskirts of the city so for the safety of the women the timings are different
plus the work profile of male and female employees are different. Anahita challenged this
disparity in Supreme Court as it is violative of her fundamental right.

(a) There is no violation of fundamental rights as timing for the male and female employees
are different.

(b) There is violation of fundamental rights.

(c) There is violation of fundamental rights as there no sufficient cause to give less salary to
women.

(d) Supreme Court has no authority to decide the matter because it is authority of the state to
make law on this issue.

Passage (Q.-Q.): A question that has vexed countless non-resident Indians (NRIs) who were
forced to remain in India on account of international travel restrictions introduced to combat
the spread of COVID-19 is: will they be considered "tax resident" in India for Indian income-
tax purposes? The incidence of income tax under the Income-tax Act, 19 ("ITA") is based on
the residence of the taxpayer and the sources of his or her income.While a person "resident"
in India is taxed on his or her worldwide income, non-residents are only taxed on their India-
sourced income. Section 6 of the ITA treats an individual to be "resident" in India in any
previous year if he or she is in India for: (a)1 days or more in that year; or (b) days or more in
that year and has been in India for 3 days or more in the four years preceding that year. It was
anticipated that an obligated stay due to the lockdown might unintentionally render NRIs,
while on visit to India during FY 2019-20, were forced to remain in India due to the
imposition the lockdown and the accompanying suspension of international flights, "resident"
in India and therefore subject their worldwide income to Indian taxation. To allay these
concerns, the CBDT issued a Circular clarifying that the period from March 22, 2020 to
March 31, 2020 would be excluded in determining the residential status of an individual
under Section 6of for FY 2019-20. Additionally, it was stated that as the lockdown had
continued into FYFY 2020-21, a further circular excluding the period of stay of individuals
stranded in India up to the date of normalisation of international flight operations, for
determining the residential status during FY 2020-21, would be issued after such
normalisation. [Extracted from ‘Tax Residency in The Time of COVID - What Does the
Supreme Court Ruling in Gaurav Baid v. Union of India Mean for NRIs?’ by
Anandapadmanabhan Unnikrishnan on 1 Mar 2021 on livelaw
https://www.livelaw.in/columns/covid-19-tax-residency-supreme-court-gaurav-baid-v-union-
of-india[1]1505]

. Mr. Worldwide is a businessman. He is involved in the trade of international export and


import of clothes in different parts of the world and frequently moves back and forth between
the United States and India. In 2019, his income from his business was geographically
divided as follows: Miami 2 million EUR, New York 10 million EUR, New Jersey 5 million
EUR and New Delhi 7 million EUR. He spent 1 days in the United States in 2019. What
portion of his income will not be taxable under the Income Tax Act, 19?

(a) 15 million EUR from New York and New Jersey

(b) 7 million EUR from New Delhi

(c) 24 million EUR from Miami, New York, New Jersey and New Delhi

(d) 17 million EUR from New York, New Jersey and Miami

. Suresh is a Non-resident Indian. He came to India on 1st January 2019 and left on 1st
September 2019. His net salary during that period was Rs 1 crore. On 1st April 2020, he
received a notice for non-payment of income tax during the Financial Year of 2019 from the
Indian Income Tax Department. Suresh believes that this notice is erroneous and incorrect
because he is not liable to pay income tax in India for that Financial Year as he does not
qualify as a resident for the purposes of Section 6 of the Income Tax Act. Is he correct in his
assessment of the situation?  

(a) No, because Suresh qualifies as a resident of India for the Financial Year of 2019 as he
spent days of that year in India and had been in India for 3 days in the four years preceding
that year

(b) No, because Suresh qualifies as a resident of India for the Financial Year of 2019 as he
spent more than 1 days in India during that time

(c) Yes, because Suresh does not qualify as a resident of India for the Financial Year of 2019
as he spent less than 1 days in India during that time
(d) Yes, because Suresh does not qualify as a resident of India for the Financial Year of 2019
as he did not spend days of that year in India and had not been in India for 3 days in the four
years preceding that year

. Suresh's brother Rakesh also accompanied him to India. However, unlike Suresh, Rakesh
left India on 1st February 2019. He again came to India on 15th January 2020. He was
scheduled to leave in March but was stuck due to the worsening situation of the pandemic.
Rakesh is worried that he may be forced to pay income tax in India, even though he
ordinarily resides in Antigua and Barbados. Is Rakesh's income taxable for the Financial Year
of 2020?

(a) Yes, because as the lockdown had continued into FY2020-21, he became a resident of
India for the purposes of income tax

(b) Yes, because the lockdown and extended lockdown period of stay of a non-resident
Indian is excluded from the determination of the residential status of an individual

(c) No, because the lockdown and extended lockdown period of stay of a non-resident Indian
is excluded from the determination of the residential status of an individual

(d) No, because even though the lockdown was extended to FY2020-21, the same was not
excluded from the period of stay of non-resident Indians

. Mohini is a famous dancer in Dubai who teaches dance to students at her academy as well
as through online classes. Although she is originally from India, she has been residing in
Dubai for the last 10 years. She came to India on 1st December 2019 to visit her family and
friends. In March 2020, a nationwide lockdown was imposed and international travel was
banned. If international flight operations were normalized on 15 May 2020, and Mohini left
for Dubai on 3rd December 2020, will her income be taxable for this period?

(a) No, because she is not an Indian resident and the worldwide income of non-resident
Indians is not taxable

(b) No, because the lockdown period in March as well as extended lockdown till
normalization of flight operations is included from the calculation of residence period for
income tax purposes

(c) Yes, because Mohini spent 1+ days in India even though flight operations were
normalised on 15 May 2020

(d) Yes, because Mohini spent 1+ days in India excluding the period of lockdown in March
2020 as well as the extended lockdown later on in the year

Passage (Q.-Q.): The Election Commission (EC) has opined in favour of 12 MLAs from
Manipur, who were facing disqualification in a 2018 ‘office of profit’ case for holding the
position of parliamentary secretaries. While the law relevant to office of profit bars MPs and
MLAs from holding a position under the government, the MLAs did not seem to commit any
violation as they had held the posts of parliamentary secretaries in the state under an
exemption granted by two laws passed over the last decade. Both Acts ceased to exist after
the Manipur High Court pronounced them invalid and unconstitutional in a judgment on 17
September 2020. Subsequently, a disqualification of these MLAs was sought on account of
holding the position of parliamentary secretaries, which qualified as ‘office of profit’. The
opinion of the EC was sought which is said to have opined that since the Acts mentioned
above were in force at the time they held the office, the MLAs could not be disqualified for
holding office of profit retrospectively. The EC is learnt to have based its view on Article
1(1) of the Constitution. The Article says a person can be disqualified as MLA “if he holds
any office of profit under the Government of India or the Government of any State specified
in the First Schedule, other than an office declared by the Legislature of the State by law not
to disqualify its holder”.  21 of 36 The case for the disqualification of the Manipur MLAs had
made a reference to the Supreme Court order in the case of Bimalangshu Roy v. State of
Assam, according to which appointing elected representatives of the land as parliamentary
secretaries violates the law. However, the Manipur MLAs case is different since there was a
protection granted to them in law earlier, which was subsequently declared unconstitutional.
[Extracted with minor revision from ‘EC sees “no ground for disqualification” of 12 Manipur
BJP MLAs in office of profit case’ by Sanya Dhingra, published 2 Mar 2021 in theprint
https://theprint.in/india/ec-sees-no-case-for-disqualification-against-12-manipur-bjp-mlas-
who-held-office[1]of-profit/43/]

. Wakanda announced a new law to regulate the conduct and salaries of its MLAs in the
legislative assembly of Wakanda. Anju and Panju were two representatives who were elected
from the constituencies of Red and Yellow. Both Anju and Panju had previously worked in
the capacity of an assistant to the earlier MLAs from these constituencies. Now, those earlier
MLAs were working in a government-run policy think tank. Anju and Panju felt that they still
had a lot to learn so in their spare time, they continued to assist the earlier MLAs in their
research work for the think tank and received a nominal stipend for the same from the think
tank itself. Are they risking disqualification by such conduct?

(a) No, because the Wakandan law to “regulate the conduct and salaries of its MLAs in the
legislative assembly of Wakanda” was in operation which explicitly does not prevent them
from working in the think tank

(b) No, because merely assisting the earlier MLAs in exchange for a nominal stipend from
the government think tank does not amount to “office of profit”

(c) Yes, because they are violating Article 1(1) by assisting the other two MLAs for their
research work in the government think tank

(d) None of the above

. The law of Wakanda which regulated the conduct and salaries of its MLAs was passed in
2019 and subsequently repealed in January 2020 after an order of the Wakandan High Court.
The law provided for an explicit exemption clause which allowed MLAs to assist research
think tanks in exchange for a nominal amount in order to help MLAs gain policymaking
experience. Anju and Panju started working in the capacity of a secretary to the earlier MLAs
for the government think tank in December 2018. Are they liable to be disqualified from their
position as MLAs?

(a) No, because Anju and Panju did not commit any violation as they had held the posts of
secretaries under an exemption granted by the Wakandan law

(b) Yes, because the Wakandan law in question was repealed in January 2020 so Anju and
Panju can be disqualified on account of holding the position of secretaries, which qualifies as
‘office of profit’

(c) No, because Anju and Panju cannot be disqualified from their positions on account of a
retrospective application of law

(d) None of the above

. Jonty Rhodes was a proficient lawyer and handled all the high profile cases in the state.
Eventually, he also contested as a candidate to represent his constituency in the Lok Sabha.
Jonty won the election by a thumping majority. However, law was still his first love so he
continued his practice as a lawyer in the Supreme Court. Katari is concerned that Jonty may
be disqualified on account of holding an “office of profit”. Is his concern valid?

(a) Yes, because Jonty is holding an office of profit by continuing his practice as a lawyer in
the Supreme Court while also being an MLA

(b) No, because continuing his practice does not amount to a violation of Article 1(1) of the
Constitution

(c) Yes, because he is in violation of the rules laid down by the Election Commission as well
as judicial pronouncements

(d) No, because he is not in violation of the judicial principle laid down in the case of
Bimalangshu Roy v. State of Assam  

. Jonty wanted to serve the state in a higher capacity than just as an MLA or a lawyer. He had
a passion for putting criminals behind bars and make the society free of miscreants and
nuisance. So, he decided to temporarily suspend his personal practice and become a public
prosecutor for the state. He believes that this jump will help him fulfil his passion as well as
work for the society’s welfare. Before finalising this switch, he asks your opinion on the
same. Would you advise to leave his personal practice and become a public prosecutor for the
state while he continues to be an MLA?

(a) Yes, because this move will help him realise his passion as well as work further for
society’s welfare and it also doesn’t threaten his position as an MLA rather it complements
the same

(b) Yes, because a becoming a public prosecutor does not amount to an office of profit under
the government which could raise disqualification concerns
(c) No, because if he becomes a public prosecutor he may be disqualified for violating Article
1(1) of the Constitution

(d) No, because an MLA cannot become a public prosecutor due to the rules provided by the
Election Commission as well as the Constitution and this threatens him with disqualification

Passage (Q.-Q.): In one of its most recent judgments, in the case of Padia Timber Company
(P) Ltd. Vs The Board of Trustees of Visakhapatnam Port Trust Through its Secretary, the
Hon'ble Supreme Court considered the issue of regarding formation of the contract between
the parties, which was the bone of contention between the parties. The decision of the Apex
Court in the above judgment is based on simply following the first principles of law of
contract. Parties must agree on the same thing, in same sense is the sin-qua-non for formation
of a valid contract. Without consensus ad-idem no valid contract can be formed. Similarly,
for examining at whether and when a valid contract was formed recourse should be taken to
the principles of 'offer', 'acceptance' and 'revocation of acceptance' as mentioned in Chapter 1
of the Contract Act. Parties by agreement cannot deviate from the said principles, and yet
contend that valid contract was formed. The offer, acceptance, revocation etc should be well
communicated to all the concerned parties. It is relevant to refer to saving to Section 1 of the
Contract Act which reads as under: "Saving. - Nothing herein contained shall affect the
provisions of any statute, Act or regulation not hereby expressly repealed, nor any usage or
custom of trade, nor any incident of any contract, not inconsistent with the provisions of this
Act." Therefore, only such contracts which were not inconsistent with the provision of the
Contract Act were saved at the time when Contract Act was brought on statute book.
Accordingly, it can be contended that all contracts must comply strictly with the principles
enunciated in the Contract Act, to be adjudged as valid. Source: Live Law

. Anandi was a widow who lived in a huge kothi outside the city. She offered her neice
Brinda to buy her estate. Brinda said that if Anandi did not hear from her for the next 7 days
then the offer on the estate would be deemed to have been accepted by Brinda. Anandi did
not agree to this. In the course of those 7 days, Anandi’s manager Chandu, mistakenly sold
the estate to Dr.Oswal. Can Brinda bring a suit for breach of contract against Anandi?

(a) Yes. Since Anandi did not hear from Brinda in the course of 7 days, the offer was deemed
to have been accepted by Brinda.

(b) No. Since Anandi did not agree to silent acceptance, the silence of Brinda for the course
of 7 days is not considered as a valid acceptance.

(c) Yes. Anandi relied upon Brinda’s acceptance and set aside the estate for her.

(d) No. There was no acceptance as in the course of those 7 days, the estate was already sold
to Dr.Oswal.  

. Ross and Chandler have been friends for the last 10 years. Ross has a sister named Monica
who is very intelligent but is suffering from lung cancer. Monica is fond of animals and
wants a dog for herself. She is also a tech-savy. Chandler has a female dog which recently
gave births to two puppies named Bentley and Mercedes. Three days back Ross promised
Monica to buy her Bentley if she gives her car Porsche (the car she got from her parents) to
Ross. Monica thought that he is talking about the puppy named Bentley. Ross was talking
about a car named Bentley. As Monica only had Ross and no one else to take care of her, she
gave in and sign the contract. Is the contract valid?

(a) The contract is valid as the object was lawful and so is the consideration.

(b) The contract is not valid as it lacked consensus ad idem as Ross and Monica did not agree
on the same thing when they entered into the contract.

(c) The contract is not valid as Monica is a strong independent woman and she doesn’t need
Ross to buy her things. Also, there was undue influence done by Ross.

(d) The contract is not valid as Chandler’s consent was not taken.

. Mamta read Sonu’s mail and replied saying: “I humbly decline your offer.” The mail
automatically went to the ‘Trash’ folder in Sonu’s email account. On 02.01.2021, Sonu’s
consignment reached Mamta’sgodown in a truck which bore the name “Sonu Fresh Fruits Pvt
Ltd”. Mamta mistook the consignment to have come from Katty, her long-time trading
partner. She received the consignment but refused to make payment.

(a) There was no option of revoking through email. The only option of revocation was to
refuse to receive the consignment. Since, consignment has been received payment must be
made.

(b) The revocation is valid since it has been put in the course of the transmission as to be out
of the control of the person making the revocation.

(c) The revocation is not valid since it has not come to the knowledge of Sonu.

(d) Since Mamta received the consignment her revocation cannot be considered valid.

. Bank of Bhubaneshwar, in its council meeting, decided to appoint Sharma as its Bank
Director. Decision was made followed by an application for the post was received from
Sharma by the Bank. Before the Council could communicate to Sharma, later was become
aware of the Council decision by Suresh, a servant at Bank of India. Later, Council decided
to revoke the appointment. Sharma challenged the revocation on the ground that proposal has
already been received by him. Will he succeed?

(a) Sharma will not succeed since there was no communication of acceptance of his job
proposal.

(b) Sharma will not succeed since there was no official confirmation made by the bank to
Sharma.

(c) Sharma will succeed since decision was communicated as soon as servant informed
Sharma.
(d) Sharma will succeed since decision was communicated as soon as Council made the
decision.

. Rahul promises to his two children that he will purchase for them a ‘Seltos’ car in case they
stand first in the ensuing examinations. Both the children stood second and third in their
respective classes. However, they claim that the examiner had some grudge against them and
mendaciously deducted their marks. Rahul refused to purchase the cars as promised by him.
Can the children sue Rahul in the court of law for breach of contract?

(a) Yes, as there was offer by Rahul and acceptance by the children by doing the required act.

(b) No, the condition for a valid contract are not met and therefore children will not succeed
in their suit.

(c) Yes, as this was a case of oral contract and such contracts are honored in the court of law.

(d) Both (a) and (c)

. Sameer the owner of a dog, announces a reward to anyone who will bring back his lost dog,
Rocky. Abhishek, a friend of Sameer, gives information which leads to recovery of the lost
dog from Mr. Jain, who had no knowledge about the reward. Later when Abhishek comes to
know about the announcement of the reward by Sameer, claims the reward. Decide.

(a) No, as Abhishek had no knowledge of the offer before performing the required task.

(b) Yes, as by performing the condition of the proposal, he gave acceptance to the general
offer made by Sameer.

(c) No, as Abhishek was only doing his duty as a friend.

(d) Yes, as Sameer has a duty to reimburse Abhishek for helping him find his dog.

Passage (Q.-Q.): Sections 3 to 402 deal exclusively to offences like robbery and dacoity.
These offences are basically aggravated and more serious forms of simple theft. This is
because they involve physical harm to the victim. In order to prevent such offences, the law
provides for severe punishment. The interpretation of the law is always done in a manner that
serves the purpose of the existence of the law. According to Section 3, there is either theft or
extortion in every robbery. In other words, the offence of robbery is an aggravated form of
either: (1) theft, or (2) extortion. (1) When theft is robbery: Theft, in simple words, means
taking a movable property out a person’s lawful possession without his consent. Robbery
generally takes place when, while committing theft or moving the property subject of theft,
the offender: (a) voluntarily causes or attempts to cause death/hurt/wrongful restraint to any
person, or (b) voluntarily causes or attempts to cause fear of instant death/hurt/wrongful
restraint. In other words, theft becomes robbery when the offender commits any of the above
two acts while committing theft. It can also happen when he completes the theft and tries to
move the property away from lawful owner possession. (2) When extortion is robbery:
Extortion basically means obtaining some kind of property from a person by instilling a fear
of injury. Extortion becomes robbery when the offender is in the presence of the victim while
committing it. The offender must also put the fear of instant death/hurt/wrongful restraint in
the victim or another person. Furthermore, the victim of this extortion must deliver the
extorted property to the offender. If all of these conditions exist, the offence of extortion
becomes a robbery. The difference between robbery and dacoity simply depends upon the
number of offenders involved. When five or more people conjointly commit or attempt
robbery, their offence becomes dacoity. Similarly, even if the number of people attempting or
abetting robbery is five or more, they are guilty of dacoity.

. Gabbar is a famous daaku. He once went to Thakur's home to take away his new jewellery.
He hit Thakur's servant and took his gold ring with him. The ring however fell from Gabbar's
pocket in Thakur's yard. Decide.

(a) Gabbar is liable for robbery for he came and moved Thakur's property and committed hurt
in the process.

(b) Gabbar is not liable for robbery as he hurt Thakur's servant and not Thakur himself.

(c) Gabbar is liable for robbery as he is a famous daku and his actions are always criminal

(d) Gabbar is not liable for he never committed theft in the first place as property was still in
Thakur's possession.

. Gabbar came in to rob Thakur of his jewellery. After he robbed Thakur of 15 gms of gold,
he ran away. Thakur chased him. Gabbar got irritated and while Thakur was chasing him, hit
Thakur on the head with the back of his gun.

(a) Gabbar is liable for robbery as he voluntarily hurt Thakur during the process of theft.

(b) Gabbar is not liable as he hurt Thakur after the process was complete.

(c) Gabbar is liable as the process of theft was not complete till the actual hitting happened.

(d) None of the above

. Gabbar kidnapped Thakur's child to extort money out of him. He called Thakur to ask him
to deliver money. During the call, he video call the Thakur to show that his child was with
Gabbar. Decide

(a) Gabbar is liable for robbery as he was in the presence of the victim at the time of extortion

(b) Gabbar is not liable for robbery as presence over video call is not sufficient presence to
cause robbery
(c) Gabbar is liable for robbery for he is an evil man and always makes money by crooked
manner

(d) None of the above  

. Thakur was scared for his child's life who had been abducted by Gabbar. He decided to pay
via Bitcoins for the ransom and transferred some Bitcoins in Gabbar's name on 24/02/2010.
Gabbar couldn't use bitcoins so changed the same into rupees on 26/02/2010 which were
credited into his account on 28/02/2010. On what date is extortion completed?

(a) The extortion is complete on 26/02/2010

(b) The extortion is complete on 24/02/2010

(c) The extortion is complete on 28/02/2010

(d) Extortion has not been committed

. Gabbar was to receive ransom money in Bitcoins. Thakur transferred the same on
24/02/2010. However, Gabbar returned the Bitcoins and threatened to send cash instead on
26/02/2010. Thakur transferred the cash on 28/02/2010. Decide

(a) The extortion is complete on 26/02/2010

(b) The extortion is complete on 28/02/2010

(c) The extortion is complete on 24/02/2010

(d) None of the above

. Gabbar had threatened to take away Thakur's jewelry. He did take away his ring but
dropped it on the way. X found the ring and took it to his place. On the way, he also hit
Thakur's child. Decide

(a) X is liable for robbery as he moved the property and also caused hurt

(b) X is not liable for robbery as the act of causing hurt is an independent act and doesn't have
to do with stealing

(c) X is liable as he moved the property of theft from one place to another

(d) None of the above

Passage (Q.-Q.): Contributory negligence means that when the immediate cause of the
damage is the negligence of the plaintiff himself, the plaintiff cannot sue the defendant for
damages and the defendant can use it as a defense. This is because the plaintiff in such a case
is considered to be the author of his own wrong. It is based on the maxim volenti non fit
iniuria which states that if someone willingly places themselves in a position which might
result in harm, they are not entitled to claim for damages caused by such harm. The plaintiff
is not entitled to recover from the defendant if it is proved that

1) The plaintiff by the exercise of ordinary care could have avoided the consequence of the
defendant’s negligence.

2) The defendant could not have avoided the consequence of the plaintiff’s negligence by an
exercise of ordinary care

3) There has been as much want of reasonable care on the plaintiffs part as on the defendants
part and the former cannot sue the latter for the same. The burden of proving contributory
negligence rests on the defendant in the first instance and in the absence of such evidence, the
plaintiff is not bound to prove its non-existence In the case of Shelton Vs L & W Railway
(16), while the plaintiff was crossing a railway line, a servant of the railway company who
was in charge of crossing shouted a warning to him. Due to the plaintiff being deaf, he was
unable to hear the warning and was consequently injured. The court held that this amounted
to contributory negligence by him. Source: Law Times Journal

. Evelyn had a farmhouse in the outskirts of the city of Delhi. Since he enjoyed spending his
weekends writing at the farmhouse, he decided to employ a servant Roshan to take care of the
house on a regular basis. A few months later, Evelyn adopted a ferocious dog named Tiger
and asked Roshan to care for Tiger as well. A few days after bringing in the dog to the
farmhouse, Evelyn asked Roshan to install a notice on the gate which read: Beware of the
dog. However, the notice fell off from the gate to the ground due to the lack of proper
installation. Despite Evelyn’s reprimands, Roshan did not get around to installing the notice
again. Meanwhile, a young couple who was passing by the farmhouse decided to enter inside
and steal some mangoes for the journey. Unaware of the dog, both the husband and the wife 
climbed through the fence and went inside, only to be attacked seconds later by Tiger, the
dog. The couple later sued Evelyn for negligence. Is Evelyn liable?

(a) Yes, since Evelyn is responsible for the wrongful actions of his servant Roshan, which in
this case, was the negligence to install a proper notice.

(b) Yes, as the owner of the property, Evelyn was responsible for putting up a notice to warn
outsiders of the dangerous dog in the farmhouse.

(c) No, since Evelyn has the right to use all means necessary to protect his farmhouse.

(d) No, since this is a case of contributory negligence and Evelyn is only partially responsible
for the harm caused to the couple.

. The claimant, Dhram and Veer were friends. After going out for an evening and few drinks
down, Veer gave lift to Dhram on his new motor bike. Veer had no expertise in driving a bike
nor license to drive. This was very much known to Dhram ye he agreed to take a lift from his
friend. They met with a brutal accident due to high speed driven by Veer. The eye witnesses
testified for the high speed and negligent driving by Veer. The accident had a brutal result as
Veer died on the spot and Dhram suffered multiple grave injuries. In an action brought by
Dhram against the representative of Veer, decide.

(a) The defence of Volenti non fit injuria may be raised as Dhram was aware of the risk
involved in sitting behind Veer in that intoxicated state.

(b) The defence of ex turpi causa non orituractio can be raised as Dhram was aware of the
illegality in Veer driving the bike at age 16 and being in an intoxicated state while doing so.

(c) The defence of contributory negligence may be raised as Dhram was also intoxicated and
hence contributed to his injuries.

(d) Both (a) and b)

. Iyer and Tiwari were neighbors. Iyer had a big Labrador named Nayyar. Tiwari hated dogs.
But Tiwari's daughter Jaimal enjoys playing with Nayyar. Every evening she generally
brought Nayyar over to their garden to play with him. One evening when Nayyar was playing
in Tiwari's garden he went and chewed up Tiwari's prized roses. Tiwari wants to sue Iyer.

(a) Iyer can be held liable because as the owner of the animal, he can be held responsible for
all the wrongful acts of her dangerous animal.

(b) Iyer cannot be held liable because dogs are not inherently dangerous animals.

(c) Iyer cannot be held liable because Nayyar was brought on to Tiwari's premises by her
daughter

(d) Iyer cannot be held liable because the plaintiff has not come with clean hands.

. Chandu carelessly left a pole protruding across a public road. Mukesh, riding a motorcycle,
saw the pole but, since he was driving at a speed substantially above the posted limit, he
collided with the pole and was injured. In an action by Mukesh against Chandu.

(a) Mukesh will win because if it had not been for Chandu’s carelessness, Mukesh would not
have been injured.

(b) Mukesh will win because Chandu had an opportunity to prevent the injury by putting up a
warning.

(c) Mukesh will lose because he was already breaking the law by driving too fast.

(d) Mukesh will lose because if he had not been speeding, he would not have been injured.  

 
Passage (Q.-Q.): The offence of criminal conspiracy is defined under Section 120-A of
Chapter V-A of the Indian Penal Code, 18. According to Section 120-A, when two or more
persons come together and agree to do, or cause something to be done, which constitutes an
illegal act or a legal act carried forward by illegal means, such persons would be guilty of the
commission of the offence of criminal conspiracy. In simpler terms, conspiracy refers to the
meeting of minds for the commission of an offence. However, no such agreement would
constitute the offence of criminal conspiracy, unless and until an act is performed in
furtherance of such an agreement. The explanation attached to Section 120-A makes it clear
that it is immaterial whether the illegal act committed in furtherance of such an agreement, is
the focal point of the agreement or, is merely incidental to the performance of the ultimate
goal of the agreement. For the establishment of the offence of criminal conspiracy, the
existence of an agreement between the parties is a sine qua non. This agreement may be
express or implied, the important factor is consensus ad idem, i.e., meeting of minds. The
agreement must be read as a whole and the object ascertained. It is not necessary that more
than one person must always be convicted for the offence of criminal conspiracy, it is
sufficient if the court is convinced that more two or persons were actually involved in the
conspiracy. The offender might join the conspiracy from the start itself or at any time before
the completion of the objective of the agreement, irrespective of the time of joining, each
party to the offence would be held equally responsible. A criminal conspiracy is said to
persist as long as the parties to the agreement continue to act in furtherance of the objects of
the agreement. [Source: https://lexlife.in/2020/06/04/criminal-law-concept-of-criminal-
conspiracy/ ]

. A and B decided to rob a house of Mr. D. In the process of doing so, A killed Mr. D’s son.
Is there a criminal conspiracy under Section 120A?

(a) Yes, both A and B are part of the criminal conspiracy.

(b) No, only A is responsible for the criminal conspiracy as there was no agreement between
A and B for the murder but only for the robbery.

(c) There is no criminal conspiracy as the essential ingredients for such an act are prior
agreement and involvement of more than two persons.

(d) B cannot be said to be part of the criminal conspiracy. B is not liable.

. In the above facts, if they blackmailed Mr. D’s guard to help them in the robbery, is it a
criminal conspiracy?

 (a) Yes, all three of them are part of the criminal conspiracy under Section 120A.

(b) Only A and B form part of the criminal conspiracy.

(c) This is a case of robbery and not criminal conspiracy.

(d) There is no criminal conspiracy since there was no prior agreement between the guard
andA & B.

 
. On Monday, Ashfaq and Bittu committed theft in a house. In addition, there were three
other persons Paras, Qasim and Rohit who happened to rob the same house on the same day.
All five of them were caught by the police. Is there a commission of criminal conspiracy?

(a) All five of them formed the part of criminal conspiracy.

(b) Only Ashfaq and Bittu formed the part of criminal conspiracy.

(c) Only Paras, Qasim and Rohit formed the part of criminal conspiracy.

(d) There is no criminal conspiracy as there is no prior agreement.

. In the above facts, Qasim wanted to kill the house owner’s son due to his old rivalry with
him but Paras and Rohit did not agree to this as they only wanted to rob the house. Qasim
killed the owner’s son after taking their all money and jewellery.

(a) All three of them committed criminal conspiracy.

(b) Only Qasim committed criminal conspiracy.

(c) There is no criminal conspiracy as the prior agreement was only of robbing the house and
not murder of the owner’s son.

(d) This is a case of murder and not criminal conspiracy

26
Passage (Q.-Q.): As per Section 4-A, “Whoever, being the husband or the relative of the
husband of a woman, subjects such woman to cruelty shall be punished with imprisonment
for a term which may extend to 3 years and shall also be liable to fine.” For the purpose of
this section, ‘cruelty’ states (a) any deliberate conduct which is of such a nature as is likely to
induce a woman to commit suicide. Moreover, it may cause a serious injury or danger to life
or health (whether mental or physical) of the woman; or (b) harassment of the woman where
such harassment is with an intention of coercing her or any person related to her to meet any
illegal demand for any property or valuable security. Moreover, it includes coercion on
account of failure by her or any person related to her to meet such demand. In order to
identify as a women, it is sufficient that the person was born a woman. In 19, domestic
violence was decided as a specific criminal offense with the introduction of section 4-A into
the IPC. This section deals with cruelty by a husband or his relatives against a married
woman. Four types of cruelties are which are under this law. Firstly, conduct which is likely
to induce a woman to suicide, secondly, conduct which is likely to cause serious injury to the
life or health of the lady, harassing with the purpose of forcing the lady or her known to give
some property, or lastly, harassment because the woman or her relatives are unable to fulfill
the demands for more money items. The various forms of cruelty are Persistent refusal of
eatables, Insisting on continuous sexual conduct, Continuous locking a lady out of the house,
Physiological violence, Taunting, demoralizing and putting down the lady with the aim of
causing mental harm, Confining the woman at the house and not allowing her normal social
interaction, Abusing children in their mom’s presence with the aim of causing her mental
harm, Threatening with divorce for dowry.

. Laxmi was married to Menon. At the time of marriage, she identified herself as a woman for
she was born that way. However, she does not identify as a woman anymore. She is subject to
cruelty by her husband. Decide

(a) Laxmi can resort to domestic violence laws for she is a woman.

(b) Laxmi cannot resort to domestic violence laws for she does not identify herself as a
woman any more.

(c) Laxmi can resort to domestic violence laws for cruelty was committed onto her (d) None
of the above

68. Rajesh was not happy with his wife for she did not cook good food. He once yelled at her
for the food did not taste good. Decide

(a) Rajesh has committed cruelty for he has induced his wife to commit suicide

(b) Rajesh has not committed cruelty for mere yelling is not enough

(c) Rajesh is liable under domestic violence act for he coerced her to meet an illegal demand

(d) Rajesh's liability will depend upon the facts of the case.

. Rajesh's wife is a smuggler and smuggles gold from Afghanistan. Rajesh asked her to get
diamonds for himself from Afghanistan to avoid the hefty import duty. Even though
diamonds were legal, his wife did not comply. Rajesh beat her up. Decide

(a) Rajesh is liable to be punished under domestic violence act as he harassed his wife for an
illegal demand of property.

(b) Rajesh is not liable for the property was legal to be imported.

(c) Rajesh is liable for diamonds could not be imported legally into India

(d) Rajesh's liability will depend on the circumstances of the case.

. Ram has a good friend Shyam who he considered as family. Shyam always took part in the
decisions of the family including who to marry Ram with. Ram's wife was an RTO officer
and Shyam always harrassed her for not getting his license processed in time. Decide

(a) Shyam is liable for he is a relative and harrassed Ram's wife


(b) Shyam is not liable for he is not a relative and did not harass Ram's wife

(c) Shyam is not liable for he is not a relative. But harassed Ram's wife

(d) Shyam is not liable for he is a relative. But he did not harass Ram's wife.

. Raman was a staunch believer in Chanakya's philosophy which stated that from 0-5 years, a
child must always be forgiven, from 5-18 years he must be beaten/rebuked for his mistakes
and after 18, he becomes a friend of his parents. Raman used to beat up his children often
when they committed mistakes. Their mother however did not like this beating at all and
often complained. Decide

(a) Raman is liable under domestic violence laws as his beating of children forms part of
cruelty

(b) Raman is not liable under domestic violence laws for his acts are not enough to constitute
cruelty.

(c) Raman is liable under Domestic Violence Act for the mental health of the mother was
affected by such beating

(d) Raman is not liable. He is only following Chanakya's philosophy.

. Raman beat up his child only because he knew their mother did not like it. He did it to get
her to sign a contract with his friend Raghav.

(a) Raman is liable under domestic violence laws for he committed cruelty to get his illegal
demand accepted.

(b) Raman is not liable under domestic violence laws for entering into a contract is not an
illegal demand.

(c) Raman is liable for domestic violence for he asked a valuable security to be delivered
illegally.

(d) Raman is not liable for domestic violence as a contract is not a valuable security.

Passage (Q.-Q.): Article 20 of the Constitution provides protection in respect of conviction of


offences and guarantees protection against self-incrimination, double jeopardy i.e. being
punished for the same offence twice and, ex post facto laws i.e. laws that declare any activity
or action as illegal, with retrospective effect. The right against self-incrimination means that
no person in the country can be forced or compelled to provide any testimony against
himself. Forced or compelled testimony is not only limited to confessions or statements but
also includes incriminatory statements. Article 21 of the Constitution provides that “no
person shall be deprived of his life or personal liberty except according to procedure
established by law”. This can be interpreted as a guarantee to live life with dignity, free from
any kind of torture or cruel, inhuman, and degrading treatment or punishment. The right to
life and personal liberty as guaranteed by the Constitution establishes that nobody’s life and
liberty can be taken away and nobody can be deprived of their right to life and personal
liberty unless there is a procedure that has been established by law. The Supreme Court of
India has further interpreted the “procedure established by law” as “due process of law” and
thus stated that as per Article 21 of the Constitution, the procedure of law must be fair, just,
and reasonable and not arbitrary, cruel, or whimsical, which thus provides a triangular
relationship between Articles 14, 19, and 21. [Extracted with revisions from an excerpt of
‘Torture Behind Bars: Role of the Police Force in India’ by Joshua N Aston, published on
scroll.in https://scroll.in/article/5552/this-book-reminds-us-of-what-counts-as-torture-by-the-
police-according-to[1]the-indian-constitution]

. Majnu was arrested for allegedly abducting Laila. The investigation was still going on and it
had not been proved that Majnu is the offender. The police officer in charge of investigating
the same asked Majnu to sign a confession that admitted that he actually abducted Laila.
Majnu was illiterate and refused to sign anything that he couldn’t understand. The police
officer didn’t say anything but forced Majnu to stamp his thumb print even though he kept
persuading him to not do that. Is this a violation of Article 20?

(a) Yes, because compelled testimony is not only limited to confessions or statements but
also includes incriminatory statements

(b) Yes, because Majnu is being forced to sign a confession against his will and his right
against self[1]incrimination is being violated

(c) No, because Article 20 does not provide protection against stamping of thumb prints on
confessions while in police custody

(d) No, because Majnu is not being forced to sign a confession against his will and his right
against self[1]incrimination is not being violated

. Dholakpur government enacted ABC law which made it mandatory for all citizens to wear a
badge of the national flag on their clothes at all times outside the house. The law came into
effect on 31st Jan 2021 but, discussions about the same had been going on since a long period
of time in both the Parliament as well as in public discourse. However, ABC law was
repealed on 28th March 2021. Jaggu was arrested and convicted on 29th March for wearing a
red badge, instead of one featuring the national flag. Is any constitutional right being violated
in this case?

(a) No, because Jaggu was not convicted under any law with retrospective application as he
was arrested on 29th March

(b) No, because such laws are in the interests of fostering a spirit of patriotism which is
exempt from the scope of Article 20

(c) Yes, because ABC law has been repealed and no one can be convicted under a repealed
law

(d) Yes, because Jaggu was convicted under ABC law with retrospective application as it had
already been repealed before the conviction

 
. Sanju was arrested and convicted under the “Terrorist and Disruptive Activities (Prevention)
Act [TADA]” for being in possession of military grade firearms. After his sentence was over,
proceedings were again started against Sanju under TADA for ‘conspiring to hurt the national
integrity of the country’. Sanju’s lawyer challenges this saying that if Sanju was convicted
again then it would amount to Double Jeopardy. Is the challenge correct?

(a) Yes, because the second investigation is a violation of Sanju’s right to protection against
ex-post facto laws

(b) Yes, because being convicted twice under TADA amounts to double jeopardy which is a
violation of Article 20

(c) No, because being in possession of military grade firearms and conspiring to hurt the
national integrity of the country are two separate offences and not the same offence (d) No,
because being in possession of military grade firearms and conspiring to hurt the national
integrity of the country are the same offence under TADA . In general terms, ‘bail is the rule
and jail is the exception’. However, when Sanju was arrested under TADA, he was not
granted any bail. He remained in police custody till the trial ended. Under TADA, an accused
can be arrested and bail may be refused. Such steps are allowed because the same is
considered to be in protection of national interest. Can the refusal of bail under TADA be
considered a violation of right to life and personal liberty?

(a) Yes, because bail is the rule and jail is the exception and nobody can be deprived of the
right to bail

(b) Yes, because the right to life and personal liberty as guaranteed by the Constitution
establishes that nobody’s life and liberty can be taken away

(c) No, because being granted bail is not a part of right to life and personal liberty

(d) No, because bail was not granted to Sanju as he was arrested under TADA which allows
such a step as per the provisions established in the law itself

Passage (Q.-Q.): The concept of mistake is mentioned in Chapter II of the Indian Contract
Act, which deals with voidable contracts and voidable agreements. The Act provides that
“Where both the parties to an agreement are under a mistake as to a matter of fact essential to
the agreement, the agreement is void”. This implies that anerroneous opinion as to the value
of the thing which forms the subject-matter of the agreement is not to be deemed a mistake
“as to a matter of fact.”“As to matter of fact” here would signify that the parties to the case
have consent but a subject matter/ fact has misled them or they weren’t aware of it. The effect
of mistakes as to law: a contract is not voidable because it was caused by a mistake as to any
law in force in India, but a mistake as to a law not in force in India has the same effect as a
mistake of fact. A leading case law on the concept of mistake is Gallaway vs. Gallaway. In
this case, both the parties were under a mistake of fact that they were married. Subsequently,
the two of them agreed to separate and thus, made an agreement. It was then found that the
man’s first wife was still alive, unknown to both of them. The court held that the separation
deed was void on the ground that the agreement had been done on the belief that they were
married to each other, but turned out otherwise with the first wife being alive. [Extracted with
revisions from ‘Mistake under the Indian Contract Act, 18’ byMohammad Shuja Uzair,
published on lexforti.com https://lexforti.com/legal-news/mistake-under-the-indian-contract-
act-18/]

. Chunnu and Munnu are two merchants involved in sea trade. Chunnu agrees to sell a
consignment of coal that was being carried by a ship named ‘Sea Queen’ from London to
Surat. They sign a contract on 24th March 2021. It was subsequently found that Sea Queen
drowned after hitting an iceberg on 23rd March 2020. Munnu sues Chunnu for breach of
contract. Chunnu claims that the contract is void. Is he correct?

(a) No, because the fact that Sea Queen drowned before signing the contract does not void the
contract as the ship was not essential to the contract

(b) Yes, because Sea Queen forms part of subject matter essential to the contract and the fact
that it had drowned before the contract was signed implies a mistake of fact

(c) Yes, because a failure to assess the status of Sea Queen at the time of signing of contract
was a mistake of law which renders the contract void

(d) None of the above

. A wants to buy B’s horse named Chetak. Although Chetak was of unsound mind, he was a
strong horse with a good pace and could run very fast. The fact that Chetak was of unsound
mind was not known to B, who was the owner of the horse. A asks B to sell Chetak to him
because he admired his fast speed and B agrees. The next day, A found out that Chetak was
of unsound mind. A claims that the contract is void while B disagrees. Is A correct?

(a) Yes, because the fact that Chetak was of unsound mind is a matter of fact which forms an
essential part of the contract

(b) Yes, because A would not have agreed to buy Chetak if he was aware that the horse was
of unsound mind

(c) No, because the fact that Chetak was of unsound mind and this was not known to A and B
while signing the contract does not constitute a mistake of fact

(d) No, because the fact that Chetak was of unsound mind is not a matter of fact which forms
an essential part of the contract

. Hari wanted to buy a painting for his wife Meena. He went to a shop and asked for ‘Mona
Lisa’. The shopkeeper showed Hari a painting as well as a necklace, both of which were
named Mona Lisa. Subsequently, he kept the necklace back and informed Hari about the cost
of the painting. He paid for the same and the shopkeeper packed it for him. Next morning,
Hari went back to the shop and told the shopkeeper that he had packed the incorrect item
since he wanted to buy the Mona Lisa necklace and not the painting. Hari asked him to return
his money as the contract had become void since there was a mistake of fact. The shopkeeper
refused to agree. Based on your reading of the passage, is the shopkeeper correct in doing so?
(a) Yes, because although there was a mistake of fact regarding the two Mona Lisas, this was
not essential to the contract between Hari and the shopkeeper

(b) Yes, because there was no mistake of fact as Hari agreed to buy the painting and the
shopkeeper agreed to sell the painting only

(c) No, because there was a mistake of fact between Hari and the shopkeeper concerning
which Mona Lisa to buy and sell respectively

(d) None of the above

. Chulbul Pandey wanted to buy a pair of custom Pay Ban sunglasses. He went to the Pay Ban
store and asked for the same. Mr. Pay helped Chulbul with his request. After the pair of
sunglasses were selected and customised, he sold the pair to Chulbul for Rs. 5000 and
Chulbul went back home. Subsequently, Mr. Pay found out that the average market cost for a
pair of custom sunglasses is Rs 10,000. He sought to set aside the contract as it was void due
to a mistake of fact. Chulbul disagreed and said that there was no mistake of fact. Is he
correct?

(a) No, because the value of the sunglasses is a part of the subject matter of the agreement
and an incorrect value can be deemed as a mistake of fact

(b) No, because an incorrect value being ascertained to goods can constitute a mistake of law
and this can be used to set aside a contract

(c) Yes, because an incorrect value being ascertained to goods can constitute a mistake of law
and this can be used to set aside a contract

(d) Yes, because the fact that sunglasses were bought for a cheaper price than the market
average does not render the contract of sale void

Passage (Q.-Q.): Theft, robbery and dacoity are three types of crimes under the law on crimes
known as the Indian Penal Code, 18. Theft is when someone steals a movable property out of
your possession with the intention of stealing it from you. For example, if Shikar steals five
hundred rupees from your wallet when you were not in the room, then it is an act of theft.
Robbery is an aggravated form of theft, so if someone attempts or causes any hurt, wrongful
restraint or death in order to commit an act of theft, it is known as robbery. For example, if
Shikar comes to your house, beats you up and then steals five hundred rupees from your
wallet, it is an act of robbery. Dacoity is a crime when 5 or more people commit or attempt to
or aid someone to commit an act of robbery together. For example, if Shikhar and his 4
friends come to your house and beat you up to steal your money and jewellery, then it is an
act of dacoity. The punishment for these crimes are also different. If you commit theft you
may be punished with jail time up to 3 years or a fine or both. If you commit robbery you
may be punished with jail time up to 10 years or a fine or both. If you commit dacoity then
you may be punished with jail time which may be for life and a fine. [Extracted with minor
revisions from ‘What is the difference between theft, robbery and dacoity?’ published on
nyaaya.org https://nyaaya.org/questions/theft/what-is-the-difference-between-theft-robbery-
and-dacoity/]
. Mogambo was a small time sandalwood smuggler. In the dead of the night, he used to go to
X’s farm where he used to cultivate sandalwood trees. He started cutting the trees and
severing them off the earth one by one. He then put all the trees in his truck. Just as he was
about to drive his truck away, police caught him and arrested him for theft. Did Mogambo
commit theft?

(a) Yes, because he had the intention of stealing X’s sandalwood trees and it doesn’t matter if
they were immovable in nature

(b) Yes, because he severed the sandalwood trees off the earth and was stealing them from X
by taking them out of his possession

(c) No, because sandalwood trees cannot be stolen as they are immovable and cannot be
taken away from someone’s possession

(d) No, because he did not commit theft as there is a difference between theft and smuggling
and sandalwood trees can only be smuggled

. Sonpari was an old lady who was walking her dog in the neighbourhood park. Suddenly, Z
came out of nowhere and showed Sonpari a knife. Z threatened to kill Sonpari’s dog if she
made any noise. Sonpari was frightened but she stood still. Z while holding the knife in his
hand, snatched her gold chain from neck forcefully and ran away. Is this an act of robbery?

(a) No, because Z did not attempt or cause any hurt to Sonpari and merely showed her a knife

(b) No, because Z committed theft and not robbery because he stole a movable property out
of Sonpari’s possession with an intention of stealing it from her

(c) Yes, because Z committed an aggravated form of theft by threatening to kill her dog and
snatching her chain while showing her a knife

(d) None of the above

. Roald and Dahl are two friends. Roald had a very good book collection so Dahl decided to
visit his house to borrow a book. However, when Dahl went there, Roald was not home. So,
Dahl picked up the book from Roald’s bookshelf and asked his mother if he may borrow the
same. Roald’s mother permitted Dahl to take the book but she forgot to inform Roald about
it. When Roald came home, he thought that Dahl had stolen his book. Can Roald accuse Dahl
of theft?

(a) Yes, because Dahl took a movable property out of Roald’s possession without informing
him

(b) Yes, because Dahl had the intention of stealing Roald’s book otherwise he would’ve
waited to take permission from Roald himself

(c) No, because Dahl asked Roald’s mother for permission before taking the book and he
wouldn’t have done that if had a criminal intention
(d) No, because merely taking a book out of someone’s possession without asking them does
not constitute theft

. Shinchan was going on a vacation to Chambal. Suddenly, Jagga Daaku and his gang of 7
members appeared on the lonely highway with guns and surrounded Shinchan’s vehicle,
forcing him to stop. They asked Shinchan to come out of the car so that they could steal all
the valuables in the car as well as petrol and other parts from the car itself. They forced
Shinchan to sit on his knees and one of Jagga Daaku’s men held a gun over Shinchan’s head.
After they had taken whatever they wished to take, they left Shinchan and rode away on their
horses. Has Shinchan been a victim of dacoity?

(a) Yes, because Jagga Daaku and his gang together stole Shinchan’s valuables while by
threatening to hurt him by holding him at gun-point

(b) Yes, because although Jagga Daaku and his friends didn’t actually threaten to hurt
Shinchan they stole movable property out of his possession

(c) No, because this was an act of theft and not dacoity as there was no force or aggravation
involved in the incident (d) None of the above

Passage (Q.-Q.): The Supreme Court said on March 3 that it is “not seditious to have views
different from the government.”The apex court was hearing a Public Interest Litigation –
which it eventually dismissed – filed against former Jammu and Kashmir chief minister
Farooq Abdullah for his remarks on the Centre’s move to read down Article 3. The Supreme
Court’s observation is significant considering the spate of sedition cases slapped against
activists and journalists recently. In a recent interview, retired Supreme Court judge, Justice
Deepak Gupta, had said Section 124A is used by governments to create fear in the citizenry
to prevent or throttle dissent. He said the law is widely misused and, worse, cases of its
misuse are increasing “exponentially”. Section 124 A of the Indian Penal Code prescribes the
punishment for Sedition which defines the offence as “Whoever, by words, either spoken or
written, or by signs, or by visible representation, or otherwise, brings or attempts to bring into
hatred or contempt, or excites or attempts to excite disaffection towards, the Government
established by law”. [Extracted with revisions from ‘'Not Seditious to Have Views Different
from Govt': SC Quashes PIL Against Farooq Abdullah’ published 3Mar2021 on the wire
https://thewire.in/law/supreme-court-sedition-government-farooq-abdullah-article-3-china]

. Risha Davi was a student activist. There were large scale farmer agitations going on in her
country so she shared a “toolkit” with necessary information and helplines on her Instagram
account. She edited two lines of the said toolkit which she had obtained from an international
activism page to make it India centric and more precise. Two days after sharing the edited
toolkit, Risha was arrested by the police on the pretext of sedition for “harming India’s
international image and encouraging unrest and rebellion”. Is the arrest valid?

(a) Yes, because sharing of toolkit amounts to exciting disaffection for the government
established by law through visual representation
(b) Yes, because sharing of toolkit amounts to bringing hatred, contempt and disaffection
towards the government established by law

(c) No, because sharing of toolkit does not bring contempt or hatred towards the government
established by law

(d) No, because sharing of toolkit by way of words does not amount to exciting disaffection
towards the government established by law

. Nemo was a famous documentary director. He was renowned for making documentaries
that depicted the "real India" in a raw manner, instead of portraying a fictional image like
commercial Bollywood movies. His latest documentary depicted violence in jails and how
policemen employ violent means to get people to confess their crimes. The film also
graphically highlighted the corruption committed by high level authorities and how they hide
police brutality in such cases. The government asked the police to arrest Nemo for portraying
it in such a bad light. Is it a case fit for sedition?

(a) No, because it is seditious to present the government in a different light than the views
advocated by it to the public

(b) No, because it is not seditious to present the government in a different light than the views
advocated by it to the public

(c) Yes, because it is seditious to present the government in a different light than the views
advocated by it to the public

(d) Yes, because it is not seditious to present the government in a different light than the
views advocated by it to the public

. The documentary was edited in the production stage before finally being made available for
free on online streaming platforms. It now contains a disclaimer in the beginning that states
"all incidents are based on true and real life incidents and may bear resemblance to real life".
Although no proof for the same has been furnished or relied upon. The end of the
documentary also encourages the viewers to take mass action against the government, even if
it turns violent, in order to control the situation. Does this amount to sedition?

(a) Yes, because the disclaimer is seditious in nature and encourages disaffection towards the
government

(b) Yes, because the documentary is seditious and encourages disaffection towards the
government

(c) Yes, because the end of the documentary is seditious in nature and encourages
disaffection towards the government

(d) None of the above

 
. The Supreme Court declared that homosexuality is not illegal or criminal in nature. The
government did not agree with this view of the Supreme Court and although it did not arrest
people for being homosexual, it did not support the same and actively discouraged it. To
celebrate the Supreme Court's decision, a pride parade was organised and people from all
walks of life participated in it. The next day, police arrested the organizer of the parade for
the offence of sedition. The official stand on the parade was that it was "violative of
government policy, Indian morality and good public behaviour". Is the arrest justified?

(a) No, because holding a pride parade does not constitute a violation of Section 124A

(b) No, because the law is being misused to create fear in the citizenry to prevent or throttle
dissent

(c) No, because it is not seditious to have views different from the government

(d) Yes, because a pride parade contrary to officially government policy may incite contempt
towards the government

. Balchandra Bala is a poet by profession. His latest poem is a narrative of all the contentious
activities and actions taken by the government in the last five years. The poem is very
descriptive in nature and was published in a national newspaper. The last stanza is
particularly elaborate which ends with a call of action towards the reader and how they must
take charge of the situation and hold the government accountable for its misdeeds. Can
Balchandra Bala be held liable for attempting to excite disaffection towards the government?

(a) Yes, because he is attempting to bring in hatred and excite disaffection towards the
government established by law by spoken words

(b) Yes, because he is attempting to bring in hatred and excite disaffection towards the
government established by law by written words

(c) Yes, because he is attempting to bring in hatred and excite disaffection towards the
government established by law by signs

(d) Yes, because he is attempting to bring in hatred and excite disaffection towards the
government established by law by visual representation

Passage (Q.-Q.): In practical terms, 'transit remand order' means on order passed by a Judicial
Magistrate remanding an arrested person to police custody for the purpose of his transit to
another state. Further, the local Magistrate authorizes the other-state police to take the
arrestee out of his home state. Section 1 lays down the procedure when the investigation
cannot be completed in 24 hours as fixed by Section 57 and requires the police officer to
transmit to the nearest Jurisdictional Magistrate a copy of the entries in the diary relating to
the case, The Magistrate in turn may authorise detention of the accused for a term not
exceeding 15 days, and if he has no jurisdiction, then he may order the accused to be
forwarded to a Magistrate having such jurisdiction. Article 22 (2) of the Constitution
prescribes that every person who is arrested and detained in custody shall be produced before
the nearest magistrate within a period of twenty-four hours exclusive of the time taken for
journey of such arrest from the place of arrest to the court of the magistrate and no such
person shall be detained in custody beyond the said period without the authority of the
magistrate. With the aim of ensuring protection of the personal liberty of an individual, the
Courts have evolved the concept of 'transit remand’ in cases where the person is sought to be
taken out of state by police from a different state. In such cases, the person ought to be
produced 23 of 36 before the nearest Magistrate soon after the arrest, and ought to be taken
out of the arrestee's home state only on the strength of a 'transit remand order' issued by such
Magistrate. Article 22(1) of the Constitution of India states that a person who is arrested,
shall not be denied the right to consult and to be defended by a legal practitioner of his
choice.

. Santosh was alleged to be the murderer of famous journalist, Kavya, which took place in
Mumbai. He belongs to state of West Bengal but was in Mumbai for some business purpose.
He was arrested and was taken into police custody for a period of 10 days. Later on, during
the investigation, police find some clue at his place, which can be strong evidence against
Santosh. For collecting the same, they took Santosh with them to West Bengal and presented
the accused before the magistrate with the evidence after 20 days. On which ground, court
can claim the irregularity in the proceedings-

(a) The police have taken the accused Santosh into police custody for period of more than 15
days which cannot be the case at all.

(b) There was no order authorised by Judicial Magistrate for transit remand to take Santosh
out of Mumbai to state of West Bengal.

(c) The police have not presented the accused Santosh before the Magistrate within 48 hours
of arrest.

(d) Santosh was allowed to consult his lawyer after taking into police custody and his
personal liberty was being protected by the investigating officers.

. Manish was alleged to be one of the co-accused in case of rape of minor girl under POCSO
and was taken into police custody. He was presented before the nearest judicial magistrate
and was investigated in the case. Police took 17 days for the investigation and presented the
accused before the court for the trial. The defence side claimed bail for Manish as he is
innocent and cannot be kept in police custody further. Also, the trial cannot be taken further
as there are certain irregularities in the proceedings. Police cannot keep Manish under
custody for period of more than 15 days. The prosecution can claim-

(a) Contention of defence to be incorrect as Manish was under police custody for period of
only 15 days and remaining period was counted under transit remand.

(b) Contention of defence to be correct as Manish was under police custody for period of
more than 15 days and it amounts to irregularity under set procedure.

(c) Contention of defence to be incorrect as Manish was under police custody with the order
of magistrate and 2 days extra under police custody is a minor irregularity to be ignored.
(d) Contention of defence to be correct as Manish cannot be under police or judicial custody
for more than 15 days and there has to be reasons given in writing for any such irregularity.

. A crime was said to be committed in Bhopal district of state of Madhya Pradesh and
accused Pontu was arrested near Indore district. A suit was filed against him in Indore
District Court and was taken into custody. Police investigated the case and held that there
have to be strong evidences against the accused to held him liable. Within 15 days of police
custody, Indore Police find direct evidence in the case and court solely relied on that direct
evidence. Pontu was convicted by the trial court and appeal was filed in Madhya Pradesh
High Court. The defence claimed that Trial Court order has to be quashed. On what ground it
can be quashed-

(a) The trial court convicting Pontu has no jurisdiction to try the case and thus the order
passed by the court is not valid and amounts to irregularity.

(b) The trial court has not authorised the police remand for further investigation to the police
officers and cannot convict the accused without enough evidences.

(c) The trial court was not presented with the police diaries relating to the case and conviction
of Pontu was done solely on the basis of direct evidence.

(d) There were no such major irregularities in the proceedings of Trial Court and order of
conviction of Pontu is valid one. It cannot be quashed.

. Monika was of unsound mind and do not possess basic understanding of a reasonable man.
She was in love with Mohit, his neighbour. Mohit was engaged to Sumona and was happy
with her. One day, Monika saw him over the terrace with Sumona and they both were making
love. She felt enraged and hit them both with a stone. Mohit was hurt badly and was taken to
hospital. He died there. Police arrested her and taken into police custody. She was presented
before the judicial magistrate after 28 hours of arrest. In the trial against her, the defence
claimed that she has to be presented before the magistrate within 24 hours of arrest and it
amounts to irregularities if not followed. It violates the personal liberty of the accused and
thus the proceedings are void. The police claimed that delay of 4 hours is due to long journey
they had to take to the way to Magistrates house. Decide-

(a) The contention by the investigating officer is incorrect as the time period of 24 hours is
inclusive of time taken to reach out to the judicial magistrate.

(b) The contention by the investigating officer is correct as the time period of 24 hours is
exclusive of time taken to reach out to the judicial magistrate.

(c) The contention by the investigating officer is incorrect as the time period of 24 hours is
exclusive of time taken to reach out to the judicial magistrate.

(d) The contention by the investigating officer is correct as the time period of 24 hours is
exclusive of time taken to reach out to the judicial magistrate.

 
. In which of the following situations, transit remand cannot be ordered by the judicial
magistrate.

(a) Where accused A was arrested in state of Uttar Pradesh and has to be taken to trial court
in Andhra Pradesh as murder was committed in that state.

(b) Where accused B was taken from Jaipur district to Jodhpur district as Jodhpur district
court has the jurisdiction to try the case.

(c) Where accused C was taken to state of Uttrakhand for the purpose of investigation and
collecting further evidences.

(d) Where accused D was to be presented before the judicial magistrate in Bikaner district of
state of Rajasthan and was arrested in Barmer district.

Passage (Q.-Q.): Section 2 (11) of Consumer Protection Act, 2019 defines Deficiency of
Service as “any fault, imperfection, shortcoming or inadequacy in the quality, nature and
manner of performance which is required to be maintained by or under any law for the time
being in force or has been undertaken to be performed by a person in pursuance of a contract
or otherwise in relation to any service and includes (a) any act of negligence or omission or
commission by such person to the consumer and (b) deliberate withholding of relevant
information by such person to the consumer.”The term ‘defect’ is defined under Section 2
(10) of Consumer Protection Act, 2019 as “any fault, imperfection or shortcoming in the
quality, quantity, potency, purity or standard which is required to be maintained by or under
and law for the time being in force or under any contract, express or implied or as is claimed
by the trader in any manner whatsoever in relation to any goods or products and the
expression ‘defective’ shall be construed accordingly “.In any buyer-seller relationship,
deficiency of service prevails. Such as legal aid, banks, railways, construction, transportation,
education, electricity, entertainment, restaurant, hospitality, etc. The consequences of
deficiency of service can range from inconvenience or harassment to mental or physical
injury to death, thereby leading to legal consequences. In Indian Medical Association vs. V.P.
Shanth, the liability of doctor and hospital management arises when a patient is admitted. The
standard duty of care must be maintained by the hospital. When a patient is admitted, he/she
is also considered as a consumer. Deficiency in services plays a vital role to a consumer as
well as the service provider. The Consumer Protection Act, 2019 has successfully managed to
incorporate as aspect of deficiency that may be found in a product or service, hence giving
the consumer the power to take any unfair practice to the court of law.

. Manohar was the driver of Charbhuja bus services. He was on duty for last 10 years. Due to
some technical bus failures, he was on leave for a week and left the bus for repairing purpose.
He resumed his services back on Monday and took the ride for Nagpur from Bombay. On the
way, suddenly a truck came into his way and he applied breaks but breaks were not working
and the bus met with an accident. Sunil, one of the injured passengers, sued Manohar and
Charbhuja bus services for negligence. Manohar contended that it was the fault of mechanic
Chintu who was given bus for repairing and had negligently ignored the breaks fault. Can
Sunil claim compensation from Chintu?

(a) Sunil can validly claim compensation from Manohar as he was negligent to check the
technical fault of bus before taking the ride.

(b) Sunil cannot validly claim compensation from Manohar as bus services are held to be
liable for negligence as it is there duty to check technical faults of the bus.

(c) Sunil can validly claim compensation from Chintu as he was negligent in repairing the
technical fault of breaks.

(d) Sunil cannot validly claim compensation from Chintu as Manohar has the duty to check
the repair works done by Chintu.

. Sheetal, a beautician was in habit of applying cosmetics. She had a fixed buyer. She
purchases it from Motilal Cosmetics Ltd. This directly supplies from the manufacturer. She
asked him for facial spa beauty product. He arranged it for her and asked for Rs.0 per pack.
She paid him. When she went home, she checked the product. It was 50 grams less. She filed
a complaint against the manufacturer in the consumer forum for such defect. Decide-

(a) She cannot claim for such defect in the customer forum as it is such a trivial defect to be
claimed for compensation.

(b) She can claim for such defect in the customer forum as the less quantity in beauty product
is a defect and damages can be claimed for.

(c) She cannot claim for such defect in the consumer forum as there was no customer service
provider relationship from Sheetal.

(d) She can claim for such defect only from manufacturer and not from the shop owner as
manufacturer was the real seller.

. Mahima, a journalist, ordered a brand new style kurti from Payzone app. She ordered it on
15th January 2021. When she got the product, it was a defective product and not up to her
satisfaction. She returned it and got it exchanged with another same product. This time it was
not fit for her size and she sent back the product and claimed for refund. Payzone refused
such refund as their policy states for one time exchange/refund/return which was clearly
visible in the invoice. Decide-

(a) She can file a complaint in consumer forum for such fraud by Payzone and get refund for
her amount duly paid.

(b) She cannot claim for such refund as no complaint can be filed against Payzone in
consumer forum and she was negligent about refund policy.

(c) She can claim compensation from Payzone along with refund for not bringing such policy
to her notice and sending defective products.
(d) She cannot claim any refund or compensation with Payzone as thei refund policy was
clearly mentioned and she consents to it.

. Rakesh, was travelling in a train from Surat to Bombay and was depart at Nagpur. One of
the passengers Suresh, left at Ahmadabad and took Rakesh’s luggage. After long journey to
Nagpur, Rakesh get to know about his luggage and asked the authorities to help him find his
luggage. He was informed that Suresh had submitted it in cloak room at Ahmadabad station.
Rakesh gets back to Ahmadabad and found his luggage lost. He asked the authorities and was
informed that he is the only one who has to take care of goods as per given in terms and
conditions in their receipts.

(a) Rakesh can claim compensation from railway authority and file a complaint in consumer
forum for such negligence on their part.

(b) Rakesh cannot claim compensation from railway authorities and cannot file a complaint
in consumer forum for such negligence.

(c) Rakesh can claim compensation from railway authority as they should inform Rakesh or
Suresh about their terms and conditions in railway receipts.

(d) Rakesh cannot claim compensations from railway authorities and he was negligent in
reading terms and conditions of railway receipt.

. The relationship between the buyer and seller can be said to be of fiduciary relationship
where there is element of trust and on breach of such trust, buyer/customer has all the rights
reserved to claim compensation or damages from the seller/manufacturer. From the following
below, identify the other essential elements of fiduciary relationship if any, which cannot be
part of buyer-seller relationship. I. Fiduciary may derive a profit for himself at the expense of
beneficiary whose interest is bound to protect. II. Fiduciary undertakes to act voluntarily and
is not getting any remuneration for his services unless contrary is provided. III. Fiduciary
office is onerous and hence, fiduciary to perform and to act for the protection of the interest
of the person whom is bound to protect as a matter of confidential obligation he has
undertaken to fulfill. IV. Fiduciary transaction must be good faith transaction.

(a) Only III.

(b) Both I and II.

(c) I, II and IV.

(d) Only IV.

Passage (Q.-Q.): The Supreme Court reiterated that the anticipatory bail once granted by the
court does not automatically end on the filing of the charge sheet. In this case, theAllahabad
High Court, on an application made by the complainant held that the anticipatory bail granted
to the accused by the Trial Court came to an end with the filing of a charge sheet, and
directed him to surrender and apply for regular bail. Challenging this order, the accused
approached the Apex Court relying on recent constitution bench judgment in Sushila
Aggarwal and Ors. Vs. State (NCT of Delhi) and Anr. to contend that there is no principle of
law which required that anticipatory bail once granted automatically comes to an end on the
filing of the charge sheet. The bench noted that in Sushila Aggarwal it was observed that
'mere fact that an accused is given relief under Section 438 at one stage, per se does not mean
that upon the filing of a charge sheet, he is necessarily to surrender or/and apply for regular
bail.' The court held that “the life or duration of an anticipatory bail order does not end
normally at the time and stage when the accused is summoned by the court, or when charges
are framed, but can continue till the end of the trial. Again, if there are any special or peculiar
features necessitating the court to limit the tenure of anticipatory bail, it is open for it to do
so”. It was further held in the said judgment that 'mere subsequent event of the filing of a
charge sheet cannot compel the accused to surrender and seek regular bail. In view of the
above, we are of the opinion that the High Court wrongly held that the anticipatory bail
granted to the accused had come to an end with the filing of the charge sheet, the bench said
while allowing the appeal.

. Manoj and Divya were in friendly relationship with each other. They get into sexual
relations too. Soon, Divya asked Manoj to marry her. He refused to do so and stated that his
family was never in favor of their relationship and he was with her for only a short period of
relationship and experience the live in relationships. She filed as frivolous case against Manoj
under sec. 354 of IPC. The police took Manoj in custody and was presented before the court
for trial. The investigating officer asked for extension of custody and defence lawyer claimed
for anticipatory bail. The court-

(a) Can grant anticipatory bail to Manoj as he has not committed the said offence and no
charge sheet has been filed by the investigating officer.

(b) Cannot grant anticipatory bail to Manoj as he has been taken into custody by investigating
officer.

(c) Can grant anticipatory bail to Manoj as he has not taken into judicial custody by
investigating officer and charge sheet has been filed.

(d) Cannot grant anticipatory bail to Manoj as he has committed the said offence and charge
sheet has been filed by the investigating officer.

. Vijay Malia, the politician of well known political party, was involved in a money
laundering case. He was being party to lots of corruption taking place within the party and
was handling the accounts for the same. He asked his assistant to keep the accounting
information hidden as a secret and should not put it in public domain. Unfortunately, his
assistant leaked the information and it was all over the media that huge scandals take place in
the political party and they are soon to be tried by the Apex Court. Vijay Malia approached
the court for anticipatory bail. Choose the appropriate contention.

(a) Anticipatory bail cannot be claimed as his assistant leaked the information and he cannot
be alleged for the said offence of money laundering.
(b) Regular bail can be granted instead of anticipatory bail. There is no such anticipation for
Vijay Malia to be arrested and tried for the case.

(c) Anticipatory bail can be claimed as there are reasonable appreciations that Vijay Malia
can be tried for the money laundering case by the Apex Court.

(d) Regular bail cannot be granted as there is no such arrest made and Vijay Malia, being a
public servant, given special treatment under law.

. Montu was a business man and was having enmity with his competitor. He played a chance
and asked his friend Shontu to approach his competitor company and became the employee
of the same. Shontu take into confirmation the opposite company and became accounting
head of the same. Soon he got a chance, he intentionally mismatched the accounting books
and made a big scam of Rs.50,000 Crore. The company gets into losses and was dissolved
soon. They get to know that these malpractices was by Montu and simultaneously filed a case
in NCLT to take action against Montu and his company. NCLT passed an order for
investigation into the case and an interim order for arrest of Montu. Decide-

(a) NCLT cannot grant anticipatory bail to Montu as it is not a court and power to grant
anticipatory bail is with courts only.

(b) NCLT can grant anticipatory bail to Montu as no arrest was made and no charge sheet has
been filed by the investigating officers against Montu.

(c) NCLT cannot grant anticipatory bail to Montu as interim order for his arrest has already
being made and bail cannot be granted in such cases of fraud.

(d) NCLT can grant anticipatory bail to Montu only when strong evidences are to be proved
against him and no reasonable doubt has been made.

. Pritam was a young boy of 21 years of age. He was not in good friend circle and was
influenced by his friend where he was forced to have sexual relation with a girl to exist in a
society. He was questioned on his virginity. He approached a girl and gets much close to her.
He gets this opportunity and one day took her out for dinner. He offered her a drink with
drugs and as soon as she lost her senses, he took her to room and sexually assaulted her. The
girl filed a complaint against Pritam and he got anticipatory bail. The investigating officer
still arrested him. He was kept in judicial custody for period of 50 days and charge sheet was
filed. He applied for regular bail, but was rejected prior to filing of charge sheet. The
investigating officer forgets to add some charges and filed a supplementary charge sheet in
the court. The defence again claimed regular bail but was rejected by the court. Choose the
incorrect option-

(a) Pritam cannot be taken and kept into custody for period of 50 days as he was granted
anticipatory bail pre filing of charge sheet.

(b) Pritam can be granted regular bail once the charge sheet has been filed and anticipatory
bail does not come to an end post filing of charge sheet.
(c) Pritam can be granted regular bail post filing of supplementary charge sheet and the
anticipatory bail does not come to an end post filing of supplementary charge sheet.

(d) Pritam cannot be granted regular bail post filing of supplementary charge sheet and the
anticipatory bail does come to an end post filing of supplementary charge sheet.

. Which among the following can be valid considerations before the court for not granting
anticipatory bail?

 I. No chances that he may threaten the witnesses and tamper the evidences.

II. Gravity of allegations and injustice to be caused.

III. Antecedents of the accused approaching for the anticipatory bail.

IV. Whether no malicious prosecution is possible.

(a) Both II and III

(b) I,III and IV

(c) Only III

(d) All I,II,III and IV

27
Passage (Q.-Q.): The Supreme Court briefly explained the scope of Section 311 of the Code
of Criminal Procedure in its two recent judgments. As per Section 311, any Court may, at any
stage of any inquiry, trial or other proceeding under this Code, summon any person as a
witness, or examine any person in attendance, though not summoned as a witness, or recall
and re-examine any person already examined if his evidence appears to it to be essential to
the just decision of the case. The Court observed that the object underlying Section 311 CrPC
is that there may not be failure of justice on account of mistake of either party in bringing the
valuable evidence on record or leaving ambiguity in the statements of the witnesses examined
from either side. Court observed “At the same time, the discretionary power vested under
Section 311 CrPC has to be exercised judiciously for strong and valid reasons and with
caution and circumspection to meet the ends of justice.” The bench referred to Swapan
Kumar Chatterjee Vs. CBI “the court has vide power under this section to even recall
witnesses for re-examination or further examination, necessary in the interest of justice, but
the same has to be exercised after taking into consideration the facts and circumstances of
each case. The power under this provision shall not be exercised if the court is of the view
that the application has been filed as an abuse of the process of law. In another case, the
bench observed that the true test for invoking Section 311 CrPC is whether it appears to the
Court that the evidence is essential to the just decision of the case. Rather, it noted that “the
length/duration of a case cannot displace the basic requirement of ensuring the just decision
after taking all the necessary and material evidence on record. In other words, the age of a
case, by itself, cannot be decisive in the matter when a prayer is made for examination of a
material witness”, the bench noted while allowing the appeal. Aim Of Every Court Is To
Discover The Truth: supreme Court Explains Scope of Section 311 CrPC (Live Law, 06
March 2021) as accessed on 07 March 2021.

. Madhya Pradesh High Court has been hearing case of a mob lynching for 7 years. Police
had filed chargesheet against 4 accused, one of which is a woman. Court, after analysing all
the evidence, gave prosecution and defence to conclude their arguments. However, defence
made an application to produce before the court a key witness. The application states that this
is a crucial witness who was present at the scene of the crime. The application was opposed
on the ground that the case has already been dragged for a long time and shall not be
extended anymore. The prosecution further contends that the defence has deliberately filed
the application to waste the time of the court. The court, however, allowed the application
and fixed a date for examining the witnesses. Choose the correct option:

(a) The court should not allow the application as the case has been heard for 7 years now and
the defence has made the application maliciously to further delay the judgment.

(b) The court should allow the application as it would be against the principles of natural
justice if the court gives a judgment without considering the defence’s version of the facts.

(c) The court should not allow the application as the court has already heard the witnesses
from both sides and there is no need to give another opportunity to either of the sides.

(d) The court should allow the application as it has been filed to present before the court a
crucial witness and listening to their testimony is crucial to preserve the instrument of justice.

. Ali, an IAS officer has been charged under Prevention of Corruption Act for taking bribes to
pass a tender for construction of roads. The defendant’s lawyer 2 witnesses before the trial
court after examination of which the court fixed the next date for concluding arguments. On
the next hearing, the defendant filed an application for presenting two more witnesses to
bring to light some crucial evidence in favour of the defence. The trial court rejected the
application, and asked parties to proceed with final hearing. Defendant has filed an appeal
before the High Court against the denial of the application; High Court has allowed the
appeal and directed the trial court to decide a date for examining the two witnesses. Which of
the following is the correct answer with respect to the order of the High Court? 

(a) High Court has rightly allowed the appeal as no party shall be deprived of an opportunity
to present all their witnesses before the court, in the interest of justice.

(b) High Court has wrongly allowed the appeal as the court had already fixed the date for
conclusion of the arguments and application for examination of the witness implies the
malicious intention behind filing the application.

(c) High Court has rightly allowed the application as the same would afford defence an
opportunity to produce a crucial witness and prevent the court from reaching a decision prior
to considering all the relevant evidence on record

(d) High court has wrongly allowed the application as the trial court felt that allowing the
application would be an abuse of the process of law.

 
. Srikanth was a minor charged under various sections of Indian Penal Code, for rioting,
criminal trespassing and abduction being tried before a juvenile court, along with some adults
whose case was being tried before Sessions Court. The prosecution presented two witnesses,
who testified against Srikanth. However, these witnesses did not support the prosecution's
version of the facts before the Sessions Court in the simultaneous trial. Srikanth filed an
application for resummoning the witnesses for cross-examination. Prosecution has not
particularly opposed the application. However, the Juvenile Court finds that the application is
malicious and rejects the request. Decide whether the application should be rejected or not.

(a) Application has been rightly rejected as after prosecution presented its witness, the
defence was given a chance to re-examine the witness and there is no further need to allow a
re-examination.

(b) Application has been wrongly rejected as the witness has given contrary testimonies at
both the trial and therefore Srikanth shall be allowed to re-examine the witness in the cause of
justice.

(c) Application has been rightly rejected as the two trials are separate ones and the testimony
of the witness in other trial has no bearing on the instant case.

(d) Application has been wrongly rejected as a minor shall be shown more consideration than
adult accused to ensure that a minor is not wrongly accused of a crime they have not
committed

. Tarun Gawli was a popular politician and gangster in Mumbai. A contractor pressed charges
against him for extortion and assault. During the trial, Gawli’s lawyer produced witnesses to
support their case. After summation of arguments, the court reserved the judgment for a later
date. In the meantimeGawli filed an application for producing more witnesses that would
corroborate his alibi, however, nothing material was stated by the witnesses to support the
defence side. Court fixed another date for judgment, but Gawli filed another application to
recall previous witnesses. The court denied the application against which Gawli has filed an
appeal before the High Court. Which of the following shall be the order of the high court?

(a) Appeal shall be allowed as there may not be failure of justice on account of mistake of
either party in bringing the valuable evidence on record or leaving ambiguity in the
statements of the witnesses examined from either side.

(b) Appeal shall not be allowed as witnesses produced by the defence have not stated
anything to support defence’s version of the facts and thus the process under section 311 is
just being used to abuse the consideration given under the law.

(c) Appeal shall be allowed as the court would be acting beyond the bounds of law if it
reaches a decision before considering the evidence or witness produced by the defence side.

(d) Appeal shall not be allowed as TarunGawli is a powerful person who could produce fake
witnesses to support his case and therefore the court shall no more consider an application for
re-examining the witnesses.

 
. Which of the following is not the objective of section 311 of the Code of Criminal
Procedure:

I. There may not be failure of justice on account of mistake of either party in bringing the
valuable evidence on record

II. No evidence crucial to a just decision by the court is missed.

III. To ensure that the just decision is reached after taking all the necessary and material
evidence on record, unless it causes any delay in the process of law.

IV. To ensure that courts are under duty to consider only those witnesses and evidence that
are produced before them with the bonafide intention. Which of the following is the correct
option?

(a) I, II and IV (b) I and II (c) III and IV (d) III only

Passage (Q.-Q.): The Bombay High Court on Thursday (04th March) directed media
organizations to observe restraint and not publish any content related to the incident of death
of a Pune woman who fell from the balcony of her house and subsequently died. The Bench
of Justice S. S. Shinde & Justice Manish Pitale also directed the media organizations to
scrupulously follow the guidelines which the Court had laid down in the case of Nilesh
Navlakha v. Union of India [Sushant Singh Rajput case] and not to give any unnecessary
publicity to the illicit relationship of the Woman. Soon after the death of the Petitioner's
daughter, allegedly, various news in print and electronic media emerged and started
circulating reports alleging that his daughter was having illicit relations with one Y. In Nilesh
Navalakha The Bombay High Court, gave judgment on a clutch of PILs regarding 'media
trials' in the Sushant Singh Rajput death case, observed that the media ought to avoid reports
touching upon an ongoing investigation and present facts that are in the public interest rather
than what, according to the media, the public is interested in. The Court in this case had laid
down various guidelines for reporting on "sensitive criminal matters, including death by
suicide, and had held that the media should avoid putting photographs of victims, depicting
the deceased as a weak character, or try to reconstruct the incident while the investigation is
underway. The division bench of Chief Justice Dipankar Dutta and Justice GS Kulkarni had
directed the print and electronic media to exercise restraint and refrain from publishing any
news item, debate, discussion on interview while reporting on certain cases or at a particular
stage of investigation. In this significant pronouncement, the Bombay High Court had held
that media trial during criminal investigation interferes with the administration of justice and
hence amounts to 'contempt of court' as defined under the Contempt of Courts Act, 19.
Sparsh Upadhyay, "Exercise Restraint And Follow 'Nilesh Navlakha Case' Guidelines On
Media Trial Scrupulously": Bombay High Court To Media (Live Law, 05 March 2021) as
accessed on 07 March 2021.

. Shanaya was a successful actress who committed suicide in her bunglow just a day before
screening of her film. After her death, screenshots of her chats with Faizaan, her co-actor in
the movie spread like wildfire. Various news channels exploited the wave of sympathy to
raise their TRP and conducted debate on whether Shanaya committed suicide because of her
affair with Faizan, who was already married. Many channels targeted Faizaan on his alleged
affair with Shanaya and alleged him to be the reason Shanaya committed suicide. Both
Shanay’s brother and Faizaan had made an application before the Delhi High Court to
restrain news channels from broadcasting any judgments on the case, while it is still under
investigation. Media houses, however, are defending that they have been broadcasting what is
already in public domain. Choose the correct option?

(a) Application shall not be allowed as the information is already in public domain and the
news channels were not the first ones to publish it, thus they cannot be restrained from
broadcasting on the case.

(b) Application shall be allowed as the reporting is defamatory to the reputation of both
Faizaan and Shanaya, and holds Faizaan guilty while the case is under investigation.

(c) Application shall not be allowed as the news channels are raising mere questions
regarding the cause of the Shanaya’s death which would aid in the investigation rather than
interfering with it.

(d) Application shall be allowed as the reporting amounts to media trial of a sensitive
criminal matter and interferes with the administration of justice.

. Assertion: Courts can restrain media houses from broadcasting any sensitive information
about a case under investigation. Reasoning: Such information will be defamatory to the
parties involved in the case. Choose the correct option:

(a) Both are correct and R is the correct explanation of (A)

(b) Both are correct but R is not the correct explanation of (A)

(c) A is true, R is false.

(d) A is false, R is true.

. Kunal Singh was one of the three popular Singh’s in the Bollywood industries, known as the
trinity of the film industry. One day, he was found dead in his apartment. ‘News Forever’ a
news broadcasting channel, disclosed videos of Kunal being with Sunil Singh the other one
of the trinity, a night before his death. The channel further reported the growing competition
between the two actors in the recent year due to which there was tension between the two
actors. Sunil has made an application before the Sessions Court to restrain News Forever
from connecting the case with his name and passing any judgment on him when the case is
under investigation. The court has held the reporting to be an interference with the
administration of justice as it changes the direction of investigation and imposed a restraint
on further reporting on the case. Which of the following is the correct option with respect to
the order of the court?

(a) The order has been rightly passed as the channel was reporting on the sensitive matters of
the case which amounts to interfering with the administration of justice.
(b) The order has been wrongly passed as the channel was merely reporting on the case and
life of the actor and broadcast did not imply any connection of Sunil with case of Kunal’s
death.

(c) The order has been rightly passed as no news channel shall be allowed to report on
delicate and complex matters such as the present one.

(d) The order has been wrongly passed as the same is violative of the right to trade and
profession and freedom of press as granted by article 19 of the Indian constitution.

. The entire nation was shook when a 14 years old girl was brutally raped and then murdered
in Noida, U.P. The family of the girl is enraged by the act and is protesting before a police
station for proper actions against the accused. Meanwhile, a news reporter managed to get a
video of the girl’s corpse and the one news channel broadcasted the entire video on the prime
time show. Although the video was broadcasted the entire video after a trigger warning, the
family of the accused is demanding that the video be not shown on TV anymore and the news
channel be prevented from publishing anymore of such content. News channel has opposed
the application stating that the video is being shown only post a warning and people exercise
their own will by choosing to watch it. Decide whether application shall be allowed or not.

(a) The application shall be allowed as publishing video of the deceased girl amounts to
reporting on sensitive criminal matters, which is prohibited by the order of Bombay High
Court.

(b) The application shall not be allowed as the channel has warned the viewers about the
triggering nature of the video and thus has attempted to avoid triggering people by its content.

(c) The application shall be allowed as news channels shall be considerate towards the
sufferings of the families of the victims prior to reporting any illicit content regarding a case.

(d) The application shall not be allowed as the channel broadcasted the video in public
interest to make people aware of the complete incident without leaving out any details.

. Recently, when Kamlesh Thadani, a business tycoon, died, all his property was inherited by
his ex-wife Salma. Police after the due investigation concluded that the reason for death is
natural causes however they are exploring murder as one of the options. Dastak, a
newschannel, boradcasted a television programme completely dedicated to case of Kamlesh’s
murder. The programme showed how police has failed to consider Salma as the primary
accused, as she is the only one who could benefit from Kamlesh’s death. Salma is now facing
a lot of resentment from the public in general for killing her own husband for money. After,
an application by Salma court has directed Dastak to refrain from broadcasting any such news
any further else they would be charged for contempt. Dastak has filed an appeal against the
order. Choose the correct option.

(a) Appeal shall be allowed as the channel is not interfering with the ongoing investigation
but aiding to explore the new dimensions to the case.
(b) Appeal shall not be allowed as the channel has been interfering with the investigation by
reporting on the case and assuming facts on the ongoing investigation.

(c) Appeal shall be allowed as the reporting by the channel has no proximate cause with the
resentment that Salma is receiving from the public and the channel has no control over the
views of the public at large.

(d) Appeal shall not be allowed as reporting by the channel is defamatory to Salma and no
person is guilty of an offence until proven so in Indian law.

Passage (Q.-Q.): The bench comprising Justices AM Khanwilkar, Indu Malhotra and Ajay
Rastogi of SC observed that the condition predicated in Section 31 of the Foreign Exchange
Regulation Act, 19, of obtaining "previous" general or special permission of the Reserve
Bank of India for transfer or disposal of immovable property situated in India by sale or
mortgage by a person, who is not a citizen of India, is mandatory. The bench clarified that the
transactions which have already become final including by virtue of the decision of the court
of competent jurisdiction, need not be reopened or disturbed in any manner because of this
pronouncement. Referring to Section 31 of the Act, the bench held that that the requirement
of taking "previous" permission of the RBI before executing the sale deed or gift deed is the
quintessence; and failure to do so must render the transfer unenforceable in law. "It is well
established that a contract is void if prohibited by a statute under a penalty, even without
express declaration that the contract is void because such a penalty implies a prohibition.
Further, it is settled that prohibition and negative words can rarely be directory. In the present
dispensation provided under Section 31 of the 19 Act read with Sections 47, 50 and of the
same Act, although it may be a case of seeking previous permission it is in the nature of
prohibition as observed by a three Judge Bench of this Court in MannalalKhetan&Ors. vs.
Kedar Nath Khetan& Ors. In every case where a statute imposes a penalty for doing an act,
though, the act not prohibited, yet the thing is unlawful because it is not intended that a
statute would impose a penalty for a lawful act. When penalty is imposed by statute for the
purpose of preventing something from being done on some ground of public policy, the thing
prohibited, if done, will be treated as void, even though the penalty if imposed is not
enforceable." [Source- ‘Contract Is Void If Prohibited By Statute Under A Penalty Even If It
Does Not Expressly Declare It Void: Supreme Court’, Live Law, , as accessed on 01st March
2021]

. Jatin, a resident of India, had a property in Chennai. He asked his agent, Kunal, citizen of
India, to sell that property for not less than RS. 25 lakh. Kunal informed a broker about the
same and asked him to find such customer who is interested to buy the land in Chennai for
Rs. 30 lakh as opening price. Broker contacted Rohan, a non-citizen of India, and tells him
about the deal. He showed his interest and negotiated to buy the property at Rs. 28 lakh. No
such prior permission was taken by the RBI before the sale deed concluded. Decide-

(a) The sale deed concluded between Rohan and Jatin require prior permission of RBI as
Jatin is not a citizen of India and thus sale of property is invalid

(b) The sale deed concluded between Rohan and Jatin need not require prior permission of
RBI as Jatin is a citizen of India as resident and citizen means the same and thus sale of
property is valid
(c) The sale deed concluded between Rohan and Jatin require prior permission of RBI as
Rohan is a non-citizen of India and thus sale of property is invalid

(d) None of the above.  

. Shagun and Mehul, non-citizens of India, entered into the gift deed on 20th September 2019
where Shagun agreed to gift her ancestral property in West Bengal to Mehul for consideration
of two famous and populated hotels in Bangalore as soon as the construction work gets over.
The deed is to be executed in month of April 2021. Shagun get to know about the recent
judgment where prior permission of RBI was made mandatory in such sale deeds. As a
lawyer, suggest Shagun-

(a) The gift deed between Mehul and Shagun is held to be invalid as prior permission of RBI
is necessary at time of execution of the deed

(b) The gift deed between Mehul and Shagun is held to be valid as prior permission of RBI is
not necessary at time of execution of the deed

(c) The gift deed between Mehul and Shagun is held to be valid as prior permission of RBI is
necessary only in cases of sale deed and not gift deed

(d) The gift deed between Mehul and Shagun is held to be invalid as prior permission of RBI
is necessary at time of entering into the deed

. Pushpa and Sunita were neighbours and they were not in good terms with each other. Sunita
deliberately played loud music at midnight to disturb and cause nuisance to Pushpa so that
she cannot sleep and get sleeping disorders in her routine. Pushpa filed a civil complaint
against Sunita for the same and claimed damages. Sunita had not paid heed to this action of
Pushpa and had not filed her written statements in the court within the prescribed limit of 30
days. Further, she has also not filed the same within the extended period of days. Prosecution
lawyer claims such provision Order VIII Rule 1 of Code of Civil procedure which states “the
defendant shall, at or first hearing or within such time as the court may permit, present a
written statement of his defense”, is mandatory in nature. Decide-

(a) The contention is incorrect as Order VIII Rule 1 of CPC has not provided any prohibition
or penalty for non-compliance; it is only directory in nature.

(b) The contention is correct as Order VIII Rule 1 of CPC has provided prohibition or penalty
for non[1]compliance; it is mandatory in nature.

(c) The contention is incorrect as Order VIII Rule 1 of CPC has not provided any prohibition
and it depends upon case to case basis whether it is directory or mandatory in nature.

(d) The contention is correct as Order VIII Rule 1 of CPC has been enacted with a lawful
purpose to compensate for wrong and thus written statements has to be filed within the time
limit. It is mandatory in nature.

 
. Manu and Kamu were best friends and were much interested in earning money from short
temporary sources. They get to know about the horse race betting and entered into a wager
where the losing party will give the other party cash reward of Rs. 20,000. Manu won the bet
and claimed Kamu the decided amount. Kamu refused to pay the same and stated that under
Indian Contract Act, Wagering agreements are void per se and Manu cannot claim anything
out of such agreement via help of court procedures. Manu asked his lawyer and was
consulted. As his lawyer, you would suggest-

(a) The agreement was void under Indian Contract Act as agreements in form of wager are
not recognised as lawful and is prohibited

(b) The agreement was not void under Indian Contract Act as agreements in form of wager
are not recognised as unlawful and is not prohibited

(c) The agreement was void under Indian Contract Act as agreements in form of wager are
not recognised as lawful but is not prohibited

(d) The agreement was voidable under Indian Contract Act as agreements in form of wager
are not recognised as unlawful but is prohibited

. Which among the following provision(s) is/are not mandatory in nature?

I. Section 1 of Cr.P.C states that no court shall take cognizance of any offence alleged to have
been committed by a public servant, judge, magistrate, or member of the armed forces.

II. Rule 27 of the Uttar Pradesh Forest Service Rules 12 states that if the Governor is satisfied
that the operation of any rule regarding conditions of service of the members caused undue
hardship in a particular case, he may consult the Public Service Commission. 22 of 40

III. Rule 47 of the Representation of the People 11 provided that a ballot paper shall be
rejected if it is spurious or if it was so damaged or mutilated that its identity as a genuine
ballot paper could not be established.

IV. Article 124(2) of Indian Constitution states that Every Judge of the Supreme Court shall
be appointed by the President by warrant under his hand and seal after consultation with such
of the Judges of the Supreme Court and of the High Courts in the States as the President may
deem necessary for the purpose and shall hold office until he attains the age of sixty five
years.

(a) Only I (b) Both II and IV (c) Only II (d) Both III and IV

Passage (Q.-Q.): In a significant verdict, the Supreme Court on Tuesday held that consumer
complaints filed before the coming into effect of the Consumer Protection Act 2019 (CPA
2019) should continue in the fora in which they were filed as per the pecuniary jurisdiction
under the previous Consumer Protection Act of 19 (CPA 19). This significant
pronouncement, which settles a widespread confusion prevailing in consumer fora across the
country, came in the case of NeenaAneja and others vs. Jai Prakash Associates Ltd. The 2019
Act had increased the pecuniary jurisdiction of consumer fora as follows- District Forum-
Increased to Rs. One Crore from Rs. Twenty Lakhs. State Commission- Increased to Rs. Ten
Crores from Rs. One Crore and for National Commission- Above Rs. Ten Crores from Rs.
One Crores. Section (2) has saved "the previous operation" of any repealed enactment or
"anything duly done or suffered there under to the extent that it is not inconsistent with the
provisions of the new legislation". Section (3) indicates that the general application of
Section 6 of the General Clauses Act is not prejudiced. Section 6 of the General Clauses Act
provides governing principles with regard to the impact of the repeal of a central statute or
regulation. These governing principles are to apply, "unless a different intention appears".
Clause (c) of Section 6 inter alia stipulates that repeal would not affect "any right, privilege,
obligation or liability acquired, accrued or incurred under any enactment so repealed". Clause
(e) of Section 6 ensures that a legal proceeding which has been initiated to protect or enforce
"such right" will not be affected and that it can be continued as if the repealing legislation has
not been enacted. The Court observed that it would be difficult to attribute to Parliament,
whose purpose in enacting the Act of 2019 was to protect and support consumers, with an
intent that would lead to financial hardship, uncertainty and expense in the conduct of
consumer litigation. [Source- Manu Sebastian, ‘All Consumer Complaints Filed before CPA
2019 Should Be Heard by Fora as per Pecuniary Jurisdiction under CPA 19: Supreme
Court’’, Live Law, , as accessed on 15th March 2021]

. Manisha was a regular customer of Bikajilal Shop in Delhi. She trusts the shopkeeper and
orders her purchase items on telephonic calls only. One day she ordered for edible food items
amounting to Rs.30,00,000. But the shop keeper made her a bill of Rs.58,00,000 and asked
his worker to collect the amount first and then deliver the product. The worker asked for the
payment but Manisha refused to do so and argued for extra Rs.28, 00,000 in the bill. She
claimed that the shop keeper is taking undue advantage of her trust and she decided to file a
complaint in consumer court on 12th September 2020. She can-

(a) File a complaint against Bikajilal Shop owner in State Commission as it has the pecuniary
jurisdiction over the matter and complaint has to be filed there.

(b) File a complaint against Bikajilal Shop owner in District Forum as it has the pecuniary
jurisdiction over the matter and complaint has to be filed there.

(c) Cannot File a complaint against Bikajilal Shop owner in State Commission as the ground
for filing the complaint is frivolous and she cannot do so.

(d) File a complaint against Bikajilal Shop owner in both District forum and State
Commission as both have the pecuniary jurisdiction over the matter.

. Mr. Ratnani was owner of largest Cotton Manufacturing Company of India. A suit was filed
long back in year 2015 against the company for claiming the cotton to be smooth and iching
free. But the quality of cotton was not matched with the market standards. The opposite party
was claiming the compensation of amount Rs.1,10,00,000 as his clothing company suffers
loss due to his bad quality cotton. In pendency of proceedings it was discovered that Mr.
Ratnani has profited himself with adulterating the cotton and mixing it with poor quality one.
The consumer party added new amount of Rs. 50,00,000 more for claiming compensation.
Decide-
(a) The pecuniary jurisdiction will be shifted from National Commission to State
Commission as limits have been amended in the new act of 2019.

(b) The pecuniary jurisdiction will not be shifted from National Commission to State
Commission as though the new act of 2019 has no retrospective effect.

(c) The pecuniary jurisdiction will be shifted from National Commission to District Forum as
limits have been extended beyond twenty lakh.

(d) The pecuniary jurisdiction will not be shifted from National Commission to State District
Forum as no limits have been amended in the new act of 2019.

. Mr. Prabhu Ramchandani was owner of largest Coffee Manufacturing Company of India
having a subsidiary in UK. A suit was filed long back in year 2010 against the company for
claiming the coffee to be of fine quality and coco rich content. But the quality of coffee was
not matched with the market standards. The opposite party was claiming the compensation of
amount Rs.7,00,00,000 as his coffee powder company suffers loss due to his bad quality
coffee. In pendency of proceedings it was discovered that Mr. Prabhu has profited himself
with adulterating the coffee and mixing it with poor quality one. The consumer party filed a
suit against the subsidiary company claiming amount of Rs. 9,00,00,000 more for
compensation. Decide-

(a) The pecuniary jurisdiction will be of State Commission only as limits have been amended
in the new act of 2019.

(b) The pecuniary jurisdiction will be shifted from National Commission to State
Commission as though the new act of 2019 has no retrospective effect.

(c) The pecuniary jurisdiction will be of both National Commission and District Forum as
limits have been extended beyond twenty lakh.

(d) The pecuniary jurisdiction will be shifted from National Commission to District Forum as
no limits have been amended in the new act of 2019.

. The Court shall be competent to entertain the proceedings. The Competency is legally
termed as jurisdiction. Pecuniary Jurisdiction is based upon the valuation of a subject matter
of the suit. With regards to territorial jurisdiction of the court, several other factors with
regard to recovery, rent, partition, sale, redemption, determination of right of immovable
property, it shall be instituted in the court within the local limits of whose jurisdiction the
property is situated. On the other hand, Subject matter jurisdiction relates to the subject
matter of the suit without its reference to the pecuniary valuation or a territorial jurisdiction
of the subject matter. In the exercise of Original Jurisdiction, a court of first instance decides
suits, petitions, application etc. Appellate Jurisdiction refers to the court’s authority to review
or re-hear the cases that have been already decided in the lower courts by way of any appeal,
revision etc. Choose among the following which is best suitable example of only pecuniary
jurisdiction of the trial court?
I. Family court of Jaipur dealing with the case of dowry death involving amount of Rs.
5,00,000 and domestic violence under sec.4 of IPC.

II. Criminal Court of Delhi convicting the accused for theft of Rs. 20,00,000 and charged
under POCSO for attempt to rape of a minor girl.

III. Cuttack District Forum dealing with a dispute between manufacturer and consumer
claiming compensation of Rs. 15,00,000. Appeal filed to state commission in Bhubaneswar.

IV. Nagpur Civil court of second class magistrate dealing cases of trivial matters up to sum of
Rs. 2500 and other petty cases of civil nature.

(a) Both I and III (b) Only IV (c) Both III and IV (d) Only II

. Choose the most appropriate option[1]

Assertion (A): The amendment in Consumer Protection Act 19 was brought by CPA 2019
which changed the pecuniary jurisdiction of respective forums keeping National Commission
to be the highest authority.

Reasoning (R): The parliament intended the amendment to be with retrospective effect so that
the public do not suffer any financial hardships and uncertainty in filing consumer litigations.

(a) Assertion is true but Reasoning is false.

(b) Both Assertion and Reasoning is false.

(c) Both Assertion and Reasoning is true and Reasoning is correct explanation of Assertion.

(d) Assertion is False but Reasoning is true.

(e) Both Assertion and Reasoning is true but Reasoning is not the correct explanation of
Assertion.

Passage (Q.-Q.): The Centre has opposed any move to accord legal sanction to same-sex
marriages in India. Decriminalization of Section 3 of the Indian Penal Code on 7 Sep 2018
does not automatically translate into a fundamental right for same-sex couples to marry, the
Centre’s counsel told the Delhi High Court in an affidavit on Thursday. The Supreme Court’s
ruling granted same-sex couples the freedom to lead a dignified private life, but allows them
only “basic right to companionship so long as such companionship is consensual, free from
the vice of deceit, force, coercion, and does not result in the violation of fundamental rights
of others,” it said, citing the court ruling in the Navtej Singh Johar case. The ruling applied to
“personal private domain of individuals akin to the right to privacy and cannot include a
public right like recognition of same-sex marriage and thereby legitimizing a particular
human conduct,” In case of issues concerning personal relationships like marriage, divorce,
adoption, maintenance, etc., either the codified law or the personal law occupies the field, it
said, suggesting that laws are enacted by Legislature’s wisdom in a manner concomitant with
“societal values” and “national acceptability”. “The fundamental right under Article 21 is
subject to the procedure established by law and the same cannot be expanded to include the
fundamental right for a same-sex marriage to be recognized under the laws of the country
which mandate the contrary.” The institution of marriage had a certain sanctity attached to it
across the country, it added. It enjoins the parties belonging to different sexes to bring up
children in the “most natural way possible”. Seeking a declaration recognizing same-sex
marriages has more legal ramifications than simple legal recognition. The parties to same-sex
marriage would not fit the legislative scheme which talks about a biological woman who is
the wife and a man who is the husband, it said. Source: THE ECONOMIC TIMES Feb 25,
2021, 09:21 PM https://economictimes.indiatimes.com/news/politics-and-nation/same-sex-
partners-not-comparable-with[1]indian-concept-government/articleshow/2028.cms

. Vibhi and Vaibhav are sharing the same room in the boy's hostel since 2020. They both
have emotional and physical intimacy between them and confessed their relationship status
publicly which offended the hostel administration, in return the administration forcefully
parted them in the hostel by changing their room. Is Hostel administration is legally right?

(a) No, the Hostel administration can’t force anyone to change room because all are paying
equally therefore they have the right to enjoy freedom.

(b) Yes, hostel administration is legally right because Gay relationship is against “societal
values” and “national acceptability in India.

(c) No, The Supreme Court’s ruling granted same-sex couples the freedom to lead a dignified
private life and allows them the basic right to companionship.

(d) No, “The fundamental right under Article 21 is subject to the procedure established by
law and the same cannot be expanded to include the fundamental right for a same-sex
marriage to be recognized under the laws of the country which mandate the contrary.

. considering the above facts now Vibhi and Vaibhav wanted to marry to avoid any societal
pressure and they read the Supreme Court judgment of decriminalizing section 3 after that
decided to marry nearby court. Decide their decision is legally acceptable with appropriate
reasoning.

(a) No, a cultured country like India will never accept the marriage of the LGBTQ
community it is against moral principles and customary laws. (b) No, Decriminalisation of
Section 3 of the Indian Penal Code does not automatically translate into a fundamental right
for same-sex couples to marry.

(c) Yes, we have the fundamental right to freedom to marry with our own choice, and the
constitution of India gives equal rights to all its citizens.

(d) No, it is against the order of nature.

. Rachel and Monica are consensually living in Bombay and having intercourse with the
consent of each other but after 1 week Rachel is threatening Monica by knife and trying to
make a physical relationship without her consent. Threatened by the conduct of her partner
she doesn’t want to live with her but Rachel is forcing her not to leave the premises. Do they
have the right to leave together? Answer considering the facts decided with legal reasoning.

(a) Yes they have every right to be in companionship because Section 3 is unconstitutional.

(b) No, they have a basic right to companionship so long as such companionship is
consensual, free from the vice of deceit, force, coercion, and does not result in the violation
of fundamental rights of others. But there is a violation of rights of Monica.

(c) Yes, they have a basic right to companionship so long as such companionship is
consensual, free from the vice of deceit, force, coercion, and does not result in the violation
of fundamental rights of others. But there is a violation of rights of Monica.

(d) It is their matter we should not bridge their right to privacy by interfering in their life.

. David and John are in a physical relationship in 2017 and publicly kissing and setting in an
intimate position in the garden. Are they having the right to do so answer considering
passage?

(a) Yes, Decriminalisation of Section 3 of the Indian Penal Code granted same-sex couples
the freedom to lead a dignified private life.

(b) Yes, The Supreme Court’s ruling granted same-sex couples the freedom to lead a
dignified private life.

(c) No, Gay relationship is against “societal values” and “national acceptability in India.

(d) No, in 2017 the section 3 of the Indian penal code is constitutional. Therefore same-sex
couples don’t have the right to be in a physical relationship.

. X and Y are in a same-sex relationship to marry each other X who is a man decided to dress
as a woman and soon developed a womanish way of living and considered him the wife of Y.
Are they legally fit in the legislative scheme of marriage?

(a) No, same-sex marriage is against the order of nature.

(b) No, would not fit the legislative scheme of marriage which talks about a biological
woman who is the wife and a man who is the husband.

(c) Yes, would fit the legislative scheme which talks about a woman who is the wife and a
man who is the husband.

(d) None of the above.

 
Passage (Q.-Q.): The Punjab & Haryana High Court on 10th March registered its disapproval
of 'new concept of contractual Live-In-Relation' backed by a deed, wherein parties state that
their live-in[1]relationship is not 'Marital Relationship'. The Counsel for the petitioners
submitted that Petitioner No.1 Moyana Khatun (aged 18 years) and Petitioner No.2 Labh
Singh (aged about 19 years) executed a deed of Live-In-Relationship dated 04th March 2021
and certain terms and conditions have been settled in the deed of live-in-relationship by way
of mutual consent. The said deed specifically stated that both the parties have agreed that
their live-in-relationship is not a 'Marital Relationship' and further, the parties will fully
cooperate with each other without any dispute and issue and will not claim anything against
each other. The said deed also stated that if any of the parties backs out from the aforesaid
deed, the other party will have a right to approach a competent Court of law for the
implementation of the same and that, in the end, on attaining marriageable age the parties
agreed to solemnize marriage. The Counsel for the states opposed the prayer of the
petitioners on the ground that such deed of live-in[1]relationship is impermissible in law
when the parties have not attained the age of performing marriage under the Prohibition of
Child Marriage Act, 2006. It was further submitted that Section 5 (iii) of the Hindu Marriage
Act, 15 prohibits marriage of a girl below 18 years and boy below 21 years of age. It was also
contended that Section 26 of the Indian Contract Act, 18 provides that an agreement in
restraint of marriage is a void agreement and therefore, it cannot be enforced as per Section
14 of the Specific Relief Act, 19. It was thus, submitted that the Live-In-Relationship
agreement set up by petitioners being void agreement cannot be accepted.

. Avantika is junior to Prateek in college. After pursuing Prateek for a year, they eventually
fell in love and now wish to get married. Both Avantika and Prateek belong to two different
communities and their parents oppose their union. When their parents met, they got into
serious argument. As a resultant, Avantika’s parents made her sign a deed that she will marry
anyone but Prateek. The agreement was drafted by a lawyer and was duly notarized. In this
case-

(a) Avantika and Prateek cannot legally marry each other because of the ongoing contract.

(b) Avantika and Prateek can marry each other but it will be considered illegal.

(c) Avantika and Prateek can marry legally and the contract will be considered as void.

(d) The parents could have allowed Avantika and Prateek to marry which would have
promoted national integrity.

. Neil agrees to pay Anshul a sum of Rs.1 lakh if Anshul marries Kavita within a period of six
months. Anshul could marry Kavita only during the seventh month as the marriage
ceremonies were prohibited due to covid 19 lockdown. Anshul claims Rs.1 lakhs from Neil.
Was the contract in restraint of marriage?

(a) The contract between Neil and Anshul was in restrain and was invalid.

(b) The contract between Neil and Anshul was not in restraint of marriage.

(c) The contract between Neil and Anshul was not in restraint of marriage as six months
duration was mentioned in the contract.
(d) The contract did not restrict Anshul to marry another girl, therefore it was not in restraint
of marriage.

. Chinamma makes three deals with her three sons. She agrees to pay the first son a lakh if he
marries Lakshmi. She agrees to pay the second son a lakh if he doesn't marry Saraswati. She
agrees to pay the third son a lakh if he marries a Hindu girl. Which among the following is a
valid deal?

(a) Deal with 1st Son.

(b) Deal with 2nd son.

(c) Deal with 3rd son.

(d) All deals are invalid

. Dashrath enters into an agreement with Janak for marrying Sita, Janak’s daughter, who was
15 years of age. Janak promises to pay a sum of Rs.2, 00,000/- and advances a sum of
Rs.25,000/- to Dashrath. On the day of the marriage, Janak refuses to pay the remaining sum
of Rs.1,,000/- to Dashrath. Are such marriages permissible?

(a) The contract between Dashrath and Janak are illegal.

(b) The marriage of a girl below 18 years is prohibited under the legal provisions.

(c) Dashrath and Janak would face legal action for getting into such immoral contracts.

(d) Such marriages are permissible only with the consent of Sita

. Bimal and Beena are high school sweethearts. They couldn’t marry because Beena is just 17
year old now. So Bimal and Beena signed a live-in relationship agreement. The clauses
mentioned their live-in partnership till they get married. Bimal to formally propose Beena
with a diamond ring on the inland cliff and their wedding to take place in Palace in Udaipur.
The agreement also mentions to not involve their respective parents throughout their
relationship and have no off spring in future. They seek your advice on the validity of their
agreement as per the law.

(a) The agreement will be notarized as it is duly signed with consent of Beena and Bimal.

(b) The object of the agreement was illegal so Bimal will not be able to get it notarized.

(c) Such agreements are considered void by the Apex Court.

(d) The agreement is valid and binding, as it was not is restrain of the marriage.

. In the above question, If Bimal made an agreement with Beena that he will not marry
anybody else but Beena if she can wait for him for 3 more years, so that he gets enough time
to establish his business and she can finish her graduation. Beena waited for 3 years and
when she asked Bimal to marry her, he refuses. Decide.

(a) The contract is void, as it restrains Bimal from marrying anybody else than Beena.

(b) The contract is void as Beena didn’t pay any money to make the contract valid.

(c) The contract is valid, as it is not in restraint of marriage.

(d) None of the above.

Passage (Q.-Q.): The long title of the new Consumer Protection Act, 2019 in the least number
of words explains the whole and sole purpose of the Act. Key features of the Consumer
Protection Act, 2019 “Advertisement” is defined as any audio or visual publicity,
representation, endorsement or pronouncement made by means of light, sound, smoke, gas,
print, electronic media, internet or website and includes any notice, circular, label, wrapper,
invoice or such other documents. • A provision for a minor being a consumer has been
introduced where the parent or legal guardian can approach the authorities through the minor
seeking relief. • “consumer” is defined as a person who “buys any goods for a consideration
which has been paid or promised or partly paid and partly promised, or under any system of
deferred payment and includes any user of such goods other than the person who buys such
goods for consideration paid or promised or partly paid or partly promised, or under any
system of deferred payment, when such use is made with the approval of such person, but
does not include a person who obtains such goods for resale or for any commercial purpose”
Thus, a consumer will now mean any person who “buys any goods” and “hires any services”
which shall include both online and offline transactions through electronic means,
teleshopping, direct selling or multi-level marketing. • The new additions include “e-
commerce” “electronic service provider” along with the prescribed liabilities in relation to
internet frauds. • under Section 2(34) of the Consumer Protection Act, 2019 as “the
responsibility of a product manufacturer or product seller, of any product or service, to
compensate for any harm caused to a consumer by such defective product manufactured or
sold or by a deficiency in services relating thereto;” Source: SSC Online Blog

. Murphy 17 year CLAT aspirant wants to crack Clat 2021 but due to coronavirus unable to
join offline coaching center therefore he has taken subscription of online clat course of
10,000 rupees from unknown clat preparation website named as” Guaranteed success”.Even
after the payment and waiting for a long time the “Guaranteed success” not given him study
material that was promised while taking a subscription. Can Murphy file a complaint against
“Guaranteed success” under the consumer protection act 2019? Answer with Appropriate
reasoning.

(a) No, Murphy cannot file a complaint against Guaranteed success because the consumer
should be aware of dire consequences before involving in such activities while taking
subscriptions from illegal and unknown sources.

(b) Yes, Murphy can file complaints as it the responsibility of a product manufacturer to
compensate for any harm caused to a consumer by such defective product manufactured or
sold.

(c) Yes, Murphy can file complaints through a legal guardian as it is the case of internet fraud
by online service provider Guaranteed success.
(d) Yes, Murphy should file a complaint as being an Indian citizen and aware consumer he
has every right to get compensations from fraud sellers.

. Beautylovely is cosmetic cream manufacturing company promoting their product through


ambassador Nami Gautam via online advertisement and vouch for the fair and glowing face
for all skin types within 1 week. Millions of Indian women purchased Beautylovely by seeing
Persuasive advertising based on the claims of the advertiser. But none of them received fair
and glowing faces even after using for 1 month. Saanvi law student aggravated by such
misleading advertisement and now want to seek relief can she do so? (a) Yes, Saanvi is
eligible to seek relief from Beautylovely because the company wasted her money and crucial
time.

(b) No, Saanvi cannot seek relief as it was not a misleading advertisement by the company
because it is presumed that beauty products don’t have % result and layman have knowledge
about what to purchase and what not.

(c) Yes, Saanvi can get relief because this comes under misleading advertisement defined
under section 2 of the Consumer Protection Act, 2019.

(d) None of above

. Beena Devi is selling sarees to women in her locality therefore is ordered 12 sarees directly
from the manufacture via WhatsApp and after delivery of the product found 2 sarees
damaged and when requested for exchange manufacturer denied. Can Beena Devi file a
complaint against the manufacturer under the consumer protection act 2019. Explain with
reasonable reasoning.

(a) Yes, because it is the responsibility of a product manufacturer or product seller, of any
product or service, to compensate for any harm caused to a consumer by such defective
product manufactured.

(b) Yes, being a consumer it is her right to get qualitative and safe delivery of the product.

(c) No, she can't sue the manufacturer because she is not the consumer according to the
definition of consumer in the Consumer Protection Act of 2019.

(d) No, she can't sue the manufacturer because she ordered sarees online and online
transactions not covered under the Consumer Protection Act of 2019.

. Saanvi aggrieved by misleading advertisement went to consumer court for filling complaint
but denied by the judge. Can she claim the right to be heard? Answer considering the
passage.

 
(a) yes, the consumer has the right to be heard and to be assured that the consumer’s interests
will receive due consideration at appropriate fora.

(b) No, she can't claim the right to be heard.

(c) yes, she has every right to file a complaint against anyone according to her wish.

(d) None of the above.

. We had The Consumer Protection Act 19 and worked effectively for 3 decades so what can
be reasons for amendments in the existing act and bringing new Consumer Protection Act of
2019.Answer with reasonable reasoning.

(a) Being around three decades old, did not inculcate the needful things that would have
solved the problems of the modern and technology-dependent consumers, which is why a
need was felt to replace the whole Act with a new one and bring a fundamental change.

(b) Democratic countries should always change law according to the dynamic nature of
society to maintain rule of law.

(c) Both A and B

(d) None of the Above

28
Passage (Q.-Q.):Necessity as a defence is defined under section in Indian Penal Code as:
“Act likely to cause harm, but done without criminal intent, and to prevent other harm.
Nothing is an offence merely by reason of its being done with the knowledge that it is likely
to cause harm, if it be done without any criminal intention to cause harm, and in good faith
for the purpose of preventing or avoiding other harm to person or property.” The explanation
of the same is that it is a question of fact in such a case whether the harm to be prevented or
avoided was of such a nature and so imminent as to justify or excuse the risk of doing the act
with the knowledge that it was likely to cause harm. The defence of necessity requires the
defendant to prove on his part that the unlawful act committed by him satisfies the following
elements:

· It was not a result of his own negligence;

· S/he acted to avoid a significant risk of harm;

· No other alternative lawful means were available;

· Harm avoided was greater than the harm which resulted due to his act;

· The harm was imminent and the act was reasonable.

[Extracted with revisions from ‘Necessity – public and private’ by Shantanu Rathore
https://www.lawctopus.com/academike/necessity-public-and-private/]

 . Ram Lorry Services was a company which provided transport services for goods all over
the country. Pyare Lal was a truck driver employed with Ram Lorry Services and he was
transporting a consignment of bananas from Kerala to Punjab. The shelf life of bananas is
very limited and so they must reach the seller within 10 days of being dispatched. Pyare Lal
was scheduled to make a timely delivery but on the way he encountered a large traffic jam.
All roads from Haryana onwards were blocked due to a religious procession. Since the
bananas would have become rotten by the time Pyare Lal reached Punjab, he decided to sell
the consignment to the nearest seller he could find in Haryana. When his employer came to
know he got angry at Pyare Lal because the seller in Punjab was scheduled to get the
consignment and he would've paid a higher price. Can Pyare Lal take the plea of necessity?

(a) Yes, because it was reasonable and necessary to sell the consignment of bananas to save
them from getting rotten

(b) Yes, because the traffic jam was long and would have wasted a lot of time

(c) No, because there was no imminent risk to the consignment if the bananas were not sold

(d) No, because there was a contract to sell the bananas only to the seller in Punjab

. Shyam Lal was also a truck driver employed by Ram Lorry Services. He was carrying a
consignment of winter clothes from Uttar Pradesh to Srinagar. On the way he encountered a
cold weather storm and as a result, the temperature suddenly dropped to minus degrees.
When he was about to reach the retailer who had contracted to buy the winter clothes, Shyam
Lal saw several poor and destitute people sitting on the road, many of whom were women
and children. Shyam Lal decided to distribute half of the winter clothes to them. In doing so,
he violated the contract with the retailer. Can Shyam Lal take the defence of necessity to
justify his acts?

(a) Yes, because he distributed the clothes in good faith without any criminal intention

(b) Yes, because the harm avoided by distributing warm clothes to the needy was greater than
the harm which resulted to the contracting parties due to his act

(c) No, because it was neither necessary nor reasonable to distribute clothes which had
already been contracted to someone else due to a change in temperature

(d) No because although the harm was imminent due to the cold weather, the act of
distributing half of the consignment of winter clothes for free was reasonable

. A ship was carrying passengers on board as well a batch of COVID-19 vaccines. Before
reaching its destination, the ship had a minor accident and started to sink. In order to stop it
from sinking, it was important to reduce some of the load it was carrying otherwise everyone
on board could have died. The captain decided to unload the precious COVID-19 vaccines
off the ship by throwing large crates of the vaccines into open sea. This caused a loss of
tremendous amount of money, resources and already limited vaccines along with causing
environmental damage. However, with this move the ship finally became light enough to
restart the engine and get it moving to reach the nearest port. Can the captain take the defence
of necessity?
(a) Yes, because it was necessary to throw the vaccines into water even though the harm was
not imminent in this case

(b) Yes, because the harm to human lives was greater than the harm which resulted as a
consequence of throwing the vaccines into water

(c) No, because the harm to be prevented was not of such a nature and so imminent as to
justify the unloading of precious vaccines

(d) No, because alternative lawful means were available and he could have waited for rescue
ships to save all the passengers as well as the vaccines

. Lakhan lived in a hut next to a big mansion which had an attached well. The owner of the
well did not allow anyone to step foot on his property. One day, Lakhan’s hut caught on fire
and all his valuable and precious material was inside. The nearby well in the mansion was the
closes source of water. Without giving it a second thought, Lakhan hurriedly drew several
buckets of water from that well to douse the fire and save his hut. When the fire was finally
put out, the owner of the mansion charged Lakhan with “trespass of property” because he
used water from his well without his permission. Can Lakhan plead the defence of necessity?

(a) No, because lawful alternatives were available and it is irrelevant that the well was the
closest source of water

(b) Yes, because Lakhan acted to draw water from the well to avoid a significant harm

(c) Yes, because there was no alternative available as the well was the closest fsource of
water which could be used to douse the fire

(d) Both b and c

Passage (Q.68-Q.): Section 4 of the Contract Act, 18 deals with the completion of a proposal,
acceptance and revocation of a contract and states: The communication of a proposal is
complete when it comes to the knowledge of the person to whom it is made. The
communication of an acceptance is complete, as against the proposer, when it is put in a
course of transmission to him so as to be out of the power of the acceptor; as against the
acceptor, when it comes to the knowledge of the proposer. The communication of a
revocation is complete, as against the person who makes it, when it is put into a course of
transmission to the person to whom it is made, so as to be out of the power of the person who
makes it; as against the person to whom it is made, when it comes to his knowledge. In the
case of postal service, the communication becomes complete when the letter is posted from
the offeror and the offeree is bounded by it. But, electronic modes such as fax and telephonic
communications become complete when the acceptance is received by the offeror. In the
Entores case it was held that the postal rule cannot be applied to instantaneous mode of
communications, such as telephone and telex. If a phone line “went dead” just before the
offeree said “yes”, it would be absurd to assume that the contract was formed and the parties
would not have to call each other back. Therefore, the contract could only be formed when
the telex was received. [Extracted with revisions from ‘is section 4 of contract act, 18 in pace
with 21st century’ by AyushPandia, published on scconline blog
https://www.scconline.com/blog/post/2018/03/13/is-section-4-of-contract-act-18-in-pace-
with-21st[1]century]

68. X sends a letter to Y on 2nd January 2021 offering to sell his cow for Rs10,000. Y
receives the letter on 3rd January 2021 proposing to buy the cow for Rs00 instead of
Rs10,000. X received Y’s letter on 4th January 2021 and send his acceptance via mail on 5th
January 2021. The acceptance reaches Y on 6th January 2021. When is the acceptance of the
offer complete against Y?

(a) On 3rd January when Y received X’s letter

(b) On 4th January when X received Y’s letter

(c) On 5th January when it was sent by X to Y via mail

(d) On 6th January when it came to the knowledge of Y via mail

. Sonu and Monu were discussing an offer over the phone. Sonu called Monu to ask him if he
would sell his car to him for Rs 1 lakh. At that time, Monu was at the metro station but he
was interested in selling his car so he continued to discuss the offer with Monu. Monu heard
Sonu’s offer loud and clear and was willing to sell his car to him in exchange for this amount.
However, just as he was confirming his acceptance, a metro train arrived on the station
creating a lot of noise and Sonu could not hear Monu. Is the acceptance of the offer
complete?

(a) Yes, because Monu was willing to accept the offer and he was about to verbally confirm
the same so the noise created by the metro does not matter

(b) No, because the noise created by the metro did not allow Monu to properly communicate
the acceptance to Sonu

(c) No, because the acceptance was not put in the course of transmission to be received by
Sonu

(d) Yes, because the acceptance was already put in the course of transmission by Monu and is
valid against Sonu

. Jalebi Bai accepted an offer by Makkhi Chand to buy a horse for Rs. 20,000. Later, Makkhi
Chand realised that Rs 20,000 was too low of a price and insufficient for selling such a fine
horse. So, he wrote a letter to Jalebi Bai for revocation on 30th May 2020. Independently,
Jalebi Bai thought that Rs 20,000 was a very high price. So, she also independently wrote a
letter to Makkhi Chand on 29th May 2020. Assuming that Makkhi Chand received the letter
on 1st June and Jalebi Bai received the letter on 2nd June, when was the communication of
revocation completed against Makkhi Chand?

(a) On 30th May 2020 when he wrote a letter to Jalebi Bai

(b) On 29th May 2020 when Jalebi Bai wrote a letter to him

(c) On 1st June 2020 when he received the letter by Jalebi Bai 

(d) On 2nd June 2020 when Jalebi Bai received his letter

. Sanjana ran a successful Instagram store and did a majority of her business online. One day,
she was talking to her supplier regarding a contract to purchase some books over a zoom call.
In the background, her young baby was making loud laughing noises while playing with his
toys. The supplier offered to sell the books at Rs 150 each and Sanjana accepted. If Sanjana’s
communication of acceptance was audible to the supplier on zoom but it was marked by loud
noises in the background, would the acceptance of offer be complete?

(a) Yes, because the zoom call line did not go “dead” as required in the Entores case

(b) Yes, because the acceptance was received as it was audible to the supplier even if there
were loud noises in the back

(c) No, because in instantaneous modes of communication the acceptance is complete when it
is received by the offeror

(d) No, because the loud baby noises in the background could have created problems in the
communication of acceptance over zoom

Passage (Q.-Q.): Insults to religion made unwittingly or carelessly without any deliberate or
malicious intention to outrage the religious feelings of a class would not attract Section 2A of
the Indian Penal Code (Sri Dulal Ghosh v. State of Tripura). A single-judge bench of Chief
Justice AkilKureshi, therefore, quashed a First Information Report (FIR) which was filed
against one Dulal Ghosh (petitioner) for a Facebook post on Bhagavad Gita. The Court was
dealing with a case, where FIR was registered against Ghosh under Section 2A of the IPC for
a Facebook post on Bhagwat Gita. The complainant contended that the petitioner- accused
made distasteful and derogatory comments on the Hindu religion by posting on Facebook in
Bengali that the Gita (sacred religious text) is a "thakbaji Gita" (deceitful). The petitioner, on
the other hand, contended that the Facebook post in question was 'deliberately twisted and
misinterpreted by the complainant. He argued that his post did not convey the meaning that
Gita was deceitful or swindling. Instead, the petitioner had put the post conveying that the
Gita is a pan that fries swindler. The Court at the outset placed reliance on the 17 Supreme
Court judgment in Ramji Lal Modi v. the State of UP to elucidate the scope of Section 2A.
"Section 2A does not penalize any and every act of insult or an attempt to insult the religion
or the religious beliefs but it penalizes only those acts of insults or attempts which have been
perpetrated with the deliberate and malicious intention of outraging the religious feelings of a
particular class," the Court said. The expression thus used by the petitioner which is in total
isolation, without virtually any background or foreground, therefore would require much
twisting to fit within the scheme of Section 2A of IPC which would be wholly impermissible,
the Court held. As a parting note, the Court said that "the petitioner can hold his personal
beliefs and within the framework of law can also express them, as long as he does not
transgress any of the restrictions imposed by law to the freedom of his speech and
expression." Source: BAR and BENCH 8 march 2021
https://www.barandbench.com/news/litigation/insults-religion-carelessly-intention-outrage-
religious[1]feelings-will-not-section-2a-ipc-tripura-high-court

. Lodi is giving a speech in his political rally and said the Muslim religion is a hindrance in
the development of India because of their stubborn religious beliefs and rituals. Their govt
will not promote such religious[1]centric activities in the country. Decide Lodi would attract
Section 2A of the Indian Penal Code or not?

(a) No, because the statement by Lodi is made unwittingly or carelessly without any
deliberate or malicious intention.

(b) Yes, said statement has been perpetrated with the deliberate and malicious intention of
outraging the religious feelings of a particular class.

(c) Yes, restrictions imposed by law on the freedom of speech and expression of political
leaders. 

(d) No, the petitioner can hold his personal beliefs and within the framework of law can also
express them.

. Janvi is a content writer in Indian times and given her opine on women rights to inter
Sabarimala temple but her article aggrieved group of people and filled petitions against her
on grounds of the malicious intention of outraging the religious feelings of a particular class.
Will Janvi be liable to decide with legal reasoning?

(a) No, the person can hold his personal beliefs and within the framework of law can also
express them, as long as he does not transgress any of the restrictions imposed by law to the
freedom of his speech and expression.

(b) No, the accused made distasteful and derogatory comments on the Hindu religion.

(c) No, there is malicious intention to outrage the religious feelings of a class that would not
attract Section 2A of the Indian Penal Code.

(d) None of the Above  

. FIR was registered against Ghosh under Section 2A of the IPC for a Facebook post on
Bhagwat Gita. The complainant contended that the accused made distasteful and derogatory
comments on the Hindu religion by posting on Facebook that the Gita (sacred religious text)
is not given rights to lower class people during Mahabharata because Karna had not given an
education. Will Ghosh attract Section 2 A?

(a) No, because he has the right to freedom of speech and expression without restrictions.

(b) No, Insults to religion made unwittingly or carelessly without any deliberate or malicious
intention to outrage the religious feelings of a class.
(c) Yes, distasteful and derogatory comments on the Hindu religion attract Section 2A of the
Indian Penal Code.

(d) Both A and B

. What Supreme Court elucidated in the 17 Ramji Lal Modi v. State of UP judgment?

(a) Section 2 does not penalize any and every act of insult or an attempt to insult the religion
or the religious beliefs but it penalizes only those acts of insults or attempts which have been
perpetrated with the deliberate and malicious intention of outraging the religious feelings of a
particular class,"

(b) Section 2A does penalize any and every act of insult or an attempt to insult the religion or
the religious beliefs but it penalizes only those acts of insults or attempts which have been
perpetrated with the deliberate and malicious intention of outraging the religious feelings of a
particular class,"

(c) The Court said that "the petitioner can hold his personal beliefs and within the framework
of law can also express them, as long as he does not transgress any of the restrictions imposed
by law to the freedom of his speech and expression."

(d) None of above

. Deeksha Ingle Assistant professor of History at National Law University Rewa giving a
brief introduction of Sati practice and said it was the heinous and derogatory practice of the
Hindu religion in the medieval period. Decide she will attract section 2A or not.

(a) Yes,attempts have been made by her with the deliberate and malicious intention of
outraging the religious feelings of the Hindu religion.

(b) No, the petitioner can hold his personal beliefs and within the framework of law can also
express them, as long as he does not transgress any of the restrictions imposed by law to the
freedom of his speech and expression.(c) No,opinion to religion made without any deliberate
or malicious intention to outrage the religious feelings of a class.(d) Both B and C

Passage (Q.-Q.): On February 25, 2021, the Ministry of Information, Government of India
enacted the Information Technology (Intermediary Guidelines and Digital Media Ethics
Code) Rules, 2021. Rule 4 entails that the intermediary is under an obligation to prominently
publish on his website or application, or both, the privacy policy and the usage of personal
data or information for its users. Furthermore, the privacy policy or user agreement should be
drafted in such a way so that it embraces that the user is under the obligation not to host,
display, upload, modify, publish, transmit, store, update or share any information which is
derogatory to the rules of the society or unethical or defamatory to the general public or
misleads the general public. Furthermore, the intermediaries are under the obligation to notify
the users that in case such unethical information is transmitted, then it would call for either
termination of their account or removal of the information that is not in conformity with the
user agreement or privacy policy. The intermediary is to publish on its website or application
or both, as the case may be, the name of the Grievance Officer and his/her contact details as
well as the mechanism by which a user or a victim may make a complaint against violation of
the provisions of Rule 4 or other matters pertaining to the computer resources made available
by it. Moreover, the Grievance Officer is under an obligation to acknowledge the complaint
within three working days and resolve the same within one month from the receipt of the
complaint. Furthermore, an intermediary is under the strict compulsion to remove
information within twenty-four hours on the complaint of a person which portrays such a
person in bad light or exposes their private parts, or shows such a person in a sexual act or
nudity. Regarding the aforementioned burden, a complaint can be filed by an individual or
any person on his/her behalf. Source:https://ssrana.in/articles/analysis-of-the-information-
technology-intermediary-guidelines-and[1]digital-media-ethics-code-rules-2021/?
utm_source=Mondaq&utm_medium

. Webnexe.com is an emerging website in India for sharing thoughts virtually on legal and
political news. What points should take under consideration by Webnexe.com to avoid
violation of IT rules 2021?

(a) Under an obligation to prominently publish on his website or application, or both, the
privacy policy and the usage of personal data or information for its users.

(b) Under the obligation to notify the users that in case such unethical information is
transmitted, then it would call for either termination of their account or removal of the
information.

(c) The intermediary should not publish on its website or application or both, as the case may
be, the name of the Grievance Officer and his/her contact details as well as the mechanism by
which a user or a victim may make a complaint against violation of the provisions of Rule 4.

(d) All of the above

. Phasebook is a social media platform and one of the Phasebook users is sharing obscene
video content which is unethical and against the user agreement or privacy policy. What is
the course of action for the same in IT rules 2021?

(a) The intermediaries are under the obligation to notify the users that in case such unethical
information is transmitted,

(b) It would call for either termination of their account or removal of the information that is
not in conformity with the user agreement or privacy policy.

(c) Both A and B

(d) An intermediary is under the strict compulsion to remove information within twenty-four
hours

. XYZ is facing abuse and defamatory messages on Phasebook by unknown Id.XYZ reported
the same to the Grievance Officer on 10 march 2021 but unfortunately not received any
response till 10 April and on 15 April received the response. Will the grievance Officer is
liable.
(a) Yes,the Grievance Officer is under an obligation to acknowledge the complaint within
three working days and resolve the same within one month from the receipt of the complaint,
and the officer has not responded within the limited time.(b) Yes, the Grievance Officer is
under an obligation to acknowledge the complaint within three working days and resolve the
same within one month from the receipt of the complaint(c) No, the victim may make a
complaint against violation of the provisions of Rule 4 and there is no violation of Rule 4(d)
None of the above. 

. Actor X who makes adult movies was portrayed in bad light her private parts on the website
named Entertainment.com without her approval. Regarding the aforementioned burden, a
complaint was filed by her against the website. Will the website will be liable and what
obligations are should be followed by the website.

(a) No website will not be liable because Actor X is making adult movies and show her
private parts in many movies Therefore Portraying her is not a Violation of Rights.

(b) Yes website is under the strict compulsion to remove information within twenty-four
hours on the complaint of a person which portrays such a person in bad light or exposes their
private parts.

(c) under an obligation to prominently publish on his website or application, or both, the
privacy policy and the usage of personal data or information for its users.

(d) Both A and B

. Unethical information is transmitted by XYZ through a page named Addsup which


conformed to the user agreement or privacy policy of Addsup. Will termination of the
account of XYZ will take place.

(a) Yes,the intermediaries are under the obligation to notify the users that in case such
unethical information is transmitted, then it would call for either termination of their account
or removal of the information that is not in conformity with the user agreement or privacy
policy.

(b) No,the information conforms to the user agreement or privacy policy of the website.

(c) No, there is a fault of the website for not drafting the user agreement carefully.

(d) None of the above

Passage (Q.-Q.): A polygraph test can be conducted only after obtaining the consent of the
accused; mere silence will not amount to consent: Karnataka High Court A polygraph test
cannot be administered without obtaining the written consent of the accused person to whom
it is to be administered, the Karnataka High Court ruled (Virendra Khanna v. State of
Karnataka). Mere silence of the concerned person will not amount to the consent of such
person, a single-judge Bench held. "Mere silence of the said person would not amount to
consent on behalf of such person. If a person was to refuse the administration of polygraph
test, no such polygraph the test could be administered and even if administered, the result of
the said the test would be void and cannot be considered by a Court of Law," the judgment
said. It also held that an investigating officer has no right to disclose the private data seized
from the smartphones or electronic gadgets of an accused to a third party, without the written
permission of the concerned court. "The consent in writing to be obtained from such a person
before directing the administration of the polygraph test. Merely because an accused is silent,
neither accepts or rejects the administration of polygraph test would not amount to consent
being provided by the accused," the judgment said. In the instant case, the Court observed
that the trial court had passed its order directing a polygraph test without there being any
application filed by the prosecution. Further, no opportunity to be heard was provided either
to the petitioner or his counsel and no consent was obtained for administration of the test, the
High Court noted. Source: 17 Mar 2021, BAR and BENCH

. Jonas was accused of committing murder and for collection of the evidence investigation
officer forced him to give consent for a polygraph test but Jonas remained to moan without
uttering a single word regarding his consent. Can the investigation officer can take polygraph
test of Jonas in these Circumstances.

(a) No, Mere silence of the concerned person will not amount to the consent of such person.

(b) Yes, a Polygraph test can be administered without obtaining the written consent of the
accused person to whom it is to be administered.

(c) No, investigating the officer has no right to take a polygraph test without the written
permission of the concerned court.

(d) Both A and C

. Jonas was accused of murder given his consent in front of the investigation officer. Will this
consent is eligible to take a polygraph test?

(a) Yes, the accused is not silent he accepts the administration of the polygraph test.

(b) No, investigating officer has no right to take a polygraph test without the written
permission of the concerned court

(c) No. A polygraph test cannot be administered without obtaining the written consent of the
accused person to whom it is to be administered.

(d) Yes, a Polygraph test cannot be administered without obtaining the written consent of the
accused person to whom it is to be administered.  

. Jonas the accused refused the administration of polygraph test and Singham new police
inspector administered it. Decide the result of the said test would be considered by a Court of
Law or not.

(a) No, the result of the said test would be void and cannot be considered by a Court of Law.
(b) No, Polygraph test cannot be administered without obtaining the written consent of the
accused person to whom it is to be administered,

(c) No, Mere silence of the concerned person will not amount to the consent of such person.

(d) All of the above

. It also held that an investigating officer has disclosed the private data seized from the
smartphones of accused Jonas to the Media with the permission of the judge in the case by
the telephonic call. Will the Investigation officer be held liable?

(a) Permission from the court is granted to the investigation officer therefore he will not be
liable.

(b) Investigating officer has no right to disclose the private data seized from the smartphones
of the accused during duty.

(c) Telephonic call is not the written permission of the concerned court. The investigation
officer will be liable.

(d) Both B and C.

. Which of the following is not true in the context of the passage?

(a) Test should administer after obtaining the written consent of the accused person to whom
it is to be administered.

(b) Mere silence of the said person would not amount to consent on behalf of such person

(c) In Virendra Khanna v. State of Karnataka no opportunity to be heard was provided either
to the petitioner or his counsel and no consent was obtained for administration of the test,

(d) All of the above are true.

Passage (Q.-Q.):1. The Supreme Court observed that the time period for investigation
specified under Section 1 of the Code of Criminal Procedure cannot be extended by seeking
to file supplementary charge sheet qua UAPA offences. The bench comprising Justices
Sanjay Kishan Kaul and R. Subhash Reddy, while granting default bail to FakhreyAlam, a
person accused under Section 18 of the UAPA Act, reiterated that default bail under first
proviso of Section 1(2) of the Cr.P.C. is a fundamental right and not merely a statutory right.
Court held that the State can take advantage of the fact that in one case there is one charge
sheet and supplementary charge sheets are used to extend the time period in this manner by
seeking to let the supplementary charge sheet qua the offences under the UAPA Act even
beyond the period specified under Section 1 of the Cr.P.C beyond which default bail will be
admissible. That period having expired and the charge sheet not having been led qua those
offences (albeit a supplementary charge sheet), we are of the view the appellant would be
entitled to default bail in the aforesaid facts and circumstances. When an investigation is not
completed in 24 hrs, it is in these circumstances that Section 1 of the Cr.P.C. provided for the
time period within which the investigation should be completed, depending upon the nature
of offences. Since liberty is a Constitutional right, time periods were specified in the default
of which the accused will have a right to default bail, a valuable right, for an offence with
punishment of death, life imprisonment or imprisonment of 10 years or more such specified
time period is days for other offences it is days. After the stipulated period every accused
shall be furnished a bail if they are prepared and do furnish bail. In Bikramjit Singh case it
was held that default bail under first proviso of Section 1(2) of the Cr.P.C. is a fundamental
right and not merely a statutory right as it is, a procedure established by law under Article 21
of the Constitution. Default Bail : State Cannot Take Advantage Of Filing One Charge Sheet
First And Seeking Time To File Supplementary Charge-sheets To Extend The Time Limit
U/S 1(2) (Live Law, 16 March 2021) as accessed on 17 March 2021.

. Kalpana was charged with murder of her employer and arrested by Police. When she was
produced before a magistrate after 24 hrs, a local magistrate allowed a judicial custody of
days. Police has been investigating since and could not find any significant evidence against
her. On the st day Kalpana filed an application for bail but before it could be accepted, Police
filed a chargesheet against Kalpana and an application for extension of custody. Both, the
chargesheet and the bail application are pending before the magistrate. Decide which of the
two shall be allowed by the magistrate.

(a) Bail application shall be allowed as when the period of days expired Kalpana had a
default right to bail. She filed an application of bail which shall be allowed if the chargesheet
has not been filed before it.

(b) Application for extension of custody be allowed as the chargesheet has been filed by the
Police before the bail application was allowed and thus the period of custody can be
extended.

(c) Bail application shall be allowed as the as after the expiry of a period of days in judicial
custody, the time period for custody cannot be extended under any circumstances.

(d) Application for extension of custody can be allowed upon the discretion of the magistrate
if the chargesheet presents enough reasons for the extension of the custody of the accused.

. The Indore Police department has been investigating a series of bomb blasts in the city just a
few days before the Prime Ministers rally. On the basis of suspicion, Police has made several
arrests and all the accused have been sent into judicial custody for days and the same period
has been allowed to the Police department to conclude the investigation. Due to its inability
to find any material evidence, Police filed a chargesheet containing charges based on
whatever little evidence they had to avoid these accused getting bails after the expiry of
days. Extension on the period of custody and time for filing supplementary chargesheet was
granted by the magistrate. Further, Police filed supplementary chargesheets later in the
process with more consolidated data and charges. The accused have raised objection against a
same in a writ petition filed before the high court. Choose the correct option with respect to
the petition.

(a) The writ petition shall not be allowed as the Police has filed a chargesheet in view of
which the custody of the accused can be extended by the magistrate.
(b) The writ petition shall be allowed as the legal provision has been exploited to avoid the
bail of the accused and to get an extension on the period to file the actual chargesheet.

(c) The writ petition shall not be allowed as a magistrate enjoys absolute discretion in the
matters of extending the custody of the accused in the interest of justice.

(d) The writ petition shall be allowed as after the stipulated period of days an accused has a
fundamental right to receive bail under article 21 of the Indian Constitution.

. Assertion: State cannot file a chargesheet and apply for an extension to file supplementary
chargesheet and extension of custody of the accused after stipulated period under section 1 of
CrPC. 

Reasoning: Under section 1 an accused can be sent into custody only for a period of or days
subject to the nature of offence alleged to be committed.

(a) Both A and R are true, R is the correct explanation of A.

(b) Both A and R are true, R is not the correct explanation of A.

(c) A is true R is false.

(d) A is false R is true.

. Porus was a politician from the opposition who has been arrested on charges of money
laundering. He was sent into judicial custody for a period of days u/s 1 of the CrPC so that
Police can conclude investigation and prepare chargesheet without Porus influencing with the
process. Before Porus’s lawyers could file an application for bail, Police filed the chargesheet
on the th day and decided to prosecute Porus for the offence of money laundering. Porus was
further sent into judicial custody after filing of the chargesheet upon order of the local
magistrate. Porus has challenged the order in a writ petition filed before the High Court
contending that he had a default right to bail, which he has been denied. Choose the correct
option regarding the order of the high court.

(a) Petition shall be allowed as chargesheet was filed after the period of days when the
accused had a default right to bail, which was denied to him.

(b) Petition shall be allowed as once the person has been sent into custody for the stipulated
period no further extension in the period of custody can be allowed.

(c) Petition shall not be allowed as the accused has a right to bail after the expiry of the
stipulated period only if they are prepared to furnish a bail.

(d) Petition shall not be allowed as once the Police files the chargesheet custody of the
accused can be extended regardless of the bail application made by the accused.

 
. A massive protest in the National Capital Region entailed a violent communal riot. Police
made 15 arrests and later dropped charges against 8 of the arrested persons and released
them. The rest of the arrested people were sent to judicial custody of days. The Police filed a
chargesheet against the arrested persons on the th day and the magistrate himself ordered an
extension on the custody further for a period of 45 days. One week later, Police filed a
supplementary chargesheet with additional charges against the accused. The accused filed an
application against the order of the magistrate, contending that Police cannot just file a
chargesheet for extension of the custody citing the judgment of the Supreme Court in the
comprehension above. Decide whether the application should be allowed or not.

(a) Application shall be allowed as an accused has a default right to bail after the expiry of
the stipulated period under section 1 of CrPC.

(b) Application shall not be allowed as the Police has already filed the chargesheet after
which extension of the custody can be allowed by the magistrate.

(c) Application shall be allowed as the Police filed a preliminary chargesheet and later filed
supplementary chargesheet only to get an extension on the period of custody

(d) Application shall not be allowed as the Police did not apply for an extension of the
custody just by filing a preliminary chargesheet and the court itself relied on the chargesheet
to order an extension on the period of custody.

Passage (Q.-Q.): Section 311 deals with the power to summon material witness, or examine a
person present. Any Court may, at any stage of any inquiry, trial or other proceeding under
this Code, summon any person as a witness, or examine any person in attendance, though not
summoned as a witness, or recall and re-examine any person already examined; and the Court
shall summon and examine or recall and re-examine any such person if his evidence appears
to it to be essential to the just decision of the case, the provision reads. The bench particularly
referred to the observations made in Swapan Kumar Chatterjee v. Central Bureau of
Investigation that the first part of this section which is permissive gives purely discretionary
authority to the criminal court and enables it at any stage of inquiry, trial or other proceedings
under the Code to act in one of the three ways, namely, to summon any person as a witness;
or to examine any person in attendance, though not summoned as a witness; or to recall and
re-examine any person already examined. The second part, which is mandatory, imposes an
obligation on the court to summon and examine or to recall and re-examine any such person
if his evidence appears to be essential to the just decision of the case. In Manju Devi v. State
of Rajasthan, a two-Judge bench of this Court noted that an application under Section 311
could not be rejected on the sole ground that the case had been pending for an inordinate
amount of time (ten years there). Rather, it noted that the length/duration of a case cannot
displace the basic requirement of ensuring the just decision after taking all the necessary and
material evidence on  record. In other words, the age of a case, by itself, cannot be decisive in
the matter when a prayer is made for examination of a material witness, the bench noted
while allowing the appeal. Aim Of Every Court Is To Discover The Truth: Supreme Court
Explains Scope of Section 311 CrPC (Live Law, 6 March 2021) as accessed on 9 March 2021

 
. Richa was an MBA student in JNK College. Her fellow student Munna, son of a very
influential politician, proposed Richa for marriage. She rejected the proposal as she was in
love with another boy named Bittu. Soon Richa and Bittu decided to marry each other. On
their wedding day, Munna stormed into the wedding hall with some of his friends and started
firing. A bullet shot Richa and she died on the spot. Bittu filed a complaint against Munna
and when the case went to the court, all the witnesses who had seen Munna firing bullets,
turned hostile. This led to the case being delayed for 8 years, but suddenly an old man named
Kiku approached Bittu and told him that he is ready to give his statement and will also tell the
court that he, along with all other eyewitnesses, was threatened by Munna and his father.
Bittu’s lawyer filed an application under Section 311 for permission to re-examine the
witness. Whether the application should be accepted or not?

I. The application filed under Section 311 should be accepted as an application to re-examine
a witness can be filed at any stage of trial to reach the just decision of the case.

II. The application filed under Section 311 should not be accepted as it was already delayed
for 8 years and such an inordinate amount of time taken indicates that there are no merits in
the case.

III. The application should not be accepted as Bittu was only trying to waste the court’s time
and blame Munna for a crime against which there is no evidence or witnesses.

IV. The application filed under Section 311 should be accepted as it cannot be rejected on the
ground that the case had been pending for an inordinate amount of time. The correct answer
is:

(a) Only I

(b) Both I & IV

(c) Only II

(d) Either I or IV

. An FIR was filed against Girish on 18.05.2008 for fraudulently using forged documents as
original and defrauding many people. After the investigation was over, a charge sheet was
filed against him as all the evidence found by the police was against him. During the trial, the
prosecution presented 46 witnesses for examination in the course of over 54 hearings to prove
that he has defrauded many people. There was only one witness from the side of Girish. The
trial court thereafter fixed the date for final hearing on 4.03.2021. Girish filed an application
under Section 311 for permission to examine 3 more witnesses present in the court. Should
the trial court accept the application?

(a) The trial court should not accept the application filed under Section 311 as Girish was
only trying to waste the court’s time by requesting to examine 3 more witnesses when the
date for final hearing is already fixed.

(b) The trial court should accept the application filed under Section 311 as the court can allow
examination of any person in attendance at any stage of trial if their evidence appears to it to
be essential to the just decision of the case.
(c) The trial court should accept the application filed under Section 311 as the court shall
allow examination of any person at any stage of trial upon the defendant’s request.

(d) The trial court should not accept the application filed under Section 311 as the
prosecution had already presented 46 witnesses for examination in the course of over 54
hearings which clearly proves that Girish is guilty of fraud.

. Mahesh was arrested for the murder of his ex-wife Rashmi. When the case was in trial at the
court, the prosecution presented Rashmi’s sister Geeta as the prime eye witness. She
explained to the court that she has seen Mahesh holding their dog’s red coloured chain and
choking Rashmi with it until she died. The prosecution also presented many witnesses to
prove that Mahesh is violent by nature and since Rashmi was going to marry someone else,
he couldn’t digest it and so he killed her. On the date of final hearing, Mahesh’s lawyer filed
an application under section 311, praying therein to cross-examine Geeta as it came to their
knowledge that the dog’s chain was of blue colour and not red. The prosecution, on behalf of
Geeta replied that the application has been filed with a view to linger on the proceedings. She
further claimed that she was cross-examined at length on all aspects averred in the application
as such; the application deserves to be dismissed. Whether the application should be
dismissed or not?

(a) The application filed under Section 311 should be dismissed as she was already cross-
examined at length on all aspects averred in the application and allowing it at such a time
would further linger on the proceedings.

(b) The application filed under Section 311 should be dismissed as such an application cannot
be entertained in the later or final dates of the trial.

(c) The application filed under Section 311 should not be dismissed as it came to the
knowledge of Mahesh’s lawyer that Geeta was lying and so it is necessary to cross-examine
her in order to know the real reason.

(d) The application filed under Section 311 should not be dismissed as even if it was filed on
the date of final hearing, the cross-examination of Geeta is necessary to reach the just
decision of the case. .

. Assuming that the court dismissed Mahesh’s application filed under Section 311 to cross-
examine Geeta, he again filed an application under section 311 praying therein to examine a
person named Ritesh as a witness present in the court. His lawyer claimed that Ritesh was
present with Mahesh on the night of his wife’s murder. He was not ready to come before the
court as they were gambling on the night and he was afraid that his family might know about
it if he gave a statement to the court. The prosecution opposed that by saying that the
application has been filed with a view to further linger on the proceedings and that too on the
day of final hearing of the case. He also contended that Ritesh was not summoned as a
witness by the court and therefore his testimony cannot be held valid. What is your opinion
on what the court should decide?

I. The court should allow the application as even if Ritesh was not summoned as a witness by
the court, the court has the power to allow the examination of any person present in
attendance of the court to reach the just decision of the case.
II. The court should not allow the application as it was filed on the final date of hearing of the
case. Any application to examine a new witness cannot be entertained at this stage of the trial.

III. The court should not allow the application as Ritesh was not summoned as a witness by
the court and no person can be examined as a witness unless they are summoned by the court.

IV. The court should not allow the application as it was filed on behalf of Mahesh with a
dishonest intention of lingering on the proceedings before the court. The correct answer is:

(a) Only I (b) Only III (c) Both II & III (d) Either I or II, III & IV

. Assertion: Section 311 imposes an obligation on the court to summon and examine or to
recall and re[1]examine any such person if his evidence appears to be essential to the just
decision of the case.

Reasoning: An application filed under Section 311 could be rejected on the ground that the
case had been pending for an inordinate amount of time.

(a) A is true, R is false.

(b) A is false, R is true.

(c) Both A and R are true, R is the correct explanation of A.

(d) Both A and R are true, R is the correct explanation of A

29
Passage (Q.-Q.): Five Hindu women and a NGO have approached the Supreme Court
challenging the practice of bigamy permitted under Muslim personal law. The petition
claimed that while there exists a clear bar under law on polygamous relationships under
Section 4 of the Indian Penal Code (IPC) which makes it punishable, such personal laws
make its provision discriminatory as other religions are unable to enjoy this benefit. The
petition has sought striking down Section 2 of Muslim Personal Law (Shariat) Application
Act, 17 which recognises the system of bigamy or polygamy prevalent in Muslim
community. It said, ―The system of bigamy is the history of past and has no place in present
day scenario at International level. There is unanimity between nations that such a practice is
unscrupulous, discriminatory, exploitative and anti-women, and must come to an end.‖
Section 4 of IPC provides that ―whoever, having a husband or wife living, marries in any
case in which such marriage is void by reason of its taking place during the life of such
husband or wife, shall be punished with imprisonment of either description for a term which
may extend to seven years, and shall also be liable to fine‖. The petition has said that there
should be unanimity in the field of penal law relating to prohibition of bigamy. The
petitioners have prayed that the words ―in any case in which such marriage is void by
reasons of its taking place‖ occurring in Section 4 of IPC be read down or struck down as the
same is discriminatory and oppressive for women and will be hit by Article 14 (right to
equality) and 15(1) (Right against discrimination) of the Constitution of India. Source: Bar
and Bench
. Uttam was married to Neema at the age of 16 and the girl‘s age was of 15. Later, when he
turned 24 years of age, he started liking another girl. He has read somewhere that marriage
performed below the age of 18 is a void marriage, therefore believing that he is not validly
married to Neema, left her without giving her divorce and marries the girl he liked. In this
case-

(a) Uttam has committed an offence of bigamy by marrying other women without divorcing
his first wife.

(b) Uttam has not committed an offence of bigamy, because he was not of major age when he
married to Neema

(c) Uttam can marry to the girl he likes because now he is of 24 years of age and can take his
own decision.

(d) These rules are bogus and against one‘s fundamental right to choose partner.

. Salim Ali at 19 years, a Muslim by religion, married to Sara (17 years), a Muslim Girl.
Later, when he turned 28 years of age, he fell in love with another girl, Valima. He left Sara
and married Valima. When Sara found out about Ali‘s second marriage, she wants to bring an
action against him for committing bigamy. Decide.

(a) Salim Ali has committed an offence of bigamy.

(b) Salim Ali has not committed an offence of bigamy, because he was not of major age when
he married to Sara.

(c) Salim Ali can marry to the girl he likes because now he is of 28 years of age and can take
his own decision.

(d) Salim Ali has not committed the offence of Bigamy.

. Atimabh is married to Maya and both are leading a happy married life. Both of them decide
to go on a vacation to Bangkok for a week for business and pleasure. During their trip, in one
of the industrial fairs, Maya and Atimabh get separated and in spite of several efforts by
Atimabh, the local police and the media, he can't locate Maya. With a heavy heart, he decides
to leave for India. Two years pass by and he meets a divorcee called Meekha. They meet
regularly to share their loneliness and eventually decide to get married. The next year, Maya
comes home and finds out that Atimabh has married Meekha. Which of the following
statements is true?

(a) Atimabh and Meekha are liable for bigamy

(b) Since Maya was still alive, Atimabh's marriage to Meekha will be considered as a case of
bigamy.

(c) Since Maya was absent for several years, a presumption will be made in law that she was
not alive. Hence, Atimabh's marriage with Meekha is valid

(d) Maya is liable for deserting her husband, hence, she is liable
 

. Sourabh and Premlata are both Hindus and got married in Punjab. Later, Sourabh went to
Ohio, US to complete his Master‘s degree. He fell in love with one of his batchmates
Manisha and wished to marry her. The local laws of Ohio allowed people to marry more than
once during the presence of their living spouse. Sourabh went ahead and divorced Premlata
without her knowledge. After a year, when Premlata went for Saurabh‘s convocation she got
the information about his second marriage to Manisha. Devastated, Premlata moved a petition
in Indian court against Saurabh for committing bigamy. Which of the following is true?

(a) The marriage of Sourabh with Manisha is valid as a competent court of law dissolved the
marriage of Sourabh with Premlata

(b) Since Sourabh has been residing in Ohio, his marriage and divorce shall be governed by
the law there

(c) Since Sourabh did not divorced Premlata before marrying Manisha, he will be held liable
for committing bigamy under Indian Penal Code.

(d) Since Sourabh got married in Punjab, the Indian law should be applicable to his case

. Rakhi got married to Daniel in 2011. Tired of Rakhi‘s publicity stunts, Daniel left her and
started living separately. Rakhi met Mika in a concert and fell in love with him. The duo even
launched their new item number which got immense popularity. Mika who is an ardent news
follower read the above article and believed that bigamy is allowed for all in the country and
not just Muslim men. He broke this news to Rakhi and the very next day they married at a
small ceremony on an isolated island and broke the news to their fans through Instagram live.
When Daniel saw the Instagram live, he went furious and filed a case against Rakhi for
bigamy. Is she guilty?

(a) Rakhi guilty as her act of remarrying is an offence under the penal codes of the country.

(b) Rakhi is not guilty as she genuinely believed that bigamy is allowed to all the citizens in
the country.

(c) Rakhi.is not guilty as she was anyway living separately from her husband making it a
valid ground for divorced.

(d) Mika is guilty for the offence of adultery.

Passage (Q.-Q.): The 2018 amendment to Section 10 of the Specific Relief Act, 19 has
significantly curtailed the power of the courts to refuse specific performance of a contract,
especially when the terms of the contract clearly warrant it. This has put a huge onus on all
entities entering into commercial contracts to expend more time and attention while
negotiating and entering into such contracts. The real significance and impact of the
amendment stems from the fact that it has made the specific performance of contracts the
(general) rule rather than an optional remedy. Prior to the amendment, Section 10 of the Act
stated that "the specific performance of any contract may, in the discretion of the court, be
enforced" since the legislative intention at the time of the enactment of the Act was to give
full judicial discretion to the courts to direct/enforce or not to direct/enforce the specific
performance of a contract. Post the amendment, the Section reads that "the specific
performance of a contract shall be enforced by the court subject to the applicability of the
provisions contained in Section 11(2), Section 14 and Section  19 of 40 16 of the Act." This
clearly shows the change in the approach of the legislature to limit or restrict the judiciary‘s
wide discretionary powers qua specific performance of contracts. It is well-established under
contract law that with respect to the interpretation of contracts, the doctrine of contra
proferentem applies against the party which drafted the terms of the contract or imposed any
specific conditions therein in that any ambiguity in the contract will be construed against the
party that drafted the contract. This holds good irrespective of the nature of the contract or the
bargaining power/position of the parties. [Extracted with minor revisions from ‗Judicial
discretion to enforce contracts in India curtailed: Effect on post-COVID litigation‘ by NL
Rajah and Aparna Raman, published 24 Fed 2021 in barandbench
https://www.barandbench.com/columns/judicial-discretion-to-enforce-contracts-in-india-
curtailed-effect[1]on-post-covid-litigation]

. A and B entered into a contract in 2016 for the sale of 10,000 pairs of plastic slippers. Under
the contract, A was supposed to make half of the payment at the time of contracting and the
other half upon delivery of the slippers. B was due to make good on his obligations before
March 2020 however, he failed to do so due to interruptions in supply chains in light of
severe travel restrictions. On 1st April 2020, A sued B for specific performance of the
contract. Is the court bound to direct B to perform the contract?

(a) Yes, because the amendment to the Specific Relief Act significantly curtailed the power
of the courts to refuse specific performance of a contract

(b) Yes, because the terms of the contract clearly warrant specific performance of obligations
by B before March 2020

(c) No, because the courts have discretion to direct/enforce specific performance by B

(d) No, because the wide discretionary powers of the court is limited with respect to ordering
specific performance by B

. In the previous factual scenario, assume that B was bound to deliver the plastic slippers
before March 2018. However, B was unable to fulfil his contractual obligations because the
truck carrying the raw material for making the slippers met with an accident and the slippers
could not be completed in time. If A sues B for specific performance of contractual
obligations, is the Court now bound to give an order in A's favour?

(a) No, because Section 10 of the Specific Relief Act restricts the judiciary‘s wide
discretionary powers qua specific performance of contracts

(b) No, because Section 10 of the Specific Relief Act states that "the specific performance of
any contract may, in the discretion of the court, be enforced"

(c) Yes, because Section 10 of the Specific Relief Act states that "the specific performance of
any contract may, in the discretion of the court, be enforced"

(d) Yes, because Section 10 of the Specific Relief Act restricts the judiciary‘s wide
discretionary powers qua specific performance of contracts
 

. Chota and Bheem entered into a contract for the sale and purchase of hand sanitizers in
January 2020. Due to the lockdown restrictions in March 2020, Bheem could neither fulfil
nor deliver the sanitizers to Chota in the time period stipulated under the contract. Chota gave
him an additional time period of one month. Bheem was now in a position to make good on
his obligations but he refused to do so because he wanted to spend his lockdown time with
his family and do no work. If the contract is not concerned with the provisions of Sections
11(2), 14 and 16, can Chota undoubtedly seek specific performance of the contract by
Bheem?

(a) Yes, because the specific performance of contracts is the general rule rather than an
optional remedy

(b) Yes, because there is a change in the approach of the legislature to limit or restrict the
judiciary‘s wide discretionary powers qua specific performance of contracts

(c) Yes, because the amended Specific Relief Act doesn't significantly curtail the power of
the courts to refuse specific performance of a contract, especially when the terms of the
contract clearly warrant it

(d) None of the above  

. Dekisuki and Nobita enter into a contract wherein Nobita offers to pay Dekisuki Rs 500 if
he completes his maths homework. The contractual performance date was surpassed and
Nobita wants to sue Dekisuki for specific performance so that he may submit his homework
on time. If this contract relates to the provisions under Section 11(2), Section 14 and Section
16 of the Specific Relief Act, can Nobita seek specific performance of the contract to submit
his homework on time?

(a) Yes, because the specific performance of the contract is enforceable when these
provisions are applicable

(b) No, because the specific performance of the contract is not enforceable when these
provisions are applicable

(c) No, because the specific performance of the contract is not enforceable when these
provisions are not applicable

(d) Yes, because the specific performance of the contract is enforceable when these
provisions are not applicable

. Harry was in the business of selling juice to retail shops. Mitsi was a retailer who used to
buy goods from wholesalers and sell them in her shop in Patparganj. Harry and Mitsi enter
into an agreement, drafted by Harry, to contract for the sale and purchase of 2 crates of "high
quality, premium orange juice". When Harry delivered the crates, Mitsi refused to accept the
goods because the juice boxes were non-pulpy. Because of this reason, she believed that the
juice boxes were not of premium quality, unlike what was promised in the contract. Both of
them sued each other for contractual performance. As a legal counsel, who do you think has a
better position to get a remedy in their favour according to the law?

(a) Harry, because Contra proferentem applies and there was an ambiguity in the contract
with respect to what qualifies as "premium"

(b) Mitsi, because Contra proferentem applies and there was an ambiguity in the contract
with respect to what qualifies as "premium"

(c) Harry, because Contra proferentem does not apply as there was no ambiguity in the
contract since he provided premium juices even though they were non-pulpy

(d) Mitsi, because Contra proferentem does not apply as there was no ambiguity in the
contract since non-pulpy juices can never be premium in nature

Passage (Q.-Q.): Damages in contract law can be defined as a sum of money paid to the
innocent party in compensation for a breach of contract. When parties make an agreement,
they will hope that they both fulfil their obligations. Therefore, the intentions of the parties
cannot usually be used in order to calculate an amount of damages that should be awarded
under the contract. Instead, the amount of damages will be awarded based on the value of the
interest the innocent party has in the contract. The purpose of awarding such damages is to
compensate parties for the loss suffered and not to improve their position more than what it
would have been if the contract had been duly performed. There are different types of
damages that are awarded by courts. Compensatory damages are an award of a sum of money
which aims to compensate the claimant for his loss under the contract. Non[1]compensatory
damages are an award of a sum of money not only to compensate the claimant for his
contractual losses, but also aim to compensate the claimant in relation to any bad conduct of
the other party including mental agony. In order to assess whether an innocent party may be
entitled to damages, there are six things that should be considered. The first step is to
ascertain the loss the claimant has suffered under the contract. The general rule is that the
claimant may only recover for his own loss. Secondly, it must be considered whether or not
the loss suffered is actionable. In order for a loss to be actionable, the claimant must show
that the breach of contract caused the loss. Causation requires both legal, and factual
causation. The next step is to figure out whether the type of loss was reasonably foreseeable.
The purpose of this step is to figure out the remoteness of the damage. The next step is if the
claimant contributed to the loss in question, the courts may reduce the amount of damages the
claimant is able to claim, proportionately in line with the fault of the claimant. The final
analysis looks at whether there was an agreed damages clause in the contract that was
breached later and such amount is usually the final amount of damages. [Source: Damages
for Breach of Contract, New York University available at
https://www.law.nyu.edu/sites/default/files/ECM_PRO_07.pdf ]  

. Naik wanted to buy agricultural land. He contracted with a surveyor Somashekhar to inspect
the land and measure its fertility and possible profits he would make on agricultural produce.
Somashekhar surveyed the land and valued it for Rs. 1 crore. S, however, failed to notice the
immediate layer of red soil below 3 feets of black soil and had he taken note of it, the
agricultural land would have been worth only Rs. 50 lakhs. Naik followed Somashekhar‘s
advice and bought the land for Rs. 1 crore and thereafter spent Rs. 40 lakhs for arranging for
black soil and irrigation system. He filed a suit against Somashekhar claiming Rs. 40 lakhs as
damages. He also asked for Rs. 1,00,000 for mental agony.

(a) Naik should get 40 lakhs and Rs. 1, 00, 000.

(b) Naik should get Rs. 50 lakhs, since he paid additional Rs. 50 lakhs on account of
Somashekhar‘s negligence.

(c) Naik should get Rs. 4 lakhs, since that loss was attributable to Somashekhar‘s negligence.

(d) Naik should not get any amount as compensation. There is no foreseeability.

. Hari enters into a contract with Krishan to construct a strong wall in his house. While this
wall was being constructed, Hari‘s son dug a hole in the wall and escaped. He went out into
the city and became a victim of an unfortunate accident which broke his leg. Hari sued
Krishan for damages due to breach of contract.

(a) Krishan is liable for damages due to such breach as had it not been for the weak wall
Hari‘s son would not have been injured.

(b) Krishan is not liable for damages as he ensured due diligence while constructing the wall

(c) Krishan is liable for damages as it has caused Hari immense mental agony

(d) Krishan is not liable for causation of injury is not linked to the breach of contract

. Please refer to the facts mentioned above. Hari had specifically mentioned to Krishan as part
of the contract that this wall was required to protect his child from going out. Yet, Hari‘s son
escaped due to a weak constructed wall

(a) Krishan would be liable for damages as the contract breach directly led to the injury

(b) Krishan would not be liable for damages as the contract breach did not cause the injury

(c) Kirshan would be liable for damages as the wall was not properly built

(d) Krishan would not be liable as he could not have foreseen the injury.

. Rakesh had asked Manjappa to construct a house with strong walls for a Delhi Metro tunnel
was nearby. In order to save cost, while preparing the cement mix, Manjappa added cheaper
ingredients which resulted in weaker adhesive bonds of the bricks and consequently weaker
walls of the house. Per the contract, it was Rakesh‘s duty to water the walls for about 10 days
regularly for a strong bond as per Rakesh‘s requirements. Rakesh however, went off to
Thailand and forgot about watering the wall.

(a) Manjappa would be liable for damages as the contract breach directly led to the injury
(b) Manjappa would not be liable for damages as the contract breach did not cause the injury

(c) Manjappa would be partly liable since Rakesh‘s negligence also contributed to the
damage

(d) Manjappawould not be partly liable since construction was solely Rakesh‘s job.

. Please refer to the facts above. The contract contained a clause that stated in the event of any
breach, the party responsible for such breach shall be liable to pay damages of INR 10,000.
Rakesh‘s son was injured when the wall fell due to vibrations from the nearby Delhi Metro.
Rakesh had to spend an amount of INR 50,000 in getting his son treated at the hospital.

(a) Manjappa would be liable to pay INR 50,000 as he was totally responsible for the damage

(b) Manjappa would be liable to pay INR 10,000 as the contract stipulated so

(c) Manjappa would be liable to pay INR ,000 as medical expenses plus damage in contract

(d) Manjappa would be liable to pay nothing as the damage was too remote.  

Passage (Q.-Q.): Free speech of the citizens of this country cannot be stifled by implicating
them in criminal cases, unless such speech has the tendency to affect public order, the
Supreme Court remarked while quashing FIR registered against Shillong Times Editor
Patricia Mukhim over a Facebook post on violence against non-tribal people in Meghalaya.
The facebook post was directed against the apathy shown by the Chief Minister of
Meghalaya, the Director General of Police and the DorbarShnong of the area in not taking
any action against the culprits who attacked the non-tribals youngsters. Sections 153 A and
505 (1) (c) of the Indian Penal Code, states that only where the written or spoken words have
the tendency of creating public disorder or disturbance of law and order or affecting public
tranquility, the law needs to step in to prevent such an activity. The intention to cause
disorder or incite people to violence is the sine qua non of the offence under Section 153 A
IPC. The intention has to be judged primarily by the language of the piece of writing and the
circumstances in which it was written and published. The matter complained of within the
ambit of Section 153A must be read as a whole. One cannot rely on strongly worded and
isolated passages for proving the charge nor indeed can one take a sentence here and a
sentence there and connect them by a meticulous process of inferential reasoning. The first
test for the Courts to apply is the hate speech prohibition objectively and in so doing, ask
whether a reasonable person, aware of the context and circumstances, would view the
expression as exposing the protected group to hatred. The second test was to restrict
interpretation of the legislative term "hatred" to those extreme manifestations of the emotion
described by the words "detestation" and "vilification". The third test was for Courts to focus
their analysis on the effect of the expression at issue, namely, whether it is likely to expose
the targeted person or group to hatred by others. Mere repugnancy of the ideas expressed is
insufficient to constitute the crime attracting penalty. Free Speech Of Citizens Cannot Be
Stifled By Implicating Them In Criminal Cases: Supreme Court

 
. Sushmita was the chief editor at E-Quality Magazine, which featured her own article titled
‗Homosexuality not against Indian Culture‘. The piece highlighted that how sculptures at
many Indian temples not only acknowledge but also celebrate homosexuality and this is a
clear indication that homosexuality is not against Indian culture. Therefore, the Indian society
should break their preconceived notions regarding the issue and have an unbiased opinion on
it. Many people across the country disputed the veracity of the fact and criticised Sushmita
online for her hurtful and reckless comment. Amidst this furore, somebody filed a complaint
against Sushmita under section 153A of the IPC for causing public disorder and affecting
public tranquility. During the trial, Sushmita argued that her. Choose whether Sushmita shall
be held guilty of the offence or not?

(a) Sushmita shall be held guilty of the offence as her article caused immense public disorder
and completely disrupted the public tranquility and in such a case intention is barely of any
significance.

(b) Sushmita shall not be held guilty of the offence as she had no intention to vilify any
religious group and her article was just an attempt to show the existence of homosexuality in
a culture since the very beginning.

(c) Sushmita shall be held guilty of the offence as the article had a evident intention to
outrage the people who take offence by such connection of homosexuality and their culture.
This led to causing public disorder and disruption of public tranquility.

(d) Sushmita shall not be held guilty of the offence as one should take into consideration the
entire article before drawing any deductions regarding the intention of writing the article.  

. Hema was an environmental and social activist, who has been working for animal rights and
their better treatment for years. At an environmental conference she presented a paper which
stated how religious practices have been a profound reason for ill treatment of animals and
for example she quoted statistics on deaths of goats in Islamic Festivals. The entire Muslim
community started showed extreme disapproval of her statements and demanded grave
punishment for her for insulting Islam. Taking into account the nationwide furore, the
Bombay High Court took the suo moto cognizance of the matter and charged Hema under
section 153A. Decide whether she shall be convicted of the offence or not.

(a) Hema shall be convicted of the offence as she exposed entire Muslim community to
hatred by pointing a particular practise and insulting the community on the basis of that
practise.

(b) Hema shall not be convicted of the offence as there is no way to deduce her intention to
cause public disorder by outraging the feelings of the community.

(c) Hema shall be convicted of the offence as sacrifice of goats is an essential practice in the
Islamic festivals and criticising it clearly amounts to vilifying the community.

(d) Hema shall not be convicted of the offence as the intention behind her statement cannot
be deduced from a single sentence she spoke and the entire context is relevant to adjudicate
guilt.

 
. Raman was a cabinet minister in the state government of Odisha. In a political rally he said
that all the people ‗Pudali‘ tribe have been joining naxals gradually and it would be in best
interest of the people if the State takes preemptive action against them. Resultantly, the
people from the Pudali tribe suffered violence and brutality from the rest of the communities
of the state. The state government eventually had to announce a lockdown to control the
statewide disorder and unrest. Opposition party has demanded a trial against Raman for
inciting disorder and violence against people of Pudali community. Raman is defending
himself arguing that he just propounded an opinion and had no intention of causing such
disruption. Which of the following is the correct option in the light of the third test given in
the above passage?

(a) Raman shall be convicted of the offence as Raman‘s statement exposed the Pudali tribal
community to hatred and violence to an extreme extent causing disruption across the state.

(b) Raman shall not be convicted of the offence as he just propounded an opinion which is a
clear indication that he had no intention to expose any group to hatred or violence.

(c) Raman shall not be convicted of the offence as he just demanded a State intervention and
preemptive action against the tribes to prevent the consolidation of naxals in the state.

(d) Raman shall be convicted of the offence as it was due to his extreme manifestations of
emotions that vilified the tribal community outraging the people of Pudali community.

. Rujhita, a feminist writer, wrote a book to highlight the celebrated judgment of the Supreme
Court wherein it upheld the dignity of the women and improve their status in the society. In
that book, she criticised the rituals that undermine the position of the women such as sati,
triple talaq or lack of coparcenary rights in various fragments throughout the book. Hemant
pressed charges against Rujhita for vilifying the Hindu religion amongst others despite the
reformative policies adopted by it. Even during the trial Hemant argued that the highlighted
practices are no more prevalent but Rujhita has highlighted them solely for the purpose of
disrespecting the religion. Decide whether Rujhita has committed the offence under section
153A and 505 (1)(c).

(a) Rujhita has not committed the offence as mere highlighting of old oppressive practices
does not establish that she had any intention of causing public disorder or affecting public
tranquility.

(b) Rujhita has committed the offence as despite such practices being held unconstitutional.
The only reason to connect these practices with a religion that no longer follows them is
disrespecting the religion and resultantly spreading hatred.

(c) Rujhita has not committed the offence as the actual intention was to celebrate the
progressive judgments by the Supreme Court. Hemant has made above mentioned
accusations by relying on different fragmentations from different parts of the book which
does not sum up the intention of the book.

(d) Rujhita has committed the offence as a reasonable person who is already aware of the
context andthese forbidden practices would perceive the book as an attempt to expose certain
religions to hatred.  
 

. Which of the following are offences under section 153A in the light of the second test given
in the passage? I. A leader giving a speech that the majority religion is trying to force the
minorities out of the country resulting into nationwide riots. II. A researcher commenting in a
conference that females have been oppressed by males for centuries, the comment outrage
many males who criticized the comment on social media. III. After the assanination of a
certain minister, two of his followers chanted slogans against the entire religious community
to which the accused belonged. IV. Akul milk published a cartoon in the newspapers showing
people of four different religions praying before a light to celebrate secularism in India on the
occasion of Independence Day, which resulted in a little violence in some parts of the nation.
Choose the correct option:

(a) II, III and IV

(b) I and III

(c) II and IV

(d) I, II and III

Passage (Q.-Q.): Under Indian law, people seeking refuge are defined as ―foreigners‖ under
the Foreigners Act, 16 as they enter the nation without valid travel papers. According to the
Foreigners (Report to the police) Order, 2001, made under the Foreigners Act 16, anyone
giving shelter to such a foreigner has to report him or her to the police within 24 hours.
Despite this, all people living within the borders of India, including foreigners, migrants,
refugees, have two very important rights under the Indian Constitution. The rights under
Article 14 and Article 21 of the Constitution are available for all persons, whether they are
citizens or foreigners, whether they are legally inside the country or not. Globally, a refugee
is defined as a special category of foreigner who is compelled to seek refuge in another
country because of persecution in the country of their origin based on certain identities like
race, creed, belief, religion , ethnicity etc. The basic protection for anyone seeking asylum
under any refugee law is the right to non[1]refoulement.The principle of non-refoulment
prohibits states from transferring or deporting individuals from their jurisdiction or effective
control when there are substantial grounds for believing that the person would be at risk of
irreparable harm upon return, including persecution, torture, ill-treatment or other serious
human rights violations. Non-refoulement is a right under refugee law which is a part of
international humanitarian law. India is a signatory to international human rights treaties and
is under obligation to respect basic human rights principles. Since India does not have a
refugee law ,UNHCR would ascertain whether a person is qualified to be a refugee and then
issues them an identity card, helps them with a stipend, and often resettles the refugees in
other countries. https://scroll.in/article//india-must-offer-shelter-to-myanmarese-asylum-
seekers-even-though-it[1]lacks-a-refugee-protection-law

. Phizo is a Burmese citizen, he is infamous in his country for running a drug cartel, he is also
involved in other criminal activities including extortion, forgery and fraud. They government
of his country designs a plan to capture him but he deceives them and escapes through the
porous border to the neighbouring nation of India. The government of India captures him and
initiates the procedure of his deportation, but he argues that his life would be in danger in
Myanmar and India should accept him as a refugee, Is his request viable?

(a) He is correct as his life would be in danger back in his country and all persons within the
territory of India possess article 21 which is right to life.

(b) He is correct as under the principle of non-refoulment, he cannot be deported.

(c) His request isn‘t viable as he does not qualify as a refugee and the Burmese government
isn‘t targeting him based on any of his identities.

(d) He is a criminal operating a drug cartel, he will not be allowed to stay as refugee.   

. Adil is a Rohingya refugee who fled Myanmar and is settled in India for the past 10 years,
In the year 2020 the government of India enters into an agreement with the government of
Myanmar according to which all refugees stationed in India will have to be deported to
Myanmar. The legality of this treaty is challenged in the court.

(a) The court will let the government proceed with this treaty as this is a policy matter and the
court does not have locus standi.

(b) India is bound by international treaties and also is liable to follow the principle of non-
refoulment, hence the court would scrap this treaty

(c) Article 14 and 21 of the Constitution are available to all persons within the territory of
India, hence the government cannot deport the refugees staying in India.

(d) India does not have a refugee law hence the country isn‘t bound to allow Rohingya
refugees in India.

. Sajid is a human rights activist working in the north-eastern part of India, he is deeply
pained by the military crackdown on Rohingyas in Myanmar and wishes to help them, he
eventually sets up a refugee camp and helps the Rohingyas in India financially. He is arrested
by the police on the charges of helping the refugees evade deportation and of hiding them in
order to protect them, is he liable?

(a) No, he is not liable as the government cannot deport refugees under international law

(b) Yes, he is liable as anyone giving shelter to a foreigner has to report him or her to the
police within 24 hours under Foreigners Act 16

(c) Yes , he is liable as India does not have a law to protect refugees and refugees are
considered as foreigners

(d) No, he is not liable as India isasignatory to international human rights treaties which call
for protection of refugees and non-refoulment
 

. A group of refuges have been residing in Delhi for several years now, some of these
refugees recently have been found to be in touch with a supposed human rights organisation
which is known to be involved in anti- India activities, due to this they have been detained , is
this infringement of their right as refugees?

(a) Yes, this is a clear violation of their right to life guaranteed under article 21 of the
constitution

(b) Yes, refugees also enjoy several rights under the Indian constitution.

(c) No, police has the right to inspect them because the organisation in suspicious in nature

(d) No, they have been booked under due process of law and not because of their status as
refugees

. Abel belongs to the Rohingya ethnicity who are often persecuted in Myanmar , growing up
he and his family also has faced discrimination and persecution. This instilled a feeling of
enmity and desire for revenge in him, at the age of 20 he formed an organisation to cater to
the needs of Rohingyas. His organisation is infamous for violence against civilians and
causing deadly riots and other war crimes. Recently he has fled to India to secure refuge and
escape action from the army of his nation, the Indian government arrests him and plans to
deport him back, he claim security under article 21, what will be the course of action?

(a) He cannot be deported back to Myanmar as the Indian constitution allows him right to life
under article 21.

(b) He cannot be deported as Non-refoulement is a right under refugee law as his life would
be under threat back in Myanmar

(c) He will be deported back to Myanmar as he is accused of war crimes and he should be
tried according to the procedure established by law.

(d) He will be deported back to Myanmar as India does not have a refugee law and recognises
them only as illegal immigrants.  

Passage (Q.-Q.): In November 2020, the state Assembly passed the Haryana State
Employment of Local Candidates Bill, 2020 paving the way for more employment
opportunities for locals in the private sector. On March 2, the Governor gave his assent to the
Bill. All the registered companies, societies, trusts, partnership firms and any person
employing 10 or more persons and an entity, as may be notified by the government from time
to time shall come under the ambit of this Act. The definition of ―employer‖ given in the
Bill means a Company registered under the Companies Act, 2013 or a Society registered
under the Haryana Registration and Regulation of Societies Act, 2012 or a Partnership Firm
as defined under Indian Partnership Act, 12 or any person employing 10 or more persons on
salary, wages or other remuneration for manufacturing or providing any service or such
entity, as may be notified by the government from time to time. It shall not include the central
government or state government or any organization owned by the central or state
government. A candidate ―who is domiciled in the State of Haryana‖ is called a local
candidate and shall be able to avail the benefit of this reservation while seeking employment
in the private sector. Every employer shall be required to employ percent of local candidates
for the posts where the gross monthly salary or wages are not more than Rs. 50,000 or as
notified by the government from time to time. An employer can claim exemption only if the
government-appointed officers believe that the employer‘s request seeking exemption holds
merit. The employer may claim exemption where an adequate number of local candidates of
the desired skill, qualification, or proficiency is not available. The employer can be fined a
minimum of Rs. 10,000 to a maximum of Rs. 2 lakh once it is established that the employer
has violated provisions of the Act.

. J and A is a newly established cyber law firm in Haryana which isa Company registered
under the Companies Act, 2013, currently working on international cyber laws and want to
seek exemptions from Haryana State Employment of Local Candidates Bill, 2020 because
not getting local experts in international cyber law in Haryana. J and A firm will be eligible
for exemption decide with legal reasoning.

(a) No,every employer shall be required to employ percent local candidates for the posts.

(b) Yes,the Employer can claim an exemption if the government-appointed officers believe
that the employer‘s request seeking exemption holds merit. J and A rightly claimed that an
adequate number of local candidates of the desired skill is not available.

(c) J and A rightly claimed that an adequate number of local candidates of the desired skill is
not available.

(d) Yes, J and A firm not come under the ambit of this Act.

. Wit and Wisdom is a Book publication company in Sonepat Haryana that comes under the
ambit of the central govt and is funded by the central govt and not giving opportunities to
local candidates of Haryana underHaryana State Employment of Local Candidates Bill, 2020.
Aggrieved by this Student filled petition against the company. Will the company be liable
under the Haryana State Employment of Local Candidates Bill, 2020?

(a) Yes, the company will be liable for the violation of the Haryana State Employment of
Local Candidates Bill, 2020 because it Company registered under the Companies Act, 2013.

(b) Yes, it will be liable because comes under the ambit of this Act.

(c) No,the act not includes the central government or state government, or any organization
owned by the central or state government.

(d) No, the act shall include the central government or state government or any organization
owned by the central or state government.  

. Legal aid society is a privately owned society in Haryana. Society is not registered under the
Haryana Registration and Regulation of Societies Act, 2012 and violating rules of the
Haryana State Employment of Local Candidates Bill, 2020. Will it be liable for compensation
and violation under the said act?

(a) Yes, All the registered companies, societies, trusts, partnership firms and any person
employing 10 or more persons and an entity, as may be notified by the government from time
to time shall come under the ambit of this Act.

(b) The employer can be fined a minimum of Rs. 10,000 to a maximum of Rs. 2 lakhs once it
is established that the employer has violated provisions of the Act.

(c) Both A and B

(d) No, because Legal aid society is not registered. Therefore,it will not come under the ambit
of the act.

. Concise Pvt Limited is Pen manufacturing company in Haryana but even after several
warnings not adhering to Haryana State Employment of Local Candidates Bill, 2020 and not
employing percent of local candidates for the posts where the gross monthly salary or wages
are not more than Rs. 50,000. Decide with the appropriate course of action.

(a) The employer can be fined a minimum of Rs. 10,000 to a maximum of Rs. 2 lakh once
because there is a clear violation of provisions of the Act.

(b)The employer can be fined a minimum of Rs. 10,000 to a maximum of Rs. 2 lakh once it is
established that the employer has violated provisions of the Act.

(c) The company should be closed because they are doing illegal work by not allowing local
candidates of Haryana.

(d) All of the Above.

. There is an amendment by govt of Haryana that percent of local candidates for the posts
where the gross monthly salary or wages are not more than Rs. 30,000 and manufacturing
companies will not come under the ambit of the above act. Considering Que 4, decide with
the appropriate course of action liability of Concise Pvt limited.

(a) The employer can be fined a minimum of Rs. 10,000 to a maximum of Rs. 2 lakh once
because there is a clear violation of provisions of the Act.

(b) The employer can be fined a minimum of Rs. 10,000 to a maximum of Rs. 2 lakh once it
is established that the employer has violated provisions of the Act.

(c) Company will not be liable underHaryana State Employment of Local Candidates Bill,
2020.

(d) Both A and B

 
Passage (Q.-Q.): The Rajya Sabha on Tuesday passed the Medical Termination of Pregnancy
(Amendment) Bill, 2020. Introduced by Union Health Minister Dr. Harsh Vardhan, the Bill
seeks to amend Section 3 of the Medical Termination of Pregnancy Act, 19, extends the
upper limit for medical termination of pregnancy to 24 weeks, from the present stipulation of
20 weeks, for certain categories of women which will be denied in the MTP Rules. These
categories will include 'vulnerable women' including rape victims. The Statement of Objects
and Reasons annexed with the Bill states, "With the passage of time and advancement of
medical technology for safe abortion, there is a scope for increasing upper gestational the
limit for terminating pregnancies especially for vulnerable women and for pregnancies with
substantial foetal anomalies detected late in pregnancy. Further, there is also a need for
increasing access of women to legal and safe abortion service in order to reduce maternal
mortality and morbidity caused by unsafe abortion and its complications." The opinion of one
doctor will be required for termination of pregnancy up to 20 weeks; and opinion of two
doctors will be required for termination of pregnancy of 20 to 24 weeks. They have to be of
the opinion that the continuance of the pregnancy would involve a risk to the life of the
pregnant woman or of grave injury to her physical or mental health; or that there is a
substantial risk that if the child were born, it would suffer from any serious physical or
mental abnormality. Confidentiality Of Identity The Bill also stipulates that the name and
other particulars of the woman whose pregnancy is terminated will not be revealed, except to
a person authorized in any law which is in force; and any person acting in  28 of 40
contravention of this provision will be punished with imprisonment which may extend to one
year, or with fine, or with both. Source: Live Law 16 March 2021 7:20 PM
https://www.livelaw.in/news-updates/parliament-passes-bill-to-increase-upper-limit-for-
legal-abortions-to[1]24-weeks-1256

. What are the reasons for amendment in Medical Termination of Pregnancy Act, 19 in the
context of the Passage?

(a) It was essential to make modification in the MTP Act, 19 to avoid serious physical or
mental abnormality to women.

(b) Due to negligence of medical experts‘ pregnancies with substantial foetal anomalies
detected late therefore to give extra time to avoid any kind of dire consequences amendment
in MTP Act 19 was vital.

(c) Due to the advancement of medical technology for safe abortion, it is possible to increase
the upper gestational limit for terminating pregnancies and reducing maternal mortality and
morbidity caused by unsafe abortion.

(d) All of the above

. Which is not untrue according to the passage?

(a) Privacy of woman whose pregnancy is terminated will be maintained.

(b) Opinion of one or two doctors will be required for termination of pregnancy of 20 to 24
weeks.
(c) In Medical Termination of Pregnancy Act, 19 stipulation limit is 5 months.

(d) Both A and C.

. X is 4-month pregnant woman and wants to terminate her pregnancy due to substantial
foetal anomalies detected in her Sonography. She terminates her pregnancy in Mumbai
Hospital and don‘t want to reveal her identity but Hospital given her name, date, and time of
termination of pregnancy to a Government officer who is authorized by law in force to collect
data on abortions from all private and government hospitals. Decide hospital will be legally
liable for violating Medical Termination of Pregnancy (Amendment) Bill, 2020 Clause of
maintaining the privacy of women?

(a) No, Hospital will not be liable because they have given her identity to a person authorized
by the law in force.

(b) No, Hospital will not be liable because MTP Bill 2020 talks about 24 weeks pregnancy
and women is only 4 months pregnant.

(c) Yes, they will be liable the bill stipulates that the name and other particulars of the woman
whose pregnancy is terminated will not be revealed.

(d) Yes, they will be liable the bill stipulates that the name and other particulars of the woman
whose pregnancy is terminated will not be revealed and will be punished with imprisonment
which may extend to one year, or with fine, or with both.

. Y is rape victim 6 month pregnant and wants to terminate her pregnancy on 6 march 2018
because she is not physically and mentally prepared to be a mother. Can she legally terminate
her pregnancy on 6 March 2021? Give the answer with legal reason?

(a) Yes, she can terminate her pregnancy after consulting with the doctor and they have to be
of the opinion that the continuance of the pregnancy would involve a risk to the life of the
pregnant woman.

(b) Yes, she can terminate her pregnancy with the opinion of two doctors and they have to be
of the opinion that the continuance of the pregnancy would involve a risk to the life of the
pregnant woman.

(c) No, She can‘t terminate her pregnancy because in the 2018 Medical Termination of
Pregnancy Act, 19 is enforceable and only termination up to 20 weeks is allowed in it but Y
is 24 weeks pregnant?

(d) No, unmarried rape victims are not legally allowed to terminate their pregnancy.  

. Z is 5 months pregnant and wants to terminate her pregnancy because she is not financially
able to afford a child in the year 2021. The doctor was of the opinion that continuance of the
pregnancy would not involve a risk to the life of the pregnant woman or of grave injury to her
physical or mental health; or that there is a not any substantial risk that if the child were born,
it would suffer from any serious physical or mental abnormality. Can she lawfully terminate
her pregnancy?

(a) No, she cannot legally terminate her pregnancy.

(b) Yes, she can legally terminate her pregnancy.

(c) Yes, she can legally terminate her pregnancy after the opinion of one Doctor.

(d) Yes she can legally terminate her pregnancy because she is not financially able,

30
Directions (Q. – Q.): Read the comprehensions carefully and answer the questions based on
it. Passage (Q.-Q.): Time has come to initiate action against persons who file frivolous
complaints under Sections 354, 354A, 354B, 354C, 354D IPC, etc. only for an ulterior
purpose, the Delhi High Court has just recently on February 23, 2021, in the latest decision
has imposed a cost of Rs. 30,000 on the petitioners with a warning not to file false and
frivolous cases. It is a national tragedy that we see that laws meant for protection of women
are fast turning into weapons of stabbing men in the most gruesome manner. It is a matter of
national disgrace that a man named Vishnu Tiwari had to suffer without committing any
offense 20 years of incarceration in connection with a false rape case filed against him by a
woman owing to an alleged land dispute in Lalitpur district in UP. Offenses under Sections
354, 354A, 354B, 354C, 354D IPC are serious offenses. Such allegations have the effect of
tarnishing the image of the person against whom such allegations are made. Sexual
harassment and punishment for sexual harassment. Section '354A- A man committing any of
the following acts[1]I Physical contact and advances involving unwelcome and explicit
sexual overtures; or ii. A demand or request for sexual favors; or iii. Showing pornography
against the will of a woman; or iv. Making sexually colored remarks shall be guilty of the
offense of sexual harassment. Section 354B - Any man, who assaults or uses criminal force to
any woman or abets such act to disrobe or compel her to be naked, shall be punished. Section
354C - Any man who watches, or captures the image of a woman engaging in a private act in
circumstances where she would usually have the expectation of not being observed either by
the perpetrator or by any other person at the behest of the perpetrator or disseminates such
image shall be punished on first the conviction with imprisonment of either description for a
term which shall not be less than one year, but which may extend to three years, and shall
also be liable to fine.

. X is boss of Y a female employee, after office hours X requested Y to watch pornography


content together for business purpose as a company is doing research on Hormonal control
methods but Y is not uncomfortable for some minutes and then she did not deny or show any
disapproval for watching porn. Will Boss is liable for sexual harassment to decide in context
of the passage.

(a) Yes, shall be guilty of the offence of sexual harassment because showing pornography
against the will of a woman is sexual harassment.
(b) Yes, he is guilty of the offence of sexual harassment as indirectly X demanding and
requesting for sexual favor from Y.

(c) No, showing pornography with the will of a woman is not sexual harassment.

(d) None of the above.

. What can be inferred from “It is a national tragedy that we see that laws meant for the
protection of women are fast turning into weapons of stabbing men in the most gruesome
manner” in context with the passage.

(a) All women are misusing laws that are meant for their protection against sexual
harassment.

(b) Law that protects women are being misused in a heinous way by some people against men
for ulterior motives.

(c) Law is enforceable in society for women's protection but misusing it in an unlawful way
is derogatory for society at large.

(d) Law is for providing justice and misuse of the same legal law is illegal in any legal
system.

. C a woman is changing clothes at a roadside Open restaurant. Unintentionally Z, an


employee in a restaurant watched her changing clothes. She filed a complaint against Z under
section 354C. Decide Z will be liable or not under Section 354C.

(a) Yes, Section 354C.Any man who watches, or captures the image of a woman engaging in
a private act in circumstances where she would usually have the expectation of not being
observed.

(b) Yes, 354C.Any man who watches, or captures the image of a woman engaging in a
private act image shall be punished on first conviction with imprisonment of either
description for a term which shall not be less than one year.

(c) No, circumstances in which she was changing the clothes, it has the possibility of being
observed.

(d) Both A and B

. Which is true in the context of the passage?

(a) Due to land dispute in Lalitpur district in UP between a woman and Vishnu Tiwari a false
rape case filed against him by a woman.
(b) False Rape allegations have the effect of tarnishing the image of the person against whom
such allegations are made.

(c) Law that protects women are being misused in a heinous way by some people against men
for ulterior motives.

(d) All of the Above

. What punishment should be given to women in the context of the passage? Who filled false
allegations against men?

(a) Fine up to Rs 30,000.

(b) Imprisonment of either description for a term which shall not be less than one year, but
which may extend to three years, and shall also be liable to fine.

(c) In accordance with the law in the force.

(d) None of the Above.

Passage (Q.-Q.): Supreme Court has issued new guidelines for all courts to adhere to while
passing bail orders in cases involving crimes against women. The judgment came on a plea
filed by nine women lawyers who contended that such bail orders have the effect of
trivializing sexual offenses. Guidelines for judges, lawyers to follow In its judgment, the
court passed seven key directions for courts to follow when bail orders are dictated. “The act
perpetrated on the survivor constitutes an offense in law and is not a minor transgression that
can be remedied by way of an apology, rendering community service, tying a rakhi or
presenting a gift to the survivor,” the court said. Among the key directions in the judgment
are that bail conditions should not mandate or permit contact between accused and victim
should seek to protect the victim, not discuss the complainant’s behaviour, dress, past
conduct, or morals and the complainant should immediately be informed about the bail order.
The bench also said courts must desist from expressing any stereotype opinion about women,
in words were spoken during proceedings, or in the course of a judicial order. Further, the top
court said the expressions that should not find their way into any verdict include phrases such
as “women are physically weak and need protection”, “women are incapable of or cannot
take decisions on their own”, “men are the head of household and should take all decisions
relating to the family” and “good women are sexually chaste”. “The causes and factors of
violence against women include entrenched unequal power equations between men and
women that foster violence and its acceptability, aggravated by cultural and social norms,
economic dependence, poverty and alcohol consumption, etc.,” the court said.

. Court gives bail order on condition to give half property of X convicted for sexual
harassment against women Y for the remedy of the offense of X and giving justice to Y. The
Bail order of the court is in line with Supreme Court new guidelines that all courts have to
adhere to while passing bail?

(a) No, It is a minor transgression that can be remedied by way of an apology.

(b) No, It is not a minor offense that can be remedied by presenting a gift to the survivor.
(c) No, this is against the law in force

(d) None of the above

. Tara was the victim of a sexual offense and during giving bail to accused bail conditions
discuss Tara's past behaviour of alleging false complaint against his boss and her Bar dancer
profession. Is this violation of guidelines issued by the Supreme Court?

(a) Yes, that bail conditions should not mandate or permit contact between accused and
victim should seek to protect the victim, not discuss the complainant’s behaviour, dress, past
conduct or morals.

(b) Yes, courts must desist from expressing any stereotype opinion about women, in words
spoken during proceedings, or in the course of a judicial order.

(c) Yes, the court should seek to protect the victim.

(d) Yes, such bail orders have the effect of trivializing sexual offenses.

. What can be inferred from “The causes and factors of violence against women include
entrenched unequal power equations between men and women that foster violence and its
acceptability, aggravated by cultural and social norms, economic dependence, poverty and
alcohol consumption, etc.”?

(a) Violence against is due to increasing alcohol consumption by their husbands.

(b) Violence against women is committed because many women are not economically
independent and they have to face abusive violence.

(c) Women are not given equal position like a man in society many spheres of their life,

(d) Both B and C.

. Which are following Guidelines are given by Supreme Court in context of the passage?

(a) That bail conditions should mandate or permit contact between accused and victim,
should seek to protect the victim, not discuss the complainant’s behaviour, dress, past
conduct, or morals.

(b) Courts must desist from expressing any stereotypical opinion about women, in words
spoken during proceedings, or in the course of a judicial order.

(c) Courts must desist from expressing any stereotype opinion about women, in words spoken
during proceedings, or in the course of giving bail.

(d) None of the above.

 
. Court in its verdict said that for ensuring safety and giving protection to women legislature,
executive and judiciary need to work together because women need protection due to
increase in crime against them. Decide in context of the last paragraph that using such the
phrase will violate guidelines of court?

(a) Yes using such phrase infers meaning that “women are physically weak and need
protection”.

(b) Court verdict does not violate guidelines because no stereotypical phrase being used in the
verdict.

(c) Verdict of the court says that women are incapable of protecting themselves.

(d) All are not true.

Passage (Q.-Q.): Negligence is the failure degree of care in conduct that a reasonable man in
situation would observe but this is just one way of formulation of this concept. Limiting
negligence to only this definition without further explanation would be obscure that there
could be inconsistent application to this principle to similar cases. One significant benefit of
anthropomorphizing the negligence test is to make vivid a negligence standard would
otherwise be abstract. Does a reasonable person standard amount only to a useful rhetorical
device for helping the fact-finder to analyse and apply considerations that are relevant? If
negligence should be understood as an unreasonable balance of advantages and disadvantages
of taking precaution, perhaps the reasonable man is just a person who does this balance. The
reasonable person formulation then adds nothing of substance to the content of negligence
test. Personification might serve 3 additional functions. First, it seems preferable to an
‘Impersonal’ Learned Hand Balancing test in accounting for cases of deficit skill in
conducting an activity. The surgeon whose hand slips during surgery, we describe their fault
if we try to characterize it as an unreasonable balance of taking precautions. Second, a
reasonable person can readily be calibrated along the dimension of civilizations in asking
what a reasonable person under all the circumstances would have done; we can relate
negligence to certain capacities and traits. This test is overly idealised and homogenous in
nature. A reasonable should be what a person would take if he/she were to have reasonable
beliefs and values. Third, this test precludes separate analysis of different dimensions of
unreasonableness. A final value of personification should include excused and justified
conduct.

. Mahadev Bus Services was the well known transport services in city of Bhopal which
covers every small routes and villages for wider services to be availed by the passengers.
They hired Prabhu as the bus conductor. He has his own private bus and asked the transport
services to use it under their services and they can pay the amount for the petrol and other
maintenance charges during his services. Prabhu, before starting his services at Mahadev
Transportation, decided to give his bus for maintenance and other repairing works to Kapil
Mechanics. After a week, he took his first ride from Bhopal to Indore. In the way, he realised
that breaks are not working properly and subsequently they met with an accident. Suraiya,
one of the passengers in the bus, filed a suit against Prabhu and Mahadev Bus Services for
negligence. Decide-
(a) She will win in her case as Prabhu and Mahadev Bus Services were negligent about the
break failures and should have applied due and reasonable care before providing the services.

(b) She will fail in her case against Mahadev Bus Services as they were not negligent, and it
was fault of both Prabhu and Kapil Mechanics who should have applied due and reasonable
care while providing bus services repairing bus respectively.

(c) She will win in her case against Prabhu and not Mahadev Bus Services as there was no
agent[1]principle relationship between them and Prabhu was negligent about the break
failures and should have applied due and reasonable care before providing the services.

(d) She will fail in her case against Prabhu and Mahadev Bus Services but can win against
Kapil Mechanic as he was the one who was negligent to check the break failures and should
have applied due and reasonable care.

. Priya Malik was a well-known athlete and she was a good swimmer. She bought a new
mansion and had constructed a swimming pool wide enough with filters to avoid
overflowing. She also hired a staff to carry out maintenance and cleaning services of the pool.
One day, she was out of the town for her fellowship programme. Staff person opened the
pipes filing the pool and left it unattended. Filters of the pool also got blocked. Soon the
swimming pool gets overflowed and the water runs out of the mansion to the public road.
Jatin, a biker and racer, was passing through outside her mansion and was in a high speed.
Due to wet and slippery road from the overflowed water, he met with an accident and crushed
through side of his face. He claimed compensation and damages from Priya for her
negligence. Decide-

(a) Priya is not liable for damages as she was not negligent. The act of overflowing of water
was just an accident and hence Jatin will fail in his suit.

(b) Priya is vicariously liable for damages as she was negligent in applying due care and
diligence. There exists principle-agent relationship between her and the staff and hence Jatin
will succeed in his suit for damages.

(c) Priya is not liable for damages as her staff was negligent in taking due care and diligence.
She cannot be held liable as principle for the negligent acts of her staff and hence Jatin will
fail in his suit.

(d) Priya is liable for damages as she was negligent. But Jatin will not succeed in his suit
because he was also negligent as he was driving bike in very high speed and he should have
taken reasonable care while driving.

. Manohar, a well known chef in Pyarelal Restaurent, was famous for his Grated Cheese
Pizza Patties dish. He usually purchases a packet of cheese cubes from the nearby shop in his
locality which is different in texture from other cheese cubes and that’s what makes his dish
delicious and different. On day, a customer, Mr. Sharma went to the restaurant and ordered
his favorite dish Grated Cheese Pizza Patties. He was served with the mouth- watering dish
and he tasted it. He didn’t like the taste as it was not as usual as before and complained the
manager of the bad dish. He smelled the same and stated that cheese used in the dish is giving
foul smell and asked to call the chef. On testing, it was found that cheese cubes brought by
Manohar were 6 months expired. Mr. Sharma gets to know this and claimed damages from
Manohar for his negligent act of using the expired cheese for his dishes. Decide-

(a) Mr. Sharma can validly claim damages from Manohar as he was negligent and he should
have taken reasonable care and due diligence before buying and using cheese in his dishes.

(b) Mr. Sharma can validly claim damages from both Manohar and restaurant as there exists
principle[1]agent relationship between them and under vicarious liability they are liable.

(c) Mr. Sharma cannot claim damages from Manohar as he was not negligent in his act and
it’s the shop owner who should have taken reasonable care and due diligence before selling
the cheese to its customers.

(d) Mr. Sharma cannot claim damages from Manohar and restaurant authorities as he had
wished for the dish and had consented for the same. He was negligent in ordering such dish
and he should have taken reasonable care for the same.

. Qui Facit per Alium Facit per se means he who does things through others does it himself.
Facts: Jetali and Mehul were best friends and were employed in the same office and under
same departmental heads. Jetali was not well and she decided to take leave for the day. She
asked Mehul to take her car to attend the meeting scheduled in their office and return back
the same to her by night. Mehul agreed to her and asked her to take rest. On the way to office,
Mehul was on a call and hit a pedestrian, Rahul who was crossing the road in a hurry. He was
injured badly and was taken to taken hospital. Decide-

(a) Rahul can claim damages from Mehul only as he was negligent while driving car and
having a call at the same time. He has a duty of reasonable care and due diligence against
Rahul and other pedestrians.

(b) Rahul cannot claim damages from Mehul as he was not negligent while driving the car.
He has not duty of reasonable care and due diligence against Rahul or other pedestrians.

(c) Rahul cannot claim damages from Mehul or Jetali as they were not negligent. Rahul must
have taken reasonable care while crossing the road.

(d) Rahul can claim damages from both Mehul and Jetali as Mehul was negligent while
driving car and having a call at the same time and Jetali being owner of the car is also liable.

. Choose the appropriate option.

Assertion (A): While determining the degree of negligence in the conduct of the person, the
standard of care by reasonable man has to be applied.

Reasoning (R): As legally defined, a reasonable person is one who acts carefully and/or
makes reasonable balance between advantages and disadvantages.

(a) Both A and R is correct and R is correct explanation of A.

(b) Both A and R is incorrect.


(c) Only A is correct and R is incorrect.

(d) Both A and R is correct but R is incorrect explanation of A.

. Various tests and principle of reasonable person has been discussed by the author in the
above passage with regards to negligence. What is the stand of the author while stating those
tests and their validity with the time?

I. The author has not find these tests to determine and apply the principle of standard
reasonable care perfect but a suitable option to be used as compared to other options
available.

II. She/he finds such test of reasonable person very interesting and appreciated the full proof
validity of the same with changing time.

III. The author seems to be very indifferent when it comes to the principle of reasonable man.

(a) The author has showcased the orthodox view while applying the test of reasonable man
and standard care for determining the negligent act. Both I and III

(b) Only IV

(c) Both II and IV

(d) I, III and IV

Passage (Q.-Q.): Full Bench of the Bombay High Court, Goa Bench on Friday held that the
Goa State Cooperative Bank Ltd. is not a 'State' nor does it fall within the ambit of 'any other
authority' for the purposes of Article 12 of the Constitution of India. The bench also went
ahead to observe that the said Bank does not discharge any public functions which would
warrant issuance of a writ in the nature of mandamus. The bench comprising of Justice M.S.
Sonak, Justice Dama Seshadri Naidu and Justice Bharati H. Dangre was dealing with a
reference order adjudicating major issues- Whether Goa State Cooperative Bank Ltd. is a
'State' or any instrumentality thereof, for the purposes of Article 12 of the Constitution of
India and whether it performs any public functions, which would warrant issuance of writ in
the nature of mandamus in discharge of its performance of the public functions? Two division
benches of the High Court in the cases of Ganesh Morto Naik vs. Goa State Co-operative
Bank and Surendra J. Kalangutkar vs. Goa State Cooperative Bank Ltd. held that the Goa
State Cooperative Bank Ltd. is a 'State' for the purpose of Article 12 and that since it is
discharging public functions, it is amenable to writ jurisdiction under Article 226 of the
Constitution of India. In another decision by the Full Bench of the HC in Shamrao Vithal Co-
operative Bank Limited vs. Padubidri Pattabhiram Bhat, it was held that Multi State Co-
operative Bank is not a 'State' within the meaning of Article 12, though it is governed by the
Banking Regulations Act, 19 and it performs public functions. By applying various tests as
developed by the Courts under Article 12, the Court observed that a writ in the nature of
mandamus would also lie against a private body, but only when such body performs any
public function. Therefore, a commercial business of banking though is a function of public
importance is not a public function. Providing life insurance cover comes under the ambit of
public function.

. Amit was the student of National Law University Jodhpur and was a studious child. One
day, he gets to know that his grandfather is sick and is admitted in ICU ward. He has to
urgently leave for his home in Amritsar to meet his grandfather. He was in dilemma as mid
semester exams were approaching in the coming week. He went to Amritsar without
informing the authorities and came back after two weeks. He was marked absent for his
exams and was marked zero for the same. He approached the administration to give him a
chance to justify his reasons for not attending the exams. Being a studious child, such action
would affect his total grades. But he was refused for any such hearing. He filed a case against
the university stating it to be a State within article 12 of the Constitution and thus, it cannot
violate his fundamental right of fair hearing. Decide-

(a) NLUJ is not a state within the ambit of article 12 of the Constitution as it was not serving
any public function and thus is not liable for violation of fundamental right.

(b) NLUJ is not a state but an instrumentality of state which is serving public function of
educating the law graduates and thus is liable for violation of fundamental right.

(c) NLUJ is not a state within the ambit of article 12 of the Constitution though it was not
serving public function of educating law graduates but is a private body and thus is not liable
for violation of fundamental right.

(d) NLUJ is a state within the ambit of article 12 of the Constitution as it was governed by the
state government and thus is liable for violation of fundamental right.

. Suyash was a agent of Life Insurance Corporation Ltd. (LIC). His oldest of all customers
was Mr. Bhide whom he had family relations too. Mr. Bhide deposits a sum of Rs. 0 every
month with Suyash for life insurance cover policy and get receipts for the same in return.
Suyash was not an honest employee and he cheated Mr. Bhide. He only deposits Rs. 500 per
month as life insurance cover and kept the remaining 500 for his personal use. This fraud
came into notice of Mr. Bhide when he went to LIC office and asked for his insurance
amount due for the remaining years. Mr. Bhide filed a suit for recovery of damages from
Suyash and compensation as well. The court decided in favour of Mr. Bhide and held Suyash
liable for the same. One of the issues brought up before the court was to held LIC to be liable
as there is principal-agent relationship between Suyash and LIC Ltd. The court went on to
seek the nature of LIC ltd. Decide-

(a) Life Insurance Corporation Ltd. is not a state as it giving life insurance cover does not
amount to public function and hence is not liable for any damages to Mr. Bhide.

(b) Life Insurance Corporation Ltd. is instrumentality of state as it discharges public function
of giving life insurance cover and hence is liable for damages to Mr. Bhide.

(c) Life Insurance Corporation Ltd. is not a state but a private body which does discharge
public function of giving life insurance cover but is not liable for damages to Mr. Bhide.
(d) Life Insurance Corporation Ltd. is not a state or instrumentality of state. The question
here for consideration is irrelevant as there do not exist any principal-agent relationship
between Suyash and LIC Ltd.

. K.L Rahul, one of the renounced cricketer of India was admitted in Indian Cricket Team by
BCCI (Board of Control for Cricket in India). For the new test series against England, BCCI
decided to give a chance to new players and had not selected K L Rahul for the series. He
was very disappointed. Seeing his last performances, he was expecting to be in the team for
this series and in this regard, he had a talk with the chairman of BCCI. He contended that he
should be in the team for this series and they can give chance to new comers in other
upcoming events scheduled. BCCI refused him again and asked him to wait for more
opportunities in the next month. He made a point to file a case against BCCI for unfair
practices in selecting players including the corruption charges. BCCI chairman asked him to
file the same and contended that BCCI is not a state that it will protect the fundamental rights
of the players. Decide-

(a) The contention of BCCI is correct as it is not a state within the meaning of article 12 as it
does not discharge any public function and hence is liable for infringing FR.

(b) The contention of BCCI is not correct as it is an instrumentality of state within the
meaning of article 12 as it discharges public function of representing India in game of Cricket
at global level and hence is liable for infringing FR.

(c) The contention of BCCI is not incorrect as it is not an instrumentality of state or state
within the meaning of article 12 as it is a private body discharging function of public
importance and hence is not liable for infringing FR.

(d) The contention of BCCI is incorrect as it is a state within the meaning of article 12 as it
discharges both public function and of public importance by representing India at global level
in game of Cricket and hence is not liable for infringing FR.

. In case of Rajasthan State Electricity Board (RSEB) vs. Union of India, it was held by the
court that RSEB is a state within the ambit of article 12 as it discharges functions similar and
under the guidance of state government of Rajasthan and can be said to be its instrumentality
for the purpose. Going by the ruling of the case, a private electricity board was set up by
name of Balaji Electricity Suppliers and was charging fewer fees for supplying electricity to
the connected areas. RSEB filed an action against the Balaji Electricity Suppliers for
violating the terms of market competition and performing similar function without any
authority. Balaji Electricity Suppliers contended that they are also the State within the
meaning of article 12 and RSEB cannot stop them from performing public function. Decide-

(a) Balaji Electricity Suppliers are not state or instrumentality of state although they perform
similar functions of RSEB. They are private body performing function of public importance.

(b) Balaji Electricity Suppliers are state or instrumentality of state as they perform similar
functions of RSEB. Though, they are private body performing public functions, they are very
well within the ambit of article 12.
(c) Balaji Electricity Suppliers are not state but instrumentality of state as they perform
similar functions of RSEB. They are performing function of public importance which are per
se public functions.

(d) Balaji Electricity Suppliers are state but not instrumentality of state. Although they
perform similar functions of RSEB but RSEB is instrumentality of state and not state per se.

. Which of the following statement(s) with respect to writ of Mandamus is/are incorrect? I. A
writ of mandamus is used to order a lower court or government agency to complete a duty to
uphold the law or to correct an abuse of discretion. II. A writ of mandamus is deemed
necessary when the actions (or inaction) of government bodies or corporate officials are so
inappropriate or egregious that immediate, emergency action needs to be taken by the legal
system. III. Scope of Writ of Mandamus also extends to Chief Justice of Supreme Court and
other distinct High Courts. IV. The writ of mandamus can be issued against the Head of the
State, that is, the President on a national level or Governor at the state level.

(a) Both II and IV

(b) Only I

(c) Both III and IV

(d) Both I and III

Passage (Q.-Q.): Section 1C(1) forbids the use of undue influence at elections. As per the
section, undue influence is the interference with the free exercise of any electoral right.
Section 1C (2) specifies the ways in which undue influence can be exercised. As per clause
(b) of the subsection, a person who induces or attempts to induce any candidate or voter to
believe that he or any other person in whom he is interested would face spiritual censure or
divine displeasure, commits the offence of undue influence at elections. The purpose of
section 1C and that of other sections under chapter IX-A (offences related to elections) of
IPC is to ensure that elections are conducted freely and the righteous freedom of any voter or
candidate (with respect to elections) is not adversely affected. To truly ensure this righteous
freedom, it is important to appreciate that undue influence at elections is not limited to overt
threats of divine displeasure. The opening of sub-section (2) which reads “without prejudice
to the generality of provisions of sub-section (1)” conveys that a specific mention of divine
displeasure in sub[1]section (2)(b) doesn’t render other religiously based acts (which can
unduly influence the elections) out of the scope of the section. In Shiv Kirpal Singh’s case,
the SC laid down a relatively liberal definition of undue influence. According to the decision
an element of threat or compulsion is not a sine qua non for establishing undue influence.
Also, it is not necessary that an actual interference with free exercise of electoral right is
caused and that even an attempt to do so would amount to undue influence. Moreover,
considering the huge diversity in India and therefore the significance of religion in Indian
politics, it is important that section 1C is not read in a literal and narrow sense.

. Meenal was contesting for upcoming Uttar Pradesh elections of 2021. She asked her
husband to help her out for votes and make her win these elections. Her husband was not
much happy with her wife contesting the elections and wants her candidature to be rejected.
They called a Priest and asked about her winning chances. The Priest forecasted that she will
win by a good majority number but goddess Durga will not be much happier of her winning.
She should not contest these elections. Decide-

(a) The forecast by Priest was in form undue influence and fall under the said section 1C
amounting to divine displeasure.

(b) The forecast by Priest was not in form of undue influence and does not fall under the said
section 1C i.e divine displeasure or spiritual censure.

(c) The forecast by Priest amounts to only undue influence but do not fall under section 1C
amounting to divine displeasure or spiritual censure.

(d) The forecast by Priest doesn’t amount to undue influence but was only a divine
displeasure not falling under section 1C.

. Banku, a minister wants to contest elections again for the upcoming Karnataka elections
2021. He filed his candidature on 1st January 2021 and starts preparing for influencing votes
by rallies, posters etc. On 5 th January 2021, he met with fortune teller and asked him that he
will lose the elections and he should not contest the same. On 7th January 2021, his
candidature was rejected due to invalidity. He filed a complaint against Prakash, the
opponent, as he got to know that fortune teller was fraud send by him only. Decide-

(a) Banku can file a valid complaint against such rejection for his candidature as it was fraud
in name of divine displeasure.

(b) Banku cannot file a valid complaint for his candidature as it was rejected for reasons of its
invalidity and not undue influence.

(c) Banku can file a valid complaint for candidature as the act of Prakash falls under
section1C i.e divine displeasure and spiritual censure.

(d) Banku cannot file any valid complaint against such rejection for his candidature as there
was no undue influence. Only spiritual censure under section 1C.

. Meenakshi was a house wife of Mr. Ratan. Rattan does not want her to get into politics and
asked her not to file candidature for upcoming Panchayat elections of their village.
Meenakshi doesn’t listen to him and made him to agree with her decision. Soon, she filed the
candidature and before the day of voting, she withdraws the same. The council asked her the
reasons and she stated that she doesn’t want to contest while making her husband and family
upset from her decision. Ratan was glad to hear that and in her place filed his nomination.
Decide-

(a) The candidature of Meenakshi was withdrawn due to undue influence exercised by her
husband and falls under section 1C.
(b) The candidature of Meenakshi was withdrawn not due to undue influence of her husband
and her family but because of her own voluntary decision. She does not want to contest the
same.

(c) The candidature of Meenakshi was withdrawn due to only undue influence of Ratan but
does not fall within section 1C.

(d) The candidature of Meenakshi was withdrawn was due to other reasons than undue
influence but they form part of section 1C.

. Anant , an MLA, of Chattarpur was again contesting elections for year 2021. His opposition
candidate Munna was not happy as he has fair chances of losing the elections. He asked
Anant to withdraw his candidature, but Anant refused to do so. Munna warned him time
again and threatened him to take back his candidature or else he will kill his family members.
Anant, to save his family, took back his candidature and eventually Munna won the elections.
Decide-

(a) Anant has taken back his candidature due to threat and compulsion of Munna and said it
to be undue influence under section 1C.

(b) Anant has taken back his candidature but not due to threat and compulsion of Munna and
cannot be said to be undue influence under section 1C.

(c) Anant has taken back his candidature due to threat and compulsion of Munna but it does
not amount to undue influence under section 1C.

(d) Anant has taken back his candidature due to threat and compulsion of Munna and amount
to undue influence but not under section 1C.

. Choose the most appropriate option. Assertion: Right to vote and right to file candidature
are one of the important part of Indian democracy and are categorised as civil rights of an
Individuals. Reason: Civil rights are secured by positive government actions in contrast to
civil liberties which are secured by restraints on government.

(a) Both Assertion and Reason are true but Reason is not correct explanation of Assertion.

(b) Only Assertion is correct and Reason is incorrect.

(c) Both Assertion and Reason are true and Reason is correct explanation of Assertion.

(d) Both Assertion and Reason are incorrect.

Passage (Q.-Q.): It was clearly laid down as early as in 13 in Raj Bahadur v. State of W.B.,
that traffic in human beings means to deal in men and women like goods, such as to sell or let
or otherwise dispose of. It would include traffic in women and children for immoral or other
purposes. The Supreme Court in Bandhua Mukti Morcha v. Union of India, elucidated the
rehabilitation of bonded labour and directed the Government to award compensation to
released/rescued bonded labour under the provisions of Bonded Labour System (Abolition)
Act, 19 after taking note of serious violation of fundamental and human rights: “The bonded
labourer who is released would prefer slavery to hunger, a world of ‘bondage and illusory
security’ as against a world of freedom and starvation.” The Constitution of India also
incorporates several safeguards in this respect. Article 23 – It specifically prohibits “traffic in
human beings and begar and other similar forms of forced labour”. Article 39 – It states that
men and women should have the right to an adequate means of livelihood and equal pay for
equal work; that men, women and children should not be forced by economic necessity to
enter unsuitable avocations; and that children and youth should be protected against
exploitation. It is enshrined in the Constitution in the form of a directive to be followed while
formulating policies for the State.

. X employs a number of street children or urchins and sends them to people’s houses on
contract basis to clean houses. He keeps the children in a bungalow where he gives them two
meals a day. Whatever money is earned by the children; X keeps all of it without giving any
share to the children. When a child turns 18, X gives an option to either continue to
participate in the business the same way or leave the bungalow and find other ways to
survive. Is this a violation of the principles mentioned in the passage?

(a) Yes, because “traffic in human beings and begar and other similar forms of forced labour”
are prohibited under Article 23 which includes making children work for immoral purposes

(b) Yes, because women and children should not be forced by economic necessity to enter
unsuitable avocations and the same is prohibited under Article 39

(c) No, because the children were working out of their own will and were not being exploited
but rather given a place to stay and food to consume

(d) None of the above

. Y lost his father at a very young age. His mother tried to apply several times to the
unemployment bureau to earn some money but she was never able to secure a job. This
forced Y’s mother to enter into prostitution, even though she detested the same. As bigger
expenses fell upon the family, such as paying for Y’s education, Y also had to resort to taking
up odd jobs like cleaning the sewers, even though he was just 10 years old. Is the condition of
Y’s family in violation of Article 39?

(a) Yes, because prostitution and cleaning of sewers amounts to begar or forced labour as
well as human trafficking

(b) Yes, because women and children should not be forced by economic necessity to enter
unsuitable avocations and should not be exploited

(c) No, because prostitution by women and cleaning of sewers by children does not amount to
exploitation as they are respectable means of earning a livelihood

(d) No, because Article 39 is a directive principle of state policy and not a fundamental right
whose violation can be mandatorily claimed
 

. A group of 30 girls were rescued from the home of a drug dealer. He used to sell these girls
to rich households in other countries so that they could be used as objects for any purpose by
those households. The girls were kidnapped or taken from their parents at a young age under
false pretences in exchange for a meagre amount of money given to their families. Is such a
business in violation of the principles mentioned in the passage?

(a) Yes, because it amounts to human trafficking as the girls are being treated like goods
which are bought and sold

(b) Yes, because the girls are being forced by economic necessity to enter unsuitable
avocations like this

(c) No, because it not begar or forced labour as the girls are not being forced to provide
services for free or a very minimum charge

(d) No, because there is no economic exploitation of the girls as their families were paid
money in exchange for their services

. Rama and Ramesh were working in the same company. Rama was paid a salary of Rs.
20,000 for her work as an assistant to the regional manager while Ramesh was paid a salary
of Rs. 35,000 for his work as assistant regional manager. Can Rama challenge this as
arbitrary and claim the same salary as Ramesh?

(a) Yes, because Article 39 provides that men and women should not be forced by economic
necessity to enter unsuitable avocations

(b) Yes, because Article 39 provides that men and women should have the right to an
adequate means of livelihood and equal pay for equal work

(c) No, because Rama and Ramesh are not doing the same work to claim the same salary so
there is nothing arbitrary about their earnings

(d) No, because Rama is being exploited as she is not being paid the same salary for the same
work

Passage (Q.-Q.): State of mind to commit a particular crime "must be visible" to determine
the culpability for offence of abetment, the Supreme Court has said while setting aside the
conviction of a man who was accused of abetting his wife's suicide in 19. The apex court said
that ingredient of "mens rea" (intention) cannot be assumed to be ostensibly present but has
to be "visible and conspicuous". A bench set aside the order of the Punjab and Haryana High
Court which had upheld the trial court's verdict convicting the man for the offence under
section 306 (abetment of suicide) of the Indian Penal Code. "As in all crimes, mens rea has to
be established. To prove the offence of abetment, as specified under sec of the IPC, the state
of mind to commit a particular crime must be visible, to determine the culpability," said the
bench. An FIR was lodged in the matter in August 19 at Barnala on the basis of statement of
the deceased's father and the prosecution had alleged that the woman was harassed after
marriage for insufficient dowry. The apex court noted that there is no direct evidence of
cruelty against the husband or the in-laws in the case. 26 of 36 "Insofar as the possible reason
for a young married lady with two minor children committing suicide, in the absence of
evidence, conjectures cannot be drawn that she was pushed to take her life by the
circumstances and atmosphere in the matrimonial home," the bench said. "In view of the
foregoing, we are persuaded to conclude that the decisions under challenge cannot be legally
sustained. Consequently, interfering with the impugned judgment of the high court and the
trial court, the appellant's conviction under section 306 IPC is set aside and quashed," it said.
Source: The New Indian Express

. Arya and Bhakti went for hunting deer. Arya knows Zia (her arch enemy) to be behind a
bush. Bhakti does not know it. Arya induces Bhakti to fire at the bush. Bhakti fires and kills
Zia. Will Arya and Bhakti be held liable for murder?

(a) Arya had mens rea but no actus reas. Bhakti had actus rea but no mens rea. No one is
guilty.

(b) Arya induced Bhakti to fire at the bush with the knowledge that Zia is there. Arya is
guilty of murder but Bhakti is not guilty of any offence.

(c) Both Arya and Bhakti are guilty.

(d) None of the above.

. Punnu was in his car when he was approached by a police officer who told him to move the
vehicle. Punnu did so, reversed his car and rolled it on to the foot of the police officer. The
officer forcefully told him to move the car off his foot at which point Punnu swore at him and
refused to move his vehicle and turned the engine off. Punnu was convicted for assaulting a
police officer, whereas Punnu argues that it was an accident Was it an accident?

(a) He is not liable because there cannot be an assault in omitting to act and that driving on to
the officer's foot was accidental, meaning that he was lacking mens rea when the act causing
damage had occurred.

(b) He is not liable as the act neither amount to an attempt nor a threat to commit an offence
that amounts to an actionable act of assault.

(c) Punnu's crime was not the refusal to move the car but that of having driven on to the foot
of the officer and decided not to cease the act, he had established a continual criminal act.

(d) He is neither liable for assault nor battery as he accidently drove his car on the police
officer's foot.

. Amit while on a jungle safari, is attacked by a lion and lion drags him while he is crying for
help. Pooja, a co-passenger in Amit’s safari picks up Amit’s gun in good faith and fires at
lion which injures Amit. Pooja has never used the gun before. Decide the liability.
(a) Pooja is liable for the injury because she knew that she can injure Amit as she has never
used any gun before

(b) Pooja is not liable as she has done the act in good faith and without any mens rea.

(c) Pooja is liable because she has not taken A’s consent before firing

(d) Pooja is liable because she has used Amit’s gun without his consent

. Osama, a terrorist having links with various extremist organizations, plans various attacks
on major metropolitans of the country during the festive week of Durga Pooja. He hires a
small-time gangster, Ballu Badshah, and instigates him by brainwashing him to help out with
the attacks. Ballu Badshah asks his brother Chota Chetan, who is a chartered accountant to
arrange funds without telling him his real motives. Chota Chetan arranges the funds and the
attacks are carried out as planned. Can Osama be held liable to abet the attacks by instigating
Ballu Badshah to arrange it?

(a) Osama can be held liable to abet the attacks by instigating Ballu Badshah since he
brainwashed him to arrange it.

(b) Osama cannot be held liable since Ballu Badshah did not directly carry out the attacks.

(c) Osama can be held liable to abet the attacks since he was the mastermind behind it.

(d) Osama cannot be held liable to abet the attacks since he did not actively engage with
Ballu Badshah to plan the same.

. Can Ballu Badshah be held liable to abet the attacks by intentionally engaging with Osama
and Chota Chetan to carry out the attacks?

(a) Ballu Badshah cannot be held liable to abet by intentionally engaging with Osama and
Chota Chetan since he was brainwashed.

(b) Ballu Badshah can be held liable to abet by intentionally engaging with Chota Chetan to
carry out the attacks but not with Osama since he was brainwashed by Osama.

(c) Ballu Badshah cannot be held liable to abet by intentionally engaging with Chota Chetan
since Chota Chetan did not know of the actual purpose but he can be liable to abetment by
intentionally engaging with Osama.

(d) Ballu Badshah can be held liable to abet by intentionally engaging with both Osama and
Chota Chetan.

. Can Chota Chetan be liable of abetment by intentionally aiding the carrying out of the
attacks by arranging for the funds for the same?
(a) Chota Chetan can be held liable for abetment by aiding since he arranged for the funds for
the attacks.

(b) Chota Chetan cannot be held liable for abetment by aiding since he did not know the true
motives of his brother and did not intentionally aid in the carrying out of the attacks.

(c) Chota Chetan can be held liable for abetment since intention and knowledge is irrelevant
to be held liable for abetment.

(d) Chota Chetan cannot be held liable for abetment since he did not actively engage in the
carrying out of the attacks

31
Passage (Q.-Q.): The bench of Justices Chandrachud, M. R. Shah and Sanjiv Khanna was
considering the contours of the power of quashing under section 4, CrPC and the power to
grant interim relief by way of bail/anticipatory bail, stay on coercive steps i.e. a stay on arrest
and investigation. "We know that the door cannot be closed completely. But to what extent it
is to be opened is the question", noted Justice Khanna. The bench had yesterday expressed
concern regarding the rising trend among High Courts across the country to routinely grant
interim relief by way of a stay on any coercive action pending a writ petition or a plea under
section 4, CrPC for quashing of criminal proceedings. "We have to balance between the
rights of the accused and the investigation", observed Justice Chandrachud. “Like you said,
there is a difference between the registration of the FIR for investigation and arrest. The latter
needs a stricter standard. Of course Sections 438 and 439 (CrPC, anticipatory bail and bail)
are there, but can it be said that in all cases, bail cannot be granted? In the Supreme Court, we
may dismiss a plea but we grant liberty to take appropriate recourse. We also say that one day
notice be given. It is our way of saying that a liberal approach be taken and that the
application must be heard”. Section 41(1) (a) of the CrPC says that the police officer 'may'
arrest (a person believed to have committed a cognisable offence). The court has construed it
to mean that the person need not necessarily be arrested. But whether there is to be arrest or
not, unless it is an anticipatory bail plea, it is not a power which the High Court will normally
exercise. When the High Court entertains 4 for quashing, and it is of the opinion that a prima
facie ground for quashing is made out, it may grant interim protection. Stay of the
investigation is very, very serious. It should not be granted unless there is something
extraordinary.

. Manohar, a boy of 19 years of age, was living in a slum area of Delhi. He was abandoned by
his family and had no sufficient resources to have his living. To serve his need, he indulged
in bad practices of theft, dacoity, robbery etc, which gives his enough earning to feed his
stomach a day. One day he was caught by police officer while committing theft. He was
stealing bread from the bakery shop. Trial was commenced and in the hearing the district
court punished him for imprisonment of 5 years for theft. But HC dismissed the trial court
order and quashed the proceedings against Manohar wholly. Also, he was acquitted and no
such trial was again commenced against him for theft. Choose the most appropriate-
(a) High court can quash the criminal proceedings against Manohar under Sec. 4 as he has not
committed the theft as of serious offence and imprisonment of 5 years for stealing bread is
not justified.

(b) High court cannot quash the criminal proceedings against Manohar under Sec. 4 as he has
been convicted by the trial court and power to quash the proceedings is not extended post trial
is over.

(c) High court can quash the criminal proceedings against Manohar under Sec. 4 as he has
committed trivial theft of stealing the bread and HC can grant interim relief to the accused
Manohar.

(d) High court cannot quash the criminal proceedings against Manohar under Sec. 4 as there
has to be balance between rights of accused and the law. He should be convicted for 5 years
to teach others not to commit such theft further.

. Pintu, renounced MLA of Bijay Janta Dal, was chairman of running NGO “We are for the
People”. Recently, in Covid-19 period, the NGO was indulged in social work to grant relief
and accommodation to people who are far away from their homes. But there were some
scams in accounting books of the NGO. A major fund was allotted to the NGO from the state
government and all activities were conducted by these funds. There was no definite source of
income for the NGO. Soon it came into notice that these funds were not utilized completely
by the NGO but was used by Pintu to purchase land in his own name in State of Goa. Soon,
CBI approached the NGO for investigation. Pintu approached the High court for stay order
on arrest and investigation and quash the proceedings against him. HC under sec. 4 passed
the interim order for the same granting relief to Pintu. Decide-

(a) High court was correct in granting interim relief to Pintu as court has the power to quash
the proceedings and stay investigation and arrest under sec. 4.

(b) High court was not correct in granting interim relief to Pintu as court has the power to
quash the proceedings and stay investigation and arrest under sec. 4 but only in extraordinary
cases.

(c) High court was correct in granting interim relief to Pintu as court can do so to protect the
rights of the accused person and thus, balance it out with its powers under sec. 4.

(d) High court was incorrect in granting interim relief to Pintu as court does not has power to
quash the proceedings and stay investigation and arrest under sec. 4 but can do so where there
are political interference as it amounts to extraordinary circumstances.

. Mrinal was having a love affair with her friend Mohan. They both were in live in
relationship for three and a half month and taking the opportunity he called his other friends
to take the moment in hand and sexually assaulted Mrinal. They gang raped her and left her
alone on the busy road. She filed a complaint against Mohan and his friends. During the trial,
the prosecution presented the evidence where cum spots were observed on Mrinal clothes and
some of the blood stains. The defence lawyer asked the court to grant them the copy of
evidences presented by the prosecution before the accused can be cross[1]examined. The
court can-

(a) Provide the accused with the evidence presented before the court as a matter of right. The
court cannot refuse the accused for copy of such evidence before cross examination.

(b) Do not provide the accused with the evidence presented before the court as accused have
no right to such evidences in such serious offences. The court can refuse the accused for copy
of such evidence before cross examination.

(c) Provide the accused with the evidence presented by the prosecution as it is an integral part
of the proceedings else it will not amount to irregularity in the proceedings and injustice to
accused.

(d) Do not provide the accused with the evidence presented by the prosecution. The court can
refuse the accused for such copy and balance out between right of accused and victim too.

. Section 4 of CrPC, which deals with the power of the court to quash criminal proceedings,
hasn’t given the details of that what exactly constitutes the inherent power of the court. In
that sense, the Code is very vague as it doesn’t lay out the grounds on which the foundations
of the inherent power of court lay. Furthermore, there has been consistent inconsistency in the
judgments of the Supreme Court of India with regard to the application of Section 4 of CrPC.
Consequently, the application of Section 4 of CrPC is a very agitated issue in litigation along
with being a strongly debated concept in the legal academic circles. In light of the passage
choose the most appropriate among the following which summarizes it-

(a) Section 4 preserves the inherent powers of the High Court to prevent an abuse of the
process of any court or to secure the ends of justice but such discretion has to be exercised
judicially.

(b) The invocation of the jurisdiction of the High Court to quash a criminal proceeding on the
ground that a settlement has been arrived at between the offender and the victim is same as
the invocation of jurisdiction for the purpose of compounding an offence.

(c) Criminal cases which have an overwhelming or predominant element of a civil dispute,
stand on a distinct footing in so far as the exercise of the inherent power to quash is
concerned.

(d) In the exercise of the power under section 4 and while dealing with a plea that the dispute
has been settled, the High Court must have due regard to the nature and gravity of the offence
and such judgment which impacts public at large.

. Choose the correct option[1]

Assertion (A): Cognisable offences are those where police officer can arrest the accused or
convict without any warrant and mostly in non-bailable offences.
Reasoning (R): Offences like Murder, Rape, Forgery, and Dowry Death are heinous crimes
and best examples of cognizable offences.

(a) Both A and R is correct and R is correct explanation of A.

(b) A is correct and R is incorrect.

(c) Both A and R is correct but R is not correct explanation of A.

(d) A is incorrect but R is correct.

Passage (Q.-Q.): When the Supreme Court decided T.T. Antony no doubts remained that a
second FIR could not be registered for the same incident.“However, the sweeping power of
investigation does not warrant subjecting a citizen each time to fresh investigation by the
police in respect of the same incident, giving rise to one or more cognizable offences,
consequent upon filing of successive FIRs whether before or after filing the final report”.
Then in Upkar Singh [(2004) 13 SCC 2] it was clarified that a second FIR for other offences
in the same transaction is not always barred when new facts come to light. A decade later in
Anju Chaudhary [(2013) 6 SCC 3] and Amitbhai Shah [(2013) 6 SCC 348], different benches
of the Court re-iterated the narrowness of the Second FIR logic, and in Anju Chaudhary it
was stated that. “It will, thus, be appropriate to follow the settled principle that there cannot
be two FIRs registered for the same offence. However, where the incident is separate;
offences are similar or different, or even where the subsequent crime is of such magnitude
that it does not fall within the ambit and scope of the FIR record first, then a second FIR
could be registered.” The Court has consistently held that a multiplicity of criminal
proceedings for the same cause of action is impermissible. At the same time, the remedy it
has crafted is necessarily post-facto and requires in-depth examination of the multiple cases
to determine their purported similarity. As it is probable that the cases may fall across state
boundaries, it also means that the only court unequivocally competent to hear such cases is
the Supreme Court itself. All of which creates a fairly long period of time in which there is no
protection for accused persons from various state police forces; police forces which spend
taxpayer money investigating into the same underlying acts.

. Abhinav and Rubina were happily married couple. Rubina’s family were not in good terms
with Abhinav’s family. Rubina’s brother Rohan thought of murdering Abhinav and planned
to consume him poison. He went to their house for business purpose asked Rubina to offer
him and Abhinav cup of tea. He secretly added poison in cup of Abhinav and administered it
to him. Soon Abhinav died. FIR was lodged against Rohan by Rubina and police investigated
the case. Trial was commenced and charge sheet was filed against Rohan for murder charges
of Rohan. Later on an evidence stage, a bill was procured by the investigating agency which
has to be presented before the court. The bill states that Rohan has purchased the poison from
the shop on the same day when Abhinav was murdered. Rubina filed another FIR against
Rohan for planning and conspiring against Abhinav. Decide-

(a) The second FIR exists as there were new charges framed against Rohan by Rubina and the
new evidence can only be presented by way of subsequent FIR.
(b) The second FIR fails as the new charges framed against Rohan by Rubina can be added to
the existing charges and their need not be any subsequent FIR for new evidence.

(c) The second FIR exists as there were new evidences evolved in the case and can only be
presented before the court by way of filing of second FIR and new charge sheet.

(d) The second FIR fails as there subsequent FIR can only be filed when there are
extraordinary circumstances in the case and not on new evidences.

. Piyush was one of the best racers in his group and bets his friends to beat him in his score to
complete the whole 500m distance in his time record. One of his enemies planned to kill
Piyush and let it be come out as accident. He planned to drive a truck with heavy load and run
it with high speed over the same track which Piyush will follow for the race. Soon the race
started, the plan was executed and Piyush get crushed under the truck. The teal stall owner
saw Ramesh to be at the spot, the driver of the truck and complained of him in the nearby
police station. A zero FIR was lodged and police get to the accident site for investigation.
Piyush’s friend also gets to know about the accident and was known that it all is done by
Ramesh. They filed another FIR against Ramesh for murder in another Police Station.
Decide-

(a) FIR lodged by friends can be considered to be second FIR and can be quashed
subsequently as first FIR has already been lodged before.

(b) FIR lodged by friends cannot be considered to be second FIR and cannot be quashed
subsequently as zero FIR is not considered to be the first FIR.

(c) FIR lodged by friends is the second FIR and has to be quashed subsequently zero FIR has
been filed by the tea stall owner with regards to the same incident.

(d) FIR lodged by friends is not a second FIR and should not be quashed subsequently as zero
FIR was lodged for accident and not murder.

. Sanjay Dutt, a popular singer and renounced Member of Parliament, was accused of charges
of corruption and had been investigated by the Central Bureau of Investigation (CBI). FIR
was lodged in this regard under Prevention of Corruption Act. Also, Enforcement Directorate
goes by the findings of the CBI and framed a case of money laundering. Another FIR was
lodged by ED against Sanjay Dutt. Trial was commenced in the trial court for corruption
charges. The defence lawyer raised contention for filing of second FIR by ED in the same
case and asked the court to quash the same. Decide-

(a) The contention put forth by the defence lawyer was correct as two FIRs have been lodged
by the CBI and ED in the same case against Sanjay Dutt.

(b) The contention put forth by the defence lawyer was incorrect as FIRs lodged by the CBI
and ED in the case against Sanjay Dutt is not same and constitutes different offences.
(c) The contention put forth by the defence lawyer was not incorrect as ED and CBI, though
are different agencies but has contended the same charges against Sanjay Dutt in both the
FIRs.

(d) The contention put forth by the defence lawyer was not correct as FIR by ED was not
valid one as it was a second FIR and only FIR by CBI will be relevant for the purpose.

. Multiple cases for the same underlying offence potentially expose a person to double
jeopardy — not only in the limited sense of double punishment, but the hazards of facing the
threat of arrest from different police forces as well during an investigation. Double Jeopardy
law in India essentially protects a person from multiple punishments or successive
prosecution based on same facts of a case where the elements of multiple prosecutions are
similar to those for which the accused has already been prosecuted or has been acquitted by
the court. Facts- Mr. A charged for smuggling foreign goods after enquiry was held by
Customs authorities, smuggled goods were seized and confiscated. He was charged with
penalties too. On petition against Mr. A, magistrate grant relief to the appellant. Mr. A
claimed before the Apex Court for violation of Double Jeopardy principle. Decide-

(a) There was violation of principle of double jeopardy as Mr. A was penalized, charged and
prosecuted by the customs authorities prior to the trial by the court.

(b) There was no violation of principle of double jeopardy as Mr. A was not prosecuted twice
for the said offence as there were different competent authorities trying the case.

(c) There was violation of principle of double jeopardy as Mr. A was prosecuted twice for the
said offence by two different competent authorities in the present case.

(d) There was no violation of principle of double jeopardy as Mr. A was not penalized,
charged and prosecuted by the customs authorities prior to the trial by the court.

. Discussed in the passage the history of Second FIR cases, which of the following make
obvious the limitations in this remedy? I. Its costs, as going to the Supreme Court and getting
a relief is not cheap and something only the rich and powerful can afford. II. The risk in
doing top-down litigation, as any observation from the Supreme Court can sway the course of
the litigation before lower courts. III. The necessarily standards-based nature of the
evaluation of facts. IV. The lack of any preventive aspect.

(a) Both I and III

(b) Only IV

(c) All of the above

(d) I, II and IV

 
Passage (Q.-Q.): The government has notified a consolidated list of rights that Overseas
Citizens of India (OCI) enjoy. The notification in the Gazette on granted OCIs the right of
multiple entry lifelong visa to India for any purpose; exemption from registration with the
Foreigners’ Regional Registration Officer (FRRO) for any length of stay in India; and parity
with Indian nationals in the matter of domestic airfares, entry fees to monuments and public
places. It also said that OCI cardholders will enjoy parity with NRIs in the adoption of
children, appearing in competitive exams, purchase or sale of immovable property barring
agricultural land and farmhouses, and pursuing professions such as doctors, lawyers,
architects, and chartered accountants. However, many of these rights have been subjected to
certain restrictions. The notifications say that the right of multiple entry lifelong visa for “any
purpose” can be enjoyed provided the OCI obtains special permission from “the competent
authority” or the FRRO “to undertake research; to undertake any Missionary or Tabligh or
Mountaineering or Journalistic activities; to undertake an internship in any foreign
Diplomatic Missions or foreign Government organizations in India or to take up employment
in any foreign Diplomatic Missions in India; to visit any place which falls within the
Protected or Restricted or prohibited areas as notified by the Central Government or
competent authority”. There will be no restriction in visiting religious places and attending
normal religious activities like attending religious discourses. However, preaching religious
ideologies, making speeches in religious places, distribution of audio or visual
display/pamphlets pertaining to religious ideologies, spreading conversion, etc. will not be
allowed,” The notification says OCIs can appear for all-India entrance tests such as National
Eligibility cum Entrance Test (NEET), Joint Entrance Examination (Mains), Joint Entrance
Examination (Advanced), or such other tests to make them eligible for admission only against
any Non-Resident Indian seat or any supernumerary seat. SOURCE: THE Indian Express

. Abdul khan is an OCI cardholder who wants to do research work in Nlsiu and entered India
without taking permission from the competent authority. Decide Abdul violated OCIs rules.

(a) No, The notification in the Gazette on granted OCIs the right of multiple entry lifelong
visas to India for any purpose;

(b) Yes, the right of multiple entry lifelong visas for “any purpose” can be enjoyed provided
the OCI obtains special permission from “the competent authority” or the FRRO “to
undertake research

(c) Yes, many of these rights have been subjected to certain restrictions

(d) None of above

. Zara Ali an OCI cardholder wanted to do worship in Jama masjid and want to deliver a
lecture on the importance of religion in human life at Delhi University. Will she be allowed?

(a) Yes, there will be no restriction in visiting religious places and attending normal religious
activities like attending religious discourses.

(b) No, preaching religious ideologies, making speeches in religious places, distribution of
audio or visual display/pamphlets pertaining to religious ideologies, spreading conversion,
etc. will not be allowed,”.
(c) No, preaching religious ideologies, making speeches in religious places, distribution of
audio or visual display/pamphlets pertaining to religious ideologies, spreading conversion,
etc. will be allowed,”.

(d) Yes, there will be no restriction in visiting religious places, and attending normal religious
activities like attending religious discourses and giving a speech in religious places is not
allowed and Delhi University is not a religious place.

. Amir Khan aware of his OCI rights in India and knowledge of OCI rights in parity with
NRIs rights want to buy unfertile agriculture land for industry purpose. Amir Khan is
eligible.? answer with reasoning.

(a) Yes, OCI cardholders will enjoy parity with NRIs in the adoption of children, appearing
in competitive exams, purchase or sale of immovable property agricultural land and
farmhouses.

(b) Yes, he will be allowed because he is purchasing unfertile agricultural land.

(c) No, OCI cardholders will enjoy parity with NRIs in the adoption of children, appearing in
competitive exams, purchase or sale of immovable property barring agricultural land and
farmhouses

(d) Both A and B

. Sameena Haider OCI cardholder is an ambitious girl who wants to do engineering from the
prestigious Indian Institute of technology and therefore she appeared in JEE mains and got
selected. Sameena will be eligible to give a competitive exam and take a seat in India.
Explain.

(a) The notification says OCIs can appear for all-India entrance tests against any seat.

(b) Only against any Non-Resident Indian seat.

(c) Both A and B

(d) None of above

. Mantasha is an OCI cardholder and wanted to do prayer in Ram mandir Ayodhya but denied
by authorities on grounds of non-eligibility of OCI to do worship in religious places in India.
Our authorities are right in their conduct?

(a) No, because there will be no restriction in visiting religious places and attending normal
religious activities like attending religious discourses.

(b) Yes, because there is a restriction in visiting religious places and attending normal
religious activities like attending religious discourses.

(c) Yes, she will be preaching religious ideologies and making speeches in religious places.
(d) Both A and B.

Passage (Q.-Q.): Failure of the State government to provide basic shelter to homeless persons
is a violation of the right to life guaranteed under Article 21 of the Constitution of India, the
Karnataka High Court said on Tuesday. "State must be conscious of the fact that its failure to
provide basic shelter to urban homeless may amount to violation of Right to Life guaranteed
by Article 21 of Constitution of India. Article 21 reads as: “No person shall be deprived of his
life or personal liberty except according to a procedure established by law.” “The right to live
includes the right to live with human dignity and all that goes along with it, viz., the bare
necessities of life such as adequate nutrition, clothing, and shelter over the head and facilities
for reading writing and expressing oneself in diverse forms, freely moving about and mixing
and mingling with fellow human beings and must include the right to basic necessities the
basic necessities of life and also the right to carry on functions and activities as constitute the
bare minimum expression of human self.” “It is the fundamental right of everyone in this
country… to live with human dignity free from exploitation. This right to live with human
dignity enshrined in Article 21 derives its life breath from the Directive 24 of 40 Principles of
State Policy and particularly clauses (e) and (f) of Article 39 and Articles 41 and 42 and at the
least, therefore, it must include protection of the health and strength of workers, men and
women, and of the tender age of children against abuse, opportunities and facilities for
children to develop in a healthy manner and conditions of freedom and dignity, educational
facilities, just and humane conditions of work and maternity relief. “These are the minimum
requirements which must exist in order to enable a person to live with human dignity and no
State neither the Central Government nor any State Government-has the right to take any
action which will deprive a person of the enjoyment of these basic essentials.”

. Kamlesh was drug-addicted therefore neighbors of Kamlesh requested a police officer to not
allow him to participate in any functions of the community. Police restricted him to come out
of his home during any festival or social gathering. Kamlesh aggravated by this behaviour
filed a writ petition in Supreme Court against the police for violation of his fundamental right
to life. Decide

(a) No person shall be deprived of his life or personal liberty according to a procedure
established by law.

(b) The Central Government or any State Government has no right to take any action which
will deprive a person Right to life.

(c) The right to live includes the right to live with human dignity freely moving about and
mixing and mingling with fellow human beings.

(d) Both B and C

. In many states of India manual scavengers have been pressed into extra duty without
adequate protection or training or remuneration causing serious bodily or mental harm to
manual scavengers. How manual scavenger’s fundamental right to life is violated?
(a) It is the fundamental right of everyone in this country to live with human dignity free
from exploitation and manual scavengers are exploited.

(b) No protection of the health and strength of workers are taken into consideration.

(c) The right to live includes the right to live with human dignity and forcing them in
scavenging violates their dignity of life.

(d) All of the above.

. How Article 21 protect the right to life?

(a) Entails being able to live a complete life of dignity and meaning.

(b) When the right to life or liberty of a person is taken away by the state, it should only be
according to the prescribed procedure of law

(c) Right to life does not include the right to die.

(d) Both A and B.

. Mr. X is in a vegetative state and therefore Doctors intentionally letting him die by
withholding an artificial life support ventilator after permission from the family of Mr. X.
Decide it is a violation of the Right to life of Mr. X.?

(a) No, It is not a violation of the rights of Mr X because passive euthanasia is allowed in
India.

(b) No, It is not a violation of the right to life because the right to life can be taken away
following the procedure established by law.

(c) Yes it is a violation of Article 21 because the right to life does not include the right to die.

(d) None of the above.

. MLA private builder is forcing workers to work on construction site for hours and giving
very fewer wages without proper Drinking water facility. Decide MLA private builders will
be liable for the violation of fundamental rights of workers.

(a) No, MLA private builders will not be liable.

(b) Yes, It is the fundamental right of everyone in this country to live with human dignity free
from exploitation and the company is violating workers' rights.

(c) Yes, protection of the health and strength of workers was not taken into consideration.
(d) The right to live includes the right to live with human dignity and forcing them to work
violates their dignity of life.

Passage (Q.-Q.): Torts are civil cases, where a person claims that they have been harmed by
someone else, in violation of a non-contractual duty. Tort cases aren’t just about the
wrongdoer performing a harmful act, they can also deal with a failure to act. One of the most
important claims under tort law is that of negligence: where a person fails to exercise the duty
of care required of them by law. But does Indian law allow us to sue government authorities?
Can municipal be held liable for negligence? English common law, which was applicable to
tortious claims in India, used to include the concept of “sovereign immunity”: that the
government, as representative of the Crown, could not be held liable for actions of its agents
because the Crown could do no wrong. Over the years, this position was thankfully diluted,
and in 18, the Bombay High Court delivered a judgment in the P&O Steam Navigation case,
where it held that the government could be held liable for tortious acts (in that case,
negligence) of public servants in situations where an ordinary employer would have been
made liable. However, the court differentiated between sovereign i.e. which was part of
government’s work as a sovereign and, non-sovereign functions; tortious liability of the state
could only arise in relation to non-sovereign functions. [Extracted with minor revisions from
‘Mumbai Bridge Collapse Shows Need for Law on Govt’s Tort Liability, by Vakasha
Sachdev, published in thequint https://www.thequint.com/voices/opinion/mumbai-foot-over-
bridge-collapse-tort-liability-of-state-law[1]needed#read-more]

. The railway officials are in charge of looking at the periodic maintenance and repair of
railway bridges. A bridge collapsed at a railway station killing hundreds of people. The
bridge was in need of repair since a long time however no repairs were made and the bridge
was left as it is. The negligence meted out to the repair work meant that the bridge had
become very weak which ultimately caused it to collapse on its own. Can the government be
held liable for the tort of negligence?

(a) No, because repairing a bridge and checking on its regular maintenance is not a part of the
sovereign function of the government

(b) No, because the government cannot be held liable for the negligence of railway officials

(c) Yes, because repairing a bridge and checking on its regular maintenance is a part of the
sovereign function of the government

(d) Yes, because railway officials are not a part of the government and maintenance of
bridges is non[1]sovereign function

. Uphaar was a famous cinema in Delhi. It was well known that the cinema was quite
cramped and did not have many exits as it took a lot of time for people to exit the cinema hall
after a show was over. One day, Uphaar caught fire due to an electrical problem. The less
number of exits and lack of any fire safety equipment caused several people to stay trapped in
the hall. During an investigation it was found that a majority of the people died not because
of being burnt in the fire but because of inhaling an excessive amount of smoke and carbon-
monoxide because they were trapped in the hall. In your opinion, can the government be held
liable in tort for its failure in ensuring a non-violation of fire safety norms?

(a) No, because ensuring a non-violation of fire safety norms is not a sovereign function of
the government

(b) No, because the people died due to negligence of the cinema hall and not because of the
negligence of the government

(c) Yes, because the government is bound to ensure that cinema halls follow all the relevant
fire safety protocols

(d) Yes, because the government was negligent in performing its sovereign function to ensure
periodic inspections to check if fire safety norms were being followed or not

. Singham was a public servant. He was getting a new house constructed for himself by a
local contractor. One day, a person was passing by the road on which the construction site
was located. A brick dropped out of nowhere and hit the person on his head. He was severely
injured and claimed that there were no proper signs anywhere to tell a passer-by that
construction work was ongoing and his hurt was due to the negligence of Singham, who
should have ensured the same. Can the government be held liable for negligence?

(a) Yes, because Singham was a public servant and construction of the house was a part of
the sovereign function of the government and makes it liable for the same

(b) Yes, because a failure to put up appropriate signs to alert a passer-by regarding an
ongoing private construction is a part of the sovereign functions of the government

(c) No, because the construction of a house by Singham for himself does not constitute a
sovereign function of the government and doesn’t make it liable

(d) None of the above

. X was a jeweller whose gold had been stolen. The police recovered the stolen gold and kept
it in police custody. A police officer was given the duty to ensure a safe custody of the gold
before it was returned to its rightful owner. The gold was again stolen from the police station.
It was found that the same police officer stole the gold and fled to Pakistan. X was angry and
filed a case of negligence against the State. Can he do so?

(a) No, because police officer was performing a sovereign function and the gold was stolen
during the performance of that sovereign function itself

(b) Yes, because recovery and safety of lost goods must be ensured by the police as it is their
sovereign function to do so
(c) Yes, because the police officer was negligent in taking care of the stolen gold

(d) No, because the police officer was not performing a sovereign function while stealing the
gold so a case of negligence can be filed

Passage (Q.-Q.): As per the text of Article 226 of the Indian Constitution, High Courts are
empowered to issue writs against “any person or authority” for the enforcement of
fundamental rights, or “any other purpose”. In view of established constitutional law cases,
the power of High Courts to issue writs under Article 226 is much broader than the Supreme
Court’s powers under Article 32. Indeed, the term “authority” must receive a liberal
construction, as held in cases including Dwarka Nath v. Income Tax Officer (14) and Andi
Mukta Sadguru v. V.R. Rudani (19). This is owing to the fact that Article 226 by its design
uses a wide language to enable High Courts to mould reliefs to meet the Indian landscape’s
peculiar cases, while also empowering them to issue writs other than prerogative writs the
Supreme Court can decree under Article 32. The ‘public function’ test, i.e., the proposition
that private entities discharging functions characterized as ‘public’ are covered under Article
226, is a product of such a liberal construction. As established by Khamroi, “every individual
retains their right to privacy over personal information, even though that information exists
only on a virtual space.” However, the question as to whether this right to privacy can be
enforced through writs against a private entity such as WhatsApp depends on establishing
that WhatsApp performs a ‘public function’. In fact, to this day, there has never been any
instance in India where a private entity was held to discharge a public function merely
because its activities are ‘fundamental’, important, or ‘public’. The entity in question must
discharge said function fully exclusively. Evidently, this is not the case with WhatsApp, since
there are several other similar social media apps which Indian users rely on, including those
not owned by its parent company Facebook.

. The Waterworks Department of Gotham City (WDGC) was sent several communications
and representations by the concerned citizens of Gotham City regarding the precarious water
situation of the city. The water pipeline in the city was in a desperate need of repairs and had
eventually broken down. This disrupted the water supply. In order to resolve their water
woes, citizens had to pay for their own water tankers as no other recourse had been provided
to them. However, the WDGC officials always cited some or the other reason and failed to
take any measure. It has been two months since this ordeal first began. Can the residents file
a writ petition against WDGC to resolve this issue?

(a) Yes, because the WDGC violated the citizens' fundamental right of access to water and
failed to take any action to resolve the situation

(b) Yes, because writs can be issued for the enforcement of fundamental rights or any other
purpose against WDGC

(c) Yes, because the term “authority” must receive a liberal construction, as held in the cases
of Dwarka Nath v. Income Tax Officer and Andi Mukta Sadguru v. V.R. Rudani

(d) No, because write remedies are available only against government authorities and WDGC
is a private entity

 
. In Gotham City, the entire business of setting network lines for broadband and mobile
phones is handled by Batman Company. Batman Company was set up in 19 to take care of
the growing network requirements and help Gotham City make progress at par with the rest
of the world. It is the only company which has the license and technical know-how as well as
resources to offer such a service. As a result, when Batman Company fails to perform its
functions appropriately or effectively address the concerns of the citizens, there is no proper
recourse available as Batman Company is the sole network provider. Batman Company has
now launched a mandatory new software which tracks the location of all Gotham City
residents. Can the citizens challenge this action?

(a) Yes, because Batman Company is performing a public function of providing network
access, even though it is a private authority

(b) Yes, because Batman Company is performing a private function of network provision,
even though it is a public authority

(c) Yes, because right to privacy is a fundamental right and network provision is a public
function being performed by Batman Company

(d) None of the above

. The economy of Gotham city was opened up in 19 in light of the ‘Liberalisation,


Privatisation and Globalisation’ policy. As a result, several relaxations were made and private
players were allowed to participate in functions that were earlier only performed by the
government. Now, Batman Company has several competitors in the network space such as
Wonder Woman Company, Spiderman Company etc. These companies also provide network
services. However, Batman Company is cheaper and still has a lot of subscribers. Can
Batman Company now be said to perform a public function?

(a) Yes, because it is still a network provider which has a lot of subscribers and is performing
a public function in the discharge of its duties

(b) No, because its activities are no longer ‘fundamental’, important, or ‘public’ in nature due
to the presence of several competitors

(c) No, because although it has several competitors now, it is still discharging the function of
network provision exclusively due to its larger subscriber base

(d) None of the above

. Hockey is the most prevalent and influential sport in Gotham City and is widely viewed and
commercialized. Board of Hockey Control (BHCI) is the only body which is responsible for
regulating the sport of hockey, its commercial viewership as well as selecting the members of
the national team for international tournaments. BHCI exercises unparalleled control over its
regulations, thus impacting millions of hockey lovers. Although BHCI receives several tax
incentives from the government of Gotham City, it is an autonomous body and not an
instrumentality of the state. Does BHCI perform a public function?
(a) Yes, because the activities of BHCI are fundamental’, important, and ‘public’ and non-
exclusive in nature

(b) Yes, because the activities of BHCI are fundamental’, important, and ‘public’ and
exclusive in nature

(c) No, because the activities of BHCI are not fundamental’, important, and ‘public’ or
exclusive in nature

(d) No, because BHCI is an autonomous body and not an instrumentality of the state which
discharges public functions

Passage (Q.-Q.): On April 8, a civil court in Uttar Pradesh asked the Archaeological Survey
of India to conduct a survey at the Gyanvapi mosque in Varanasi to determine whether it was
built over a temple. the Places of Worship Act, 19was Passed in the backdrop of a growing
Ramjanambhoomi movement that demanded that a Ram Temple be built in Ayodhya at the
site of the Babri Masjid, the law sought to protect the character of all places of worship. As
per the law, the character of a place of worship has to be maintained as it was on August 15,
17 and any legal proceeding that seeks to change the nature of the religious place would cease
with the commencement of the law in July 19. The law provides certain other exceptions. To
understand these, a reading of Section 4 provisions is necessary. In simple words, Section 4
(1) states that any legal proceeding that seeks to change the character of a place of worship
shall abate with the commencement of the Act and the religious character shall be maintained
as it was on August 15, 17. Section 4 (2) maintains that if a suit or appeal or any other
proceeding is instituted claiming that conversion of a place of worship took place after
August 15, 17, it should be disposed of by ensuring the character of the place of worship as it
existed on August 15, 17. Moreover, if a place of worship of any religion is covered under the
definition of ancient and historical monuments or an archaeological site or remains, this could
be exempted from the ambit of the Places of Worship Act as well as when consensus is
formed. As per the Ancient Monuments and Archaeological Sites and Remains Act, 18, an
“ancient monument” is “any structure or monument, or any tumulus or place of interment, or
any cave, rock-sculpture, which is of historical, archaeological or artistic interest and which
has been in existence for not less than one hundred years” https://scroll.in/article/90/analysis-
could-asi-survey-of-gyanvapi-mosque-lead-to-it-being-exempted[1]from-places-of-worship-
act

. An edifice situated in the capital of the country has been long been disputed between the
followers of the green religion and that of the blue religion as theirs, ending the dispute the
government in the year 10 turned the structure into a common place for worship after
negotiations, this was accepted by the leaders of both sects, a petition is filed in 2008
demanding that the structure be handed over to the blue religion followers, what would be the
outcome?

(a) The court would restore the building to into its original status that existed in the year 10.
(b) Since the decision of the place being a common worship area was accepted by both sides,
it will not be reversed.

(c) The court should order ASI to conduct a survey to find out more evidence.

(d) The petition would collapse as it was filed in the year 2008.

. The bluebook community is a tribal group living in the dense forests of Chhattisgarh , they
worship a eleven decade old holy tree located in the centre of their village, the villagers over
the years have constructed boundaries around the tree and created other temporary structures,
the government decides to cut the tree for road construction, the devastated villages claim
that the tree and the dwellings nearby are ancient monuments. What would the courts rule?

(a) The courts would rule in the favour of the villagers as the tree would be considered as an
ancient monument under the above mentioned act.

(b) The dwellings around the tree would be considered as ancient monuments hence the tree
also would be considered as ancient.

(c) The government shall proceed with the road construction as the tree would not be
considered as an ancient monument under the Ancient Monuments and Archaeological Sites
and Remains Act, 18 as the act does not cover trees.

(d) The tree is eleven decade old, hence would be covered under the act passed in 18.

. ASI discovers ruins of a 400 year old temple structure are found in a deserted part of Thar
desert, several activists ask for restoration of the structure into a temple. Would such
restoration be valid?

(a) Yes, it would be classified as an ancient structure and can potentially be restored as a
temple as it will be exempted from the places of worship act.

(b) Since it’s a ruin site, the government will not be able to convert it back into a temple.

(c) Its status was not certain as of august 15th 17, hence it cannot be converted back into a
temple under places of worship act.

(d) The government should conduct more research on the site to gather more evidence.

. A church situated in Kerala is made of precious and rare marble in Indo-gothic style and is
of great artistic value. It was commissioned in 15 and came into being in march 10, would
this be classified as an ancient monument?

(a) No, as monuments constructed in 10 are not ancient.


(b) Yes, it is a monument of artistic value so under the Ancient Monuments and
Archaeological Sites and Remains Act, 18 it would be categorised as an ancient monument.

(c) Yes, it is of great importance and has deep personal and historical values

(d) No, it does not fulfil the requirement of hundred years of existence.

. Followers of blue religion demolished a cathedral in 19 and established their place of


worship there, the matter was raised in court in 2009 demanding construction of cathedral
again, what would the courts rule?

(a) The courts would maintain status quo and quash the petition.

(b) The courts would order that cathedral should be constructed according to Places of
Worship Act, 19

(c) The courts would initiate a settlement between both parties.

(d) None of the above.

Passage (Q.-Q.): On 4 January 2021, WhatsApp announced its new privacy policy for India.
As a result of this privacy update, WhatsApp will get permission to share with Facebook the
metadata of users and their messages with business accounts. In 2019, the Ministry of
Electronics and Information Technology tabled the Draft Personal Data Protection (PDP) Bill
in Parliament. It is currently with a Joint Parliamentary Committee and has not passed into
law. the Clause 11(2)(c) of the PDP bill says that data principals must give specific consent.
Clauses 5 and 6 mandate that the data fiduciary can collect data only for the purpose to which
the data principal has consented and the data shall be stored within the national vicinity.
Clause 11(3)(c) requires the data fiduciary to take consent for processing sensitive personal
data separately for each different purpose. This provision would prevent WhatsApp from
taking consent for both purposes together, because the metadata and chats with a business
account that WhatsApp would have been able to share with Facebook could reveal sensitive
personal data like health information, sexual orientation, etc. 30 of 40 It has also been ruled
several times that when a person “has no choice or rather no meaningful choice” other than
signing on the dotted line and accepting the unfair clauses of a contract, then such a contract
must be considered unreasonable and unconscionable and this provision shall be kept in mind
by the government while drafting the bill Clause 40 of the PDP bill empowers the Data
Protection Authority to include certain data fiduciaries in a regulatory sandbox who would be
exempt from the obligation of taking the consent of the data principal in processing their data
for up to 36 months. Such a sandbox might be required to provide relaxations to certain
corporations, such as those that deal with Artificial Intelligence so that they can test their
technology in a Sandbox environment. Data fiduciaries would be approved by the
government and selection shall not be irrational. Considering that the bill has become the law,
answer the following questions https://www.theleaflet.in/whatsapp-privacy-controversy-and-
indias-data-protection-bill/

. WhatsApp’s privacy policy was recently amended to add the clause that if the users are not
willing to accept the terms and conditions of the app, they are free to leave the app.
Distressed users have questioned this change. Is this change in policy by WhatsApp legal?
(a) Yes, the users always have the choice to withdraw themselves from the platform than
asking for a change in policy.

(b) The contract would be construed as unreasonable and unconscionable, hence imposing it
on the users is a dereliction of the law.

(c) The change of policy does not include clauses which are illegal or violate the user rights
hence the changes are legal.

(d) WhatsApp is giving its users no real choice which is not permissible by legal standards.

. Tooter is a popular social media app. The app when downloaded asks the user to accept its
general rules and conditions. Recently the app also launched its own clothing brand and has
explicitly been using statistics of users acquired from Tooter to target prospective consumer
groups. They have been sued. They argue that they had obtained general consent of the user
in their terms and conditions, was their action legal?

(a) No, they should have asked for specific consent as the users had not consented for sharing
of their personal data with another entity.

(b) No, the information shared might be sensitive in nature hence general consent does not
fulfil the requirement of the law.

(c) Yes, The information and statistics shared with Tooter were not sensitive in nature hence
the action of the app was not against the law.

(d) Yes, the actions of Tooter were legal as both entities are owned by and general consent
was already acquired.

. Facebook is a social media app with millions of followers in India. In 2021 they announced
that they have created a mechanism to store all the back-up data from the app. This backup
data will not be utilised for any purpose and will be stored in Facebook headquarters in
California. The location of storage was challenged in court.

(a) Facebook’s action do not violate the law as they data will not be utilised for any purpose
and will just be stored.

(b) Facebook’s action are definitively violative as they did not obtain the consent of the data
principal.

(c) Facebook’s action are violative as data can only be stored in the within the country as per
Clauses 5 and 6 of PDP law.

(d) The data in question is back-up data, hence is can be stored outside the country.

 
. The government has decided to create a database for a new unique identification and
intelligence technology of the citizens, to execute this they have classified themselves as a
data fiduciary and granted access to themselves in one such sandbox of a tech social media
giant. This has been challenged in court, what would the courts rule?

(a) The governments actions are violative of tenets of data protection bill.

(b) This action of the government lacks rationality as they can misuse the data , hence the
violate the law.

(c) The government can process the data in the sandbox for 36 months and it would not be
illegal.

(d) This action of the government would certainly not be illegal as the purpose is testing a
new technology which is an exception according the PDP bill for three years.

. Right to privacy is a fundamental right, does clause 40 of PDP bill violate the right to
privacy?

(a) Yes, government cannot allow access to personal data without specific consent of the
users as it is violation of right to privacy.

(b) No, the access provided by the government is only for a limited period of time hence it is
not violative of right to privacy.

(c) No, the reason behind the government providing access is to test a new technology and
framework hence it is not a violation.

(d) Yes, storage of data and creation of a sandbox for it by the platform is in itself a violation.

32
Passage (Q.68-Q.): The Supreme Court observed that it is permissible to hold discrete/open
enquiry, at pre FIR Registration stage in corruption cases. In appeal, the bench Justices DY
Chandrachud and MR Shah considered the question whether such an enquiry at pre-FIR stage
would be legal and to what extent such an enquiry is permissible? Referring to Lalita Kumari
vs. Government of Uttar Pradesh the bench observed thus: an enquiry at pre[1]FIR stage is
held to be permissible and not only permissible but desirable, more particularly in cases
where the allegations are of misconduct of corrupt practice acquiring the assets/properties
disproportionate to his known sources of income. After the enquiry/enquiry at pre-registration
of FIR stage/preliminary enquiry, if, on the basis of the material collected during such
enquiry, it is found that the complaint is vexatious and/or there is no substance at all in the
complaint, the FIR shall not be lodged. However, if the material discloses prima facie a
commission of the offence alleged, the FIR will be lodged and the criminal proceedings will
be put in motion and the further investigation will be carried out in terms of the Code of
Criminal Procedure. Therefore, such a preliminary enquiry would be permissible only to
ascertain whether cognizable offence is disclosed or not and only thereafter FIR would be
registered. Therefore, such a preliminary enquiry would be in the interest of the alleged
accused also against whom the complaint is made. The court added that, “It is enough if the
information discloses the commission of a cognizable offence as the information only sets in
motion the investigative machinery, with a view to collect all necessary evidence, and
thereafter to take action in accordance with law. Therefore, as such, holding such an enquiry,
may be discrete/open enquiry, at preregistration of FIR stage in the case of allegation of
corrupt practice of accumulating assets disproportionate to his known sources of income,
cannot be said to be per se illegal”. Both Money laundering and Corruption are cognizable
offences and the accused can be arrested by police officers without any warrant.

68. Ramdhari Dinkar, education minister in state of Rajasthan, was indulged in taking an
ample amount of money for transferring teachers and lecturers in government schools. Those
who want to be transferred to their nearby places and schools, has to deposit the mentioned
amount accordingly. For primary school teacher its Rs.15,000, for third grade teacher, its
Rs.20,000; for second grade teacher its Rs.30,000 and for first grade teacher/lecturer its
Rs.50,000. Also if someone wants for deposition can submit Rs.25,000. This all was handled
by his assistant and other ministers at local level. Eventually, the accounting book
maintaining all such records came out in the public domain and the public demands for
enquiry into the case. No complaint as such was filed against the issue but a huge troll starts
over the social media. Decide-

(a) Preliminary inquiry cannot be taken out by the investigating officers as there is no prima
facie ground available for valid enquiry at pre registration of FIR.

(b) Preliminary inquiry can be taken out by the investigating officers as there is prima facie
ground available for valid enquiry at pre registration of FIR.

(c) Preliminary inquiry cannot be taken out by the investigating officers but based on the
complaint and social media trolls, a case for corruption and money laundering can be made.

(d) None of the above.

. Dharmendra Textiles and Co. Ltd was dealing in manufacturing and selling of textiles
product and is the biggest leading company in India. The government of India has taken out a
contract with the Company to deliver 0 tonnes of clothing material rich in Khadi and pure
cotton. The total amounting bill was made of Rs.150 Crore and the government paid Rs.2
Crore and gets the impure material too from the company to showcase it to be of pure quality
and sell it to the customers at government subsidized retailer shops at fixed price. The
opposition party took the matter in discussion in session of parliament and asked the
government about the extra expenditure of Rs.120 crore which was missing from the book of
accounts. The government stated the said expenditure to be incurred in maintaining the
situation of COVID-19 under control and is paid from PM relief fund. A complaint has been
made for such enquiry into the case. Decide-

(a) Preliminary enquiry can be taken out by the investigating agency as there is prima facie
evidence as to the corruption by government in book of accounts.

(b) Preliminary enquiry cannot be taken out by the investigating agency as there is no prima
facie evidence as to the corruption by government in book of accounts.
(c) Preliminary enquiry can be taken out by the investigating agency as valid complaint has
been filed against the government and it cannot be done before filing of FIR.

(d) None of the above.

. Lok Manya Janta Hostipal, a government run hospital in state of Madhya Pradesh, was one
of the best hospitals to cure heart related diseases and had number of experienced surgeons
for the same. One fine day, Mr. Mohan died due to silent heart attack and a certificate was
issued by the local private hospital. His wife took his husband again for check up in Lok
Manya hospital and she was informed that he was alive and she needs to arrange money for
his surgery. She was in shock but had done what she was asked for. She got them the required
medication and prescribed surgery was taken out. She deposited an amount of Rs.50,000 as
surgery fees and later asked for extra amount of Rs.40,000 as her husband is dead and if she
wants his body back. She filed a complaint against the corruption going on in public hospital
and had validly given the death certificate of her husband. Decide-

(a) Preliminary enquiry can be taken based on complaint of women as it is substantial and on
finding of prima facie evidence, subsequent action can be taken against the authorities.

(b) Pre registration of FIR, enquiry cannot be taken in the case as the complaint filed by
women is not substantial and strong prima facie case has to be built for investigation.

(c) Enquiry in the case can be taken only after filing of FIR and there has to be strong
evidence with the lady apart from the death certificate for valid enquiry.

(d) There is no prima facie evidence with the lady to hold out a legal enquiry either
preliminary or pre registering the FIR and hence no action can be there against the
authorities.

. Charul, Priya and Mohini were good friends and were preparing for UPSC exams 2020.
They all cleared the written exams with good pointers and rank and were called for next stage
of selection i.e Interviews. Their interviews were not so satisfactory and all three of them lose
the hopes for getting selected for the post of IAS officers. Soon the selection list was released
and Priya got selected out of the group. Mohini’s father was also a minister and he applied his
approach and asked the selection board to take his child also for the post. He urged them to
adjust some ranks and release another rank list with Mohini’s name. He also gave a sum of
Rs. 3,00,000 for the said purpose. Soon a rectified list was released with Mohini getting
selected for the post. Charul get to know about this adjustment and she filed a complaint
against the same claiming the selection of her friend to be cancelled and also the board is
corrupted in taking huge sums for such posts. Decide-

(a) Preliminary enquiry cannot be taken out in this case and no FIR can be filed for such
corruption charges as the complaint filed by Charul is vexatious and not valid.

(b) Preliminary enquiry can be taken out in this case and FIR can be filed for such corruption
charges as the complaint filed by Charul is true and valid too.
(c) Preliminary enquiry cannot be taken out in this case but FIR can be filed for such
corruption charges as the complaint filed by Charul is true and valid.

(d) Preliminary enquiry can be taken out in this case but no FIR can be filed for such
corruption charges as the complaint filed by Charul is vexatious and invalid.

. Choose the most appropriate. Assertion (A): Enquiry at pre registration of FIR and
Preliminary enquiry can be taken out in cases of money laundering or corruption under
Prevention of Corruption Act. Reason (R): Both Money laundering and Corruption are
cognizable offences and the accused can be arrested by police officers without any warrant.

(a) Both A and R is correct and R is the correct explanation of A.

(b) Both A and R is incorrect.

(c) Both A and R is correct but R is not the correct explanation of A.

(d) Only A is correct and R is incorrect.

Passage (Q.-Q.): Macaulay‘s Draft Penal Code 17 consisted of section 113 that corresponded
to section 124A IPC. The punishment proposed was life imprisonment. Sir John Romilly,
Chairman of Second Pre-Independence Law Commission commented upon the quantum of
the punishment proposed for sedition in India, it should not be more than five years.
However, this section was not included in the IPC when it was enacted in 10. This was
surprising for many. Mr. James Stephens when asked about this omission referred to the letter
written by Sir Barnes Peacock to Mr. Maine, where he had remarked that: “I have looked into
my notes and I think the omission of a section in lieu of section 113 of the original Penal
Code must have been through mistake. I feel however that it was an oversight on the part of
the committee not to substitute for section 113.” Mr. James Stephen thereafter set out to
rectify this omission. Consequently, sedition was included as an offence under section 124A
IPC through special Act XVII of 18. This section was in line with the Treason Felony Act 18
that penalised seditious expressions. One of the reasons cited by Mr. Stephen for introducing
this section was that in the absence of such provision, this offence would be penalised under
the more severe common law of England. Therefore, the adoption of this section was
projected as an obvious choice for protecting freedom of expression from the stricter
common law. According to Mr. Stephen, the adopted clause was much more compressed,
much more distinctly expressed, and freed from great amount of obscurity and vagueness
with which the law of England was hampered‘. The intent of the section was to punish an act
of exciting feelings of disaffection towards the government, but this disaffection was to be
distinguished from disapprobation. Thus, people were free to voice their feelings against the
government as long as they projected a will to obey its lawful authority.

. Ms. Vaijayanti, a feminist leader, started a organisation called “We the Women” and works
to promote and upgrade the position of women and transgender in the society and awareness
about their role in bringing about the change. In her public speech on International Women’s
Day, she mentioned about the significant role of women in a society and how it has been
restricted to the gender roles ascribed to them. She also talked about the emerging
transgender community and active roles to transform the orthodox mindset of the society. She
also raised pertinent questions to the government where no such representation is sought for
women in the Parliamentary Indian Democracy and called them as ill minded patriarchs. She
sharply pointed out how Indian Parliament discriminates against the women representation
and asked the people to demand for 50 % reservation for women in each public department
including the Indian Parliament and asked everyone to move the idea forward with large
violent protests and revolt against government. Decide-

(a) She cannot be charged with offence of sedition as her speech was only a part of dissent
against the present status quo of condition of women and their representations.

(b) She can be charged with the offence of sedition as her speech was with intention to incite
hatred and disaffection towards the government and to revolt against it with violent protests.

(c) She cannot be charged for offence of sedition as her speech was only a formal speech as a
leader and she has spitted out the truth, though were of nature to instigate people to revolt
against the government.

(d) She can be charged with the offence of sedition as her speech constitutes disapprobation
more than disaffection and was sufficient enough for inciting people against the government.

. In state of Jumritalaiya, government has officially passed an act to persecute the tribal
people involved with the Naxalites. This was done with regards to the recent incident where
many soldiers were martyred by these gangs. Under this act, such tribes have to leave the
state by end of year 2022. The act become a burning issue soon and was also criticized by
various renounced politicians and officials. Constitutionality of the act was also challenged in
courts. Violent protests took place in state of Jumritalaiya and various slogans of anti-
government were raised. A lot of abuse was spoken about the government in power and
public places were destroyed. This was all done to strike down the act to be invalid. Soon day
after the protests, Mr. Kanhaiya shared a document file, by mistake, over Flutter, a social
media platform, which unfolded the planned conspiracies of anti-nationalists behind such
protests. Document had contained words “This government need to be thrown out and power
has to be demolished by violent means”. Government filed against Kanhaiya for sedition.
Decide-

(a) Kanhaiya is not liable for offence of sedition as he has shared the document over Flutter
by mistake and the text in the document is also in form of dissent towards the arbitrary
actions of the government.

(b) Kanhaiya is liable for sedition as she was not part of the protests and was a formal
criticism to such government action against tribal people and had used violent means to
disturb public peace.

(c) Kanhaiya is not liable for offence of sedition as there has to be violence or actual threat to
overthrow the government from power and document was not real evidence as it was from an
unknown source.
(d) Kanhaiya is liable for sedition as the document shared by him over flutter is inciting and
cause disaffection towards the government with conspiracy to overthrow the government
from power.

. Government of India has passed Farmers Bill 2020 and it was criticised a lot with many
violent protests in the country. The bill was also challenged in courts for its
unconstitutionality and to strike it down. Mr. Athawle, Member of Parliament of opposition
party, criticized the government action over a television interview on famous news channel.
He stated that government acts arbitrarily and always proves to be a weak hand when it
comes to support agricultural sector in the country. Agriculture is one of the primary sectors
on which whole economy of India depends as most of the Indians rely on agricultural income.
With such covid-19 times, government is solely responsible for decreasing GDP rates. He
also emphasized on low budget allocations and fake policies claiming to support the farming
sector in India. He alleged finance minister of the country to be incompetent and asked
people of the country to make her resign from the cabinet if she cannot control the ongoing
economic crisis in the country. Decide-

(a) The interview of Mr. Athawle does not amount to offence of sedition as it’s his personal
views in nature of dissent towards actions of government though amounts to disaffection or
hatred of violent nature.

(b) The interview of Mr. Athawle does amount to offence of sedition as he is inciting people
to revolt against the finance minister and has made oblique remarks in that regard.

(c) The interview of Mr. Athawle does not amount to offence of sedition as he was merely
exercising his right to freedom of free speech and expression and his statements were mere
remarks criticizing government.

(d) The interview of Mr. Athawle does amount to offence of sedition as his statements were
more than criticisms and were real threat to overthrow the government in power.

. Aman was a student in IIT Guwahati. He was an intelligent student and was among the
toppers. In his final year of his studies, he gets in relationship with Mahira. One day Mahira
asked her to remain with her and marry her to which he denied. Mahira approached the
sexual harassment cell of her college and complained of Aman raping her. She also presented
the frivolous chat messages. Soon the news gets spread in college and Aman felt ashamed of
his relationship with such girl. He committed suicide in his hostel room. All students marched
rally in the college demanding justice for Aman and asked the authorities to investigate into
the matter. Soon the education minister of Assam was approached and government
interference was sought. The students were so enraged that they started peaceful protests and
requested all to overthrow the government if no justice is arrived at. Decide-

(a) The conduct of students cannot be said to be offence of sedition as it was all done under
the umbrella of freedom of speech and expression and demand for justice.

(b) The conduct of students can be said to be offence of sedition as it was inciting general
public to overthrow the government and disturb the public peace and order.
(c) The conduct of students cannot be said to be offence as it was mere form of dissent out of
enraged students as Aman has committed suicide due to frivolous case of Mahira.

(d) The conduct of students can be said to be offence of sedition as it cause real threat to the
government and was more of kind of disapprobation and disaffection towards government.

. While inserting section 124A in IPC, Mr. Stephen made certain remarks and also stated that
it was a most obvious choice to include this section under the code as to protect the freedom
of expression. What stand does he intends to make here?

I. Mr. Stephen wants to convey that Indian Sedition laws are much more severe in nature and
punishment than the laws of England.

II. He contends that Indian Sedition law is much more ambiguous and strictly followed when
comes to its implementation.

III. Mr. Stephen found Indian Sedition law to me much more subjective as compared to
sedition laws of other jurisdictions. IV. Mr. Stephen stated that Indian Sedition Law is clear
and lucid with less subjectivity in hands of authorities.

(a) Both I and IV

(b) Both II and III

(c) Both I and III

(d) Both II and IV

. Choose the most appropriate options.

Assertion (A): Sedition laws in India are implemented with reasons to punish those who do
not obey the lawful authority and deter them to voice against the government.

Reasoning (R): Article 19 provides for fundamental right to freedom of speech and
expression but is not absolute and are restricted.

(a) Both A and R is correct and R is correct explanation of A.

(b) Both A and R is incorrect.

(c) Only R is correct and A is incorrect.

(d) Both A and R is correct but R is incorrect explanation of A.

Passage (Q.-Q.): Read the passage carefully and answer the following questions[1]Under
Section 31 of the Indian Contract Act, a contingent contract is a contract to do or not to do
something, if some event collateral to such contract does or does not happen. Essentials of
contingent contract[1]I. Depending on happening or non-happening of a certain event The
contract is contingent on the happening or the non-happening of a certain event. The contract
will be valid only if it is about performing or not performing an obligation. II. Performance of
the contract must be conditional The situation for which the contract has been entered into
must be a future event, and it should be uncertain. If the performance of the contract is
dependent on an event, which is although in the future, but certain top happens, then it’ll not
be considered as a contingent contract. III. The event must be collateral to the contract. It is
important that the event on whose happening or non-happening the performance of the
contract is dependent should not be a part of the consideration of the contract. It should exist
independently. IV. The event should not be a mere will of the promisor The event cannot be
wish of the promisor and should be totally a futuristic and uncertain event. Contingent
contracts to do or not to do anything if an uncertain future event happens, cannot be enforced
by laws unless and until that even has happened. If the event becomes impossible, such
contracts become void. A contingent contract might be based on the happening of an
uncertain future event. If the happening of the event becomes impossible, then the contingent
contract is void. Similarly, wagering contracts are void under the law. Contingent contract is
different from wagering contract, in the sense that, in wagering,  a. Uncertainty of event b.
‘Mutual’ chances of gain or loss (Excerpt from Law of Contract, Ashok Jain, Kinds of
Contract, pp 8-11)

. Which of the following situation/s do/does fulfil the conditions of a continent contract? I.
Avi enters into contract with Vikas and promises to deliver a watch to him. Vikas promises to
pay Rs. 2000 upon delivery. II. Avi agrees to pay Vikas Rs. 10, 000 if Sun rises from the
west. III. Avi agrees to pay Vikas a sum of money if a certain ship does not return by the next
day. The ship is sunk on the very same day.

(a) Only 1

(b) 2 and 3

(c) Only 3

(d) 1 and 3

. Avi promises to pay Ragini, Rs. ,000 if she marries Vikas. She agreed for the same. While
they were about to marry, Vikas died. Which of the following requirement of continent
contract is not fulfilled in this situation?

(a) The event was mere will of the promisor

(b) The event was not collateral to the contract

(c) It is restraint of marriage and invalid under Indian Contract Act

(d) The condition could not be fulfilled

. Which of the following statement/s differentiate the contingent contract with wagering
contract? I. Contingent contract is mainly depending upon uncertainty of an event; whereas,
wagering contract is just a matter of chance II. Contingent contract are enforceable under
Indian Contract Act; whereas, wagering contract is unenforceable

(a) Both 1 and 2

(b) Only 1

(c) Only 2

(d) Neither 1 nor 2

. A bought a lottery from a vendor; in case, the number mentioned in the lottery come in
lucky draw, he will win Rs. 1, 00, 000. Fortunately, his number came in the lucky draw. But
the vendor refused to pay the winning amount. Analyse the case as per the above-passage.

(a) This is contingent contract depending upon the lucky draw number

(b) This is contingent contract as the condition is independent to the consideration

(c) This is wagering contract as it is just matter of chance; neither party has control over it.

(d) It is illegal under Indian Penal Code

. Avi announces a reward of Rs. 0 to be paid to anyone who finds his lost pet. Vikas finds the
pet. Vikas’ act of finding the pet is acceptance to the offer as well as of the performance of
the contract. Hence, it is

(a) A contingent contract

(b) An absolute contract

(c) Not contract under the law

(d) Wagering contract

Passage (Q.-Q.): Read the passage carefully and answer the following questions[1]The
United Kingdom Supreme Court held on Friday that Uber drivers are "workers", not
independent for the purposes of employment legislation. Hence, Uber can be vicariously
liable for the wrongful act of drivers. The court stated three elements of “worker contract” as
(1) a contract whereby an individual undertakes to perform work or services for the other
party; (2) an undertaking to do the work or perform the services personally; and (3) a
requirement that the other party to the contract is not a client or customer of any profession or
business undertaking carried on by the individual, then proceeded to analyze UBER working
system. As a result, five factors listed below were concluded and those factors concluded in
the classification of drivers as “workers”: I. UBER sets the fare. The drivers may accept
lower fee but cannot anything higher. UBER can change the fare or percentage of driver fee
due to local conditions or user request/complaint. II. The Service Agreement is imposed to
drivers. Such concept would not have occurred if the powers were balanced. III. While a
driver can freely choose where and when to work by logging onto the app, there is a
constriction on whether to accept a request for transportation. One of control mechanisms is
keeping track of acceptance and cancellation number of the driver. When cancellation
reaches a certain number, several penalties are applied such as being logged out of the app.
IV. UBER has total control over how the services are delivered and there is a rating system to
supervise drivers. Unlike booking sites rating system does not offer users any preference, is
only used to measure drivers’ performance. V. Finally, UBER restricts any kind of
communication between driver and user. Thus, the driver is not free to offer services
independently. The court also highlighted that Uber monitors a driver’s service through a star
rating basis, and holds the authority to terminate the relationship if after repeated warnings
the ratings don’t improve. Volenti Non Fit Injuria acts as a defence to Vicarious Liabi
(Excerpt from The Leaflet, February 19, 2021, Uber Drivers are workers, not independent
contractors, rules the United Kingdom’s Supreme Court unanimously.

. Plaintiff Jame Dew brought the suit against Uber Tecnologies Inc. (Uber) for sexual assault
that plaintiff alleged they suffered at the hands of Uber driver.

(a) Uber cannot be vicariously liable as sexual assault falls outside the scope of an employee
duties

(b) Uber can be vicariously liable as they have the right to control the manner and means of
employment

(c) Uber can be held liable because there is breach on the part of Uber

(d) Uber cannot be held liable because it is a criminal wrong on the part of the employees

. Mr ‘A’, a mechanic, operated a company named ‘Cool Runnings Refrigeration Pvt. Ltd’.
The mechanic was contracted by ‘B’ for the servicing of a refrigerator. The refrigerator door
collapsed and struck ‘S’ causing her to have injuries to her head, neck and hand.

(a) A was an employee of B hence B could be vicariously liable for the injury caused to Ms S

(b) A was not an employee of B hence B could not be vicariously liable for the injury caused
to Ms Sweeney.

(c) A conducted his own business and there is no Respondant Superior principle will apply

(d) A is not liable as it is an unforeseen incident

. A was qualified shot-firers at B, the employer’s quarries. The employees had been short of
wire to test a circuit for firing explosives from a sheltered distance. He disobeyed their
employer’s instructions and mandatory regulations and tested the electric circuit with the
insufficient wiring, causing himself and a passerby injury.
(a) B is vicariously liable as an employer because A was acting in his quarries and under
manner prescribed by B.

(b) B is able to rely on volenti non fit injuria as a defence to vicarious because A has acted in
deliberate neglect of the employer’s instructions.

(c) B cannot be held for vicarious liable as he was a skilled worker and was working in his
own way and manner without B’s direct supervision.

(d) B is liable it was his quarries which escaped the risk.

. A is the owner of many trucks and employs many drivers to drive them, B is one of them.
One night, B gets into an accident because of his rash driving, and he was in voluntarily
drunken state at that time. Who will be held liable for the accident?

(a) A will be vicariously liable because B was working under the authority of A

(b) A will not be vicariously liable as B was driving recklessly and he was under the
influence of intoxication

(c) A will be held vicariously liable because it was his truck, which B was driving

(d) A will not be vicariously held liable because B is liable under criminal law for rash
driving

. The term ‘contract of service’ differs from ‘contract for service’ by way of the degree of
agency and authority of the hirer on the worker. While perusing the conditions given in
above-passage, which of the following condition/s does not denote ‘contract for service’? I.
The worker is not free to offer services independently to any third person without any
restrictions of the employer. II. The worker can freely choose where, when and how to work
III. The worker has the ultimate control over how the services are to be delivered

(a) 1,2, and 3

(b) 2, and 3

(c) 1, and 3

(d) 1, and 2

Passage (Q.-Q.): The Supreme Court on Friday said people are free to choose their own
religion, even as it lashed out at a “very, very harmful kind” of “public interest” petition
claiming there is mass religious conversion happening “by hook or by crook” across the
country. Instead, a Bench led by Justice Rohinton F.Nariman said citizen of India have a right
under the Constitution to profess, practise and propagate religion. Justice Nariman asked
petitioner-advocate Ashwini Kumar Upadhyay. Justice Nariman reminded Mr. Upadhyay of
the fundamental right under Article 25 of the Constitution to freely profess, practise and
propagate religion, subject to public order, morality and health. “Why do you think there is
the word ‘propagate’? “Justice Nariman asked the petitioner. Religious conversion is being
done through a “carrot[1]and-stick” approach, Mr. Uapdhyay had claimed in his petition.
Justice Nariman said every person is the final judge of their own choice of religion or who
their life partner should be. Courts cannot sit in judgment of a person’s choice of religion or
life partner. Religious faith is a part of the fundamental right to privacy. Justice Nariman
reminded Mr. Upadhyay of the Constitution Bench judgment which upheld inviolability of
the right to privacy, equating it with the rights to life, of dignity and liberty. Mr. Upadhyay’s
petition was dismissed as withdrawn. His pleas to approach the Law Commission or the High
Court with the plea were not expressly allowed by the Bench. The petition alleged that the
court should direct the Centre and the States to control black magic, superstition and religious
conversion being done through threats, intimidation or bribes. Further, the petition said the
Centre and States were obligated under Article 46 to protect the SC/ST community from
social injustice and other forms of exploitation.

. Navratri which is a pious festival in India is celebrated at night by doing the Garba. Garba
mandli has organised a Garba party in Jamnagar and has played loud music mid-night
disturbing the tranquillity of the area. Hamid an artisan doesn’t share good relations with
Garba mandli, Therefore, he filed a complaint against their conduct of celebrating festival
midnight along with other members of the community who are facing health issues due to
loud music. Decide can Garba Mandli be restricted from doing the Garba considering the
above situation?

(a) Citizen of India have a right under the Constitution to profess, practise and propagate
religion therefore garba mandli have every right to celebrate Navratri and do the Garba.

(b) They are not disturbing public order and due to personal reasons, Hamid filed a complaint
against Garba mandli. Hence legal restriction is invalid.

(c) Constitution granted the right to practice religion without violating public order, morality,
and health. Hence, reasonable restriction is valid.

(d) Celebrating festivals at midnight is against public order, morality and health. Hence
reasonable restriction is valid.

. Mr.Groff an American is propagating his religious beliefs in India and practicing them
without disturbing public order, morality, and health but forcefully he was restricted by
Police from practicing his religion. Is the Police violating Mr Groff’s rights Under Article
25?

(a) No, Police are not violating his rights under Article 25.

(b) Yes, Constitution granted the right to practice religion without violating public order,
morality, and health. Hence Police violating his right to practice religion.

(c) Yes, a person is the final judge of their own choice of religion therefore police cannot
restrict them.
(d) No, a citizen of India has a right under the Constitution to profess, practise and propagate
religion.

. Kaju and Badam are fighting on the land dispute and out of vengeance Badam forces Kaju
to convert his religion into Christianity. Failing to convert religion will result in the loss of
land of Kaju. Elaichi wife of Kaju filed a writ petition against Badam on behalf of her
husband's forceful conversion. Decide Legal remedy for Kaju.

(a) Badam cannot force Kaju to convert his religion because the person has the right to
choose their religion freely.

(b) Badam can force Kaju to convert his religion because every person is the final judge of
their own choice of religion.

(c) Badam cannot force Kaju to convert his religion because a citizen of India has a right
under the Constitution to profess, practise and propagate religion.

(d) It is a personal matter between the parties therefore court is not bound to interfere.

. What is the reason for the petitioner to file a petition. Answer in the context of the passage.

(a) He doesn’t want mass religious conversion in India.

(b) Petitioner was of the notion that mass religion conversion is done in unethical ways.

(c) Petitioner was of the notion that religious conversion being done through threats,
intimidation, or bribes.

(d) Both B and C

. Hoshangabad district of Madhya Pradesh is populated with ST and SC community people.


The XYZ Religious organisation influencing the SC and ST community people to convert
into ABC religion by giving financial Aid. Mr Vakil aggravated from the same and filed a
complaint against XYZ Religious organisation for exploiting SC and ST community. Decide
XYZ will be liable for Exploitation and forceful conversion of SC and ST community.

(a) Yes, XYZ will be liable because a citizen of India has a right under the Constitution to
profess, practise and propagate religion freely

(b) Yes, XYZ is liable because Article 46 protects the SC/ST community from social
injustice and other forms of exploitation.

(c) Both A and B

(d) XYZ will not be liable.

 
Passage (Q.-Q.): Force majeure has become a fairly common ground to avoid the
performance of contractual obligations in a post-COVD-19 world. At a time when lawyers
and parties seek to examine their force majeure clauses and reassess their contractual
arrangements, here is a report studying how courts have treated force majeure claims. The
term ‘force majeure’ has been defined in Black’s Law Dictionary, as ‘an event or effect that
can be neither anticipated nor controlled. It is a contractual provision allocating the risk of
loss if performance becomes impossible or impracticable, especially as a result of an event
that the parties could not have anticipated or controlled.’ After examining a total of 0 cases, a
qualitative set of 1 cases were extracted to form the core data set for the report. The report
categorizes force majeure claims that have been allowed into broad descriptive heads, to give
a clearer understanding into the nature of force majeure claims that have been allowed. These
heads are: Orders by the government, Permission and regulatory clearance, Change in law,
Court order, Act of God, Foreign Events and a residuary category of Disturbances in
Contract. The report finds that of the 1 cases, a force majeure claim was allowed in 68 cases –
i.e. 38.4% of the cases we surveyed. Accordingly, a force majeure claim was rejected in the
remaining cases – i.e., 61.5% of the cases we surveyed. The lockdown measures imposed in
India to contain the spread of COVID-19 have had a naturally disruptive effect on the
performance of contracts. With government and regulatory bodies already invoking force
majeure clauses and extending the timelines for performance of contracts, there is an
increasing trend of force majeure claims being made.

. Aparna books a ticket to New Delhi on an Indigo flight 1. A usual late riser, Aparna had to
stay up all night to board her 5 AM flight for it was very expensive. As Aparna reaches the
airport and checks in her luggage, she is informed that the flight is cancelled due to a massive
locust attack on the Delhi Airport Runway. Indigo neither delayed the flight nor provided any
alternative. Aparna drags Indigo to court and Indigo claims that they did not breach any
contract as it was a situation of force majeure. Decide

(a) Indigo cannot claim force majeure. Even if this was an unexpected event, it does not
fundamentally alter the nature of obligations. They could have delayed the flight.

(b) Indigo can claim force majeure. The unexpected event changes the nature of obligations
of the contract which was to land in a safe environment.

(c) Indigo cannot claim force majeure. It hurts passenger interest.

(d) Indigo can claim force majeure for Aparna had to wake up early morning.

. Bombay Dyeing entered into an agreement with Cotton and Co. for purchasing raw cotton.
The contract was through the exchange of letters among them. The seller used to write them
which were confirmed by the buyer. At one instance, Bombay Dyeing wrote 4 letters to
Cotton and Co. during 15th May 2000 to 10th June 2000. The confirmation for the same was
provided on 20th June 2000. Bombay Dyeing after providing a notice exercised the right to
resale. Bombay Dyeing had also made a claim of the amount of debit note which was Rs.
57,000. The note was returned by Cotton and Co. along with the letter of 20th June stating
that the contract was void and illegal. Cotton and Co. denied the performance of the
contractual obligations. The claim made by Cotton and Co. was the usage of the words usual
force majeure clause in the contract. Decide

(a) The claim is justified due of the existence of a force majeure clause in the contract.

(b) The claim is not justified as the terms involved are vague and uncertain.

(c) The claim is justified as the party did not have control on the unfolding of events.

(d) The claim is not justified on account of non-fulfillment of obligations within a reasonable
timeline.

. A contract was signed between Vladimir Industries and Gupta Industries and the governing
law of the contract was Indian law. As a result of heightened import duties, the company’s
profit margin was cut down by around 2.5 percent which inconvenienced their quarterly
charts. The CEO of Vladimir wishes to not perform the obligations of the contract and claim
force majeure.

(a) Vladimir can claim force majeure. This is a natural calamity and an unforeseeable event.

(b) Vladimir cannot claim force majeure. This event does not significantly impact their
profitability.

(c) Vladimir can claim force majeure. This event is covered within the dicta of the Supreme
Court.

(d) Vladimir cannot claim force majeure. This event is merely onerous to them and cannot
vitiate performance of the contract.

. Krishna Nigam Power Generators is a huge company which generates electricity using coal
and has tie up with government of Madhya Pradesh to whom it sells the electricity. This
company used to mainly import coal from Australia as it would cost them cheaper than here
in India. Due to change in government in Australia, the export duty on coal was increased
significantly, thus making it very costly for Krishna Nigam Power Generators to purchase
coal from there. Krishna Nigam Power Generators went to the Supreme Court against the
Madhya Pradesh government claiming relief under force majeure. Decide whether it will
succeed or not?

(a) No, as increase in export duty doesn’t qualify as an event which would make contract
impossible.

(b) Yes, it will succeed because their inability to perform the contract arises out of a
government decision and is beyond their control.

(c) No, it will not succeed because it is a not a case of force majeure but a case of frustration
which has made the performance impossible.
(d) Yes, it will succeed because the communication of their inability to perform their part of
the contract due to an unforeseeable event was made as soon as it occurred and not ex post
facto.

. Mohandas is a merchant based in Chennai deals in cashew, which he procures from Indian
as well as foreign markets mainly from Sri Lanka. Chandanlal, a confectioner in Delhi enters
into a contract with Mohandas for supply of 0 kilograms of best quality cashew at the rate of
Rs.500 per kilogram every week. The relations between India and Sri Lanka turned sour and
both countries snapped business ties soon after Mohandas agreed to supply cashew to
Chandanlal. Mohandas did not deliver cashew to Chandanlal because he mostly procured
cashew from Sri Lanka, which were comparatively cheaper. He cannot supply to Chandanlal
at the mentioned rates after buying from Indian markets. When asked by Chandanlal,
Mohandas said that due to no trade and war like conditions between Indian and Sri Lanka he
is not in a position to supply him cashew. Chandanlal files a suit for breach of contract and
claims compensation.

(a) Chandanlal will succeed.

(b) Mohandas will succeed.

(c) The contract will be declared void.

(d) The contract shall be re-drafted in existing situations.

. TSeries contracted with famous singer Mrs. Krishna Murti for Taj Exclusive Club members'
event. After the contract, Mrs. Murti was diagnosed with Covid 19 and was advised not to
perform for the next six months. Despite her willingness, Mrs. Murti could not sing for Taj
Exclusive Club members' event. TSeries filed a suit against Mrs. Krishna Murti for breach of
contract. Decide whether the diagnosis of bronchitis rendered the contract void?

(a) No, as Mrs. Murti was willing to perform for Taj Exclusive Club member's event.

(b) No, as TSeries was not responsible for medical condition of Mrs. Krishna Murti.

(c) Yes, as the medical condition of Mrs. Murti made it a case where the situation is beyond
one’s control.

(d) Yes, as doctor's advice ought to be given preference to contractual obligations.

Passage (Q.-Q.): The adoption of the Information Technology (Intermediary Guidelines and
Digital Media Ethics Code) Rules, 2021 (“Intermediary Guidelines”), the government has
adopted a liability centred approach and raised several questions. Content on the internet
typically concerns three parties: (1) ‘content originators’ who post or share content; (2) online
intermediaries who provide a platform for users to share content on; and (3) ‘content
recipients’ who view the content posted by content originators. Conceptually, it is important
to distinguish between websites that post their own content, and online intermediaries that
merely provide a platform .When (inevitably) the material posted by a content originator
violates the law, it raises the question of whether the online intermediary should be held
secondarily liable as well. Section (1) of the IT Act states that an online intermediary shall
not be liable for this facilitative role but in case of active promotion of content shall be. This
ensures that intermediaries do not have to scrutinise each user post for potential liability that
may be foisted on them, ensuring the free flow of user content on platforms such as Facebook
and Twitter. However, given the wide reach of the internet, governments also typically want
intermediaries to act as gatekeepers against some of the worst forms of content. As a result, in
India, the immunity offered to intermediaries by Section (1) requires intermediaries to satisfy
certain obligations. Crucially, under Section (2)(c), online intermediary must “observe due
diligence”. This is where the Intermediary Guidelines become relevant, as Rules 3 and 4 of
the Guidelines prescribe the ‘due diligence’ obligations of online intermediaries. To do this
service providers often take the help on automated regulator and checkers who review
content posted on the platform.

. Sudesh is an Instagram influencer who often posts objectionable content on his profile,
recently he posted a video where he was urging the citizens to protest for the secession of an
Indian state which is an illegal act. This post was reported several times by users but wasn’t
taken down by the app. Instagram claims they have put up a caveat before the video to inform
the viewers about the content. Was the necessary procedure followed?

(a) The process of due diligence was not followed, since the video invited liability it should
have been taken done which was not done.

(b) Caveat or warning put up by Instagram shows that it fulfilled its duty as an intermediary.

(c) No, the steps taken by Instagram were not adequate enough.

(d) The app is a free space and it cannot violate the freedom of expression of the citizens.

. Aditya is a student of medical sciences and deeply passionate about the legalisation of
marijuana for medicinal purposes in India, he often writes articles and makes short videos on
this cause to post on twitter. Dack Jorsey is an internationally acclaimed and very popular
actor and this cause of Aditya resonates with him. He often retweets his tweets and helps him
reach to a wider audience, the right wing government is troubled by this as this is against the
ideology of the ruling party and imposes a fine of twitter for not following due diligence,
were they correct?

(a) Yes, the government was correct as twitter should follow the guidelines laid down by
them

(b) Yes, twitter shall be liable as they failed to follow the prescribed law

(c) No, the content posted on the medium was not objectionable in nature, hence not violative
of the guidelines

(d) None of the above


 

. In the wake of a health emergency, twitter became a hub of fake news and rumours which
were also promoted by several celebrities and famous personalities. To ensure that this stops
twitter hires several experts who scrutinise these accounts, their data and personal chats ,
pictures etc, when the news gets out one such personality sues twitter for violation of privacy,
was twitter at fault?

(a) Yes, right to privacy if fundamental and scrutinising information like pictures and chats is
violative of right to privacy

(b) Yes, it failed to maintain due diligence.

(c) No, the app was following the procedure mentioned in the intermediary guidelines, this is
part of maintaining due diligence.

(d) No, the app was just fulfilling its legal obligation by being a responsible intermediary.

. Rafael and Roger are very popular celebrities and international stars, it is also very well
known that they are arch rivals. Recently, Roger posted objectionable and vulgar content
about Rafael on his Facebook page, this garnered a lot of attention and millions of likes.
Rafael claims that the content is defamatory and objectionable in nature and along with Roger
also sues Facebook, will Facebook be liable

(a) Yes, Facebook should have removed the content on the request of Rafael

(b) No, the it’s not Facebook’s duty to check whether the nature of the content

(c) Yes, Facebook would be held liable as the process of due diligence was not followed, the
content should have been taken down as it was objectionable in nature

(d) No, only roger would be liable as he posted the defamatory content and Facebook was
just the intermediary

. Toland Drump is the chief minister of an Indian state, after losing the 2019 Vidhan Sabha
elections, he was agitated and mobilised his supporters through twitter, his supporters
eventually attacked and pillaged the assembly hall. Will twitter be liable?

(a) Yes, twitter’s actions were not responsible. They should have been more careful.

(b) No, freedom of expression is fundamental in nature. It cannot be abridged.

(c) No, the pillage was not foreseeable by twitter , hence the two events are unrelated.

(d) Yes, the app should have fulfilled its duty by restraining the tweet, due diligence was not
followed and the consequences was a directly related to the tweets.

 
. Do these intermediary guidelines violate right to privacy?

(a) Privacy is a fundamental right, by keeping surveillance on the content posted by creators
is violative of right to privacy.

(b) No, to maintain due diligence, it is important to keep a check.

(c) No, Right to privacy would be violated when personal content of individuals is under
check which includes chats , pictures , contact details etc

(d) None of the above

33
Passage (Q.68-Q.): The word “nuisance” is derived from the French word “nuire”, which
means “to do hurt, or to annoy”. One in possession of a property is entitled as per law to
undisturbed enjoyment of it. Nuisance is an unlawful interference with a person’s use or
enjoyment of land, or of some right over, or in connection with it. Nuisance is an injury to the
right of a person in possession of a property to undisturbed enjoyment of it and result from an
improper use by another person in his property. Trespass is direct physical interference with
the plaintiff’s possession of land through some material or tangible object. Trespass is
actionable per se, while nuisance is actionable only on proof of actual damage. Section 268 of
the IPC, defines public nuisance as “an act or illegal omission which causes any common
injury, danger or annoyance, to the people in general who dwell, or occupy property, in the
vicinity, or which must necessarily cause injury, obstruction, danger or annoyance to persons
who may have occasion to use any public right.” Generally speaking, Public Nuisance is not
a tort and thus does not give rise to civil action. On the other hand Private nuisance is an
unlawful interference and/or annoyance which cause damages to an occupier or owner of
land in respect of his enjoyment of the land. The law of nuisance has grown and expanded
through interpretation and through a plethora of judgments. Indian Courts in the matters of
nuisance have borrowed quite intensively from the English principles as well as from the
decisions of the common law system along with creating their own precedents. This has
resulted in a sound system of law being developed that ensures fairness and well being of all
i.e. the parties and the society at large. [

68. Mrs. Kunti Sharma and Mrs. Lalita Mishra were neighbours living in a well maintained
locality. They both were jealous of each other from the very first day they met each other.
One day an argument started between them with respect to use of the common land outside
their private flats. This argument has turned into grave enmity and all people around the
locality gathered to witness the fight. Another day Mrs. Mishra stated the conversation to her
husband and enraged with the same, Mr. Mishra went to Mr. Sharma’s house and threw some
small stones and broke the mirrors, lamp posts etc. of Mrs. Kunti’s house. Decide- (a) The act
of Mr. Mishra amounts to tort of trespass as there was direct physical interference with Mr.
Sharma’s house and actual damage is caused to Mr. Sharma. (b) This act of Mr. Mishra does
not amount to trespass as there was no direct physical interference and no actual damage was
caused to Mr. Sharma. (c) This act of Mr. Mishra amounts to private nuisance as there was
annoyance and disturbance caused to Mr. Sharma with respect to his right of enjoyment to the
property. (d) This act does not amount to private nuisance as though there was damage
caused but no actual interference was there with Mr. Sharma’s right of enjoyment to the
property. . In Ushaben vs. Bhagyalaxmi Chitra Mandir the plaintiffs sued the defendants for a
permanent injunction to restrain them from exhibiting the film “Jai Santoshi Maa”. It was
contended that exhibition of the film was a nuisance because the plaintiff’s religious feelings
were hurt as Goddesses Saraswati, Laxmi and Parvati were defined as jealous and were
ridiculed. Also the film must be struck down on the constitutional grounds as there was
violation of Article 25, right to religion and its promotion. Being the defence lawyer, you
would have argued that- (a) Exhibition of the film is not a nuisance as there was no strict or
direct proof of actual damage caused to the plaintiffs and religious hurt is not nuisance. (b)
Exhibition of the film is not a nuisance as there was no unlawful interference with his right of
freedom of religion and hence no violation of Article 25. (c) Exhibition of the film is not a
nuisance as tort of nuisance is only applicable to cases where person in possession of
property is caused unlawful disturbance as to its enjoyment and right over such property. (d)
Exhibition of the film is trespass as hurting religious sentiments is actionable per se and is not
consequential but direct act of defendant.

. Prashant, a well known builder and constructor, was having a big house on highway. One of
the windows of his house has an opening outwards on the main road. This window was
damaged due to much exposure to sunlight and storms and hanging partly on highway side.
Prashant was not aware of the same. On one stormy day, people were rushing to avoid the
storm and suddenly the window nuts get loose and the window fell down upon Mr. Kamal
Hasan. He was injured badly and taken to hospital. He filed case against Prashant for
compensation and damages. In your view- (a) Claims for compensation and damages were
valid as Mr. Kamal was injured due to negligence of Prashant. He should have taken
reasonable care and due diligence as his house is on main highway. (b) Claims for
compensation and damages were valid as the act of falling down of damaged window
amounts to public nuisance and Prashant was liable for compensation to Mr. Hasan. (c)
Claims for compensation and damages were valid as the act of falling down of window
amounts to private nuisance and Prashant was liable for compensation and damages to Mr.
Kamal. (d) Claims for compensation and damages were not valid as knowledge plays
important role in nuisance and Prashant was not having proper and complete knowledge of
damaged window. . Astha, a housewife, was a specialist in putting tasty pickles and soon she
started a business for the same. She got an order for 50 kg red chilli pickle and she started
working for the same. She went to market and bought red chillies and other essential
ingredients. She put the red chillies on her terrace to get dried up. She also put some red chilli
powder with the same. Due to high winds, red chilli powder spreads to the nearby houses too
and caused injury to the neighbours. All complained Astha for the same. Decide- (a) The act
of Astha amounts to public nuisance as it was interference with the enjoyment right of the
neighbours and caused actual damage to them. (b) The act of Astha does not amount to public
nuisance as there was no interference with the enjoyment right of the neighbours. Astha was
only using her terrace for her person use and no actual damage was caused to them. (c) The
act of Astha amounts to private nuisance as it was interference with the enjoyment right of
the nearby neighbours only and not the general public. (d) The act of Astha does not amount
to private nuisance as it was interference with the enjoyment right of the public at large and
had caused actual and real damage to them. . Which among the following is/are suitable
example/s of public nuisance? I. Siya resided in a house next to a Roman Catholic Chapel of
which Shyam was the priest and the chapel bell was rung at all hours of the day and night. It
causes disturbance to Siya and her family. II. Priyam established an electric flour mill
adjacent to Priya’s house in a bazaar locality and the running of the mill produced such noise
and vibrations that Priya and her family, did not get peace and freedom from noise to follow
their normal avocations during the day. III. Mehta and Sons Pvt. company carrying on
calcining operations in any manner whereby noxious vapors would be discharged on the
pursuer’s land, so as to do damage to his plantations or estate. IV. Priyank, a doctor,
complained that sufficient quantity of dust created by the defendant’s brick powdering mill
enters the consultation room and causes discomfort and inconvenience to him and his
patients. (a) Only I (b) Only IV (c) Both II and III (d) I, II and IV . Choose the most
appropriate option. Assertion (A): Where the thing is not material and tangible or where
though it is material and tangible,but it is not direct act of the defendant but merely
consequential on his act, the injury is not trespass but merely a nuisance actionable on proof
of actual damage. Reason (R): Nuisance is an injury to some right accessory to possession but
not possession itself. (a) Both A and R is correct and R is correct explanation of A. (b) Both
A and R is incorrect. (c) Only A is correct and R is incorrect. (d) Both A and R is correct but
R is incorrect explanation of A.  

Passage (Q.-Q.): In Hirday Nath Ray vs. Ramachandra Sarma, the Calcutta High Court
defined the term “jurisdiction” – as “the authority by which a Court has to decide matters that
are litigated before it or to take cognizance of matters presented in a formal way its decision”.
The Court shall be competent to entertain the proceeding. They are categorized mainly into
four types of jurisdictions. Pecuniary Jurisdiction is based upon the valuation of a subject
matter of the suit. With regards to territorial jurisdiction of the court, several other factors
have to be taken into consideration. If the suit is with regard to recovery, rent, partition, sale,
redemption, determination of right of immovable property, it shall be instituted in the court
within the local limits of whose jurisdiction the property is situated. Where a wrong has been
caused to a person, or any damage has been caused to a movable property, then the suit may
be instituted either in the place, where wrong or damage has been caused, or in the place,
where the defendant resides, where there is a dispute in business, agreement or any other kind
of civil dispute, except matrimonial matter, then the suit may be instituted either, in a place,
where the defendant resides or carries on business, or in a place, where the cause of action
has arisen, i.e. where the dispute or wrong took place. On the other hand, Subject matter
jurisdiction relates to the subject matter of the suit without its reference to the pecuniary
valuation or a territorial jurisdiction of the subject matter. In the exercise of Original
Jurisdiction, a court of first instance decides suits, petitions, application etc. Appellate
Jurisdiction refers to the court’s authority to review or re-hear the cases that have been
already decided in the lower courts by way of any appeal, revision etc. [Source- Diva Rai,
“How to Identify the Jurisdiction Civil Courts in India”, , accessed on 08th April 2021]

. Mr. Khurshid living in city of Indore had visited Nagpur for his business purpose. There he
entered into a contract with one of his brothers, Abu Khurshid, for transfer of property
situated in Indore. Due to unavoidable circumstances Abu was not in capacity to perform the
contract. The contract was breached and subsequently a suit was filed in civil court of
Nagpur. Abu Khurshid claiming his share in a property dispute contended that he had not
breached the contract deliberately and no such suit can be filed for the same in Nagpur civil
court due to lack of its jurisdictional power to entertain the suit. Decide-

(a) Contention raised by Abu is invalid as cause of action i.e. contract for transfer of property
was entered between him and Mr. Khurshid in city of Nagpur only. Hence it has the
jurisdiction to try the suit.
(b) Contention raised by Abu is valid as Indore Civil Court has the subject matter jurisdiction
over the property matter in issue and not the Nagpur Civil Court. Hence it lacks the
jurisdiction.

(c) Contention raised by Abu is not valid as Nagpur Civil Court has both pecuniary and
subject matter jurisdiction over the property matter in issue. Hence it does not lack
jurisdiction over the suit.

(d) Contention raised by Abu is not invalid as Nagpur Civil Court do not have pecuniary
jurisdiction over the property matter in issue. Hence it lacks jurisdiction over the suit.

. Maniratnam was residing in city of Bhopal, carrying business of automobiles in metro city
of Mysore. He visited Jodhpur for his business purpose and there he met Mr. Praful
Shantiram, who lives in Dehradun and was one of the directors of best selling Rice
companies in Hyderabad, India. Maniratnam entered into a contract with Mr. Praful for sale
of Basmati rice at fixed price of Rs. per kg and time was the essence of the contract. Mr.
Praful Shantiram had not completed the order on time due to certain circumstances and as a
consequence Mr. Maniratnam had filed a suit against him for breach of contract and claiming
the damages for the same. Decide-

(a) Maniratnam cannot file a suit for breach of contract claiming compensation in civil courts
of city of Hyderabad and Dehradun as they lack jurisdiction.

(b) Maniratnam can file a suit for breach of contract claiming compensation in civil courts of
city of Hyderabad, Jodhpur and Dehradun only as they have valid jurisdiction.

(c) Maniratnam cannot file a suit for breach of contract claiming compensation in civil courts
of city of Bhopal, Jodhpur and Dehradun as they lack jurisdiction over the suit.

(d) Maniratnam can file a suit for breach of contract claiming compensation in civil courts of
city of Mysore or Dehradun only as they have valid jurisdiction over the suit.  

. Harman and Barman were brothers under Hindu Joint Family and their father had adopted
another son Manan. In a property dispute for the ancestral property, they have decided to
share it in a ratio of 2:2:1. The same arrangement was agreed upon by two brothers and was
not stated to Manan. In a civil suit against the claim of equal share over the property, Manan
contended that his share in the property is less and both Harman and Barman have committed
fraud by not disclosing such ratio of division of the property. The two brothers stated about
the arrangement scheme passed and approved by the head of village Panchayat as the
property in issue is situated in the outskirts of city of Cuttack and Bhubaneswar. Manan
contended that only Cuttack civil court has the jurisdiction over the matter. Decide-

(a) Contention is incorrect as only Bhubaneswar civil court has the jurisdiction over the
property matter in issue, though such property is situated in outskirts of Cuttack.

(b) Contention is correct as only Cuttack civil court has the jurisdiction over the property
matter in issue because such property is situated in outskirts of Cuttack.
(c) Contention is not incorrect as Bhubaneswar civil court has only territorial jurisdiction
over the property matter in issue unlike Cuttack civil court which has both territorial and
pecuniary jurisdiction over the suit.

(d) Contention is not correct as both Bhubaneswar and Cuttack civil court have the
jurisdiction over the property matter in issue and suit can be entertained by either of the
courts.

. Hemant was a foodie and loves to cook and eat his favorite Red Sauce Pasta. He went to the
nearby grocery shop and purchased cheese, red tomato ketchup and pasta. On returning back
at home, he found the tomato ketchup bottle to be expired one and went back to the store to
exchange it. When he again went to the edible foods row, he found all tomato ketchup bottles
were of expired dates as due to long covid-19 outbreak, owners have not checked and
changed the old stocks. Thus, he filed a complaint against the negligence of the shop owners
to sell the expired edible food items in their store. For such claim of damages he can
approach which court?

(a) Hemant can approach any nearby civil court having the territorial jurisdiction over the
matter and can file a suit claiming damages.

(b) Hemant can approach any nearby civil court having the both territorial and pecuniary
jurisdiction over the matter and can file a suit claiming damages.

(c) Hemant can approach any nearby civil court having the subject matter jurisdiction over
the matter and can file a suit claiming damages.

(d) Hemant can approach any nearby civil court having the all pecuniary, territorial and
subject matter jurisdiction over the matter and can file a suit claiming damages.

. Which of the following is not a suitable example of original jurisdictions?

I. Supreme Court of India hearing a case under special leave petition for violation of Article
14, 19 and 21.

II. High Court of Karnataka reviewing a case which was decided by the lower court which
having no jurisdiction over the matter decided it. III. Supreme Court of India advising the
President of India as to appointment of judges of High Court of Rajasthan. IV. High Court of
Punjab and Haryana issuing a writ of mandamus in a case where a government or agency of
government has certain duty but has failed to do so.

(a) Only III

(b) Both II and IV

(c) I, II and III

(d) I, III and IV  


 

. Choose the appropriate one.

Assertion (A): Subject Matter jurisdiction is one which refers to subject matter of the issue
and is inclusive of both pecuniary and territorial jurisdiction.

Reasoning (R): Employees of Bharat Gas Ltd. filing a case in labour court against the
employers for not providing the sufficient wages under the Industrial Disputes Act is an
example of subject-matter jurisdiction.

(a) Both A and R is correct and R is correct explanation of A.

(b) Both A and R is incorrect.

(c) Only A is correct and R is incorrect.

(d) Both A and R is correct but R is incorrect explanation of A.

Passage (Q.-Q.): Disdain of Court is an issue which respects that equity ought to be managed
reasonably and it likewise rebuffs any individual who intends to hurt the poise or authority of
the legal councils. This intensity of responsibility plainly portrays a similar responsibility the
Supreme Court has these days under the Indian Constitution against its hatred. The term
‘Contempt of Court’ is a non[1]exclusive term illustrative of lead corresponding to specific
procedures in an official courtroom which will in general undermine that framework or to
restrain residents from benefiting themselves of it for the settlement of their questions. This
term can be effectively comprehended as when we are ill bred or defiance towards the official
courtroom which implies that we wilfully neglect to comply with the court request or lack of
regard the lawful specialists. In civil contempt, defence can be lack of knowledge of the
request, the non-compliance or the rupture done ought not to be or the request that has
resisted ought to be dubious or uncertain. A well known Advocate and Activist Prashant
Bhushan, was held guilty for contempt of court on August 14 for two tweets against present
Chief Justice S A Bobde and past four CJIs which the Supreme Court discovered depended
on twisted realities and added up to a vulgar and noxious assault on the Apex court and
destabilized the establishment of the legal executive. He was fined with 1Re. for such
contempt and in default of payment of a fine by September 15, Bhushan needs to go through
imprisonment of three months and will face a practice ban of three years, a bench headed by
Justice Arun Mishra said in the judgment. SC said the tweets brought the organization of
equity into offensiveness and authority of the establishment of Supreme Court all in all and
the workplace of the Chief Justice of India specifically, according to the general population
on the loose. [Source- “A Study on Contempt of Court”, , The Daily Guardian, as accessed on
14th April 2021]

. Mrs. Dobarwal, young activist and feminist lawyer, created an online platform over a social
media handle and shared invites to the practicing and experienced advocates of various High
Courts and Supreme Court to critically examine and comment on the rulings of courts. It is
with an object to prevent court from relying upon such bad precedents in law which leads to
way of injustice. Mr. Purshotam, practicing lawyer of Karnataka High Court, wrote an article
commenting on the recent Bombay HC judgment on embarking the limits of sexual
harassment under POCSO and IPC. He criticised the way HC conclude its reasoning with
making the definition of sexual assault under POCSO limited to skin-to-skin contact. He also
mentioned about the SC precedents where POCSO Act finds its valid grounds and
appreciation. Can Bombay HC punish Mrs. Dobarwal and Mr. Purshotam for contempt of
court?

(a) Yes. Both Mrs. Dobarwal and Mr. Purshotam are liable for contempt of court as article of
Mr. Purshotam was of obnoxious nature and it goes beyond the exercise of Article 19 i.e
freedom of speech.

(b) No. Mr. Purshotam and Mrs. Dobarwal are not liable for contempt of court as the article
published on social media was only in form of opinion and within the ambit of Article 19.

(c) Yes. Only Mrs. Dobarwal is liable for contempt of court as she has created the platform
with such objective which only works to bring about hatred against the authorities in courts.

(d) No. Only Mr. Purshotam is liable for contempt of court as he had intention to defame the
authorities in power in Bombay HC and brings hatred for them while pointing their
incompetency to rule out such decisions.  

. Avinash was an adopted son of Mr. Sharma. Mr. Sharma was a cancer patient and had no
biological child of his own. Soon his wife died and he married to another women. They
together have a son and a daughter. Mr. Sharma died one day due to being COVID positive.
The biological children of Mr. Sharma were not in good terms with Avinash and never
treated him as their brother. In a property dispute case, they refused to give any share of
property to Avinash as he was an adopted child and their father never adopted him legally.
The court ruled out that adoptive son is to be treated as biological son and as a consequence
he is entitled to property of his parents. The court awarded him right to the parental property
of his adoptive parents. When Avinash approached the family to take his share, they refused
to divide the property and asked him to live with them. Decide-

(a) Avinash can file a suit against the family for contempt of court as they have disregarded
the decision of court and refused to give him his share from the property.

(b) Avinash cannot file a suit against the family for contempt of court as they have asked him
to live with them and in no way disregarded the decision of court.

(c) Avinash can file a suit against the family for contempt of court as they have no intention
to follow the decision of court and it amounts to contempt.

(d) Avinash cannot file a suit against the family for contempt of court as they have agreed to
give his share but for the time being asked him to live with them.

. Prasoon, a construction builder, was engaged in big government project of constructing


World’s biggest hospital in state of Lucknow. Owing to the situation of COVID, the
government thought of building such infrastructure for further protection and improving the
medicinal and health situation in the country. Soon, the government finds a way to divert the
funds of public and mismanaged the accounts for the same. Prasoon was given an amount of
Rs.25,00,000 for such project but was shown only a limited investment in this project. This
corruption was reported by the activist, Mr. Jahangir and suit was filed against the same. The
court on the prima facie reports and evidences of books of accounts passed an injunction
order to the builders and asked the government to file their written arguments for the same.
But Prasoon continued the construction work of hospital. Decide-

(a) Prasoon is liable for contempt of court as he has disregarded the injunction order of the
court and has continued the construction work for the Hospital.

(b) Prasoon is not liable for contempt of court as he has not disregarded the injunction order
of the court and has continued the construction work for the Hospitals to complete the project
on time.

(c) Prasoon is not liable for contempt of court as he has disregarded only the injunction order
and not the final decree or decision of the court. So there is no contempt.

(d) Prasoon is liable for contempt of court as he has disregarded the injunction order of the
court which is equivalent to the final decree or decision of court and hence it amounts to
contempt.

. Advocate Sunil Chetri, young activist of lawyers association, wrote a text on his Flitter, a
social media handle. In his text he stated that “When historians in the future look back at the
last six years to see how democracy has been destroyed in the country even without a formal
Emergency, they will particularly mark the role of the Apex Court and its Chief Justices’ in
this destruction”. In another text on same day, he pointed that “The master of roaster rides a
Rs -lakh motorbike belonging to a highly influential political party and without wearing a
mask or helmet, at a time when he keeps the Apex Court on lockdown mode denying citizens
their basic fundamental right to access justice!”. A suit was filed against Mr. Sunil for
contempt of court. Decide-

(a) Only the first text will amount to case of Criminal Contempt and fine of hefty amount
should be imposed by the court with further imprisonment of 3 months and ban on default.

(b) It is not the case of Civil Contempt as both texts of Mr. Chetri on Flitter were obnoxious
and imposed on baseless allegations on authorities and court of equity.

(c) Both the texts were not the case of Criminal Contempt as Mr. Sunil was exercising his
right of freedom of speech under Article 19(1)(a) and the texts were within its limits.

(d) Only the second text is the case of Civil Contempt but defence of lack of knowledge and
dubious request of court can be taken into consideration to reduce the liability.  

 
. From the following situations, find out which situation(s) would not amount to Contempt of
Court. I. Sheela Barse, a feminist lawyer, tweeting about the injustice court has supported in
recent rape case of Uttar Pradesh and the practiced corrupted environment the Supreme Court
is working in. II. Mrs. Swara, member of opposite party, in her speech criticised the
inclination of the Apex Court and other HCs in deciding matters in favour of the respective
government. III. Ajay Singhania, an activist, criticising the Apex Court’s decision on recent
farmers’ bill as bad judgment in law and against the principles of natural justice. IV.
Shubham selling his ancestral property despite of the interim stay order of the family court
for the pending case of the property dispute.

(a) Both I and II

(b) Both III and IV

(c) Only III

(d) Only II

. Choose the most appropriate option.

Assertion (A): Article 19 of the Indian Constitution enshrines the fundamental right of free
speech and expression but such right is not absolute and comes with limitation under Article
19(2). One of such limitation is of Contempt of Court.

Reasoning (R): Article 129 of the Indian Constitution empowers the Apex Court as court of
record with power to punish for contempt of all courts including contempt of itself.

(a) A is correct and R is incorrect.

(b) Both A and R is correct and R is not incorrect explanation of A.

(c) Both A and R is correct but R is not correct explanation of A.

(d) A is incorrect but R is correct.

Passage (Q.-Q.): The government has notified Copyright (Amendment) Rules, 2021 and
included a new provision that eliminates the requirement of publication in the Official
Gazette. To encourage accountability and transparency, new provisions have been introduced
to deal with the undistributed royalty amounts and use of electronic and traceable payment
methods while collection and distribution of royalties. In India, the copyright regime is
governed by the Copyright Act, 17 and the Copyright Rules, 2013. The Copyright Rules,
2013 were last amended in the year 2016. “The amendments have been introduced with the
objective of bringing the existing rules in parity with other relevant legislations,” the
commerce and industry ministry said in a statement. As per the amendments, a new rule has
been introduced, whereby the copyright societies will be required to draw up and make public
an Annual Transparency Report for each financial year and is aimed to reinforce transparency
in working of copyright societies. The amendments have harmonised the Copyright Rules
with the provisions of Finance Act, 2017 whereby the Copyright Board has been merged with
Appellate Board. The time limit for the Centre to respond to an application made before it for
registration as a copyright society is extended to 1 days, so that the application can be more
comprehensively examined. “The amendments which were proposed in the draft rules of
2019 in the context of internet broadcasting are missing and probably rightly so, since the
principal statute that is the Act itself may need changes before the Copyright Rules are
amended. The interpretational conundrum with regard to internet broadcasting thus
continues,” Nadkarni said. As per the statement, the move aims to ensure smooth and
flawless compliance in the light of the technological advancement in the digital era by
adopting electronic means as primary mode of communication and working in the Copyright
Office.

. National Copyright Society, Mumbai has more than new copyright registrations this year
and therefore they want to make a transparent report of a newly registered copyright. What
will be the right course of action according to Copyright (Amendment) Rules, 2021?

(a) They have to make a systematic annual report for the smooth functioning of an
organisation.

(b) They have to publish an annual transparency report on their website.

(c) They can allow concerned people on their request to see the Annual transparency report.

(d) They can send Annual reports to a limited number of educated people through the post.  

. Why there is a need for bringing new provisions through Copyright (Amendment) Rules,
2021. Answer in the context of the passage.

(a) There was a lack of proper distribution and transparency in royalty amounts.

(b) To bring transparency in the functioning of copyright offices.

(c) To enhance digital transaction in royalties.

(d) All of the above.

. JVG copyright society made an application before central on 1st January 2021 for
registration as a copyright society. The authorities of the central government are confused
regarding giving a response to the JVG copyright society. Suggest them a solution under
Copyright (Amendment) Rules, 2021.
(a) They have to give a response before April 2021.

(b) They have to comprehensively examine the application and give a response as soon as
possible.

(c) They have to give a response publicly to maintain transparency.

(d) They have to comprehensively examine the application and give a response before 30
June 2021.

. What are the possible positive effects of Copyright (Amendment) Rules, 2021?

(a) Help in following proper standards and ethical practices in the digital era.

(b) More people will apply for copyright.

(c) New provisions will help in getting royalties easily.

(d) All of the Above.

. Which is correct in the context of the passage.

(a) Copyright Act was last amended in 2016.

(b) Copyright Board and appellate Board will work together.

(c) Copyright Rules 2013 eliminate the requirement of publication in the Official Gazette.

(d) None of the above.

Passage (Q.-Q.): Among various affirmative actions taken by a government for the
underprivileged, reservation is an extreme measure – in contrast to other steps such as
providing free education, housing, etc. – because it implies segregation of some posts for
certain class of persons, which otherwise might have been secured by other persons. The
problem of non-representation of the Scheduled Caste (SC) and Scheduled Tribe (ST)
communities in public employment at the time of independence was of this nature. And the
reason for their deprivation from state services was neither non-availability of candidates nor
lack of educational qualifications nor poverty. Rather, it was the age-old tendency of savarnas
to oppress a class whom they treated as untouchables or indigenous people who remained
outside mainstream society. Reservation is a manifestation of the fundamental right to
equality of opportunity in matters of public employment under Article 16(1) read with Article
14. It is the duty of the state under Article 46, read with Article 37, to promote the economic
and educational interests of the SCs and STs, and to protect them from social injustice and all
forms of exploitation. Despite all the safeguards, if the state shies away from its constitutional
obligations and makes a “policy” abolishing the reservations, will the judiciary remain a mute
spectator on such arbitrary actions? We are told that the Supreme Court plays the role of
sentinel on the qui vive (watchful guardian during a crisis). Summing up, it is emphasised
that the Supreme Court must change its approach with respect to reservation and see things
from the perspectives of the backward classes. SOURCE: The Wire 1st April 2021
https://thewire.in/law/supreme-court-caste-reservation-approach-change-social-justice

. Why there is a perception that reservation gives benefits to some persons at the cost of
others. Answer in the context of the passage.

(a) Due to segregation of some posts for a certain class of persons.

(b) Providing education facilities to a certain class of persons only.

(c) Government is doing inequality by giving reservations.

(d) Only the backward class citizen is given financial aid.  

. “The reason for their deprivation from state services was neither non-availability of
candidates nor lack of educational qualifications or poverty. Rather, it was the age-old
tendency of savarnas to oppress a class whom they treated as untouchables or indigenous
people who remained outside mainstream society.” What do the above historical insights
clarify?

(a) Due to the economic condition of a certain class of people reservation is provided.

(b) The above historical insights clarify that reservation was not provided on the ground of
economic backwardness, because economic advancement is possible through other policies
also.

(c) The above historical insights clarify that reservation is against the right to equality and
should not be practiced.

(d) The above historical insights clarify that the economic condition of a person should
decide their eligibility for reservation.

. Why author is not supporting the views that were expressed by the judges of a constitution
bench of the Supreme Court.?

(a) Author belongs from the backward class therefore he doesn’t support the views of judges.

(b) Economic conditions of SC and ST community are not good.

(c) Because of caste-based discrimination which is still prevalent in India.

(d) Government is doing nothing for backward classes.

. Government of India is not providing educational support and reservation in government


jobs for SC and ST community which is facing inequality and other social trauma for
centuries and the situation is still the same. Decide
(a) Government should not give reservations in government jobs because it will be an
injustice for other serious aspirants.

(b) Government should give financial support to the SC and ST community instead of
reservation.

(c) Both (a) and (b)

(d) Government should introduce a measure to provide jobs and education to ST and SC
community to safeguard fundamental rights.

. What are the views of the author on reservation in the passage?

(a) Caste-based discrimination is still prevalent in India; therefore, providing reservation will
help in the social upliftment of a particular community.

(b) Purpose of the reservation is not to provide economic support but social support which
can secure ST and SC community from exploitation.

(c) Both (a) and (b)

(d) Reservation should never end in India.

Passage (Q.-Q.): The Principle of Natural Justice is a principle under a legal system which
protects people against the arbitrary exercise of power to ensure fair play. The term ‘Natural
Justice’ assures people fairness, reasonableness, a good conscience, equity and equality.
Aiming to prevent a miscarriage of justice and arbitrariness, natural justice is a scientific term
for the ‘Nemo judex in causa sua’ which means rule against bias or that no man should be a
judge in his own case and “audi alteram partem” which means “hear the other side” or “no
man should be condemned unheard” or the right to fair hearing that has largely been extended
to “duty to act fairly”. Grounded in the Constitution of India, the doctrine of Natural Justice
protects the fundamental rights of people and to feature the concept of fairness by the
administrative authorities. In the case of Mohinder Singh Gill v. Chief Election
Commissioner, it was held that the concept of fairness should be in every action whether it is
judicial, quasi-judicial, administrative or quasi[1]administrative work.

. Rajinder was a judge at the Saket District Court. A matter came before him which
concerned a property dispute Mohinder and Manmeet. Once the matter was listed, Rajinder
found out that Mohinder was actually his distant cousin. Rajinder decides to hear the matter
and the petition is listed. Is this a violation of the rules of natural justice?

(a) Yes, because it goes against the principle of audi aulterem partem as Manmeet is not
being heard

(b) Yes, because it goes against the principle of nemo judex in causa sua as Rajinder is acting
as a judge in a family matter and may be biased
(c) No, because Mohinder was actually a distant cousin of Rajinder so there is no possibility
of any bias since Rajinder wasn’t even aware that he had such a cousin

(d) None of the above

. Sohan and Mohan were fighting a Court battle against each other. One day Sohan’s lawyer
filed an injunction i.e. a plea to stop Mohan from taking a certain action. Mohan did not
challenge the injunction order when he was notified that such a plea was being filed in Court.
The judge in this matter also sent a notice asking if Mohan wanted to challenge the same but
Mohan still did not respond. In the next hearing, the judge granted the injunction to Sohan
even though Mohan was not present. Is this a violation of the principles mentioned in the
passage?

(a) Yes, because it goes against the principle of audi aulterem partem as Mohan was not
heard and the injunction was granted without him being present

(b) Yes, because it violates the principle of Nemo judex in causa sua as the judge is biased
against Mohan in granting the injunction

(c) No, because the judge has acted fairly and natural justice principle has not been violated
in this case

(d) None of the above

. Tamil Nadu and Karnataka approached the Cauvery Water Tribunal for settling a question
of distribution of river waters between the two states. The Tribunal invited both the states one
by one and heard them separately before taking up the matter. When both the states were
separately called, they only presented their side of the story. Has the Tribunal treated both the
states fairly?

(a) Yes, because even if both the states were called independently, they were given the same
equal and reasonable opportunity to present their case

(b) Yes, because calling the states individually helped the Court to prevent bias as the other
state could not interfere while the first state was laying down its premise

(c) No, because they were both called independently and not together thereby violating the
principle of audi aulterem partem or ‘hear the other side’

(d) No, because both states were called independently and not together but it does not matter
because the principle of natural justice does not apply to Tribunals

 
. A and B took a flight together from Calcutta to Bombay. They were complete strangers who
just happened to be seated together because their tickets were booked accordingly. During the
flight they made some casual conversation. Once the flight landed they never spoke again or
kept in touch at all. Years later, A appears before B in a court case. Should B recuse himself
from hearing the matter?

(a) Yes, because it violates the rule of bias or the rule of Nemo judex in causa sua as A and B
know each other and are not complete strangers

(b) Yes, because it violates the rule of bias or the rule of Nemo judex in causa sua as A and B
are friends

(c) No, because it does not violate the rule of bias or the rule of Nemo judex in causa sua as A
and B are not friends and are not even remotely related to each other

(d) No, because it violates the principle of audi aulterem partem as the party opposite to A
will not be treated equally since A and B are friends  

. X was accused by a co-worker in a case of sexual harassment at the office. An Internal


Complaints Committee (ICC) was appointed to investigate the matter. The ICC heard the
victim’s side of the story but did not give Pramod a chance to be heard and directly
suspended him without even giving him a chance to defend himself. The ICC believed that
they had enough evidence and did not need to hear Pramod anymore. Is this a violation of
natural justice?

(a) Yes, because it goes against the right to be heard and Pramod should have been heard at
least once even if the evidence was against him

(b) Yes, because it violates nemo judex in causa sua as the ICC is acting as a judge in its own
internal matter and may be biased so an independent judge should be appointed

(c) No, because there is no unreasonableness or unfairness that is happening as the ICC is
functioning in a fair matter by giving the victim the main priority

(d) No, because accountability to the accused is not a part of the principle of natural justice so
the ICC may choose to not hear Pramod at all

Passage (Q.-Q.): Bihar’s new law seeks to set up an armed police force in the state, named
the Bihar Special Armed Police (BSAP). It will be responsible to maintain public order,
combat extremism, ensure security of specified establishments, etc. The most controversial
provision of the Act is section 7, which provides that any Special Armed Police Officer
entrusted with securing an establishment may, without a magistrate’s order or warrant, arrest
any person who causes, threatens to cause, or attempts to cause harm to an employee of the
establishment or the officer himself. Further, any person can be arrested either based on
“reasonable suspicion” or if they commit or attempt to commit a cognizable offence. Section
8 provides for any Special Armed Police Officer to detain a person even on the basis of
“reason to believe” that an offence has been or is being committed. He would have the right
to search the offender and his or her belongings and arrest them. Article 312 establishes the
Indian Police Service. Therefore, the state governments are empowered to maintain the police
force for the sake of “public order” and that only, hence the role of the police cannot be
strengthened in administration and management of the state. The Act places an undue burden
on ordinary individuals, who could end up being subjected to extreme abuse of power. The
burden of proving innocence is also laid up upon the accused which is unconstitutional. This
is why Article 21 of the Constitution protects the right to life and personal liberty of every
individual and reminds the state and people of the requirement to follow procedures
established by law. No court shall take cognizance of any offence under this Act where the
accused person is a Special Armed Police Officer except on a report in writing of the facts
constituting such offence and with the previous sanction of an officer authorized by the
Government in this behalf. https://www.theleaflet.in/is-bihar-a-police-state-in-the-making/

. Adit is a professor in Bihar university, he has been very critical of the Bihar government on
issues related to the tribal natives and local indigenous groups and his criticism has often
gained traction and troubled those in power. He often organises protests and seminars to
highlight his cause. In 2021 the BSAP has arrested him on the grounds of reasonable
suspicion of having links with the tribal Maoists and insurgents. Was this action of the BASP
correct?

(a) No, being supportive of a cause and voicing protest against the government on public
forums cannot indicate reasonable suspicion.

(b) Yes, his support and sympathy for these groups qualifies as reasonable suspicion.

(c) No, he has been critical of the state government and his arrest is symbolic to stifling of
dissent.

(d) Yes, the law states that the police can arrest anyone without warrant based on the charges
of reasonable suspicion.  

. Ashok is a BSAP officer and he has been found accepting bribes and extorting businesses
asking for money and other favours. Agitated shopkeepers and businessmen approach the
courts against him, the government refuses to sanction his report. Under the above mentioned
law what would the courts do?

(a) The courts would continue the case against him as these charges are covered under the
Indian penal code.

(b) The courts will not be able to proceed as they require government sanction to prosecute
BASP officials.

(c) The courts are constitutional bodies hence they can continue with the prosecution of
Ashok.

(d) Courts will not be able to prosecute because he has legal immunity as he is a BSAP
officer.

 
. The government of Bihar has decided that several districts in the state which are troubled by
Naxals and Maoists would only and completely be administered by the officers of the BSAP.
The officers would be responsible for everything from public order to sanitation and
providing educational services. This decision of the government has been challenged in court.

(a) The court does not have the jurisdiction to rule on the matters relating to BASP according
to the above mentioned law

(b) The courts would rule in favour of the petitioner as the role of the police is to establish
public order and it cannot be involved in the administration.

(c) Since these areas are infested with Naxals and Maoists, the courts will allow the BSAP
officer to administer these districts.

(d) Administration of districts is not under the jurisdiction of BASP.

. Does this law passed by Bihar government go against the Article 21 of the Indian
constitution?

(a) Yes, this law puts undue burden on the accused which is a violation of rights.

(b) No, the restrictions and conditions that the law subjects the citizens to are reasonable in
nature.

(c) Yes, this law is against Article 21 of the Constitution as this violates the principles of
natural justice and personal liberty and does not follow the procedure established by law.

(d) No, the law has been passed to curb the problems of extremism, extortion etc. the law was
an essential requirement for the state.

. Akhil is a member of the opposition party in Bihar and is agitated by the government’s
alleged misgovernance. He is often seen raising his opposition on public platforms. The
government is disturbed and claims that his opposition stems out of his relations with
members of various gangs. In the month of March 2021 he was arrested under BSAP on the
charges of disturbing the peace of the state and having links with members of extortionary
gangs. Moreover he is asked by the BSAP officers to produce evidence against his will. He
says his rights have been violated. Is that true?

(a) His rights have not been violated as the procedure established by law while arresting him
was followed.

(b) His rights have been violated as he has been asked to produce evidence against himself
and undue burden is laid on him.

(c) The purpose of the BSAP is to fight these extortionary gangs, Akhil’s arrest and the
responsibility of burden of proof on him are legal.
(d) He has been forced against his will to produce evidence, this violates his right against
self[1]incrimination. His rights have indeed been violated as per the Constitution.

34
Passage (Q.64-Q.67): In a recent judgment, the Supreme Court stated that “when the
protector of people and society himself instead of protecting the people adopts brutality and
inhumanly beat the person who comes to the police station, it is a matter of great public
concern”. There are two provisions in the Constitution in this regard. Article 20(3), a pro-
liberty provision, protects an accused of any offence from being a witness against himself.
Article 22(4) regulates the preventive detention of a person by permitting it under certain
circumstances but that to for a maximum of three months. Article 20(3) is the safeguard
against custodial torture. In the D K Basu case in 19, the Supreme Court said that “the word
torture today has become synonymous with the darker side of the human civilisation” and
also declared protection from torture “within the inhibition of Article 21” — the right to life.
Torture includes both forceful and non-forceful measures. The methods of torture used in
2019 included hammering iron nails into the body, stretching legs apart, striking genitals,
stripping, waterboarding, electric shocks, and even more horrific examples. These victims of
custodial torture are mostly left “damaged” for life, unable to find closure, justice or even the
treatment they need in its aftermath. [Extracted from ‘Torture, an issue ignored’ by MR
Shamshad, published in the Indian Express https://indianexpress.com/article/opinion/torture-
an-issue-ignored-008/]

64. Shakti was a known gang leader. Once the police finally caught her, she was taken in
police custody. However, despite long hours of questioning, she never revealed or confessed
anything. Finally, the police decided to stop giving her food until she started to answer their
questions. On her lawyer’s next visit, she told him that she was being tortured. Are her
fundamental rights being violated?

(a) Yes, because she was being kept in preventive detention by being forced to stay in police
custody

(b) Yes, because depriving Shakti of food till she started answering questions amounts to
custodial torture

(c) No, because merely stopping the provision of food does not amount to custodial torture as
the police never used any physical or verbal force on her

(d) No, because Shakti could still choose to stay silent and no one is compelling her to be a
witness against herself

65. Chulbul Pandey beat Chhedi Singh black and blue. In fact, he beat him so bad that Chhedi
Singh ended up confessing even to those crimes which he did not commit or was even
remotely aware of. On the basis of his confession before the magistrate, the trial court found
him guilty on all the pleaded ground. The matter went to the High Court and Chhedi Singh
now challenges his conviction. Is he correct in doing so?

(a) Yes, because Chhedi was being kept in preventive detention by being locked up in police
custody for so long giving Chulbul Pandey to get such a confession
(b) Yes, because Chhedi Singh was compelled to be a witness against himself and was
physically tortured in police custody

(c) No, because being beat up in police custody is normal and does not amount to any kind of
torture or compulsion

(d) None of the above

66. Moti was standing outside a shop. When the shop was empty, he walked in and
threatened the shop owner to give him all the money from the cashbox. Frightened at the
sight of a long, sharp knife near his face, the shopkeeper started putting the money in a bag.
He also smartly managed to press the emergency button on his phone and before Moti could
escape, police arrived on the spot. When Moti was in police custody, he was frequently beat
up and given electric shots so that he would confess his crime. Are Moti’s fundamental rights
being violated in this case?

(a) Yes, Article 20(3) which protects an accused of any offence from being a witness against
himself

(b) Yes, Article 21 which is the right to life and also protects against torture of an individual

 (c) Both of the above

(d) None of the above

67. Abu Hummus was a student activist and was the leader of a student mob that turned
violent. To prevent further disruption, he was charged under the Unlawful Activities
Prevention Act (UAPA) which allows police to detain people even if they have not
committed any crime but there is an apprehension that the person may do some unlawful
activity. Abu Hummus challenged this detention by arguing that preventive detention is a
violation of his fundamental rights. Is he correct?

(a) Yes, because preventive detention is a violation of the right to life and personal liberty
under Article 21

(b) Yes, because preventive detention of a person amounts to custodial torture as Abu
Hummus is unnecessarily being kept in police custody where he may be beat up

(c) No, because Abu Hummus is a possible criminal so preventive detention in such a case is
allowed as he could make the students commit unlawful activities

(d) No, because preventive detention is allowed in certain cases as per the Constitution and in
this case UAPA allows preventive detention of Abu Hummus

Passage (Q.68-Q.): In the midst of the recent electoral campaign in Tamil Nadu, a movement
to free Hindu temples from state control gained some traction. Ask the proponents of the
movement who they see as the state’s successor and the answer you get is the community of
Hindus. But by that, what do they mean? Who selects the personnel that will constitute this
community? A lack of clear answers to these questions suggests that what the programme
really seeks is to plant in the state’s position powerful private interests, and in the process,
reinstate some of the hierarchical divisions that might have been negated, albeit not nearly
entirely, through the Constitution and its processes. It was with these ends in mind that
various caveats and provisos were written into Articles 25 and 26 of the Constitution. The
former makes the freedom of conscience and the right freely to profess practise and propagate
religion subject to public order, morality and health. What is more, this right, the provision
clarifies, will not stand in the way of the state making laws regulating any economic,
financial, or other secular activity associated with religion, or in the way of the state making
laws providing for social welfare and reform. Article 26, on the other hand, protects group
rights. It grants to every “religious denomination” the right to establish institutions; to
manage its own affairs in matters of religion; to own and acquire property; and to administer
that property in accordance with law. This right too is bound by considerations of public
order, morality, and health. A plain reading of these provisions shows us that a religious
denomination has substantial freedom over matters concerning its faith. But this right does
not override the state’s power to make laws regulating the management of properties
belonging to these denominations. The state’s authority is wider still in attending to religious
institutions that part takes a public character. The protections of Article 25 are expressly
restricted to matters within the domain of religion. Government has every authority to
regulate and restrict a secular function performed by a public religious institution. Source:
The Hindu 22 April 2021 https://www.thehindu.com/opinion/lead/the-last-word-on-the-state-
and-temples/article343441.ece 68. The government of Madhya Pradesh submitted a report to
Jabalpur high court stating that in 0 temples in the state, original idols have been stolen and
replaced with counterfeits and there is no pooja or ritual taking place as there is no revenue.
Therefore, the government of Madhya Pradesh wants temples under their control for
management. The Hindu community outrage at the decision of the government claiming that
a temple is not a business but the soul of the community. Decide can the state make the law
for the same.

(a) The very fact that the Hindu community is targeted and cannot keep its place of worship
amounts to nothing less than apartheid.

(b) Every citizen under Article 26 has the right to establish institutions; to manage its affairs
in matters of religion. Therefore, the government cannot take temples under control by
making laws.

(c) Government of Madhya Pradesh can take temples under its control for social reform in the
state.

(d) Government can take temples in their control as they are a good source of revenue.  

. Mr.Budhram Baghel the collector of Gadia district issued an order to demolish the on-going
construction of the church of Christ in the district because the land belongs to the government
and there is a Supreme Court order that no religious structure can be brought up on the
government land. The Christian forum initiated a mass movement against the government for
violating their Right to Religion. Decide.

(a) Government is violating the rights of the community by restricting them to practice and
profess their religion.

(b) Christians are part of India and have every right to build a church on government land.
(c) The citizens of India have the right to establish institutions and to manage their affairs in
matters of religion; to own and acquire property.

(d) Illegal construction is not permissible therefore the government is not violating the right
of citizens.

. Female Genital Mutilation ritual with both a religious as well as a cultural background is the
most brutal practice that exists even today in some parts of India. Mutilation of women parts
causes’ serious mental and physical health issues, therefore privately governed NGO
Samadhan restricts this practice and punishes those who practice Female Genital Mutilation.
People from a Community filled a writ petition for violation of their fundamental right to
religion. Decide.

(a) Samadhan NGO cannot restrict people to practice their religious rituals.

(b) Restrictions of NGOs are valid because the right freely to profess practise and propagate
religion subject to public order, morality, and health.

(c) Heinous practices like female genital mutilation should be restricted in every society to
protect the fundamental rights of women.

(d) NGO Samadhan has no authority to put restrictions on any religious rituals therefore it is
violating the rights of the community who practice female genital mutilation.

. Om Peaceful Om is a religious organisation and propagating its religious practices


nationally. They inspire teenagers to leave their family, career, and social life behind to
embrace faith in God and feel the tranquillity of the universe. Many teenagers started
following this path after attending a session with Om peaceful Om. Savvy young law
aspirants filled a petition against Om Peaceful Om for misleading teenagers and ruining the
youth of India by propagating religion that is against public morality in the opinion of Savvy.
Decide

(a) Om peaceful Om cannot propagate this type of religious practices which is against public
morality.

(b) Om peaceful Om has the right to freely profess practise and propagate religion without
violating restrictions of Article 25.

(c) Om peaceful Om should be encouraged at the national level because they are propagating
religious activity which will help in building the peaceful nation.

(d) Both B and C.

. A new religion, Moksha’s has this practice of the first child in each family being sent to a
religious school to learn the practice of Moksha’s voluntarily or involuntarily. The religious
school demands that all entrants join the school at the age of 14 and stay there till they
complete their education by the age of 25. Post this, student would be allotted one of the
schools Moksha runs where the students should be attached to till the age of 60. Students
should dedicate their lives to Moksha and disown their own families. Supreme Court receives
a petition from adherents of Moksha requesting that this practice be banned.

(a) The Supreme Court should not be interfering as Article 26 explicitly guarantees the right
of religion.

(b) The Supreme Court does not intervene as this is an essential tenet of religion Moksha.

(c) The Supreme Court does not intervene as Article 25 protects the right of any individual to
follow any religion.

(d) The Supreme Court should intervene to protect young children from being oppressed and
excluded from normal life by force.  

Passage (Q.-Q.): In the case of Romesh Thappar v State of Madras (10) the court held that
there can be no doubt that freedom of speech and expression includes freedom of propagation
of ideas, and that freedom is ensured by the freedom of circulation. There are circumstances
that warrant courts cautioning the media about the manner of its reportage of judicial
proceedings. An example of such a circumstance is when the media conducts a media trial in
criminal proceedings. However, the general view is that the public has a right to know about
what is happening in open court, and it is the duty of the media to disseminate information.
With the contents of the special leave petition and the role of the media and the EC in mind,
the judgement delivered by Justice Chandrachud in the recent case of The Chief Election
Commissioner of India v M.R. Vijayabhaskar & Ors (2021) .In its judgment, the court
recognises that both the EC and the high court are constitutional bodies and proceeds to
embark on striking a balancing act. The court reiterates the freedom of the press being
encapsulated in Article 19(1) (a) and being subject to the restrictions specified in Article
19(2) of the Constitution which are contempt of court, security of the state, incitement to an
offence and public order. The Supreme Court stresses the importance of reporting
proceedings that are conducted in an open court; these cases are vital sources of public
information about the activities of the legislature and the executive. Not all cases are heard in
open court; cases involving privacy of parties, such as matrimonial cases, or matters
involving child sexual abuse are held in-camera. While the court refuses to restrain the media
from reporting court proceedings and the dialogue between the bar and the bench, which
takes place during the proceedings, it sounds a word of caution for the judges of the high
court to refrain from making off-the cuff-oral remarks which may be misinterpreted by the
media. Thus, in conclusion, the decision of the court re-emphasises the important role the
media plays in our democracy. Source: The Wire 08/MAY/2021 https://thewire.in/law/why-
media-cant-be-blamed-for-reporting-on-judicial-proceedings

. Government of India fixes a maximum no of pages which a newspaper would be entitled to


print and prohibits a newspaper to publish a supplement or a new edition without permission
of the government. How government is violating freedom of speech and the expression of the
press?
(a) Government is not violating freedom of speech and expression of the press as regulating
maximum no of pages of newspaper control unfair competition among the newspaper.

(b) By imposing restrictions government curtails their right to the propagation of ideas as the
government is imposing a burden on the press which endangers its existence or circulation.

(c) Press have the right to comment on public affairs and to criticize the government
including its defence policy and the conduct of armed force.

(d) Restriction on media helps in maintains peace in the nation therefore the government is
not violating freedom of the press.

. Kal Tak media house is live streaming the case of two reputed business companies in the
Supreme Court. Kal Tak is live streaming the case proceedings considering it constitutional
and national importance case which should be a live stream to know the perspective of all
Indians and show the reality of business companies to the world at large. Kal Tak was
restricted by Hon’ble Supreme Court judges to live stream particular cases. Decide

(a) Supreme Court should not restrict media to live stream case proceedings because opening
the vista of the courtroom can epitomize transparency, good governance, and accountability.

(b) Freedom of the press is a subset of freedom of speech and expression of Article 19,
freedom to propagate ideas by way of circulation of press and media should not be restricted
by Hon’ble court.

(c) The Hon’ble court can restrict Kal Tak as privacy of parties should be safeguarded and
limiting the recording and broadcasting of its proceedings in certain cases ensure better
access to justice.

(d) Live streaming of court proceedings is manifested in the public interest and public interest
should be preserved through the constitution.  

. Selection commission of India is the central government body for recruitments of candidates
at central offices and facing legal challenges and battles in court these days for its prior
misconducts. The selection commission filed a plea in the court to stop media from reporting
oral observation during selection commission court proceedings. What will be the right
course of action? Answer following the passage.

(a) Court has to remain alive to evolving technology of media, it is not just if it restrained
media from reporting judicial proceedings excluding certain nature of cases.

(b) Media discharges an important function in adding vitality to democracy and it cannot be
restricted from covering court proceedings in any manner.

(c) Selection commission is a national body and all national government bodies should be
protected from media abuse to maintain pillars of democracy.
(d) Independence of judiciary is of utmost crucial for the functioning of the court and live
streaming of cases will hamper with the independence of judges and judiciary. Hence courts
have to restrain media from live streaming judicial proceedings.

. Famous cartoonist Vaibhav was charged with sedition by the Government for posturizing a
cartoon where the parliament was depicted as an unhygienic lavatory. He had done this to
protest against the corruption of the then government. The government took his right to
publish cartoons in the newspaper or social media. Decide.

(a) His picture is against the decency and morality of the country. Hence the government is
right in their conduct to take away Vaibhav fundamental right to speech and expression.

(b) As per the limitation to Article 19(2), it was an indecent depiction of the national
parliament of India.

(c) Vaibhav is not violating any limitation of Article 19(2) and didn’t incite or recommend
violence against the state.

(d) Vaibhav motive of the speech is only to protest against the government and disturb public
order.

Passage (Q.-Q): The term “Acceptance” has been defined under Section 2(b) of the Indian
Contract Act,18. According to the Section, an offer or proposal is said to have accepted when
the person to whom the proposal or offer to do or not to do an act is made if gives his assent
to such an act or omission. Therefore, acceptance of the contract is said to have taken place
when the person to whom the offer is made gives his assent or consent to the terms of the
contract. Acceptance to be legally enforceable must be absolute and unqualified. Acceptance
must be expressed or implied. Section 7(1) of the Indian Contract Act provides that in order
to turn an offer into an agreement the acceptance to the offer must be absolute and
unqualified. The logic behind the principle that the acceptance to the offer must be absolute
and unqualified is that when acceptance is not absolute and is qualified it results into a
counter offer which leads to the rejection of the original offer made by the offeror to the
offeree. If the offeree makes any variations in the original terms of the contract proposed to
him and then accepts the contract, such an acceptance would result in the invalidity of the
contract. It is also important that the acceptance made by the offeree should be in toto, i.e.
acceptance should be given to all the terms and conditions of the offer as acceptance of only a
part of the offer is not a good acceptance under the law. Source:
https://blog.ipleaders.in/acceptance-of-proposal/

. Mark bought expensive new LeBron shoes for $120 for his basketball match in school. The
shopkeeper told him that they were the best available in the market. However, he later found
that he had in fact bought an older model and better ones were available online. He decided to
sell the shoes online and buy better ones. He put up an advertisement with a picture of the
shoes, stating, “Brand new pair of LeBron basketball shoes on sale for $. Please contact if
interested.” Zack mails Mark offering to buy the shoes provided he reduces the price further.
Which of the following statements is correct?

(a) Anyone who contacts Mark regarding the shoes first would be bound by contract to buy
them.

(b) The advertisement by Mark constitutes a proposal and Zack’s mail converted it into a
promise.

(c) Both (a) and (b).

(d) Neither (a) nor (b).  

. Mark refuses to lower the price of the LeBron shoes. Zack finally consents to buy the shoes
at the originally set price of $.

(a) Mark is bound by contract law to sell the shoes to Zack for $.

(b) Zack’s mail to Mark was in the nature of a counter-offer.

(c) The original offer by Mark is still subsisting until Mark puts an end to it.

(d) Mark cannot sell the shoes to a third party over Zack.

. Tiger, a contract killer, was offered to kill a Home Minister by a stranger. He agreed to
accept the contract when he was acquitted by the court in a murder case which was under
trial. However, even after his acquittal, Tiger, refused to kill the minister. The stranger wants
to sue him for damages.

(a) The stranger cannot sue him for damages as contract killing is an illegal act.

(b) The stranger can sue him as he has breached the contract.

(c) The stranger cannot sue him as there was no contract in the first place due to the lack of
qualified acceptance.

(d) None of the above.

. Mary Jane offers to sell a farmhouse to Stanley for a consideration of Rs. 30 lakhs. Stanley
visited the farmhouse and found that the farmhouse was in a bad condition and therefore he
wrote to Mary Jane that he can buy the farm but for nothing more than Rs. 20 lakhs. Which
of the following options best describes the situation?

(a) Stanley has accepted the offer of Mary Jane

(b) Stanley has rejected the offer of Mary Jane


(c) Stanley has made a counter offer to Mary which amounts to rejection

(d) Stanley has partially accepted the offer of Mary Jane

. In the previous question, if Stanley had written in his letter to Mary Jane that ‘I promise to
pay you Rs. 20 lakhs only after deduction of the costs for imminent cleaning of weeds’;
which of the following options would have best described the situation—

(a) Stanley has rejected Mary Jane’s offer by express words

(b) Stanley has agreed to Mary Jane’s offer impliedly

(c) Stanley has entered into a promise with Mary Jane by express words

(d) Stanley has rejected Mary Jane’s offer impliedly

. A contract is signed between an Indian and Argentinian party. There is an arbitration clause
in the agreement. Immediately after signing the contract the Indian party objects to the
arbitration clause and writes a letter to the Argentinian party. Argentinian party doesn't reply
to it but permits the work to go on. What is the status of the arbitration clause now?

(a) The arbitration clause stands deleted now because the Argentinian party impliedly
accepted the Indian party's objection.

(b) The entire contract stands deleted now.

(c) The offer and acceptance apply to contract only. Not to alteration of contract.

(d) The Argentinian party's silence on the matter doesn't convey agreement and hence
agreement clause stands.

Passage (Q.-Q.): Any intentional false communication, either written or spoken , that harms a
person’s reputation; decreases the respect, regard or confidence in which a person is held; or
induces disparaging, hostile or disagreeable opinions or feelings against a person is known as
defamation.Defamation is the act of making untrue statements about another which damages
his/her reputation.It is a statement that injures someone’s reputation. Defamation is the act of
saying false things in order to make people have a bad opinion of someone. Defamation may
be defined as a communication to some person, other than the person defamed, of the matter
which tends to lower the plaintiff in the estimation of right thinking persons or to deter them
from associating or dealing with him. Defamation is a wrong done by a person to another’s
reputation by words, written or spoken, sign or other visible representation. If the statement is
made in writing and published in some permanent and visible form, then the defamation is
called libel otherwise Slander. Defamation is also an offence under  20 of 36 criminal law.
Section 4 Of IPC:- Whoever by words either spoken or intended to be read, or by signs or by
visible representations, makes or publishes any imputation concerning any person intending
to harm, or knowing or having reason to believe that such imputation will harm, the
reputation of such person is said to defame that person.Law makers have decided that one
cannot sue for defamation in certain instances when a statement is considered privileged.
Also, a fair and Bonafide comment on a matter of public interest is a defence in an action for
defamation. If it is not a statement of fact but an comment or criticism, then also defamation
is not attracted. The law of defamation is supposed to protect people’s reputation from unfair
attack. In practice its main effect is to hinder free speech and protect powerful people from
scrutiny. Defamation law allows people to sue those who say or publish false and malicious
comments. [SourceNehaMalhotra,“Defamation”,, as accessed on 17th April 2021]

. Mr. Balot was chief director of Mangaldas Company and was also an ex-husband of Mrs.
Poonam. Mrs. Poonam was chief executive officer of her fathers’ company, Amarchand. Her
company was a growing business and was earning just the double profits than those of Mr.
Balot’s Company. Mr. Balot was jealous of this quality of his ex-wife and wants to make the
company fall down and become insolvent. He prepared posters stating that Amarchand
Company Pvt. Ltd. is selling all the assets at lowest price of all and had turned out to be
bankrupt. Soon the CEO and the directors of the company were to be arrested. After seeing
this, shareholders reported the company and have sold out their shares in the company to
private individuals and Amarchand Company suffered losses. Decide-

(a) Mr. Balot is liable for offence of defamation as he has communicated the false statements
due to which the company has to suffer losses and its reputation has been affected.

(b) Mr. Balot is not liable for the offence of defamation as he has not communicated any false
statements about the company and company has suffered only monetary loss and not
reputation loss.

(c) Mr. Balot is liable for offence of defamation as he has communicated the true statements
about the company due to which it has to suffer monetary losses.

(d) Mr. Balot is not liable for the offence of defamation as he has communicated the false
statements about the company and not about the person.

. Harman and Bipin were good friends from their childhood. They were living together
nearby to each other and even play together sometimes. They were in same school and one
day during recess time, Pintu, another boy, asked Bipin about some secrets to be revealed
about Harman which no one knows. Bipin refused to do so and said he will ask Harman first
and then will them. He went to Harman and asked him if he could share any such secret with
other friends to which Harman get angry and asked him if he does that he too has his hidden
secrets. Bipin argued that he will go and tell everyone that Harman still pee in his pants when
he sees any horror dreams in night. Harman slapped Bipin and it get turned into a big fight.
Decide-
(a) Bipin is liable for defamation of Harman as the statement made by him was very much
capable of being hurting the reputation of Harman in front of his other friends.

(b) Bipin is not liable for defamation of Harman as the statement made by him was the truth
and Bipin can claim the defense of stating the truth about Harman.

(c) Bipin is liable for defamation of Harman as the statement made by him was with intention
to hurt the reputation of Harman among his other friends and can do so.

(d) Bipin is not liable for defamation of Harman as the statement made by him was very
much capable of being hurting the reputation of Harman but was made to Harman only.  

. Meenakshi, a Member of Parliament, was a Bollywood actor of her times and was selected
to be part of Upper house of Parliament nominated by the President. In one of the
parliamentary proceedings of Rajya Sabha, she was addressing her speech on the recent
Farmers Bill 2020 and pointed towards the Prime Minister while saying “Vibhishan has led to
the end of Ravan and lead Lanka to win by Ram, but the PM is no less than a Vibhishan who
wants to sell this country to the Capitalist Countries”. In reply to her speech Prime Minister
said that“Meenakshi was herself played the role of Surpankha and she should not talk about
the same”. Meenakshi filed a suit for defamation in Delhi High Court. Decide-

(a) Both the statements made by Meenakshi and Prime Minister amounts to offence of
defamation as they hurt the reputation of each other and hence both are liable for damages.

(b) Only statement made by Meenakshi amounts to defamation as she started defaming the
Prime Minister and reply of PM was a reaction to action of Meenakshi.

(c) Only statement made by Prime Minister amounts to defamation as PM should not speak
anything in the parliament which hurts the reputation of any member in eyes of reasonable
person.

(d) Both the statements made by Meenakshi and Prime Minister do not amount to defamation
though they were not capable of hurting the reputation of each other in eyes of reasonable
person.

. Priyanka was a student of class 12th and she was very intelligent in her studies. Rohan, a
student of class 11th was in love with her and he proposed her. Priyanka want to study more
and remain focused for her boards so decided not to engage herself in such relationships and
hence she refused Rohan’s proposal. Rohan friends tease him every day and night for being
rejected by a girl and his senior. Rohan was upset by this act and wrote a letter to Priyanka’s
friend stating that Priyanka is a characterless girl having many boyfriends and was having
sexual relation too with some of the boys. Also, he stated that Priyanka is ready to sleep with
him also and when he proposed her, she refused. He kept this letter on her friend’s desk and
went to play. The letter get wet due to water on the desk and her friend cannot read exactly
what was written except few letters of Characterless, sexual relations and ready to sleep. She
complained to the principal office and on inquiry the writing matched with Rohan. Decide-

(a) Rohan is not liable for defamation as he has no intention to defame Priyanka’s friend and
had wrote the letter for Priyanka which was mistaken by her friend for her.
(b) Rohan is liable for defamation as the statements written by her amounts to degrading
one’s reputation in eyes of others and is immaterial whether such statement is written for
Priyanka or her friend.

(c) Rohan is not liable for defamation as Priyanka’s friend misunderstood the letter for her
but was actually for Priyanka and hence there is no loss of reputation.

(d) Rohan is liable for defamation as he has done it to affect the reputation of Priyanka and
had made the false statements too in his letter.

. Choose the most appropriate option. Assertion (A): Section 4 of IPC defined defamation as
an offence and also attracts criminal liability and punishment under section 500 of Criminal
Law. It is not violative of freedom of speech. Reasoning(R): Article 19(1)(a) provides
fundamental right to free speech and expression but such right is not absolute in any way and
reasonable restrictions are imposed on such right under Article 19(2).

(a) A is correct and R is incorrect.

(b) Both A and R is correct and R is not incorrect explanation of A.

(c) Both A and R is correct but R is not correct explanation of A.

(d) A is incorrect but R is correct.  

. Choose from the following the most appropriate essential(s) of offence of Defamation. I.
The person about whom the defamatory statement is made must be ‘damaged’ by the
statement and damage must be actual, not mental. II. Statement must be communicated to the
public at large including the person defamed and must be published in sense that is available
in public domain. III. The statement, which must be about another person, which may or may
not be false. IV. In the context of libel or slander, ‘per se’ means defamation that is
intrinsically damaging.

(a) Both II and III

(b) I, III and IV

(c) Only IV

(d) Both I and III

Passage (Q.-Q.): In common law, assault is a tort, an act of the defendant which causes to the
plaintiff reasonable apprehension of the infliction of a battery on him by the defendant. When
the defendant creates his act by an apprehension in the mind of the plaintiff that he is going to
commit battery against the plaintiff, the wrong of assault is completed. The wrong consists of
an attempt to do harm rather than the harm being caused thereby. In assault charges must
include conduct that is offensive or causes another person to the fear of their safety. This
clearly means that one can be guilty of assault even if he/she did not physically harm the
victim. Elements of the crime of assault are: An act or conduct intended to created: To prove
a criminal attack, the defendants’ behaviour must be motivated to create a situation of fear or
danger in the victim’s mind. A reasonable apprehension: Further, the victim must reasonably
believe that the defendant’s conduct will harm or humiliate him. Of imminent harm: The
victim’s fear must be a direct response to a threat that is imminent. In addition, there must be
some kind of perceived physical threat to the victim in the loss; It is believed that the
defendant’s actions would cause physical danger or abusive behaviour to the victim. All of
the above elements must be present and the evidence must be supported with evidence if
found guilty for the attack. It can be difficult to prove whether the defendant actually
intended the attack. Similarly, judges often spend a lot of time determining whether a
defendant’s actions are considered harmful or abusive. In determining this, they will consider
what an average person may perceive as harmful or aggressive.

. Farhan was sitting in his car parked in a non-parking zone and soon he was approached by a
police officer who asked him to take the vehicle. Farhan did so what was asked. While doing
so he overturned his car and rolled over a police officer’s leg. The officer forcefully asked
him to remove the car from his leg, to which Farhan swore him and refused to take the
vehicle and shut down the engine. A suit was filed against Farhan by the police officer for
assault. Decide-

(a) Farhan is not liable for assault on Police officer as his act was motivated or created by the
Police Officer himself and was not intended by Farhan.

(b) Farhan is liable for assault on Police officer as his act has caused physical danger to
police officer and was abusive behaviour in not removing the car.

(c) Farhan is not liable for assault on Police officer as his act was no having reasonable
apprehension of danger or threat to the police officer.

(d) Farhan is liable for assault on Police officer as his act was offensive rather than causing
actual danger or harm to the police officer.  

. Sarita and Salman were in love with each other and want to get married but their families
were against it. Sarita decided to move ahead in her life and get married to Pranav. For a
period of almost two years, Salman followed the Sarita home from work, made numerous
silent phone calls, wrote her over 0 letters, drove past her house, visited her house without her
consent, and wrote offensive words on her house’s door three times. Following these actions,
she received two additional letters with threatening language. She was soon diagnosed by a
doctor as suffering from clinical depression and anxiety due to man’s actions and letters.
Decide-

(a) The Act of Salman amounts to assault as his conduct was created with an apprehension
and has offensively caused imminent mental harm to Sarita.

(b) The Act of Salman does not amount to assault as his conduct was not created with an
apprehension and was not offensive in nature.
(c) The Act of Salman amounts to assault as his conduct was motivated and had caused actual
and real physical damage to Sarita.

(d) The Act of Salman does not amount to assault as his conduct has not caused any actual or
much apprehension to Sarita to qualify assault.

. Manoj and Sarla were married couple. One day an argument has taken place between them
and subsequently it was turned into a big fight. The matter was about the dowry demand
Manoj and his family was demanding from him. Soon the mother of Manoj interfered the
fight and asked both of them to keep quiet and remain silent. Sarla was having knife in her
hand and enraged with anger she showed it to her and ask her to remain out of this quarrel.
Her mother-in-law get apprehended from such act and get fever the next day. Decide-

(a) Sarla is liable for assault on her mother-in-law as she has shown her a knife which has
caused reasonable apprehension or threat of harm.

(b) Sarla is not liable for assault on her mother-in-law as she has by mistake shown her a
knife when she was in anger which might has caused reasonable apprehension or threat of
harm.

(c) Sarla is liable for assault on her mother-in-law as she has intentionally apprehended her
mother-in[1]law as she was also angry with her demand of dowry.

(d) Sarla is not liable for assault on her mother-in-law as she has shown her a knife which has
caused reasonable apprehension or threat of harm but no such actual damage was caused to
her mother-in[1]law.

. Chintu was the naughty boy of his class and always remain engaged in sports and playing
activities. He was clever but not intelligent. His mother was not happy with his behaviour and
was always in fear of his future. One day, he was assigned with a home work which he had
not completed on time and when asked by the teacher he argued with her. The teacher
shouted on him and asked him to complete the same by end of week or she will beat her in
the next class. Chintu get into fever from this scolding. Chintu’s mother filed a suit against
teacher for assault. Decide-

(a) There is no assault on Chintu by the teacher. Mere scolding and future threat of beating
would not amount to sufficient apprehension or fear with no imminent harm.

(b) There is assault on Chintu by the teacher. Scolding and giving future threat of beating
would amount to sufficient apprehension of threat or fear with imminent harm.

(c) There is no assault on Chintu by the teacher. He should have completed the work on time
and this was with mere intention to make Chintu work in disciplined manner.

(d) There is assault on Chintu by the teacher. There was actual damage caused to Chintu
mentally and there was sufficient threat or fear.  

 
. Which among the following can be valid ground of defence in cases of assault? I. Using the
amount or display of force that is appropriate in the situation and in proportion to the force
being used against them. II. Intoxication when affects a person’s ability to act intentionally.
III. Coercion- force to attack under threat of harm. IV. The elements of proof are found or
supported with the correct evidence.

(a) Only III

(b) Both I and II

(c) Only IV

(d) I, II and III

. Choose the most appropriate option.

Assertion (A): Threat of violence is enough for assault. No physical contact is necessary.
Also, we can say assault is an attempt to commit battery.

Reasoning(R): Battery includes intentional application of force to another person without any
lawful justification. There should be use of force with physical harm.

(a) A is correct and R is incorrect.

(b) Both A and R is correct and R is not incorrect explanation of A.

(c) Both A and R is correct but R is not correct explanation of A.

(d) A is incorrect but R is correct.

Passage (Q.-Q.): The law of tort is said to be the development of the maxim Ubi jus ibi
remedium. The word “jus” means legal authority to do something or to demand something.
The word “remedium” means that the person has the right of action in the court of law. The
literal meaning of the maxim ‘Ubi jus ibi remedium’ is where there is a wrong there is a
remedy. This maxim also says that there is no remedy without any wrong and the persons
whose right is being violated has a right to stand before the court of law. This principle also
states that if the rights are available to a person then it is required to be maintained by that
person only and remedy is available only when he is injured in the exercise of duty or
enjoyment of it; It is useless to imagine and think a right without a remedy. It is necessary to
keep in mind that both rights violated and the remedy sought or to be obtained should be
legal. There are many moral and political wrong but are not actionable or it does not give
many sufficient reasons to take legal action as they are not recognized by law. Thus, the
maxim does not mean that there is a remedy for every possible wrong. It is appropriately said
by Justice Stephen that maxim would be correctly stated if it were to say that “where there is
no legal remedy, there is no legal wrong.

. X and Y enter into a contract. It is standard practice and is always preferred to enter into
contracts on stamp paper as it ensures greater enforceability and validity of the contract.
However, it is not illegal and perfectly legal even if a contract is made on a tissue paper,
provided it has all the essentials of a contract. X makes the contract on a ruled sheet. In the
event of a breach of contract, Y goes to court and finds it difficult to get the contract to be
taken seriously because it wasn’t made on a stamp paper. Can Y claim the applicability of ubi
jus ibi remedium against X for making the contract on a ruled sheet?

(a) Yes, because a legal wrong has been committed against Y by not making the contract on a
stamp paper

(b) No, because there is no legal wrong and Y does not have any legal right which has been
violated so he cannot claim a remedy for the same

(c) Yes, because making a contract on a random sheet of paper is a moral wrong and where
there is a wrong there is a remedy

(d) None of the above  

. Jethalal was a registered voter. He was going to cast his vote but when he reached the
polling booth, he was detained from giving a vote by the police officer in-charge. The party
he wanted to vote for had won the election but Jethalal filed a suit against the police officer
stating that he was detained from giving a vote and his right to vote was infringed. Can he
claim a remedy for not being allowed to cast a vote?

(a) Yes, because Jethalal has a legal right to vote and he must be allowed to seek a remedy
when this right was violated by the police officer

(b) Yes, because although the right to vote is not a legal right, not being allowed to cast a
vote is still a wrong which must be remedied

(c) No, because the party he wanted to vote for won the election so there is no real wrong that
has been caused to Jethalal

(d) No, because not being allowed to vote is anyway not a legal wrong to be compensated for

. Jayaben was an elected Member of the Legislative Assembly (MLA). She was on her way to
attend a parliamentary meeting when her car was stopped by the police and she was taken
into custody. She was not produced before a magistrate and by the time she was released the
next day, she had missed the parliamentary meeting. It was later found that the arrest was also
wrongful in nature. Can Jayaben seek a remedy under the principle of ubi jus ibi remedium?
(a) No, because no legal right has been violated to claim a remedy against as a missing a
meeting does not amount to violation of a legal right

(b) Yes, because missing a parliamentary meeting amounts to infringing of a legal right as it
is her right as an MLA to attend such meetings

(c) Yes, because her fundamental right to be produced before a magistrate after being in
custody was violated
(d) Both (b) and (c)

. Mohan deposited some money in his account because he was due to receive a cheque and
was told that to receive the cheque amount he must have sufficient balance in his account.
However, when the cheque was deposited, the bank manager refused to honour the same and
did not allow the cheque amount to be transferred. Is the bank manager in violation of the
principles mentioned in the passage?

(a) No, because not transferring the cheque amount is not a legal wrong

(b) No, because bank managers have the power to refuse customers even if it is something
they are entitled to receive

(c) Yes, because failure to transfer the cheque amount despite sufficient balance is a violation
of Mohan’s rights as a customer

(d) Both (a) and (b)

Passage (Q.-Q.): The offence of criminal conspiracy is defined under Section 120-A of
Chapter V[1]A of the Indian Penal Code, when two or more persons come together and agree
to do, or cause something to be done, which constitutes an illegal act or a legal act carried
forward by illegal means, such persons would be guilty of the commission of the offence of
criminal conspiracy. In simpler terms, conspiracy refers to the meeting of minds for the
commission of an offence. However, no such agreement would constitute the offence of
criminal conspiracy, unless and until an act is performed in furtherance of such an agreement.
The explanation attached to Section 120-A makes it clear that it is immaterial whether the
illegal act committed in furtherance of such an agreement, is the focal point of the agreement
or, is merely incidental to the performance of the ultimate goal of the agreement. In the case
of State (Delhi Admn) v. V.C.Shukla, the court observed that: “To prove a criminal
conspiracy which is punishable under S. 120-B of IPC, there must be direct or circumstantial
evidence to show that there was an agreement between two or more persons to commit an
offence. This envisages that there must be a meeting of minds resulting in an ultimate
decision taken by the conspirators regarding the commission of an offence. [Extracted from
‘Criminal law: Concept of Criminal Conspiracy’ by Disha Tulsyan and Salonee Nayak
published on lexlife https://lexlife.in/2020/06/04/criminal-law-concept-of-criminal-
conspiracy/]  

. Hema, Rekha, Jaya and Sushma are four friends. One day they all meet in a park and are
having a casual conversation about what it feels like to be rich. Hema jokingly suggested that
they should all rob a bank so that they can be happy for life. All the four friends daydream
about this plan and later laugh it off and go back home. Next day, Hema alone actually
executes the plan from the previous day and is caught. Can Hema, Rekha, Jaya and Sushma
be arrested for criminal conspiracy?

(a) Yes, because all four of them planned about the robbery and had a common intention even
though it was only executed by Hema
(b) Yes, because there was a meeting of the minds between all four of them even though only
one of them committed an act in furtherance of that intention

(c) No, because although there was meeting of the minds between the four friends only one of
them committed an act in furtherance of that intention

(d) No, because there was no meeting of the minds between the four friends for robbing a
back even if one of them committed an act in furtherance of that intention

. Roy and Raja are two friends who decide to rob an old lady. Roy drives the car and stands
outside the house to keep a watch while Raja goes inside and steals all the valuables. Roy
now feels bad about robbing an old lady and decides to let Raja keep all the valuables
because he didn’t want to be part of something like this. Before Raja could sneak out of the
house, an alarm is sounded and the police come on the spot. Can Roy be arrested for
conspiracy?

(a) Yes, because Roy and Raja together agreed to rob the old lady and it doesn’t matter if Roy
didn’t go inside the house or later decided to let Raja keep all the valuables

(b) No, because there is no theft and the act could not be performed as the police came on the
spot before the two could escape

(c) No, because there is no meeting of the minds as Roy later decided to let Raja keep all the
valuables

(d) No, because Roy merely drove the car and stood outside the house and didn’t himself
commit any act in furtherance of the common intention

. Popat and Lal decide to bomb a building. The stay up all night coming up with plans and
making a chemical bomb. Next morning, Popat and Lal place the bomb in the basement of
the building and take the remote with them. They drive to a distance from the building and
press the blast button on the remote. They wait for 10 minutes but no blast happens.
Disappointed, they leave the spot. Later on police comes and arrests them for terrorist
conspiracy. Is the charge of conspiracy valid?

(a) No, because no bomb blast actually happened so there was no act in furtherance of their
plan to bomb the building

(b) Yes, because even though the blast didn’t happen there was meeting of the minds and
they still acted in furtherance of the same by making the bomb and placing it

(c) Yes, because placing the bomb was incidental to bombing the building

(d) No, because they can be charged with terrorism and not conspiracy as they didn’t have
any agreement in place
 

. P, Q and R sell guns in Indiana. It is legal to sell weapons like guns in the open market in
Indiana. One day, they were running short on supply so they decided to procure gungs from
Paka. It is forbidden to have any business relation with Paka in Indiana. P drives to the
border, Q talks to the dealer and R drives back home. Can they be charged with conspiracy
even though they have not yet sold the weapons?

(a) No, because it is legal to sell weapons in Indiana and it does not matter from where they
are procured

(b) No, because there was no meeting of the minds and no single act in furtherance of that
intention as P, Q and R did separate jobs

(c) Yes, because P, Q and R are selling guns which is an illegal business and procuring guns
for the same is acting in furtherance of a common intention

(d) Yes, because even though they have not sold the guns and selling them is also legal, the
guns were procured illegally and all of them acted in furtherance of that common intention

35
Passage (Q.68-Q.): Agreement in restraint of trade is void under Section 27 of the Act. That
is, any agreement that debars one person from starting or continuing his trade or profession,
in return for some consideration is void. Therefore, any agreement stopping a person from
trading in the manner he likes or wherever he likes, on an agreement with other party, in
which the other party benefits from him stopping his trade or profession, will be called an
agreement in restraint of trade. All agreements in restraint of trade are void. The background
for delegitimizing an agreement in restraint of trade lies in the history of conflict between
free markets and the freedom of contracts. Ensuring freedom to the contract would mean
legitimizing agreements in restraint of trade, which would result in parties agreeing to curb
competition. In common law, a test of reasonability is followed. An agreement in restraint of
trade is valid, if: 1. There is a valid interest that the party imposing the restraint is trying to
protect. 2. The restraint is no more than that which is necessary to protect this interest. 3.
Restraint is not contrary to public interest. Section 27 of the Indian Contract Act declares all
agreements in restraint of trade are void with the only exception being Sale of Goodwill. Yet,
it is important to understand that these agreements are void.Unlike the common law, even
partial agreements in restraint of trade or reasonable restraint are not valid under the Contract
Act.Goodwill is an important asset because a customer is expected to engage with the same
favorable firm, that he was engaged with earlier, because of its name and reputation. After a
sale of goodwill, the seller continues to enjoy the right to carry out a competing business.

68. Hussain and Farhan were two neighbors and were also having same business of Private
Tourist Buses and other transportation facilities. Hussain business was a bigger one as
compared to Farhan. But Farhan had famous business instead of a bigger one. Farhan want to
leave the city and went to another city due to his family issues. In lieu of this, he entered into
agreement with Hussain where he transferred all his business to Hussain and decided that
Farhan would not engage in same business with same name for another two years. Soon
Farhan was leaving the city, he asked Hussain for complete payment as termed in the
agreement. Hussain defaulted in the payments. Farhan sued Hussain for restoration of his
business but Hussain contended that agreement is void since it is in restraint of trade. Decide

(a) Hussain contention is not correct as the agreement was not in restraint of trade but a valid
agreement and Farhan can claim his remaining amount from Hussain.

(b) Hussain contention is correct as the agreement was in restraint of trade and hence a void
agreement. Farhan cannot claim his remaining amount from Hussain.

(c) Hussain contention is not incorrect as the agreement was not in restraint of trade but a
void agreement. Thus, Farhan cannot claim his remaining amount from Hussain.

(d) Hussain contention is incorrect as the agreement was in restraint of trade but a valid
agreement. Farhan can claim his remaining amount from Hussain.

. Bodhi and Manu were both barbers and were good friends too. They have very big salon
shops in city of Delhi, Old Rajendra Nagar. Since both were in same profession, they decided
mutually that for first 10 days of a month, Bodhi will open his shop. For next 10 consecutive
days, Manu will open his shop and for remaining days of each month, 10 or 11, they both will
open their respective shops together. On starting of month of April 2021, Bodhi opened his
shop for 10 consecutive days till 10th April 2021 and till that time Manu had not opened his
shop for a single day. On 11 th April Manu opened his shop and saw Bodhi also opened his
shop. He asked him to shut down his shop as decided in the agreement. Bodhi refused to do
so and stated the agreement is not valid. Decide –

(a) The agreement entered between Bodhi and Manu was void as it was restraining both
Bodhi and Manu to have trade in their respective businesses.

(b) The agreement entered between Bodhi and Manu was valid as it was not restraining either
Bodhi or Manu to have trade in their respective businesses but was balanced arrangement.

(c) The agreement entered between Bodhi and Manu was voidable at the option of either
party as it was restraining both Bodhi and Manu completely to have trade in their respective
businesses.

(d) The agreement entered between Bodhi and Manu was not void as it was not restraining
either Bodhi or Manu completely to have trade in their respective businesses.

. Coca Cola Pvt. Ltd. Company was engaged in manufacturing of famous soft drink Coca
Cola and had advertised about new and fresh flavors including lime and mint. The company
thought to change its bottle designs and entered into contract with another Pvt. Company
manufacturing bottles for such soft drinks. Coca Cola had put a term in the agreement where
the company would bottle the soft drinks for Coca Cola and it would not bottle any other soft
drinks of the competitor companies of Coca Cola during the period of this existing contract.
The bottling company challenged the validity of the contract. Decide –

(a) Contract between bottling company and Coca Cola is void as it was clearly in restraint of
trade or business of bottling company to bottle soft drinks for other companies.
(b) Contract between bottling company and Coca Cola is valid as it was not in restraint of
trade or business of bottling company to bottle soft drinks for other companies.

(c) Contract between bottling company and Coca Cola is not void as it was in restraint of
trade or business of bottling company but a reasonable restriction on such trade.

(d) Contract between bottling company and Coca Cola is not valid as it was in restraint of
trade or business of bottling company but for the benefit of Coca Cola Company.

. Legal Principle - Any restriction on trade can be valid if it restricts for operating similar
business within local limits and should be a reasonable restriction.

Factual Situation: Mr. Ramesh, Prakash and Suresh were good friends and thought of
carrying a new trade or business of supplying fresh vegetables at door step of each
individuals living in Old Rajendra Nagar, New Delhi. They started the business and soon find
Mr. Ratan as their competitor who had already in the business for past 3 years and was selling
vegetables at cheaper prices. They entered into contract with Ratan for not selling vegetables
for next 4 years anywhere in India and he was given good amount of consideration for the
same. Soon after a year, they found Ratan selling famous Nagpuri Tomatoes in Old Rajendra
Nagar. He argued that tomatoes are fruits and not vegetables. Decide –

(a) The contract between Ramesh, Suresh, Prakash and Ratan was valid and they can sue
Ratan for breach of contract as in general parlance tomatoes are vegetables.

(b) The contract between Ramesh, Suresh, Prakash and Ratan was void and they cannot sue
Ratan for breach of contract as biologically tomatoes are fruits.

(c) The contract between Ramesh, Suresh, Prakash and Ratan was voidable at the option of
Ratan and they can sue Ratan for breach of contract but only during pendency of contract.

(d) The contract between Ramesh, Suresh, Prakash and Ratan was void and they cannot sue
Ratan for breach of contract though he was selling tomatoes which in general parlance are
vegetables.

. Choose the correct legal position as per the Indian Contract Law with respect to Restraint of
Trade. I. Agreements in restraint of trade under section 27 of Indian Contract Act are void per
se and illegal in nature. They cannot be entered into between the parties. II. Agreements
under section 27 of Indian Contract Act are unlawful and are void. III. Agreements in
restraint of trade is not void per se but is against the public interest at large, which makes
them void. IV. Agreements in restraint of trade are unenforceable in court of law and are
void.

(a) Both I and II

(b) Only IV

(c) Both III and IV


(d) Only II

. Which of the following maxims is best suitable to define the nature of void agreements
under Section 27 of Indian Contract Act?

(a) Actio Personalis Moritur Cum Persona

(b) Pro Tanto

(c) Ei Incumbit Probation Qui Dicit Non Qui Negat

(d) Nunc Pro Tunc

Passage (Q.-Q.): In ancient India, it was an undisputed principle that no one is exempted from
the operation of law. This liability to equal punishment extended even to the king, relative of
the king, a judge or an ordinary citizen. The rule of law was considered supreme and binding
on everyone alike. The important functions of the king were concerned with protection of
people, punishment of crimes and maintenance of dharma or social order. In the medieval
Indian history the personal liability of officers for their wrongs was more vogues with
evidences showing equality between the ruler and the ruled subject. Only when the king
considered it proper to undertake the burden of public officer, it was then the state treasury
used to pay the compensation. Sovereign functions are those actions of the state for which it
is not answerable in any court of law. Therefore, they are not amenable to jurisdiction of
ordinary civil court. The State is immune from being sued, as the jurisdiction of the courts in
such matters is impliedly barred. A consideration of the pre-Constitution cases of the
Government’s liability in tort begins with the judgment of the Supreme Court of Calcutta in
the case P. & O. Steam Navigation Co. v. Secretary of State. The principle of this case holds
that if any act was done in the exercise of sovereign functions, the East India Company or the
State would not be liable. It drew quite a clear distinction between the sovereign and non-
sovereign functions of the state. State’s liability for the acts or omissions of statutory
authorities arises only in cases where the statutory authority acts outside his legal authority
while purporting to act pursuant to the legal authority conferred upon him and the act or
omission, which causes or results in damage to a person, is not within the ambit of the
statutory protection, if any, contained in such enactments. This rule is evolved for the obvious
reason that an act done under a statute and in accordance with the statute can never amount to
a tort as was said by the Supreme Court in Martin Burn Ltd. Vs. Calcutta Corporation. The
Court said “A result flowing from a statutory provision is never an evil”.

. In state of Jhumritaliya, population has increased with high magnitude in past 10 years and
in such times of COVID-19 it becomes very difficult for the citizens to move from one place
to other. Railways, in such times, become the only reliable means of transportation. Thus,
government of Jhumritaliya decided to expand the routes for better utilisation and to solve the
situation of crisis. Government also allowed the privatisation of railways and purchased 4
latest super fast electrical trains from state of Ratrani. Routes of these trains were in vicinity
of residential area and due to their high speed vibrations build some cracks and starts forming
space in these well maintained houses. People sued the government of Jhumritaliya for
compensation. Decide –

(a) Government of Jhumritaliya cannot be sued for the damages by the resident people
because they were performing within their sovereign functions and hence not liable.

(b) Government of Jhumritaliya can be sued for the damages by the resident people because
they were performing within non-sovereign functions and hence liable.

(c) Government of Jhumritaliya cannot be sued for the damages by the resident people.
Though they were performing within non-sovereign functions but for wider public interest.

(d) Government of Jhumritaliya can be sued for the damages by the resident people. Though
they were performing within their sovereign functions but had caused damage to private
individuals

. Madhusudan, a business, was famous for his jewellery designs all over the world. They use
traditional and typical designs in jewellery and use pure quality of gold and silver metals for
the same. Soon it came into notice that the government is thinking to impose national
lockdown in the country from March 2020 in lieu of COVID-19 and hence all his gold has to
be settled in form of stocks before it. He approached RBI, Reserve Bank of India, to take the
gold bars from him and issue gold bonds instead for the same. And whenever the situation
gets back to normal, he will return the bonds to RBI and take back his gold bars. Soon the
situation gets back to normal in November 2020 and Madhusudan approached the RBI. RBI
returned him gold bars and takes back the bonds. He saw that gold bars were less in amount
for Rs.150 Crores and sued RBI along with Government of India (GOI) for his lost gold bars.
Decide –

(a) Government of India is not liable and Madhusudan cannot claim damages for lost gold as
issuing gold bonds by RBI is a sovereign function within GOI.

(b) Government of India is liable and Madhusudan can claim valid damages for lost gold as
issuing gold bonds by RBI is a non-sovereign function within GOI.

(c) Government of India is not liable but Madhusudan can claim damages for lost gold from
RBI as RBI is not a statutory body under GOI and hence liable for its own acts.

(d) Government of India is liable but Madhusudan cannot claim damages for lost gold from
RBI. Though RBI is a statutory body under GOI but vicarious liability is on GOI for act or
omission of RBI.

. Rustom, a Naval Officer, was commanded by his battalion leader to get the Indian slaves
release which were illegally detained by the state of Jhumritaliya. Officer went to the state of
Jhumritaliya where these Indian slaves were engaged forcefully in terrorist activities and
other terror funding business. He successfully helped the slave to exit from the detained place
and also in addition set fire to the nearby area where these people were detained. He did so in
order to rescue easily from the state. Soon the fire extended to the nearby residential area and
cause heavy damages to the other citizens burning their homes. State of Jhumritaliya sued
Government of India (GOI) and Indian Navy Force for the damages. Decide –

(a) Rustom has acted in excess to his statutory power which was not commanded by the head
of battalion and thus is a non-sovereign function. GOI is liable for his acts.

(b) Rustom has not acted in excess to his statutory power and was commanded by the head of
battalion and thus is a sovereign function. GOI is not liable for his acts.

(c) Rustom has acted in excess to his statutory power but was necessary to rescue the slaves
safely. This is a sovereign function and GOI is not liable for his acts.

(d) Rustom has not acted in excess to his statutory power but caused the inevitable damage to
the citizens of Jhumritaliya. This is a non-sovereign function and GOI is liable for his acts.

. In the case of Kasturilal vs. State of Uttar Pradesh, the plaintiff had been arrested by the
police officers of UP on a suspicion of possessing some stolen property. Upon investigation,
a large quantity of gold was found and was seized under the provisions of the Code of
Criminal Procedure. Ultimately, he was released, but the gold was not returned, as the Head
Constable in charge of the maalkhana, where the said gold had been stored, had absconded
with the gold. The plaintiff thereupon brought a suit against the State of Uttar Pradesh for the
return of the gold or alternatively, for damages for the loss caused to him. It was contended
that the concerned police officers had failed to take the requisite care of the gold seized from
the plaintiff and hence government of Uttar Pradesh should be held liable for compensation.
Decide –

(a) Police officers were acting within the ambit of sovereign function of the state and hence
no liability occurs on the state for such lost gold by their constable.

(b) Police officers were not acting within the ambit of sovereign function of the state and
hence liability occurs on the state for compensating such loss to the plaintiff.

(c) Police officers were acting within the ambit of sovereign function of the state but state is
liable for the omission of police officers to take due care of the seized property of the plaintiff
and thus, compensate him.

(d) Police officers were not acting within the ambit of sovereign function of the state but state
is not liable for the unlawful act of an individual police officer.

. As compared to the ancient time where even king was subject to punishment for his wrongs
and was not above law, in medieval times this fact has changed and evolved the distinction
between sovereign and non-sovereign functions of the state and hence liability as to such
wrongs has been decided accordingly. Which among the following maxim(s) is/are best
suitable with the above context of vicarious liability of medieval times? I. Respondeat
Superior II. Caveat emptor III. Rex Non Potest Peccare IV. Juris Et De Jure

(a) Both I and IV


(b) Only I

(c) Both I and III

(d) I, II and IV

. Which among the following is/are sovereign function(s) and would not attract(s) state’s
liability?

(a) Government of India had mismanaged the COVID-19 funds by diverting them to
investments in other countries in order to strength its foreign affairs.

(b) Dr. Nagendra, gynaecologist in government hospital, had been negligent in taking out the
sterilization operation of a lady due to which she conceived another child.

(c) Jagdish, a driver, was appointed for Mr.Abhilash, collector of Udaipur City, had knocked
down a pregnant lady while returning from the workshop after car repair.

(d) Both (a) and (c).

Passage (Q.-Q.): The victim was residing at Spoorthi Adoption and Fit Institution where she
was given in adoption but because of some differences between the adopted child and the
family, adoption was cancelled and the child started staying in Spoorthi Institution. The
petitioner used to enter the institution during night hours and have sexual intercourse with the
victims who are staying in the said Institution. Cases of sexual assault were also reported.
The Court further observed that Section 35 of the POCSO Act consists of two parts: firstly, it
deals with the period for recording of evidence of the child and disposal of case. Section
35(1) states that the evidence of the child shall be recorded within a period of thirty days of
the Special Court taking cognizance of the offence and reasons for delay, if any, shall be
recorded by the Special Court. Secondly, clause (2) prescribes the period of one year from the
date of taking cognizance of the offence for the purpose of completion of the trial. Of course,
the said period prescribed is to be complied with, as far as possible, by the Special Court. The
POCSO Act being a special piece of legislation must over-ride the general legislation.
Section 164 of Cr.P.C. deals with recording of confessions and statements by any
Metropolitan or Judicial Magistrate made to him in the course of an investigation, the same is
relatable to Sections 25 and 26 of the POCSO Act. But, Section 35 of the POCSO Act does
not deal with recording of statement of a child, but recording of evidence of the child and
disposal of the case. A statement under Section 164 of Cr.P.C. is during the course of
investigation or at any time afterwards before the commencement of the trial. But, the
evidence recorded before the Special Court under Section 35 of the POCSO Act is during the
course of the trial. [Source- ‘Can the statement recorded under Section 164 of Cr.PC be
considered to be evidence under Section 35 of the POCSO Act? Whether the accused is
entitled to bail in case the mandate under Section 35 of the POCSO Act has not been
completed?’, SCC Online, , as accessed on 22nd May 2021] 20 of 40 . Abhimanyu, a boy
aged 18 years of age, wants to get married with Sheetal, who was a minor girl. He asked his
family to consent this marriage. Soon, Sheetal got married with Abhimanya and they together
had consensual sexual intercourse. Sheetal’s father lodged an FIR, on November 2020,
against Abhimanyu under POCSO. The officials arrested the accused and have taken him into
custody. Abhimanyu denied the allegations and contended that the sexual intercourse was
with the consent of Sheetal, who was still a minor. Sheetal was not ready to give any
evidence against Abhimanyu and also not supported the fact that she had consented to their
relationship. The court asked for evidence recording from Sheetal to which she refused.
Later, her family forced her to speak up and finally she recorded her evidence against
Abhimanyu on May 2021. The Court relied on the evidence of Sheetal and ordered
imprisonment for 2 years to Abhimanyu. In an appeal to this case, Abhimanyu can contend

(a) Abhimanyu can defend the order of lower court and validity of his imprisonment on
grounds of irregularity in the procedure given under section 35 of POCSO.

(b) Abhimanyu cannot defend the order of lower court as they were correct in their decision
as Sheetal had given evidence to the court. Appeal would fail.

(c) Abhimanyu can defend the order of lower court as it should have disposed the trial under
the said provisions of POCSO within a year and this amount to irregularity in trial procedure
under section 35 of POCSO.

(d) Abhimanyu cannot defend the order of lower court as they have followed the due
procedure of law as given under section 35 of POCSO and section 164 of Cr.P.C.

. Mayur and Meena were in 10th standard and were studying in St. Josephs Sr. Sec. School.
They were good friends to each other. Mayur was much interested in Meena and want her to
be his girlfriend. He was in love with Meena. Meena, on the other side, was a sincere and
hardworking girl. She was among the toppers of the class and had no such intention to ever
had affair with someone in her school days. Mayur’s friends asked him to prove his
masculinity and for this he had to have sexual intercourse with Meena. Mayur caught Meena
outside the washroom and forced her to have sexual intercourse. He undressed her forcefully
and tried kissing her. She ran away and stated this to her mother. An FIR was lodged against
Mayur. The trial commenced and the court summoned Meena for witness and evidence
recording. Meena was mentally tortured and was in a state of depression. Her medical officer
in-charge recommended recording the evidence after 15 days, till the time Meena probably
can overcome this trauma. The court took note of this fact and took evidence after a month. If
Mayur gets convicted for the alleged offence under POCSO, he can contend appeal on what
grounds, if any. Decide –

(a) Mayur cannot go for appeal for the conviction order of lower court on any ground as the
lower court was correct in its decision and there were no procedural irregularities in this case.

(b) Mayur can go for appeal in higher courts against the conviction order as the lower court
was incorrect in its decision. The evidence recorded from the child has to be within time
frame of 30 days and not beyond that.

(c) Mayur cannot go for appeal for his conviction against the order of lower court as he had
done wrong and committed grave offence on Meena. (d) Mayur can go for appeal for his
conviction against the order of lower court on the ground that lower court was lethargic in
adhering to the set procedure given under section 35 of POCSO and section 164 of Cr.P.C
 

. Shrikant and Suchi were married couple and had an adopted daughter Dhriti. She was in 1st
year of her law college and Suchi was a good caring mother. Shrikant was not a man of noble
character. One day, he finds an opportunity and saw Dhriti alone at home. He wants to have
sexual relations with her. He forced her to have an intercourse with him and later forced her
not to speak up about this to her mother Suchi. Dhriti was much afraid and went to
depression. Suchi somehow gets to know about this and she filed an FIR against Shrikant.
The investigating officers arrested him and started their investigation. Suchi made Dhriti
believe that she would be helping her to overcome this trauma. But Dhriti refused to give any
evidence to the police officials. The trial commenced and the court summoned Dhriti for
witness. Dhriti told about her torture and in the final verdict, the court punished Shrikant with
imprisonment for 4 years. Decide –

(a) Statement of Dhriti would be considered to be recording of evidence under section 35 of


POCSO as the case falls within the provisions of POCSO and not Cr.P.C.

(b) Statement of Dhriti would be considered to be recording of confession under section 164
of Cr.P.C as the case falls within the provisions of Cr.P.C and not POCSO.

(c) Statement of Dhriti would not be considered to be either recording of evidence under
section 35 of POCSO or section 164 of Cr.P.C even though the case falls within the
provisions of POCSO.

(d) None of the above.

. Rahul and Disha were good friends from their school time. They were in love with each
other and had been living in live-in relationship from last 3 years. Disha asked Rahul to get
married to her and they will live a happy life. Rahul denied to this proposal of Disha. One
day, she checked his cell phone and found that Rahul was having an affair with Neha, his
colleague. Disha planned to kill Rahul and answer his betrayal. She planned with her brother
Pratim to murder Rahul. She helped Pratim in procuring drugs which he administered to
Rahul with drinks and soon he stabbed him with a sharp knife. Rahul’s mother filed a suit and
after investigation, Disha and Pratim were arrested as suspects. In a trial for murder charges,
Disha admitted that she helped her brother Pratim in procuring drugs but had not helped to
administer them to Rahul. Decide –

(a) The statement of Disha would be considered to be recording of evidence under section 35
of POCSO as the case falls within the provisions of POCSO and not Cr.P.C.

(b) The statement of Disha would not be considered to be either recording of evidence under
section 35 of POCSO or section 164 of Cr.P.C even though the case falls within the
provisions of POCSO.

(c) In any case, the special legislation will always prevail over the general legislation and
hence, the POCSO provisions will be applicable. The statement is covered under section 35
of POCSO Act.
(d) The statement of Disha would be considered to be recording of confession under section
164 of Cr.P.C as the case falls within the provisions of Cr.P.C and not POCSO.

. Choose the most appropriate option.

Assertion (A): Section 35 of POCSO and Section 164 of Cr.P.C cannot be equated and are
not on the same plane when it comes to procedure and nature of statements made by the
victim.

Reason (R): A point of difference between POCSO and Cr.P.C is that POCSO is a
substantive law with set procedure whereas Cr.P.C is only a procedural law which creates
Criminal Machinery function in India.

(a) Both A and R is correct and R is the correct explanation for A.

(b) Both A and R is incorrect. (c) Only A is incorrect but R is correct.

(d) Both A and R is correct but R is not the correct explanation of A

. In light of the facts of the case discussed in the passage, what intention the POCSO Act was
enacted with?

(a) The intention of the POCSO act is to protect the vulnerable minor girls from grave
offences of sexual assault and sexual harassment.

(b) The intention of the POCSO act is to protect the rights of minor victims. It has much
wider ambit than the Indian Penal Code as it is gender neutral.

(c) The intention of the POCSO act is unambiguous and not clear which made the judicial
rulings to interpret it in more liberal ways.

(d) The intention of the POCSO act is protecting the children from grave offences of sexual
assault and sexual harassment.

Passage (Q.-Q.): Respondent 1 filed a suit for declaration and permanent prohibitory
injunction against the appellants alleging that the Will in question executed by the mother of
appellant 1, respondents and grandmother of the appellants 2 to 4 in respect of properties
owned and possessed by her in favour of the appellants 2 to 4 is illegal and wrong. The Court
thus observed that the law of Will, one of the ways to dispose property, as prescribed in
Chapter-III, Rule 1 of the Mohammedan Law makes it clear that a Will may be executed in
writing or oral, showing a clear intention to bequeath the property. However, a limitation is
also prescribed that a valid Will by a Mohammedan will not be for more than 1/3rd of the
surplus of his/her estate and that to a non-heir. The Court relied on judgment Miyana Hasan
Abdulla v. State of Gujarat, wherein it was observed that on perusal of Section 67 of the
Evidence Act, 18 it is amply clear that where the document is written by one person and
signed by another, the handwriting of the former and the signature of the later have both to be
proved in view of Section 67 of the Evidence Act. What Section 67 of the Evidence Act
refers to, is the signature of a witness who counter signs a document as a person who was
present at the time when the document was signed by another person. The Court observed
that no exception can be taken to the findings recorded by the learned first Appellate Court,
whereby it drew an adverse inference against the appellants for not examining the scribe of
the document Shri Shamshad Ahmed Qureshi, who was very much alive at that time and
even, in case, he was suffering from ailment, his statement could have conveniently been
recorded on commission.

. Wahida, a Muslim girl, gets her Nikaah with Rehman, a Muslim boy. They both were living
in a joint family. Rehman’s grandmother was not in good terms with his father and trust
Rehman more than anyone else in the family. She was suffering from venereal disease and
had decided to transfer all her ancestral property to Wahida. She signed a Will deed and
asked Rehman to testify the same. Rehman was in doubt as he wanted the property. He
signed the Will deed at that time. When his grandmother died, he planned to get the Will
transferred in his name. He approached the court and was asked to prove that the Will is a
valid one. On what grounds Will can be invalidated, if any. Decide –

(a) The Will by Rehman’s grandmother can be proved to be a valid Will as he has to only
prove his signature as he was the only to testify the same.

(b) The Will by Rehman’s grandmother cannot be proved to be a valid Will as his
grandmother had died and he cannot prove her handwriting in the court.

(c) The Will by Rehman’s grandmother can be proved to be an invalid one as it was against
the principles of Mohammedan Law on Wills.

(d) The Will by Rehman’s grandmother cannot be prove to be invalid as Will was valid as per
Mohammedan Law and as per Evidence Act, Rehman can prove his signatures as he did
while testifying the will

. Zara, a Muslim girl, was taken in adoption by her parents. Her family was not much
attached to her as she was not a biological child. She was treated very indifferently in her
family. She was only being loved by her parents and her grandmother. She was very close to
her and was like best friends together. Her grandmother died due to silent heart attack. In a
family dispute as to share of property, Zara’s mother claimed that Zara too will have a share
in the property and since she was much more closer to her grandmother, will have an extra
share in the same. No one agreed to this proposal. She contended that it is implied from the
grandmother’s Will that she had given an extra share of property to Zara. The Will states
“Whenever I die, share the property left behind in portion of 1/6th among you all and give
that extra share to my beloved one”. Decide –

(a) Zara is entitled to an extra share in her grandmother’s property as it was explicitly and
clearly written in the Will to give that extra part to Zara.

(b) Zara is not entitled to an extra share in her grandmother’s property as the Will was invalid
under Mohammedan Law and has no effect as to the share of property.
(c) Zara is entitled to an extra share on her grandmother’s property as she was very close to
her and was the most beloved. The Will implied it for Zara only.

(d) None of the above.

. Shajida was married to Farhan. They had love marriage and Farhan’s family was much of
orthodoxical view. They do not approve such marriage and asked them to live apart from the
family. Shajida took Farhan to her family and started living with them. Farhan was much
happier in her family. They lived a good life. Shajida’s mother transferred all her property to
her husband as she was the only child to her parents. The Will states “As soon as I turn 60 in
my age, my all property is deemed to be transferred to Farhan, my son-in-law”. Shajida’s
mother turned 60 in year 2020 and Farhan approached the court in January 2021 to execute
the Will. Decide –

(a) Farhan can execute the Will deed as it was a valid Will and he had to prove the
handwriting of Shajida’s mother and the signature of the testator.

(b) Farhan cannot execute the Will deed as it was a invalid Will. He can prove the signatures
of the testator but not the handwriting of Shajida’s mother.

(c) Farhan can execute the Will deed as it was a valid Will and it can also be implied that
Shajida was the only daughter to whom her mother would transfer the property.

(d) Farhan cannot execute the Will deed as it was a invalid Will as per the Mohammedan
Law. Only 1/3rd part of the property can be transferred through such Will.

. Aly, a businessman, had three sons Salman, Sharukh and Aamir. They were among rich
Muslim families. All his sons had their own empires and were not bothered about their father
Aly. Aly was very disappointed by their irresponsible behaviour for their parents and decided
to give all his property to NGO, Samridhi. Salman gets to know about this decision of Aly
and went to meet him. He asked him not to do so and influenced Aly to transfer 1/3rd of
estate to him. Aly refused to it. Salman forcefully took the signatures of Aly on Will deed and
transferred 1/3rd of his property in his name. Soon Aly too died. Later, Sharukh and Aamir
gets to know about the property issue and filed a suit against Salman. Salman presented a
fraud Will in the court to prove the transfer of property by Will of Aly. Decide –

(a) Salman will succeed as he had the valid Will and the transfer of property was also valid.
He need not prove the handwriting of Aly in the court as he is already dead.

(b) Salman will not succeed in this case as the Will was not valid as per the principles of
Mohammedan Law and thus, he had invalid transfer of property by Will deed.

(c) Salman will succeed in the case as he can prove the signatures of his father Aly and his
handwriting on the Will deed and hence, it was valid transfer of property.

(d) Salman will not succeed in the case as the Will deed was invalid under Evidence Act but
not as per the Mohammedan Law. Thus, invalid transfer of property
 

. Choose the most appropriate option.

Assertion (A): A Will, also called ‘testament’ is an implement which enables a person to
dispose of his own property to someone whom he wants to give after his death. A Will is a
legal declaration of transfer of property by a person after his death.

Reason (R): To make disposition of property in Islamic Law, a Muslim can dispose of his
property only by accessing his testamentary powers i.e. by making a Will.

(a) Both A and R is correct and R is the correct explanation for A.

(b) Both A and R is incorrect.

(c) Only A is correct but R is incorrect.

(d) Both A and R is correct but R is not the correct explanation of A.

. What are the essential(s) of Will under the Mohammedan Law? I. The person making the
Will should have the property in her/his name before transferring it to another person. II.
Person making the Will under Islamic Law can transfer less than 1/3rd of his/her
property/estate to anyone before he/she dies. III. Will under Islamic Law can be in any form
i.e written, oral or implied form. It cannot be negated if made by any other means or form.

(a) Both I and II

(b) Both II and III

(c) All of the above

(d) Only I

Passage (Q.-Q.): Trespass means the wrongful disturbance or interference in the possession
of land or goods of another person. A person who intentionally and without consent enters
another person’s property is a trespasser. In the case of trespass to land, the unlawful land
infringement must be direct, intentional and actionable in itself. The entry must be intentional
in the sense that the trespasser intended to go onto that particular land. The trespasser’s
intention to trespass is not at all necessary. Illustration: A parachutist’s entry into the land
accidentally blown by the wind is unintentional and there is no liability for trespass. Trespass
to land may be committed in three situations. In each case, the entry must be without
justification. The cases are: · In order to constitute a trespass, entry is essential. · Entry must
be without permission. · The land must be in possession of the plaintiff; it may be actual or
constructive. · Entry must be voluntary which means not against a person’s will or by force. ·
Entry must be intentional. If the defendant consciously enters a land that he believes is his
own but that turns out to be the plaintiff’s land, he is still liable for trespass. It is irrelevant
that the defendant made a reasonable mistake and was not negligent.
. Maninder was obsessed with aviation technology. For his 18th birthday, his father bought
him a state-of[1]the-art drone which was equipped with a camera and could be controlled by
a remote. He went to his balcony to test it out. As soon as he pressed the button, the drone
took off and started hovering in the air, moving forward. Maninder could see where the drone
was flying through a screen on his remote. But as soon as the drone disappeared from his
sight, Maninder lost control and did not know how to manoeuver the drone. He eventually
found the drone to be in his neighbour’s land. Can Maninder’s neighbour sue him for trespass
because his drone landed on his property?

(a) Yes, because the drone actually entered his property without any permission

(b) Yes, because Maninder knew that the drone would land on his neighbour’s property as he
saw the camera screen but he still didn’t do anything about it

(c) Both (a) and (b)

(d) No, because the entry was not intentional in nature

. A and B had adjacent farms, but they did not have a very evident boundary to delineate
where one’s land starts and the other’s land stops. One day, A was mowing his own land, but
he mistakenly crossed the boundary and mowed the land of his neighbor, B, believing it was
his own land. As a result, he mowed some of the grass on B’s land. B sued A for trespass.
Based on your reading of the passage, do you think the Court will rule in B’s favour?

(a) Yes, because A made a mistake in crossing the boundary and did not intend to mow the
grass on B’s farm

(b) No, because A made a mistake in crossing the boundary and did not intend to mow the
grass on B’s farm

(c) Yes, because even if A mistakenly crossed the boundary, he voluntarily and intentionally
mowed the grass on B’s land

(d) No, because A did not intend to enter B’s land and did not mow the grass voluntarily

. Tom and Jerry were best friends but occasionally they indulged in brute fights. In one such
fight, Tom caught hold of a Frisbee. He attached Jerry to the Frisbee with the help of a strong
rope and, flung it across the ground. The Frisbee landed in the house of Bruto. He did not like
any intruders on his property. Just as Jerry was trying to wriggle himself out of the rope once
the Frisbee landed, Brutus saw him. Can Brutus file case of trespass against Jerry?

(a) No, because the trespass was against Jerry’s will and occurred only because Tom flung
him across through a Frisbee

(b) No, because there is no trespass as there has been no “wrongful interference” with
Brutus’s property
(c) Yes, because the fight between Jerry and Tom was intentional and that is the reason Tom
flung Jerry across

(d) Both (a) and (b)

. Sunio was getting his bathroom refurbished. In order to do so, he had to get the entire
underground sewage system dug up so that fresh pipes could be laid down. The work was
scheduled to last several days. On the first day, the workers were merely able to dig the
ground and this left the pipes exposed. After the workers left for the night, one of the pipes
broke and started projecting sewage water. The stream of the water was such that it directly
landed in the adjacent land which was owned by Jian. Is Sunio liable for trespass?

(a) No, because he didn’t intend for the pipes to project sewage water on Jian’s land

(b) No, because Sunio did not enter Jian’s land without his permission to interfere with his
possession

(c) No, because it was the workers who caused the pipe to burst and they are the ones who are
liable

(d) None of the above

. Jack and Jill used to play in their neighbour’s garden every day from 9 am to 5pm. Their
neighbour was a sweet old man but he only gave them permission to play till 5pm because he
thought that too much play time would take away from their time to study. One day, Jack and
Jill were playing hide and seek. Till 4:55 p.m., Jack was unable to find Jill. As soon as his
watch showed 5 p.m., he announced that he was leaving. Meanwhile, Jill continued to hide in
the garden till 6 p.m. Can the neighbour sue Jill for trespass?

(a) Yes, because Jill intentionally continued to hide even though it went past the permitted
time

(b) Yes, because Jill voluntarily stayed back on her neighbour’s land without his permission

(c) Yes, because Jill unlawfully infringed on her neighbour’s right to enjoy his property

(d) All of the above

Passage (Q.-Q.): Amongst all the fundamental rights guaranteed by Part III of Indian
Constitution, Article 19(1)(a), i.e., freedom of speech and expression is the most sensitive one
and is prone to controversy. If exercised negligently, without abiding by the limitations
prescribed under Article 19(2), a person, on one hand can be booked for defamation and on
the other, hangs the probability of prosecution for “Criminal Contempt”. Article 129 of the
Constitution, makes the Supreme Court “a court of record” and confers power to punish for
contempt of itself. Similar powers have been vested in the High Court by the virtue of Article
215 which makes the High Court “a court of record”, implying that only Supreme Court and
High Courts are empowered to adjudicate criminal contempt proceedings. “Criminal
Contempt means the publication (whether by words, spoken or written, or by signs, or by
visible representations, or otherwise) of any matter or the doing of any other act whatsoever
which─ (i) scandalises or tends to scandalise, or lowers or tends to lower the authority of any
court; or (ii) prejudices, or interferes or tends to interfere with, the due course of any judicial
proceeding; or (iii) interferes or tends to interfere with, or obstructs or tends to obstruct, the
administration of justice in any other manner;” However, the Court can permit justification
by truth as a valid defence in any contempt proceedings if it satisfied that such a defence is in
the public interest and the request for invoking the defence is bona fide. Truth should
ordinarily be allowed as a defence unless the Court finds, that it is only a camouflage to
escape the consequences of the deliberate attempt of scandalising the Court. [Extracted with
revisions from ‘Criminal Contempt of Courts [What is contempt and what isn’t? Where falls
the line?]’ by Kamini Sharma published on scconlineblog
https://www.scconline.com/blog/post/2021/03/16/criminal-contempt-of-courts/]

. A comedian, Runal Ramra was notorious for making tweets on every contemporary issue. In
one of his tweets, he said that, “honour has left the building (Supreme Court) long back” and
“Supreme Court of the country is the most Supreme joke of the country”. Along with this, he
also posted a picture of the Supreme Court painted in saffron colour with the flag of the
ruling party, the BJP. Can the Supreme Court initiate contempt proceedings against him?

(a) No, because it does not prejudice or scandalise the image of the Supreme Court

(b) Yes, because it does prejudice and scandalise the image of the Supreme Court

(c) No, because it falls under the freedom of speech and expression enshrined under Article
19(1)

(d) None of the above

. Two communities, X and Y were engaged in a historical land dispute. The matter went to
Court and the trial was ongoing for over 20 years. There was a lot of tension surrounding the
dispute and emotions of both the communities were charged up. Right before the decision
was scheduled to come out, Cerus, a sympathiser belonging to community X announced, “If
the Supreme Court gives a decision in favour of community Y then it will lose all its
integrity”. Can Cerus’s announcement be categorised as contempt of court?

(a) Yes, because it interferes with the due course of the ongoing judicial proceedings

(b) Yes, because it may obstruct the administration of justice in the ongoing proceedings

(c) No, because it does not threaten the due course of the ongoing proceedings or the
administration of justice

(d) Both (a) and (b)

. P and Q were embroiled in a tough legal battle. The District Court gave a decision in favour
of P. This angered Q who had spent a lot of money and resources to fight this dispute. As a
result, Q went on social media and started passing a lot of maligning comments against the
Court. Is the Court empowered to initiate contempt proceedings against Q?

(a) Yes, the Court is empowered to initiate criminal contempt proceedings under Article
19(2) of the Constitution

(b) Yes, the Court is empowered to initiate criminal contempt proceedings under Article 129
of the Constitution

(c) Yes, the Court is empowered to initiate criminal contempt proceedings under Article 215
of the Constitution

(d) None of the above

. Faye was a famous journalist. She published an article in a leading newspaper condemning
the decision of the High Court. In her article, she revealed that the decision was biased
because the judge had taken a bribe from one of the parties. Her article created an uproar and
the High Court initiated contempt proceedings against Faye. When the trial began, Faye
furnished proof of the judge’s corruption. If the allegations against the judge are true, will
Faye be liable for contempt?

(a) No, because her article does not tarnish, scandalise or prejudice the image of the High
Court

(b) No, because her article was true which is a valid defence for any contempt proceeding

(c) Yes, because even if her article is true it is not in public interest

(d) Yes, because even if her article is true her request for invoking the defence is nor bone
fide

Passage (Q.-Q.): Mr. Balfour is the appellant in the present case. He used to live with his wife
in Ceylon, Sri Lanka. During his vacations in the year 15, they came to England. His wife
became ill and needed medical attention. She was advised by her doctor to stay in England.
They made an agreement that Mrs. Balfour was to remain behind in England when the
husband returned to Ceylon (Sri Lanka) and that Mr. Balfour would pay her £30 a month
until he returned. This understanding was made while their relationship was fine; however,
the relationship later soured. The parties subsequently divorced and an issue arose as to
whether agreement was enforceable and soon after that Mrs. Balfour sued him for alimony
equal to the amount her husband had agreed to send. The Court held that “the agreement
between the Balfours was not a legally enforceable contract but merely an ordinary domestic
arrangement. There was no intention to create legal relations and Mrs. Balfour could not sue
for the alleged breach of it.” The Court was of the view that mutual promises made in the
context of an ordinary domestic relationship between husband and wife do not usually give
rise to a legally binding contract because there is no intention that they be legally binding.
However, the Court did concede that there may be circumstances in which a legally binding
agreement between a husband and wife may arise. For a contract to be valid or enforceable,
the two necessary elements are valid consideration and an intention to create legal
obligations. [Extracted with revisions from ‘Case Analysis of Balfour vs. Balfour’ by Shelal
Lodhi Rajput published on lawlex.org https://lawlex.org/lex-bulletin/case-analysis-of-
balfour-vs-balfour-19-via-irac-method/1]

. A husband and wife filed a complaint against each other in a Court of law. They both agreed
to the arrangement that the husband will provide monthly maintenance to his wife and in
consideration, his wife will not take loans from anyone. Under English law, a husband is
considered to be liable for his wife’s debt from a third party. The husband didn’t want to take
this burden. Is there a valid contract in this case?

(a) No, because there can be no intention to create legal obligations between a husband and a
wife by virtue of their domestic relationship

(b) Yes, because there can be no intention to create legal obligations between a husband and a
wife by virtue of their domestic relationship

(c) Yes, because there was an intention to create legal obligations and there is a valid object
and consideration

(d) Both (b) and (c)

. X’s daughter Y was living in Washington with her son Z. Y was employed as an accountant
in Washington. X wanted Y and Z to live with her in England and that Y continues her
education therein. So, X entered into a contract with Y to pay for her expenses and education
in England and in return, Y agreed to come back to England and stay in her mother’s house
while she completed her education. Over time, Y dropped out of her course and also got
remarried. Her husband also moved in to X’s house and X continued to bear everyone’s
expenses. The relations between X and Y soured and X asked her to leave her house. Can Y
sue for enforcing the contract?

(a) No, because Y failed to fulfil her obligation to continue her education in return for X
paying her expenses and allowing her to live in her house

(b) No, because there was no intention to create legal obligations in this situation

(c) Yes, because it is irrelevant that Y dropped out of her course as she still came to England
in pursuance of the contract only

(d) Yes, because X cannot ask Y to leave the house since it was consideration in exchange for
Y completing her education in England

. Two Companies called Dunder Mifflin and Staples were dealing in the business of paper.
Dunder Mifflin was based in America and Staples was Canada-based. They both agreed in
explicit written terms in their contract that, in the event of any dispute, they would not be
bound by the jurisdiction of any Court. That is, they agreed that there would be no legal
consequences to anything. Subsequently, Staples breached the contract by failing to deliver
on time. Can Dunder Mifflin sue Staples for enforcing this contract?

(a) Yes, because Dunder Mifflin and Staples intended to create legal obligations while
entering into the contract

(b) Yes, because the contract between Dunder Mifflin and Staples is valid and enforceable

(c) Both (a) and (b)

(d) No, because the contract between Dunder Mifflin and Staples is not enforceable

. Mr and Mrs Khan jointly owned a house on a mortgage. Mr Khan left the house to live
independently because he wanted to become a ‘brahmachari’ and attain ‘sanyas’ or
renunciation. Consequently, Mr and Mrs Khan entered into a written agreement that Mrs
Khan would pay Rs. 10,000 as a monthly sum and once the full mortgage amount is
completed, Mr Khan would transfer the house in the name of Mrs Khan. The full mortgage
amount was paid however, Mr Khan refused to transfer the house. Can Mrs Khan sue Mr
Khan for breach of contract?

(a) No, because there was no intention to create legal obligations between Mr and Mrs Khan

(b) No, because contracts between domestic partners like a husband and a wife cannot be
valid or enforceable

(c) No, because transferring the house in the name of Mrs Khan for a monthly sum of Rs.
10,000 to clear the mortgage does not constitute valid consideration

(d) None of the above

36
Passage(Q.-Q.): As Indian courts were flooded with unresolved cases and the cases were
piling up in the court of law the requirement was felt for a process that was less costly,
speedy, and much flexible as compared regular judicial process. To fulfil these requirements
Alternate Dispute Resolution was introduced in India. Arbitration is a form of alternate
dispute resolution. Arbitration is a form of alternate dispute resolution in which the parties to
the dispute by mutual agreement or with the help of the court appoint a third party knows as
the arbitrator for resolution of the dispute. Arbitration is an adjudicatory process. Arbitration
is of two types’ Ad hoc arbitration and institutional arbitration. In Ad Hoc arbitration all the
aspects such as the number of arbitrators, process of appointing of those arbitrators,
procedural for conducting the arbitration, etc. are determined by the parties. Whereas in
institutional arbitration parties mutually agrees on a specialized institution to take on the role
of administering the arbitration process. An arbitrator is an individual third party that presides
over an arbitration proceeding. An arbitrator is like a private judge hired by the party or
sometimes appointed by the court at the request of a party to resolve a dispute through an
arbitration proceeding. According to Section 11 of the arbitration and Conciliation Act, any
person of any nationality can be an arbitrator unless otherwise agreed by the parties to an
arbitration agreement. If in case parties to an arbitration agreement on commencement of
arbitration are not able to mutually agree on an appointment of the arbitrator within 30 days
than any of the party to an arbitration agreement can approach the court and file an
application under Section 11 for the appointment of an arbitrator. Once the arbitrator is
appointed by the court the parties shall not appeal to such a decision. Though if any of the
parties to arbitration during the arbitration hearing finds that the arbitrator is partial, biased,
prejudiced against them, the procedure is not met with, etc. it can file a challenge to such
arbitrator.

. Rawl’s Legal firm and Nibble Content firm decided to administer the arbitration process
mutually at the office of Arbitrators to reduce the cost; both firms negotiated a complete set
of rules, procedures that fit their particular needs. But parties lack the necessary knowledge
and expertise which resulted in misinformed decisions and a lack of cooperation between the
parties therefore they agreed to appoint XYZ specialized institution with experienced
arbitrators to administer their arbitral process. Find out which process of arbitration is
followed by Rawl’s Legal firm and Nibble Content firm.

(a) Institutional arbitration is followed as the process of arbitration is completed in the office
of arbitrators.

(b) Institutional arbitration is followed as the parties were not involved during the process of
arbitration.

(c) Ad hoc arbitration is followed when parties determined the aspects in the arbitrator's
office and institutional arbitration is followed when XYZ institution is appointed.

(d) Ad hoc arbitration is not followed by the firms only institutional arbitration is followed as
in both the situations they appointed the experienced arbitrators.

. A French Biotech Company entered into a license and development agreement with a large
Indian Pharmaceutical company and the parties have not included in their contract a clause
stating the arbitrator to resolve the disputes which arise in the future and unfortunately,
dispute arise but parties were not mutually agreed to appoint the same arbitrator and 3 months
have passed in dispute. Decide.

(a) The Parties should appoint the arbitrator under the terms and conditions of an arbitration
agreement.

(b) Any party can file an application to the court for the appointment of the arbitrator within
the time.

(c) Parties have to mutually agree to avoid the delay of the arbitration process.

(d) Parties can again draft an arbitration agreement to decide the arbitrator.

. Arbitration agreement of company read as “In the event that employer and employee are
unable to resolve the dispute, and should either desire to pursue a claim against the other
party, both employer and employee agree to have the dispute resolved by final and Binding
Arbitration. The employer and employee agree that the arbitrator shall be from the county
where the Employee recently worked for an employer” The dispute arises between employer
A and the employee B on 1st march 2021 and B recently worked in Pakistan for the A and
currently he works in Sweden for A. Employer decided to appoint an arbitrator from Pakistan
but it was refused by the B on 3rd March 2021 because he considers Pakistan a biased
country which will not do justice to his case. Decide.

(a) According to Section 11 of the arbitration and Conciliation Act, any person of any
nationality can be an arbitrator. Hence the Arbitrator should be decided mutually by A and B.

(b) Arbitrator is partial, biased, prejudiced against B therefore he can file a challenge to such
arbitrator.

(c) B can approach the court and file an application under Section 11 for the appointment of
an arbitrator.

(d) Arbitrator from Pakistan should be appointed as it was agreed by the parties to an
arbitration agreement.

. UUC and XXZ companies file an application under Section 11 for the appointment of an
arbitrator and the arbitrator Mr. Mudit is appointed by the court which is a former employee
of the UUC and is not Disqualified to act as an arbitrator but during the arbitration
proceedings, it was noticed that Mudit was biased towards XXZ as he has past grudges with
UUC company. Decide.

(a) Mudit had a business relationship with the parties therefore he is disqualified by the law to
act as arbitrator.

(b) Parties can file a challenge to the arbitrator if they found him biased towards any one of
them.

(c) Once the arbitrator is appointed by the court the parties shall not appeal to such a decision.

(d) The present serving employee is stand disqualified to act as an arbitrator therefore Mudit
can’t act as the arbitrator.

. What is the reason for the increasing demand for Alternate dispute resolution in commercial
cases?

(a) It resolves the dispute cost-effectively, less time-consuming, and produces the desired
result.

(b) There is no right of appeal even if the arbitrator makes a mistake of fact or law, therefore,
benefitting the one party.

(c) Lack of legal expertise makes the process informal and more effective.

(d) There are no precedents made or followed this makes it easy to decide an issue.

 
Passage(Q.-Q.): Elections bring the Election Commission of India (ECI) into sharp focus as
this constitutional body superintends, directs and controls the conduct of elections. It is the
constitutional duty of the ECI to ensure that the elections held are free and fair. Article 324
contains plenary powers to ensure free and fair elections and these are vested in the ECI
which can take all necessary steps to achieve this constitutional object. The model code of
conduct issued by the ECI is a set of guidelines meant for political parties, candidates and
governments to adhere to during an election. This code is based on consensus among political
parties. Election Symbols (Reservation and Allotment) Order, 18 says that the commission
may suspend or withdraw recognition of a recognised political party if it refuses to observe
the model code of conduct. The Constitution has clothed the ECI with enough powers to do
that. Thus, the code has been issued in exercise of its powers under Article 324. The model
code is observed by all stakeholders for fear of action by the ECI. Further, Article 324 does
not confer untrammelled powers on the ECI to do anything in connection with the elections.
If transfer of officials is a power which the ECI can exercise without the concurrence of the
State governments, the whole State administration could come to a grinding halt. The ECI
may transfer even the Chief Secretary or the head of the police force in the State abruptly. In
Mohinder Singh Gill’s case (supra), the Court had made it abundantly clear that the ECI can
draw power from Article 324 only when no law exists which governs a particular matter. It
means that the ECI is bound to act in accordance with the law in force. Transfer of officials,
etc. is governed by rules made under Article 309 of the Constitution which cannot be
bypassed by the ECI under the purported exercise of power conferred by Article 324.

. The media plays a major role in keeping the citizenry abreast of current events and raising
awareness of various issues in society. In India, during parliamentary elections, only a few
reputed party candidates were allowed to debate with each other on national television and
communicate their message to the electorate. The election commission of India doesn’t
intervene in media plays. Decide this is a fair election?

(a) Media are free to act independently and have the right to prompt a particular political
party. Hence this is fair elections.

(b) This is not fair elections as media should be an open forum for debate and discussion to
act as public educators. The election commission should ensure its duty.

(c) This is a fair election as the public is fully able to participate and is unhindered in
exercising choice.

(d) This is not a fair election as paid political advertising is a violation of free and fair
elections in Indian democracy.

. Using its Powers under Article 324, the Election Commission of India had prohibited
newspapers and news channels from publishing results of pre-election surveys and exit polls
during a certain period during Lok Sabha elections and Assembly elections in a few states.
However, the ban was challenged in the Supreme Court. Decide the validity of the election
commission decision.
(a) The Prohibition is justified as because the exit polls are always motivated and sponsored
by rivals of political parties and has a distorting effect on the choices voters make in the
protracted election.

(b) Prohibition is not justified as it violates the fundamental right of free speech and
expression of print and electronic media.

(c) Prohibition is justified as pre-election surveys and exit polls can be controversial if the
agency conducting them is perceived to be biased and it is the constitutional duty of ECI to
ensure fair elections.

(d) The prohibition is not justified because the election commission cannot enforce such
guidelines.

. The Model code of conduct is issued by the election commission in Lok Sabha elections, the
MCC issued guidelines like on polling day, all party candidates should cooperate with the
poll duty officials at the voting booths for an orderly voting process and before using
loudspeakers during their poll campaigning, candidates and political parties must obtain
permission or license from the local authorities. During the election, the CRPF deployed at
booth no 7 Mumbai claims that BB party workers are attempting to take control of EVM and
are rigging the booth. What actions can be taken by ECI?

(a) The model code is observed by all stakeholders as the fear of action by the ECI which is
not justified in democratic elections.

(b) Model code of conduct issued by the ECI is a set of guidelines meant for political parties,
candidates, and governments to which can be escapable.

(c) The State commission can suspend the recognition of the BB political party.

(d) The Election Commission can suspend the recognition of the BB political party.

. Before the East Bengal election the election commission had shaken government machinery
in the East Bengal by suspending senior officials and pulling out officials from all election
duty to seeking immediate disciplinary action against some officers as no law exists which
governs this matter. The reason cited for EC action is a dereliction of duty by officials.
Decide the legal enforceability of ECI actions in East Bengal.

(a) ECI can draw power from Article 324 to suspend and take actions against the state
officials.

(b) Transfer of officials, etc. is governed by rules made under Article 309 of the Constitution
which cannot be bypassed by the ECI under the purported exercise of the power conferred by
Article 324.
(c) Article 324 does not confer untrammelled powers on the ECI to do anything in connection
with the elections.

(d) Both B and C  

Passage(Q.-Q.): Perusal of Section 361 of the IPC reveals that, in addition to defining the
minority of the girl, there must be an act of tempting or taking and care/keep of a legal
guardian (being sixteen for boys and eighteen for girls) and care/keep of a legal guardian.
Such ‘enticement’ does not need to be overt or instant in time and can even be by indirect
acts such as winning over a minor girl’s love. Nevertheless, the simple retrieval of a lost
minor from a stranger’s care does not ipso-facto establish the crime of abduction. It allows
his/her guardians the power to make important decisions about a minor’s physical welfare.
Hence, the infatuation of a minor girl with her supposed kidnapper should not be permitted as
a security of its own, for the same would amount to unexpectedly dismantling the defensive
nature of the kidnapping crime. Similarly, IPC Section 366 postulates that if the prosecutor
leads evidence to establish that the abduction was with the intention/knowledge of the girl’s
compel marriage or to force/induce her to have sexual intercourse, the 10-year additional
sentence as provided for there under will be attracted. The cited judgment, therefore, cannot
be of any assistance without establishing: first, knowledge and capacity with the minor of her
actions; second, voluntary abandonment on part of the minor; and third, lack of inducement
by the accused.

. Simran, a sixteen year girl and studying in the first year of law, reveals her intention to
marry her neighbor, Raj (aged 22 years) when caught having a telephonic conversation with
him by her father, Mr. Gabbar. In response, Mr. Gabbar sends her to her aunt’s house in
Kanyakumari. At around 10:00am, she left the house and telephoned Raj, to meet her at a
particular road in that area and, she walked to that place. They met at the rendezvous and
Simran got into the car. Next day, they later registered their marriage at the Registrar’s office.
Mr. Gabbar infuriated with the news filed the case of Kidnapping against Raj. Premising on
the passage and the facts presented, decide whether Raj is guilty for kidnapping Simran?

(a) Raj is guilty of kidnapping Simran as she is a minor.

(b) Raj is not guilty of kidnapping as there was consensual affair.

(c) Raj is not guilty of kidnapping as the element of ‘taking’ from lawful guardianship is
missing.
(d) Raj is not guilty of kidnapping as Simran’s aunt is not her lawful guardian and hence, she
was not taken from the lawful guardian.

. Shiva, the neighbor of Mr. Gabbar deposed during the proceedings that he communicated
the message of Raj to Simran to meet at the rendezvous a day before she eloped and about his
promised to marry later. Depending upon the developments in the case, what would be the
effect on the culpability of Raj?

(a) Raj is guilty of kidnapping as Simran was enticed from her lawful guardian.

(b) Raj is guilty of kidnapping as he was delivered the message to meet at the rendezvous.

(c) Raj is not guilty of kidnapping as there was no assistance in leaving the house offered by
him to Simran and she voluntarily abandoned the custody.

(d) Both a and b

. What are the mitigating factors that could aid the case of Raj?

(a) It was a consensual affair.

(b) He never intended Simran to abandon the custody of her parents and had no intention of
marrying her.

(c) Simran is an educated, matured and capable adult.

(d) Both b and c.  

. If Raj after a few days drops Simran back to Mr. Gabbar, what will be the offence he is
liable for?

(a) He is not liable for any offence as he returned Simran back to the lawful guardian.

(b) He is not liable for any offence as there was consensual affair and she abandoned the
custody on her own accord.

(c) He is liable for the offence of kidnapping as the offence is complete as soon as Simran
abandons the guardianship of her parents.

(d) Both a and b

Passage(Q.-Q.): We are a democratic republic. It is not open to the Government of the day to
arbitrarily give away hills and hillocks for exploitation. Merely because the process of
issuance of mining lease was conducted in consonance with the statutory procedure, that
would not confer any immunity against judicial scrutiny. Unless there are supervening public
interest considerations, hills and hillocks cannot be given away for mining. While a hill can
be allowed to be exploited under certain circumstances, "post-exploitation, the hill must still
remain.” The ways of Nature are inscrutable. Only after tsunami struck us, we were told that
mangrove forests act as natural barriers against such oceanic onslaughts. Villages in a
particular District never faced storms because a hill range acted as protection wall. (Extracted
with requisite revisions and edits from ‘Judicial scrutiny not excluded even if mining lease
granted as per law; Govt can't arbitrarily give away hill for exploitation: Madras High Court,
Bar and Bench https://www.barandbench.com/news/litigation/hills-and-hillocks-exploitation-
mining-lease-madras-high[1]court-mining-thiruppani-rock). . Tender was floated for mining
and quarrying activities to be carried out on 30-meter hillock in a Thiruppani hillock at
Rajapalayam by the government. It was a dry land. Mr. Mittal & Co won the bid and all the
statutory requirements envisaged in Tamil Nadu Minor Mineral Concession Rules, 19 have
been complied with. There is an approved mining plan. The District Level Environment
Impact Assessment Authority has granted clearance on 01.05.2021. This land was rich in blue
metal which is part of construction material and is being extracted by using explosives by the
Company. A suit was brought against this lease. Decide the fate of the suit.

(a) The suit shall fail as all the legal compliances have been adhered to by the Company.

(b) The suit shall fail as the land was already barren and belonged to the government.

(c) The suit shall fail as construction presumes a public interest.

(d) The suit shall succeed as irreparable harm will be caused to the hillock.

. During the proceedings, it was brought to light that the land was leased earlier to a Women
self-help group and hence the present suit is the outcome of malice by the rival companies.
Decide the outcome of the suit.

(a) The suit shall succeed as the Hillock will suffer irreparable exploitation because of
explosives.

(b) The suit shall fail as it is a result of malicious prosecution by the rivals of the leaseholder.

(c) The suit shall succeed as the Hillock is a rich Heritage for the nation.

(d) The suit shall fail as it was earlier leased to a women self help group.

. Before this lease, the majority of the blue metal was exported from Africa. And with this
project the government aims to use this mineral for construction of houses in the village.
Decide which of the following is true:

(a) Construction of houses involves general public interest and hence lease will be valid.

(b) The exploitation of the hillock is beyond restoration and hence the lease is invalid.

(c) The owner of the land has the authority to appropriate the land as s/he deems fit as we are
a democratic republic.
(d) Exploitation is always contingent on inter-generational equity.  

. The hillock was barren, so the Company decided that after extracting the mineral, it shall
plant fruit[1]bearing trees in acres of land and provides a contingent promise of
compensation in case of any post-exploitation effects. Which of the following statements is
true?

(a) Post-exploitation of the hillock also the hill will remain and will not be completely
destroyed.

(b) Planting in acres of land is an insufficient area to restore and compensate the loss.

(c) Inter-generational Equity shall be maintained by plantation of fruit bearing trees.

(d) Hillock despite barren will suffer lasting damages.

Passage(Q.-Q.):Reproductive rights of individuals and couples in India can be located in laws


and polices relating to heath, employment, education, provision of food and nutrition, and
protection from gender-based violence. These fundamental rights are guaranteed under Part
III of the Constitution of India, but the right to health or reproductive rights are not expressly
recognised as fundamental rights. On many occasions our Hon'ble Supreme Court has
interpreted the right to health as an integral part to the right to life. In Suchita Srivastava and
Another vs Chandigarh Administration, the Court held that reproductive autonomy is a
dimension of personal liberty as guaranteed under Article-21. It is important to recognise that
reproductive choices can be exercised to procreate as well as to abstain from procreating. The
crucial consideration is that a woman's right to privacy, dignity and bodily integrity should be
respected. Furthermore, women are also free to choose birth-control methods such as
undergoing sterilisation procedures. Taken to their logical conclusion, reproductive rights
include a woman's entitlement to carry a pregnancy to its full term, to give birth and to
subsequently raise children. Several provisions under Part IV of the Constitution are also
related to issues of health, which are not enforceable in any court, but create an obligation on
the State to apply these principles in making laws and policies, such as Article- 39(e), 39(f),
42, 45 and 47. (Extracted with requisite revisions and edits from ‘Women's Reproductive
Rights In India And Beyond’, https://www.livelaw.in/columns/rights-of-women-world-
health-organisation-who-14)

. Mrs. Kaushik, a woman in her late 40s got pregnant for the first time after 12 years of
marriage. However, owing to her age she decides to abort the child in the second week of
pregnancy. Mr. Jeetendra, her husband taken aback with this decision protests but to no avail.
Unfortunately she proceeds with the termination of the pregnancy. Mr. Jeetendra now brings
an action against her. Which of the following is true?

(a) Abortion of a child is a criminal offence and Mrs. Kaushik shall be punished accordingly.

(b) Mrs. Kaushik has the Right to procreate which does not include the right to terminate one.
(c) Mrs. Kaushik has the autonomy to decide the fate of her pregnancy.

(d) Reproductive rights are enshrined solely under the Directive Principles of State Policy
and hence Mrs. Kaushik is not protected.

. Assume Mrs. Kaushik entered into a contract at the time of her marriage not to abort her
pregnancy under any circumstances and thus partially waiving her reproductive rights
otherwise entitling her to damages. On termination of pregnancy, a suit for damages is
brought against her. Decide the fate of the suit.

(a) There is breach of Contract and hence Mr. Kaushik is eligible for damages.

(b) Mrs. Kaushik has the right to reproductive choices and no damages shall be awarded.

(c) Partial damages shall be allowed as there is breach of Contract which is a Central
Legislation which supersedes Directive Principles.

(d) Termination of pregnancy is not a facet of reproductive autonomy.  

. Mrs. Kaushik during her youth and before marriage had previously got pregnant with the
child of Mr. Arora. She decides to keep the pregnancy. However, Mr. Arora fearing his
reputation and the backlash he might receive from his family persuades her to terminate the
pregnancy which she denies. Which of the following is true?

(a) Mrs. Kaushik has reproductive autonomy which is protected under the Constitution.

(b) Mr. Arora has equal say in the retaining or termination of the pregnancy.

(c) Mrs. Kaushik’s right to personal liberty is guaranteed under the Directive Principles of
State Policy.

(d) Decision by the Supreme Court will have overriding effect on all contradictory decisions
by the lower courts.

. Mr. Arora infuriated by the refusal of Mrs. Kaushik to terminate her pregnancy intentionally
administers certain medications resulting in the termination of pregnancy. On being caught he
cites the Doctor’s report which had provided for the ill effects if the pregnancy is continued.
Based on the passage, which of the following statements is true?

(a) Mr. Arora shall be liable for culpable homicide not amounting to murder.

(b) Mr. Arora’s act was to further the protection of Mrs. Kaushik’s health and reputation, thus
is not ill intended.

(c) The Doctor’s report is insufficient and inconclusive evidence.

(d) None of the above.


 

Passage(Q.-Q.): According to the data provided by the National Crimes Record Bureau
(NCRB), an average of 52 cases were registered under section 304B, IPC nation-wide
between 2016 and 2019. Section 304B provides that when a woman dies due to burns or
bodily injury or of unnatural circumstances, within seven years of her marriage and it is
shown that soon before her death she was subjected to cruelty or harassment by her husband
or his relatives in connection with any demand for dowry, such death shall be called ‘dowry
death’. The husband and his relative/s shall be presumed to have caused her death. The Bench
has based its judgment on the premise that the phrase ‘soon before her death’ has proved to
be contentious and aided acquittals. The SC chides trial courts for construing ‘soon before’ to
mean ‘immediately before’ the death of the woman, an interpretation which is often difficult
to prove for the prosecution. The SC goes on to say that the term ‘soon before’ should be
determined by establishing a ‘proximate and live link’ between the cruelty and the
consequential death of the victim. The demand of dowry, cruelty or harassment based upon
such demand and the date of death should not be too remote in time which, under the
circumstances, be treated as having become stale.

. A young woman, Ms. Preeti was married to Mr. Kabir Singh sometime in the year 2017.
However the family was not supportive of the marriage and hurled abuses and mistreated
Preeti. In January, 2021, within four years of the marriage, Preeti consumed Aluminium
Phosphide, which is a pesticide, as a result of which her young life was snuffed out. On the
same day, an FIR was lodged against the husband, and the family members. Which of the
following statements are true?

(a) Husband and the relatives shall be presumed to have caused her death under Section
304B.

(b) The death of Ms. Preeti establishes the proximate and live link test.

(c) The death of Ms. Preeti is not hit by Section 304B of the CrPC.

(d) The criteria of ‘soon before’ are established amply in the factual matrix.  

. In the first year of marriage, demands of money were made by the family knowing the
financial instability of Ms. Preeti’s family. After 3 years, a vehicle was given as peace
offering but the mistreatment didn’t cease with repeated demands of materialistic
requirements ultimately leading to Preeti’s suicide. Which of the following statements are
true?

(a) Demand of dowry and the suicide are remote and have become stale.

(b) Death of Ms. Preeti is not caused by burns or is under unnatural circumstances and hence
Section 304B shall not be attracted.

(c) Husband and the relatives shall be presumed to have caused her death.

(d) The principle of innocent unless proven guilty shall be applicable refuting the
presumption in Section 304B.
 

. While Ms. Preeti was mistreated, Mr. Kabir Singh was merely a silent spectator and didn’t
protest against the act. Moreover, the demand of dowry was only made by the family
members and not by Mr. Kabir Singh. After her suicide, FIR was to be lodged. Mr. Singh
pleaded his non-involvement in the mistreatment and hence not to be addressed as a co-
accused in the case. Can he do so?

(a) Yes, there was no involvement of Mr. Kabir Singh in mistreating Ms. Preeti.

(b) No, there exists a presumption against the Husband and the relatives.

(c) Yes, the demand of dowry was never raised by Mr. Kabir Singh.

(d) No, silence will be equivalent to the commission of the act in this scenario.

. Assume Ms. Preeti tormented by the continuous mistreatments and demands for dowry is
diagnosed with depression and leaves her in-laws and begins living at her parents’ house.
Meanwhile the in-laws continuously insist on certain payments or to continue her stay at her
parents’. After almost 4 months, she commits suicide at her parents’ house. Hence, the police
refused to take any action against the in[1]laws and her husband under Section 304B. Is the
act of the police justifiable?

(a) Yes, because cruelty and her date of death are remote and do not fulfill the proximate and
live link.

(b) Yes, the death was independent of dowry demands but a result of mental breakdown.

(c) No, the police are required to register an FIR and investigate in cases involving
cognizable offences.

(d) No, the death satisfies the ‘proximate and live link’ test.

Passage(Q.-Q.):India, which is facing shortages of vaccines and medicines, can do better if


the government decides to invoke two provisions of the country’s Patents Act, which
Parliament included to address public health emergencies. While amending India’s Patents
Act after the country joined WTO, lawmakers had included important instruments such as
special compulsory licences (Section ) and government use of inventions (Section ). These
amendments were introduced by the National Democratic Alliance (NDA) government in
2001. Section allows the Centre to issue compulsory licences any time after the grant of a
patent “in circumstances of national emergency or in circumstances of extreme urgency”. The
Controller of Patents is authorised to grant a licence to any person interested in producing the
patented product and on such terms and conditions as the Controller deems fit. The provision
of Section can be applied in case of “public health crises…or other epidemics”. No one,
including India’s lawmakers, could anticipate a pandemic such as Covid-19; the worst case
scenario for invoking this Section, according to them, was, therefore, “epidemics”. Section
enables the Centre, or any person authorised by it, to use the invention “at any time after an
application for a patent has been filed at the patent office or a patent has been granted”. In
other words, if a vaccine or medicine has been granted a patent in India, or an application for
the grant of a patent in India has been made, the government can authorise anyone to produce
such a product in the country. While introducing the Patent Amendment Bill in the
Parliament, the then commerce and industry minister, Murasoli Maran, had explained that the
government can use this provision “only in the interest of public health system. They can
procure [patented medicines] and sell [them] to hospitals or they can give it to third parties”
for producing in the country.

. The central government granted compulsory licenses to more pharma companies for the
manufacture of dicloxacillin, erythromycin, and tetracycline as they are in high demand in the
international market for enhancing the beauty of skin and granted licenses for Oral antifungal
drugs include fluconazole and itraconazole for tackling future outcome of black fungus.
Decide the validity of government under Section of India’s Patents Act.

(a) This is valid because the Controller of Patents is authorized to grant a license to any
person interested in producing the patented product and on such terms and conditions as the
Controller deems fit.

(b) This is invalid under Section of the India Patent Act.

(c) This is valid as the black fungus is epidemic and the provision of Section can be applied
in case of public health crises and epidemics.

(d) The government decision is valid to tackle the problem of black fungus.

. Pandemic calls for the suspension of IP rights to keep prices low and to address supply
shortages, particularly in low and middle-income countries. To increase the supply of
vaccines in India the government of India authorized the Health institute of India Pune Under
Section to manufacture NatraZaneca the US invented vaccine. Can Government authorize to
manufacture NatraZaneca in India?

(a) Yes, Section enables the Centre to authorize anyone to produce such a product in the
country.

(b) Yes, Section enables the Centre to authorize anyone to produce such a product in the
country only in the interest of the public health system.

(c) No, Government cannot authorize the Health institute of India Pune Under Section to
manufacture NatraZaneca because of patent issues.
(d) No, Government cannot authorize the Health institute of India Pune Under Section to
manufacture NatraZaneca as a patent has been granted.

. To increase the supply of covid medicines in India the government of India authorized the
Health institute of India Pune Under Section to manufacture Imemdesivir injection an Italy
develop injection and application for Imemdesivir patent rights was filled in the patent office.
Can Government authorize to manufacture of Imemdesivir in India?

(a) Yes, Section enables the Centre to authorize anyone to produce such a product in the
country only in the interest of the public health system.

(b) Yes, Section enables the Centre to authorize anyone to produce such a product in the
country.

(c) No, Government cannot authorize the Health institute of India Pune Under Section to
manufacture Imemdesivir as no patent has been granted.

(d) No, Government cannot authorize the Health institute of India Pune Under Section to
manufacture Imemdesivir.

. Covid 19 outbreak drastically vanishing lives of people worldwide and shortage of medical
essentials is deteriorating the situation in India, Pharma companies handed over a list to the
government to tackle drug security concerns, Highlighted that India may face a shortage of
antibiotics such as Azithromycin, Ofloxacin, and vitamin tablets and capsules such as B12
and E which is manufactured in the international market and essential for dealing current
health emergency. What course of action government can follow to avoid the shortage of
above mention Drugs?

(a) Government can authorize anyone to produce such essentials covid drugs in the country.

(b) Amending India’s Patents Act will help in increasing manufacturing nationally.

(c) India, which is facing shortages of vaccines and medicines, can do better if the
government decides to invoke two provisions of the country’s Patents Act.

(d) India, which is facing shortages of vaccines and medicines, can do better if the
government decides to invoke two provisions of the country’s Patents Act which is section
and .  

Passage(Q.-Q.):As per the Indian law, the consumption or possession of marijuana is a


criminal act and those found guilty can face a jail term up to 10 years or a fine of Rs. 1 lakh
or both. India’s approach towards Narcotic Drugs and Psychotropic Substances is enshrined
in Article 47 of the Constitution of India which mandates that the ‘State shall endeavour to
bring about prohibition of the consumption except for medicinal purposes of intoxicating
drinks and of drugs which are injurious to health’. The NDPS Act prohibits cultivation,
production, possession, sale, purchase, trade, import, export, use and consumption of narcotic
drugs and psychotropic substances except for medical and scientific purposes in accordance
with the law. Preparation to commit certain offences is punishable as is attempt. Accessory
crimes of aiding and abetting and criminal conspiracy attract the same punishment as the
principal offence. The NDPS Act lays down the procedure for search, seizure and arrest of
persons in public and private places. Safeguards such as prior recording of information,
notifying a superior, limiting powers of arrest to designated officers, informing the person
being searched of her/his rights have been scrupulously enforced by the courts, in light of the
stringent punishments prescribed under the Act. In 19, the NDPS Act was supplemented by
the Prevention of Illicit Traffic in Narcotic Drugs and Psychotropic Substances Act to
provide for preventive detention of people suspected or accused of involvement in drug
trafficking. Section 32B of the NDPS Act lists ‘the fact that the offence is committed in an
educational institution or social service facility or in the immediate vicinity of such institution
or facility or in other place to which school children and students resort for educational,
sports and social activities’ as one of the aggravating factors which may be considered by the
Court for imposing higher than the minimum penalty prescribed for the offence. Source: The
Daily Guardian May 12, 2021 https://thedailyguardian.com/drug-abuse-in-india-policy-
refinement-for-efficient-control/

. In the small towns of the Himalayas, the plant is considered as part of their tradition
therefore Mr Kisan farmer in parvat District of Himachal Pradesh grown marijuana a narcotic
drug in a small pot for personal consumption without making any commercial profits and Mr
Kisan is also thinking to do scientific research of marijuana for medical purpose. Will Mr.
Kisan be held liable under the NDPS Act?

(a) Growing Marijuana in small quantity at home does not fall under the NDPS Act. Thus, its
possession is not an offence that is punishable under the Law.

(b) Mr Kisan was not doing any commercial selling of marijuana and grown that for medical
purposes and scientific research. Hence he will not be liable under NDPS Act.

(c) He will be held liable under the NDPS Act for illegal possession of Marijuana.

(d) He will be held liable for the huge cultivation of Marijuana which comes under the
narcotic drugs and psychotropic substances.

. The Mishmi Hills in Arunachal Pradesh – part of the eastern Himalayas in northeast India –
have become notorious for illegal opium cultivation over the past two decades. This has led
to the destruction of large amounts of forestland in a global biodiversity hotspot and major
social problems in the remote region of India. Decide

(a) There should be an official survey to estimate the scale of illegal cultivation across
Arunachal Pradesh.

(b) There must be efforts to spread awareness about the opium menace. The government
should initiate drug rehabilitation centers, and local police, district administration, and NGOs
to organised various awareness campaigns.

(c) Both A and B

(d) Officials have to destroy all areas where opium is grown.  


 

. Earlier this month, police were called to a local school in Haryana following a report that a
suspicious food item had been recovered from the students during a bag search. The item
turned out to be an edible ganja a narcotic drug snack made with cereal. Police further
investigated and arrested Mrs XYZ she is selling this Ganja snack to school students in the
school canteen. (Those found guilty of selling Ganja in India can face a jail term of up to 10
years or a fine of Rs. 1 lakh or both under the NDPS Act). Decide Mrs XYZ's punishment
under NDPS Act.

(a) Mrs XYZ can face a jail term of up to 15 years with a fine of Rs 1 lakh under the NDPS
Act if the judiciary want.

(b) Mrs XYZ can face a jail term of up to 10 years or a fine of Rs. 1 lakh under the NDPS
Act.

(c) Mrs XYZ is not guilty of the offence under NDPS Act because she was not selling pure
Ganja.

(d) None of the Above.

. The STF chief of Punjab detains 62 persons who were suspected of supplying drugs and
supporting the drug trade directly or indirectly. The families of Detained person filed a
petition in Punjab and Haryana high court for illegal detention on grounds of suspicion only.
Can Officials detain a person under NDPS Act? (a) Officials cannot detain the person without
the trial.

(b) NDPS Act provides for preventive detention of people accused of involvement in drug
trafficking.

(c) Detention is necessary to deal effectively with the drug malady under NDPS Act.

(d) After 19 NDPS Act provides for preventive detention of people suspected or accused of
involvement in drug trafficking

Passage(Q.-Q.):The sedition law under Section 124A the Indian Penal Code (hereinafter the
IPC) says, “Whoever, by words, either spoken or written, or by signs, or by visible
representation, or otherwise, brings or attempts to bring into hatred or contempt, or excites or
attempts to excite disaffection towards, the government established by law in India, shall be
punished with imprisonment which may extend to three years, to which a fine may be added;
or, with fine.” It is sometimes claimed in popular discourse that the Constituent Assembly
was dead set against the inclusion of sedition as a restriction on free speech and expression,
and therefore, it is argued that the presence of Section 124A of the IPC penalising sedition is
an insult to the intent of the framers of the Constitution. It would help to go a few steps
before the Constituent Assembly and take a look at the evolution of Indian thought on
sedition before we allow pre-conceived notions to dictate the course of our opinions. Did the
Drafting Committee, under the stewardship of Dr. Ambedkar, remove sedition from the
grounds of restrictions on free speech and expression? No. Here’s Clause 15 based on the
proceedings of the Drafting Committee on October 31, 17: Clause 15 (i) Subject to public
order, health and morality, every citizen shall have the right— (a) To freedom of speech and
expression; *[Provided that the publication or utterances of seditious, slanderous, libellous or
defamatory matter shall be actionable or punishable in accordance with law.

. Rajni, a journalist, wrote an article on the irresponsible attitude of the government on the
issue of migrant workers during lockdown and stated, “The government has no right to be in
power”. She was charged under sedition under Section 124A the Indian Penal Code
(hereinafter the IPC). Can she take any defence under the law? If yes, which of the following
will be the most appropriate one?

(a) She cannot take the defence as the statement is an attempt to incite the hatred towards the
government

(b) She cannot take the defence as it may cause disruption in public order

(c) She can take the defence under Article 19 (1) (a) as she has right to freedom of speech and
expression

(d) She can take the defence as she has no intention to incite or attempt to incite violence
under Section 124A IPC  

. Dr. Kamila Thapar, a well-known historian participated in an assembly organized by JNU


students criticizing the government’s policies. She got charged for sedition as the act
amounted to excite the hatred towards the government. Can she be charged under Section
124A of the IPC?

(a) She cannot be charged under sedition as her act has amounted to cause or attempt to cause
violence

(b) She cannot be charged under sedition as she was exercising her right to freedom of speech
under Article 19 (1) (a) of the Constitution

(c) She can be charged under sedition as the act amounted to excite the students for causing
hatred towards government

(d) She can be charge under sedition as she has the intention to cause hatred towards the
government

. A student was charged under Section 124A for sloganeering in a rally “Hindustan
Murdabad”. He took the defence of right to freedom of speech and stated to not have the
intention to incite violence. Can he avail the defence against the sedition charged?

(a) Yes he can avail the defence as he had no intention to incite violence against the
government

(b) Yes he can avail the defence as he has not acted for causing or attempting to cause hatred
towards the government
(c) No he cannot avail the defence as the intensity of sloganeering was sufficient to bring the
sedition charges

(d) No, he cannot avail the defence as he committed act against the sanctity and integrity of
the Nation

. Nitya, after doing her MBA from Boston University, came back to India. She wanted to do
something for her people, so she decided to contest Panchayat’s election. While making her
debut speech in the election rally in the gram panchayat, she condemned the government for
the recent farm laws with strong words by saying that the government is conspired with the
industrialist and wants to exploit the poor farmers. She further urged people to overthrow the
government. Will she be held liable for sedition?

(a) Yes, because she intentionally tended to incite violence against the government

(b) No, because she was exercising her freedom of speech and expression

(c) Yes, because she instigated the people to take action against the government

(d) No, because She was merely showing her disapproval with the government policies

. A public servant in a newspaper article regarding the government’s NirogyaSetu App put
forth the lacunae in the app and suggested the government should follow the solution given
by him. He got charged under Section 124A of the IPC. How far the charged as valid as per
the law given in the passage?

(a) The charges are valid as the government officials cannot criticize the government though
a public platform

(b) The charges are valid as the words amounted to causing disruption in belief system of
common public towards the government

(c) The charges are invalid as he was exercising his right to freedom of speech and expression
under Article 19 (1)(a) of the Constitution

(d) The charges are invalid as mere criticism towards government policies does not amount to
causing hatred towards the government

37
Passage (Q.68-Q.): The Aurangabad Bench of the Bombay High court was informed on
Monday that the treatment for Mucormycosis (Black fungus disease) is now covered under
the Mahatma Jyotiba Phule Jan Aarogya Yojana (MJPJAY) and the Pradhan Mantri Jan
Aarogya Yojana (PMJAY) for eligible beneficiaries. The Court directed the State government
to give wide publicity to this decision so that the "poorest of the poor, the illiterate, and
persons living in remote areas" are aware of these facilities. The Bench of Justices RV Ghuge
and BU Debadwar also ordered that since the State had come up with a specific policy for
free treatment of the fungal disease, the identified private hospitals be restrained from raising
astronomical bills. The Court was informed on Monday by Advocate Satyajit Bora, amicus
curiae in the case, that the MJPJAY, which covers people below the poverty line, is limited to
Rs. 1, 50,000 per patient. He submitted that the treatment for the fungal disease involved
hospitalization of 4-6 weeks to recover completely. After factoring in the drugs for treatment,
the total cost to be borne by a patient comes to Rs 8-10 lakhs. He submitted that patients from
the economically weaker sections of society would find "death to be cheaper than the
treatment". Accordingly, it was informed by the State that the scheme includes free treatment
of Mucormycosis. "Irrespective of whether any person in the State of Maharashtra is a card
holder or not, all would be eligible for the treatment under the MJPJAY, in selected
hospitals," the Maharashtra government clarified. The State also clarified that there would be
capping of expenses for the treatment of the disease and that 130 hospitals have been
identified as facilities for treating it. 

68. Upon learning about the same, a public spirited individual, one Mr Diwan started
spreading the word regarding the order of the High Court which led to several slum dwellers
storming the gates of multiple private as well as government hospitals and started demanding
free of cost treatment for Mucormycosis. Are the slum dwellers eligible for such treatment?

(a) The slum dwellers will be eligible for the treatment in both private and public hospitals.

(b) Only those slum dwellers below the poverty line would be eligible for receiving treatment
in select public hospitals.

(c) The slum dwellers below the poverty line will be able to receive treatment for
Mucormycosis only in all public hospitals since the courts cannot order private hospitals to
obey.

(d) The slum dwellers below poverty line will be able to receive treatment for Mucormycosis
under any of the two hospitals as have been mentioned above, with discretion of the State.

 . Nandan is a rickshaw puller who barely earns minimum wage and sleeps on an empty
stomach some days. He takes care of his aged mother who has contracted the fungus, and
wife and is the sole bread earner of the family. Upon hearing of such news, he takes his
mother to Sheelavati Hospital, a very new state of the art medial facility which is owned by
the American playboy-billionaire-philanthropist Melon Husk. Upon admitting his mother in
the hospital he receives medical bills upwards of 25 lakhs, which included costs for treatment
of other diseases as well which his mother was suffering from. The hospital told Nandan that
the majority of the costs went in treating her for glaucoma and arthritis. Will he be liable to
pay?

(a) Nandan clearly falls within the ambit of the people covered under the two schemes as he
barely manages to earn a minimum wage like amount. Considering the same he should not be
made liable to pay.

(b) Nandan was not made aware of the hospital’s other treatments; had they only treated her
for Mucormycosis the costs surely would have not exceeded the bandwidth of 8-10 lakhs.
(c) The State has specified a certain capping of expenses occurred and the waiver of the
expenses thereto. Should he fall under the cap, his expenses shall be waivered. ,subject to the
hospital being declared as a site for treatment.

(d) The Hospital has not been notified as a place wherein such treatment shall be offered free
of cost, had it been notified his expenses would’ve been waivered subject to the capping on
expenses

. Jignesh is a landlord in the heart of the city and runs a big chain of hotels also. He has a
sprawling mansion and has a dozen of hired help to maintain his mansion. One of his servants
is Raju, who has contracted the fungus but does not have the funds for treatment. Jignesh,
being a philanthropic person, prepares to fund his treatment on his own credentials unaware
of the Court order. Upon the fulfillment of the treatment he gets to know about the order and
demands that the expenses be waivered as his servant fell under the eligibility criteria. Should
he be made liable to pay?

(a) Jignesh is a well to do magnate, he doesn’t require the waiver as he can easily pay off the
expenses.

 (b) Raju is obviously an impoverished individual and thus, should be made eligible to not
pay the expenses of his treatment as Jignesh had taken care of those out of affection and
philanthropic sentiments. However, since a mechanism for such respite was in force, the
hospital should not have charged him.

(c) Jignesh got to know about the eligibility scheme after the treatment was done, and
therefore did not possess such intention as to avail scheme. Moreover, the eligibility criteria
did not encompass well to do individuals such as Jignesh.

(d) It is unclear as the data above does not provide information about Raju’s financial
situation and the hospital being one that is specified for such schemes.

. Considering the situation in the aforementioned question, had Jignesh known about the
health schemes and the waivers in place, would he still be liable to pay for the treatment of
his servant, Raju, who fell under the eligibility criteria?

(a) Jignesh was sponsoring Raju’s treatment with philanthropic sentiment, on his own
credentials, and therefore, would be liable to pay regardless of his knowledge of the schemes
and the waivers.

(b) Raju was eligible for the waivers in place as, being a servant, he was extremely poor.

(c) The treatment in question was free of cost, regardless of the person affected or paying for
the same. Therefore, the expenses should be waivered by the Hospital in furtherance of public
duty.

(d) Statutory mechanisms such as healthcare schemes are for the benefit of the public at large,
and therefore, such schemes should be strictly enforced. Therefore, Jignesh should not be
made liable to pay.

Passage (Q.-Q.): A Supreme Court bench headed by Chief Justice NV Ramana ruled that
though Section 438 of Code of Criminal Procedure (grant of anticipatory bail by HC or
Sessions Court should be read liberally, courts should not normally grant protection from
arrest for a particular time period where no ground for anticipatory bail is made out. Such a
power can be used only in exceptional circumstances. "The Court must take into account the
statutory scheme under CrPC. and balance the concerns of the investigating agency,
complainant and the society at large with the concerns/interest of the applicant. Therefore,
such an order must necessarily be narrowly tailored to protect the interests of the applicant
while taking into consideration the concerns of the investigating authority. Such an order
must be a reasoned one." the Court noted. However, such discretionary power cannot be
exercised in an untrammelled manner, the Court underscored. Hence, the Court set aside
orders passed by the Allahabad HC, which dismissed the anticipatory bail application of the
accused but also granted them days to surrender. "A period of days cannot be considered
reasonable in these circumstances," held the top court. The reasons for setting aside the
orders were: 1. After the dismissal of the anticipatory bail application, on the basis of the
nature and gravity of the offence, the High Court has granted relief to the respondents. 2. In
granting the relief for a period of days, the Court has not considered the concerns of the
investigating agency, complainant or CrPC. which necessitates that the Court passes such
protection order for the shortest reasonable time.

. Balwan is a Bollywood actor prominently known for his illogical films and has often been
hailed as ‘the leader of fools” because of his massive fan following in spite of his terrible
movies. His latest movie ends up earning more than 50 crores on the opening day itself and
he decides to go partying with his friends. It gets late when he decides to finally drive home
after drinking heavily with his friends. He decides to drive and despite his driver being
insistent upon driving for him, he takes the wheel, only to then crash his car into a temple
where 3 people die at the gates. Upon approaching the trial court, his lawyer pleads for an
anticipatory bail of 2 months since his latest movie was doing well and the news of his
potential trial could damage the earnings. Will this action succeed?

(a) Balwan was intoxicated out of his wits and did not possess the intent to kill the people.
His driver should have been more careful, the Court should take cognizance of his plight as
his movie could end up making losses.

(b) Balwan should not be granted an anticipatory bail just because he is a movie star.

(c) Balwan should be granted bail as he has approached the trial court with appropriate
authority and has rightfully applied for the same. Moreover, he was intoxicated heavily and
thus, could not possess a guilty intent.

(d) Balwan should not be granted anticipatory bail as the grounds that have been made out
above are not grave enough to warrant such measures.

. When Balwan appears before the court, the Court is notified by the CBI that this, being a
high-profile case, should be handed over to them and their courts. The trial court, however,
proceeds to hear the pleas of Balwan and decides to grant him 6 weeks’ worth of time to
surrender before any action is taken against him. This decision of the trial court is appealed
before the HC, does the decision hold any substance?
(a) The trial court is correct in assuming jurisdiction as it has pronounced its decision only
after considering the relevant facts and evidences on record.

(b) The trial court has made an error in pronouncing its decision as it did not take into
account the claims of the CBI.

(c) The trial court has failed to take into account the concerns of the police and the other
investigating agencies but still the decision holds substance as the trial court has jurisdiction
to hear the matter.

(d) The decision does not hold any substance as the trial court did not take into account
relevant investigating authorities and was therefore, unreasonably exercising power.

. Ravish is a 23 year old guy who usually visits the Boca Bar on Weekends. On one Sunday,
Ravish got over-drunk. Another customer Ravi had just entered the Bar when he accidentally
bumped into Ravish, who was in a completely intoxicated state. This led to an altercation
between the two and Ravish broke a beer bottle on Ravi’s head. Ravi sustained serious
injuries and was admitted to hospital. The next day, Ravish started receiving death threats
from anonymous numbers, and later received a call threatening to charge him for an attempt
to murder. In this situation, could he be granted an anticipatory bail for protection from the
death threats?

(a) The courts could provide him with anticipatory bail through a reasoned order as his life is
in danger in the case.

(b) The courts could provide him with anticipatory bail as he as threatened by the anonymous
callers of being accused attempt to murder and he had valid fears with regards to being
falsely accused and arrested.

(c) Ravish wouldn’t get an anticipatory bail as he ultimately based his pleas on the murder
threats and not the attempt to murder accusations.

(d) Ravish could get an anticipatory bail if the false accusations are solified against him.

. Considering that the teenager that threatens Ravish files a complaint with the police
accusing him of rape, and the police arrests him on those counts, could he plead the courts for
an anticipatory bail?

(a) Ravish could plead the courts for an anticipatory bail as he has been arrested for being
accused of rape.

(b) Ravish might be granted an anticipatory bail

(c) Ravish cannot plead or be granted an anticipatory bail.

(d) Since the accusations hold no substance, Ravish should not be worried about bail as there
is no evidence on record
. Dinesh is a watchman for a mall and keeps watch mostly during the night shift, in the late
hours. One particular night, he is faced with a bunch of thugs who threaten him with their
guns unless he steps aside and gives him the keys to the backdoor entrance. Dinesh, however,
does not surrender quietly and decides to confront them physically. They are all engaged in a
scuffle when suddenly one of their pistols shoot and one of those thugs and a bystander is
shot killing both of them. The thugs frantically run away and Dinesh checks his gun to find
that only one shot was fired from his gun and ultimately, deduces that the other shot must
have been fired from one of the thugs’ guns. The next day he is brought in to the concerned
police station for questioning and is told by the police to not leave the country at any costs.
He starts panicking and he eventually files for an anticipatory bail, could he be granted one

(a) Dinesh could be granted an anticipatory bail as he is rightly apprehensive of being


arrested.

(b) Dinesh could not be granted an anticipatory bail as he was just brought in for questioning
by the police.

(c) Dinesh is a prime suspect in this case of homicide. He was a security guard and he
should’ve been more careful than to engage unruly thugs. Scuffles could have led to possible
public shooting.

(d) Dinesh could be granted anticipatory bail, depending upon the discretion of the court, via
a reasoned order.

Passage (Q.-Q.): The MCG decided unanimously in favour of imposing a closure of meat
shops every Tuesday citing religious sentiments. The decision of MCG raises Constitutional
dissection as the Corporation (MCG) constitutes ‘State’ under Article 12 of the Indian
Constitution. The particular direction given out by the MCG brings into question a citizen’s
right to practice any profession or to carry on any occupation, trade, or business as expressly
provided under Article 19 (1) (g) of Part III of the Indian Constitution. No right under Part III
is absolute and free from restrictions. Therefore, Article 19 (6) of the Constitution permits the
state to impose reasonable restriction in the ‘interest of general public’. If a person owning a
meat shop is guaranteed the right under Article 19(1) (g), it is pertinent to decide whether
meat sales/meat shops constitutes 'profession’, 'occupation', 'trade' or 'business', because only
if it constitutes any one of these will he/she enjoy protection under Article 19(1) (g). Once
established that meat shops function as a 'trade', the question of whether the MCG is justified
to prohibit the sale of meat on a particular day of is the second question to be answered. The
Supreme Court has held that the restrictions laid down under Article 19 have to be tested on
the anvil of the test of proportionality and that theArticles 19 and 21 of the Constitution
mandate that any 'State action' should satisfy five important criteria, which are : (a) State
actions to be backing by a law, (b) legitimacy of purpose, (c) A rational connection of the act
and object, (d) The necessity of the action, and finally (e) when the above four are
established, then the test of proportionality. [1]

. The Ajanta Municipal Corporation is empowered under Dimdima Municipal Corporation


Act passed by the state of Dimdima to remove the prostitutes from certain demarcated
locality of the city without giving them opportunity to be heard, for the convenience of
general public and public health. Can this provision be challenged under Article 19 (1) (g) of
the Constitution?

(a) No because the authorities are empowered to facilitate the public health

(b) No because the prostitution is not a profession for the purposes of Article 19 (1) (g) of the
Constitution

(c) Yes because the prostitutes are not given the reasonable right to be heard

(d) None of the above

. The state government of Bhangarh in the Union of Bairagistan came up with a rule that the
government teachers of first and second grades will not provide the service private tuition for
any cash or kind. The said rule was made to regulate the services of the government teachers
and uplift the level of education in the government schools. Shanti, a second grade teachers,
who provides free tuition to the children of poor families nearby her house, challenged the
government’s action stating that it is violative of her Fundamental Right under Article 19 (1)
(g) of the Constitution. Can she claim her Fundamental Right?

(a) She cannot claim her right as the government’s action is justiciable on the ground of
public interest

(b) She can claim her right as the government’s action of unreasonable and discriminatory

(c) She cannot claim her right as the government’s action is to promote education in
government schools

(d) She can claim her right as she is providing the tuition for free

. Internet services were suspended in Jammu and Kashmir by the Central Government in
apprehension that it might promote the frivolous and illegal activities in the valley. Sajid, a
Tradesperson, who runs an e-commerce business of pashmina, challenged the said Order
claiming that it is an infringement to his right under Article 19 (1) (g). Decide the matter as
per the given information in the passage.

(a) The act of the government is justified under Article 19 (6) to put reasonable restrictions

(b) The act of the government is not justified as it is not causing any public inconvenience or
public disorder

(c) The act of the government is not justified as it is based on mere apprehension

(d) The act of the government is justified as it may cause grievous risk to others’ right to life

. The central government of Union of Ujalapur is very concern of the health issues among the
consumers of tobacco products, such as, bidis. It put a ban on the manufacturing of bidi to
curb the spread of diseases caused by Bidis. Considering that the law of the Union of
Ujalapur is pari materia to the law of the Union of India. Can this action of government be
covered under reasonable restrictions under Article 19 (6) under public health?

(a) No, because total prohibition does not amount to reasonable restriction

(b) No because it is against the economic health of the state

(c) Yes, because consumption of bidis is causing grave health issues

(d) Yes, because it causes inconvenience to the general public

. The state government of Kunthala in the Union of Mahismati amended Kunthala Police Act,
which prohibited dance performance in the eating areas and bars with proviso, which says
such dance practices are permitted in hotels not below three stars. The objective of the
amendment is to enforce them to obtain license. Considering that the law of the Union of
Mahismati is pari materia to the law of the Union of India. How do you see the amendment in
the Act?

(a) The amendment is constitutional as the objective of the amendment is to curb


unauthorized trade

(b) The amendment is not constitutional as it is discriminatory and arbitrary

(c) The amendment is constitutional as the alleged act is detrimental to public health and
women’s safety

(d) The amendment is not constitutional as the amendment is abridging the right to trade of
those place and the dancers

Passage (Q.-Q.): Haryana Assembly passed ‘The Haryana Recovery of damages to property
during disturbance to public Bill, 2021’ a Bill that will allow authorities to recover
compensation from protesters for any damage to property. The Bill provides for recovery of
damages to properties caused by persons during disturbances to public order by an assembly,
lawful or unlawful, including riots and violent disorder”. It has provision for the constitution
of Claims Tribunal to determine the liability, to assess the damages caused and to award
compensation. Uttar Pradesh was the first state to bring a Bill for recovery from the people
for damages caused to public property during agitation and protests Vij said the Bill was
meant to hold accountable those who burn shops and damage other public and private
property during protests. “We are not bringing this Bill against people’s democratic right to
hold a protest, “he said, adding, “the actual perpetrators of such acts of violence and crimes
as also the instigators, organisers be made liable for damages to public and private property
and the cost of the forces requisitioned from outside the state for controlling such acts of
violence and crimes”. In a democracy everyone has the right to 20 of 40 speak and protest
peacefully, but no one has the right to damage property. Former chief minister claimed that
the “purpose and reasons of the proposed law” have “clearly written that the government
wants to instil a sense of fear among the common man. Section-14 of the Bill states that the
recovery will not only be made from those who indulge in violence but also from those who
want to lead the protest, the organizers, those involved in its planning, those who provide
encouragement, and the participants.” he said.

. The state of Haryana experienced a lawful protest by members of the Jat community for
political and economic reservation from the government. In some parts of the state, protesters
clashed with security forces and damaged government buses. Will protesters be legally liable
under The Haryana Recovery of damages to property during disturbance to public Bill, 2021?

(a) No, they will not be liable because they were doing lawful protests for their rights.

(b) No, the constitution of India has given its citizen the right to protest peacefully.

(c) Yes, Lawful protest also comes under the ambit of the Haryana Recovery of damages to
the property during disturbance to public Bill, 2021.

(d) Yes, the Bill provides for recovery of damages to properties caused by persons during
disturbances to public order by an assembly, lawful or unlawful.

. Farmer Leader Jitu Singh inspired and encouraged farmers of Haryana to protest against
three Farm acts that were passed by the Parliament of India but due to prior commitments,
Jitu Singh didn’t take part in the protest. The peaceful protest soon turned into a violent
protest at a certain point as protesting farmers drove through the barricades and clashed with
the police damaging public property. Decide Jitu Singh will be liable for compensation or
not?

(a) He is not liable because he was not present during the protest.

(b) In a democracy everyone has the right to speak and protest peacefully therefore he is not
liable.

(c) He will not be liable because he is not the actual perpetrator of such acts of violence.

(d) He will be liable for damages because he leads the protest.

. Students of consortium protesting against increase in fee in the state of Haryana. In name of
agitation they vandalise private university which is not governed by the state of Haryana.
Students will be liable to damage private property?

(a) Yes they will be liable.

(b) Yes damaging public property is a nuisance.


(c) No, they will not liable because they vandalise a private university that is not governed by
the state of Haryana.

(d) No, because it is common to damage some property during the protest.

. What can be inferred from Former CM's arguments for not supporting the bill?

(a) written that the government wants to instil a sense of fear among the common man.

(b) Every protesting person is not guilty because of the act of few protesters who damages
property, considering every person liable for compensation is not valid.

(c) There is only a political motive behind the Former CM argument.

(d) Government wants to take the democratic rights of a citizen.

. Which is not false according to the passage?

(a) Bill is not against the people’s democratic right to hold a protest. 

(b) Haryana is not the first state to bring a Bill.

(c) Bill was meant to hold accountable those who burn shops and damage other public and
private property during protests.

(d) All of the above 

Passage (Q.-Q.): Candidates/students have the right to access their own answer sheets as per
the provisions of the Right to Information Act. The Central Information Commission stated
that the issue involved in the present matter has been long settled by the Supreme Court in the
decision of CBSE v. Aditya Bandopadhyay, wherein following was held: “…every examinee
will have the right to access his evaluated answer-books, by either inspecting them or taking
certified copies thereof unless the same was exempted under Section 8 (1) (e) of the RTI Act,
2005.” Further, the definition of information" in Section 2(f) of the RTI Act refers to any
material in any form which includes records, documents, opinions, papers among several
other enumerated items. The term "record" is defined in section 2(i) of the said Act as
including any document, manuscript or file among others. When a candidate participates in
an examination and writes his answers in an answer-book and submits it to the examining
body for evaluation and declaration of the result, the answer-book is a document or record.
When the answer-book is evaluated by an examiner appointed by the examining body, the
evaluated answer-book becomes a record containing the ‘opinion’ of the examiner.
Therefore, the evaluated answer-book is also an ‘information’ under the RTI Act. Hence, in
view of the above, the Commission held that the present issue involved a larger public
interest affecting the fate of all the students/candidates who wished to obtain information
regarding their own answer sheet which would have a bearing on their own career and in turn
would ostensibly affect the right to life and livelihood. A restriction to this right arises when
the concerned body is not a state entity or the information is related to national security,
internal government discussions etc.
. Riddhi was a student at XYZ School in Madhya Pradesh. The School's beneficiary was Mr.
Mukesh Ambani and the school was run under the patronage of the Ambanis. Once Riddhi
received the marks for her final exams, she was astonished to see that she had secured less
than 50% marks even though she had prepared very well. Can Riddhi file an RTI to request
the school authorities to see her answer sheet?

(a) Yes, because it falls within the definition of Sections 2(f) and 2(i) of the RTI Act

(b) Yes, because every examinee has the right to access their evaluated answer-books as per
the decision given in CBSE v. Aditya Bandopadhyay

(c) Yes, because evaluation of answer sheets affects the larger public interest concerning the
fate of all the students/candidates in the country

(d) None of the above

. An Environmental Commission was set up by the government to gain a clear picture of the
worsening environmental crisis and suggest solutions for the same. Soon after, based on some
rumours, news spread that the Commission was allowing the owners of big industries to
provide a clear picture of their business practices so that they could be exempt from any
proposed punishment through some or the other technicality. The alleged reason for the same
was that these industries provided the government with a lot of revenue through taxes. Disha,
an environmental activist, heard about this and wanted to seek information about the same. If
she files a request for information under the Right to Information Act, can her request be
denied?

(a) No, because the information that she has sought is important in light of public interest

(b) No, because the information that she has sought falls within the ambit of Sections 2(f) and
2(i) of the RTI Act

(c) Yes, because the concerned Environmental Commission is not a state body but an
independent entity

(d) Yes, because the information that she has sought concerns internal government
functioning as the Environmental Commission has not given out any conclusive document or
statement

. Survi made several rounds of the panchayat office to get her grandfather's death certificate.
She desperately needed the certificate so as to complete post-death formalities related to
pension, insurance etc. However, every time she went, the official there would tell her to
come later or pay a bribe of Rs 500 to get the work done. Can Survi file an RTI inquiry to ask
why the death certificate hasn't been given to her yet?

(a) No, because only information related to answer sheets can be sought and not those related
to death certificates

(b) No, because her request of information related to a death certificate doesn't fall within the
ambit of Sections 2(i) and 2(f) of the RTI Act
(c) Yes, because her request falls within the ambit of Sections 2(i) and 2(f) of the RTI Act

(d) Yes, because her request is in line with the decision of the Supreme Court in the case of
CBSE v. Aditya Bandopadhyay

. Hussain ran a small business in Delhi. He saved all his profits so that he could visit his
parents one day, who lived in the Maldives. In February 2020, he applied for a passport
because he wanted to visit his parents. He waited for 12 months till February 2021 and visited
the passport office several times. However, he still did not receive his passport. Can he file an
RTI to find out the reason behind this delay?

(a) No, because the Supreme Court decision in CBSE v. Aditya Bandopadhyay was limited to
filing RTI requests only for answer sheets and not for other matters

(b) No, because passport delay is not related to any material in any form which includes
records, documents, opinions, papers among several other enumerated items

(c) Yes, because passport delay is not related to any material in any form which includes
records, documents, opinions, papers among several other enumerated items

(d) Yes, because passport delay is related to any material in any form which includes records,
documents, opinions, papers among several other enumerated items

Passage (Q.-Q.): The Supreme Court of India recently heard a plea seeking the protection of
‘academic freedom’ from raids and seizures of police and investigative agencies.
Destruction/distortion could be comparable to willingly destroying the sources of someone’s
livelihood and thereby disrupting his life which defies Article 21 of the Constitution under
which a person cannot be “deprived of their life and liberty”. The Supreme Court must be
mindful of the same in further hearings of the case, as it is the premier guardian of the
Constitution that swears to protect the right to freedom of speech and expression through
Article 19 (1) (a), guaranteeing academic liberty. Yet, Article 19 has always been subject to
reasonable restrictions such as national interest and public interest. Another imperative point
highlighted by the petition is provision of a ‘seizure memo’. A seizure memo is a copy of the
evidence or the materials that are seized by the investigative agencies in a form that cannot be
“overwritten or changed” while the original stays with the accused. The prime basis for the
petitioners' argument, that seeks protection for academic freedom, is their reference to the
verdict in the case of Justice KS Puttaswamy v. Union of India — a landmark judgment in the
jurisdiction of 'right to privacy'. This judgement read the right to privacy as an expansion of
the fundamental right to life. However, in its judgment, the Supreme Court held that the right
to privacy derived from Articles 14,15 and 21 is not ‘absolute’ — and will always be
subjected to some degree of restriction. Therefore, as the matter proceeds, the petitioners will
have to find peculiar references from the Constitution to make their case stronger.
. Professor X was a renowned scientist. He had spent years working on his project which he
finally published in 2021. His work explored the harmful agrarian practices encouraged by
the government that had resulted in a decline of crop yields, soil fertility as well as
environmental damage. Two days after he published his work, armed policemen came to his
door who detained him. Over the next 24 hours, they thoroughly investigated Professor X and
asked him about his sources. Can he contest this instance as a violation of his privacy?

(a) Yes, he may do so because right to privacy was recognised as an extension of Article 21
in the case of Justice KS Puttaswamy v. Union of India

(b) Yes, he can because the right to privacy is fundamental and absolute in nature

(c) No, he cannot because the right to privacy derived from Articles 14,15 and 21 is not
‘absolute’

(d) No, he cannot because the right to privacy derived from Articles 14,15 and 21 is subject
to restrictions

. While investigating Professor X’s case, the police took his research work and made a copy
of the same as a seizure memo. However, later on he was astonished to find that the seizure
memo which had been submitted as part of the evidence against him, was not an identical
replica of his work. Several footnotes and references, where he had cited his sources had been
deleted. This deletion meant that the arguments in his work now seemed like they had been
his own arbitrary claims which were not backed by anything. Can he challenge this seizure
memo?

(a) Yes, because the seizure memo must be in a form that cannot be changed or overwritten

(b) No, because the seizure memo must be in a form that cannot be changed or overwritten

(c) Yes, because the seizure memo must be in a form that can be overwritten or changed

(d) Yes, because the seizure memo must be in a form that can be changed but not overwritten

. As Professor X’s trial went on, the police unearthed more evidence from his house. This
included different drafts and versions of his final work, as well as a draft of his follow up
research in a pen drive. This new draft provided statistics and interviews of several farmers
who had adopted new agrarian practices suggested by the government but suffered a loss as a
consequence of the same. When this draft was presented before the Court, the police had
omitted the statistics and interviews. They only presented the conclusion of the study and
deleted the rest. Is this action of the police valid?

(a) Yes, because omission of statistics and interviews did not change the substance of the
work and the integrity of the work remained the same

(b) Yes, because the seizure memo in this case has neither been overwritten not changed
(c) No, because the police have not compromised the integrity of the work by causing such an
omission

(d) No, because the distortion and omission of the original draft may be in violation of Article
21

. Barkha Dutt was a famous journalist. She had a special niche in reporting war stories and
loved to associate herself with the Kargil war. She published an article on her blog where she
unearthed secret military locations of the Indian army which had been used as vantage points
in several wars and scuffles. Within a minute of posting the article, her blog was shut down
by the Ministry of Information and Broadcasting and all the traces of that article were
subsequently deleted. Barkha contested that this was a violation of Article 19. Is this a
violation of principles mentioned in the passage?

(a) No, because Barkha’s article was in contravention of national interest as it revealed
confidential information related to the Indian army

(b) No, because Barkha’s article was in contravention of public interest as it revealed
confidential information related to the Indian army

(c) Yes, because Barkha’s article was not in contravention of national interest as it revealed
confidential information related to the Indian army

(d) Yes, because Article 19 protects the right to freedom of speech and expression through
Article 19 (1) (a), guaranteeing academic liberty

Passage (Q.-Q.): The Lok Sabha on Wednesday passed the Juvenile Justice (Care and
Protection of Children) Amendment Bill, 2021. The Bill inter alia seeks to re-dene the
category of "serious offences" under the Juvenile Justice (Care and Protection of Children)
Act, 2015 and further, empowers the District Magistrates to pass adoption orders. The Bill
proposes to amend Section of the Act to the effect that offences punishable with
imprisonment between 3-7 years shall be non-cognizable and non-bailable. (At present, such
offences are cognizable (where arrest is allowed without warrant) and non-bailable). Section
58 of the JJ Act prescribes the procedure for adoption of children by prospective adoptive
parents. The procedure involves a seal of approval by the Civil Court, which passes the
adoption order. The Bill provides that instead of the court, the District Magistrate (including
Additional District Magistrate) will issue such adoption orders, both for intra-country and
inter-country adoptions. The Bill further proposes that any person aggrieved by an adoption
order passed by the District Magistrate may file an appeal before the Divisional
Commissioner within a period of 30 days. Endeavour shall be made to dispose of such
appeals within 4 weeks. The Bill seeks to strengthen the Child Welfare Committees (CWCs)
by incorporating provisions relating to educational qualifications for its members and
stipulating eligibility conditions for selection of the Committee. The Bill proposes certain
additional criteria for appointment of CWC members. It provides that no person shall be
eligible for selection as a member of the CWC if he: • has any past record of violation of
human rights or child rights • has been convicted of an offence involving moral turpitude, and
such conviction has not been reversed or has not been granted full pardon in respect of such
offence. • has been removed or dismissed from service of the Government of India or State.

. Morona 13-year-old boy fetched a shotgun he had found under the house and ran inside. He
aimed it at everyone in the room. Then he pointed it at a 12-year-old girl and fired but the
attempt was unsuccessful because the boy missed the mark and the bullet killed the dog.
Police arrested him under an offence punishable with imprisonment of 5 years. Can police
arrest him without a warrant considering Juvenile Justice (Care and Protection of Children)
Amendment Bill, 2021?

(a) Yes, this offence is cognizable where an arrest is allowed without a warrant.

(b) No, a 13-year-old boy is a minor therefore will not be liable for an attempt to murder.

(c) No, Police cannot arrest the boy without a warrant considering JJ amendment Bill, 2021.

(d) Yes, a murder attempt is a serious offense under the JJ act. Police can arrest him without a
warrant.

. Mr Kriplani the retired IAS officer wants to join as a member of Child Welfare Committees
(CWCs). In 2012 he was convicted of subjecting her wife to cruelty and attempt of killing her
for dowry. Decide Mr. Kriplani shall be eligible for selection as a member of the CWC or
not? 

(a) No, he is not eligible because he has a past record of violation of child rights. 

(b) No, he is not eligible because has been convicted of an offence involving moral turpitude. 

(c) No, he is not eligible because has been convicted of an offence involving moral turpitude,
and conviction has been granted a full pardon. 

(d) All of the Above.

. Mrs. Sharma and Mr. Sharma the Indian couple want to adopt the girl child from Pakistan.
Who will issue such adoption orders under the JJ act? 

(a) No adoption order will be given to them because inter-country adoption is not allowed in
India. 

(b) The Bill provides that the District Magistrate (including Additional District Magistrate)
will issue such adoption orders. 

(c) The JJ Act prescribes civil court to issue such adoption order. 

(d) The Divisional Commissioner within 30 days will issue an adoption order

. Which of the following amendments under the Juvenile Justice (Care and Protection of
Children) Act, 2015 are not true? 
(a) Offences punishable with imprisonment between 3-7 years shall be non-cognizable where
police can arrest without warrant. 

(b) The Bill provides that instead of the court, the District Magistrate (including Additional
District Magistrate) will issue such adoption orders. 

(c) Offences punishable with imprisonment between 3-7 years shall not be cognizable where
police cannot arrest without warrant. 

(d) Strengthen the Child Welfare Committees (CWCs) by incorporating provisions.

. How the Bill seeks to strengthen the Child Welfare Committees (CWCs)? 

(a) By including provisions for educational qualifications for its members. 

(b) Not allowing the person who has been removed or dismissed from service of the
Government of India or State. 

(c) Not allowing the person who has any past record of violation of human rights or child
rights. 

(d) All of the above. Passage

(Q.-Q.): The Allahabad High Court has held that discontinuation of electricity supply which
in turn results in the interrupted supply of water to agricultural fields is a violation of the
fundamental right to trade and profession of the agriculturalists. Any eventuality that
adversely affects supply of water to agricultural fields Virtually amounts to violation of
Article 19 as that affects business and Profession of agriculturists. The Constitution of India,
therefore taking into consideration the need and the importance of work, enshrines and has
guaranteed a fundamental right under Article 19(1) (g) to practice any profession, or to carry
on any occupation, trade, or business, to all the citizens residing with the jurisdiction of the
country. Under this article, every citizen has the right to choose employment, or take up any
trade or occupation, etc. as per his free will. The State cannot compel any citizen to run a
business which is against his will. But as no right is absolute, the right to close a business is
also not an absolute right. It is something which the State has the right to control by law for
the welfare of the public. However, if a person is involved in some business, he can be
compelled by the State to close it down, for the concern of the general public. Under Article
19(6), the State has the right to impose certain limits (a) Impose reasonable restrictions upon
the freedom of trade, business, occupation, or profession in the interest of the general public
(b) Prescribe the professional or technical qualifications necessary for carrying on any
occupation, trade, or business. (c) Carry on any trade, business, industry, or service, by itself
or through a corporation controlled by the State – to the exclusion of private citizens, wholly
or in part.

. ABZ is Japanese private company demand for operation of Solar Energy Plants in India but
restricted by a Government officer. Is it a violation of the Rights of ABZ Company under
Article 19 of the constitution of India?

(a) Yes, every citizen has the right to choose employment, or take up any trade or occupation.
(b) No, if a person is involved in some business, he can be compelled by the State to close it
down, for the concern of the general public.

(c) No, there is not any violation of the fundamental rights of the company.

(d) Both (b) and (c)

. In pursuance of an order passed by the Bar Council of India (BCI) and approved by the
central government, Prohibit Law College to wind down its operation if any batch of college
has completed its graduation, it is mandatory for college to run it administration till all
current batches not complete their final year. Ecstasy Law College Pune wants to wind down
its operation in 2020 due to unanticipated financial challenge caused disruption of business
and 2019 batch will complete its final year in 2024. Decide whether guidelines of BCI
forcing the Law colleges to operate against their will are a violation of their rights under
Article 19?

(a) Yes, the State cannot compel any citizen to run a business which is against his will.

(b) Yes, for the welfare and wellbeing of oneself they can close.

(c) No, the state can compel not to close business.

(d) No, the state can compel not to close business for the welfare of the general public.

. ICSI prohibits a person from practice the profession of Company Secretary unless such
person is fluent in English and has the certificate of fluency from the ICSI. Mr. Kapil, an
Executive Programme passed student of Company Secretary Ship Course challenge the ban
as it violates his right to carry his profession. Decide

(a) Institute of company secretary of India has a superior right to ban.

(b) The ban is justified as the right of Mr. Kapil to carry his profession is not absolute.

 (c) The ban is not justified as the ICSI cannot deprive any person of his right to carry on his
profession.

(d) The ban is not justified because every person in India doesn’t know the English language.

. According to the passage what could be the reason behind Allahabad High Court to hold
that discontinuation of electricity supply to fields is a violation of the fundamental right to
trade and profession of the agriculturalists.

(a) the interrupted supply of water to agricultural fields adversely affects the supply of water
to agricultural fields result in the loss of crops.

(b) Loss of crops affects the livelihood of farmers because agriculture is only their source of
income. (c) Farmer should be given rights because the country depends on them for food.

(d) ALL of the above.


. MCI forces Doctors to continue their profession for a lifetime. No doctor can stop practicing
even if he is not willing to carry the profession. Decide legal validity of MCI guidelines
according to passage.

(a) MCI is not violating any right because the state can compel not to close business for the
welfare of the general public

(b) The State cannot compel any citizen to run a business which is against his will.

(c) Lifetime restriction is not a reasonable restriction.

(d) Both (b) and ©

38
Passage (Q.67-Q.): In many situations, a criminal defendant may wish to argue that he or she
never intended to commit a crime and that the criminal act that occurred was a result of a
mistake of facts regarding the circumstances of the crime or a misunderstanding concerning
the law at the time. Such mistakes of fact can be applied to a variety of criminal activities, but
mistakes of law are only rarely allowed as full defenses to a criminal conduct. Mistakes of
fact arise when a criminal defendant misunderstood some fact that negates an element of the
crime. However, this mistake of fact must be honest and reasonable. Likewise, the mistake
must be one that would appear reasonable to a judge or jury. Mistake of law is a defense that
the criminal defendant misunderstood or was ignorant of the law as it existed at the time. The
onus is generally placed on individuals to be aware of the laws of their state or community,
and thus this defense only applies in very limited circumstances. Specifically, mistake of law
can be used as a defense in limited circumstances: When the law has not been published or
the defendant relied upon a law or statute that was later overturned or deemed
unconstitutional or the defendant relied upon a judicial decision that was later overruled or
the defendant relied upon an interpretation by an applicable official. Additionally, the
defendant’s reliance on any of these sources must have been reasonable, much like mistake of
fact. It is also important to note that, while reliance on an interpretation of an official may
include judges or federal or state agencies, it does not include reliance on the statements of a
private attorney. It is therefore important to ensure that any attorney from whom you obtain
advice is knowledgeable and trustworthy.

67. Rex, a foreign citizen, visited India during winters and went for a tour to the Shimla Hill
Station in month of January 2021. In the morning time, he saw that a few people at the Hill
Station had set fire on road side to get their body warm in such cold weather. Next day, he
saw two workers in his hotel and asked them to help him out to cut a dry tree from backyard
of hotel. They agreed to do it for him and demanded a sum of Rs. 250 for the work. Rex paid
the amount in Indian Rupees only. They helped Rex in cutting down a tree and cut it further
into smaller logs and pieces. Rex too set fire and sat along with the workers to get their body
warm. Soon, Police officials patrolling in the nearby area watched them and arrested Rex
with the two workers for causing harm to the environment and cutting down a tree. Rex came
to know that cutting down a tree is a criminal offence in India. Decide –

(a) Rex can plead mistake of law as a valid defence as he was not aware about the Indian Law
of cutting down a tree as a punishable offence and hence is not liable of the said offence.
(b) Rex cannot plead mistake of law as valid defence as he should be aware about the Indian
Law before visiting India and hence he is liable for punishment for said offence.

(c) Rex can plead mistake of law as defence under the exceptional circumstances as the law
in issue was not a major law which has to be known to Rex and thus, he is not liable for the
offence of cutting down a tree.

(d) Rex cannot plead mistake of law as a defence as it does not fall in any of the exceptional
circumstances and Rex should be known of Indian Law before he commits any such offence.
He is liable for the said offence.

68. Mr. Ratnani was a well known merchant and a businessman. He was indulged in imports
and exports of gold across the globe. He also had an illegal business of selling impure gold to
countries having the higher tax jurisdictions. By this way, he was also earning a lot of black
money. He was travelling on 28th January 2021 from Malaysia to Singapore via Bombay in a
flight. He was carrying with him a bag full of gold approax 40 Kgs. He had not disclosed it in
the list of items taken in record by the airport authorities before boarding the flight. On 20th
January 2021, Government of India notified that no more exemption is being given to the
travellers on carrying gold and minimum limit prescribed was being scraped. Now each item
has to be disclosed before the passenger boards his respective flight. The notification was
made effective from six months after the date of the notification. Mr. Ratnani was caught at
the Bombay Airport and was arrested for not disclosing the gold in the list of items taken in
Malaysia. Decide –

(a) Ratnani can claim the defence of mistake of fact as he was not aware about the recent
notification and hence, he is not liable for carrying the gold without listing it with the airport
authorities.

(b) Ratnani cannot claim the defence of mistake of law as he should be aware of the recent
notifications of Government of India before travelling in the flight and he is liable. Hence,
should be prosecuted.

(c) Ratnani cannot claim the defence of mistake of law as he is not liable for carrying the
40Kgs gold from Malaysia to Singapore despite of the notification of Government of India.

(d) Ratnani can claim the defence of either the mistake of law or mistake of fact as he was
indulged in this business from so long and hence should be prosecuted for the same.

. Pintu a boy aged 17 years was living in the slum area of Delhi. He used to perform on
streets and earn bread for his family. He was good at playing with knives and shooting the
targets with perfection. He sets his target, chalks out the circles on the tree and started
practicing there. He practices throwing the knives on a trunk of a tree in the garden daily.
People in the park used to watch his play every evening. Chintu, a boy aged 10 years also
comes to park with his mother to enjoy the swings. Pintu does not like Chintu much as he
was very notorious and irritating. One day, Pintu threw a knife which missed the target and
hit Chintu who was playing just behind the target tree. Chintu was grievously hurt and
wounded by the knife. Chintu’s mother complained to police officials of Pintu. Pintu was
taken to police station. Decide –
(a) There is no defence available with Pintu as he had clear intention or enough mens rea of
hurting Chintu grievously as he had enmity with him. Pintu is liable for offence.

(b) Pintu can avail the defence of mistake of fact as he had no such intention to hurt Chintu
grievously and had hit him by mistake. He is not liable for the offence.

(c) Pintu can avail the defence of mistake of law as he was not aware that throwing of a knife
can also be a criminal offence and punishable. Hence, he is not liable.

(d) Pintu is liable for his acts and cannot take any defence of either mistake of law or mistake
of fact. He had mens rea to hurt Chintu and hence he is liable for the offence.

. Pankaj, an Indian citizen but a foreign resident, visited India in the year 2018. He came for
his business purpose and found it to be suitable to trade in Indian Jurisdiction. In year 2021,
he planned to go back to his resident place and spend some time with his family living there.
He had to catch the flight on 23rd March 2021, but was arrested by the authorities and reason
stated was that he was boarding with an expired visa and an outdated passport. He contended
that he has his fundamental right to freedom of movement and the authorities cannot infringe
upon his right. The authorities argued that the right which he is talking about has been
amended long back by Indian Parliament. A notification dated 15th August 2019 had
modified certain restrictions on the fundamental right to free movement. The notification was
being effective from January 2020. Pankaj further contended that the Parliament cannot
amend the fundamental rights as they are part of the basic structure as held in Keshavanand
Bharti Case which was upheld in many more cases. The authorities asked him to get himself
acquainted with the Indian Laws and then defend his unlawful acts. Decide –

(a) Pankaj can take the defence of mistake of law as his conduct falls within the exceptional
circumstances where the law has not been published properly by the Indian Parliament.

(b) Pankaj can take the defence of mistake of law as his conduct falls within the exceptional
circumstances where he relied upon a landmark judicial decision of the court which was later
overruled.

(c) Pankaj cannot take the defence of mistake of law as he is liable for his conduct. It is
reasonable to expect that he should have knowledge of Indian Laws and well acquainted with
them.

(d) Pankaj can take the defence of mistake of law as his conduct falls within the exceptional
circumstances where he relied upon a law or statute which was later deemed unconstitutional
by the Indian Parliament.  

. Choose the most appropriate answer.

Assertion (A): If the criminal defendant can prove that the mistake reasonably negated an
element of the crime, the defense of mistake of fact will usually be held to apply and absolve
the defendant completely of his liability.
Reasoning (R): The act of defendant lacks mens rea or an intention to commit an offence or
to do something wrong and they are committed under good faith then defence of mistake of
fact can be availed under Criminal law.

(a) Both A and R is correct and R is the correct explanation of A.

(b) Both A and R is correct but R is not the correct explanation of A.

(c) Both A and R is incorrect.

(d) Only A is incorrect but R is correct.

. In which of the following situations, defence of mistake of fact cannot be relied upon by the
defendant.

I. Mr. Somesh filed a suit in the family court for a property dispute of an ancestral property
between him and his four brothers. The court heard both the sides and decided in favor of his
brothers. The court decided that no part of ancestral property is to be shared with Mr.
Somesh. Somesh, still believes that the property belongs to him, commits forgery and took
the false signatures of all his brothers on the transfer deed of the property in issue. All four
brothers filed a joint suit against Somesh.

II. Sunita, owner of an alcohol shop, was notified by a government order not to sell alcohol to
any of the minor child in her locality. Meera, a minor, represents herself to be above 18 years
of age, asked Sunita for two bottles of beer. Sunita gave the beer bottles to Meera and later
found her to be a minor. Sunita was arrested by the officials.

III. Piyush and Simran were married to each other. But from last two years, they were not in
comfortable position with each other and decided not to live like husband and wife anymore.
They both filed a suit in family court for divorce by mutual consent. In the meanwhile,
Piyush gets married to Mrinal, his ex[1]fiancé. Simran charged Piyush for the offence of
bigamy.

IV. Ram Prakash was at a position of SP in Thane Police Station. He ordered his subordinates
to arrest Mohan, a habitual offender. His subordinates arrested Sohan who looks similar to
Mohan. Sohan was taken into police custody and was kept in prison for 7 days. On inquiry, it
was found that the person arrested was Sohan and not Mohan. Sohan filed a suit for
compensation from the Police Officials.

(a) Both II and IV

(b) Both I and III

(c) Both III and IV

(d) All of the above

 
Passage (Q.-Q.): The specific case put forth by the petitioner was that the summons, alleged
to have been sent through WhatsApp to his mobile phone, had never reached him, as he had
not downloaded the WhatsApp application on his phone. Section 62 of CrPC, dealing with
the mode of service of summons, prescribe the following: “62. Summons how served- (1)
Every summons shall be in writing and served by a police officer, or subject to such rules as
the State government may make in this behalf, by an officer of the Court issuing it or other
public servant. (2) The summons shall, if practicable, be served personally on the person
summoned, by delivering or tendering to him one of the duplicates of the summons. Going by
Section 65 of CrPC, if service could not be effected as provided under Section 2, the serving
officer shall affix one of the duplicates of the summons to the conspicuous part of the house
or homestead in which the person summoned ordinarily resides.” The Bench stated, above
provisions do not provide for service of summons through WhatsApp. No doubt, the
revolutionary changes in the field of communication calls for a more pragmatic approach
regarding the mode and manner of service of summons. In this regard, the insertion of
Section 144 in NIA for the purpose of overcoming the delay in serving summons, which
provide for service of summons by speed post or by approved courier service. In Indian
Banks Assn. v. Union of India, the Supreme Court had alerted the Magistrates about the need
to adopt a pragmatic and realistic approach while issuing process and had directed to issue
summons by post as well as by email. In the case at hand, summon was stated  to had been
issued through WhatsApp, which is not an accepted mode of service. As such, the court
should not have issued non-bailable warrant against the petitioner on the assumption that he
had failed to appear after receiving the summons.

. Yash, a well known businessman, was director and CEO of a soap manufacturing company,
THOR. His company was found to be indulged in many illegal activities and concrete
corruption charges were made against the company. Also, anti-competition market practices
were alleged to have been adopted by the company to establish its monopoly in the market in
order to earn larger profits. Soon, a suit was filed in NCLT (National Company Law
Tribunal) against THOR Co. Ltd to prevent these malpractices in the market and avoid
further loss to the other companies in competition. The bench of NCLT hearing this case was
headed by two retired judges of High Court. Ret. Jst. Sharma issued and served summons to
Yash to appear before the Tribunal and present his defence arguments. Yash refused to
appear for the hearing as he claimed the due procedure under section 62 of Cr.P.C to be
violated. Decide –

(a) Yash is incorrect in his contention as the summons served to him was valid and proper.
There is due procedure followed by the NCLT given under sec.62 of Cr.P.C.

(b) Yash is correct in his contention as the summons served to him was not valid and
improper. The NCLT has not adhered with the due procedure given under sec.62 of Cr.P.C.

(c) Yash is not correct in his contention as the summons served to him was invalid though
properly served. There is minor deviation from the due procedure under sec.62 of Cr.P.C
which does not affect the validity of such summons.

(d) Yash is not incorrect in his contention as the summons served to him was invalid and
improper as they do not follow the essential requirements of serving the summons under
sec.62 of Cr.P.C.
 

. Madhuri, a minor, was in love with a gangster Munna. Munna was aged 30 years and that is
the reason their families were not ready for their marriage. Munna asked Madhuri to run
away with him and no one can do anything once they are married. Madhuri consented to this
offer of Munna and they ran away together to get married. Soon Madhuri’s father gets to
know about this and he filed a case against Munna for kidnapping her daughter. The court of
magistrate took cognizance of the case and issued summons to Munna. The court officer went
to Munna’s house to serve the summons and found empty house. He pasted summon on the
bottom side of the entrance gate near the sewage pipe of the house. Munna had not appeared
for the hearing and court issued arrest warrant against Munna. Munna on being arrested
contended that he had not been called upon by the court and summons served were improper.
Decide –

(a) Contention of Munna will fail as the summons was served to Munna properly. They were
valid and in accordance to requirements given under sec.62 of Cr.P.C.

(b) Contention of Munna will succeed as the summons was served to Munna improperly.
They were valid but not reasonable as per the requirements given under sec.62 of Cr.P.C.

(c) Contention of Munna will fail as the summons was served to Munna properly. They were
invalid though as it has to be served in person to Munna or to any of his family members in
absence of Munna.

(d) Contention of Munna will succeed as the summons was served to Munna improperly.
They were invalid and not as per the requirements given under sec. 62(2) of Cr.P.C.

. Priyanka, an actress, was famous diva in the Bollywood industry. She was married to a well
known athlete, Virendra Ganguly. Virendra was not happy with her career. He does not like
his wife to have romantic scenes on screen with other co-actors and he warned her for the
same. Priyanka never thought that her husband would be such an old-school type person and
wants to get divorced. She filed a divorce petition before the family court in Bombay. She
claimed divorce on the grounds of cruelty and mental harassment. The court ordered for a
cooling off period of 6 months before the couple finally decides to get divorce. Priyanka and
Virendra went together for a foreign tour before the court grants the final decree. In the
meantime, an FIR was lodged against Virendra alleging murder of wise businessman due to
grave enmity. The session court of Bombay issued summon in name of Virendra and when
the officer went to serve the same, he gets to know that Virendra will return only after 3
months break. Decide –  

(a) The session court has to wait till the time Virendra returns back from his foreign tour after
a break of 3 months and then issue fresh summons again.

(b) The session court need not wait till the time Virendra returns back from his trip. They can
serve the summons to his family members or can affix it at conspicuous space of his house.

(c) The session court need not wait for Virendra to return back after 3 months from his tour
and instead they can send summons issue via Whatsapp or other social media handle.
(d) The session court need not wait for Virendra till the time he returns back from his trip.
Instead summons can be issued via speed post or over e-mail to Virendra.

. Bahadur and Shamsher were brothers. They were engaged in the business of manufacturing
a soft drink in Bhopal, Madhya Pradesh. Bahadur usually handles the accounting part and
monitors the meetings and other documentation work for the company. Shamsher mostly
goes for foreign tours for business purpose and controls management affairs outside India. A
foreign company in India filed a suit against Bahadur for sexual harassment of one of its
employees. The Bhopal Criminal Court took the cognizance of the matter and issued
summons for Bahadur to appear for hearing. The police officer went to serve the summons
and gets to know that Bahadur lives on rent and his actual place of residence is in Jaipur,
Rajasthan. The officer further inquired and informed the court officer that Bahadur is busy in
a board meeting. Thus, he will be serving the summons after a week. Decide –

(a) There was irregularity in the procedure as prescribed under section 62 of Cr.P.C and
summons has to be served on date when it is issued.

(b) There was no irregularity on part of police officer as he went to serve the summons and
had inquired sufficiently. It is reasonable to serve summons to Bahadur after a week since he
was busy in board meetings.

(c) There was irregularity in serving the summons to Bahadur by the Police Official as it is
not reasonable to serve him the summons with delay for a week as he was busy with his
board meetings.

(d) There was no irregularity in the set procedure under section 62 (1) of Cr.P.C. The court
has issued the summons validly and has to be served properly as per requirements of section
62.

. In the case of State of Uttar Pradesh vs. Singhara Singh, the court held that when a power is
conferred then same has to be exercised in the mode prescribed in the statute and in no other
way. Choose from the following the accepted modes of serving the summons.

I. Serving summons through Twitter Account.

II. Serving summons by way of fax messages.

III. Serving summons over a channel created by the officer of court on Telegram App.

IV. Serving summons over registered address of a company or to the residential place of
director of the company.

(a) Both I and IV.

(b) Both II and IV.


(c) Both III and IV.

(d) None of the above.

. Which among the following is not correct statement for Summons under section 62 of
Cr.P.C?

(a) Summons is served with the purpose to inform the accused that an action has been taken
against him and he has to appear before the court.

(b) Summons is sent by either police official or any other officer of the court. It can also be
sent by any other public servant of the State Government.

(c) A duplicate copy of summon has to be issued and sent to the victim to provide a proof that
accused has been summoned before the court.

(d) In case accused is not at home, such copy of summon can be issued and served by
affixing it to the conspicuous part of the house of the accused.  

Passage (Q.-Q.): Article 32 deals with the ‘Right to Constitutional Remedies’, or affirms the
right to move the Supreme Court by appropriate proceedings for the enforcement of the rights
conferred in Part III of the Constitution. It states that the Supreme Court “shall have power to
issue directions or orders or writs, including writs in the nature of habeas corpus, mandamus,
prohibition, quo warranto and certiorari, whichever may be appropriate, for the enforcement
of any of the rights conferred by this Part”. The Article is included in Part III of the
Constitution with other fundamental rights including to Equality, Freedom of Speech and
Expression, Life and Personal Liberty, and Freedom of Religion. Only if any of these
fundamental rights is violated can a person can approach the Supreme Court directly under
Article 32. When it comes to violation of fundamental rights, an individual can approach the
High Court under Article 226 or the Supreme Court directly under Article 32. Article 226,
however, is not a fundamental right like Article 32. The five kinds of writs are: · Habeas
corpus - related to personal liberty in cases of illegal detentions and wrongful arrests ·
Mandamus — directing public officials, governments, courts to perform a statutory duty; ·
Quo warranto — to show by what warrant is a person holding public office; · Prohibition —
directing judicial or quasi-judicial authorities to stop proceedings which it has no jurisdiction
for; and · Certiorari — re-examination of an order given by judicial, quasi-judicial or
administrative authorities.

. Barbie went on a jail-visit as part of her research project. When she went there and
interacted with the prisoners, she was astonished to find that some of the prisoners, who were
insane at the time of trial but subsequently declared sane, were not released due to inaction of
State Authorities. Consequently, at this point, they had remained in jail for almost 20 to 30
years. Shocked at this discovery, Barbie wrote a letter to the Supreme Court. Can the
Supreme Court initiate writ proceedings in this case?
(a) Yes, the Supreme Court can issue a writ of prohibition to stop the jail terms of the
prisoners;

(b) Yes, the Supreme Court can issue a writ of Habeas Corpus to restore the liberty of the
prisoners;

(c) No, the Supreme Court cannot issue a writ of Habeas Corpus because there has been no
illegal detention or wrongful arrest;

(d) No, because there has been no violation of fundamental rights to initiate writ proceedings.

. Sonam always wanted to study in a school but it was far away. When the government
opened a school in her district, she was delighted to finally be able to fulfil her dream. When
she went to enquire about the admission process, she was informed that there were many
vacant seats. Right to Education is also a fundamental right. However, when she went to
complete the final admission process with her mother, she was refused admission without any
due process. Can Sonam file a writ petition in this case?

(a) Yes, she can file the writ of mandamus because the school authorities are not fulfilling
their statutory duties;

(b) Yes, she can file the writ of habeas corpus because her personal liberty is being violated
by being denied admission;

(c) Yes, she can file the writ of quo warranto to ask the authorities the reason for denying
admission;

(d) Both (a) and (b).

. District Courts have the jurisdiction to hear disputes only if their value is less than Rs. 1
crore. If the amount involved in the dispute falls in a range greater than Rs. 1 crore, then the
High Courts have the jurisdiction to hear the dispute directly, without going to the lower
courts. A and B were involved in a property dispute amounting to Rs. 1.25 crores. A filed a
case in the Rampur District Court and the Court commenced a trial. Can B exercise any writ
remedy to stop the proceedings?

(a) Yes, B can file certiorari so that the High Court takes up the matter and stops the
proceedings in the District Court

(b) Yes, B can file prohibition to stop the proceedings in the District Court

(c) Either (a) or (b)

(d) None of the above  

. Mr K.N. Srivastava was appointed as a judge of the Guwahati High Court. X was a
concerned citizen who looked into the background of Mr. K.N. Srivastava. He found out that
Mr K.N. Srivastava did not meet the experience which was required to become a High Court
judge. Consequently, he decided to file a writ of Prohibition to stop this decision. Is he
correct in exercising this writ remedy?

(a) No, he should have filed the writ of Quo Warranto to question the appointment of the
Judge;

(b) No, because the writ of Prohibition is filed only for courts overstepping their jurisdiction;

(c) Both (a) and (b)

(d) Yes, he is correct in filing the writ of Prohibition because it can stop the appointment of
the judge.

Passage (Q.-Q.): Section 63 of the Indian Contract Act, 18 provides the framework for the
doctrine of waiver in India. It recognises the right of the promisee to dispense with, extend
the time for or accept any satisfaction in lieu of the contractually agreed performance.
Although the provision does not expressly refer to "waiver", the principle has been repeatedly
interpreted as such by the Indian courts. The Supreme Court has described waiver as an
intentional relinquishment of a known right or advantage, benefit, claim or privilege which
except for such waiver the party would have enjoyed. A person is required to be fully
cognizant of his rights before waiving them. Voluntary intention is the essence of waiver.
Even the slightest indication of waiver being involuntary may render it invalid. A law which
is made for individual benefit can be waived by an individual, but when such law includes an
element of public interest / policy, such rights arising from the law cannot be waived because
the same becomes a matter of public interest / policy. Consequently, a statutory right may be
waived only if it can be materially proved that the right relates to a private benefit. Most
importantly, fundamental rights cannot be waived.

. Income Tax must mandatorily be paid by all Indian citizens. X defaulted on his tax payment
to the tune of Rs. 4 lakhs. In a bid to avoid a harsher penalty, he reached a settlement with the
tax authorities to pay Rs. 3 lakhs over the course of three months. Subsequently, the Supreme
Court declared that charging Income Tax is a violation of fundamental rights. X challenged
his settlement with the tax authorities. Will he succeed?

(a) No, because he waived his right to avoid paying income tax by specifically reaching an
agreement with the tax authorities;

(b) No, because he was fully cognisant and voluntarily waived of his fundamental right by
way of the agreement;

(c) Yes, because charging of Income Tax is in violation of fundamental rights;

(d) Yes, because charging of Income Tax includes an element of public interest.

. Sam and Dean entered into a contract in 2007. One of the clauses of the agreement provided
that, "In the event of a dispute, no party shall seek legal remedy in any court of law. All
disputes must be resolved by way of an out of Court Settlement". In 2020, the relations
between Sam and Dean turned sour and Sam sought to terminate the contract. He sent Dean a
legal notice. Can Dean challenge the validity of this notice based on the Doctrine of waiver?
(a) Yes, because Sam and Dean had voluntarily waived their right to approach the Court for
individual benefit; (b) Yes, because Sam had voluntarily waived his right to approach the
Court while signing the contract; (c) No, because parties cannot waive their right to seek legal
remedy as it is against public policy; (d) No, because parties cannot waive their right to seek
legal remedy as it is a violation of fundamental rights.  23 of 40 . Hrithik was staying in a
rented accommodation. His landlord was Shah Rukh. Due to COVID-19, Hrithik was unable
to pay rent. Under the India Rent Act, “All tenants are statutorily required to pay rent without
any excuse, else they shall become liable for legal action.” Shah Rukh reduced the rent owed
by Hrithik and waived his right to seek legal recourse. Can he do so?

(a) Yes, because the India Rent Act can be waived for individual benefit as it would help
Hrithik avoid legal action;

(b) Yes, because the India Rent Act can be waived for individual benefit as it would help
Shah Rukh avoid legal action;

(c) No, because the India Rent Act includes an element of public interest to protect landlords
and their rent;

(d) No, because the India Rent Act includes an element of statutory rights to protect landlords
and their rent.

. Chunnu was a vaccine manufacturer. He signed a contract with the government for the
provision of vaccines. One of the provisions of the contract waived Chunnu's responsibility to
conduct any trial of his vaccines. Under the contract, all vaccines produced by him could
directly be procured by the government for mass vaccination, without any prior testing. Is
such a contractual waiver permissible?

(a) No, because such a contractual waiver would be contrary to public policy;

(b) No, because such a contractual waiver would infringe fundamental rights like right to
health in light of mass vaccinations;

(c) Yes, because such a contractual waiver would be for individual benefit and not infringe
public policy;

(d) Yes, because such a contractual waiver would be for individual benefit and not infringe
fundamental rights.

. The standard time period for manufacturing and procuring of vaccines is three months. In
the previous case, the government decided to increase the time period to six months to allow
for proper testing of the vaccines by Chunnu before their roll-out. This deviated from the
standard practice but the government feared backlash from the citizens so it decided to take
this drastic step. Is it legal for a promisee to extend the time for a contractual performance?
(a) Yes, because the doctrine of waiver can be applied in this case;

(b) No, because the doctrine of waiver can be applied only for individual benefit and this is
not a private matter;

(c) No, because such a waiver would be contrary to public policy and public interest;

(d) None of the above.

. A large number of slum dwellers lived under a flyover in Delhi. The government agreed to
speak with them and in exchange, it sought to demolish their dwellings by way of a new law.
The talks did not go well and the slum dwellers decided to take action. They also did not
allow the government to demolish their houses by contesting that it violated the right to
shelter. When the matter went to Court, the government contested that the slum dwellers
cannot prevent them from demolishing the illegal dwellings as they had waived their right to
do so. Is the government's argument valid?

(a) Yes, because they waived their right in exchange for a private benefit not contrary to
public policy or fundamental rights;

(b) Yes, because they waived their right in exchange for a private benefit contrary to public
policy or fundamental rights;

(c) No, because they waived their right in exchange for a private benefit not contrary to
public policy or fundamental rights;

(d) No, because they waived their right in exchange for a private benefit contrary to public
policy or fundamental rights.  

Passage (Q.-Q.): The Manipur high court has granted safe passage to seven persecuted
citizens from Myanmar, including three minors, to seek protection of the United Nations
High Commissioner for Refugees (UNHCR) in New Delhi by recognising the right of non-
refoulement. As underlined by the High Court, ‘non-refoulement’ is a widely accepted
principle of International Law that provides a refugee or asylum seeker a right against
expulsion from the territory in which he or she seeks refuge. It bars the forcible return of a
persecuted asylum seeker to the country of origin where he or she faces a threat to life or
freedom because of race, religion, nationality, membership in a social group, or political
opinion. Moreover, the court affirmed the findings of the Supreme Court in Louis De Raedt
v. Union of India and State of Arunachal Pradesh v. Khudiram Chakma to observe that even a
foreigner is entitled to the right to life and personal liberty under Article 21. On non-
refoulement, the Gujarat High Court has held that there is substantial, if not conclusive,
authority that the principle is binding on all States, independently of specific assent of the
Refugee Convention. The Delhi High Court in 2015 had also extended protection to
Myanmar’s refugees against immediate deportation by reading non-refoulement as a part of
Article 21, subject to a national security exception.

. Ronny’s country was war-torn as different factions and ideologies of the government had
divided the population. As a result, there was constant destruction and threat to life due to the
violence. Ronny decided to escape his country by sea as all the other modes had strict
government surveillance. Over the course of three days, he made a boat with the help of 0
packets of Lay’s chips. The packets had so much air in them, that the makeshift raft was able
to float even after it was subjected to the weight of a human being. After 2 weeks of sailing,
Ronny reached Italy. The Italian Coast Guard refused to permit Ronny to enter the country. Is
this action valid?

(a) Yes, because countries can refuse permission to uninvited foreigners who arrive by way
of sea;

(b) Yes, because Ronny was not a persecuted asylum seeker as he did not face a threat to life
or freedom because of race, religion, nationality, membership in a social group, or political
opinion.

(c) No, because Ronny was a persecuted asylum seeker who faced a threat to life or freedom
because of race, religion or nationality,

(d) No, because Ronny was a persecuted asylum seeker who faced a threat to life or freedom
country was at war due different factions and ideologies of the government

. Malai lived in Afghanistan at a time when the country was in the control of Taliban, a
violent terrorist group. She was extremely passionate about education and loved to go to
School. However, the Taliban were anti-female education and did not believe that girls need
to educated. One day, they caught Malai going to school and shot her 13 times. Malai
survived the attack but the family feared their safety. If they seek refuge in Europe, can they
be denied asylum?

(a) No, because Malai’s family faced a threat to their life if they were to continue staying in
Afghanistan;

(b) No, because the principle of non-refoulement provides a refugee or asylum seeker a right
against expulsion from the territory in which he or she seeks refuge;

(c) Both (a) and (b)

(d) None of the above

. Tiger was a RAW agent from India and Zoya was an ISI agent from Pakistan. In a mission
abroad, both of them fell in love with each other, even though they were from different
countries and had fundamentally different beliefs. After their mission was completed, the two
decided to marry each other and settle in India. However when they landed in India, Zoya
was taken away by RAW officials who were worried that a Pakistani ISI agent had been
made privy to confidential information. Zoya was locked in a cell for 7 days, without being
allowed to meet Tiger or anyone from her own country. Is this a violation of the principles
mentioned in the passage?  

(a) Yes, it is violation of non-refoulement as Zoya is being expelled from India;


(b) Yes, it is a violation of Article 21 as Zoya is entitled to the right to life and personal
liberty;

(c) Yes, it is a violation of non-refoulement which provides that an asylum seeker has a right
against expulsion from the territory in which he or she seeks refuge;

(d) No, it is not a violation of the principles mentioned in the passage.

. Lakhs of refugees tried to escape Mangoland and seek refuge in Appleland. The refugees
belonged to religion X and were a minority in Mangoland. The people of Mangoland did not
like members of religion X. As a result, there was large scale violence against members of
religion X, irrespective of their age or gender. Consequently, they escaped the country and
fled to Appleland. The government of Appleland seeks to send these refugees back to
Mangoland because they do not have any space for them in their country and would not be
able to afford their living. Can the refugees be deported to Mangoland?

(a) Yes, because non-refoulement bars the forcible return of persecuted asylum seekers to
their country of origin;

(b) No, The refugees cannot be deported to Mangoland because they are being persecuted in
their country of origin on account of their religion.

(c) Yes, because even foreigners like members of religion X are entitled to right to life and
personal liberty under Article 21;

(d) No, because non-refoulement can be read into Article 21 only if it does not go against
national security and members of religion X threaten the security of Appleland.

Passage (Q.-Q.): As per the copyright law in India, governed by the Copyright Act of 17, the
creators who produce musical, dramatic, artistic or literary works and the producers of sound
recordings or cinemas have an exclusive right to protect their work known as copyrights.
These rights may include the rights of adaptation, reproduction, communicating to the public
and translation of the work of such a creator/producer. All these rights when clubbed together
take the form of copyrights, however, the intensity at which these rights shall be applicable
depends on the work of the creator/producer. However, just making a few changes to the
original work of an owner to come off as an original creator does not mean that the copyright
infringement can be escaped. As important as it is to grow your business, it is equally
important to protect it and one such way is to create a trademark for it. The trademark not
only protects your business, it also increases brand recognition and goodwill. In the case of
Hearst Corporation v Dalal Street Communication Ltd., it was held that a trademark is said to
be infringed when a person for the purpose of his business uses a mark that is deceptively
similar or identical to the trademark of another company’s registered trademark.

. Starbucks was a famous coffee chain which served different varieties of coffee and other
snacks. It was extremely popular and had become a beloved brand since its inception ten
years ago. In light of this, Starbucks had registered for a trademark over its name. Recently, a
new café chain opened up in the same city by the name of “Sardar Baksh”. This new chain
also served coffee and other snacks, besides other menu options which were not available in
Starbucks. Can Starbucks sue Sardar Baksh for trademark violation?

(a) Yes, because the trademarked logo of Sardar Baksh is deceptively similar to Starbucks
and could confuse the customers;

(b) Yes, because the trademarked name of Sardar Baksh is not deceptively similar to
Starbucks and could confuse the customers;

(c) Yes, because the trademarked name of Sardar Baksh is deceptively similar to Starbucks
and could confuse the customers;

(d) Yes, because the trademarked logo of Sardar Baksh is not deceptively similar to
Starbucks and could confuse the customers.  

. Harry Potter is a tale of magic and wizardry. After the book series was published in 19, the
first movie of the franchise came out in 2002. Over the years, Harry Potter has become a
beloved brand with a registered copyright and trademark. In 2005, Mirchi Movies released a
film called ‘Hari Puttar’. The plot of the film revolved around a young Punjabi boy who lived
with his family in United Kingdom. However, when his family went for a trip to France, they
forgot Hari at home. The film followed Hari’s adventures as he tried to protect his house
from getting robbed. Can the Harry Potter Franchise file a case of copyright and trademark
infringement against Hari Puttar?

(a) No, there may be a copyright infringement by Hari Puttar but not a trademark
infringement,

(b) No, there may be a trademark infringement by Hari Puttar but not a copyright
infringement;

(c) Yes, there is both copyright and trademark infringement by Hari Puttar;

(d) No, there is neither a copyright nor a trademark infringement by Hari Puttar.

. Zara was a popular international fashion brand. Young people loved the brand so much that
they would buy their clothes from Zara, even if they were extremely expensive. A few years
later, a new restaurant in Delhi opened up and it was named after the renowned fashion
brand. Although Zara restaurant had no involvement with the Zara fashion brand or fashion
in general, the fashion brand sued the restaurant for infringing its trademark by using the
same name. Will Zara succeed in its suit?

(a) Yes, because the name of the restaurant and that of the fashion brand is identical;

(b) Yes, because the name of the restaurant and that of the fashion brand is deceptively
similar;
(c) No, because the name of the restaurant and that of the fashion brand is not deceptively
similar;

(d) No, because the name of the restaurant and that of the fashion brand is not identical.

. Godaddy.com was a popular website provider. The brand became so well known that nearly
50% of all new customers making a website for their companies used Godaddy.com to
purchase and create a domain name. To compete with Godaddy.com, X created a new
company to provide websites at much cheaper prices. He named his company,
‘Gomommy.com’. Can Godaddy.com sue Gomommy.com for trademark infringement?

(a) Yes, because Gomommy.com may confuse the customers of Godaddy.com

(b) Yes, because Gomommy.com will confuse the customers of Godaddy.com

(c) No, because Gomommy.com cannot confuse the customers of Godaddy.com

(d) None of the above

Passage (Q.-Q.): Article 13 aids the court and citizens to keep the powers of the legislature
under preview. Article 13 of the Indian Constitution describes the means for judicial review.
It enjoins a duty on the Indian State to respect and implement the fundamental right. And at
the same time, it confers a power on the courts to declare a law or an act void if it infringes
the fundamental rights. Article 13 is the guardian of Rights mentioned in Part III of our
Constitution. Clause (1) of the said Article emphasises that all inconsistent with the existing
laws become void from the commencement of the Constitution. As per the Supreme Court,
Article 13(1) does not make existing laws inconsistent with the fundamental rights, or void ab
initio, for all purposes. But Article 13 makes such laws’ ineffectual and void’ prospectively
based on its exercise of fundamental rights on and after the commencement of the
Constitution. Therefore, if a law becomes void from the applicability of the Constitution, as it
is inconsistent with the rights imbibed in part III, it will still apply to citizens and non-citizens
who committed an offence under the said law. Article 13(2) of the Indian Constitution says
that the State must not make laws that are not in ‘consonance’ with the Constitution. And if
the law drafted intervened with an individual’s fundamental rights, the said law will become
void to the extent of the contravention. This is an express ground for judicial review after
legislation.

. The 15 Education Act inter alia provided that children belonging to caste X will not be
granted admission. Although the Act had several good provisions to promote and encourage
education in India, this one particular provision was extremely discriminatory. When the
Constitution came into effect in 10, Article 14 came into force. Article 14 guaranteed all
persons with the right to equality and prohibited discrimination. In light of these facts, does
the 15 Act become void?

(a) The 15 Education Act became void in 17 when India became independent;

(b) The 15 Education Act became void in 10 when the Constitution came into effect and
Article 14 came into the picture;
(c) The 15 Education Act was void ab initio or void from the beginning because it was in
violation of a fundamental right;

(d) None of the above.

. The Government of Gujarat sought to increase industrial development and economic


activity in the state. As a result, the Government passed a new law that did not allow workers
in a factory producing cement to form trade unions. The motive behind this law was to
encourage businessmen to set up industries in the state, without being deterred by possible
legal hassles. The workers protested by saying that it is a violation of Article 19(1)(c) of the
Constitution which provides that, “All citizens shall have the right to form associations or
unions”. The Government contested their argument. Is the Government’s contention valid?

(a) Yes, because the new law is in consonance with the Constitution as it sought to increase
industrial development and economic activity in the state;

(b) No, because the new law is not in consonance with the Constitution since it violates
Article 19(1)(c);

(c) No, because the new law is not in consonance with the Constitution since it violates the
right to equality of other business owners who employ workers;

(d) Both (b) and (c).

. The legislature propagated a New Education Policy (NEP) in a bid to improve the literacy
rate and quality of education in the country. One of the rules of NEP observed that education
in institutions like madrasas and shakhas, where children were schooled in religious
knowledge, were outdated. Further, such institutions provided little to no practical knowledge
and did not adequately prepare children to compete with those students who were taught in
the regular school system. As a result, the rule prohibited education through such institutions.
Can this rule be challenged?

(a) Yes, because it is contrary to the right to freedom of religion and not in consonance with
the Constitution;

(b) No, because it promotes right to education which is a part of right to life and in
consonance with the Constitution;

(c) No, because it promotes the promote right to equality and in consonance with the
Constitution;

(d) Both (b) and (c).

. Indi and Pak were two countries at war against each other. In this context, in order to avoid
any information from reaching Pak about the policies being adopted in Indi and to present a
unified Indi front, the Government of Indi passed a new law that prohibited all media houses
from publishing anything related to the Government or its policies. This prohibition was
temporary in nature, and only expected to last till the war persisted. A newspaper challenged
this decision to be in contravention of Part III of the Constitution. Based on your reading of
the passage, is this law void?

(a) Yes, because it violates the Article 19(1)(a) which provides that “All citizens shall have
the right to freedom of speech and expression”;

(b) No, it is only partly void because it is parity in consonance with the Constitution;

(c) No, because the prohibition was temporary in nature and such reasonable restrictions are
in consonance with the Constitution;

(d) None of the above.  

Passage (Q.-Q.): On 3 June, the Supreme Court quashed the sedition case filed against
veteran journalist Vinod Dua, filed by a minister in the Central Government for being
criticized for the implementation of safety guidelines. While providing relief the SC said that
even journalists should benefit from the high threshold of sedition set in the Kedar Nath case.
In the case the Supreme Court was hearing a question that arose in criminal appeals filed by
persons including Kedar Nath: Is S.124A of IPC (laying down the offence of sedition)
violative of the fundamental right to free speech (Article 19(1)(a))? Kedar Nath was
convicted under section 124A IPC. He subsequently moved an appeal before the Patna HC
which was rejected. Therefore, he finally moved the SC. In the appeal, he challenged the
constitutional validity of S.124A. The Supreme Court very categorically clarified that Section
124A penalizes only that speech which is intended or has the tendency to create public
disorder or incitement to violence. While upholding the constitutional validity of the sedition
law, the SC said that s.124A “strikes the correct balance” between individual’s fundamental
rights and the interest of public order. “Criticism of public measures/comment on government
action, would be within reasonable limits and would be consistent with freedom of speech
and expression. It is only when the words, written or spoken, etc. having the tendency or
intention of creating public disorder or disturbance of law and order that the law steps in in
the interest of public order.” Source: https://www.thequint.com/news/law/getting-to-know-
the-kedar-nath-case#read-more

. Vikram is an author of a very renowned book ‘Scam Country’ which discusses the
propaganda schemes that various governments have been accused of formulating over the
years. This book of his is banned in India however; Vikram claims to have not named any
particular person in his book but has referred the governments by their ruling periods and
their speeches. Javed comes into possession of this book through backdoor means and upon
reading it, he starts preaching that the ruling government should be overthrown and that
‘Lord Vikram should be given the Throne!’ This causes 400 people to riot in front of the
parliament and disrupts the traffic network for 2 days straight. Could Javed be held liable for
sedition?

(a) Javed cannot be held liable as he was only preaching what Vikram wrote.

(b) Javed should be held liable as his speech led to the riots.
(c) Javed should be held liable as he was in possession of an illegal substance that led to the
riots.

(d) Vikram should be held liable as he was the one who published such seditious views.

. In the above question, had there been no riots, but only peaceful protests urging the
parliament to resign as ‘the people have lost faith in them’. Would Javed be liable for inciting
sedition?

(a) Javed would be liable as he still made contentious statements on the government and
incited protests.

(b) Javed would not be liable as the protestors did not disrupt any public or lawful machinery.

(c) Javed will not be liable as Vikram was the one who wrote the comments on the
government

(d) Javed will be liable as book was banned for good reasons, he flouted the laws, thereby
disrupting public order.

. Moira and Johnny Rose are the party leaders of JZB, a labour union party made in
furtherance of communist beliefs in India. Alexis and David are two journalists who work at
The Nightly Bugle as on the road correspondents who specialized in political journalism.
Seeing that the JZB was on a rise, they both decided to interview the party leaders since the
elections were almost upon them. Upon meeting for the interview they began asking Johnny
and Moira questions, to which, they gave controversial answers, often lingering on the
bounds of criticizing the Central government in power in a rude manner. When the interview
got published in the newspapers, the supporters of the Centre lit up fires in several newspaper
shops and grocery stores as an act of aggression, claiming that they want to oust the JZB from
the country. Could this act of the aggressors be termed as sedition?

(a) These acts of the aggressors can be termed as being seditious as they were inspired by the
vile comments of the JZB.

(b) These acts cannot be termed as seditious as they were not aimed at disrupting public
order.  

(c) These acts cannot be termed as being seditious since the JZB did not make comments
about overthrowing the government.

(d) These acts can be termed as seditious since they were incited by comments from the
opposition party.

. In the above question, had Moira and Johnny Rose presented their criticisms and complaints
in an organized and respectful manner, but at the same time towards the end of the interview,
made an emphatic speech asking the citizens of the country to rush to the streets and take
what was rightfully theirs since the all systems were rigged; would Moira and Johnny be held
guilty for such an act?

(a) No, as they voiced valid criticisms in a respectful manner.

(b) Yes, they would be held guilty for only the last part of the interview.

(c) Yes, as the act was seditious.

(d) Yes, as their act had a tendency to create public disorder, and they were liable of sedition
under S.124.

39
Passage (Q.64-Q.68): The term foreign judgments means the judgment of a foreign court, and
Section 13 of the code provides the criteria for enforcement of such judgement in the Indian
Court. There are three primary sources of law about the enforcement of foreign judgments in
India:

· Laws passed by the parliament: Section 44A of the Code explains the constitutional
principle that the judgment handed down by a Superior Court of the Reciprocating Territory
is implemented in India as though it were a ruling enacted by the Indian District Courts. That
being said, a judgment emerging from a non-reciprocating territory cannot be immediately
mandated in the same way and a fresh case should be issued for its regulation in which such a
judgment is of only evidentiary value.

· Bilateral treaties with the reciprocating countries concerning recognition and enforcement
of foreign judgments to which India is a party; and

· Judicial precedents: A foreign judgment not emanating from a superior court of a


reciprocating territory cannot be executed in India without the filing of a new suit in which
the said judgment has only evidentiary value. One of the fundamental requirements of
recognition is that a foreign judgment must not be inconclusive under the Code. According to
Section 13 of the Code, a foreign judgment will be inconclusive if it:

· is pronounced by a court that was not of competent jurisdiction;

· is not given on the merits of the case;

· appears to be founded on an incorrect view of international law or a refusal to recognize


Indian law (where applicable);

· violates principles of natural justice;

· is obtained by fraud; or

· Sustains a claim founded on a breach of Indian law.

64. Rakesh, an Indian National, is a trustee to property of Manoharam, a Srilankan National


in India for about ten years. India and Srilanka have the bilateral treaty between them for the
enforcement of judgment since 2004. Recently, a war started between both the countries and
Srilanka has been declared an enemy State. Rakesh decided to sell the property as the
trusteeship agreement stands suspended due to such circumstance. Manoharan sought the
injunction passed by the Superior Court of Srilanka. He filed an application before the Civil
Court in India to enforcement the injunction as Srilanka is the Reciprocating Territory.
Rakesh argued that the decree cannot be enforced Srilanka has been declared as enemy State.
Can Manoharan seek the enforcement the decree for injunction before the Indian Civil Court?

(a) Yes, because the bilateral treaty was entered into prior to the occurrence of war

(b) Yes, because the trusteeship agreement was signed prior to the occurrence of war

(c) No, because Srilanka has been declared as enemy as all the contract stands suspended

(d) No, Srilanka is not a Reciprocating Territory

65. Mohanlal and Sarita married in Chennai in 2008, in compliance with the Hindu Marriage
Act. The couple last settled in New Zealand together and then the husband moved to
Australia. Later, in 2014, he filed a petition for the dissolution of marriage in Australia and
secured an irreversible breakup of marriage decree by legally meeting the condition of days
of residency in Australia. The wife had made it known that she would not consent to the
Australian Court’s jurisdiction. She lodged a criminal lawsuit against the husband before the
Family Court, Chennai for bigamy after he married other woman. Can the Court enforce the
foreign decree assuming the fact the Australia has the Reciprocating Territory in the Gazette
of India?

(a) Yes, the foreign decree is admissible in the Indian Court as Australia is a Reciprocating
Territory

 (b) Yes, because the parties have complied with the conditions prescribed under the
Australian law for seeking the decree for dissolution of marriage

(c) No, because the marriage can be dissolved only as per Hindu law

(d) No, because the marriage took place in Chennai as per the Hindu law, hence, Indian
Courts have the jurisdiction for the dissolution of marriage

66. Aramco, a Saudi Company, entered into a contract on February 17, 2017 with Oil and
Natural Gas Commission (ONGC), a Government of India undertaking, for carrying out
structural oil drilling in India. The contract provided that amount payable to the Aramco on
fee or any default etc. Shall be computed in Riyal and the ONGC shall pay % of that amount
in Riyal and remaining in Indian Rupee using a fixed conversion rate. The disputes and
differences, which arose between the parties, were referred to arbitration. The award made on
May 28, 2020 directed certain payments to be made in Riyal, but did not specify the rate of
exchange for conversion into Indian Rupee. The Delhi High Court passed a decree on
January 2021 in terms of the award simpliciter without fixing any date for conversion of the
Riyal into Indian rupees. Decide as to which date shall be taken into account to compute the
conversion rate?
(a) The date of the entering into contract shall be taken as conclusive date to decide the
conversion rate

(b) The date of the award given be the Umpire in arbitration shall be taken as conclusive date
to decide the conversion rate

(c) The date of the decree passed by the Delhi High Court shall be taken as conclusive date to
decide the conversion rate

(d) The decree passed by the arbitration is not conclusive as per Section 13 of the CPC

67. Anurag, an Indian and Swetlana, a Dutch, married in 2017 in Pune under the Hindu Law.
Prior to the marriage, they signed the pre-nuptial agreement in the Netherlands, wherein it
was decided that in case of default by one party, the other party has the right to dissolve the
marriage unilaterally. Swetlana was granted the decree to dissolve the marriage by the
Appellate Court of the Netherlands. Anurag filed an application for the injunction before the
Family Court, Pune. Can the Family Court enforce the decree passed by the Supreme Court
of the Netherlands, considering that the Netherlands is not a Reciprocating Territory?

(a) The Court can enforce the decree because it is valid as per the Netherlands law

(b) The Court can enforce the decree because the parties mutually entered into the agreement

(c) The Court cannot enforce the decree because the agreement is against the public policy of
India

(d) The Court cannot enforce the decree as the Family Court Pune has the jurisdiction to
decide on the validity of the marital agreement i.e. the pre-nuptial agreement

68. Aaditya works as an Associate in the Linklaters LLP, a multinational law firm in London.
The firm has claimed 4000 £ have been misappropriated by Aaditya while in the employment
of the firm, before the High Court London. As Aaditya was in India at that time, did not
appear at the hearing. The High Court passed a decree based on the documentary evidence
presented before the Court. The firm filed the enforcement suit in the Delhi High Court
against Aaditya. Decide will the firm succeed in the enforcement of the decree, considering
the fact that the UK is the Reciprocating Territory?

(a) Yes, because the decree was passed by the highest Court in London

(b) Yes, because the decree was passed by the Court after perusing appropriate evidence

(c) No, because the decree is not conclusive in nature

(d) No, because the firm had the higher bargaining power

 
 

Passage (Q.-Q.): On 9th April 2021, the Kerala High Court in Treasa Josfine v. State of
Kerala directed the State authorities to consider an application submitted by the petitioner (a
female trainee engineer) for the post of a Safety Officer in Kerala Minerals and Metals Ltd,
on the grounds that she had been denied opportunity on the basis of her sex. The counter
affidavit filed by the State argued that the notification was in compliance with Section 66(1)
(b) of the Factories Act, 18. Section 66(1) (b) states “no woman shall be required or allowed
to work in any factory except between the hours of 6 A.M. and 7 P.M”. The submission of
the State was that the post of a Safety Officer required the person so engaged to work round
the clock, even during the night if required. Therefore, women could not be hired for this
position under the provisions of the Factories Act. In Treasa Josfine, Justice Anu Sivaraman
agreed that Section 66(1) (b) is a beneficial provision intended to protect women. However,
the Court observed that the Factories Act was enacted at a different time and in a different
socio-economic context, particularly with respect to the roles played by women in society.
Given this context, Section 66(1) (b)’s force could only be utilized to protect women, but
would not constitute a reason to deny them engagement and opportunity if they are fully
qualified [paragraph 15]. On this basis, the Court set aside the notification and held it to be
violative of Articles 14, 15 and 16. Notably, states such as Maharashtra and Kerala have
altered the position under Section 66(1)(b) by permitting the employment of women post 7
p.m. provided that all safety and security safeguards are met by the employer.

. Which of the following inference can be drawn be perusing the information given in the
passage?

(a) Section 66 (1) (b) is per se in contravention to Article 14, 15 and 16 of the Constitution

(b) The enactment of provisions, such as, Section 66 (1)

(b), was very much essential for socio-economic development of women in 18

(c) The Court Order leaves the employment of women entirely to the option of the employer,
but does little to incentivize them

(d) The State must identify and attempt to remedy specific forms of disadvantage

. A statute was passed by the state of Jinjibar in the Union of Mastana that prohibits women
from working for more than 10 hours a day, keeping in view of their performance of maternal
functions and household responsibilities. A group of male employees challenged the Statute
on the ground of violation of Article 14, 15 and 16. Considering that the laws of the Union of
Mastana are pari materia to the laws of the Union of India, which of the following could be
the most appropriate argument in favour of the Statute?

(a) The Statute would ensure that the State must identify and attempt to remedy specific
forms of disadvantage

(b) The Statute mandate safeguards and security for women at the workplace without
removing them from the workplace altogether
(c) Women have to devote time to the home, because it further entrenches the public-private
divide.

(d) Women holding household duties is not a universal phenomenon

. Supposedly, the state government of Punjab has made an amendments in the Section 30 of
the Punjab Excise Act, 14 that prohibits employment of "any man under the age of 25 years"
or "any woman" in any part of such premises in which liquor or intoxicating drug is
consumed by the public. Ramesh, a member of Hotel Association of India challenged the
validity of the said provision on the basis of violative of Articles 19(1) (g), 14 and 15 of the
Constitution of India. Decide the validity of the provision as per the legal information given
in the passage.

(a) The provision is constitutional under the law as it seeks to protect the vulnerable groups in
such kinds of services

(b) The provision is unconstitutional under the law as it does not have reasonable nexus with
the objective of the provision

(c) The provision is unconstitutional under the law as it prohibits the women from seeking
employment in hotel industry

(d) The provision is constitutional under the law as it does have reasonable nexus with the
objective of the provisions

. In the Union of Shahistan, there is a law i.e. Air Shahistan Employees' Service Regulations;
and Regulation 12 (1) (a) of Air Shahistan Employees' Service Regulations provides that the
services of the Air Hostesses, except the cabin crew members of similar status, would stand
terminated on first pregnancy. Considering the fact that the law of the Union of Shahistan is
pari materia to the Union of India, decide the constitutionality of the Regulation keeping in
view the kinds of requirements pertaining to the services of an airhostess.

(a) The Regulation is constitutional having regard to the nature of job functions, the mode of
recruitments of Air Hostesses, their qualifications, the Air Hostesses fall within a category
separate from the other

(b) Te recruitment of the Air Hostesses is actually sex based recruitment made not on the
ground of sex alone but swayed by a lot of other considerations and hence it is Constitutional
in nature

(c) The Regulation is unconstitutional as The impugned provisions appear to be a clear case
of official arbitrariness

(d) The Regulation is unconstitutional as it is a hostile discrimination and not reasonable


classification

. Smt. R. Vasantha, who is employed in a textile mill and is willing to work during day time
or at night time on shift basis, depending on the workload. The Mill owner has informed her
that the management will render dormitories for comfortable stay and food within the factory
premises, transport if necessary and other facilities to enable the women employees to work
in the night shifts. However, due to the operation of Section 66(1) (b) of the Factories Act,
18, she is unable to carry on her chosen employment between 7.00 P.M. and 6.00 A.M. and
hence she wants to challenge Section 66(1) (b) of the Factories Act, 18. Support her with
requisite arguments as per the legal information given in the passage.

1. The provision discriminates the factory workers based on sex and gender and such gender
discrimination is arbitrary and without any basis

2. The said piece of legislation is one of the instances of romantic paternalism by which men
wanted to restrict women to household activities so that they can retain their economic
superiority in the homes

3. Good intent or absence of discriminatory intent does not immunize an employment


practice from the equal opportunity requirement under Articles 15 and 16 of the Constitution
of India

(a) All of the above

(b) Only 1 and 3

(c) Only 1 and 2

(d) Only 2 and 3

Passage (Q.-Q.): A group administrator cannot be held vicariously liable for an act of a
member of the group, who posts objectionable content, unless it is shown that there was
common intention or pre[1]arranged plan acting in concert pursuant to such plan by such
member of a Whatsapp group and the administrator. When an online platform is created, the
creator thereof cannot expect any of the members thereof to indulge in defamation and
defamatory statements made by any member of the group cannot make the Administrator
liable therefor. It is not as if without the Administrator's approval of each of the statements,
the statements cannot be posted by any of the members of the Group on the said platform. An
admin, by merely creating a group, does not publish or transmit material over the internet.
Thus, the Court drew a distinction between a mere group admin and an intermediary under
the Act. The term "intermediary" refers to any person who, on behalf of another person,
receives, stores or transmits an electronic record or provides any service with respect to that
record. Generally, the admin of a group does not store or receive electronic records on behalf
of another. Rather, it is WhatsApp itself which is arguably an intermediary as it acts as a
medium between two or more persons. (Extracted with requisite revisions and edits from
‘Can the administrator of a WhatsApp group be held liable for objectionable posts?’
https://www.barandbench.com/columns/administrator-whatsapp-group[1]liable-
objectionable-posts)

. Suresh created a whatsapp group for the preparation and discussion of CLAT related topics.
Various students were added in that group. One day, during one such discussion, the
discussion got intense and one Raju passed some sexually colored remark along with some
obscene stickers against another group member, Megha. Mr. Suresh observed all this but did
not object to the remarks made by Raju and later even after several requests by Megha, failed
to remove him from the group. Can he now be held liable for the act of Raju?

(a) Yes, he will be liable because he failed to remove Raju from the group despite several
requests of Megha after he passed colored remarks against her.

(b) Yes, since he did not object to such colored remarks by Raju, it indicates the presence of
common intention.

(c) No, he was neither involved in making such colored remarks nor supporting them when
passed by Raju.

(d) No, he cannot be held liable for acts of Raju because there was no common intention.

. Two day ago, Raju and Suresh were studying and during the discussion they discussed the
arrogance of Megha and her habit/ want of always being correct and certain colored remarks
were also made. Two days later, the incident of Raju passing such remarks happened. Is
Suresh liable for the content posted?

(a) Yes, his discussion two days ago indicates the presence of common intention.

(b) Yes, his discussion two days ago indicates the likelihood of existence of a pre-arranged
plan.

(c) No, there was no common intention shared by Raju and Suresh for the comments passed
or content posted in the group.

(d) No, he is merely the admin of the group and created the group in good faith.

. One of the stickers being shared by Raju in the group was previously shared in a personal
chat with Raju by the administrator of the group Suresh and hence it was argued that Suresh
is also acting as an intermediary in this situation and would be liable. Is the argument correct?

(a) Yes, Suresh has acted as the intermediary as the stored the sticker and shared it with Raju.

(b) No, Whatsapp is acting as the intermediary and not Suresh.

(c) Yes, Suresh has acted as the intermediary between Raju and Megha.

(d) No, Raju is the intermediary as he transmitted the sticker from Suresh to Megha.

. Bombay Police Act was recently amended to provide under Section 68 that “all persons
shall be bound to conform to the reasonable directions of a Police officer given in fulfillment
of any of his duties under this Act.” Directions against the whatsapp admin on posting of
objectionable content by members of the group were issued by the police officer. Would such
directions be deemed reasonable?

(a) No, neither the group administrator’s approval is required nor he has any control over the
statements posted in the group by the participants.
(b) Yes, since the Bombay Police Act is enacted by the parliament it shall be deemed to have
been well contemplated and deliberated and thus be valid.

(c) No, the directions are unreasonable and actions are required to be undertaken against such
excruciating and ill-thought out directions.

(d) Yes, the Bombay Police Act has empowered the police to issue any directions as deemed
appropriate by them.

Passage (Q.-Q.): It is clarified that the fact of the petitioner having converted to Islam would
not be a relevant factor while ensuring that there is no interference in the liberty of the
petitioners unless there is any allegation by petitioner regarding forcible conversion. We,
therefore, direct that the administration/police authorities throughout the country will see to it
that if any boy or girl who is a major undergoes inter-caste or inter-religious marriage with a
woman or man who is a major, the couple are not harassed by any one nor subjected to
threats or acts of violence, and anyone who gives such threats or harasses or commits acts of
violence either himself or at his instigation, is taken to task by instituting criminal
proceedings by the police against such persons and further stern action is taken against such
persons as provided by law. It is also clarified that petitioners, if major, are entitled to live
together even if not married and, therefore, proof of their marriage would not be required and
shall not be insisted upon by the concerned Police Officer. (Extracted with requisite revisions
and edits from ‘Can the administrator of a WhatsApp group be held liable for objectionable
posts?’ at https://www.barandbench.com/news/litigation/conversion-islam[1]irrelevant-grant-
protection-couple-allahabad-high-court)

. Mr. Parvez (initially named Mr. Parma Chauhan) married Ms. Zoya Qureshi after
converting into Islam. They had initially eloped as their family was against these inter-
religious ties. Parma later converted into Islam and their nikah (marriage) was solemnized
with all the Islamic rituals. They have been living together for about 6 months and then their
family got hold of their location. They started threatening them of their lives as their
relationship was affecting their family name. They approached the authorities for the
protection of their life and liberty. Are they entitled to any protection?

(a) Yes, it was a marriage between two adults and mere conversion is not relevant.

(b) No, there might be presence of forcible conversion which has not been ruled out and
could only be conclusively determined after some investigation is done.

(c) Yes, criminal proceedings should be instituted against the perpetrators.

(d) No, there were merely threats in the absence of any actual violence.

. It was later brought into light by the mother of Parma Chauhan that there was forcible
conversion involved and hence the police authorities are not obligated to grant them any
protection. The Police agreed to the argument of her mother that forcible conversion was
involved denied the protection sought by the couple. Is the act of police justified?
(a) Yes, there is an allegation of forcible conversion of Mr. Parma.

(b) No, allegation of forcible conversion has not been made by Mr. Parma.

(c) Yes, the marriage is between adults who should be granted protection and conversion
whether forcible or not is immaterial.

(d) No, the police are under an obligation to grant protection to all adults irrespective of the
existence of a valid marriage.

. It was revealed that the marriage was contracted without the requisite number of witnesses,
which is an requisite under the Islamic laws and thus making the marriage irregular (also
called fasid). Hence, when they sought aid from the police to protect their interests and
liberty the police denied any protection as their marriage was not recognized even by their
religion. Is the act of police justified?

(a) Yes, since the marriage is irregular according to Islamic laws.

(b) No, the protection is to be granted to every major and they are allowed to live together
regardless of marriage.

(c) Yes, the police are under an obligation to protect only those who have been under a proper
wed-lock and their liberty is at stake.

(d) No, the protection has to be granted to every individual as it has been mandated through
various judicial pronouncements.

. They were granted protection at that time and 7 years passed after their marriage and they
started living in another locality. There was however certain threats by their new locality
once they got to know that there was an inter-religious marriage between the couple. Zoya
approached the police to protect them since such threats and take criminal action against
those involved. Parma, this time, however admits that his mother was right in her argument
about the forced conversion and he did not speak about it due to the political associations of
Zoya. Would this new factor being disclosed by Parma, a relevant factor?

(a) No, there has already been a lapse of 7 years to raise any such contention.

(b) No, nobody can sleep over his rights.

(c) Yes, allegation of forcible conversion has been made by Parma and not anybody else.

(d) Yes, if the contentions of Mr. Parma turned out to be correct this will have a bearing on
his life and liberty.

Passage (Q.-Q.): Draft Rules released by the Supreme Court e-Committee on Monday for
live[1]streaming and recording court proceedings propose a 10-minute delay in transmission
and exclusion of communally sensitive cases and matters that involve sexual offences and
gender violence against women. Justice Chandrachud said the right of access to justice,
guaranteed under Article 21 of the Constitution, “encompasses the right to access live court
proceedings”. The Rules intend to balance between access to information and concerns of
privacy and confidentiality. Matrimonial matters, cases under the Protection of Children from
Sexual Offences Act (POCSO) and under the Juvenile Justice (Care and Protection of
Children) Act would also be exempted from livestream. The Bench can exempt, for reasons
recorded in writing, any case it considers antithetical to the administration of justice. Personal
information such as date of birth of parties, home address, identity card number, bank
account information, and the personal information of related parties, such as close relatives,
witnesses and other participants, will be deleted or muted during live-streaming. “No
person/entity (including print and electronic media, and social media platforms) other than an
authorised person/entity shall record, share and/or disseminate live-streamed proceedings or
archival data,” the Rules proposed. (Extracted with requisite revisions and edits from ‘Draft
Rules for live-streaming, recording of court proceedings out’ at
https://www.thehindu.com/news/national/draft-rules-for-live-streaming-recording-of[1]court-
proceedings-out/article3454.ece)

. Firm R&R was undergoing a dispute pertaining the assets of the company post its demerger
under the leadership of two of its founding partners. In the upcoming proceedings, there was
a requirement for the presentation of various documents providing the details of CSR
activities of the company, its balance sheets, and details of various tax related information all
of which has been mandated to be provided in the website of the company under the
Companies Act, 2013. Post Covid and in pursuance of the draft Rules, the Court decides to
live stream the proceedings. One of the partners opposed this but his request was denied by
the Bench. Was the decision of the Bench flawed?

(a) Yes, business confidentiality will apply to the said hearing.

(b) No, the discretion to exempt the live-streaming resides solely with the bench to cater to
the administration of justice.

(c) No, right to access of justice mandates the live streaming of the proceedings.

(d) No, the documents to be presented in the proceedings are already in the public domain
(website) and no business confidentiality will be applicable.

. In another case before the trial court, a renowned celebrity Mr. Golden khan was accused to
molestation of a minor and booked under POCSO Act. The court decides the same to be held
in-camera and no live streaming of the same. Mr. Khan, however, approached the court and
quoted Justice Chandrachud that “right of access to justice, guaranteed under Article 21 of
the Constitution, encompasses the right to access live court proceedings.” Will his argument
succeed?

(a) Yes, Article 21 being a Fundamental Right cannot be subject to any restrictions.

(b) No, the case involves the offence against a minor and POCSO Act’s applicability will
prohibit the live streaming of the same.
(c) No, it’s merely a statement by Justice Chandrachud and will not have any binding effect
on any court or its proceedings.

(d) Yes, administration of justice shall permit the live-streaming of such proceedings.

. Despite several attempts, the proceedings were recorded and leaked in an online platform. It
was however put down from the platform. Few days later, a twitter user and law student, Ms.
Sanjeevni posts a clip from the said leak. A suit is brought against her. Is she liable for breach
under the draft Rules?

(a) Yes, she is not an authorized person to record or share live-streamed proceedings.

(b) Yes, recording, sharing or disseminating of live-streamed proceedings is prohibited for


everyone.

(c) Yes, the clip shared involves sharing of sensitive matter.

(d) No, the recording and initial sharing was done by a third party and she has only re-shared
a part in the clip.

. One of the arguments given by Ms. Sanjeevni was on the premise that the purpose of her re-
sharing of the clip was educational involving a research on a particular field requiring the
survey into the opinions of twitter users and is not motivated by any commercial benefits.
Will her argument hold well?

(a) Yes, the re-sharing of the clip was not in any print/electronic media platforms.

(b) Yes, the re-sharing was merely educational and not commercial.

(c) No, she was an unauthorized person to share or disseminate the clip.

(d) No, it was illegal to share a sensitive matter involving POCSO Act.

Passage (Q.-Q.): “Ignorance is no justification for normalizing any form of discrimination”


said Justice Anand Venkatesh in a judgment which issued a slew of guidelines to ensure the
protection of LGBTQIA+ persons in consensual relationships from police harassment. The
Court observed that legislative changes are needed to eradicate the social discrimination
against LGBTQIA+ community and to ensure the protection of their life and dignity. In this
regard, the court mentioned how legislative interventions are changing the attitude towards
persons with disabilities and mental illness. The hostilities they face are due to the fact that
their relationship does not enjoy societal sanction. "...the actual problem is not the fact that
the law does not recognise a relationship but that the sanction that is accorded by the society
is not available. It is only for this reason, I strongly feel that the change must take place at a
societal level and when it is complemented by a law there will be a remarkable change in the
outlook of the society by recognising same-sex relationships. Till the legislature comes up
with an enactment, the LGBTQIA+ community cannot be left in a vulnerable atmosphere
where there is no guarantee for their protection and safety. This gap is now sought to be filled
in by way of issuing guidelines till law takes over and guarantees safety and protection.
(Extracted with requisite revisions and edits from 'My Upbringing Treated "Homosexual",
"Gay", "Lesbian" As Anathema' : Madras HC Judge Explains How He Overcame Prejudice
Against LGBTQIA+ Community at https://www.livelaw.in/top-stories/lgbtqia-homosexual-
gay-lesbian-madras-hc-anand-venkatesh[1]overcome-prejudice-1342) 24 of 40 . Ms. Eena
and Ms. Meena, a lesbian couple decided to elope from their parents who were unsupportive
of their relationship. They through the help of an NGO managed to obtain an accommodation
and were living there for about 4 months. However, their parents filed missing complaints of
those girls. The police tried to bring them to their families but they resisted the authorities.
They approached the Court for relief from what they called police harassment. Should the
Court grant them any relief?

(a) No, there exists no case of discrimination against the couple.

(b) No, the case in question is about the missing complaints of the girls and is independent of
any police harassment or discrimination.

(c) Yes, ignorance of law by the police and family is no excuse for harassment.

(d) Yes, the Court should ensure the protection of their life and dignity.

. The Landlord of their apartment filed a police complaint for their eviction as after giving
them notice for payment of rent of last 2 months they failed to comply. The couple however
alleged that this eviction amounts to discrimination on the ground of their sexual orientation
as the landlord was not favorable of same sex relationships. In a suit brought against such
eviction, will their argument hold well before the Court?

(a) Yes, they have been discriminated on the ground of their sexual orientation.

(b) No, there is no discrimination on the basis of their sexual orientation and lack of societal
sanction.

(c) Yes, their relationship lacked societal sanction from the landlord and the police.

(d) No, there is no police harassment against them.

. Before they found this accommodation, they were denied accommodation by multiple
people. One such person was Mr. Gangadhar who denied them the accommodation as
according to him even law doesn’t recognize same-sex relationship and hence even his
neighbors denied renting them accommodation. Is any liable for this conduct?

(a) Yes, Ignorance of law is no excuse.

(b) Yes, Ignorance cannot rationalize any form of discrimination.

(c) No, Societal sanction has already been ensured through legal frameworks and individual
discrimination is not a ground of relief.

(d) Yes, Section 3 IPC has been read down by the Apex Court.
 

. During their visit to a temple, they were denied to perform certain rituals as a couple as they
believed that it would anger the sacred deities and bring havoc on Earth and were thus
forcibly removed. A suit was brought against such discrimination. They pleaded their
independence to manage their affairs as a Freedom guaranteed under Fundamental Rights
which cannot be interfered with in the absence of any legislative frameworks existing. Will
their argument succeed?

(a) Yes, Freedom to manage religious affairs is guaranteed under Article 26 of the
Constitution.

(b) Yes, no legislative framework exists to ensure them the liberty to exercise such a
freedom.

(c) No, despite the absence of any legislation to this effect they cannot be left abandoned
without any protection and safety.

(d) No, society should evolve and give sanction to their relationship.

Passage (Q.-Q.): Section 16 of the U.P. Sugarcane (Regulation of Supply and Purchase) Act,
13 is not repugnant to Section 3(2)(c) of the Essential Commodities Act, 15 and Clause 3 of
the Sugarcane (Control) Order, 16 as, as observed hereinabove, the price which is fixed by
the Central Government is the “minimum price” and the price which is fixed by the State
Government is the “advised price” (or remunerative price) which is always higher than the
“minimum price” fixed by the Central Government and therefore, there is no conflict. It is
only in a case where the “advised price” fixed by the State Government is lower than the
“minimum price” fixed by the Central Government, the provisions of the Central enactments
will prevail and the “minimum price” fixed by the Central Government would prevail. So
long as the “advised price” fixed by the State Government is higher than the “minimum
price” fixed by the Central Government, the same cannot be said to be void under Article 254
of the Constitution of India. (Extracted with requisite revisions and edits from ‘State
Government cannot fix the “minimum price” of sugarcane once Centre has already fixed it’ at
https://www.scconline.com/blog/post/2020/04/24/state[1]government-cannot-fix-the-
minimum-price-of-sugarcane-once-centre-has-already-fixed-it/)

. Central Government having exercised the power under Entries 33 and 34 List III of seventh
Schedule fixed the “minimum price” of jute at Rs. 6000/ kgs. However, the State
Government owing to the pandemic decided to decrease the minimum price to Rs. 5500/ kgs.
The same was brought into question before the court of law. Will the action against the State
Government succeed?

(a) Yes, advised price cannot be lower than the minimum price fixed by the Central
Government.

(b) No, by virtue of List III, State Government is empowered to fix the prices.

(c) No, State Government has fixed the price on the basis of pandemic.
(d) Facts are insufficient.

. State Government of UP set the price of sugarcane @ Rs 35/ Kg which was lesser than the
price set by the Central Government. They argued that under since it is a subject under
Concurrent list; it is at an equal pedestal in setting the minimum price for the sugarcanes and
their pricing shall prevail. Will their argument succeed?

(a) No, advised price cannot be lower than the minimum price fixed by the Central
Government.

(b) Yes, List III empowers the State Government to fix the prices.

(c) No, Central Government has the monopoly in fixing of prices of commodities like
sugarcane.

(d) No, Prices fixed by Central Government cannot be altered by the State Government.

. State Government of UP argued that there exist a repugnancy under List III (as well as
under the State Laws) as both Centre and State can set the minimum price at which sugarcane
can be brought or sold. Is there argument True or False?

(a) True.

(b) False.

(c) Partially true.

(d) Cannot be determined.

. The remunerative price of sugarcane was set @ Rs 40/ Kg for the season. However, later the
Central Government decided to increase the minimum price to Rs 50/ kg due to the failure of
crops in many parts of the country. The State Government of UP also changed the price to
Rs. 45/ Kg. It was disputed that since the price notified by the State Government is through a
later notification, such a pricing should prevail over the minimum price set by the Central
Government. Is this argument correct?

(a) Yes, the State Government’s advised price is through a later notification and hence it
should prevail.

(b) No, Central Government has the authority of setting the minimum price.

(c) No, the price set by State Government is lower than the minimum price; the price fixed by
the Central Government shall prevail.

(d) Yes, State Government can set the remunerative price as per its discretion.

 
 

Passage (Q.-Q.): No fundamental right is conferred on any religion to degrade the other
religion. I have already pointed out that while professing any religion, the religious heads or
professing by any person should not degrade other religion. No doubt, while setting the law
in motion invoked Section 2(A) of IPC regarding deliberate and malicious acts, intended to
outrage religious feelings of any class by insulting its religion or religious beliefs and after
the investigation, the Investigating Officer invoked Section 2 of IPC (uttering words to hurt
religious feelings) uttering, words, etc. with deliberate intention to wound the religious
feelings of any person. Having taken note of the complaint averments and also the statement
of witnesses, with deliberate intention to wound the religious feelings of other religion words
are uttered while propagating. (Extracted with requisite revisions and edits from ‘No religion
has fundamental right to degrade other religions: Karnataka High Court rejects plea by 2
Christians accused of insulting Quran, Gita’ at
https://www.barandbench.com/news/litigation/fundamental-right-religion-karnataka-high-
court-christian[1]quran-bhagavad-gita)

. Ms. Mary during a religious procession was heard using the statement that “Jesus created
the world and can destroy it when he wishes and even the greatest of saint cannot stop the
ragnarok”. A case for hurting the religious sentiments was brought against her. Is she liable?
(a) No, there is no degradation of other religions in the statement of Ms. Mary.

(b) Yes, there is degradation of all other religions by virtue of such a statement.

(c) No, she only professed the idea of the creation and destruction of the world without any
reference to any specific religion.

(d) Yes, a malicious statement is made to outrage the sentiments of other religions.

. Ms. Mary in the past had also elicited a lot of controversy when she said that "only Bible
can tell the future and no other religious scripts give any information and tsunami is coming
in the future." Should she be made liable for such a statement?

(a) No, there is no degrading of other religions as no specific religion has been targeted and a
general statement has been made.

(b) Yes, religious feeling will be affected irrespective of the intention involved while making
such a statement.

(c) No, merely telling the coming of tsunami or telling of future doesn’t mean degradation of
other religion.

(d) Yes, professing one religion should not be at the cost of degrading other religions.

 
 

. Another leader of a religious group during a gathering said “Sabka Malik Ek (There is only
one God) who is the supreme deity and its time when everyone learns to identify the real god
abandoning all the beliefs from those phony scriptures”. Should any liability be attributed to
him?

(a) No, there is no degrading of other religions as no specific religion has been targeted and a
general statement has been made.

(b) Yes, professing one religion should not be at the cost of degrading other religions.

(c) Yes, one cannot identify as to who can be called the Supreme Deity and it will invoke
religious turmoil.

(d) No, no particular religion has been professed through such a statement.

. A statement “Neither Bhagavad Gita nor Quran will provide any peace of mind or come to
the rescue of any person and it is only the Bible which provides the most authentic and
pragmatic view of the creation of the world”. Whether this will be struck by Section 2(A) or
Section 2 of IPC?

(a) Yes, such a statement is degrading and will result in insulting of other religious beliefs.

(b) No, such a statement is incapable of wounding the religious feelings of others.

(c) Yes, the statement is targeted as a result of hatred towards other religions.

(d) No, it is a general statement without any substance devoid of instilling any effect/
opposition by followers of other religions.

Passage (Q.-Q.): The Delhi HC ruled that photographs taken from social media and uploaded
on pornographic websites without the consent of the concerned person amounts to an offence
under Section 67 of the IT Act. The act, regardless of the photograph being
offensive/obscene, amounts to a breach of a person's privacy; the court may in appropriate
cases pass an order of restrain. The court also observed the role of intermediaries, and held
that they are mandated to disable access to such content once they receive a notification by
the appropriate government or its agency. If the intermediary fails to do so, they lose the
exemption from liability. The court said, "In the first instance, an intermediary cannot say
that it is unable to remove/disable access to offending content despite knowledge as
contemplated in law,” An intermediary is now required to inform its users that they must not
host, display, upload modify, publish, transmit, store, update or share any information that
'belongs to another person and to which the user does not have any right' being 'invasive of
another's privacy’,” The court observed that a time-bound grievance redressal mechanism is
provided of i.e. 24 hours from the receipt of a complaint given to intermediary to 'take all
reasonable measures to remove or disable access to such content. The law enforcement body
should be directed to obtain all unique user information relating to the offending content, and
the same must be not later than hours. Source: https://www.thequint.com/news/india/delhi-
hc-issues-directions-on-removal-of-offensive-web[1]content

. Rachit is a sophomore in College. He had recently attended a party in college where he had
clicked several pictures with his friends and batch-mates. He later uploaded several of those
pictures on his social media handles, out of which, one was of him and a classmate Rahul.
Rahul decided to prank Rachit and took the picture of both of them and uploaded on a
pornographic website. This infuriated Rachit, and now he wants to take legal actions against
Rahul. Will his action sustain?

(a) Rachit cannot take any action against him because that picture had both of them in it.

(b) Rachit can take legal actions against him because it was uploaded without his consent

(c) Rachit cannot take any legal actions because the picture wasn’t per se obscene or
offensive, and was merely a picture taken at a party.

(d) Rachit can take legal actions because the picture was taken from his social media.

. Apoorva is a web designer in WebFor Solutions. She recently got into a heated argument
with one of her colleagues, Manoj, on bringing some new changes to their website. In the
heat of the argument, she called Manoj an “uptight idiot” in front of several other staff
members. Embarrassed by this, Manoj went home, took one of the pictures from Apoorva’s
LinkedIn profile, morphed the face of a frog on it and shared it on a pornographic website.
Apoorva came to know about this and when she confronted Manoj on this and threatened to
take legal actions against him, Manoj defended himself saying that her face is not visible in
the picture, thus, he hasn’t committed any offence.

(a) Manoj’s argument is valid because no one can identify it’s Apoorva in the picture as her
face is not visible.

(b) Manoj’s argument is invalid because he morphed the picture of a frog onto her picture
without her consent.

(c) Manoj’s argument is valid because if no one can identify Apoorva in that picture, that
would not amount to breach of privacy.

(d) Manoj’s argument is invalid, because he took Apoorva’s picture, morphed it and uploaded
it on the website without her consent.

. Considering in the above question, had Manoj not morphed her picture but had he just
written her name on the picture of a frog eating flies, would that have warranted the attention
of the Act?

(a) Yes, as he had defamed her which amounted to offensiveness under the act.
(b) No, as Manoj did not use her picture or private information but had just used a common
name.

(c) No, as the picture did not contain any offense or obscene pictography.

(d) Yes, as he did so with the intention of implicating her.

. Akash, an engineering student, uses the Social Media platform to voice his opinions on
various controversial topics. Once, when the discussions around the deportation of the
Rohingya Muslims were going on, he used twitter to protest against this virtually, calling out
the Government for being ‘insensitive’ and ‘hypocrite’. Soon, he became the subject-matter
of several memes. In one of the memes his face was morphed into a picture with a man
standing cloth-less, covering himself with a stack of cash while the Indian Flag was being
burned. The meme was posted as stories on Photogram and would disappear after 24 hours.
Akash complained the Cyber Cell regarding this who then instructed Instagram to take down
the content. Photogram did not take any action to take down those stories thinking that they
would eventually disappear after 24 hours. By this, would Photogram be exempted from any
future liability?

(a) Yes, because all the memes were posted as stories and thus, would eventually disappear
after 24 hours.

(b) No, because it did not fulfil the mandate of taking down the photo immediately after
being asked to do so.

(c) Yes, because the purpose of the new provisions of law is to take down all the offensive
content and in this case, the photos would automatically disappear, thereby fulfilling the
purpose.

(d) No, because it allowed an offensive meme to be uploaded.

. Considering in the above question, had Photogram not taken cognizance of the memes due
to their third party encryption policy which allows users to have complete control over what
they post or see, without letting the application monitor such content,. However, they do have
an override feature which can be used, but would open the application up to hackers and
subsequent lawsuits. Would they still be liable for failure to remove the content?

(a) They will be liable to remove such content from their portal as soon as they receive such
notification.

(b) They will be liable to remove the content regardless of the consequences.

(c) They cannot subject their users to malware/ransom ware attacks, and thus, would not be
liable.

(d) Photogram uses third-party encryption and thus should be exempted from the laws as they
do not mention these laws.
40
Passage (Q.68-Q.): Diminishing the scope of the law hasn’t really worked. The Supreme
Court, in 12, had upheld the constitutionality of the law (Section 124A dealing with sedition
law) saying Article 19(2) that imposes “reasonable restrictions on freedom of speech”
protected the sedition law from the “vice of unconstitutionality”. It nevertheless
circumscribed the application of the law to “acts involving intention or tendency to create
disorder, or disturbance of law and order, or incitement to violence.” It even specifically
narrowed down its scope in order to protect dissent, saying, “Comments, however strongly
worded, expressing disapprobation of actions of the Government, without exciting those
feelings which generate the inclination to cause public disorder by acts of violence” wasn’t
sedition. Yet, the application of the law has been, more often than not, atavistic—reflecting
the colonial-era mindset to control political expression and curb dissent. The judiciary has
tried to tame the governments’ (both State and the Centre) trigger-happy application of the
law—indeed, in the Disha Ravi matter, the court said that citizens “cannot be put behind the
bars simply because they choose to disagree with the State policies. The offence of sedition
cannot be invoked to minister to the wounded vanity of the governments”. (Extracted with
requisite revisions and edits from ‘Sedition law must go: Supreme Court reviewing its
constitutionality; limiting scope hasn’t worked’,
https://www.financialexpress.com/opinion/sedition-law[1]must-go-supreme-court-reviewing-
its-constitutionality-limiting-scope-hasnt-worked/22445/)

68. Mr. Yedu Gandhi, leader of opposition, during his political campaign used pro-Pakistani
slogans to highlight how the present government has failed its subjects, the efficacy of
Pakistani system and how every nation-loving person should unite and revolt against the
Prime Minister to resign. He was shortly thereafter arrested for sedition. Is he liable for
sedition?

(a) Yes, as it has the tendency to create disorder and incitement to violence.

(b) No, the statements are merely dissent though strongly worded.

(c) Yes, as his followers will revolt against the government.

(d) Yes, Mr. Yedu forms a part of the opposition and has the liberty to criticize the existing
government.

. Mr. Yedu Gandhi on being arrested quotes that he had no intention to instill hatred against
the government but it was to learn from the experiences of our neighboring country.
Moreover, his speech was targeted against the PM and not the government as a whole. Will
his defence succeed?

(a) No, the speech was worded to create disorder and incitement to violence.

(b) Yes, there was absence of ill intention against the government but only the PM.

(c) No, comparison with Pakistan invokes sedition.

(d) Yes, the speech is protected under Freedom of speech and expression.
. Soon after his arrest, a renowned news channel Masala TV kept telecasting his speech at
Prime Time. They were thus slapped with complaints in the court to stop further telecasting
of the speech as it will instill hatred and incite violence amongst the viewers against the
Government. Whether the act of news channel seditious? 

(a) Yes, because it intended to incite violence and create disorder.

(b) No, the news channel asserted reality and truth is a defence under the law.

(c) Yes, as their act was to overturn the government.

(d) No, the telecasting does not involve creation of disorder or incitement to violence.

. The news channel was known to be anti-establishment and the anchor had at various
interviews criticized the policies of the government and saying “Look at the economy, the
drop is steeper than while coming down a rollercoaster. All credits to the present government.
I urge everyone to use their brains properly before voting next time.” Will this amount to
sedition?

(a) Yes, the speech is intended to overthrow the present government and is seditious.

(b) Yes, the speech is hit by reasonable restrictions under Article 19(2) of the Constitution.

(c) No, it is merely a form of dissent irrespective of being strongly worded.

(d) No, Freedom of speech and expression grants unfettered rights and power to the media.

Passage (Q.-Q.): It is the statutory right and even the duty of the police to investigate into the
cognizable offence and collect the evidence during the course of investigation. There may be
requirement of a custodial investigation for which the accused is required to be in police
custody (popularly known as remand). Therefore, passing such type of blanket interim orders
without assigning reasons, of not to arrest and/or “no coercive steps” would hamper the
investigation and may affect the statutory right/duty of the police to investigate the
cognizable offence conferred under the provisions of the Cr.P.C. Therefore, such a blanket
order is not justified at all. The order of the High Court must disclose reasons why it has
passed an ad-interim direction during the pendency of the proceedings under Section 4
Cr.P.C (Saving of the inherent powers of High Court). Such reasons, however brief must
disclose an application of mind. (Extracted with requisite revisions and edits from ‘The
Thanedar and the Constitution’, https://www.barandbench.com/columns/the-thanedar-and-
the-constitution)

. Radhe, an acclaimed actor was accused of the offence of murder, a cognizable offence of his
former employee. The informant (Mr. Oberoi) went to file the FIR against him; however, the
police denied registering a FIR and further investigate as he lacked sufficient evidence to
prove his case. Moreover, it was well known that Mr. Oberoi and Radhe have had a bitter
rivalry over the title of land which went in favor of Radhe two months ago and Mr. Oberoi
had been heard to set scores straight. Is the act of police justified?

(a) Yes, Mr. Oberoi failed to prima facie present any evidence to back up his allegation.

(b) No, the offence alleged is a cognizable offence and investigation by the police is
obligatory.

(c) Yes, the allegation was leveled due to past rivalry and was an attempt to set scores
straight.

(d) No, irrespective of the offence been cognizable or non-cognizable, it is mandatory for
police to conduct investigation.

. Later, the FIR was lodged. Radhe was granted anticipatory bail. However, the matter
reached the High Court on appeal during the pendency of the anticipatory bail application
(opposing its repeated extension by Oberoi) in the Sessions Court. High Court passed the
impugned interim order directing that “no coercive measures shall be adopted against the
Radhe in respect of the said FIR”. Is the action of High Court desirable?

(a) No, it amounts to a blanket interim order deprived of any reasons for “no coercive”
measure.

(b) Yes, High Court exercised its inherent powers enshrined under Section 4 CrPC.

(c) Yes, it was to ensure the smooth trial at the Sessions Court.

(d) No, High Court exceeded it power granted under Section 4 CrPC.

. Radhe while shooting his upcoming movie was taken into remand on the basis of the FIR
registered. Radhe however requested that the same shall hamper his repute and he is willing
to cooperate and give his statement after his return from abroad in two days. Despite the
assurance, he was taken into remand due to the fear in loss of valuable information due to
delay. Which of the following statements are true?

(a) The remand was unreasonable as Radhe was willing to cooperate. (b) There exists a
requirement of custodial investigation in order to avoid delay and extract information.

(c) Reputation shall act as one of the determining factor before taking any action against
Radhe.

(d) Custodial Investigation is a part of investigation of a cognizable offence.

. Consider the following statements: i. Passing of orders prohibiting arrest is illegal and
should be discouraged. ii. For investigation, remand by the police is pre-requisite. iii.
Direction of ‘no coercive measures’ or of no arrest with brief reasoning hampers investigative
powers of the police. Which of the following statements according to the passage are true?
(a) Only i.

(b) Only iii.

(c) All the statements are true.

(d) None of the statements are true.

Passage (Q.-Q.): A resignation by a director implies a relinquishment of his office. This is a


unilateral act which unless the Articles of Association otherwise provide, is not contingent on
the acceptance by the company. Directors act as agents of the company and are, thus, also
entitled to terminate their agency. The act of resignation or relinquishment of the office
would not require the consent of the company and, therefore, would become effective from
the time when the intention to relinquish the office as a Director is communicated. Section
168(1) of the Companies Act, 2013 specifies that a director may resign by giving a notice in
writing to the Company and the Board of Directors (Board) shall, on receipt of such notice,
take note of the same. Further, the Section also specifies that resignation of a director shall
take effect from the date on which the notice is received by the Company or the date, if any,
specified by the director in the notice, whichever is later. (Extracted with requisite revisions
and edits from ‘Resignation of directors during the Corporate Insolvency Resolution
Process’, https://www.barandbench.com/columns/resignation-of-directors-during[1]the-
corporate-insolvency-resolution-process)

. Mr. Amdani, a renowned businessman and director of Freelance Corporation decided to


resign from the office and informs, in his official capacity, during the meeting with the Board
his intention to resign from the coming week. From next week, he stops coming to the office
and fails to attend an important meeting with a business tycoon leading to the Corporation
facing major loss. An action is brought against him against such irresponsible behavior. Is
Mr. Amdani’s resignation valid?

(a) Yes, the information of resignation was intimated to the Board in advance.

(b) No, the resignation was not in writing.

(c) Yes, it is implicit from his conduct about him wanting to relinquish office.

(d) No, despite the meeting scheduled for him, he did not protest against the same.

. Another director of the company, Mr. Athanni, was bedridden and so he got his resignation
typed and attested it. The Corporation knowing his significance to them was not willing to
accept his resignation. Which of the following could favour the Corporation?

(a) The resignation was not in a written format but typed and hence can be rejected.

(b) The Corporation can delay the acceptance of the resignation till another alternative
candidate can be finalized.
(c) Article of association, if they contain a contradictory provision can come to the rescue of
the Corporation.

(d) The Corporation has no say in the resignation by its employees.

. Assume the resignation of Mr. Athanni contained the date of his relinquishment from office
from June 5, 2021. However, due to delay in dispatch the resignation reached the Board on
June 10, 2021. Unaware about the delay, Mr. Athanni had stopped coming to the office since
June 5 only. However, the Corporation accepted the resignation only from June 10, 2021.
Decide the date when the resignation can take effect?

(a) June 5, 2021 as it is the date mentioned in the resignation letter.

(b) June 10, 2021 as it is the date on which the letter/ notice is received by the Corporation.

(c) Due to the delay, the Corporation has the discretion to choose the date it deems
reasonable.

 (d) June 5, 2021 as Mr. Athanni was unaware about the delay in dispatch and the same was
not intimated by the Corporation as well.

. The Article of Association of the Corporation, as amended, provided for an advance notice
of 15 days before the date of resignation for the Corporation to approve the resignation. But
due to delay in dispatch, the notice reached only 10 days before the date of resignation
mentioned. However, the intent to resign was already communicated verbally by Mr. Athanni
to the Board atleast a month ago. Now, the Corporation quoted their Article of Association
for not accepting the resignation from the desired date. Is the stance of the Corporation
correct?

(a) Yes, Article of Association would govern the acceptance of resignation.

(b) No, the delay in dispatch was unintended and unforeseeable by Mr. Athanni.

(c) No, Mr. Athanni had previously communicated his intent to resign which shall suffice.

(d) Yes, Mr. Athanni is bound to be aware of all the recent amendments to the Articles of
Association.

Passage (Q.-Q.): “What’s in a name? that which we call a rose by any other name would
smell as sweet”, said Juliet. This quote from William Shakespeare’s “Romeo and Juliet” is
unarguably one of the most iconic dialogues in classical literature. It conveys that the natural
characteristics of an individual are more important than his/her artificial/acquired
characteristics. To put it differently, name is an intrinsic element of identity.” Identity is an
amalgam of various internal and external including acquired characteristics of an individual
and name can be regarded as one of the foremost indicators of identity. And therefore, an
individual must be in complete control of her name and law must enable her to retain as well
as to exercise such control freely “for all times”. Such control would inevitably include the
aspiration of an individual to be recognized by a different name for a just cause. However,
going by the very nature of rights under Article 19, the right to get changed name recorded in
the official (public) records cannot be an absolute right and as a matter of public policy and
larger public interest calls for certain reasonable restrictions to observe consistency and
obviate confusion and deceptive attempt. (Extracted with requisite revisions and edits from
‘Right to control one’s identity a fundamental right; CBSE must permit change of name for
just cause: SC’, https://www.scconline.com/blog/post/2021/06/03/right-to-control-ones-
identity-a-fundamental-right-cbse[1]must-permit-change-of-name-for-just-cause-sc/)

. Megha Verma, completed her schooling in 2021. The marksheet was released. However,
Megha decided to change her name to Megha Nilesh Verma i.e. adding her father’s name too.
However, the CBSE byelaws provided that “The details of the candidate can be changed only
if the same has been indicated and recognized in her certificates in lower classes. Otherwise
the same shall be denied.” However, Megha’s decision to add her father’s name was recent
and cannot be brought about now in every certificate. She decides to bring a suit against these
bylaws of CBSE. Will she succeed?

(a) No, CBSE’s byelaws do not prohibit the changing of the name but only provide for a
precondition to be fulfilled.

(b) Yes, law should enable her to retain as well as to exercise such control freely “for all
times”.

(c) No, the right to change her name is not an absolute right.

(d) Yes, she has the absolute right to bring any change in her identity in the public records.

. Another candidate Radha wanted to bring about correction in the name of her mother from
Jaya to Java Devi. However, as CBSE noted, her mother was named as Jaya until now in all
the academic records of Radha and couldn’t comprehend any reasoning behind the same.
Hence, the application to change the name was rejected. Is the act of CBSE justified? (a) No,
name is an intrinsic attribute and one should have freedom to control it at all times.

(b) Yes, Radha has the freedom to control her name and not the name of her mother.

(c) Yes, sudden change in name appears to be ill-intended and would create confusion.

(d) No, changing of name in the official records is an unfettered right of every person.

. CBSE provides for a deadline of one year to make any requests for any corrections/
rectifications as after those students would have to approach different authorities. Megha
sends her application after this deadline. She claims that it is her right to change her name and
no deadline can be made curbing her right. Is her argument correct?

(a) Yes, everyone has the freedom to control their name at all times.

(b) No, there is no absolute prohibition in changing the name.

(c) Yes, no reasonable deadline has been provided and this is against public policy.
(d) No, it is merely a hindrance in changing/ correcting the name.

. CBSE through a notification provides that any changes in the official documents have to be
supported either by ID proofs like Aadhar and in its absence through an Affidavit. According
to Medha and Radha this is a clear breach of their freedom and autonomy over the name.
They brought an action against CBSE. Will their argument hold well?

(a) No, CBSE hasn’t prohibited the changing of name absolutely.

(b) Yes, there is a breach of freedom over name and absolute control over the same.

(c) No, changing of name at a later stage will create confusion and ambiguity,

(d) Yes, presentation of ID proofs and Affidavits brings into question the freedom over
controlling one’s name. 

Passage (Q.-Q.): The petitioner is the second wife of the deceased husband. Since the second
marriage was solemnized during the first, a complex question w.r.t to validity was presented.
The second marriage took place while the first wife was still alive. The first wife later passed
away. The petitioner’s husband made an application to make his first wife the nominee to his
pension account. However, he passed away before the process was completed. After her
husband’s demise, an application made by the petitioner to the Audit Officer for withdrawal
of the pension amount from his account was rejected. Consequently, the current petition was
filed before the High Court. It was observed by the Court that the Rules applicable require the
existence of a valid marriage in order to extend pension benefits. However, since the
marriage was solemnized before the first wife’s demise, the applicability of the Rules was in
question. The Hindu Marriage Act, 15 does not permit second marriage while the first wife is
still alive. However, as per the Domestic Violence Act, 2005: “Even without marriage, when
a live-in-relationship between two genders is established, it is held to be legal, and with time,
the woman attains status of a wife. But, after the demise of the husband, if two wives are
alive, the second one will not attain the legal status of 'wife' unless Personal Law permits."
Hence, the second wife attains the deeming status of a wife from the date of demise of the
first wife, in case, the husband is alive on the date of demise of the first wife.” Source:
https://www.barandbench.com/news/litigation/wife-second-marriage-pensionary-
benefits[1]husband-madras-high-court-larger-bench

. Rakesh and Rubina was a married couple wherein Rubina was suffering from a chronic
heart disorder and was diagnosed with only 6 months left to live. This led to Rakesh
becoming a philanderer as he was never really attached to Rubina, only to her massive
wealth, which was bestowed upon him, once she had succumbed to her condition. It was a
known fact that he had multiple concubines who could lay claim to his estate upon his demise
or other events and thus, it just so happened that on his way to work one day, he met with an
accident and died in the same. This lead to his concubines claiming monetary assistance from
his estate for the rest of their lives as many of them had borne him children who required
financial support. Would they be able to claim such support successfully?
(a) The concubines have a valid claim to his estate as Rakesh had made them his live-in
partners.

(b) They do not have a claim to his estate as this was not a live-in relationship.(c) They will
not have a sustainable claim as multiple people cannot be considered to be live-in partners.

(d) They will have a valid claim as they have borne him children.

. Considering, in the aforementioned set of facts, Rakesh had been bigamous from the very
beginning and had a second wife while he was married to Rubina. However, on the way back
from her date of diagnosis, due to a heart failure, he had passed away without leaving
anything for his second wife. In this case, could she be considered equal to the status of a
‘wife’?

(a) She will be considered as a wife since she was married to Rakesh and the Hindu Marriage
Act provides her with legal status as the same.

(b) She will not be considered as a wife since Rubina was not alive at that point in time.

(c) She will not be considered as a wife since the laws in India do not grant her that status.

(d) It is unclear whether Rubina was alive or not, and it depends on that condition solely.

. In the aforementioned question, had Rakesh been and Rubina had died, would his second
wife have attained the status of a ‘wife’ and would have a claim to his estate?

(a) The second wife would have attained the status of a wife but wouldn’t have any claim to
his property.

(b) The second wife would have a claim to his property but wouldn’t be termed as a wife.

(c) The second wife would neither be considered as a wife, nor would she have any claim to
her property.

(d) The second wife would only have a claim in his property if Rakesh had applied to the
Audit offier for the same.

. Had Rubina and Rakesh not been married, and had Rakesh died of the heart attack before
Rubina, and had they both intended to marry out of love. Could Rubina have laid a claim to
the pension that Rakesh was to receive, being a government employee?

(a) According to the Rules, the marriage needs to be solemnised first. Hence Rubina would
not have been able to claim the pension.

(b) Rakesh should have applied for her to become a nominee first.
(c) Every pensioner requires a nominee to be added to his/her pension account. Hence Rubina
being his fiancé, she would have had a claim to it.

(d) The law requires only a semblance of a marriage even if such relationship is of a live-in
nature; hence on account of being about to marry each other, Rubina would have been able to
claim the pension.

. Jagan and Bhagan are a homosexual couple who are in a live-in relationship and do not
intend to marry but have been together for almost 12 years, ever since the first year of their
college. Jagan works for a multimillionaire who runs an automobile conglomerate by the
name of ‘Khosla’ wherein his job perks include a steady income even after he retires, or to
his spouse, if he dies untimely. Unfortunately, the worst happens and he dies in a car crash
under a bridge collapse. This leaves Bhagan distraught and he becomes depressed and is
diagnosed with being suicidal. When he tries to claim the amount of Jagan’s post-
employment income, he is denied such finances, against which, he files for the violation of
her fundamental rights. Will he be successful?

(a) Bhagan will not be successful since he is not a spouse of Jagan’s.

(b) Bhagan should be given the amount of post-employment finances since he is akin to a
spouse of Jagan’s under the law.

(c) Bhagan cannot succeed as an action of Fundamental rights cannot be brought against
private entities.

(d) Bhagan is entitled to the post-employment income as he is the closest kin to Jagan on
account of having been together for 12 years.

. In the above set of facts, had Jagan been employed in a PSU owned by the government,
wherein the same post-employment income policy was established, would Bhagan have been
able to claim it?

(a) Bhagan would have been able to successfully claim the amount as the settled position of
law would term him as the spouse of the diseased.

(b) Bhagan would not have been able to claim the amount as he wasn’t the ‘spouse’ of Jagan.

(c) Bhagan would have been able to claim the amount since the policy extended to spouses
and not just ‘wives’.

(d) Bhagan would have failed to claim the amount on account of being in a homosexual
relationship.

Passage (Q.-Q.): Mere claim of two adults living together for a few days will not be sufficient
to attach legitimacy to a live-in relationship. The Court noted that the concept of live-in-
relationship of opposite gender has got recognition in India as the legislature has injected
legitimacyand liberally defined “domestic relationship”. “To attach sanctity to such a
relation, certain conditions are required to be fulfilled by such partners. Merely because the
two adults are living together for few days, their claim of live-in-relationship based upon bald
averment may not be enough to hold that they are truly in live-in-relationship,” the Court
ruled. The Court also considered, the tendency of young people to file petitions before the
Court claiming to be in live-in relationship. Herein the matter pertained to a seventeen year
old boy and a 14 year old girl. “Majority of petitions contain grounds with imaginary cause of
action, and are rarely founded upon existence of threat, these types of cases consume
considerable time of this court, at the cost of many cases waiting in line for hearing.” The
Court said that two adults have the right to live together which legitimizes such relationships.
However, there are conditions to attach legitimate sanctity to such relationships. This
includes duration of relationship, shared household, pooling of resources and financial
arrangements, domestic arrangement, sexual relationship, children, socialization in public etc.
It further stated that though, The Court ordered the Police to ensure that the custody of the
minor girl is restored to her parents. Source:
https://www.barandbench.com/news/litigation/claim-two-adults-living-together-few-
days[1]legitimacy-live-in-relationship-punjab-haryana-high-court

. Ravi and Chhavi are a couple aged 19 and 18, respectively and study at IIM Ahmedabad.
Chhavi, is a bright girl and has been consistently scoring high in all semesters, however, her
parents are against her studying so far away from home. They want her to marry but she does
not want to do so. One day she gets a phone call from her family stating that they are looking
for a suitable boy for her. This scares her and she tells, Ravi. Ravi suggests that they should
start living together so that they can dissuade her parents and can show a live in relationship
in a legal manner. Could this move of theirs succeed in a court of law?

(a) Ravi and Chhavi will be liable to be termed as a live in couple, and therefore should be
granted the same status.

(b) Ravi and Chhavi are not living together currently; therefore, they won’t be regarded as a
live-in couple.

(c) Ravi and Chhavi are consenting adults and of the opposite genders, therefore, this should
be considered as a live in relationship.

(d) Chhavi was being influenced heavily by her parents and therefore decided to live with
Ravi under extreme duress. They should be termed as being in a live-in relationship. 23 of 36

. Notwithstanding the above set of facts, had Ravi and Chhavi been living together for more
than 2 years, and had they been having a joint bank account that they used together for
sustenance. Would this arrangement have been eligible to be termed as a live in relationship?

(a) Ravi and Chhavi on account of being consenting adults should be termed as being in a
live in relationship.

(b) Both of them have certain factors governing their relationship, ergo; they should be
termed as being in a live in relationship.

(c) Both of them live in a miserly state, therefore, they cannot sustain themselves and should
be denied the status of a live in relationship.

(d) Both of them started living together while they were minors, and hence, they shouldn’t be
termed as being in a live in relationship.

 
. Bhau and Bhouni are two consenting adults who have been living together for 5 years after
their graduation but have not been able to marry because of the absence of Indian laws to that
extent. One day, while roaming in the park together they are faced with a political group
called ‘Jhabua ke sher’ who is notoriously infamous for beating up couples for living
together. On being faced with them, they start unreasonably shouting and threatening Bhau
and Bhouni with lathis on account being homosexuals and couples. Bhau and Bhouni
approach the High Court under Article 226 for redressal on account of their fundamental
rights being violated; can they be recognized as a live in couple?

(a) No fundamental rights have been violated by Jhabua k sher and therefore they cannot be
given such redressal.

(b) They have been living together for more than 5 years now and are consenting adults,
therefore, they should be recognized as a live in couple.

(c) They wouldn’t be recognized as a live in couple.

(d) Such shamelessly fascist political groups should not be allowed to run freely and
therefore, should be provided redressal against.

. The HC of Tamil Nadu is appealed to regarding a petition filed by a couple named Dolly
and Pritam. The facts are considered by the HC as being that Pritam had recently lost his job
and was unemployed for 3 months with nearly no savings to sustain him in the face of
adversity. On account of being in such a miserly state Dolly files for divorce as their
arrangement is no more sustainable. Can the HC grant them such relief?

(a) The HC has the power to grant adequate relief under its jurisdiction and therefore should
grant such relief.

(b) The HC cannot grant them such relief they are not a married couple but only a live-in
couple.

(c) The HC can grant them relief as the question does not mention them being married or not.

(d) None of the above. 

Passage (Q.-Q.): Karnataka HC opined that Courts will have to be circumspect while
selecting matters for live streaming and that cases involving personal allegations are not
chosen for the same, but the cases involving questions of law are a good choice for live
streaming. “The subject of litigation, we have to be very careful, like no cases of personal
allegations involved (should be live-streamed),” AAG Chinnappa told the Bench that live
streaming of cases was a good move considering the current COVID-19 pandemic, as it
facilitates an open court culture. In fact, many people log in to the High Court’s YouTube
channel to view the proceedings, pointed out Chinnappa. Chinnappa, however, also expressed
some reservations stating that lawyers may be hesitant to engage in courtroom 'banter' or
'humour' as everything would be under scrutiny. “Even during sessions of Courtroom
humour, we have to be careful, everything will be recorded,” he said in lighter vein.
Chinnappa opined that while it may be beneficial for law students to actually witness
courtroom proceedings in real time, it may “take away some amount of informality” in
courtrooms. 24 of 36 The Court also informed on Monday that it is examining the Draft
Rules issued by the Supreme Court for live streaming. The e-committee of the Supreme
Court of India had recently released the Draft Model Rules for Live-Streaming and
Recording of Court Proceedings on public domain, inviting inputs and suggestions from all
stakeholders. Source: https://www.barandbench.com/news/litigation/courts-should-be-
cautious-choosing-matters-live[1]streaming-karnataka-high-court

. A matter comes up for hearing before the SC which involves an alleged exchange of
sensitive information between an ISI and RAW officer that could impact a deal that is to take
place for the Chagos Archipelago between Mauritius and United Kingdom. The media on
both sides have been heavily demanding the courts in the country to make the hearing a
public event as it is imperative for the citizens of both India and Pakistan to know what
exactly happened and how the officers consorted treason. The SC, under overwhelming
pressure, gives in and allows the hearing to be streamed online. Is this in accordance with the
passage above?

(a) Since this is a matter of national importance, and the information does not pertain to the
countries involved in the hearing, the matter should be streamed.

(b) The SC has erred in allowing the hearing to be streamed.

(c) The SC has rightly allowed for the matter to be of public knowledge as the information
therein is of public importance and should be made known to the masses.

(d) The passage does not talk about matters of cross-border interests, and therefore the SC
can allow the hearing to take place in a discretionary authority.

. In the above set of facts, had the hearing only concerned the counts of treason alleged
against the Indian agent, would the SC have allowed a public hearing to take place?

(a) Yes, as the hearing in question is of national significance and citizens of India should be
made aware of the same.

(b) Yes, as the SC has complete authority to decide which matter to stream and which to not.

(c) Yes, as the matter in question involves a personal allegation as against the crimes
committed on the country.

(d) No, as it is a matter of personal allegations.

. An extremist faction is caught on the outskirts of the Spiti Valley distributing seditious texts
to the people residing there who were in desperate need of food and were given the same if
they pledged to preach the texts given to them. Upon noticing such seditious and unlawful
behaviour the local police there arrests the faction and legal action is taken against them for
inciting rebellion and sedition. When the matter is put up for hearing, they demand that the
hearing be made public so that the whole country can listen to their ideals and appreciate their
views. Is the court bound to comply to such demands?
(a) Yes, as the matter in question could help answer important questions of law and could
help practitioners, scholars and students learn.

(b) Yes, as the court owes the public at large the duty to hear and adjudicate matters
expediently and lawfully which a public hearing would ensure.

(c) No, the courts are not bound to comply.

(d) No, as the matter involved could hurt the sentiments of the public at large and could
impact the practitioners, students, etc. in a negative manner

. The Kerala HC is faced with a very important question of law regarding awarding the death
penalty in a case of outraging the modesty of a woman. The case involved a man of 40 years
exploiting the person of a 15 year old mentally-challenged girl who could not defend herself
being handicap. The matter has gone on for 13 years with numerous hearings, pardon
petitions, appeals, tons of paperwork and immense mental agony for the family of the victim.
A number of prominent jurists, scholars and top legal minds have opined on this debating
whether death penalty should be awarded in his case or not. Considering that the courts have
never been faced with such a question of law, should this hearing be made public?

(a) Yes, as the question of law being considered herein is of a novel nature and warrants
nationwide recognition.

(b) Yes, as the matter listed could help in advancing the legal fraternity and its stakeholders
in a multi[1]fold manner considering that such an important question of law has arisen.

(c) No, as the matter in question pertains to personal allegations on a man and could severely
hamper his identity.

(d) No, as the matter in question could adversely affect the identity of the victim and set a bad
precedent.

. Shri Bhagwanta Dash ‘KKD’ Moolchandwaale is a self-declared godman who is accused of


murder and playing fraud on his followers in the name of ‘bhakti’. One anonymous person
however, files a PIL in the SC providing concrete proof of KKD’s crimes. The SC lists the
matter up for hearing and allows the public to attend the hearing as it wants to disburse the
blind-followers of the godman to know the reality. Is the SC correct in doing so?

(a) The SC is wrong in allowing the matter to be attended by the public.

(b) The SC is correct in allowing the followers to be disbursed of their beliefs.

(c) The SC should consider limiting the scope of the hearing as it could impact the
nationwide masses.
(d) The SC is correct in allowing the hearing to be heard by the masses as it solves an
important question of law.

Passage (Q.-Q.): State or its Agencies under Article 12 of the Constitution cannot claim
financial constraints or impact of the Pandemic as a ground for dispensing the services of its
employees" observed the Delhi High Court in its judgment quashing Air India's decision
terminating pilots by directing their reinstatement with back wages. "State or its Agencies
under Article 12 of the Constitution cannot claim financial constraints or impact of the
Pandemic as a ground for dispensing the services of its employees, in the manner adopted in
the present case. State has a fiduciary duty to perform towards the citizens under Article
19(1)(g) and Article 21 of the Constitution and thus it becomes the bounden duty of a
Welfare State to secure the rights of livelihood of the citizens.” Therefore, directions were
sought to reinstate the pilots who were serving as permanent employees, with all
consequential benefits of continuity of service, seniority, back wages etc. The court opined on
the resignations tendered by employees as that State or its Agencies under Article 12 of the
Constitution cannot claim financial constraints or impact of the Pandemic as a ground for
dispensing the services of its employees. “In the garb and guise of accepting resignations,
quite clearly Respondent has found an easy path to dispense with the services of the
Petitioners, without following any procedure known to law and without having to bear the
monetary consequences and liabilities thereto." The Court directed Air India to reinstate the
Petitioners who are Permanent Employees with continuity of service from the date of expiry
of the six months’ notice period. Source: https://www.livelaw.in/news-updates/financial-
constraints-pandemic-not-grounds-for-state[1]agencies-sack-employees-delhi-high-court-air-
india-pilots-case-

. The nations of Viraj and Ambuja have been struck with a genetically engineered chicken
pox strain capable of rendering its victims blind. These nations declare it an epidemic and
thus suspend all activities in the nation and initiate a lockdown in their territories. The WHO
announces a relief package for them wherein 20 countries pledge to donate $20 Bn as the
nationwide suspension of services has caused people to become unemployed and go hungry
for days on end. Could the affected people bring an action against the Central Government
alleging wrongful termination?

(a) The nations have rightly announced the safety measures and as such, cannot be brought an
action against.

(b) The nations are only affected by an epidemic and not a pandemic, and thus cannot
suspend their operations.

(c) The nations in the question have liberty to rule on the topic of the welfare of their citizens,
and as such, wrongful termination cannot be sought here.
(d) The nations have announced the lockdown measures in furtherance of public safety and
not any other ancillary measures, as such; no action for wrongful termination shall lie against
them.

. Considering the earlier question, had the above-mentioned nations not initiated a nationwide
lockdown, but had rather heavily expended their resources in the treatment, leading to a
country wide financial crunch, resulting in massive layoffs (people being terminated), could
that have been termed as a violation of Article 12, considering the laws to be pari passu that
of India?

(a) No, as such financial crisis affecting the nations cannot be equated with a global
pandemic. 

(b) No, as the countries are on the brink of collapse, and being a welfare state, they have to
consider health treatment before employment.

(c) Yes, as they should have instated a lockdown to prevent such crunch.

(d) No, as the employment agencies have not been identified as state agencies. .

. Considering that during the pandemic, the nation of Viraj declares massive layoffs in the
government regulated tobacco products sector that amasses 200 crore people. When these
people challenge such termination by their private companies, the Apex Court decides to not
allow reinstatement and cites the reasons as being a strong financial crunch being faced by
the companies. Is this decision of the apex court a good decision?

(a) This decision is a good decision as the reinstatement of these people and subsequent pay
to them will not be possible for the companies in the current situation.

(b) This decision should be struck down by a larger bench as financial crunch cannot be cited
as a reason for termination of employees.

(c) This decision is a good one as the companies involved are not state agencies.

(d) This decision is a good one as the financial crunch has crippled these organizations.

. Considering that in the above question, had Coal Viraj United, a company having 54% stake
of government, declared massive layoffs but was ordered by the Supreme Court to reinstate
the employees in their respective positions. Should Coal Viraj United comply with the same?

(a) Yes, as the order has been made by the Supreme Court and there exists no other appellate
mechanism.

(b) Yes, as the company in question is a government company.

(c) Yes, as the company cannot claim a financial crunch due to the epidemic

(d) Yes, as the company has an obligation to abide by the laws and the judiciary of India.
. Considering that in the above circumstances, had Jignesh, a high-level executive in the
company, resigned due to the financial crunch pre the layoffs, would the company still have
been under an obligation to reinstate him?

(a) No, as he had resigned of his own volition and had not been terminated from his position

(b) No, as they only have to reinstate the terminated employees.

(c) Yes, as he had resigned due to the same reason as the layoffs.

(d) Yes, as he was driven to the limit of resignation by the financial crunch itself, as was the
reason for the layoffs.

41
Passage (Q.68-Q.): A citizen has a right to criticize or comment upon the measures
undertaken by the Government and its functionaries, so long as he does not incite people to
violence against the Government or with the intention of creating public disorder, the
Supreme Court said while quashing sedition case registered against journalist Vinod Dua.
The bench of Justice Uday Umesh Lalit and Vineet Saran observed that it is only when the
words or expressions have pernicious tendency or intention of creating public disorder or
disturbance of law and order that Sections 124A is attracted. The court noted that the
statements made by Dua can at best be termed as expression of disapprobation of actions of
the Government and its functionaries so that prevailing situation could be addressed quickly
and efficiently. In the case Kedar Nath Singh vs State of Bihar(12), the Supreme Court had
read down Section 124A IPC and held that the application of the provision should be limited
to "acts involving intention or tendency to create disorder, or disturbance of law and order; or
incitement to violence". They were certainly not made with the intent to incite people or
showed tendency to create disorder or disturbance of public peace by resort to violence. The
petitioner was within the permissible limits laid down in the decision of this Court in Kedar
Nath Singh . The court held that every Journalist will be entitled to protection in terms of
Kedar Nath Singh, as every prosecution under Sections 124A of the IPC, sedition and 505 of
the IPC, statements conducing public mischief must be in strict conformity with the scope
and ambit of said Sections as explained in, and completely in tune with the law laid down in
Kedar Nath Singh.

68. Indian Government accepted continuation of all the British laws in India after its
independence. One such law was Indian Penal Code under which one case before the
Supreme Court of India is criminal proceedings against Tara Singh. When partition was
agreed to by India and Pakistan, India opened its doors for anyone who wishes to stay within
its bounds. Tara Singh was against the idea of Muslims living in the county and called for
protests against this policy. His speeches in the demonstrations led to nationwide riots, of
which he was an integral part. Due to the protests a girl, a Sakeena lost her life. The state has
charged him with law of sedition and the court has reserved the order for pronouncement for
today. Choose the most appropriate decision by the court.

(a) Tara Singh is guilty of sedition as he gave the speech against the government with an
intention to cause public disorder and violence, which it eventually concluded into.
(b) Tara Singh is not guilty of sedition as mere criticism of the policies of the government can
be characterised as sedition to render the citizens bereft of their right to freedom of speech
and expression, and freedom to demonstrate.

(c) Tara Singh is guilty of sedition because regardless that he did not have any intention to do
so, his speech resulted into mass violence causing even the demise of a girl.

(d) Tara Singh is not guilty of sedition as there is no evidence to suggest that he had any
intention or his actions had any tendency to either disrupt public order or cause violence as
result of his speech.

. After the assassination of Indian Prime Minister Seema Bose by her two bodyguards, riots
broke out in the nation. Her assassination was a vengeance for not allowing partition of
Indian state of Sindh to form a separate nation altogether. Balwant Singh and two others were
nabbed by the Police for shouting slogans ‘hail khalistan, khalsa will rule, we will liberate
Sindh from India’ in front of a government office. The court upon analysing the evidence
produced by the prosecution found out that the slogans did not attract any attention and the
accused left the place after almost half an hour of demonstration. Decide whether the court
would find them guilty of the offence of sedition.

(a) The accused shall be found guilty of sedition as they had the intention of causing violence
because at a time when the entire nation was struggling against the riots even small
demonstration and slogans criticizing state policies could turn violent.

(b) The accused shall not be found guilty of sedition as there was no intention to cause
violence because there was no overt act of violence and their slogans did not attract any
attention or caused any disruption of public order.

(c) The accused shall be found guilty of sedition because demanding a separate nation from
India is against the integrity of our nation and thus sedition in itself.

(d) The accused shall not be found guilty of sedition as even if there was an intention to
create any disorder they failed to convert it into any form of action.

. Assertion (A): The intention to cause violence and the overt acts of violence are the
mandatory requisites of convicting someone of sedition.

Reasoning (R): Any such intentionally done act when leads to disruption of public order is
seditious in nature.

(a) Both A and R are correct, R is the correct explanation of A

(b) Both A and R are correct, R is not the correct explanation of A

(c) A is incorrect and R is correct

(d) Both are incorrect

 
. Laxman Sharma was a renowned artist who drew a sketch showing politicians getting
treated in AIIMS hospitals while the rest of the citizenry is waiting in queues before
hospitals. It was aimed at pointing out the failure of institutional machinery in arranging
medical facilities for the people infected in a nationwide pandemic. He was arrested soon
after and charged with sedition for intending to cause disorder by inciting people against the
country. In the court, Laxman contested that the sketch was only for the purpose of satire and
he did not intend to disrupt the public order. Choose the correct ruling for the present matter.
(a) Laxman shall be convicted for sedition as during a pandemic such an act even if done for
the purpose criticizing the government is most likely to cause unrest and cause public
disorder.

(b) Laxman shall not be convicted for sedition as it is the intention of the accused and not
what their actions lead to which is relevant. Thus, if intention was only to present a satire on
the governmental failure.

(c) Laxman shall be convicted for sedition as during these difficult times when there is
already disorder, even mere criticism of government shall be punished as sedition.

(d) Laxman shall not be convicted for sedition as the sketch was intended to criticize the
failure of the government in providing medical facilities and it did not result into any
violence or disruption of public order.  

. Bilal Ahmed, an alleged militant, was nabbed by the police from a hotel very close to a
serial bomb blast site. He was also found in possession of a gun along with two cartridges.
State imposed on him charges under various acts including Indian Penal Code, Arms Act and
Prevention of Terrorist Activities Act. Under IPC he was charged under section 124A for
overt acts of blast and causing public disorder to incite feelings of contempt and
disapprobation against the Government of India. Decide whether he shall be convicted of
sedition or not?

(a) Bilal shall be convicted of sedition as bomb blast is an overt act of intentionally causing
public disorder or disrupting law and order.

(b) Bilal shall not be convicted of sedition as there is absence of any overt act for which
intention can be deduced and he can be charged with sedition.

(c) Bilal shall be convicted of sedition as even carrying a revolver, if he is not involved in
blasts, would be an act with an intention of disrupting the public order.

(d) Bilal shall not be convicted of sedition as the entire criminal charges against him are
based on assumption that he is a militant.

Passage (Q.-Q.): The Allahabad high court said that irrational, indiscriminate arrests are a
gross violation of human rights, accepting an anticipatory bail plea in a corruption case. The
courts have repeatedly held that arrest should be the last option for the police and it should be
restricted to exceptional cases where arresting the accused is either imperative or their
custodial interrogation is required. There was an allegation against the applicant that he was
taking money from the passing trucks, an FIR was lodged against him under section 7/13 of
the Prevention of Corruption Act, 19. After the lodging of an FIR, the arrest can be made by
the police at will. There is no definite period fixed for the police to arrest an accused against
whom an FIR has been lodged. Irrational and indiscriminate arrests are gross violations of
human rights,” the court observed, allowing anticipatory bail plea of the applicant. “In the
event of arrest, the applicant shall be released on bail till cognizance is taken by the court on
the police report, if any, under section 1 (2) Criminal Procedure Code before the competent
court on furnishing a personal bond of Rs 50,000/- with two sureties each in the like amount
to the satisfaction of the Station House Officer of the police station/court concerned,” it said.
The high court has put some conditions on the applicant, that he shall make himself available
for interrogation by the police officer as and when required. He shall not, directly or
indirectly, make any inducement, threat or promise to any person acquainted with the facts of
the case to dissuade from disclosing such facts to the court or to any police officer, it said.

. Saqib owned a transportation agency in Bhopal that rents out car for journeys. In a criminal
investigation Police finds that his car has been used in a bank robbery. Thus the police came
to his house for arrest. He claims that he is in the business of renting out cars, and thus one of
his cars was found with the offenders. He further argued that he has copies of the identities of
the people to whom he rented out the car and promises to appear for interrogation when the
police demands. Upon inquiry by the police it was also found that Saqib has no past criminal
record and has been a permanent resident of Bhopal for over 12 years. Decide whether he
shall be arrested or not.

(a) Saqib shall not be arrested as he was just carrying out his business in renting out the car
and there is no need for a custodial interrogation by the Police.

(b) Saqib shall be arrested as he is an accessory to the crime and thus accused of robbery
himself, therefore he shall be arrested so that he discloses names of other accused who
committed the robbery.

(c) Saqib shall not be arrested as there is no evidence with the police that he rented out the car
with a malicious intention or had any knowledge that the car would be used to commit an
offence.

(d) Saqib shall be arrested as he would be able to tamper with the evidence or threaten the
witnesses to influence the conclusion of the investigation.  

. Zahid is an IAS officer who was nabbed red-handed receiving bribe by the anti-corruption
bureau. The police has made an application before the magistrate to arrest. The Bureau has
contended that there have been previous complaints against the accused but allegedly he was
protected by his influence every time. Thus the department seeks to ensure that he does not
get away this time until the investigation is concluded. Decide whether the warrant shall be
granted or not.

(a) The warrant shall be granted as Zahid has been arrested red handed and there is no need
for further evidence to prove that he is guilty of the offence.

(b) The warrant shall not be granted because there are no exceptional circumstances or need
of custodial interrogation that warrant arrest of Zahid.
(c) The warrant shall be allowed as Zahid is most likely to influence the outcome of the
investigation by tampering with the evidences and manipulating the witnesses.

(d) The warrant shall not be allowed as Zahid is an officer at a decorated rank, and every
incident of arrest of such an officer would set a wrong precedent for the nation.

. Baba Ramprasad is an Indian Yoga Guru who while leading a protest against the corruption
pervasive in the government made hateful speeches that led to violence. When police arrived
to nab him, he escaped. Almost a week later he made an application before the Delhi High
Court for anticipatory bail. The application has been challenged by the State on the ground
that Baba will abscond and might also try to influence the outcome of the investigation. The
court has reserved the judgment for today. Decide whether the bail be granted or the arrest
shall be allowed.

(a) Bail shall be granted because there is no evidence that Baba will influence the
investigation by tampering with evidence if let out on bail by the court.

(b) Bail shall not be granted because Baba has absconded before when the police tried to
arrest him and if released on bail might abscond again instead of cooperating with police.

(c) Bail shall be granted because Baba cannot be put in jail unless he is actually found guilty
of the offence after a trial and sentenced by the court itself.

(d) Bail shall not be granted because Baba was leading a protest against the state itself which
is treason and therefore he shall be locked up and later sentenced for the unforgivable
offence.

. In which of the following circumstances shall the Police avoid arresting the person.

I. Police went to interrogate a person who tried to escape after seeing the police.

II. After collision of two vehicles the one at negligence was trying to help the victims and
called the police to report the incident.

III. A police constable charged with receiving bribes

IV. A husband booked for inflicting domestic abuse on her wife Choose the correct answer:

(a) IV only

(b) I and IV

(c) II, III and IV

(d) I only

 
. Imran has been arrested for being part of a violent protest that took after an MP tweeted a
communally hateful post. The Police has identified 11 perpetrators so far and Imran is one of
them. After his arrest, he made an application for the bail that was rejected by the Sessions
Court without stating any reasonable ground. The court ruled that bail is an exceptional rule
where the arrest would be against the interest of the accused and defying the cause of justice.
Thus, in absence of any such circumstances Imran shall be in police custody for forthcoming
period. Imran has made an appeal before the high court against the order. Choose the correct
option.

(a) Appeal shall be allowed as bail remains to be the general norm except in circumstances
where arrest is imperative, and since there is no particular reason to keep Imran in custody he
shall be released on bail.

(b) Appeal shall not be allowed as there is no reason to believe that arrest will cause any
prejudice to the interest or defy the cause of justice.

(c) Appeal shall be allowed as Imran’s act was only a reaction to the tweet by the MP, thus he
is not a criminal by nature and be released rather than him being incarcerated without any
substantial reason. (d) Appeal shall not be allowed as Imran is likely to abscond and then
influence the investigation if released on bail.  

Passage (Q.-Q.): The Government of National Capital Territory of Delhi (Amendment) Bill,
2021 was introduced in Lok Sabha to amend the Government of National Capital Territory of
Delhi Act, 19. The Act provides a framework for the functioning of the Legislative Assembly
and the government of the National Capital Territory (NCT) of Delhi. Restriction on laws
passed by the Assembly: The Bill provides that the term “government” referred to in any law
made by the Legislative Assembly will imply Lieutenant Governor (LG). Rules of Procedure
of the Assembly: The Act allows the Legislative Assembly to make Rules to regulate the
procedure and conduct of business in the Assembly. The Bill provides that such Rules must
be consistent with the Rules of Procedure and Conduct of Business in the Lok Sabha. Inquiry
by the Assembly into administrative decisions: The Bill prohibits the Legislative Assembly
from making any rule to enable itself or its Committees to (i) consider the matters of day-to-
day administration of the NCT of Delhi and (ii) conduct any inquiry in relation to
administrative decisions. Further, the Bill provides that all such rules made before its
enactment will be void. Assent to Bills: The Act requires the LG to reserve certain bills
passed by the Legislative Assembly for the consideration of the President. These Bills are
those: (i) which may diminish the powers of the High Court of Delhi, (ii) which the President
may direct to be reserved, (iii) dealing with the salaries and allowances of the Speaker,
Deputy Speaker, and members of the Assembly and the Ministers, or (iv) Relating to official
languages of the Assembly or the NCT of Delhi. The Bill requires the LG to also reserve
those Bills for the President which incidentally cover any of the matters outside the purview
of the powers of the Legislative Assembly.

. Legislative assembly of Delhi passed a bill to provide free water and electricity facilities to
residents of Delhi barring persons with more than 15 lakh of income. The bill passed after the
National Capital Territory of Delhi (Amendment) Bill, 2021 is enforced in Delhi. Decide
validity of decision taken by the legislative assembly of Delhi.
(a) National Capital Territory of Delhi Act, 19 provides a framework for the functioning of
the Legislative Assembly.

(b) The Bill prohibits the Legislative Assembly from making any rule.

(c) The Bill prohibits the Legislative Assembly from making any rule consider the matters of
day-to-day administration of the NCT of Delhi.

(d) The Bill prohibits the Legislative Assembly from making any rule consider the matters of
day-to-day administration of the NCT of Delhi without permission of LG.

. A Bill passed by the Legislative Assembly of Delhi without the consideration of the
President but with consideration of LG. The bill diminishes the powers of the District Court
of Delhi. Decide the legal validity of the above-mentioned bill in the context of a passage.

(a) It violates the GNTCD bill because the Act requires the LG to reserve certain bills passed
by the Legislative Assembly for the consideration of the President.

(b) It is not legal because consideration of the President is mandatory.

(c) It is legal to diminish the power of the District Court of Delhi by LG without
consideration of the president.

(d) None of the above.

. Lieutenant Governor of Delhi Shri Anil Baijal promulgated Ordinance to cut the salary of
legislators by 30 percent and this reduction is a temporary measure applicable for a year only.
Can LG do so?

(a) Yes, with the advice of the president.

(b) No, the Bill requires the LG to reserve such Bills for the President.

(c) Both A and B

(d) None of the above.  

. Is the central government taking away power from Delhi Government bypassing the
National Capital Territory of Delhi (Amendment) Bill, 2021? Answer with the context of the
passage.

(a) Yes, the LG will only enjoy all-dominating powers.

(b) Federal structure is violated because indirectly union will control NCT Delhi.

(c) Delhi assembly will not able to form any committees to scrutinize actions.

(d) None of the above . Legislative Assembly of Delhi appointed a commission of inquiry to
enquire into irregularities and fraud committed by certain persons managing the affairs of
industrial companies by changing governance tactics (Administrative decision) governed by
the Delhi government. Government of the Delhi commission is allowed or not to do above
mention inquiry? Decide.

(a) Yes they are allowed because they have powers to do so.

(b) No, they are not allowed.

(c) No, they are not allowed because The Bill prohibits the Legislative Assembly from
making any rule to enable itself or its Committees to consider the matters of day-to-day
administration of the NCT of Delhi.

(d) No, they are not allowed because The Bill prohibits the Legislative Assembly from
making any rule to enable itself or its Committees to consider the matters to conduct any
inquiry in relation to administrative decisions.

Passage (Q.-Q.): Delegation of powers means the powers passed on by the higher authority to
the lower authority to make laws. Delegated legislation means the powers given by the
legislature to the executive or administration to enact certain laws. The simple meaning of the
expression “delegated expression” may be: When the function of the legislation is entrusted
to organs other than the legislature by the legislature itself, the legislation made by such
organs is known as delegated legislation. Delegated legislation, also referred to as secondary
legislation, is legislation made by a person or body other than Parliament. Parliament,
through an Act of Parliament, can permit another person or body to make legislation. An Act
of Parliament creates the framework of a particular law and tends only to contain an outline
of the purpose of the Act. By Parliament giving authority for legislation to be delegated it
enables other persons or bodies to provide more detail to an Act of Parliament. Parliament
thereby, through primary legislation (i.e. an Act of Parliament), permit others to make law
and rules through delegated legislation. The legislation created by delegated legislation must
be made in accordance with the purpose laid down in the Act. The Legislature cannot
delegate Essential Legislative Functions which consist in the determination or choosing of the
Legislative Policy and of formally enacting that policy into a binding rule of conduct.
Essential Legislative Functions include the power to repeal or modify a law and cannot be
delegated. The underlying object of parliamentary control is to keep watch over the rule-
making authorities and also to provide an opportunity to criticize them if there is abuse of
power on their part. Parliament has control in that the enabling or parent Act passed by
Parliament sets out the framework or parameters within which delegated legislation is made.
In India, the question of control on rule-making power engaged the attention of the
Parliament. The Parliamentary control over delegated legislation in USA and India is not as
effective as in UK. [Source: Amit Kumar, ‘Delegated Legislation Development and
Parliamentary Controls’, Legal Services India, , as accessed on 20th June 2021]  22 of 40 . In
March 2020, Parliament enacted an act titled Taxes on Exports Act 2020 which gives
discretionary power to State governments to decide upon the tax slabs and other rules for
items exported from their respective states. One of the provisions of the Act states that
“Every State shall have the power to decide upon the respective tax slabs on the list of items,
given under Schedule I of the Act, to be exported from the state after 1st May 2020”. The
State of Uttar Pradesh passed Taxes on Export Rules 2020 where the tax slab for all items is
fixed at 20% of their MRPs. Many manufacturers and businessmen contest the Rules set out
by the government to be arbitrary and misuse of power. Also it was contended that the Rules
must be struck down as they are ultra vires and beyond the power of the State Legislature.
Decide –

(a) The Contention was incorrect as the power to decide the tax slabs is being given to the
State governments and the act is not ultra vires the delegated legislation.

(b) The Contention was correct as the power to decide the tax slabs is being given to the State
governments but the act is ultra vires the delegated legislation.

(c) The Contention was incorrect as the power to decide the tax slabs is not an essential
legislative function and it can be delegated to the state governments.

(d) The Contention was correct as the power to decide the tax slabs is not an ancillary
legislative function and it cannot be delegated to the state governments.

. In the year 2016, the Parliament of India enacted Insolvency and Bankruptcy Code where
only certain parts of the code were made effective for the time being. Section 4 of IBC reads
as “This Part shall apply to matters relating to the insolvency and liquidation of corporate
debtors where the minimum amount of the default is one lakh rupees; Provided that the
Central Government may, by notification, specify the minimum amount of default of higher
value which shall not be more than one crore rupees”. With regards to this, the Central
Government passed a notification dated March 2020, where the minimum default amount
was reduced to Rs. 50,000 due to the COVID Pandemic situation. Many corporate bodies
criticized this move and claimed the power delegated to be excessive in nature. Decide –

(a) The power given to the Central Government to notify the default amount timely is not an
essential legislative function and it can be delegated validly. There is no excessive delegation.

(b) The power given to the Central Government to notify the default amount timely is an
essential legislative function and it cannot be delegated validly. There is excessive delegation.

(c) The power given to the Central Government to notify the default amount timely is not an
ancillary legislative function and it can be delegated validly. There is no excessive
delegation.

(d) The power given to the Central Government to notify the default amount timely is an
ancillary legislative function but it cannot be delegated validly. There is excessive delegation.

. The Uttar Pradesh Cabinet on 8th May, 2020 approved the Uttar Pradesh Temporary
Exemption from Certain Labour Laws Ordinance, 2020. The Ordinance exempted all
factories and establishments engaged in the manufacturing process from compliance under 35
out of 38 Central Labour Laws for a period of 3 years in lieu of Covid 19 pandemic. People
of Uttar Pradesh contested the suspension on the grounds of excessive delegation and misuse
of power. Also the State government was not having such power of suspension to which the
authorities replied that The Factories Act and the Payment of Bonus Act allow suspension
under the Act for a maximum period of 3 months and for a public emergency. Decide –
(a) The Ordinance passed by the Uttar Pradesh Government is ultra vires and should be
struck down as it was in excessive delegation of power.

(b) The Ordinance passed by the Uttar Pradesh Government is not ultra vires and should not
be struck down as it was delegation of ancillary legislative functions and not excessive
delegation of power.

(c) The Ordinance passed by the Uttar Pradesh Government is not ultra vires as power to
suspend the Central Laws is given explicitly and is not excessive delegation of power.

(d) The Ordinance passed by the Uttar Pradesh Government is ultra vires as there is
delegation of essential legislative function and it should be struck down with immediate
effect.  

. Section 5 of Essential Commodities Act, 15 states that “The Central Government may, by
notified order, direct that the power to make orders or issue notifications under section 3
shall, in relation to such matters and subject to such conditions, if any, as may be specified in
the direction, be exercisable also by such officer or authority subordinate to the Central
Government, or such State Government or such officer or such authority subordinate to a
State Government, as may be specified in the direction”. In lieu of the same, the State of
Andhra Pradesh directed Municipal Authorities of Telangana to frame certain rules and
procedures to regulate the pricing, selling and exporting of essential commodities in the state.
Telangana Municipal Corporation enacted Rules 2021 which was contended to be ultra vires.
Decide –

(a) The Rules framed by the Telangana Municipal Corporation was ultra vires the Essential
Commodities Act as the power delegated was excessive delegation to the authorities.

(b) The Rules framed by the Telangana Municipal Corporation were not ultra vires as the
Essential Commodities Act explicitly states about such power to be delegated to local or
subordinate authorities to State Governments. There was no excessive delegation to the
authorities.

(c) The Rules framed by the Telangana Municipal Corporation were ultra vires as the State
government does not have the power to delegate. It is only with the Central Government to
delegate this and hence was excessive delegation to the authorities.

(d) None of the above.

. Which among the following is a suitable example(s) of only delegated legislation?

I. Central Government under Sec.7 of Companies Act notified the requisite list of documents
for memorandum at time of Incorporation of the Company.

II. Section 44A of Civil Procedure Code gives power to the Central Government to notify in
the official gazette as to inclusion or exclusion of reciprocating territory.
III. Section 65 of the Competition Act gives power to the Central Government to pass and
publish an order in the official gazette to remove any difficulty arising giving effect to the
provisions of the Act.

IV. Section 4 of the Companies Act gives power to the Central Government to make rules for
carrying out the provisions of the Companies Act 2013. The Ministry of Corporate Affairs
framed Companies (Incorporation and Dissolution) Rules.

(a) Both I and II

(b) Only III

(c) All of the Above

(d) I, III and IV

. Delegated Legislation in India has not been observed and explored much as compared to the
UK. Delegated Legislation needs more growth and concrete legislation to regulate the
administrative affairs in India. What can be the valid reason behind the concept of Delegated
Legislation?

(a) Subject matter of legislation is technical in nature. So, assistance from experts is required.
The legislative power may be conferred on experts to deal with the technical problems.

(b) Parliament can foresee all the contingencies while passing on enactment. To satisfy these
demands some provisions are required to be made. A legislative amendment is a swift
process.

(c) In times of emergency, quick action is required to be taken. The legislative process is
equipped to provide for urgent solutions to meet the situation.

(d) The complexity of modern administration and the expansion of the functions of the state
to the economic and social sphere have rendered it unnecessary to resort to new forms of
legislation and to give wide powers to various authorities on suitable occasions.  

Passage (Q.-Q.): Read the following passage and in the light of the same answer the
questions that follow. Section 161 Code of Criminal Procedure, 19 headed as “Examination
of witnesses by police” provides for oral examination of a person by any investigating officer
when such person is supposed to be acquainted with the facts and circumstances of the case.
The purpose for and the manner in which the police statement recorded under Section 161
can be used at any trial are indicated in Section 162. This section empowers the police to
examine witnesses during the course of an investigation. Any person who is supposed to be
acquainted with the facts and circumstances of the case may be examined orally. The words
“any person” used in Section 161 (1) also include a person who may be accused of the crime
and suspects. This section as well as Article 20(3) of the constitution gives protection to such
person against questions exposing him to a criminal charge. Article 20(3) lays down a
fundamental right that no person accused of any offence can be compelled to be a witness
against himself. In Nandini Satpathy v. P.L Dani, the Supreme Court held that the accused
person cannot be forced to answer questions merely because the answers thereto are not
implicative when viewed in isolation and confined to that particular case. No oath, sign or
affirmation is required in an examination of witnesses under this section. It is not mandatory
for the investigating officer to reduce in writing the statement of the person examined as
sub[1]section (3) prohibits making of précis of a statement recorded under Section 161 of the
Code. It is also provided that statement made under this sub-section may also be recorded by
audio-video electronic means. According to the law of evidence this is not evidence of fact,
so it cannot be treated as substantive evidence. Section 162 bans the use of the statements
collected by the police in the course of investigation for corroboration.

. Vijay Malia, a renowned businessman, has been alleged to commit fraud with the major two
banks of India. ICICI Bank pressed charges for forgery and criminal conspiracy as he
intentionally deposited all his money in the Swizz Bank Account and went to Ireland as India
does not have the MOU signed with the country to hand over their criminal offenders back.
Also, the State Bank of India alleged him for the same charges. The amount to be recovered
from Malia is in thousand crore. An intelligence report informed the Custom and Excise
officers that Vijay Malia will be visiting India in the name of Arvind Sahoo to take back his
authentic documents. He will be taking his flight back from Bombay Airport. Custom
Officers caught a person named Arvind Sahoo and enquired him. They have investigated his
visa and passport too. They have taken him into custody. In the trial, the authorities used this
information against Mr. Arvind. Decide –

(a) The statements made by Mr. Arvind to the Custom Officers cannot be under section 161
of the code as they have enquired him with no evidence and out of custody.

(b) The statements made by Mr. Arvind to the Custom Officers can be under section 161 of
the code as they have investigated him and such statements can be used to contradict the
accused.

(c) The statements made by Mr. Arvind to the Custom Officers cannot be under section 161
of the code as they have not been taken as per the said requirements of the section.

(d) The statements made by Mr. Arvind to the Custom Officers can be under section 162 of
the code as they have been used to contradict the accused and not for corroboration.  

. Manoj and Arti were classmates. Manoj used to like her. He also proposed to her on
Valentine’s Day but Arti refused him. She doesn’t want to be in such a relationship as of
now. Manoj was very upset by this and he planned to murder Arti as she has done wrong with
him. He invited her anonymously to his farm house and blackmailed her to share her vulgar
pictures. Arti went to the farm house and was murdered there. On investigation, the local
Police Officers enquired the nearby Tea Seller about the crime scene. The seller stated that he
saw a boy with black shirt and a mask on his face having a knife in his hand and washing it
on the roadside. It seems like there was a lot of blood on the knife. The Officers checked the
CCTV footage and saw no such boy with the knife. After a week, Police arrested Manoj and
was taken into custody. In the trial, the police presented the Tea Seller as a witness and stated
that he saw Manoj cleaning the knife after murder. Decide –
(a) The statement of the tea seller cannot be taken into consideration by the Court as it
discloses facts related to the case and hence is not a piece of substantive evidence.

(b) The statement of the tea seller cannot be taken into consideration by the Court as there
was no such footage covered in the CCTV camera when checked by the Officials and hence it
is not substantive evidence.

(c) The statement of the tea seller can be taken into consideration by the Court as the
statement is covered under the scope of section 161 of the code and can be used to determine
the facts of the case.

(d) The statement of the tea seller can be taken into consideration by the Court as there is a
probability that the Court can inquire Manoj on these lines and need not completely rely on
these statements.

. Sheetal and Suraj were married to each other. They were happy in their relationship. Sheetal
was also in good relations with her friend Anup. Suraj doubted their relation and planned to
kill her and take his revenge. He poisoned her food and she died. On investigation, Police
Officers get to know about this arrangement and arrest Suraj as suspect. Suraj stated that he
was in the city but was out for a business meeting at his friend’s house the night Sheetal was
murdered. In the trial of the case, Suraj was called as a witness and was enquired. He stated
that he was not in the city. The Police Officer contested that Suraj is lying as the statement he
made during investigation was different. Decide –

(a) Statement made by Suraj to the Police Officer is covered under section 161 of the code
but cannot be used at time of trial for contradiction.

(b) Statement made by Suraj to the Police Officer is not covered under section 161 of the
code and cannot be used at time of trial for contradiction.

(c) Statement made by Suraj to the Police Officer is covered under section 161 of the code
and can be used at time of trial for contradiction.

(d) Statement made by Suraj to the Police Officer is not covered under section 161 of the
code but can be used at time of trial for contradiction.

. Meenal and Sunita were good friends from their college times. Sunita helped Meenal a lot in
her financial problems and also got her a job. Meenal had taken a loan of Rs.1,00,000 from
Sunita and never thought of returning it back. On being asked by Sunita, Meenal stated that
she won’t be able to pay the same but can pay only 20,000 out of this amount. Sunita was in
urgent need of the money and was aware that Meenal is creating the drama. Sunita went to
her home and found her to be alone. She killed Meenal with a sharp long knife and ran away
with some jewellery. On investigation, the Police Officer found the CCTV recording where
Sunita was seen and she was the last person to meet Meenal. Sunita was taken into custody
and after a week she stated the truth to the Police Officers. In the trial, the prosecution lawyer
cross-examined her and asked her about the truth. She moulded the facts and refused to
confess her guilt. The Police personnel submitted the statement made by Sunita in custody.
Decide – (a) Statement made by Sunita to the Police Officer is covered under section 161 of
the code and can be used at time of trial for corroboration.

(b) Statement made by Sunita to the Police Officer is not covered under section 161 of the
code and cannot be used at time of trial for contradiction.

(c) Statement made by Sunita to the Police Officer is covered under section 161 of the code
but cannot be used at time of trial for contradiction.

(d) Statement made by Sunita to the Police Officer is not covered under section 161 of the
code but can be used at time of trial for corroboration.  

. Choose the appropriate option.

Assertion (A): Article 20(3) of the Indian Constitution provides right against self
incrimination i.e no accused can be compelled to be a witness against himself/herself.

Reasoning (R): Article 20 also provides for the right to remain silent as the protection extends
to the person accused not to speak or answer questions which are inculpatory.

(a) Both A and R are correct and R is the correct explanation of A.

(b) Both A and R are incorrect.

(c) Both A and R are correct but R is not the correct explanation of A.

(d) Only A is correct and R is incorrect.

. Which among the following statement(s) is/are not true with regards to section 161 of the
code? I. If a statement is recorded by police, under the provisions of Section 162 the accused
must not be asked to sign the same. Though it is not barred by the section, but violation of
this provision may sometimes diminish the value of the testimony of the witnesses when they
come to court. II. If some statement has been recorded of a person Y, such statement can be
used to contradict or to support such witness in the later stage of trial. III. If a person X is
supposed to be acquainted with the circumstances of a case, a police officer investigating the
case is allowed to examine such a person orally and may reduce the statement made by such
person into writing. IV. If a person C was asked questions self-incriminatory in nature during
investigation by police, such a person is not bound to answer those questions by the virtue of
Section 161 and 162 of the code.

(a) Both I and II

(b) Only III

(c) Both II and IV

(d)I, III and IV

 
Passage (Q.-Q.): The Information Technology (Intermediary Guidelines & Digital Media
Ethics Code) Rules, 2021 ["2021 Rules"] make a distinct departure from the erstwhile 2011
Rules while dealing with obligations and duties of intermediaries by classifying them into
two groups namely, social media intermediary and a significant social media intermediary.
Rule 2(w) defines social media intermediary as an intermediary which primarily or solely
enables online interaction between users and allows users to create, upload, share information
using its services. A significant social media intermediary means a social media intermediary
having registered users above the threshold as notified by the Government, which is 50 lakh
registered users. Under Rule 3(1)(d) of the 2021 Rules, an intermediary after receiving 'actual
knowledge' through a court order or by being notified by a government agency, is bound to
remove information that is prohibited by law within a duration of thirty-six hours. Therefore,
Rule 3(1)(d) essentially prescribes a procedure for take down/blocking of content in the
public domain. Rule 3(2)(b) of the 2021 Rules require an intermediary upon receipt of a
complaint made by an individual to remove any content which prima facie shows explicit
content relating to that individual within a period of 24 hours. Rule 4(2) entails at some stage
the decryption of information by the intermediaries such as WhatsApp or Telegram on whose
platform the objectionable content has been shared or transmitted as those platforms use End-
to-End encryption technology. Rule 4(2) of 2021 Rules casts the obligation on the
intermediary to provide the decrypted information sought for, irrespective of whether it is in
possession of the decryption key or not.

. Quitter is a popular social media platform with nearly 49 lakh registered active users per
month. X posted the contact details of the National Security Advisor on Quitter, even though
the said details were confidential, sensitive and not meant for the public eye. X obtained the
details while he was trying to schedule an interview with the National Security Advisor. As
soon as X posted the details, Quitter received a notice from the Department of Defence to
remove the post. Is Quitter liable to remove the post?

(a) No, because Quitter is not a significant social media intermediary as it has less than 50
lakh registered users

(b) No, because although Quitter has received a notice from the Department of Defence, it
does not have 'actual knowledge' of the offence under the 2021 Rules

(c) Yes, because Quitter is a significant social media intermediary and is subject to Rule 3(1)
(d) of the 2021 Rules

(d) Yes, because Quitter has received actual knowledge to remove the contact details which
were prohibited to be shared in the first place

. In the previous question, assume that the Department of Defence did not send any notice to
Quitter, and it is not inclined to do so in the future either. X's post containing the personal
contact details of the National Security Advisor is still up on his profile. Is there any other
provision which is prescribed under the 2021 Rules, that can be invoked to remove the post?

(a) Yes, Rule 4(2) of the 2021 Rules can be invoked by the National Security Advisor
(b) Yes, Rule 3(2)(a) of the 2021 Rules can be invoked by the National Security Advisor

(c) Yes, Rule 3(2)(b) of the 2021 Rules can be invoked by the National Security Advisor

(d) None of the above

. A and B were suspected to be a part of Lashkar-e-taiba, a notorious terrorist group. The


police had received special intel regarding a bomb blast. They were suspicious that A and B
may be a part of the conspiracy. Since A and B were free men, the police could not directly
track their phones. In the hope of finding something to quash the alleged terrorist conspiracy,
the police ordered WhatsApp to decrypt all the conversations, if any, between A and B. Are
the police empowered to do so under the 2021 Rules?

(a) No, because WhatsApp uses end-to-end encryption technology which makes it impossible
to share private conversations

(b) No, because a private conversation between A and B cannot be deemed as “objectionable
content”

(c) Yes, because decryption of the WhatsApp conversation between A and B is made possible
by Rule 4(2)

(d) Yes, because A and B are terrorists and decryption of their conversation is essential for
national security

. Disha Ravi was a student activist. She was extremely concerned about the procedural
safeguards in place to protect the people protesting against a contentious law. In an attempt to
raise awareness and provide strategies on how others could help the cause, she edited a freely
available toolkit and circulated it among her contacts and posted it on social media. Among
other things, the toolkit encouraged people in other countries to protest in front of the nearest
Indian Embassy in their country and also provided links to the websites of various
stakeholders participating in the protest. Next day, the police arrested her for sharing
"objectionable content that threatened India’s international position and national security”.
Now, the Government seeks access to her conversations and has asked Whatsapp to provide
that information by any means necessary. Is Whatsapp liable to comply with this directive
under the 2021 Rules?

(a) Yes, because intermediaries on whose platform the objectionable content has been shared
are not obligated to provide decrypted information

(b) Yes, because intermediaries on whose platform the objectionable content has been shared
are obligated to provide decrypted information

(c) No, because WhatsApp is not in possession of the decryption key in light of the
technological safeguards in place

(d) No, because no decryption is required as Disha posted the toolkit on her public social
media account  
 

. Shyama and Sundar were good friends. However, the two of them got in a fight when
Shyama refused to help Sunder cheat in their college examinations. To get back at Shyama,
Sunder posted some morphed explicit pictures of Shyama on his Instagram page. Shyama
immediately filed a complaint with Instagram however, even after 36 hours, Instagram failed
to take those pictures down. Is Instagram in violation of the 2021 Rules?

(a) Yes, Instagram is in violation of Rule 3(1)(d) of the 2021 Rules as it failed to take the
pictures down even after 36 hours (b) Yes, Instagram is in violation of Rule 3(2)

(b) of the 2021 Rules as it failed to take the pictures down even after 36 hours

(c) Yes, Instagram is in violation of Rule 3(2)(b) of the 2021 Rules as it failed to take the
pictures down even after 24 hours

(d) Yes, Instagram is in violation of Rule 3(1)(d) of the 2021 Rules as it failed to take the
pictures down even after 24 hours

Passage (Q.-Q.): The Supreme Court in the late s and early s developed a strong definitive
jurisprudence around the restricted use of handcuffs, which culminated into directives that
require agencies of law enforcement to ask the court’s permission before handcuffing a
person. In Sunil Batra vs Delhi Administration, the Supreme Court held that Article 21
forbids deprivation of personal liberty except in accordance with the procedure established by
law and curtailment of personal liberty to such an extent as to be a negation of it would
constitute deprivation. The court held that the minimum freedom of movement which even an
under-trial prisoner is entitled to under Article 19 of the Constitution cannot be cut down
cruelly by application of handcuffs or other hoops. In Prem Shankar Shukla vs Delhi
Administration, the court examined the rationale behind fetters and held that prima facie
handcuffing is inhuman and hence unreasonable as well as arbitrary in absence of fair
procedure and objective monitoring. “To prevent the escape of an under-trial is in public
interest, reasonable, just and cannot, by itself be castigated. But to bind a man hand and foot,
fetter his limbs with hoops of steel, shuffle him along in the streets and stand him for hours in
the courts is to torture him, defile his dignity, vulgarise society and foul the soul of our
constitutional culture.” The court observed that when there is no compulsive need to fetter a
person’s limbs it is sadistic, capricious, despotic and demoralising to humble a man by
manacling him. Thus, the court concluded that handcuffs must be the last refuge and not a
routine regimen.

. Khalid was detained by the police under the Terrorist and Disruptive Activities (Prevention)
Act (TADA) in Guwahati at a hospital. He had been admitted to the hospital as he was
suffering from an acute gastric disorder. There, Khalid had been handcuffed to his bed to
prevent any possibility of escape as the charges levelled against him were related to terrorism
and hence, very serious in nature. This measure was in addition to the room being bolted and
several policemen were also guarding the room. Is the handcuffing of Khalid legal in this
situation?

(a) Yes, because Khalid had severe charges against him and handcuffing him to his bed was
necessary for public safety
(b) No, because other extreme security measures like police guards and bolting the room have
already been taken

(c) No, because the bare minimum right to freedom of movement that Khalid is entitled to is
being violated

(d) Both (b) and (c)  

. In the previous situation, Khalid continues to be chained to his hospital bed. One of his legs
is cuffed through iron fetters on the railing of the bed. However, his remaining limbs are free.
Whenever he needs to use the washroom, an armed guard unlocks his fetters and escorts him
to the washroom. The washroom has no window and hence, there is no possibility of escape.
Is it legal to escort Khalid to the washroom without any iron fetters?

(a) Yes, because handcuffs must be the last refuge and there is no need to use handcuffs while
escorting Khalid to the washroom

(b) Yes, because handcuffing Khalid to his bed is in violation of Articles 19 and 21

(c) No, because prima facie handcuffing is inhuman and unreasonable as well as arbitrary in
absence of fair procedure

(d) No, because preventing the escape of an under-trial is in public interest, reasonable, just
and cannot, by itself be castigated

. Parminder and Joginder were detained in relation to their involvement with the Delhi riots
of 2020. Prior to being produced before the Court, the government sought permission to
produce them in handcuffs. The government justified this request by saying that the detainees
were to enter a public place with a lot of innocent civilians and were hardcore activists of a
banned organisation and were accused of terrorist and disruptive activities. Is this a sufficient
ground for handcuffing them?

(a) Yes, because it is in accordance with the procedure established by law

(b) Yes, because there is a compulsive need to fetter Parminder and Joginder

(c) Yes, because preventing the escape of detainees is in public interest, reasonable, just and
cannot, by itself be castigated

(d) None of the above

. Pikachu Dubey was a notorious criminal operating in Uttar Pradesh. After years of being
hidden outside police radar, the police received a tip that he was hidden in a temple in
Madhya Pradesh. After reaching the venue, a brief scuffle ensued between the police and
Pikachu Dubey. He was finally captured and stripped of all his weapons. He was arrested for
his involvement in over a murder, rape and drug trafficking cases. Can the police handcuff
him while transporting him to a jail in Uttar Pradesh?

(a) Yes, because he has been charged with grave offences such as murder, rape and drug
trafficking

(b) Yes, because he is violent and there has already been a scuffle between Pikachu and the
police

(c) No, because he has already been stripped of all his weapons and there is no compulsive
need to handcuff him

(d) No, because Pikachu Dubey is not a threat to society

. A few days after the police arrested Pikachu Dubey, they received information about the
whereabouts of his equally notorious criminal brother Vikas Dubey. The police had arrested
Vikas in several instances but he always managed to escape before reaching the venue. They
managed to arrest Vikas and cleared the entire route to the police station to avoid any mishap.
This time, they also handcuffed Vikas to prevent escape in transit. Is this legal?

(a) Yes, because Vikas’s right to personal liberty and freedom of movement is not being
violated

(b) Yes, because Vikas had a prior record of escaping and which creates a compulsive need to
handcuff him

(c) Both (a) and (b)

(d) No, because handcuffing must be a last resort and the police can adopt less stringent
measures

42
Passage (Q.67-Q.): The 5-judge bench of the Calcutta High Court on Tuesday continued its
hearing in the Narada case. Sol-Gen. appearing for the CBI argued that a Court proceeding
must not only be free of any Judicial-bias but must also reflect as being such to the common
man. "Principle of 'Justice must seem to be done' is engrained in Jurisprudence of many
countries, including ours. This is more so in criminal proceedings, which are in rem and
affect fabric of the society in large," he argued. The proceedings included the factual matrix
being discussed followed by an account of protests being led in front of the court complex by
radicals. The effect of terror engineered outside CBI office and Court had an effect not only
on investigating agency but also on administration of justice," he submitted. He stated that
mobocracy has not been disputed as this is admitted that members of legislative assembly
were protesting there. He insisted that these events are reasonably likely to leave an
impression on the minds of the common man that the Justice system was stifled with. “In our
country, there is a concept of open Courts. In our democracy, peaceful assembly and protests
are allowed against executive action. So how does it create an impression in common man
that such a protest will influence the judiciary?" the judge asked. "It is not about what the
Judge felt, he may not have known that people have gathered outside. But it is about what a
reasonably informed common man would feel”

67. A minister of local renown in Safdarjung area of Delhi is arrested over embezzlement of
funds from the central reconstruction project. On the day of his hearing, his followers gather
around his house, where they intend to silently protest against his arrest. However, as the
hearing scheduled is delayed by 10 minutes, they become irritated upon waiting and thus,
fisticuffs start being noticed between them and local vendors near his house. Could this
violence be construed as being influential on the decision of the court?

(a) The hearings are being conducted virtually; therefore, no question of the decision being
‘influenced’ by the protestors arises.

(b) The protestors went with a peaceful intent; therefore, no common man would have
thought the decision to be influenced by the protestors. Silent protests cannot be equated with
heated protests.

(c) The intent of the protestors being peaceful, the subsequent fisticuffs after a mere 10
minute delay could easily be construed as bearing influence on the decision.

(d) The protestors got into fisticuffs with the vendors outside his house, and near the court
complex, in which case, influence could have been noted.

68. A matter is listed for hearing in the Delhi HC wherein a three judge bench is formed
consisting of Justice X, Y & Z. The parties in question, A and B, appear before the court in
furtherance of seeking redressal of a tenancy dispute between the two, wherein A is the
landlord and B is the tenant in the said dispute, it is apparent that B would win, however, in a
desperate bid to win, A files for the same. As it turns out, A is shocked that X, his sister in
law, is actually one of the judges hearing the matter. Upon learning this, A relaxes and
becomes assured that the matter is bound to be decided in his favour, however, the matter is
decided in B’s favour, reinstating him as the tenant, with the ratio of 2:1. Is this a case of the
judiciary being bias-free?

(a) The matter was adjudicated in the favour of B, and therefore, no apparent bias can be
made out.

(b) Notwithstanding the ratio, the matter was riddled with bias.

(c) There could have been no possibility of ratio being biased as B would anyway have won,
since X could not have influenced Y and Z to vote in his favour.

(d) Nemo judex in sua causa should be upheld, and the matter should be relisted. Head
Office:

. The Enforcement Directorate has been tasked with investigating the matter of Chinmay
Moghul, an economic offender who absconded from India after defrauding the Indian Bank
of Saparghat. Being the promoter of a billion-dollar company, he was well-known by the
Indian masses, as a result of which, when he was being questioned at the HQ of the ED,
hundreds of people gathered around the ED to protest against the investigative actions of the
ED. When the matter reached the courts, people from across the country, who were employed
by his company, went on a strike in front of the ED. Could the decision of the judiciary be
influenced by such actions of the employees?

(a) The strike that took place outside ED cannot be termed to be influential on the courts as
they did not have any nexus with the courts. The protesters were unhappy.

(b) The strikes that were taking place would act as influential factors upon the bearing of the
decision of the courts.

(c) The strikes were happening so far away from the courts, that they could not have been
influential on the court proceedings.

(d) The ED acts as an investigating authority, therefore, the procedure for the judiciary to
follow does not get affected.

. Upon committing robbery, Dhaniya and his accomplices are caught and are kept in police
custody before being presented before the magistrate. When his family gets to know about
this, four of them storm into the nearest police station and refuse to leave until Dhaniya is
acquitted honorably. Could such disruption in the police functions of the investigating
authority be termed as bearing influential upon the proceeding before the investigating
authority?

(a) The police are more than capable of dealing with 4 unarmed civilians; this would not end
up influencing the magistrate.

(b) The fabric of criminal law terms such factors as being influential since it affects the
society in rem.

(c) The family members have a reasonable right to protests however; this shouldn’t bother the
investigating authority.

(d) The police cannot proceed with their policing functions if the civilians act in a haphazard
manner and threaten to disrupt their working.

Passage (Q.-Q.): The Allahabad HC held that irrational, indiscriminate arrests are a gross
violation of human rights, accepting an anticipatory bail plea in a corruption case. The courts
have repeatedly held that arrest should be the last option for the police and it should be
restricted cases where arresting the accused is either imperative or where custodial
interrogation is required. There was an allegation against him that he was taking money from
passing trucks, an FIR was lodged against him under Prevention of Corruption Act, 19.
Counsel for the applicant submitted that there is no forensic report and probe into the matter
is still underway and he has definite apprehensions that he may be arrested by the. “After the
lodging of an FIR, the arrest can be made by the police at will. There is no definite period
fixed for the police to arrest an accused against whom an FIR has been lodged. Irrational and
indiscriminate arrests are gross violations of human rights,” the court observed. “In the event
of arrest, the applicant shall be released on anticipatory bail till cognizance is taken by the
court on the police report, under section 1 CrPC before the competent court. The HC said that
he shall make himself available for interrogation by the police officer when required. He shall
not, directly or indirectly, make any inducement, threat or promise to any person acquainted
with the facts of the case to dissuade from disclosing such facts to the court or to any police
officer. The court said the applicant shall not leave India without the previous permission of
the court and if he has a passport. Source: https://thewire.in/law/irrational-indiscriminate-
arrests-are-gross-violation-of-human-rights[1]allahabad-hc Head Office:  

. Amit is a construction worker involved in contracted employment at a construction site.


There he works with 30 other such workers, out of which he does not get along with Ranjit,
often getting in fisticuffs with him. One morning, while on his way to work, he is called by
his foreman to come quickly as there has been an accident. When he reaches there, he sees
that the police is present there, talking to the foreman. He is told by another worker that
Ranjit has been killed and has been found dead behind the site itself. The police start
questioning Amit regarding the same, and arrest him for further questioning in the station. Is
the police correct in doing so?

(a) The police are correct in arresting him, as they can question him for being linked to the
murder as he did not have good relations with Ranjit and is a prime suspect.

(b) The police are incorrect in doing so, as there was no reason to arrest him.

(c) The police are correct in arresting him as they have reason enough to suspect them.

(d) There is no evidence on record to show that Amit should be termed as a suspect to be
taken into custodial interrogation, hence, the police are incorrect in arresting him.

. Raman is an Indian journalist who is known for his coverage of wartime nations and human
rights issues. When India is at the brink of waging war with Bangladesh, an emergency is
declared with complete national lockdown except for army and medical personnel, he quickly
flies out to the border, and tries to jump the frontlines to get a better view of what is
happening. Amid all the commotion, he gets trapped in a pit and is then rescued by the army
personnel stationed nearby. Upon being rescued, he is arrested by the police force stationed
near the army camp and is straight away locked up until he can be presented before the
jurisdictional courts. Could he challenge the action of the police as being discriminate and
irrational?

(a) He cannot challenge the action of police as he was in a very sensitive area without proper
cause or legal means.

(b) He cannot challenge the action of the police as they were right in taking such quick
action.

(c) He cannot challenge the action of the police as they were acting in furtherance of the
orders given to them.

(d) He cannot challenge the action of the police as the border was a very sensitive area and he
could have been a spy or an enemy solider.
 

. Surabjo is one of the prime accused in a case of sedition and has been suspected of inciting
rebellion in the minds of impressionable youth to riot against the Central Government,
although no evidence of the same is found. However, he absconds from police custody a
night before his appearance before the Trial Court. Two years later, a group of politically
driven individuals aligning with the Central Government, find him near a fruit vendor’s shop
and bring him straight to the police officials. The police arrests him at once and produces him
before the magistrate where he pleads that he has been wrongfully arrested. Can his action
sustain?

(a) His action will not sustain as he is rightly accused of inciting sedition in the youth.

(b) His action will sustain as he has been arrested irrationally.

(c) His action will not sustain as the police is rightly exercising their discretion in arresting
him.

(d) His action will sustain as a lot of time has passed since his last arrest, and the police lack
evidence.

. Ajmal Wasabi is a Jharkiswani terrorist, a nation that India troubles with for a couple of
years. Owing to the troubles his group of militants have caused in the northern region of
India, the government releases a press release with his name as being affiliated to a dangerous
terrorist group called ‘Wingbuts of Jharkiswan” and announces immediate arrest and
detention for him, and a prize money of 24 lakhs for any person having information about
him. This creates uproar in the news channels and they run to broadcast the press release. One
news channel, DNTV Ahsas, runs the press release but fails to mention the full name of the
terrorist and only mentions the first name of the terrorist. When Subedaar Khan, a constable
of impeccable record, comes to know of this, he goes on a lookout for him. After a week long
search, he finally decides to return to his station when he finally hears someone calling out
the name ‘Ajmal’ after someone running with a bag full of explosives. Subedaar rushes after
him and catches him Head Office:  by tackling him into the ground and arrests him. Upon
subsequent questioning, he gets to know that this is not the terrorist they were looking for, but
is only a local goon. Could Ajmal allege the actions of Subedaar as being a gross violation of
human rights?

(a) Subedaar was acting irrationally by arresting the first person he saw by the name of
Ajmal. His actions are a gross violation of human rights.

(b) Subedaar was rightly under the impression that the person is a terrorist. His actions were
not irrational or discriminate and, as a result, not in violation of human rights.

(c) Subedaar is right in arresting him, even if Ajmal was not a terrorist, he still arrested a
goon.

(d) Subedaar was correct in arresting him as he was only carrying out his public duty towards
the society at large.

 
Passage (Q.-Q.): The legal basis for challenging internet shutdowns in India is solid but does
not result in relief for the citizens despite several rounds of litigation. In a landmark case last
January, the Supreme Court recognised that the fundamental right to freedom of speech and
expression under Article 19(1)(a) of the Constitution, and the right to carry on any trade or
business under 19(1)(g), using the medium of internet is fully protected. However, reasonable
restrictions on fundamental rights continue to operate. Although the court held that
suspending internet services indefinitely is impermissible, this was not applied to the
shutdown in Kashmir that had been continuing for more than five months at that time. The
only relief granted was a direction given to the state to review all orders suspending internet
services forthwith. The Court did recognise that the ban constituted an indefinite ban on the
internet. In view of the fundamental nature of the rights at stake, the Court has also
emphasised the requirement of transparency, mandating that shutdown orders be published.
Nonetheless, most of the orders are not published nor is the public informed of any imminent
shutdown. Most districts in Kashmir continue to have merely 2G access since March 2020,
further complicating lives during the pandemic. The government’s 2017 rules concerning
“Temporary Suspension of Telecom Services” provide for a review of shutdown orders every
15 days. But the review committee is staffed entirely by officers of the executive and
continues to extend orders every fortnight.

. Lumbini was a small town on the outskirts of Maharashtra. The town had become notorious
due to frequent bouts of communal violence that broke out amongst the people. The source of
this communalization was the construction of a church on a site that was previously a
mosque. Since the decision of the Court was pending on this front, the local government
ordered a complete ban on internet till the Court gave its news to prevent the spread of fake
news. It has been 2 years and the decision is still pending. Is the order of the local
government a constitutional exercise of power?

(a) Yes, because it is a reasonable restriction on the fundamental right to use the internet.

(b) No, because it is not a reasonable restriction on the fundamental right to use the internet.

(c) Yes, because internet is not being banned indefinitely but only till the Court delivers a
verdict.

(d) No, because in effect the internet is being shut down indefinitely in Lumbini as the
decision of the Court remains uncertain. Head Office:  

. Brijwasi is a local artisan. He recently started selling his arts and crafts online. The business
was going well and now a majority of his sales happen online only. Due to a fault in the
network lines, the local municipality ordered the re-laying of cable lines. This disrupted
internet in the area for two weeks. In this time, Brijwasi lost business worth Rs. 25,000. Can
he sue for a violation of fundamental rights?

(a) Yes, because internet access is a fundamental right

(b) Yes, because the right to carry on any trade or business under Article 19(1)(g), using the
medium of internet is fully protected
(c) No, because the ban on internet is a reasonable restriction on fundamental rights

(d) None of the above

. Due to an imminent threat of a terrorist attack, the government was supposed to take
immediate measures in national interest. This included a temporary shutdown of internet
services to try and disable the network of the alleged terrorist. However, the length of this ban
couldn’t be predicted till the crisis was averted. The government was so busy trying to
manage the situation that internet was shutdown suddenly without any notice. Can citizens
appeal against this decision?

(a) Yes, because it is a violation of the government’s duty to be transparent;

(b) Yes, because shutdown orders must mandatorily be published;

(c) No, because it is a reasonable restriction on fundamental rights;

(d) No, because the inability to publish the notice was due to an imminent security threat that
affected national interest.

. In an alternate situation, in the previous question the government announced the ban within
2 hours of declaring it via loudspeakers. They asked the people to stay inside their homes as
there was a terrorist threat. However, due to a paucity of time, the notice could not be
published on paper even though it was announced through loudspeakers in all public places.
Can the citizens still protest against this decision?

(a) Yes, because the court has emphasised the requirement of transparency in announcing
internet bans;

(b) Yes, because the ban was announced on loudspeakers in public places;

(c) No, because the inability to publish the notice was due to an imminent security threat that
affected national interest;

(d) None of the above.

Passage (Q.-Q.): In a short judgment regarding the petitions filed earlier in the year against
the Shaheen Bagh protests, the Supreme Court has held that “public ways and public spaces
cannot be occupied in such a manner and that too indefinitely.” While acknowledging the
right to dissent, the court stated that “demonstrations expressing dissent have to be in
designated places alone.” Noting the inconvenience caused to commuters by the protesters at
Shaheen Bagh, the judges go on to say that “We have, thus, no hesitation in concluding that
such kind of occupation of public ways, whether at the site in question or anywhere else for
protests is not acceptable and the administration ought to take action to keep the areas clear of
encroachments or obstructions.” This all-encompassing ban on protests in public spaces
except designated areas doesn’t just go against the court’s own judgments from the past, it
also runs contrary to international law. A UN Special Rapporteurs’ report on the right to
freedom of peaceful assembly notes that while restrictions to the right of peaceful assembly
can be made in the interest of national security or public order, these must be lawful,
necessary and proportionate to the aim pursued. It also notes that these restrictions are to be
the exception, not the norm, and, very importantly, that they “must not impair the essence of
the right.” Completely blocking a road for an extended period of time may eventually become
an unacceptable impediment to freedom of movement, but this cannot be grounds to prevent
any such protest right at the start.

. A large mass of protestors gathered on the road in front of the Parliament. The crowd were
protesting against the inefficiency and inability of the government to take sufficient action to
prevent crimes against women and hold the perpetrators accountable. In a period of just 3
days, the crowd covered the entire road as the numbers of protestors increased by thousands
with each passing day. The crowd says that they have a fundamental right to protest on the
road in front of the parliament. Is this true?

(a) Yes, because it is a designated place of expressing dissent against the government;

(b) Yes, because the road has not yet been blocked for an “indefinite period to cause an
impediment to freedom of movement;”

(c) No, because the road in front of the Parliament constitutes “public ways;”

(d) No, because the government ought to take action to keep the areas clear of encroachments
or obstructions.

. As the protest on the road in front of the Parliament grew larger, the government ordered the
police to control the crowd. The police asked the crowd to move however they did not budge.
Eventually, the administration had to order a lathi charge to clear the road and restore
movement of cars. Is the action of the administration valid?

(a) Yes, because it is empowered to take action to keep designated protest places clear of
encroachments or obstructions;

(b) Yes, because the administration is empowered to take action to keep public ways clear of
encroachments or obstructions;

(c) No, because the government cannot take arbitrary or coercive actions to keep the areas
clear of encroachments or obstructions;

(d) No, because “demonstrations expressing dissent have to be in designated places alone”.

 
. The chief minister of Janakpur had announced a new policy of taxation wherein the middle
classes would have to pay for the upkeep and concessions of the poorer classes. The people
of Janakpur decided to protest this action as arbitrary and unfair onto the pockets of the
hardworking middle classes. The protest was organised in Lal Park, which was a popular
protest site. Lal Park was opposite to the Chief Minister’s house. Can the Chief Minister ask
the protestors to vacate the land as his right to enjoy freedom of movement around his house
is being violated?

(a) No, because the chief minister’s freedom of movement is not actually being violated;

(b) No, because completely blocking a road for an extended period of time may eventually
become an unacceptable impediment to freedom of movement;

(c) No, because Lal Park is a designated protest site;

(d) No, because Lal Park constitutes a “public way” and the freedom of movement of the
chief minister is being hampered.

. People decided to protest against the hike in prices of petrol and diesel. The protestors
contended that they could not afford such a hike in prices especially because COVID-19
related lockdown which affected their economic capacity. The protestors continued to gather
on the designated site and the government was concerned that it may lead to people
contracting COVID-19 because they were in close proximity. In order to disperse the crowd
in the interest of public health, the government ordered tear gas, lathi charge and firing of
rubber bullets. Is the governmental action valid?

(a) Yes, because it is lawful and necessary

(b) Yes, because it is in the interest of protecting public health which is a part of the
exception of “public order”

(c) Both of the above

(d) None of the above Head Office:  

Passage (Q.-Q.): The Madras HC dismissed an anticipatory bail plea by former State Minister
in a case of rape on false pretext of marriage, cheating and forced miscarriages. HC held that
it is not desirable to grant the former Minister anticipatory bail since the investigation was at
the preliminary stage given his influential status. “ There is reasonable apprehension that the
petitioner may tamper with the evidence/witnesses as the investigation is in a preliminary
stage. There is also a possibility that the petitioner may flee from justice using his influence
and money power, if anticipatory bail is granted at this stage “ When the investigation is in a
preliminary stage, courts should act with circumspection while deciding for anticipatory bail.
There will also be public outcry if the petitioner is released on anticipatory bail when the
investigation is in the beginning stage,".. HC opined that it may have considered the
anticipatory bail if the petitioner was only charged with the relatively minor offences.
However, he was accused of the more grave offences of rape and causing forced miscarriage.
" If the accused is equipped with an order of anticipatory bail even before interrogation, it
would greatly harm the investigation and would impede the prospects of unearthing all the
ramifications involved in the alleged commission of the offences and public interest may also
suffer as a consequence." the Court reasoned. The truth will have to be unearthed only after
the investigation and trial, the Court further emphasised, before ultimately concluding that the
balance is tilted against the petitioner when it comes to the grant of anticipatory bail. Source:
https://www.barandbench.com/news/litigation/madras-high-court-dismisses-anticipatory-
bail[1]former-tn-minister-m-manikandan

. Javali Babu is a suspect in a murder case with allegations of having murdered 3 innocent
people looming large over his head. The FIR also includes charges for dacoity and criminal
trespass as well. Javali is a very well known and feared bandit of Sambalpur and infamy
precedes him. His gang is also well known for escaping with crimes and evading justice.
Considering this, the HC of Odisha denies him anticipatory bail suggesting that he has a past
track record of evading the noose and shall do so again if given the chance. Can the courts
take away his right to file for anticipatory bail?

(a) The courts are wrong as every individual has a right to file for anticipatory bail.

(b) The courts are correct in considering his history of run-ins with the police and courts.

(c) The courts are incorrect as he is only a suspect and not an accused.

(d) The courts are correct as he is a heinous criminal and should be denied bail on the same
account.

. Kheda Ram is the minister for Health and Sanitation in the cabinet of the current PM. He is
a very influential person in the country and has held various prestigious union and state
government portfolios before as well. One morning, on his way to a press conference, due to
a traffic jam, he is captured by the media urinating in the open, for which the laws entail a
10,000 rupees fine and 3 month incarceration simultaneously. When this is brought to the
notice of the police they hesitated to arrest him on account of him being a very influential
person. However, the matter finally reaches the magistrate, along with Kheda Rams
anticipatory bail application. The courts deny him bail on account of being an influential
person who could evade the courts. Have the courts treated him fairly?

(a) The courts have not treated him fairly as his is a minor offence.

(b) The courts have rightly considered his influence as being an important factor in the
hearing.

(c) Kheda Ram has been publicly seen as urinating unlawfully, he should be denied bail.

(d) None of the above. Head Office:  

. In the above set of facts, when Kheda Ram is informed of his bail being rejected, he has to
attend court hearings on the day of the Union Budget being announced, as a result of which
he misses vital information pertaining to his portfolio. Enraged, he addresses the courts and
the police as donkeys and chaukidaars in the press conference which leads to the counts of
contempt of court being initiated against him. Would these offences warrant a bail application
once again being denied by the courts?

(a) The courts have once again erred in reasoning.

(b) The courts have rightly disallowed his bail application.

(c) The courts have rightly taken cognizance of his malfeasance and have rightly held him in
contempt of the court.

(d) None of the above.

. The government of India declares the possession and consumption of beef as being illegal
and punishable with imprisonment of up to 12 years on account of being insensitive towards
the religious beliefs of one of the most prominent religious sects in the country. However,
before being declared illegal, beef was being rampantly sold and eaten in India as a delicacy,
as a result of which, well known personalities like ministers, celebrities, etc. all used to eat it.
One such person was Dahiya Rasam, an MP minister, who could not dispose of the meat in
time, and was brought in for questioning by the police. Seeing the current state of affairs,
people were being beaten up in streets and were lynched for the possession of the meet, the
minister got scared out of wits and filed for an anticipatory bail fearing arrest. Can the courts
allow his action?

(a) The minister is guilty of a heinous crime herein and should not be allowed to evade
judicial action by way of anticipatory bail.

(b) The minister herein did not aim to possess the meat upon being declared illegal, and thus,
rightly fears arrest.

(c) The minister should be allowed anticipatory bail on account of the state of affairs in the
country.

(d) The state of affairs is not conducive for the minister to be allowed bail, upon receiving
which, he could be publicly lynched

. Jeevandev is a very well known actor whose movies are very well liked all across the world
and have received critical acclaim internationally. This leads to him being approached by a
very big director, Peven Pielburger, who has been nominated for an Oscar 7 times and
received 2. During the shooting, amid an action scene, Jeevandev has to fight with 4 extras in
order to get into the car and escape. The best action directors were directing this scene when a
mishap took place. The latex knife which he was supposed to use was switched out for an
actual knife by a miscreant who deliberately anted Jeevandev to be convicted for murder.
Jeevandev gets very scared due to this and goes to his lake house for hiding. However,
someone brings this to the notice of the police and they set out in search of him. Should he
apply for an anticipatory bail, would he be granted the same?
(a) Jeevandev is an influential person, his followers could create unrest in the country and
hamper the ourse of justice, hence, he shouldn’t be granted the bail.

(b) Jeevandev should be granted the bail as he did not intend to murder the extra and has been
framed falsely.

(c) Jeevandev shouldn’t be granted bail as the investigation has not even started yet and the
matter is at a very preliminary stage.

(d) There are no facts on record to show that Jeevandev intended to order anyone, therefore,
he should eb allowed the bail. Head Office:  

. During the course of investigation, the police find out that the knife which was seized at the
crime scene was taken from Jeevandev’s home and had his initials engraved on it, which he
later attested to. This leads to a warrant being issued in his name for arrest, as against which,
he files for anticipatory bail. Could his action succeed?

(a) As has been established earlier, the influence that Jeevandev carries is enough to create
unrest in the whole country, to avoid which, the courts should allow his bail.

(b) Jeevandev is being implicated in this conspiracy and should be allowed respite y way of
bail.

(c) The matter herein is still in preliminary stages and therefore, a bail application being
granted could hamper the course of investigation.

(d) Jeevandev could use his reputation to evade justice and therefore, should not be allowed
bail.

Passage (Q.-Q.): The Union Government has amended the Cable Television Network Rules,
19, in order to set up a three-tier mechanism for redressal of grievance relating to content
broadcasted by Television channels in contravention of Programme Code or Advertising
Code. They have come into force with their notification in the Official Gazette today. Similar
to the complaint redressal structure under the social media rules framed under the IT Act, the
present rule for regulation of broadcast content provide for: - A self-regulation by
broadcasters; Self-regulation by the self-regulating bodies of the broadcasters; and lastly, -
Oversight mechanism by the Central Government. So, any person aggrieved by the content of
a programme of a channel may file his complaint in writing to the broadcaster who will issue
an acknowledgement to the complainant, within 24 hours and dispose of the complaint within
15 days. In case the grievance is not resolved, s/he may appeal to the self-regulating body of
which such broadcaster is a member, within fifteen days therefrom. Every such self-
regulating body shall be constituted by a minimum of 40 broadcasters, and will have to
register itself with the Central Government within a period of 30 days. Where the
complainant is not satisfied with the decision or where the broadcaster fails to comply with
the guidance/advisory of the self-regulating body, s/he may prefer an appeal to the oversight
Mechanism, within 15 days. This body will inter alia, be empowered to take action for non-
compliance of its orders or directions and that of the self-regulating body. Source:
https://www.livelaw.in/news-updates/centre-amends-cable-tv-rules-complaints-
redressal[1]mechanism-18

. Dharamkosh United is a group of channels dedicated to showing scriptures in a cinematic


form all day long on their channels. Due to this very reason, as there is no programme
diversity on their channel, their TRP falls below average and they start making losses. Due to
this, they are approached by Chaalubagh Industries group who specialises in advertisements
and marketing. They propose to run such exciting advertisements on their channels that their
ratings would go through the roof. On hearing this, the board of directors of Dharamkosh
unanimously agrees to give Chaalubagh all their ad slots. However, in between the scriptures
being shown on the TV, Chaalubagh runs A-rated movie promos, B-class advertisements and
other offensive material which, even though leads to a huge spike in their ratings, causes
unrest among Dharamkosh’s prominent followers. You are to advise them on the action they
can take against Dharamkosh and/or Chaalubagh.

(a) The complaint lies against the Chaalubagh Group only as they are the ones who duped
Dharamkosh into believing that they will ‘magically’ raise their ratings.

(b) The complaint here lies against both Chaalubagh Group and Dharamkosh United as they
were both equally responsible in running the ads.

(c) The complaint here lies only against the Dharamkosh Group as they approved the ads to
be run on their channels.

(d) The consumers herein can file an appeal against both the parties since the gravity of the
crime is severe herein.

. In similar vein, upon receiving the complaint on 10 am Friday, Dhramakosh united issues an
acknowledgement on Monday as they remain closed on Saturdays and Sundays since people
do not watch ‘bhakti serials’ on weekends, and this move of theirs is to cut the losses that
they had been making. Is Dharamkosh United in the right here?

(a) Dharamkosh is right by issuing a fast acknowledgement as soon as possible as this is


required by the law to be done by them.

(b) Dharamkosh United is absolutely correct since they have to manage their losses as well
and in rider to do that they have to size down their operations.

(c) Dharamkosh is a prudent group and has acted in an orderly manner here y issuing a fast
acknowledgement for the complaint that was registered.

(d) None of the above.

. In the acknowledgement that the Dharamkosh group issues, they clarify their stance as being
one of mitigating their losses, which led to them contracting with the ad agency, which has
now become ‘vital’ for their ratings and profits, in the absence of which, they cannot survive.
This leads to a nationwide uproar as devout followers of the channel are now left unsatisfied
and would like to approach a higher authority. Can they do so?

(a) Dharamkosh has violated the laws in place and its viewers have appellate remedy by way
of filing an SLP in the Supreme Court.

(b) An appeal lies against the group by way of a writ to the High Court of the concerned
place.

(c) An appeal lies to the Central Government.

(d) An appeal herein shall lie to the body of broadcasters.

. The viewers of Dharamkosh decided to take their complaints to the self-regulating body of
the broadcasters, which is responsible for level II resolution of complaints, as they were not
satisfied by the behavior of the broadcaster itself. Upon hearing their pleas, the body decides
to criminally penalize Dharamkosh group and grants a permanent stay on the objectionable
content being displayed on their channels. However, when the broadcaster receives this
instruction, he does not comply and further alleges that the body answering him is not a
registered one. Is Dharamkosh right in doing so?

(a) Dharamkosh is right in doing so as an unregistered body does not have the power to
adjudicate claims of the masses by appeal.

(b) Dharamkosh has gravely erred here as the body was merely following principles of
equitable justice.

(c) Dharamkosh is a highly capitalistic entity and has acted in a severely erroneous manner
thereby neglecting the demands of its demographics.

(d) Dharamkosh has rightly contested the order as being of authority.

. The popular social networking website known as Finstagram has a feature known as FGTV
wherein people make videos, save them on the network for the people to view at large and
earn by getting viewers. This mode of broadcasting to the people at large has become very
famous and millions of people use the FGTV feature to get famous and rich. One such user is
the handle of @HarveerFernando a popular photographer who is known for his archaic yet
modern style of photography and he regularly uploads and streams his shoots on FGTV. One
such shoot was of the calendar that he was making in partnership with a Beer-Mogul Named
Bijoy Walia, who was a renowned name in the same industry. This shoot was being viewed
live by 200 million people across all ages and had garnered more than 1 million rupees. One
of the viewers was Ratan Debi, a mother of three children who objected to the obscenities
being shown in the form of models. Could she appeal to Finstagram for removing the content
that troubles her?

(a) Yes, as Finstagram is a very well-known broadcasting network.


(b) No, as Finstagram does not qualify as a broadcasting network here.

(c) Yes, as Finstagram is a very well circulated broadcasting network with a reach of millions
across the globe.

(d) None of the above.

. In the above question, when Ratna Debi had appealed to the Finstagram Network for
removing such objectionable material but Finstagram had failed to comply with her appeals
which led to her appealing to the Self-regulating body. However, to her dismay she could not
find any such body and therefore, had to directly appeal to the Union Government of India.
Will her pleas be heard?

(a) No, as she failed to observe due process of law and jumped the gun by directly appealing
to the central government.

(b) No, as there exists no level-2 appellate mechanism, and she should drop the matter.

(c) No, as Finstagram does not come under the ambit of the laws mentioned above.

(d) No, as she should apply to the central government of the home country of Finstagram.

Passage (Q.-Q.): Last month, an interim order was issued by the High Court of Delhi
directing Google and Indian Kanoon to remove access to a judgment from their portals. The
High Court had applied the Right to Be Forgotten ("RTBF") principles. At present, the data
protection regime in India does not contain the RTBF principles but there is a compelling
argument that this right must be read as part of Article 21 of the Constitution of India. Such
an interpretation is a result of the law laid down in Justice (Retd.) K.S. Puttaswamy and Anr.
vs Union of India and Ors (2017) case which confirmed the right to privacy to be part and
parcel of the fundamental right to life and personal liberty enshrined in the Constitution.
Specifically, the concurring judgment authored by the Hon'ble Mr Justice SK Kaul talks
about the "right of an individual to exercise control over his personal data and to be able to
control his/her own life would also encompass his right to control his existence on the
internet." The High Court judgment further records that "the existence of such a right does
not imply that a criminal can obliterate his past, but that there are variant degrees of mistakes,
small and big, and it cannot be said that a person should be profiled to the nth extent for all
and sundry to Know” In some cases, the Supreme Court has held that anonymity can help
protect victims of sexual offence from social ostracism. In Sri Vasunathan v The Registrar
General (2017), the Karnataka High Court recognized RTBF principles in sensitive cases
involving women in general as well highly sensitive cases involving rape or affecting the
modesty of the person.

. Which of the following is incorrect in light of the above-mentioned facts in the passage?

(a) The data protection regime in India does not consider the right to be forgotten as a
fundamental right.
(b) The K.S. Puttaswamy Case states that the Right to be Forgotten should be a part of
fundamental rights enshrined in the Constitution.

(c) The Right privacy means that an individual has the right to regulate his existence on the
internet.

(d) The Delhi High Court Judgement states that both the victims and the criminal have the
right to be forgotten.

. The Parliament passed the “Information and Data Act” that provided for the right to be
forgotten, however, it stated that a person’s data regarding a matter cannot be erased from the
public domain only if that affects the general interest of the public. Ajay Kapoor is a well-
known actor in Bollywood, with several successful films in his name. The tabloids recently
posted that Ajay would be getting a divorce from his wife. The media started publishing
details about his divorce case. One of the articles published that the divorce stemmed from a
huge fight that the couple had during their last vacation. All search engines were flooded with
articles on the famous fight. Ajay files a case to take down the articles publishing such
personal details on his account.

(a) He has a successful right to be forgotten request in relation to publications about the
divorce case details because it does not affect the public interest.

(b) He does not have a successful right to be forgotten request because he is a public figure
and his actions influence the public.

(c) He does not have a successful right to be forgotten request because his fans have a right to
be informed.

(d) He has a successful right to be forgotten request in relation to publications about the
divorce case because it affects its personal life.

. In light of the above-mentioned facts, a video was also released by a news website in which
Ajay Kapoor could be seen hitting his wife with a vase during a fight. Ajay Kapoor moves
the Court to remove the video from the internet under the Information and Data Act under the
claim that it tarnishes his public image.

(a) He has a successful right to be forgotten request in relation to publication of the video
because it does not affect the public interest.

(b) He does not have a successful right to be forgotten request because he is a public figure
and his actions influence the public and thus, the public interest.

(c) He does not have a successful right to be forgotten request because his fans have a right to
be informed.

(d) He has a successful right to be forgotten request in relation to publication of the video
case because it affects its personal life.
 

. Vikram Singh is the manager of a successful Indian Asset Reconstruction Company. He


started the business 15 years ago and is a known figure among the Indian financial
investment industry. Recently he got named in one of the most famous corporate scams.
Later, he got acquitted by the Court since the charges against him could not be proved.
However, whenever someone searched his name in any search engine, the first posts to come
up would be the old articles stating his involvement in the scam. This affected his reputation
and ultimately shadowed the business of his company as well. He moved the Court to order
the websites to remove the old articles under the Information and Data Act.

(a) He has a successful right to be forgotten request in relation to the deletion of the old
articles because it does not concern the public interest anymore.

(b) He does not have a successful right to be forgotten request because people should be
aware of his actions so as to make informed decision while dealing with his company.

(c) He does not have a successful right to be forgotten request because he was involved in the
scam

(d) He has a successful right to be forgotten request in relation to the deletion of the old
articles because he has already been acquitted by the Court.

. Abby Sheldon is a registered nurse and is currently working for a drug rehabilitation charity
program. She never published anywhere that she was a member of the Church of Scientology
or that she was a member of any other religious group. One day, Abby was contacted by Rita
who falsely presented herself and claimed to be enquiring about the services of the drug
rehabilitation charity on behalf of a member of her family but was actually inquisitive about
the links the charity had with the Church of Scientology and whether some of the employees
of the charity were practising scientologists. Upon confirming her doubts, Rita published
several blogs bashing her for practicing scientology and condemned Abby and the charity she
was working for as immoral and unethical. Anyone who carried out a search on Google for
Abby’s name or for any of the organisations she was associated with was bound to find the
negative posts by Rita as they appear very high on Google search results. Abby moves the
Court to order the websites to remove the blogs under the Information and Data Act.

(a) Abby does not have a successful right to be forgotten request because people should be
aware of her religious beliefs so as to make informed decision while dealing with her drug
programme.

(b) Abby does not have a successful right to be forgotten request because she is a member of
a criticised religious group.

(c) She has a successful right to be forgotten request in relation to the deletion of the blogs
because there is no relation between her religion and her work.

(d) Abby has a successful right to be forgotten request in relation to the deletion of the blogs
because it deters the people from getting enrolled in the drug rehabilitation programme which
thereby affects the public interest. .
 

. In the aforementioned question, had it been a known fact that followers of scientology often
use shock therapy as a form of punishment to the people who stray from their rules, would
Abby have a successful right to be forgotten request?

(a) Yes, because the blogs deter the people from getting enrolled in the drug rehabilitation
programme which thereby affects the public interest

(b) No, because she may inflict pain on people.

(c) Yes, because the shock-therapy forms a part of her religion and there is no relation
between her religion and her work.

(d) No, because people should be aware of her religious beliefs that including the rituals of
shock therapy so as to make informed decision while dealing with her drug programme.

Passage (Q.-Q.): The Bombay High Court recently sought the response of the Maharashtra
government on a plea seeking a declaration that "Ludo is a game of chance and not a game of
skill". It was submitted that a possibility of a 3-year-old winning the game could not be
discounted and, hence, Ludo cannot be considered a game of mere skill but is a game of
chance. The entire game is completely an uncertain future event and the occurrence or non-
occurrence of a particular result is absolutely based on luck, or in other words based on
'chance'. It was also contended that players could play by betting money in the game. The
term ‘Gambling’ is understood as the ‘act of wagering or betting for money’s worth’. Section
12 of the Public Gambling Act provides that “game of mere skill” does not fall under the
ambit of gambling. The five characteristics of gambling are: the exchange of money or
something of value; a future event that determines the result of this exchange and the
outcome of this event is unknown at the time that a bet is made; chance at least partly
determines the outcome of the exchange; losses can be avoided by simply not taking part;
winner gains at the expenses of the losers. Skill can include any of the following: A learned
or developed ability; Strategy, or tactic; Physical coordination or strength; Technical
expertise; Knowledge and means of accomplishing a task.

. Dobble is a game that recently gained popularity across the globe. The game includes two
opposing teams where one team would have to throw a metal ball weighing around 10
kilograms across the other side of the playing field while the opposing team defends and
stops the team from doing so. The decision as to which team goes first is done by a simple
toss of a coin. Nine out of ten times, the team that goes first wins the game because the team
to defend first is usually tired during the second innings to successfully throw the ball to the
other side while also dealing with the other team. With the rising popularity of this game,
more and more people started betting on their teams. Alok moves the Court seeking a ban on
this game on the pretext that this game encourages gambling as the result of the game is
dependent on the toss.

(a) Alok’s claim would sustain because the fact that the team to go first is usually the one to
win the game shows that Dobble is a game of chance based on a toss.
(b) Alok’s claim would not sustain because the game ultimately depends on the skills of the
players.

(c) Alok’s claim would sustain because the game is completely based on luck or chance.

(d) Alok’s claim would not sustain because the game the result is ultimately unknown while
the bet is made.

. Guess Chess is a newly launched gaming app by Gaming.inc. In the game, the players need
to pay Rs. /- to play one round in the game. In this game, the moves from both the sides
would be controlled by the in-built administrator. The app is so developed that the
administrator would only make the best possible move from both the sides. The players
would have to guess the next possible move of the administrator. In each round, 10 players
would be competing against each other and the player to get 25 right guesses first wins the
game and receives cash prize of Rs. 250/-. After 3 months into the launch of the app, it was
challenged in the Court for allowing ‘gambling’ on the platform.

(a) The claim against the app would not sustain as the players have to necessarily use their
skills to guess the next best possible move.

(b) The claim against the app would sustain because it is a complete game of chance.

(c) The claim against the app would not sustain because 10 players are competing in the
rounds to test their skills which does not amount to gambling.

(d) The claim against the app would sustain because the players need to pay Rs. to play the
game.

. Ludo Supreme is a gaming app that allows player to play ludo on the platform while also
rewarding the players with bonus tokens for every win. The app needs the players to a do a
one-time payment of a nominal sum of Rs. 150 to create an account. Thereafter, the players
have unlimited access to play the game and win prizes. Mr. Batra challenged the App,
seeking a ban on the App for promoting gambling.

(a) The App does not promote gambling as there is no question of money being gambled on a
game of chance like this.

(b) Ludo is a game of chance and since the players have to pay the money to play the game, it
amounts to gambling.

(c) The App does not promote gambling as the players have to make a one-time payment
only.

(d) Since in such apps, the administrator makes the next moves, paying money to win, the
game amounts to gambling.

 
 

. Club Maestro is a Horse-riding club. The club hosts a grand horse race event annually in
which the horses from several clubs participate to show their valour. The event is attended by
noted personalities who often bet huge sums of money on these horses. Many win stacks of
cash while many lose every penny that they entered the club with. Does this betting amount
to gambling?

(a) Yes, because people bet money on the outcome of the race which is completely a game of
chance.

(b) No, because the people are betting on the horses that they are sure would win.

(c) Yes, because people lose huge sums of money when the outcome of the race is not in their
favour.

(d) No, because horse race is a game of skill and depends on various other factors other than
chance.

43
Passage(Q.68-Q.): The Delhi High Court in a recent judgement noted that that the right to
protest is a fundamental right that flows from the constitutionally guaranteed right to
assemble peaceably and without arms enshrined in Article 19(1)(b) of our Constitution, thus,
the right to protest is not outlawed and cannot be termed as a ‘terrorist act’ within the
meaning of the Unlawful Activities Prevention Act (UAPA), unless of course the ingredients
of the offences under sections 15 of the UAPA are clearly discernible from the factual
allegations in the case. The Court acknowledged that making of inflammatory speeches,
organising chakka-jams etc. were common in the face of widespread opposition to
governmental or parliamentary actions and even if such activities cross the permissible line of
peaceful protests, it would still not amount to commission of a ‘terrorist act’ as understood
under the UAPA. Section. 15 of the UAPA reads: Whoever does any act with intent to
threaten or likely to threaten the unity, integrity, security, economic security, or sovereignty
of India by using hazardous substance of whatever nature to cause or likely to cause:

(i) death of, or injuries to, any person or persons; or

(ii) disruption of any supplies or services essential to the life of the community in India or in
any foreign country; or

(iii) damage to, the monetary stability of India by way of production or smuggling or
circulation of high quality counterfeit Indian paper currency, coin or of any other material;

(iv) damage or destruction of any property in India that may be used or intended to be used
for purposes of the Government of India; Commits a terrorist act.

68. Several hundreds of railway employees had gathered in several railway stations to protest
against the unfair wage paid to them that was below the minimum wage criteria. During the
protest, the workers stopped the functioning of the railway stations. Due to this, several
essential commodities like food supplies that were being transferred to cyclone-hit places
came to a halt. The Railway authorities claimed that the workers should be booked under the
UAPA, The claim:

(a) Can be sustained in the court of law for direct violation of Section 15 of the UAPA for
disruption of any supplies essential to the life of the community in India.

(b) Can be sustained in the court of law because they could have protested elsewhere where
the chances of disruption of essential services on account of the protest would be low.

(c) Cannot be sustained because it was a peaceful protest.

(d) Cannot be sustained because the workers did not have the intention to threaten the unity,
integrity, security, or sovereignty of India by stopping the functioning of the stations.

. Huge masses of people had gathered in several streets of Delhi to protest against the
Citizenship Act that had been recently passed by the Government. The protest continued for
over a week. Several police troops were deployed to ensure that the crowd is in control and
the protest continued in a peaceful manner. Suddenly, there was an altercation between the
police personnel and a few of the protestors and the police started beating and pushing the
protestors behind the barricades. In this, several police officers were injured along with
several protestors. The State charged a few protestors under Section. 15 of the UAPA.

(a) The charges against the protestors would sustain as they injured several police officers.

(b) The charges against the protestors would not sustain because they had the right to protest.

(c) The charges against the protestors would not sustain because the injuries sustained by the
police officers were on account of an incident that wasn’t planned to disrupt the security of
the country.

(d) The charges would sustain because the protest was not intended to be a peaceful protest.

. Considering the facts of the previous question, had the protestors carried explosives with
them that exploded during the altercation, injuring several people, would the charges brought
against them under Section 15 of the UAPA sustain?

(a) Yes, because they had carried hazardous substances with them during the protest.

(b) Yes, because carrying such explosives shows that the protestors did not intend the protest
to be a peaceful protest.

(c) No, because they did not intend for the explosives to go off.

(d) No, because the right to protest is not outlawed and cannot be termed as a ‘terrorist act’

 
. Sudip is a renowned journalist in a huge Media production house. To show his
dissatisfaction for the recently passed Citizenship Act, he asked his viewers to conduct rallies
to peacefully protest against the unjust Act passed against the minorities in the country.
Pursuant to that, several school and college students entered the streets to protest against the
Act. After some time, the rally got violent and the protestors started throwing stones at the
police troops deployed there. During this, several protestors sustained few injuries and
Deepak, a college student, was hit by a large stone. He was rushed to the hospital but
eventually succumbed to his injuries. The state charged Sudip under Section 15 of the UAPA
for instigating people to protest against the Government as a result of which Deepak lost his
life.

(a) The charges against Sudip would sustain because he had instigated people to protest
against the Act, which soon turned violent.

(b) The charges against Sudip would not sustain because he cannot be charged for merely
asking people to protest against the Act.

(c) The charges against Sudip would not sustain because he did not facilitate the protest.

(d) The charges against Sudip would not sustain because he asked the people to exercise their
right to protest.

Passage(Q.-Q.): The Patna High Court recently set aside a judgment of a Trial Court which
had awarded death sentence to an accused in a dowry death case upon finding that the
evidence on record did not show any proof that the deceased was ever subjected to cruelty on
grounds of dowry. In a separate case, the Calcutta High Court had repeatedly reiterated that
“voluntary presents given at or before or after the marriage to the bride or the bridegroom, as
the case may be, of a traditional nature, which are given not as a consideration for marriage
but out of love, affection on regard, would not fall within the mischief of the expression
'dowry' made punishable under the Act.” It has repeatedly been emphasized by the Supreme
Court that the Courts and Judges must make a dispassionate assessment of evidence and that
the Courts and Judges should not be swayed by the horror of crime and the character of the
person accused. A judge, while discharging judicial duties, should not be influenced by his
own imagined norms of the functioning of the society, the Court added. The supreme
requirement of a judgment is reason, which is the rational to the conclusion, the Court said.
"Reasoning is the mental process through which a Judge reaches to his conclusion. All
conclusions should be supported by reasons duly recorded. The finding of fact should be
based on legal testimony and should be based on legal grounds. Neither the finding of fact
nor the decision should be based upon wild suspicion, hypothetical presumption, surmises
and conjectures," the Court said.

. A FIR was lodged regarding a slaughter of a stolen cow in Bulandshahr, however, no one
was named in the FIR. Four days post the lodge of the FIR, one Ram Phal, the only witness in
the said case, recorded that complicity of a man named Zaheed has been surfaced in the crime
only to the tune that the witnesses Ram Phal has claimed to have seen Zaheed with a cow of
in the dying hours of night, and subsequently Zaheed was arrested. It was later found out that
the time gap between the time when Zaheed was seen with a cow and the time of death of the
stolen cow is so vast that the entry of any other person could not be ruled out. Zaheed moves
the Court seeking bail which the Court allows for. In this regard:

(a) Zaheed should not have been granted bail because the statement made by Ram Phal was
the only evidence present in this case.

(b) Zaheed should not have been granted bail because Ram Phal saw him with a cow.

(c) The statement given by Ram Phal does not give direct evidence that Zaheed slaughtered
the cow and thus, is not enough to keep him in custody.

(d) Since there were no other witnesses, the Court was right in granting bail to Zaheed.

. Rita was diagnosed with borderline personality disorder and she frequently experienced
manic episodes that include symptoms such as high energy, reduced need for sleep and loss
of touch with reality. Rita recently lodged a complaint in the police station that her
psychiatrist, Mr. Raj, had behaved in a sexually inappropriate manner with her.” Thereafter,
Mr. Raj was arrested. Mr. Raj moved the Court seeking bail.

(a) Statements made by Rita are sufficient to convict Mr. Raj.

(b) As a psychiatrist, Mr. Raj is expected to have higher standard of professionalism, and
thus, by making Rita uncomfortable, he should be rightfully convicted.

(c) Even though the statements by Rita do account for some value, it cannot be made a sole
ground to convict Raj of sexual assault.

(d) Statements made by Rita are not admissible because she was diagnosed with borderline
personality disorder.

. Abhishek works night shifts in a local bar in Mumbai. One night, a couple of rowdy
teenagers had visited the bar to celebrate one Manish’s birthday. While making payments,
Manish deliberately withheld a thousand rupees note from the payment amount. Upon several
request made by Abhishek to clear the payment, Manish still stood there with the thousand
rupees note in his hand and started teasing Abhishek. This led to an altercation between the
two and, in the heat of the argument, Abhishek pushed Manish who hit his head on the side of
a table. He was severely hurt and was rushed to the hospital. Manish sues Abhishek for
causing grievous hurt. Manish’s lawyer presented past records of Abhishek where he was
accused of being involved in giving bribe to some bank official to get his work done. The
record is under consideration in Court.

(a) The record should be accepted by the Court as a valid evidence of Abhishek’s past actions
that are relevant in the present case.

(b) The record should not be accepted by the Court because it does not put Abhishek in a fair
position.
(c) The record should be accepted by the Court because it is crucial to show that Abhishek is
capable of committing heinous crimes.

(d) The record should not be accepted by the Court because it holds no relation with the
present case.

. In the abovementioned question, during investigation, the police also found past record of
Manish being involved in beating up a traffic police at night when he had stopped and fined
Manish for driving above the permissible speed limit. This report is submitted before the
Court and is under consideration.

(a) The record should not be accepted by the Court because it holds no relation with the
present case.

(b) The record should be accepted by the Court as a valid evidence of Manish’s past actions
that are relevant in the present case.

(c) The record should not be accepted by the Court because it does not put Manish in a fair
position.

(d) The record should be accepted by the Court because it is crucial to show that Manish is
capable of committing heinous crimes.

. Mohan and Suman have been married for 11 years now. Suman had recently filed for
divorce from Mohan, however, Mohan insisted on reconciling to save their marriage. Even
after repeated persuasion by Suman, Mohan denied to go with the divorce. A week after this,
Suman filed a case of domestic violence against Mohan stating the violence as the reason for
divorce. Mohan was immediately arrested. During the investigation, it was found that Suman
was having an extra-marital affair with a man named Manoj for over a year. The Court found
no other evidence against Mohan. Mohan was acquitted by the Court after a careful
examination of all evidences and records.

(a) The acquittal was based on careful examination of evidence because the Court took
Suman’s extramarital affair into consideration.

(b) The acquittal was not based on careful examination of evidence because Suman’s extra-
marital affair was not relevant in the instant case.

(c) The acquittal was based on careful examination of evidence because the Court took
cognizance that there were no other evidence against Mohan and considered Suman’s extra-
marital affair as well.

(d) The acquittal was not based on careful examination of evidence because Mohan got
acquitted on the basis of simple lack of evidence.

 
 

Passage(Q.-Q.): According to Section 10 of the Indian Contract Act, “All agreements are
contracts if they are made by the free consent of the parties, competent to contract, for a
lawful consideration with a lawful object and are not expressly declared to be void. However,
the Section does not per se include an electronic contract. In the recent crisis of pandemic,
social distancing and virtual business meetings are the new norms leading the electronic
contract as an alternative to the existing conventional paper-based contracts. In electronic
commerce and trade, various types of the agreement such as Browsewrap, Shrinkwrap and
clickwrap agreement are most commonly used in the electronic platform. Browsewrap
agreement is a hyperlink or website containing the terms and conditions covering the access
or the usage of the materials available on a website or downloaded product. These terms and
conditions state that by the usage of this website one person agrees to accept the terms and
conditions on the web page and the person has consented to be bound by the terms and
conditions as stated. Shrinkwrap agreements are license agreements or other terms and
conditions that enable the consumer only when he reads or accepts after opening the product.
Moreover, it is observed that after loading the product on the electronic device, if the
consumer does not agree to those additional terms and conditions then he has a choice to
return the software product. It is also noted that as soon as the consumers uncover the
packaging of the product, due to the agreement he receives protection by indemnifying the
manufacturer of the product from any Intellectual Property Rights violation. This agreement
is discovered as part of the installation process of software packages and is also known as a
“click-through agreement” or “take-it-or-leave-it” contract.

. Ramesh loved to eat 6star chocolate. He decided to buy one from the local store. When he
reached the store, which was owned by his friend Suresh, Suresh offered him a drink. The
drink was mixed with alcohol and Ramesh wasn’t aware. After drinking it, Ramesh decided
to buy the 6star chocolate in bulk. He ended up buying 5 boxes for Rs. 10,000. Next morning,
he seeks to get out of the contract. Can he do so?

(a) No, because the 5 boxes are a lawful object and Rs. 10, 000 is a lawful consideration for
the same

(b) No, because it was made by the free consent of Ramesh and Suresh, both of whom are
competent to contract

(c) Yes, because this contract has not expressly been declared to be void by anyone or any
authority

(d) None of the above

. In the previous question, in an alternate world, Ramesh still bought the 5 boxes for Rs.
10,000 but had not drunk the beverage offered by Suresh. Next day he realized that he paid
too much money for merely 5 boxes of chocolate. Can he avoid the contract under these
changed circumstances?
(a) No, because the 5 boxes are a lawful object and Rs. 10, 000 is a lawful consideration for
the same

(b) No, because it was made by the free consent of Ramesh and Suresh, both of whom are
competent to contract

(c) Yes, because there is absence of any “free consent’ as the price is too high for just 5 boxes

(d) None of the above

. Pamazon is a new e-commerce website. It sells a large variety of products such as food
items, clothes, shoes, accessories, home goods etc. Once a buyer logs on to the Pamazon
website, they agree to be bound by the conditions of sale laid down by Pamazon such as those
regarding delivery, quality of goods etc. If a buyer purchases a shoe from Pamazon for Rs ,
what kind of agreement will it be?

(a) It is not an agreement but a mere transaction of sale

(b) It will be a standard contract as there is a lawful consideration for a lawful object

(c) It is a shrinkwrap agreement because the agreement is happening on an online software

(d) It is a browsewrap agreement because of the conditional nature of the agreement

. Pappu purchases a new jacket off of Pamazon for a discounted price of Rs. 0. Before
confirming his order, Pamazon took him to a page wherein it asked Pappu to accept the
necessary terms of service of Pamazon such as those concerning period of delivery and that
Pamazon bears no responsibility for the quality of the good sold. Pappu clicked “I accept”.
When he received the jacket, he found out that the price tag was of Rs 0 and he had
unnecessarily paid extra. Can he get out of the contract?

(a) No, because there is a lawful object and cannot be void under the Contract Act

(b) No, because there is lawful consideration of Rs 0 and cannot be void under the Contract
Act

(c) No, because Pappu himself consented to the browsewrap agreement

(d) Yes, because Pappu didn’t explicitly consent to the browsewrap agreement and price is
not mentioned in the terms of the contract

Passage(Q.-Q.): Read the passage carefully and answer the following questionsThe
Chhattisgarh High Court has observed that existence of a serious dispute between a husband
and wife is not a prerequisite for grant of divorceby mutual consent under Section 13B of the
Hindu Marriage Act, 15. The provisions contained in Section 13-B of the Act, 15 does not
provide for existence of a ground like the ones contained in Section 13 for grant of divorce by
mutual consent. There need not be a serious dispute between married couples for seeking a
divorce by mutual consent. If an application is otherwise duly constituted and properly
presented before the Court, it is not for the Court to search for a ground or a reason, which
has compelled the parties to seek divorce by mutual consent. Jurisdiction of the court should
not be a major issue in filing for divorce as the petition can be filed within the local limits of
the ordinary civil jurisdiction of where the marriage was solemnized or where either of the
parties currently resides. The parties to a marriage must be living separately for at least one
year before filing the petition. After filing a petition for divorce by mutual consent, the Court
may grant the parties a waiting period of 6 months, also known as a cooling period and it may
extend up to 18 months. The phrase “having regard to the circumstances of the case” requires
the Trial Court to find out the circumstances which compels it to pass a decree for judicial
separation. Unless such circumstances exist, the Trial Court is not entitled to pass a decree for
judicial separation in a mechanical manner. It may happen in a given case that there is no
quarrel or dispute between the couple but yet their actions and behaviour are not compatible
with each other for living a happy and peaceful married life, therefore, they may seek divorce
by mutual consent

. Seema and Arun got married in June 2018 and due to their profession they are living in
different cities. On mutual agreement, they decided to take divorce, as they found no meaning
to their marriage. And, they’ve been living separately for last two years. Can they get divorce
under Section 13B?

(a) No, because merely living separately does not grant the permission under Section 13B

(b) Yes, because the divorce can be granted on mutual agreement under Section 13B

(c) No, because marriage is not a contract under the Hindu Marriage Act

(d) Yes, because the circumstances of the case compel them to do so

. Rashi and Rohan have, during lockdown, realized that they have huge compatibility issues,
which have been creating chaos in their lives since they got married in 2019, which
solemnized in Jaipur. They were living separately for past six months; Reema is in Jaipur and
Rohan is in Delhi. They mutually decided to file a divorce application in Jodhpur. Decide the
matter?

(a) They cannot be granted divorce as living separately for at least one year prior to the filing
of application is statutory requirement under the Act

(b) They cannot be granted divorce as the application can be filed only in Jaipur, where the
marriage was solemnized

(c) They can be granted divorce as they mutually decided to get separate through divorce
under Section 13B

(d) None of the above

 
. While the Family Court was granting six months of cooling period for a divorce petition,
Jyoti had challenged the application on the ground that her consent had been obtained by
deceit and fraud. Decide the matter.

(a) Divorce petition can be dismissed as the parties need to live together

(b) Divorce Petition cannot be dismissed as the Court has already decided the merits of the
petition

(c) Divorce petition cannot be dismissed as unilateral taking the consent back is not
permissible under the law

(d) None of the above

. Atreya and Shreya, a Hindu couple, married under Special Marriage Act. They are living
separately for more than one year due to non-compatibility in their marital relations. They
want to seek divorce by mutual consent under Section 13B. Suggest them with appropriate
legal solution.

(a) They can get the divorce under Section 13B because they have mutually decided to do so
and circumstances strengthen their application

(b) They cannot get the divorce as they married under the Special Marriage Act

(c) They cannot get the divorce as Section 13B does not apply in their case

(d) They can get the divorce under Section 13B because even if they married under Special
marriage Act, Hindu marriage is governed under Hindu Marriage Act

. According to the above-passage, which of the following shall be the theme of the passage?

(a) Section 13B should be read not as a statutory mandate, but only as a directory

(b) Section 13B permits the Hindu couple to get divorceby mutual consent

(c) Lack of compatibility in the marriage leads to the evolution of Section 13B in the Act

(d) Section13B provides an ample room to the Courts to decide over divorce applications

Passage(Q.-Q.): Read the passage carefully and answer the following questionsThe Centre’s
Animal Husbandry department has suggested adding to the Prevention of Cruelty Act 10
(hereinafter the PCA Act) a stringent new Section that addresses the killing of animals and
“gruesome cruelty” towards them. Its proposed new Section, meanwhile, has the following
provisions— Section 11 (A): Gruesome cruelty or life-threatening cruelty against animals, for
which the penalty is Rs 50,000 per animal or the cost of the animal as determined by a
jurisdictional veterinarian. This carries imprisonment of one year which may extend to three
years or both. 22 of 36 Section 11 (B): Killing of an animal other than for consumption for
which the penalty is Rs ,000 per animal or three times the cost of the animal as determined by
the jurisdictional veterinarian, whichever is more, with imprisonment of three years which
may extend to five years or both. Section 11 (C): Exceptions (exemption to Section 11 (B)
killing of an animal): i) accident ii) in defence of self or property (iii) by an act of god or war
(iv) any other unforeseen circumstance outside the control of any person in general. Under
Section 12, which deals with the practice of the injection or any substance to improve
lactation— the draft proposes Rs ,000 as the penalty with imprisonment of three years which
may be extended to five. It also seeks to increase the limitation of prosecution under the PCA
from three months to two years.

. Nearly 19,000 birds fall prey to the high-tension electric wires every month at the Thar
Desert area in Jaisalmer-Barmer region. Will this issue cover under the provisions of
Prevention of cruelty Act? Read the above-passage and analyze the situation?

(a) No, the authority cannot be held responsible; it is matter of need of the population and
economy and no one has control over the same

(b) Yes, the deaths of birds at such huge number is an irresponsible behavior on the part of
the authorities

(c) No, this is merely an accident covered under Section 11 (C), which is an exception for the
purpose of the Act

(d) Yes, the power department and owners of the windmill companies are liable under the act
for failing to take requisite precautions

. Sunda, a tribe in the state of Saitisgarh has a custom of scarifying a great Indian Busturd,
which is ‘endangered’, every time on the birth of girl child in their families. And, they are
practicing this custom for generations. Some of the animal activists went to the Court
claiming it amounts to be cruelty under the Act. Decide the matter by looking into the right to
religion of the tribe and the relevant provisions in the above-passage.

(a) animal sacrifice cannot be treated as a fundamental to follow religious faith as it is not an
‘essential practice’ under the religion

(b) sacrificing an animal, which is on the verge of extinct, cannot be allowed in the realm of
right to religion

(c) the sacrifice of the animal for generations in a particular religion is protected under their
right to practice religion

(d) government has the authority to come with reform laws, and judiciary cannot interfere in
such matters

 
. Atmaram indulges in the practice of doom dev i.e. the process of blowing air into a cow’s
vagina to induce production of more milk. Will he be held liable under the Act? If yes, under
which of the above-mentioned provision?

(a) Yes, he will be held liable under Section 11 (A) for imposing gruesome cruelty or life-
threatening cruelty against cows

(b) Yes, he will be held liable under Section 12 for indulging in the practice of using
unnatural substances to improve lactation

(c) No, he will not be held liable under the Act, as he is merely fulfilling his duty by
providing requisite amount to milk to the people

(d) No, he will not be liable under the Act, as he is using natural process to enhance the
production

. Ramdin challenged the practice of sacrificing goat on the occasion of Bakr-i-Id by the
Muslims before the Court. Decide the matter as per the given provisions in the passage?

(a) The practice of killing any animals is prohibited under the Act, hence this govern come
under Act

(b) The practice of sacrificing goat is an essential practice under the religion, hence cannot be
challenged under the Act

(c) The practice of killing for any purpose other than consumption is prohibited, hence the
alleged act is protected under the Act

(d) The practice of sacrificing goat on a specific day does not make it an essential practice
under the right to practice religion

. Preclinical and clinical trial is a controversial subject. Sateesh, a member of PETA, the
Organization for animal protection, is against it because he believes that it is cruelty on
animals. He filed a petition before the Supreme Court for putting a ban on the practice of
clinical trial. He argues that animal testing is not effective and it should be banned whereas
supporters of animal testing argue that animal is necessary for the development and discovery
of new drugs. As per the above-passage would be the rationale of this matter?

(a) Clinical trials amounts to cruelty on animals as there are clinical safeguards for animals,
hence amounts to cruelty under the Act

(b) Clinical trials are requisite part of pharmaceuticals and hence, cannot be curbed as it is a
necessity to the society

(c) Balanced approach needs to be taken under the Act to safeguard the better interest of the
stakeholders
(d) It is an obligation on the part of the government to come with a requisite regulation for the
same

Passage(Q.-Q.): Read the passage carefully and answer the following questionsSection 26 of
the RBI Act states that, ‘every banknote shall be legal tender at any place in India in payment
or on account for the amount expressed therein, and shall be guaranteed by the Central
Government’. The virtual currency is not guaranteed by the Central Government, so, in order
for any virtual currency to be declared legal tender, it will have to be expressly guaranteed by
the Central Government. There is no crypto regulation in India and neither these transactions
are illegal; and since these transactions take place online and are encrypted, they are also
vulnerable to illegal and subversive activities such as terror-funding, smuggling, drug
trafficking and other money-laundering Acts, which the user who has limited knowledge may
or may not know about since these transactions are completely encrypted. In March 2020,
India’s Supreme Court struck down a 2018 order by the central bank forbidding banks from
dealing in crypto-currencies, prompting investors to pile into the market. It has stated the fact
that as far as validity of Smart Contracts is concerned, they are proper and valid contracts
under The Indian Contract Act, 18. The Apex Court further stated that an agreement, which is
made by the free consent of competent parties for a lawful consideration with a lawful object,
is a valid contract. (Excerpt from India to propose crypto currency ban, penalizing miners,
traders – source, Reuters, March 15, 2021, https://www.reuters.com/article/uk-india-
cryptocurrency-ban-idUSKBN2B60QP)

. Government of India, to manage the financial recession due to the COVID-19, directed the
RBI to conduct Open Marketing Operation for selling the Government Securities in exchange
to the crypto currency. Can the RBI negate the Directives of the Government as per the
Supreme Court judgment?

(a) Yes, the RBI can negate the Directives as the Supreme Court held that there is no crypto
currency regulations in India

(b) Yes, the RBI can negate the Directives as the Act does not consider crypto currency as
legal tender

(c) No, the RBI cannot negate the Directives as the Supreme Court set aside the RBI Circular
against crypto-currency

(d) No, the RBI cannot negate the Directives as the Supreme Court stated that RBI cannot
restrict banking services to those, who deal in it

. Rajesh, who invested his hard-earned forty-two lakh rupees to purchase a bitcoin. He wants
to sale the bitcoin in the Indian market and claims that bitcoin should be declared a legal
tender. However, the RBI argues that the bitcoin does not fulfil the criteria for a legal tender.
According to the above-passage, whether bitcoin fulfils the criteria of legal tender?

(a) No, because it is not valid for payment of debt

(b) No, because it is not guaranteed by the central government


(c) Yes, it has decentralized monetary value

(d) Yes, it is applicable for all the statutes in India

. Rainforest, an E-Commerce platform, provides an option to the purchasers to pay through


cryptocurrency i.e. smart currency. And, one of Services Clause says that Indian Contract Act
shall be applicable for all the transactions. A, the purchaser, is not happy with the product,
wants a refund for the same. Can he enforce the contract under Indian Contract Act? Answer
the question according to the above-given principle.

(a) No, he cannot enforce the contract as the usages of crypto-currency is not permissible in
Indian Contract Act

(b) No, he cannot enforce the Contract as the Supreme Court judgment says that those who
transact in crypto currency are not protected under requisite law in India

(c) Yes, he can enforce the contract under Indian Contract Act as the alleged contract fulfils
the given principles given-above

(d) Yes, he can enforce the contract under Indian Contract Act as the alleged contract is a
smart contract

. Which of the following inference can be brought into by perusing the above-passage?

(a) There is nothing under Indian law that prohibits trading in virtual currencies

(b) RBI does not have power to decide on the legal tender

(c) Mere non-regulation does not make any ‘currency’ illegal per se

(d) All transactions relating to crypto-currency are valid under Indian law

. The government of Skalia came up with an Order that transactions of agricultural land in
crypto currency, for safeguarding the public interest of the farmers. The laws of Skalia are
pari materia to the Union of India. Considering that right of freedom of trade or business is
guaranteed under Article 19 (1) (g) of the Constitution. Can the Order be challenged under
the said Article?

(a) No because the objective of the restriction is to safeguard public interest

(b) No because the Central government is empowered can make rules to regulate the crypto
currency
(c) Yes, because it is discriminatory under Article 14 of the Constitution and arbitrarily
curtails rights of some persons; while allowing the others

(d) Yes, because these transactions are protected under Article 19 (1) (g) of the Constitution

Passage(Q.-Q.): The benefit of Section I.P.C. is available to a person who by reason of


mistake of fact in good faith, believes himself to be justified by law in doing an act. Good
faith requires due care and attention. The standard of care and caution must be judged
according to the capacity and intelligence of the person whose conduct is in question. It is
only to be expected that the honest conclusion of a calm and philosophical mind may differ
very largely from the honest conclusions of a person excited by sectarian zeal and untrained
to the habits of reasoning. The law does not expect the same standard of care and attention
from all persons regardless, of the position they occupy. (Extracted with requisite revisions
and edits from State of Orissa v. Ram BahadurThapa at https://indiankanoon.org/doc/14567/)

. Ed and Lorraine Warren, professors in science and paranormal activities in London were
known for demystifying the superstition of people and unnatural phenomenon worldwide
with science, visited Bhangarh, India which is known for haunted experiences to its visitors
to unravel the science behind these occurrences. At midnight, a time after which any entry
was prohibited, they trespassed into the area and noticed a flickering light at a distance of
about 400 cubits from the path-way. There was a strong wind blowing and the movement of
the light gave an impression that it was not ordinary light but 'will-o' the wisp.' They also
found some apparitions moving around the flickering light. Seeing those apparitions,
Lorraine, in order to unearth the truth, ran towards the apparitions with a knife and repeatedly
stabbed the apparitions. It was later found that it was two of the villagers with lanterns
worshipping a sacred tree. She was charged with culpable homicide not amounting to murder
and blames the ghostly circumstances leading to this and takes the defence of Section IPC
and that she cannot be treated differently than a person in a like situation. Would her
argument succeed?

(a) No, the standard of care and caution expected of her is higher than that of other ordinary
person similarly situated.

(b) Yes, the offence was committed under a mistaken belief of ghosts and her act to be
justified by law.

(c) Yes, the surrounding circumstances created an eerie atmosphere which would affect the
intelligence of anyone despite the philosophical or educational qualifications.

(d) No, there cannot be a mistaken belief of stabbing being justified by anyone despite the
eerie surroundings.

. During the proceedings, the defence cited a case where a tribal, at midnight, was caught in
the similar scenario and believed the apparitions to be ghosts and frightened by that, he ran
away throwing stones at those figures in order to ward them off leading to fatal injuries to
another villager worshipping the tree. He was acquitted in that case by the Trial Court.
Whether the defence was correct in citing the said instance?
(a) No, the tribal will possess a comparatively lower level of mental acumen than a person
educated in science.

(b) Yes, any person be it tribal or person of science/ educated shall be put at a similar
pedestal in the like circumstance.

(c) No, a frightened person untrained with the habits of reasoning cannot be expected to be
bearing the same level of intelligence as a person of science trying to demystify superstitions.

(d) Yes, the law will expect a similar level of due care and diligence of a person caught in a
like circumstance.

. During the proceedings the case of the tribal itself came into question and his acquittal was
challenged. Was the decision of the Trial Court earlier to acquit the tribal correct? (a) No, any
tribal is to have a minimal degree of knowledge of the events taking place in the village and
cannot seek ignorance of the same. (b) Yes, the conduct of the tribal was rightfully
ascertained keeping in mind the sectarian zeal created by the eerie atmosphere and being
untrained to the habits of reasoning. (c) No, a minimum standard of care and caution has to
be expected from any person able to form reasoned choices irrespective of the level of
acumen.

(d) No, Good faith requires due care and attention and the action of the tribal was rash.

. It was revealed that the villagers who were killed by Lorraine were trespassing in the
Bhangarh fort and their worshipping of the sacred tree involved a sacrificial offering (of a
minor). Lorraine pleads acquittal on this ground. Is the argument correct?

(a) Yes, the villagers were trespassing into the area which any reasonable person cannot be
expected to have ascertained.

(b) Yes, the villagers act involved a sacrificial offering and hence their act was illegal and
Lorraine averted this misfortune.

(c) No, the standard of care and due diligence expected of a reasoned person would be
irrespective of the illegal activities the villagers were involved in.

(d) No, the act by Lorraine was unjustified under any law and punishable irrespective of the
surrounding circumstances.

Passage(Q.-Q.): Rejection of the candidature of the respondent from the OBC category on the
mere ground of belated submission of the relevant OBC-NCL certificate will only result in
virtually throwing out a meritorious candidate with extensive research experience. This will
only lead to a situation where the rights and opportunities guaranteed to the respondent under
Articles 14(equality before law) and 16 (Equality of opportunity) of the Constitution of India
will be flagrantly violated. The Division Bench observed after reading the relevant clauses of
the selection notification that what was directed to be attached along with the hard copy of
the application were certificates to prove educational qualification, experience and
community. Therefore, it was not necessary to include OBC-NCL certificate at the time of
submitting the application, the Court said. The Court opined that from a reading of the
government norms with regard to the relevant topic pursuant to the decision of the Supreme
Court in Union of India v. Abdul Rasheed, it was clear that non production of OBC-NCL
certificate along with the application before the last date, cannot be a ground to summarily
reject the application, at the threshold, where the candidate had claimed reservation benefit in
the application. (Extracted with requisite revisions and edits from ‘Rejection of candidature
only on ground of delay in submitting OBC certificate violative of Articles 14, 16: Kerala
High Court’ at https://www.barandbench.com/news/litigation/rejection-candidature-delay-
submitting-obc-certificateviolative-articles-14-16-kerala-high-court)

. Mr. Debarshi, a PhD student wished to apply for the position of teacher assistant at the
University of Doctors under the OBC-NCL category. However, due to the power failure in
his area he was unable to submit the application on time. Next day, he went to the University
to request them to understand the scenario and accept his application however the University
turned a deaf ear. He was a meritorious student during his schooling and college life and has
an infallible academic record. He approaches the Court of law and claims such a rejection
amounts to violation of Article 14 and 16 are violated. Is there university liable?

(a) Yes, the Fundamental Rights guaranteed to him are brazenly violated by the university by
not accepting the application.

(b) Yes, the university should take into account the unavoidable circumstances and Mr.
Debarshi had an infallible academic record.

(c) No, there has been failure to submit the application before the due date.

(d) No, the university has discretion to accept or reject applications of candidates.

. After a lot of deliberation and discussion and keeping in mind the academic record of Mr.
Debarshi, the University decides to accept his application and requests to submit all the
relevant and updated documents as till two months before the date of the application relating
to his qualification, category and experience to be submitted in original to their Department.
However, he fails to add any OBC-NCL document which results in the disqualification of his
application as the vacancy was only for candidates in OBC-NCL category. A suit is brought
against the University for the violation of Fundamental Rights of the applicant. Is the
University liable?

(a) No, there is no violation of Fundamental Rights by the University by the rejection of
application.

(b) Yes, there is violation of Fundamental Rights of Mr. Debarshi by rejection of the
application by the University.

(c) No, the University had already taken the extraordinary circumstances of Mr. Debarshi
earlier but he failed to completely comply with their guidelines.
(d) Yes, the meritorious academic record of Mr. Debarshi should be taken into account.

. Assume he had filed the application and the category certificate but it was found that the
category certificate was dated three years ago and was not updated as per the requirements
stated by the University. This leads to the rejection of the application. Can the University be
made liable for this act?

(a) Yes, there is violation of Article 14 and Article 16 of the Constitution of India.

(b) Yes, the document is not obsolete but merely dated 3 years ago.

(c) No, updated documents were to be submitted by the applicants but Mr. Debarshi failed to
comply with the instructions.

(d) No, University can exercise sole authority over matters governing acceptance/ rejection of
applications.

. The University was also accepting applications for the position of Research Associate for
the OBC-NCL candidates with the application requiring details as to qualification, experience
and agreeing on whether the candidate belongs to the Non-creamy OBC category. Ms. Seema
had identified herself with the category but had not submitted any document as to the same as
it was not a mandatory field. She however brings the certificate at the time of submission of
other documents in hardcopy. However, she found that the University had already rejected
her application due to absence of the certificate in the online application. Is the act of
University incorrect?

(a) Yes, there is violation of Article 16 by the University as there was no mandatory
requirement of all the documents at the time of filling the online application.

(b) No, there is no violation of any Fundamental Rights by the University as Ms. Seema
failed to submit the category certificate at the time of submission of online application.

(c) Yes, the rejection of application is arbitrary and without any intelligible differentia.

(d) No, the university has the sole discretion on acceptance and rejection of applications.

44
Passage (Q.-Q.): 'Drugs', has been defined under Section 3 (b)(ii) of the Drugs and Cosmetics
Act, 10, as follows: "such substances (other than food) intended to affect the structure or any
function of the human body or intended to be used for the destruction of vermin or insects
which cause disease in human beings or animals, as may be specified from time to time by
the Central Government by notification in the Official Gazette." Consequently, the Central
Government had notified disinfectants as a drug, under S.3 (b)(ii) of the Drugs and Cosmetics
Act, 10. Disinfectants had to be given a wider meaning in order to take into account, all the
chemical agents that would destroy or inhibit the growth of pathogenic micro-organisms. A
broader and much objective interpretation of disinfectants will bring in its fold, several
products including antiseptic lotions and sanitizers. According to Section 18(c) of the Act, no
person, either by himself or on behalf of another person shall manufacture any drug or
cosmetic, for sale or stock or distribution or exhibition, except under a license that has been
issued for the same. There is an exemption on disinfectants that permits them to be stocked
and sold unescorted by a drug license. The prime function of a 'Disinfectant', which is
essentially a mixture of chemicals of an appropriate proportion; is the annihilation and
inertness of the microorganisms, particularly that on inanimate objects. However, this does
not indicate that a disinfectant is solely used on inanimate objects. The moment it is used on
an animate object, it ceases to take the title of a 'disinfectant' and becomes an 'Antiseptic'. Its
use on animate objects is merely external, with the same intent as that of a disinfectant.
(Extracted with requisite revisions and edits from ‘Do Hand Sanitizers Require A Drug
License To Be Sold In Stores?’ at https://www.livelaw.in/columns/do-hand-sanitizers-
require-a-drug-license-to-be-soldin-stores-16)

. A famous pharma company came up with a drug, Cancerocin which had a high efficacy
over cancer causing virus. But the drug has to be administered with a non-spicy and bland
food (list of which was also provided by the company) to extract the maximum potential
which was a mandatory requirement. It was contended that this cannot be called as a drug as
a major component to extract its potential is the intake of a bland food item and food are
excluded from the definition of the Act. Will Cancerocin amount to a drug according to the
10 Act?

(a) No, Section 3 (b)(ii) of the Drugs and Cosmetics Act specifically excludes food from its
definition.

(b) Yes, food was merely a supplement to extract maximum potential of the drug and the
drug Cancerocin is still independent of the food for the destruction of virus causing the
disease.

(c) No, Cancerocin was dependent on the food item being administered and cannot be taken
as independent of the food item.

(d) Yes, although food is excluded from the definition under the Act but the food being
prescribed with the drug is a specific bland food lacking flavours which is also an inventive
step.

. Avantika, a nurse stole few quantities of Cancerocin to help her friend Meesha who was
suffering the disease and gave them to her out of love and concern for her. This drug,
according to the government was not a disinfectant. Will this act of giving the drug to
Meesha in the absence of Avantika having drug license amounts to an illegal act?

(a) Yes, the drug was sold in the absence of any drug license and thus amounts to an illegal
act.

(b) Yes, only a disinfectant is excluded from the requirement of a drug license in order to sell
or stock it and Cancerocin is not a disinfectant.

(c) No, the elements of both manufacture and sale or stocking are absent in the transaction
between Avantika and Meesha.
(d) No, the drug was given out of love and concern and should thus not be deemed illegal.

. Central government came up with a notification that included some food items also as a
drug due to their natural properties. However, the same was protested as food items are
excluded from the definition of drug. Is the act of Central Government correct?

(a) Yes, Drugs may be specified from time to time by the Central Government by notification
in the Official Gazette.

(b) Yes, the government has the right to amend the laws it makes.

(c) No, this would amount to unfettered discretion being granted to the Government and
would disturb the separation of powers.

(d) No, food items are excluded from the definition of drug and Central Government can only
indicate those substances as drugs which fulfill the qualifying criteria of the definition.

. Bovine Spongiform Encephalopathy (BSE), a degenerative and transmissible disease among


cattles was impacting large areas of Northern Areas. Cattlesis was invented which was
prescribed to be used to clean the stable that will help in killing the virus. Gajodhar, a general
store manager started selling this drug although he did not possess any requisite drug license.
Can an action be brought against him for this breach?

(a) No, disinfectants are exempted from the said restriction which allows them to be stocked
and sold unescorted by a drug license.

(b) Yes, the act of selling any drug without a license to that effect is prohibited by virtue of
Section 18(c) of the Act.

(c) No, the drug was sold for a purpose well intended and no one should be punished for
being considerate.

(d) Yes, the drug was sold in the absence of any license for sale and thus amounts to an
illegal act.

. Betol disinfectant (as the company named it), is used to clean surfaces of various objects
like tabletops, handrails etc. It can also be used for external lacerations on a human body and
for pre-operative preparation on human body. It was claimed that since the Betol is used in
animate objects, it is an antiseptic and not a disinfectant and hence cannot enjoy the
exemption for disinfectants. Is this argument correct?

(a) No, despite its usage over animate objects it still possesses features of a disinfectant.

(b) Yes, as soon as it is used on an animate object, it ceases to take the title of a 'disinfectant'
and becomes an 'Antiseptic'.
(c) No, the usage of the Betol continues to be external and thus it would qualify as a
disinfectant.

(d) Yes, Betol could either claim itself to be a disinfectant or an antiseptic and cannot
manufacture one in the name of another.

. Betol claimed that since the effect of Betol was the annihilation and inertness of the
microorganisms, it is immaterial on which surface i.e. whether animate or inanimate it is used
and irrespective of that they should be granted the exemption. Is the argument flawed?

(a) Yes, the effect of Betol at the end was the annihilation and inertness of the microorganism
regardless of the surface used.

(b) No, the drug was used over an animate object and is not a disinfectant.

(c) No, as soon as it is used on an animate object, it ceases to take the title of a 'disinfectant'
and becomes an 'Antiseptic' despite having similar properties.

(d) Yes, a disinfectant can be given a wider connotation so as to include even antiseptics used
on animate surfaces.

Passage (Q.-Q.): Copyright Law and the Museum industry may sound like an odd fit. But in
truth, they are intricately linked in their purpose and the object, which is essentially to
promote and encourage the progress of Science and art in order to constantly maintain the
value and heritage attached to certain works. Museums have been defined by the Institute of
Museum and Library Sciences as- "an organised and permanent non-profit institution,
essentially educational for aesthetic in purpose, with professional staff, which owns and
utilizes tangible objects, cares for them, and exhibits them to the public on some regular
schedule". (Extracted with requisite revisions and edits from ‘Copyright Law &Museum
Industry’ at https://www.livelaw.in/columns/copyright-law-museum-industry-17)

. Mr. Cooper was a collector of rare artifacts and precious objects. Since he travelled a lot, he
decided to organize an Exhibition-o-wheel where these artifacts were displayed in his
Campervan. He also temporarily hired some professionals at every place of exhibition in
order to bond well with the locals and exhibit his collection to the local populace. There was
a kind of taxation which was payable for all commercial activities but for educational and
museum activities. He claimed benefit under this but was denied the benefit as a museum by
the local authorities due to lacking permanency. Is the decision by the authorities on the said
ground correct?
(a) Yes, organizing the exhibition in a campervan lacks permanency in structure.

(b) No, the exhibition-o-wheel lacked professional staff which is an important essential
according to the definition by the Institute of Museum and Library Sciences.

(c) Yes, recognizing an exhibition in a moving campervan will lead to subversion of the
taxation laws of the area.

(d) No, the permanency referred to in the definition doesn’t pertain to the permanency of
place.

. Exhibition-o-wheel was later turned into a museum at a designated place in DC. However,
this time it was only opened to a select elite group of people and not to the public at large. It
claimed certain benefit under the tax scheme for museums but it was disputed that since it is
not open to the public at large it would not qualify as a museum. Is the contention correct?

(a) No, despite being opened to the select elite group of people it is still opened to the public
regardless of the number of people allowed.

(b) Yes, the benefit of the educational and aesthetic appearance of the museum artifacts
should be intended towards the public at large and not to a select group of people.

(c) No, the aesthetic and educational experience is still imparted to the audience based on the
affordability.

(d) Yes, the definition by the Institute of Museum and Library Sciences specifically provides
for exhibition to the public.

. A branch of Exhibition-o-wheel was opened in New York. This was dedicated to the raw
forms of natural artifacts in an attempt to find beauty in a pristine form. Ms. Carla, however
contended that since the artifacts are not what one might unanimously call ‘aesthetic’, hence
as per the definition by Institute of Museum and Library Sciences it doesn’t qualify as a
museum. Is this argument correct in light of the definition by Institute of Museum and
Library Sciences?

(a) Yes, the artifacts are not what one might unanimously call ‘aesthetic’, an element in the
definition of the Institute.

(b) No, aesthetic pertains to the visuals the artifacts present to its audience and can include
elements in their pristine form.

(c) Yes, the term ‘aesthetic’ has been used in order to maintain a level of quality and skill
over the artifacts being displayed in the museum.

(d) No, the artifacts are natural which automatically makes them aesthetic in themselves.

. Due to the popularity of the museum, multiple branches were opened throughout. However,
to tackle the crowd, different timings were scheduled for each day of the week except
Sunday. It was claimed that since there are no regular timings of the Museum, it doesn’t
qualify as a museum according to the definition by Institute of Museum and Library
Sciences. Is the argument correct?

(a) No, regular schedule in the definition by the Institute doesn’t mean same schedule every
day.

(b) Yes, the museum lacked a regular schedule to display the artifacts to its audience.

(c) No, the difference in timings is with the intent to tackle the growing crowd.

(d) Yes, the criteria of regular timings as per the definition are for the convenience of the
public at large for whose educational a\purpose the artifacts are exhibited.

. ‘The Exhibitionists’ is a popular museum in the District of Colombia which was famous for
showcasing perfumes of the most natural and with odors of some daily objects like books.
They have been all the benefits of a museum. But post the definition by the Institute of
Museum and Library Sciences, it was contended that ‘The Exhibitionist’ doesn’t qualify as a
museum as odor is intangible object and thus, doesn’t fulfill the condition of the definition. Is
the argument correct?

(a) Yes, the definition by the Institute states that museum is one which owns and utilizes
tangible objects and odor would thus be excluded under the definition.

(b) Yes, nobody including the Exhibitionist can own or utilize odor.

(c) No, it is not the odor that is being showcased by the museum but the perfume which is a
tangible object.

(d) No, odor although being intangible possesses the qualities of being aesthetic and thus
owning the odor would make The Exhibitionist a museum.

Passage (Q.-Q.): Mandamus is a Latin word, which means "We Command". Mandamus is an
order from a superior court to a lower court or tribunal or public authority to perform an act,
which falls within its duty. It is issued to secure the performance of public duties and to
enforce private rights withheld by the public authorities. Simply, it is a writ issued to a public
official to do a thing which is a part of his official duty, but, which, he has failed to do, so far.
This writ cannot be claimed as a matter of right. It is the discretionary power of a court to
issue such writs. The primary purpose of this writ is to make the Government machinery
work properly. An order of mandamus is a command directed to any person, corporation or
an inferior tribunal, requiring them to do some particular thing which pertains to their/his
office and which is in the nature of a public duty. The public servants are responsible to the
public for the lawfulness of their public duties and their actions under it. If a public authority
fails to do what is required under law or does beyond what was to be done, a writ of
mandamus may be issued to make him do what was required under law. Mandamus may also
be issued where there is a specific legal right, without specific remedy for enforcement of
such right and unreasonableness has no place. The Supreme Court in various decisions has
held that the doctrine of legitimate expectation is akin to natural justice, reasonableness and
promissory estoppel. Thus the writ or order in the nature of mandamus would be issued when
there is a failure to perform a mandatory duty. [Source: Tanu Kapoor, ‘Writs in Indian
Constitution’, Latest Laws, , as accessed on 22nd June 2021]

. The increasing cases of COVID-19 pandemic has left the city of Delhi in much more danger
as there were no beds available for the patients and lack of oxygen cylinders has left many
people suffering from this disease. The air pollution in the city is another major issue which
has been in discussion for the last 3 years. Srikant Tiwari, a journalist, filed an application to
the Supreme Court of India alleging the situation to be a full governance failure and also
requested to issue a writ to the National Green Tribunal of India. He contended that NGT is
also responsible for the current situation of Delhi as it was formed with the objective to frame
and plan out solutions to such issues and it should have acted in order to monitor the
increasing air pollution. Decide –

(a) The Supreme Court will accept the petition of Srikant Tiwari and will issue a writ of
Mandamus to the National Green Tribunal as they failed to perform their duty.

(b) The Supreme Court will not accept the petition of Srikant Tiwari and will not issue a writ
of Mandamus to the National Green Tribunal as they have not failed to perform their duty.

(c) The Supreme Court will accept the petition of Srikant Tiwari and will issue a writ of
Mandamus to the National Green Tribunal as they are the public authorities.

(d) The Supreme Court will not accept the petition of Srikant Tiwari and will not issue a writ
of Mandamus to the National Green Tribunal as they are neither courts nor public authorities.

. Dhriti is a big fan of Ajinkya Rahane and she used to love cricket because of him. She also
has a crush on him and she likes his attitude the most. Recently, the Indian Cricket Team was
selected for the World Test Championship Series 2021 and she gets to know that Ajinkya
won’t be playing the series this year as he was not selected in the team. She was very upset
and angry too. She filed a petition in the Apex Court to issue a writ of mandamus to the BCCI
(Board of Control of Cricket in India) to publish the criteria of selection of Cricketers for the
series and state authentic reasons for not selecting Rahane in the main team. Decide –

(a) Dhriti will lose in the petition and the Apex Court will not issue the writ of mandamus to
the BCCI as they are not well within the ambit of public authorities.

(b) Dhriti will succeed in the petition and the Apex Court will issue the writ of mandamus to
the BCCI as they failed to perform their legal duty and violated the legal right of Dhriti.

(c) Dhriti will lose in the petition and the Apex Court will not issue the writ of mandamus to
the BCCI as they have not failed to perform their legal duty. But there was a violation of the
legal right of Dhriti.

(d) Dhriti will succeed in the petition and the Apex Court will issue the writ of mandamus to
the BCCI as they are equivalent to the public authorities and accountable for their actions.

 
. Suchi was a renowned businesswoman and was very keen to have good investment plans.
She recently put her money in the share market and kept them as holdings. She was also new
to the Crypto market and decided to invest there. She bought around 250+ crypto currencies
amounting Rs.20,000. Soon, she saw a newspaper headline where the RBI (The Reserve
Bank of India) has stated that they are thinking of Crypto currencies as viable digital assets
but has not reached the decision yet. Probably the Government is inclined to ban crypto
currencies in India. She immediately filed a petition in the Supreme Court and asked to issue
mandamus to RBI as they cannot decide upon banning Crypto Currencies in India. Decide –

(a) The Supreme Court will reject the petition of Suchi and will not issue the writ of
mandamus to the RBI as they are not covered under the ambit of public authorities.

(b) The Supreme Court will not reject the petition of Suchi and will issue the writ of
mandamus to the RBI as they have acted beyond the limits of their legal duty.

(c) The Supreme Court will reject the petition of Suchi and will not issue the writ of
mandamus to the RBI as they have not acted beyond the limits of their legal duty.

(d) The Supreme Court will not reject the petition of Suchi and will issue the writ of
mandamus to the RBI as they are covered under the ambit of public authorities.

. Sajid was working as a government officer in the state forest department of Odisha. He had
a good salary. He used to fill his Income Tax on time. For the year 2021, he paid income tax
around Rs.,000. But the Income Tax Department has charged him a penalty of Rs.2,000 for
being late. The Income Tax Tribunal heard both the sides and decided in favour of the IT
Department and stated that the penalty was charged appropriately as per IT rules. He applied
for reviewing the decision to which the tribunal refused. He filed a petition in Odisha High
Court to issue a writ of mandamus to the Income Tax Tribunal to review his case again.
Decide –

(a) The High Court of Odisha cannot issue the writ of mandamus to the Income Tax Tribunal
as High Court does not have the jurisdiction and power to issue such writ.

(b) The High Court of Odisha can issue the writ of mandamus to the Income Tax Tribunal as
it is well within their duty to review the decision again.

(c) The High Court of Odisha cannot issue the writ of mandamus to the Income Tax Tribunal
as the tribunal is itself equivalent to HC in power and only the Supreme Court can issue the
writ.

(d) The High Court of Odisha can issue the writ of mandamus to the Income Tax Tribunal as
High Court has the appropriate jurisdiction and power to issue such writ.

. Choose the appropriate option.

Assertion (A): In entertaining the writs, the High Court enjoys wide and open powers as a
matter of discretion. It is a plenary power of the High Court without any fatter from any
provision of the Constitution.
Reasoning (R): The scope of Article 226 is wider than that of Article 32 because the
operation of Article 226 is not limited to violation of fundamental rights only, but it can be
operated for other purposes also.

(a) Only A is correct and R is incorrect.

(b) Both A and R are correct and R is the correct explanation of A.

(c) Both A and R are incorrect.

(d) Both A and R are correct but R is not the correct explanation of A.

. Which among the following statement(s) is/are not true about the writ of mandamus?

I. The writ of mandamus can be granted only in cases where there is a statutory duty imposed
upon the officer concerned, and there is a failure on the part of that officer to discharge the
statutory obligation.

II. The writ of mandamus cannot be granted in case when the duty involved is merely
discretionary or on the prerogative of the authorities.

III. A writ of mandamus can be granted to enforce an obligation arising out of contract.

IV. The writ of mandamus can be issued to both public and private authorities to restrain it
from acting under a law which has been declared unconstitutional.

(a) Both II and IV (b) Both III and IV (c) Only I (d) I, II and IV

Passage (Q.-Q.): According to section 2(aa) of the Juvenile Justice (Care and Protection of
Children) Amendment Act, 2006, “adoption means the process through which the adopted
child is permanently separated from his biological parent and becomes the legitimate child of
his adoptive parents with all right, privileges and responsibility that are attached to the
relationship”. In India, an Indian whether he is married or single, Non-Resident Indian (NRI),
or a person belonging to any nationality (foreigner) may adopt a child. The guidelines and
documentation process for each group of adoptive parents may differ. Under Hindu Adoption
and Maintenance Act, any male Hindu who is of sound mind and not a minor is eligible to
adopt a son or a daughter. But if such male has living spouse at a time of adoption then he
can adopt a child only with consent of his wife unless she has renounced the world
completely or becomes unsound mind or has been declared incompetent by the court. In case
of adoption of a son by any Hindu male or female, there should not be any living son in the
succeeding three generation of the party at the time of adoption. In case of adoption of a
daughter by any Hindu male or female, they should not have any daughter or son’s daughter
at the time of adoption. Where there is an adoption of a daughter by a male or female then the
adoptive father/mother should be at least twentyone years older than the child. Personal laws
of Muslim, Christian, Parsis and Jews do not recognise complete adoption. This statute only
makes a child a ward, not an adoptive child. According to this statute, the movement child
turns to the age of 21, he is no longer consider as a ward and treated as individual identity.
[Source: Ajay Thakur, ‘All you Need to Know about Adoption Laws in India’, Ipleaders, , as
accessed on 23rd June 2021]

. Maloti and Manan got married in 2002. They tried to have a child but there were some
biological issues with Maloti. She was not able to conceive the child. She gets pregnant twice
but the child dies within the womb itself due to lack of nutrition and incapability of the
internal organs to support the food supply to the unborn child. Finally they decided to adopt a
child. Manan applied to the judicial court for adopting a child. Manan adopted Sameer, a boy
aged 5 years. Maloti was not happy with this decision of Manan as he had not taken her
consent before such adoption. She wants a baby girl and not a boy. Manan contested her
argument and said the adoption is valid now and is legal too. Decide –

(a) Manan was correct in his contention as there was a valid and legal adoption of Sameer as
Maloti was incompetent to give birth to a child and hence her consent was not necessary.

(b) Manan was incorrect in his contention as the adoption of Sameer was neither valid nor
legal. He has to take consent of Maloti before such adoption.

(c) Manan was not incorrect in his contention as the adoption of Sameer was valid and legal
too. The court had given the adoption after verifying the facts and consent of Maloti was not
necessary.

(d) Manan was not correct in his contention as there was no valid and legal adoption of
Sameer. Consent of Maloti has to be taken before the adoption even though she is
incompetent.

. Anand and Purnima were in love with each other. They got married against the will of their
families. Soon Purnima gets pregnant but due to some complications she gives birth to a still
born child. She was very upset and depressed. Anand suggests an idea to adopt a child. She
agreed to that. Before adoption could take place, Anand died in an accident. Purnima was left
alone with no one to support her. She went to court and filed a petition for adoption of a
child. Her father asked her to get remarried as she has to live a long life and it would be a bit
difficult to live without any family. Also the child to be adopted will need care and family
protection for good upbringing. Purnima decides to get remarried. She adopted a child, Yash
and then got remarried to Parth. Parth was also a widower and had a baby boy from his first
marriage. Decide –

(a) The adoption by Purnima is not valid as she has to take consent of her husband Parth
before such adoption and thus Yash was not adopted legally.

(b) The adoption by Purnima is valid as she need not take the consent of her husband Parth
before the adoption and Yash was adopted legally.

(c) The adoption by Purnima is legal but not valid as Parth already had a son from his
previous marriage and thus Purnima cannot adopt another son.

(d) The adoption by Purnima is valid but not legal as she had not disclosed the fact to the
court that she will remarry and Parth already had a son from his first marriage.
 

. Sajid and Suraiya get Nikaah and are a happily married couple. Their family forced them to
have a child to continue their generation. Suraiya was not ready for this and asked Sajid to
marry another girl to have his child. Sajid gets Nikaah with Salma. They tried to have a child
together but Salma was incapable of conceiving the child. Sajid married Sahiba for the same
purpose. Soon Sahiba gets pregnant and she gave birth to a baby girl. After 5 years, Suraiya
asked Sajid to plan their child. Sajid refused her and asked her to go for adoption. She found
a couple who were ready to give in to adoption. Decide –

(a) Suraiya cannot adopt a child as she is competent to give birth to a child and Sajid cannot
deny her right to have their own biological child.

(b) Suraiya can adopt a child as she is of sound mind, a major and she had a valid consent of
her husband for such adoption.

(c) Suraiya cannot adopt a child but can provide parenthood to a ward. Consent of Sajid for
such parenthood is immaterial in such cases.

(d) Suariya can adopt a child but only a male child and not a female child as Sajid previously
had a daughter from his third marriage i.e from Sahiba.

. Mr. Mehta was an old retired journalist and had no child till the age of 65 years. His wife
died at a very early age. He gets married to Kashmira, a resident of the UK. He also adopted
two sons, named Rahul and Praful, with consent of Kashmira. He transferred half of his
property to Kashmira and the remaining half to his sons. Soon he died of cancer. Kashmira
thought to take all the property from Rahul and Praful and signed an adoption deed where she
gave Rahul and Praful in adoption to some other couple. Rahul and Praful contested this issue
in the court and contended that they both have equal share in their father’s property and
Kashmira cannot take their share of the property. Decide –

(a) The contention raised by Rahul and Praful is incorrect as they have been given in adoption
to another couple and are no more children of Mr. Mehta and Kashmira.

(b) The contention raised by Rahul and Praful is correct as they have been adopted first by
Mr. Mehta and Kashmira and hence they have right in the share of the property.

(c) The contention raised by Rahul and Praful is not correct as they were not the biological
children of Mr. Mehta and once given in adoption to others, they lose their rights to such
share of property.

(d) The contention raised by Rahul and Praful is not incorrect as they have to be given share
in their father’s property but do not have any other rights once given in adoption to others.

. Choose the appropriate option.


Assertion (A): Muslim Personal Laws does not recognise adoptions legally. However,
Muslims follow the Kafala system of parenthood where they can adopt and take care of the
ward till he/she attains majority.

Reasoning (R): An adoptive child is treated the same as a biological child of his or her
adoptive parents under Muslim law. According to law, the adoptive child has the same legal
rights to benefit from the property as that of a biological child and can also claim stakes on
their property.

(a) Both A and R are correct and R is the correct explanation of A.

(b) Both A and R are incorrect.

(c) Both A and R are correct but R is not the correct explanation of A.

(d) Only A is correct and R is incorrect.

. The Juvenile Justice Act 2000 has been amended to include extensive definition of
adoptions. It includes inter-country adoptions as well. The act also extends to children who
are in need of care and protection. Choose among the following, who all can be considered to
be the Child in need of Care and Protection. I. Child who is found working in contravention
of labor laws. II. Orphaned and abandoned Child III. Child whose parents or guardians are
unfit to take care of him. IV. Child at imminent risk of marriage before attaining the lawful
age for the same.

(a) Both I and III

(b) Only II

(c) I, III and IV

(d) All of the above

Passage (Q.-Q.): Under the Fault theory or the offences theory or the guilt theory, marriage
can be dissolved only when either party to the marriage has committed a matrimonial
offence. It is necessary to have a guilty and an innocent party, and the only innocent party can
seek the remedy of divorce. However, the most striking feature and the drawback is that if
both parties have been at fault, there is no remedy available. Opposite of this is No-Fault
theory of divorce where there is divorce without any fault of either of the parties. Another
theory of divorce is that of mutual consent. The underlying rationale is that since two persons
can marry by their free will, they should also be allowed to move out of the relationship of
their own free will. However, critics of this theory say that this approach will promote
immorality as it will lead to hasty divorces The third theory relates to the irretrievable
breakdown of the marriage. The breakdown of marriage is defined as “such failure in the
matrimonial relationships or such circumstances adverse to that relationship that no
reasonable probability remains for the spouses again living together as husband & wife.”
Such marriage should be dissolved with maximum fairness & minimum bitterness, distress &
humiliation. Irrespective of the three remedies available to parties that is: restitution of
conjugal 27 of 44 rights, judicial separation, and divorce. The judiciary in India is demanding
irretrievable breakdown of marriage as a special ground for divorce, as sometimes courts face
some difficulties in granting the decree of divorce due to some of the technical loopholes in
the existing theories of divorce. In the opinion of the Supreme Court and Law Commission of
India, it is very essential to make it a special and separate ground mission that introduction of
irretrievable breakdown of marriage, as a special ground will do any public good. [Source:
Simran, ‘Divorce under Hindu Law’, , as accessed on 24th June 2021]

. Rajesh and Mamta were in love with each other. They decided to get married. In 3 years of
marriage they found it to be a bit difficult to live with each other and either of them was not
happy with this tie. Rajesh was a psycho and was cruel to Mamta sometimes. He used to beat
her when drunk and usually slap her several times in a week. Mamta’s friend Piyush gets to
know of this situation and he sympathises with her for suffering. Soon Mamta found her
much comfortable and safe with Piyush. She starts visiting Piyush weekly and spends more
time with him. They got close to each other and had physical intimacy too. Piyush advised
Mamta to get divorced and live with him for rest of her life. Mamta filed a divorce petition on
grounds of cruelty. Decide –

(a) The case falls under the scope of Fault Theory of divorce as Rajesh was guilty of cruelty
and Mamta can validly claim divorce on these grounds.

(b) The case falls under the scope of Irretrievable Breakdown of marriage as Mamta was not
happy to be with Rajesh and they cannot live together anymore.

(c) The case falls under the scope of divorce by Mutual Consent Theory as both Mamta and
Rajesh were not happy with this tie and it would be better for them to get divorced.

(d) The case does not fall under any Theory of divorce and there is no judicial remedy
available with Mamta including the divorce.

. Shaheer and Shameena were engaged together. They have not met each other before their
Nikaah. Shaheer was in love with Shameena and excited to marry Shameena as she was very
beautiful, pretty and tall enough. He decided to give her every bit of happiness. He
transferred all his ancestral property in name of her. He also put all his jewellery, gold, and
cash transferred to Shameena. Soon Shaheer and Shameena get Nikaah and are tied in a
wedding knot. On the wedding night, Shameena saw Shaheer for the first time and was
shocked. Shaheer was a short and ugly faced man. Shameena always admired her beauty and
wished for a handsome groom. The next day, she gave Talaq to Shaheer and went back to her
family. Decide –

(a) Shameena had taken Talaq on the Mutual Consent Theory of divorce as they both were
not happy to be with each other. Shameena was pretty and beautiful and wished for a
handsome groom and couldn’t be the wife of ugly man.

(b) Shameena had taken Talaq on the Fault Theory of divorce as Shaheer should meet
Shameena before the Nikaah so that she could decide accordingly. Shaheer knowingly kept
this fact hidden from Shameena.
(c) Shameena had taken Talaq on the No-Fault Theory of divorce as they had not met each
other before the Nikaah and she did not find Shaheer to be a suitable groom for her.

(d) Shameena had taken Talaq on the Irretrievable Breakdown of marriage as there are no
chances for the couple to repair the loss and live together as Husband and Wife.

. Sagar and Anusha were friends from their college days. They were in love with each other
and wanted to marry. Anusha was not sure to spend her life with Sagar and asked him to have
a live-in relationship first before they got tied in a wedding knot. They had given 3 years to
live-in relationship and finally got married in 2020. In a few months, they had a lot of fights
and all because of their ideological differences. Anusha was a believer of religion and always
forced Sagar to visit nearby temples and explore the divine places on earth. Sagar on the
other hand is atheist and moreover against the Hindu religion. He does not like the religious
differences and mob lynching in name of religion. He believes in supreme humanity. In a
year, Anusha and Sagar found each other to be incompatible and wanted to end this marriage.
If their divorce takes place, decide –

(a) Their marriage would end under the ambit of Fault Theory of Divorce as they both were
at fault. They should adjust and compromise with each other’s ideological views and continue
their conjugal relationship. It cannot be a ground for divorce.

(b) Their marriage would end under the ambit of Divorce by Mutual Consent as their
ideological views are not similar and that becomes the main issue of their incompatibility.

(c) Their marriage would end on the ground of Irretrievable Breakdown of marriage as they
both have strong opinions and their own ideological thinking. However, it can be changed
and hence is repairable.

(d) Their marriage would not end under any of the Theories of Divorce as they both are at
fault and there is no remedy available in such a case. Though it can be a ground for divorce
but marriage cannot end in this case.

. Arjun and Draupdi were married to each other. Draupdi was an independent woman and
wanted to do a job. Arjun was of much orthodoxical views and was not in agreement to
permit women to do men’s duty. He believes women are meant to do only household work
and fulfil their responsibilities of being a wife. Draupdi has applied for a job in Hastinapur
Consultancy Firm and was hired for a monthly income of Rs.50,000. She hides this fact from
Arjun and accepts the job. She went to her parental home and started working in the firm. It’s
been over a year since she was living with her parents. Soon Arjun gets to know about this
and he asks her to return back, else he will be forced to give her divorce. Draupdi refused to
come back. Arjun filed a divorce petition on the grounds of desertion. Decide –

(a) Divorce can take place on the grounds of desertion as Draupdi has left Arjun from long
back and the case will be covered under the Fault Theory of Divorce.

(b) Divorce cannot take place on the grounds of desertion as Draupdi has not left Arjun.
Rather, it can be a case of Divorce by Mutual Consent as Arjun had warned Draupdi for
divorce and knowingly, she refused to come back.
(c) Divorce can take place on the grounds of desertion as Draupdi has left Arjun for more
than a year and such divorce cannot be on ground of Irretrievable breakdown of marriage.

(d) Divorce cannot take place on the grounds of desertion as Draupdi has not left Arjun and
this case can be covered under No-Fault Theory of Divorce.

. In the case of Naveen Kohli vs. Neelu Kohli, the Court states that “Once the marriage has
broken down beyond its essence, it would be unrealistic for the law not to take notice of that
fact, and it would be harmful to society and injurious to the interests of the parties. The
marriage becomes a fiction, though supported by a legal tie. By refusing to sever that tie the
law in such cases does not serve the sanctity of marriage; On the contrary, it shows scant
regard for the feelings and emotions of the parties.” Which theory(s) of divorce is the Court
directing to? I. No-Fault Theory of Divorce II. Fault Theory of Divorce III. Mutual Consent
Theory of Divorce IV. Breakdown of Marriage Theory of Divorce

(a) Both I and IV

(b) Only IV

(c) I, III and IV

(d) Both II and IV

. Priyanka and Piyush get married in 2019 and after a few years of marriage found it to be
difficult to adjust with each other. There were religious differences also in their families and
they decided to get divorce. A petition was filed in the family court for divorce by mutual
consent. Cooling off period of 6 months was given to them by the court. Priyanka gets to
know that she should have claimed maintenance from Piyush and his family and applied for
withdrawal of her divorce application by mutual consent. She filed divorce petition on the
grounds of cruelty and bestiality. The court refused such a petition and continued the
proceedings under divorce by Mutual Consent. What is the rationale behind the idea of
divorce by Mutual Consent on which the Court might have relied upon while rejecting
Priyanka’s application?

(a) Divorce by Mutual Consent suggests that parties can get into marriage by their
will/consent and can come out of such a tie with their consent easily.

(b) Consent of parties filing a petition for divorce by mutual consent cannot be withdrawn
once the cooling off period is over and proceedings will continue in the same manner.

(c) Consent of parties to the divorce petition under section 13 of Hindu Marriage Act cannot
be withdrawn without sufficient cause and should not be misused.

(d) Unilateral withdrawal of consent in divorce petition by mutual consent is only permitted
to the Husband and not the wife.

 
Passage (Q.-Q.): According to the standing committee on copyright and related rights
(SCCR) administered by WIPO, specific kinds of categories of exceptions and limitations
that can be made to the copyright regime: I. Use of works in Exhibition Catalogues-
Advertisement of items in a museum can attract legal issues by the virtue of copyright law.
To enable expansion of the reach and presence of museums, they must be exempted in
respect of advertisement. II. Preservation of artwork in a digital form must be allowed.
Collecting the paintings, drawings, sculpture, Artistic craftsmanship is not the only task
which can justify the purpose and object of the existence of a museum but to constantly
maintain that value the artwork should be protected against deterioration, theft, etc.
Reproduction of works must be permitted in order to preserve and restore the collections.
Digital Technologies can be very helpful in this regard. III. When the author or owner of the
work is unknown, it becomes difficult to obtain their consent. Hence, such orphan works
must be allowed an exception, as is allowed in museums in the European Union. IV.
Communication to the public on the premises of the museum- Besides the conventional
method of having an attractive and state-of-the-art collection in the galleries of the museum,
museums must also be allowed to communicate their own work to the public in other ways.
The digital medium is one of the best ways for such communication. (Extracted with requisite
revisions and edits from ‘Copyright Law &Museum Industry’ at
https://www.livelaw.in/columns/copyright-law-museum-industry-17)

. A new museum was opened in California. In order to attract customers to its art collection it
prepared certain catalogues which contained certain notable pictures from ancient era which
were available for display this week. Certain catalogues were also of live size. A claim was
made by an art lover who protested against these live paintings as a violation of copyright. Is
the live size catalogues a violation of copyright?

(a) Yes, due to the live size of the paintings it cannot be called as an exhibition catalogue and
it would not be exempted from the copyright.

(b) No, exhibition catalogues are exempted from the copyright to enlarge the reach and
presence of the museums.

(c) Yes, the catalogue contained the pictures from the notable collection which would be the
violation of the copyright.

(d) No, there would not be any violation of copyright as the ultimate owner of the painting is
the museum itself.

. The famous painting ‘The Potato Eaters’ by Vincent Van Gogh was to be displayed by the
museum this Monday. A lot of people bought the tickets at extraordinarily high prices just to
get a peek of the painting. However, instead of displaying the original painting, the museum
displayed a replica of the painting with the original painting being on display in a white
screen as it was aware of the likelihood of theft of such a painting. It was contended that this
is a sheer violation of the copyright law as the painting instead of being displayed in original
is being displayed in a white screen. Is the argument correct?

(a) Yes, the museum had charged a hefty amount for displaying the original painting.

(b) Yes, the digital technology was used to circumscribe the copyright law.
(c) No, digital technology can be adopted to protect the value of the artwork against theft.

(d) No, reproduction of works must be permitted in order to preserve and restore the
collections.

. The museum, because it charged exorbitant entry fee from its customers decided that it will
give 3 replicas/ duplicates of the painting ‘The Potato Eaters’ to 5 of its audience through a
lucky draw with the payment of a minimal amount. Is there a violation of the copyright?

(a) No, it is to enhance the reach and presence of the museum.

(b) No, the museum is the ultimate owner of the paintings so it is free to create duplicates of
the painting.

(c) Yes, there is a violation of copyright since replicas of paintings have been made.

(d) Yes, there is a violation of copyright as the payment is also being taken after the lucky
draw.

. The museum also displayed some ancient paintings with no owner to account for. They
called this segment of the museum as “Lost in Time”. This was a very popular segment of the
museum. They received these paintings from anonymous sources however, after obtaining
them it only circulated the news of the features of the paintings without displaying the
original painting asking the owner or author, of such a painting to make a claim within one
month. After this the paintings was displayed. Is this violation of copyright?

(a) Yes, one month time is not a reasonable period of time for circulation of the news and for
the author to make a claim.

(b) No, Orphan works are protected from the copyright regime hence there is no violation of
copyright.

(c) Yes, displaying the paintings without the owner’s consent is the violation of copyright.

(d) No, the paintings are merely displayed to enhance the reach and presence of the museum
which is an exception to the copyright.

. After the painting was displayed in the ‘Lost in time’ segment of the museum and post the
one month duration of claiming the title of the painting, One Mr. Robin came to claim title
over the property. After making all the necessary checks and verification, it was found that
Mr. Robin was the actual owner of the painting. But the museum claimed that since no claim
was made in the first month of the news publication, the title belongs to them. Mr. Robin
sought relief under the copyright law. Is there an infringement of the copyright by the
museum?

(a) Yes, the painting after the verification of the author is no longer an orphan painting.
(b) No, orphan works are protected from the copyright regime hence there is no violation of
copyright.

(c) Yes, the time period given to claim authorship is unreasonably small.

(d) No, Mr. Robin has failed to claim title over the painting within the said duration
prescribed by the museum.

Passage (Q.-Q.): It is true that in the complaint the respondent has not disclosed the legally
enforceable debt or liability in discharge whereof he has received the cheque from the
petitioner. That, however, cannot vitiate the complaint for the simple reason that under
Section 139 of the Negotiable Instruments Act, there is presumption that holder of the cheque
received the cheque of the nature referred to in Section 138 (when the cheque is dishonored
for insufficiency of funds or for any of the prescribed reasons, the one who is at defaulter can
be punished with imprisonment for a term which may extend to two years, or with fine which
may extend to twice the amount of the cheque, or both) for the discharge, in whole or in part,
of any debt or other liability unless of course the contrary is proved. Under Section 204 of the
Code of Criminal Procedure (CrPC), the Court shall not issue summon or warrant against the
accused unless list of prosecution witnesses is filed. However, the defect of not supplying the
list of prosecution witnesses is only an irregularity unless failure of justice has in fact been
occasioned thereby. (Extracted with requisite revisions and edits from ‘Failure to disclose
liability of drawer, supply list of witnesses do not vitiate complaint under Section 138 NI Act:
Jammu & Kashmir High Court’ at https://www.barandbench.com/news/litigation/failure-
disclose-liability-drawer-vitiate-complaint-section138-ni-act-jammu-kashmir-high-court)

. A complaint has been made by Ashutosh, real estate businessman under Section 138 against
Mr. Devgan, the allottee/ home buyer on the ground that Mr. Devgan had issued a cheque of
Rs 30 lakhs to clear the dues for the purchase of the house which was drawn on his account
maintained with the Punjab National Bank. However, when the cheque was deposited for
encashment it was returned due to insufficiency of funds in Mr. Devgan’s Account. Even
after serving a demand notice to him to complete the payment within 15 days, no payment
was made. Hence, the complaint was filed before the trial court. The Trial Court admitted the
complaint and after recording the statements of Ashutosh and other witnesses issued an order
to proceed against the home buyer. The complaint was disputed by Mr. Devgan on the
ground that in the complaint Ashutosh did not specifically claim that he had received the
cheque in discharge of some legally enforceable debt or liability. Will his argument succeed?

(a) Yes, there would be no presumption as there is failure of justice caused to Mr. Devgan by
the act of Mr. Ashutosh. (b) No, there is a presumption that the holder received the cheque for
the discharge, in whole or in part, of any debt or other liability.

(c) Yes, Ashutosh had failed to specify that in the claim he had received the cheque in
discharge of some legally enforceable debt or liability.

(d) No, there is a presumption in favour of holder under Section 138 Negotiable Instruments
Act.

 
. It was also later contended by Mr. Devgan that the complaint lacks the list of witnesses and
this will affect his defence leading to the failure of the justice and hence the complaint should
be quashed. Is his argument correct?

(a) Yes, the complaint lacks the list of witnesses which will weaken the defence of Mr.
Devgan.

(b) Yes, Section 204 CrPC mandates that the Court shall not issue summon or warrant against
the accused unless list of prosecution witnesses is filed.

(c) No, it is a mere irregularity and quashing of complaint will not be required.

(d) No, quashing of the complaint will lead of failure of justice for Mr. Devgan.

. Assume when the cheque was presented before the Bank, it was revealed that the amount
the Bank has been authorized to honour is Rs 20 lakhs and not more than that and due to
which the cheque was dishonored. Even after making a demand notice to Mr. Devgan, there
was no response as he had left for a foreign trip. Whether Section 138 NI Act is applicable
against Mr. Devgan in the case where the dishonor is not due to insufficiency of funds?

(a) Yes, Section 138 is not limited to only insufficiency of funds but for other prescribed
reasons when payment is defaulted.

(b) Yes, Mr. Devgan did not pay the amount on demand notice.

(c) No, the Bank has defaulted in the payment and not Mr. Devgan.

(d) No, Section 138 is applicable only against the defaulter which is the Bank.

. Ashutosh was involved in another transaction with Ms. Shobha where he had a liability of
Rs. 20 lakhs. He is believing that in few days he would be receiving money from Mr. Devgan
issued a cheque to Ms. Shobha informing her to encash the cheque only after 3 days. After 3
days, when Ms. Shobha goes to the Bank, but the cheque is dishonored due to insufficiency
of funds. According to Ashutosh, this is a case of good faith and Mr. Devgan should be held
liable as a defaulter and not him. Is this argument correct?

(a) Yes, this involves good faith principle and hence the liability should be invoked only
against Mr. Devgan.

(b) No, there is dishonor of cheque issued in favour of Ms. Shobha by Ashutosh.

(c) Yes, the proper course of action should involve an action to be brought against by Mr.
Devgan by Ms. Shobha.

(d) No, Ashutosh failed to recover the amount from Mr. Devgan within the time prescribed to
Ms. Shobha for encashment of cheque.

45
Passage (Q.-Q.): Mere claim of two adults living together for a few days will not be sufficient
to attach legitimacy to a live-in relationship. The concept of live-in-relationship between two
adults of opposite gender has got recognition in India also, as the legislature has injected
some legitimacy in this kind of alliance, while promulgating “Protection of Women from
Domestic Violence Act, 2005” and liberally defined “domestic relationship” in Section 2(f).
Majority of such petitions contain formal symbolic averments, grounds with imaginary cause
of action, and are rarely founded upon 'actual' or 'real' existence of threat, and these types of
cases consume considerable time of this court, that too at the cost of many other cases
waiting in line for hearing. Merely because the two adults are living together for few days,
their claim of live-in-relationship based upon bald averment may not be enough to hold that
they are truly in live-in-relationship. As per the 2013 judgment of the Supreme Court in Indra
Sarma v. VKV Sarma, there are conditions to attach legitimate sanctity to such relationships.
This include duration of relationship, shared household, pooling of resources and financial
arrangements, domestic arrangement, sexual relationship, children, socialisation in public,
intention and conduct of the parties etc. (Extracted with requisite revisions and edits from
‘Mere claim of two adults living together for few days not sufficient to attach legitimacy to
live-in relationship: Punjab & Haryana High Court’ at)

. Rama, aged 14 years and Kant, 21 years claimed protection of their life and liberty from
their relatives who disapproved of their relationship. They claimed they are in a live-in
relationship and have decided to live together till Rama comes of age. They have also not
established any sexual relationship but have portrayed themselves as a couple in front of
public and social gatherings. Does their relationship qualify as ‘domestic relationship’?

(a) Yes, Rama and Kant have portrayed themselves as a couple in social gatherings
(socialization of public).

(b) No, the concept of live-in is recognized only between two adults in India.

(c) Yes, the life and liberty of every individual should be protected and they have been living
together with plans to marry when Rama comes of age.

(d) No, there was no sexual relationship between Rama and Kant.

. Rashmi Trivedi and Guddu Shukla in order to get a rented apartment meant only for couples
portray themselves as engaged couples having their marriage scheduled in a year. They
attended various events in the society as a couple. After few days, Guddu fled without having
cleared his rent and even took certain valuables from the house of the landlord. The landlord
decided to make Rashmi accountable for this as according to him their relationship qualified
as a ‘domestic relationship’ but Rashmi denied. Is the argument of the landlord correct?

(a) No, intention of Rashmi and Guddu was not to come into a live-in relationship.

(b) Yes, they portrayed themselves as a couple in society functions.

(c) No, their relationship lacked considerable duration to constitute a ‘live-in’.

(d) Yes, the loss has to be compensated by Rashmi.


 

. Renee and Gautam, have been living together in a flat in Bombay for 3 years. They also
opened a joint bank account in the State Bank of India. They had at occasions portrayed
themselves to be in a relationship in the company events and outside. After some time Renee
became pregnant with the child of Gautam. Gautam at that time revealed that he is already
married having a wife in Pune and hence their relationship cannot qualify as that of a live-in
relationship. Can their relationship qualify as a live-in relationship?

(a) No, Gautam was already married and no legal sanctity can be attributed to his relationship
with Renee.

(b) Yes, they had lived under the same roof for considerable amount of time with shared
finances.

 (c) No, giving legal validity of the relationship goes against the traditional viewpoint of the
idea of marriage.

(d) Yes, best interests of the child mandate giving of recognition to the relationship between
Renee and Gautam.

. The appellant had alleged that he married Kumkum according to Hindu rites and
ceremonies. The respondent Sukanya filed a petition under Section 125 Cr.P.C. where she
claimed that the appellant and she have been living together in her father’s house for 3-4
years and consummated their relationship. After this the appellant started living at his own
native place but would occasionally visit her. Later she was deserted by the appellant. Due to
lack of any means of livelihood, she was unable to maintain herself while the appellant
worked as a Senior Associate in a reputable law firm. Hence, she requested for maintenance
money from the appellant. Can she claim maintenance on the premise of live-in relationship?

(a) No, they lived together intermittently and hence their relationship lacked the element of
shared household.

(b) Yes, there was the element of shared household and consummation of relationship.

(c) No, the marriage with Kumkum is with all the rites and rituals and will prevail over any
relationship, if any, with Sukanya.

(d) Yes, the act of deserting was done by the appellant and he was in a better standing to
maintain Sukanya.

. Post the decriminalization of homosexuality, Mr. Ahuja and Mr. D’souza were having a
live-in relationship for about two years. Mr. Ahuja lost his job so he contended for
maintenance money from Mr. D’souza who had deserted him 3 months ago as their
relationship would constitute as a domestic relationship under 2005 Act. Is his argument
correct?

(a) Yes, they had lived in a shared household for a considerable duration of time.
(b) No, the relationship had ceased to exist for claiming any maintenance money from Mr.
D’souza.

(c) Yes, parity between the parties and a better financial standing requires that payment of
maintenance.

(d) No, live-in-relationship is recognized only between two adults of opposite gender in India
and the relationship in hand will thus not qualify as live-in.

Passage (Q.-Q.): The Advertising Standards Council of India (ASCI), on 27 May 2021, has
released the final 'Guidelines for Influencer Advertising in Digital Media' (Guidelines)
applicable for all posts published by influencers post 14 June 2021. The primary objective of
these guidelines is to enable consumers to identify paid promotional content from the other
content posted by influencers. This is aimed at preventing business owners and influencers
from misleading customers through promotional posts or campaigns on digital media
platforms. 'Influencers' are individuals with a large number of followers on social media
platforms and are recognised as a budding category of individuals who have a large follower
base and credibility in the social media space. In the case of a material connection between
the influencer and the advertiser, the influencers are required to put a clear, prominent, and
upfront disclosure label (such as Advertisement, Ad, Sponsored, Collaboration, or Free Gift)
on their posts identifying them as advertisements. Material Connection includes monetary
incentives, free products, discounts, gifts, trips, hotel stays, and family or employment
relationships between the influencer and the advertiser. Even in cases of unbiased or original
content by the influencer about any product, the influencer is required to make the
aforementioned disclosures. The influencers are required to carry out due diligence before
posting any content to ascertain that the advertiser is able to substantiate any claims made in
the promotional post or advertisement such as health benefit claims or any other claim of any
product. Thus, the posts containing misleading claims by any advertiser must not be posted
by an influencer.

. Azeeba is an 18 year old mass media student who has acquired an Instagram following of
around 2 million. She usually posts content on various new and incoming web based
applications. She is known for paid collaborations with tech start-ups for their promotions on
social media. In all her posts, she assures her followers that she tests everything before
putting out good words on any product or service. Recently, she put out several post
announcing her collaboration with CryptForm, a new cryptocurrency exchange platform. In
her posts Azeeba referred to CryptForm to be a “trusted” platform. Taking her by her words,
her followers started making crypto-transactions on that platform, including purchase of
crypto coins. After a couple of months, the platform suddenly disappeared on the face of the
web and along with that all the money that people had invested in it. Azeeba put out a
statement saying that she herself wasn’t aware that CryptForm is a fraudulent platform since
she had invested money in it which was lost too. Has Azeeba violated the ASCI guidelines?

(a) No, because the information given out by her weren’t misleading information since she
believed the platform to be a genuine one.

(b) Yes, because under the guidelines, she was required to conduct due diligence before
promoting anything.
(c) No, because she had already disclosed it to her followers that it was a paid promotion.

(d) Yes, because people trusted her by her words and made purchase on the platform.

. Zerxes is a fitness enthusiast who has a YouTube subscriber count of 0K. He recently
announced to his followers that he would be trying out Cocoatini’s newly launched low
calories protein bars. Subsequently, he posted a 20 minutes long video whereby he could be
seen trying out Cocoatini’s different flavored protein bars. In the end, he says that the protein
bars are completely safe and fit for daily consumption. Seeing the video, Vinod bought three
protein bars and immediately consumed them. After 10 mins, he started noticing red patches
on his body and loss of breath. When it became worse, he was rushed to the hospital. Turned
out, he had peanut allergy. He sued Zerxes for violating the ASCI Guidelines for spreading
misleading information. Are his claims well-founded?

(a) Yes, because Zerxes had stated that the bars were completely safe and fit for consumption
that led to Vinod trying them out.

(b) No, because Zerxes had said that the bars were safe after trying them out in the video.

(c) Yes, because he failed to take due diligence before posting the video.

(d) No, because Vinod had peanut allergy which is completely circumstantial and his allergic
reaction was due to the peanuts in general and not because of the bars being unsafe for
consumption.

. Kavya Dutta is a well-known 45 year old Instagram influencer who has more than a million
followers. Recently, she made a video where she showed her followers different products that
she uses in her skincare routine. She also said that after using the Del cream, she has noticed
visible changes in the overall quality of her skin. She also shared a before and after picture of
using the cream in a sponsorship post by Del. Several of her followers purchased the cream
and reached out to her saying that they have noticed similar results and would continue
buying the cream. In a later post, Kavya stated that the Cream has been “clinically proven”
and “boosted new genes” to give visibly younger skin, without any actual scientific evidence.
Does this amount to violation of the ASCI guidelines?

(a) No, because her follower also tried the cream and got the promised results.

(b) No, because she did not misled her followers since she showed her own results after using
the cream and shared the next post only after getting positive reviews from her followers.

(c) Yes, because there was no scientific evidence to show that the cream actually boosted new
genes.

(d) Yes, because she made the statements after only a few reviews from her followers.

 
. Alviya started a weight loss journey video series on her YouTube Channel, also for paid
promotion of SPIKE, a workout apparel brand, and she would live stream her workout
routines and what she ate in a day. Her subscribers noticed that she was regularly consuming
a dietary supplement called “Carbs-go” before her workouts. After a couple of weeks into the
series, her subscribers started noticing visible weight loss in Alviya. Subsequently, several of
her subscribers purchased Carbs-go and even though it was expensive, they thought of giving
it a try. However, none of the subscribers noticed any change in their weight. They started
blaming Alviya for misleading her subscribers. In this regard:

(a) Kavya has not misled her subscribers because she merely showed what she actually ate in
a day which was not a part of the paid promotion of the clothing line.

(b) Kavya has misled her subscribers because she was seen consuming Carbs-go in the same
video that was promoting SPIKE.

(c) Kavya has misled her subscribers because they tried the expensive product after seeing
her using it in her videos.

(d) Kavya has not misled her subscribers because she actually consumed Carbs-go that
actually showed visible results.

. On the facts mentioned in the above question, had the video been a paid promotion of
Carbs-go, would her subscriber’s claims regarding her videos being misleading be valid as
per the ASCI guidelines?

(a) Yes, because after trying Carbs-go, her subscribers failed to see any results.

(b) No, because she actually consumed Carbs-go in the video series and there was actual
noticeable weight loss.

(c) No, because she did not explicitly ask her subscribers to try it themselves.

(d) Yes, because she is an influencer and her followers believe her.

Passage (Q.-Q.): There are a number of laws pertaining to hoarding of commodities,


including medicine and medical equipment. The Prevention of Black Marketing &
Maintenance of Supplies of Essential Commodities Act, 19 was enacted with an objective to
allow Central and State governments to pass orders to detain persons who seek to control the
production, supply or distribution of any commodity which is an essential commodity as
defined under the Essential Commodities Act of 15. Section 3 of the said Act states that
whoever instigates any person to commit any offence punishable under the Essential
Commodities Act, or any other law enforced for the time being related to production, supply
or distribution of an essential commodity, with an intention to make gain in any manner, shall
be punished. It is pertinent to note here that masks, medicines and medical equipment are
covered under the Essential Commodities Act. Section 4 states that if a person is found to
avoid the order of detention or is absconding, the government or officer shall draft a report in
writing to the Metropolitan Magistrate or Judicial Magistrate First Class, who shall pass an
order against such person under Sections , , and of the Code of Criminal Procedure (CrPC).
The provisions of Section 4 are also applicable once the authorities have an apprehension that
the person against whom orders of detention have been made, is absconding. An individual
can also approach the local court to file a criminal complaint against such hoarding and
illegal holding of such medicines under Section 200 of the CrPC. . Rautam Rambhir was a
famous cricketer who entered politics after his retirement. While he was serving as an MLA,
a deadly virus called Coronavirus hit the world, leading to a pandemic. The virus was
extremely contagious and a large number of people were struggling to find enough resources,
especially FabiFlu which was an essential part of the treatment. Rautam Rambhir stored a
large quantity of FabiFlu at his house. He distributed some of this stock to people in his
constituency. However, a shortage of FabiFlu continued to plague the market due to hoarding
behavior by some people. Is Rautam Rambhir in violation of the laws mentioned in the
passage?

(a) Yes, because he stored large quantities of FabiFlu with an intention to make unlawful
gains

(b) Yes, because he sought to control the production, supply and distribution of FabiFlu by
storing and distributing it as per his whims

(c) No, because he stored and distributed FabiFlu to the people of his constituency in the
spirit of public health and welfare

(d) No, because he did not instigate any person to commit any offence punishable under the
Essential Commodities Act

. Rautam’s brother Sautam was also an influential politician. He used his money and power to
purchase 10,000 boxes of Crocin. Unlike FabiFlu whose cost and availability were always
dubious due to hoarding and black marketing, Crocin was a cheap, freely available medicine
which was also used in the treatment of Coronavirus. Following the raid at Rautam’s house,
the police raided Sautam’s house where they discovered a 5ft tall tower of Crocin boxes. Can
the police arrest Sautam under the same charges as his brother?

(a) No, because controlling the production, supply and distribution of Crocin or hoarding it is
not illegal

(b) No, because Sautam did not instigate any person to commit any offence related to
production, supply or distribution of an essential commodity, with an intention to make gain
in any manner

(c) Yes, because Sautam tried to control the production, supply and distribution of Crocin
which was an essential commodity

(d) None of the above

. X was a famous restauranteur and ran a popular food chain called ‘Khan Chacha’. Due to
the popularity of the chain, X had become very rich and influential. There was an acute
shortage of oxygen cylinders and oxygen concentrators in the market and black marketing of
these commodities had reached an alltime high with no cap on the price. The local police
noticed long lines outside Khan Chacha outlets and conducted a raid. They found that X was
using the outlets as a place to store large quantities of oxygen concentrators and was acting as
a middleman to sell concentrators in the market. Is X in violation of the Essential
Commodities Act?

(a) Yes, because he intentionally disrupted the production, supply and distribution of oxygen
concentrators in the market

(b) Yes, because he intentionally disrupted the supply and distribution of oxygen
concentrators in the market

(c) Yes, because he intentionally disrupted the production and distribution of oxygen
concentrators in the market

(d) No, because he did not disrupt the production, supply or distribution of oxygen
concentrators with an intention to make unlawful gains

. Assume that X was found to be in violation of The Prevention of Black Marketing &
Maintenance of Supplies of Essential Commodities Act, 19. The police conducted a thorough
enquiry and found that X had been selling the oxygen concentrators in the black market at
exorbitant prices. An order was passed against X and he was asked to surrender himself to
judicial detention. However, as soon as X found out that he might have to go to jail, he flew
to Antigua and Barbados. Is this conduct of X punishable?

(a) No, because the Act only criminalizes hoarding and black marketing of essential
commodities and does not prescribe a procedure for non-cooperation

(b) Yes, because a Magistrate can pass an order against such a person under Section 200 of
the CrPC

(c) Yes, because Section 3 of the Act prescribes the procedure in case a person against whom
orders of detention have been made, is absconding

(d) Yes, because Section 4 of the Act prescribes the procedure in case a person against whom
orders of detention have been made, is absconding

Passage (Q.-Q.): In a recent matter, the State Consumer Disputes Redressal Commission,
Telangana reiterated the legal position that if the insured is found to have suppressed the
information which was material for the insurer to decide about the issuance of the policy is
made out, the insurance company cannot be made liable to indemnify the insured on the
ground that contractual obligations between insured and insurer are based purely on good
faith and if insured has knowingly failed to reveal the information which was within her
exclusive knowledge, the insurer could not be said to be liable to indemnify the insured. In
the present case, the insurance company contended that even before taking the policy, the
insured was suffering from a serious ailment and was undergoing treatment and evidence was
placed on record with regard to the said contention. The Commission held that perusal of the
crucial documents on record leaves no room for doubt that the insured was aware that she
was suffering from a serious ailment for more than 6 months prior to taking the insurance
policy and suppressing all those facts, she took the policy. The Commission observed that “If
the insurer can show that prior to the date of declaration of being healthy, the insured was
suffering with ailment which was within her knowledge but was suppressed, then the
insurance company is well within its right to repudiate the claim on the ground of suppression
veri.” [Extracted with revisions from 'Can insurance company repudiate claim if insured
suppresses fact of suffering from ailment while taking policy? Telangana State Consumer
Forum answers' by Devika Sharma, published 22 June 2021 on scconline blog]

. Mona decided to take a life insurance policy from LIC. When she was filling the forms, she
realized that the policy was very expensive and she would not be able to afford it. As a result,
when asked if she suffers from any illness or if she had undergone any surgery in the past,
she gave answers that would help bring the cost down. Most of her answers in the form
reflected that she was a healthy person with no medical history, even if that wasn’t true.
Subsequently, Mona fell ill and required medical support. Can LIC deny her medical claim?

(a) Yes, because Mona did not suppress any ‘material information’ from LIC while filling the
forms

(b) Yes, because Mona was not aware that she would fall ill after filling the insurance form

(c) No, because Mona was aware that she was ill and hid this information from her insurer
LIC

(d) Yes , because Mona suppressed facts and filled incorrect answers in the insurance form
provided by LIC

. Rama was employed in a construction company as an on-site builder. She recently


purchased health insurance and paid full premium for the same. While purchasing the policy,
she had clearly shared her occupation with the insurer in order to get the most suitable health
plan for herself. Two months later, an accident occurred on her work site as a boulder fell on
her head. Can she claim the amount necessary for her treatment from her insurer?

(a) Yes, because Rama did not reveal the information which was within her exclusive
knowledge (b) Yes, because she did not suppress any information related to her occupation
while purchasing the insurance

(c) No, because she was aware that her job was risky and accident-prone before purchasing
the insurance

(d) No, because prior to the date of declaration of being healthy, Rama was employed in a
construction company . Mukesh was a chain-smoker. He had picked up the habit of smoking
at the age of 18. He purchased health insurance for himself at the age of 24, when he got his
first job. As he was generally in good health, he did not find it necessary to reveal his
smoking habit in the insurance forms. He developed lung cancer at the age of 30 years and
seeks to claim this amount from his health insurance. The insurers found out that his lung
cancer was a consequence of chain-smoking. Can they deny his claim?
(a) No, because Mukesh knowingly failed to reveal the information which was within his
exclusive knowledge

(b) No, because although he suppressed the fact the he was a smoker, it was not material as
Mukesh was generally in good health

(c) Yes, because he suppressed the fact that he was addicted to smoking which was material
information

(d) Yes, because Mukesh did not knowingly fail to reveal the information which was within
his exclusive knowledge

. Samuel’s grandfather suffered from diabetes. His father inherited the disease from his
grandfather, and was also diagnosed as being diabetic. When Samuel was filling his
insurance form, he did a full body profile test and found that he was in good health and
without any disease. However, he did not reveal his family history of diabetes. If he is
subsequently diagnosed with diabetes and suffers from any related diseases, could Samuel’s
insurer deny his claim?

(a) Yes, because Samuel failed to reveal his family history of diabetes which was immaterial
to his case

(b) Yes, because Samuel failed to reveal his family history of diabetes which was material to
his case

(c) No, because Samuel failed to reveal his family history of diabetes which was material to
his case

(d) No, because Samuel failed to reveal his family history of diabetes which was immaterial
to his case

. Amarchand was diagnosed with Pneumococa, a rare genetic disease even though no one in
his immediate family suffered from it. When he tried to claim his treatment cost from his
insurer, he was vehemently refused. The insurer revealed that they carried out a detailed
investigation and found that Amarchand’s distant relative Manu also suffered from
Pneumococa. Amarchand did not know that Manu suffered from Pneumococa, or that he
even had any relative names Manu. Can Amarchand hold the insurer liable for not
reimbursing his medical costs? Yes, – not within exclusive knowledge

(a) No, because Amarchand failed to reveal material information that was within his
exclusive knowledge at the time of applying for the insurance

(b) No, because Amarchand suppressed information regarding the inheritance of a gene
causing Pneumococa

(c) Yes, because Amarchand did not suppress information regarding the inheritance of a gene
causing Pneumococa
(d) Yes, because the material information was not within Amarchand’s exclusive knowledge
at the time of applying for the insurance

Passage (Q.-Q.): Patent Class 426 takes into account the protection of food sources and plans.
The language of this standard covers food sources and consumable materials. The law sees
food as a piece of tissue, which is one of the classifications qualified for licenses. An
innovator can make another synthesis that imaginatively changes the construction. As such, a
culinary expert can demonstrate inventiveness by making a formula that nobody else has at
any point made previously. The appropriate legal dispute that tests this law is the decision in
Graham v. John Deere (16). The requirements are as follow: It should have creativity: One of
the main necessities for creation is freshness. Anything recently created is ineligible for a
patent. The equivalent is valid for food sources and plans. The new creation should have
recognizing highlights that demonstrate nobody has at any point made it previously. In the
field of food, this is something extreme to demonstrate. The most delectable pizza on the
planet probably has a large number of similar fixings and cooking ventures as different
pizzas. The patent candidate should show exhaustively what makes the new, more delightful
pizza unique from all the others that preceded it. It should have utility: A development should
have esteem. A creation with no object is futile. Any food that people can eat securely has a
specific degree of significant worth. A few plans will have more worth than others, yet the
overall principle is that all new food varieties and plans ought to fulfil the utility necessity.
Complete honesty is required: An extra advance for food manifestations is to tell the best way
to make the item. The creator should give a point-by-point clarification of the interaction.
This depiction should train others such that anybody could mirror it without performing
unnecessary experimentation.

. Ronald ran a popular burger chain. He was bored of selling regular circular burgers and
wanted to do something different to attract more customers to his restaurants. After several
years of experimentation, he succeeded in making a unique triangular burger. Not only was
the cheese slice and patty in a triangular shape, even the bun was conical with a triangular
base. Although people have succeeded in making triangular baked goods in the past, no one
has ever baked a triangular bun to make such a burger. Can Ronald patent his burger and
burger bun?

(a) Yes, because the burger and burger bun are creative, useful and unique

(b) Yes, because the burger and burger bun are creative, useful, honest and unique

(c) No, because only the burger is unique, creative and useful and the bun is an afterthought
to the burger

(d) No, because only the bun is unique, creative and useful and the burger is a consequence of
the bun

 
 

. Harbhajan was years old. Her grandchildren always loved to eat her delicious pickles,
which she made at home with a lot of tender love and care. They encouraged her to sell her
pickles in the market and she received raving reviews from customers. She used age-old,
traditional and secret spices to season her pickles. In particular, her lemon pickle always
remained out of stock due to the high demand. Can Harbhajan apply for a patent to protect
her lemon pickle recipe?

(a) Yes, because her lemon pickle is unique, creative and made of secretive spices

(b) Yes, because others cannot replicate her pickle anybody without performing unnecessary
experimentation.

(c) No, because her lemon pickle is neither unique nor the first such pickle in the market

(d) No, because her lemon pickle is not creative, fresh or more delightful than all the other
pickles in the market

. Richie Rich ran a high-end bakery and ice-cream parlour in South Bombay. His
establishment was popular amongst the elite for its premium ingredients, even though the
food was very expensive. He launched a new offering in his menu – an ice-cream sundae
called ‘Shahi Tijori’. It contained two scoops of premium vanilla ice-cream made from
imported French vanilla beans, topped with a 26-carat gold leaf, shaved truffles and dessert
caviar. Richie wants to apply for a patent to protect his creation. Will his application
succeed?

(a) Yes, because Shahi Tijori is unique, creative and cannot be replicated by anyone else due
to the premium nature of the dessert

(b) Yes, because Shahi Tijori has recognizing highlights that demonstrate nobody has at any
point made it previously

(c) Yes, because the premium and expensive nature of Shahi Tijori implies that the dessert
has a lot of utility

(d) None of the above

. Ritu recently completed an advanced baking course and received her diploma. Right after
completing her course, Ritu started baking different kinds of exotic breads at home and
selling them through her Instagram Store. Over time, she perfected her recipe to increase the
shelf-life of her goods so that she could deliver them to different parts of the country, without
getting spoilt. Ritu’s breads could remain fresh for 15 days without being refrigerated. In
comparison, regular breads had a shelf life of merely a week. Is Ritu’s recipe capable of
being patented?
(a) Yes, because her bread is more delicious than all the other breads in the market

(b) No, because the nature or characteristics of the bread have not changed and there is
nothing original about it

(c) Yes, because extending the shelf life of bread by 8 days is a significant development that
can be patented

(d) No, because extending the shelf life of bread by 8 days is not a significant development
which could be patented

. Sofia was a food scientist by profession. When she researched about the exploitative nature
of the meat industry, she was devastated and decided to become vegetarian. Over time, she
missed the taste of meat and did not like any vegetarian alternatives. She spent months in the
lab and finally created ‘Meatless meat’ – a unique meat alternative made with a blend of
soya, tofu, paneer and proteins in a specific proportion. Is Meatless Meat capable of patent
protection?

(a) Yes, because it is creative, novel, useful and has recognizable highlights that cannot be
easily reproduced

(b) Yes, because making something vegetarian amounts to improving food sources which is a
requirement for patent protection

(c) Both (a) and (b)

(d) None of the above

. Rajinder da dhaba was famous for its Butter Chicken. It had a cult-like following since the
past 25 years, when it first opened. In 2021, Maninder opened a restaurant next to Rajinder da
dhaba called Maninder da dhaba. The Butter Chicken at Maninder da dhaba became even
more popular for its better, creamier taste. Can Maninder apply for a patent to protect the
recipe of his Butter Chicken?

(a) No, because it is not creative or novel and can be easily reproduced by others

(b) No, because improved taste is subjective and not creative by itself

(c) No, because Maninder’s recipe has no utility and can be mirrored by others

(d) All of the above

Passage (Q.-Q.): The Hindu Marriage Act, 15 enshrined nine fault grounds in Section 13(1)
on which either the husband or wife could sue for divorce and two fault grounds in Section
13(2) on which wife alone could seek the divorce. In 14, by an amendment, certain clauses of
Section 13(1) were amended in the form of Section 13(1A), thus recognizing two grounds of
the breakdown of the marriage. The 19 amendment Act inserted two additional fault grounds
of divorce for wife & a new section 13B for divorce by mutual consent. Of them cruelty is
one such ground. The concept of cruelty is a changing concept. The modern concept of
cruelty includes both mental and physical cruelty. Acts of cruelty are behavioral
manifestations stimulated by different factors in the life of spouses, and their surroundings
and therefore; each case has to be decided on the basis of its own set of facts. While physical
cruelty is easy to determine, it is difficult to say what mental cruelty consists of. Perhaps,
mental cruelty is lack of such conjugal kindness, which inflicts the pain of such a degree and
duration that it adversely affects the health, mental or bodily, of the spouse on whom it is
inflicted. In Pravin Mehta v. Inderjeet Mehta, the court has defined mental cruelty as ‘the
state of mind.’ Desertion is also a ground for divorce which means the rejection by one party
of all the obligations of marriage or the permanent forsaking or abandonment of one spouse
by the other without any reasonable cause and intentionally without the consent of the other.
It means a total repudiation of marital obligation. Contagiousness of leprosy and repulsive
outward manifestations are responsible for creating psychology where man not only shuns the
26 of 40 company of lepers but looks at them scornfully. Thus, it is provided as a ground for
divorce under Hindu Marriage Act but was amended by the Parliament in 2019.

. Shubham and Sarita were in good relations from their college days. They got into a live-in
relationship for 2 years and finally got married in 2020. Shubham’s family was a middle class
family and was very grounded. Their base source of income is farming only. On the other
hand, Sarita belongs to a modern class family. However, there was good compatibility
between Shubham and Sarita. After their marriage, one day Shubham asked Sarita to help his
mother in putting cow dung to prepare the organic Cow Dung Cake. Sarita refused him as she
does not like to even put her hands into the cow dung (Gobar). Shubham asked her to do it as
it is the purest thing she will be doing and cow dung is good for skin too. He continuously
asked him for the same. Sarita was helpless and finally helped her mother-in-law for this and
later filed a divorce petition on grounds of cruelty. Decide –

(a) The divorce petition filed by Sarita will succeed on the grounds of cruelty as Shubham
forced her to prepare Cow Dung Cake with his mother.

(b) The divorce petition filed by Sarita will not succeed as there was no cruelty committed on
part of Shubham and the divorce petition was filed on frivolous grounds.

(c) The divorce petition filed by Sarita will succeed on the grounds of cruelty as Shubham
tortured her to help her mother-in-law in putting Cow Dung Cake and there was imminent
threat to her life.

(d) The divorce petition filed by Sarita will fail on the grounds of cruelty as there was neither
mental nor physical cruelty committed by Shubham and her allegations were false.

. Dev and Anjali were in a happy marital relationship. Anjali was working in a famous IT
firm in Bangalore and Dev was a manager in a good Consultancy firm. Dev was also
attracted towards her assistant Priya. He used to flirt with her and Priya also enjoys the same.
One day, Dev invited Priya for lunch in Sunshine Resort. Coincidentally, Anjali was also in
the same resort to attend a meeting. She sees Dev with Priya and gets to know more about
their friendly relationship. She was also sure that Dev might have physical intimacy with the
girl. Soon she gets some CCTV recordings from the resort and finds her husband to be in an
adulterous relationship. She decided not to continue this wedding. She continues to live with
Dev in their home but she discontinued all her conjugal duties. It goes on for a year and Dev
filed a petition for divorce on the grounds of desertion. Decide –

(a) Dev’s petition for divorce will fail as there was no desertion on the part of Anjali. She was
living with Dev in their house but has only discontinued her conjugal relations with him.

(b) Dev’s petition for divorce will not fail as there was desertion on the part of Anjali as she
had discontinued her conjugal relations with Dev and it has been more than a year.

(c) Dev’s petition for divorce will fail as there was no desertion on part of Anjali even though
she has discontinued all her conjugal obligations of the marriage.

(d) Dev’s petition for divorce will not fail as there was desertion on the part of Anjali as she
has abandoned all her marital obligations with Dev.

. Madhu and Manav got an arranged marriage in 2020. Their families were close friends from
long back and had decided to get them into marital tie once they attain the age of marriage.
Manav and Madhu had not seen each other before their marriage. On the night of their
marriage, Manav saw Madhu for the first time. He saw some discoloration of skin on her
neck and some parts of hand. Madhu told her that she was suffering from leprosy from her
adolescent phase. She also had disfigurement in her toes. Manav gets angry and he does not
want to continue this marriage. He filed a divorce petition on the grounds of leprosy. He also
wished to file a criminal case on Madhu and her family for fraud and hiding the material facts
before marriage but his family asked him not to do so. Decide –

(a) The divorce petition filed by Manav will fail on the grounds of leprosy as divorce cannot
be granted on such ground and his marriage continues with Madhu.

(b) The divorce petition filed by Manav will not fail on the grounds of leprosy as divorce can
be granted on such ground and his marriage will not continue with Madhu.

(c) The divorce petition filed by Manav will fail on the grounds of leprosy as discoloration of
skin and disfigurement of some body parts does not amount to leprosy.

(d) The divorce petition filed by Manav will not fail on the grounds of leprosy as
discoloration of skin and disfigurement of body parts are temporary in nature and are curable.

. Arjun and Subhadra got married to each other. They truly love each other. Subhadra was a
bit possessive towards Arjun. Soon she gets to know that Arjun was having an affair with his
colleague and who is also her best friend. She was not sure of the same but wanted to clear
her doubts. She investigated and saw that Arjun met her friend in the Resto Bar a night before
and they stayed in the same room for the whole day. When she met Arjun she lost her temper
and shouted at him a lot. She said he is impotent and cannot even have a child. Arjun was not
sure of the situation and tried to handle it but was not able to do so. After a big fight, he
decided to get divorced as Subhadra had alleged him of serious and false allegations. Decide

(a) Arjun can claim divorce from Subhadra on the grounds of cruelty as she spitted false
allegations which affects his character and inflicts pain to Arjun.

(b) Arjun cannot claim divorce from Subhadra on the grounds of cruelty as she was correct
on her part and such temper outburst is usual in marital relationships.

(c) Arjun can claim divorce from Subhadra on the grounds of cruelty as she was not sure of
the allegations and trusted them without inquiring about them once from Arjun.

(d) Arjun cannot claim divorce from Subhadra on the grounds of cruelty as she had not
inflicted any such pain to Arjun and was well within her limits and rights of a Wife.

. To prove desertion and claim divorce there are certain essentials which have to be proved in
the court of law. Choose from the following those essential(s) to prove Desertion

I. Spouses must be living separately from each other.

II. Aimus Deserdendi

III. Spouse must desert the other one with reasonable cause. IV. Spouse deserting must take
consent of the other party before desertion.

(a) Both I and II

(b) Only II

(c) I, III and IV

(d) Only I

. Choose among the following situation(s) which does/do not amount to Cruelty. I. Mr.
Sharma refusing to have marital intercourse with his wife Mrs. Sanjana. She want to have a
child but Mr. Sharma does not agree to her II. Wife’s writing false complaints to the
employer of the husband. III. Mr. Manoj asked his wife to resign from her job and live with
him. Mrs. Nirmala outbursts of temper without rancor on Manoj and refuses to resign from
her job. IV. Sushil and Sarla face incompatibility of temperament in their married life and it
seems like their wedding is an irretrievable breakdown of marriage.

(a) Both I and IV

(b) Only III

(c) I, II and IV

(d) All of the above


 

Passage (Q.-Q.): Read the following passage and in the light of the same answer the
questions that follow. The maintenance proceedings is not to punish a person for his past
neglect, but to prevent vagrancy leading to the commission of crime and starvation by
compelling those who can do so to support those who are unable to support themselves and
who have a moral claim to support. As per Section 125(l) (a) of the Code, if any person
having sufficient means neglects or refuses to maintain his wife, unable to maintain herself, a
Magistrate of the first class may, upon proof of such neglect or refusal, order such person to
make a monthly allowance for the maintenance of his wife. The wife for the purpose of the
section may be of any age and means a legally married woman. Under Section 125(l) (a) of
the Code, maintenance allowance cannot be granted to every wife who is neglected by
husband or whose husband refuses to maintain her, but can only be granted to a wife who is
unable to maintain herself. In Mohd. Ahmed Khan v. Shah Bano Begum, it is declared that a
Muslim husband having sufficient means must provide maintenance to his divorced wife who
is unable to maintain herself. Such a wife is entitled to the maintenance even if she refuses to
live with the Muslim husband because he has contracted another marriage within the limits of
four wives allowed to him by Quran. According to Section 125(1) of the Code of Criminal
Procedure, the person from whom maintenance is claimed must have sufficient means to
maintain the person or persons claiming maintenance. As per Section 125(4) of the Code of
Criminal Procedure, no wife shall be entitled to receive an allowance for the maintenance or
the interim maintenance and expenses of proceeding, as the case may be, from her husband
under Section 125 if she is living in adultery or is living separately with mutual consent. A
divorce decree by mutual consent to live separately cannot disentitle the wife to claim
maintenance.

. Rahul and Disha were good friends from their school days. They appeared for CLAT 2016
and were in the same NLU i.e NLIU, Bhopal for five years. Their friendship has grown into a
beautiful relationship and they both were happy with this transition. They were in a live-in
relationship from their 3rd year of Law College and finally they decided to get married. Disha
was a bit confused as she was not sure whether Rahul would be happy to hear that she was
pregnant. She wanted to tell him but before she could do so, Rahul confessed to Disha that he
loves to be with her but he does have other sexual relations too with her friends. Disha was
very upset to hear this and she left Rahul. She simultaneously filed a suit in the family court
of Bhopal claiming maintenance for her and the unborn child of Rahul. Decide –

(a) Disha cannot claim maintenance from Rahul and her suit will fail because she is not
entitled to such maintenance right from Rahul either for her or the unborn child.

(b) Disha can claim maintenance from Rahul and her suit will succeed because she is entitled
to such maintenance right from Rahul, both for her and her unborn child.

(c) Disha cannot claim maintenance from Rahul and her suit will fail because she has not
divorced from Rahul which is the essential requirement of section 125 of the code.

(d) Disha can claim maintenance from Rahul and her suit will not fail because she does not
have the sufficient means to maintain herself and her unborn child.
 

. Sonu and Akash got married to each other due to the family pressure but they don’t like
each other. Akash was a playboy from his college days and Sonu was an ambitious girl who
always wanted to be independent and achieve the best in the world. They were not happy
with this tie but Sonu was the one who was suffering a lot due to the orthodoxical patriarchal
setup in her family. She tried to talk to her father in this regard but was not sure whether he
would understand her. She deserted Akash and started her own IT Firm in Kanpur. Akash
was also happy with this arrangement. Soon they mutually decided to live separately and not
to inform their families about the same. Akash promised to send her a fixed amount of money
monthly till the time her company starts earning good profits. Soon, Sonu filed a suit for
maintenance from Akash as she thought she could invest that money in her firm to explore
the market more. Decide –

(a) Sonu cannot file a suit for maintenance and claim it from Akash as she is not entitled for
maintenance under section 125 of the code.

(b) Sonu can file a suit for maintenance and claim it from Akash as she is legally wedded to
Akash and has no sufficient means to maintain herself.

(c) Sonu cannot file a suit for maintenance and claim it from Akash as she has sufficient
means to maintain herself and she started her company independently.

(d) Sonu can file a suit for maintenance and claim it from Akash as she is entitled for
allowance or interim maintenance from Akash under section 125 of the code.

. Bunty and Mini were good friends. Bunty’s father was close to Mini’s father and they
decided to get them married. Mini was happy to hear that and agreed readily for the same. On
the other hand, Bunty was not sure of this wedding tie and was confused. He informed her
paramour Chinu that his family wants him to get married to Mini. Chinu advised him to get
married and continue his physical intimacy with her. He did the same. Soon after the
marriage, Bunty falls in love with Mini and decides not to meet Chinu. Chinu filed a case
against Bunty claiming maintenance from him. Decide –

(a) Chinu cannot claim maintenance from Bunty and her suit will fail because she is not
entitled to such maintenance right under section 125 of the code.

(b) Chinu can claim maintenance from Bunty and her suit will succeed because she is entitled
to such maintenance from Bunty under section 125 of the code as any woman can claim
maintenance under the said provision.

(c) Chinu cannot claim maintenance from Bunty and her suit will fail because she has not
divorced from Bunty which is the essential requirement of section 125 of the code.

(d) Chinu can claim maintenance from Bunty and her suit will not fail because she does not
have the sufficient means to maintain herself.

. Shahid and Zara got into Nikah and were living a happy life. Soon their family demanded
for a child and gets to know that Zara is incompetent to conceive the child. They asked
Shahid to marry Suraiya for the child. Zara was not happy with this decision of Shahid and
filed for a divorce petition. Suraiya was also not able to give birth to a child due to some
complications and Shahid gets Nikah with Zoya. Suraiya and Zara both succeed in claiming
maintenance from Shahid. Zara remarried to Shaheer. Zoya gave birth to Shahid’s child but
when she gets to know about Zara and Suraiya, she divorced Shahid and claimed
maintenance. Decide –

(a) Only Suraiya can claim maintenance from Shahid as she was legally married to him and
can have valid claim under section 125 of the Code.

(b) Both Suraiya and Zara can have valid claims of maintenance from Shahid under section
125 of the code as they do not have sufficient means to maintain themselves.

(c) Both Suraiya and Zoya can have valid claims of maintenance from Shahid under section
125 of the code as they were entitled to claim maintenance.

(d) All three of them, Suraiya, Zoya and Zara can claim maintenance from Shahid under
section 125 of the code as they all were legally wedded to Shahid and do not have sufficient
means to maintain.

. Choose the most appropriate option.

Assertion (A): Maintenance provisions can be found under personal laws and Code of
Criminal Procedure as well. The rationale behind maintenance laws is to prevent the wedding
tie between the husband and wife from being broken down.

Reasoning (R): The embargo of section 125 of the code is wider as it includes every woman
who is legally wedded to her husband and was neglected or refused to maintain despite
having sufficient means to maintain.

(a) Both A and R are correct and R is the correct explanation of A.

(b) Both A and R are correct but R is not the correct explanation of A.

(c) Only A is correct and R is incorrect.

(d) Both A and R are incorrect.

. Which among the following statement(s) is/are incorrect in context of Maintenance laws in
India?

I. The court will consider the financial position of the husband and reason for which the wife
is separated from her husband, before awarding maintenance to her.

II. A working woman is not entitled to claim maintenance from her husband.
III. Any woman can claim maintenance under section 125 of the Code of Criminal Procedure
irrespective of his/her personal laws.

IV. A Class II Magistrate may order a person to pay a monthly allowance for the maintenance
of his wife if she does not have the sufficient means to maintain herself.

(a) Both I and III

(b) Only IV

(c) Both II and IV

(d) I, III and IV

 
 

You might also like